You are on page 1of 274

j

j
.

"' §!iii
::e
'·• ~-·. ,, .• , .' ·,·,. ,., ',, ' ,.,,. '.

. . : .· ,-.~~.t'~l',•i1;~•

,( ,t '

'

., ~-±WJL ~ .
f;J
I'
•'
g
. . ;iisn;v1, b;'j,1;;~~mt w:1c:~i !li:

.,.
··c .
(/··: ,/'....?""t;·'',...,,--
.
,,: I ,,.•' ·'

',\
/
l /. & ,,,
,
,,,,.•---•"'"''"·•·
,..,..-⇒·-
.(,~·,·
. . . . ,.

• ··",., ,
.-::.

... .,., w,. ..~,._,


,,,,
. \,-y
?~.,.
·
/✓
. , . INS'ilTUTE FOP, IASIIFoS E.)'/1J1lltlATION
. ' '

'' . . . .
i \.1:. Dcflmhons :
.
NEW D!'i.l.i-11• ",oocg
,

Mon1erYts and ·.Pro.dtJcts


.
• Mob: 09991l1~7€12!l
,of:,-h:te·ttla
. ...
0
'

t"'~ ~:.:·\~~· '


·

· .
'

.
).

·:• · ·.
, ,


d•_,._.:1

·;
.• l

.,,
'

.............
;~,!
')j
't•

,,1
,JI (~) Rigid· l.locfy. A rigid bod)' ira cc-llcc1lcn of pw1kl:.~~-,.rn1.:,1nitu11.,,th1,1 .
··I' ~ ....;,,,~t1•
.t,:1':id
4,,;:::::·.·~ ...., ....,. ,,..,........ , ......... •' ····•.. qjsw1ic.e between.an)', rwo par1ic!es. of :hej bod)• re11ia1'11f·ahw1)1~:1hc .sat.Ill'. •
,, (b) Motl).~\'\t...of:Irnirtlii of.a. -p~rtk!!l, ·

' ·:;' The moment 0:f iMl'llO. of a•partielc


o( moss m !It ,lhc point p, . atio:ul :.(!J'j/.'.: <,
the lin~ A,!1·i~ dcl'inc~ by . ,·f''
··? ,,..
·-)i~JvJecRAMGS.~ • I= 111r • 1 •
1 •
I /
.¢,

.,=.;,,,-- . :a;=::: , where· r 'is :the _.pcl'pe.nclic\.llar- ..djstaJ1c6


of P from·the line AB.
. (c) M0cm_cnt o(I*~t\~. oC a sy~lprn -Q( . <
"('~'!:/. :·.-,~~:.:.:' ,. ...}•,· r·•.::\ :,{,::'.··,(1-;,.~r ··
particl~. -Toe,~<()/;l'i'J:l,~;,ol in4~ttAt:,"
system o(. ,parl,i\:l~s , Ot' . ni~si::J ,)t,1tt/i\:;::,
iti~~•:: ;
mi, mt, .. ,·, •nl{a'~./Jist.~n.c.c$, .. · : ..

' , .c, :ifj£f


. r1• l 2,. ,., r11 respccti,iic(y f(0.11'1· foe .
AB, about,1h1.1 .frne
AB. ls. Jefh,1~:I by .-,.
I~ 111 1 1+ mi rt+ ....... + m;,,~ ,:,
" 2
. = '>
i ,:..,1111.fi.··
[. l•I . .
(d} Mom~nt of inertia: of 11 bciby. U,
6111 be t~ m~s~ _of an elecne,mary . . .~~Pt"·,·•: .~,m
,,i.. ' ' ..• ''.
. .f POl'\iOn of_ the ·q~y 'and r, l\s'-cislaJ)Ce
. from<tne !i'ne,AJ),':'.tl\e'n -Ui:0@001e111 of; •, A"'!-. : ·2~:_.,
. .. ..,,.,
-.•

inenia or i11c' lliaS~"Om a.bout Lne lir,c


'
lf,
B . '~ .. .'""I""".'"''~
~ . IS ( ii(, ' . .. . , ,. , ....
:. 'Tni! m'olnei1l,·0~1ncrtill'·'Of.'\he ·ho<ly Q,bOUl
. . .. . . ·I =·f Jii,~. " .......
whcr~ (he: ihttigrati~n ·1f·lti!i.cn -over ·.he wholc•'.body •.
(ti)· R:idil!S ·:of ,Gy.rntioo,•Thi:.mo·nwnt of Inertia of u,body.
AB is ~iven by-· . , ·· · ·
?rim )=J
rr the. to.{,:11 mas~ of \he body 'is M :,nd .r< n ~l/Ch\th:it
;.;,: ~ ~· ,,

.. i-'.i ~:::

1
' ,: ·~- •i':; ',: :·,' .: ·~ ·."J~ri·.~!';~·:~·~l~,~·~·~~-·~~.~,J¼),~:rm~ut~~~rt-:~.:~:1';f''?lt;~·~r:1f1"'~.•:>1: 1:::,t-:,{j•'' :••::•

',;.,?'.:~;: ,, .
' ;;··i ,:~~'.~;: '•. ;~ •.j }, )' !i : · ~ :::-· ;~ ;~; '.,'.;,;,,:;•,
··~~~~:~?:;,··. . : •, .. ::-,·:: ..... , , :~ '·.~~•:.. :·!··~':<F·~~;~"'
:, .. ;.~•;.:,'• :, .
·,.::, ... ),::: :5: ..
-,

0
_________ ·······'"""',1!'"'··-"'··•..(•~·•...........~."'""-··: ~::.~............. - ........_~~... ~;.....;+.:.;i~-.s,;:..----------~ - ~ - - - - - - - -..:·-··-···-·-...·"~-l..'!~,_,-.,.,1...."'.. ~'"~Y ..1•···•~......... -"'·

l
·.z -Dynamics 'Of Rigid Dod)• Moments Md Pri>d11c/J of /11mia,
..·:·

•J
'{
~ .
'f ~ f'!Kl
f r' Thus ihe sum A+ 8 + C is independent of the dire,cti~;lS er ms, and.
tlicp,,J j~ paµca, t~e
tpqJ.ry ,6/,'8,~'(0\!a:1-, 01; ,!h~ .?,OdY,APtl_U_l,,1~~-- li~e.,AB. is equ\l) 10 twice '1he mome.1t of inertia about t!ie given point:·. · ·
-tfr.~~~¢t,~-9mfiertllif ·.J!.~1ti(xh' iM 1BB1·, H\ ·:t.-U-' r•:t~f:;,.fii1:;t 1
Prop, Ill. T/ie $Uni <if the ,i,ome//lJ of inertia >lJ/a body wiih reference IO.
C(l,Or.i:llrie.'tcs.bf n,,q)I\S$: m, with.:rcsp~cl.lO two . .. "1' Q/1)' p/atlf! th1rmgll a gi11en poinJ and irs nortiia1 .at I/lat po/111 is COtlS/Q/1/
multii!J_l.r, pcrpcndJcular !Ines OX 'nnd OY ·as e.ful i: equal 10 1/Je 11wmc111 of lner.tla of 1/te /Joi1j• with respect to the poi11/.
·axes;· tn,7~:.,h~. ~r-od_~ct of lnenlo of IT\ass :m ••• .x.: •• ,-.~:!+-~ ,~~·yj , .·, Lei ihe given point O be. taken u tlie urigln, and the plahc ns· XY phinc,
.,,,lth:t~~~i::\·16 th~'l!ncs O~ ond>O·Y.ls defined , , . . ·,: ,, :,• ,;, .•· _ If C' is the m<'t1T)e.n1 of inertia of the body nboµ.t thcX}''plane ond C \1'c
~y-,1i~•i:'.·'-'"_::··,•;,., ·..-'····· ·. ··,.·j;"··.'·:··. ' . . _.Y!: moment of ln~rt!a__ of the body ~bout Its n,ormal nt .Q_;whii::h is Z•axis,. thcn .
. I( (x'.~?-b~ th~ cqoI~mat~s _of_th~'m~; :0 ... .. .. · .. , X C'=Imt2 an4 C=tm(?+/), '···
mof ~11 elerr;~nt~y P,9,tt_itin ~H!,e b~)'--W1\h · · ·.," : .... •··, ,', C''+'C=l:m_(i+l/z2)=Lmr' .
rcsp·ecf to .\l:e· ?~.~il1icii\~~ a~e_s 0~ :a~d'·O.i'\ 1· • •• ,., •··., ·. ,' =M.'!'. .or the 'body-about O; · .
wen tbo pro~Nct-o< {,:ier.tl_ll. of the b_oa'y.;i.bout . ·. "\:,-,,, ... ·..-
, tli1se axes,·O,X arid OY Is de/inell'by Im,;cy, . .. · • • --. ,. . .
C
Thtti ·;;;·, + l$.)6~e.penderit of· the plane· thr6ugh O and 'is constnnr
~qua! to tb~ morl)en( of'jnerli~. of the.bQdy about .the point. · · •
~ 1.~. ~o'qi',~h·1 n'.nµ ;P,roduct of Irl'er.tla-.'.~11~ ·rc(pcd:to, th~?-riliitu,a·!iy '.\1 No'tt ti>' P~op, .we n; h.ave
A ~.'~ :+- C !" 2ll mr2 ,
pc~endicular nxts, - · - _ · ...•.. ,. ·· ... , : , ,.,. :· ·: .
.: ··Let (i, ,)',:)be the coordinntes_of the .m·nss 'in tif.·a ..bbdy'wit~•ttspeot t
and by prop, ill, we lfa~:,;e:p C'= l: mt.2 _ ·.
to three ·mut_ua-!ly porpencliculat oxes-OX, 'O,Y1.oz·In··space.s' 'I'non.we.,shatl ,', ·c.,,C: (ii+ B + Cf or Ct =} (A + B - C)'. . .
denote by A, B". C the niomehts of iner1i11•-of'the body ·about ·the cootdina'!e T11~s if A'. B', C' denot.e the momcnis of inertia of the bod)' v.·ilh
axe; ox, or, oz rcspec1!vely and by b, E, ft l~e:ptod\l,cts'--of'ineriia ab.out respect 10 1h¢ planes YL, ZX and XY respcectivelY,, theh
the axes OY;,-OZ; OZ, OX and· OX, _QY. .r,;sp~cti.v~ly. .'J:he.se>moments .nnd f( i(
A '-= B+ C:..:.. A), E '= C + A - B) and C' "'t
(A + B ~ C)
pro'diieJs ,of foertia are given· by . · .. , · . . . · · ' Prop, !V, A > 2D, B > 2£ and C > 2F.
A •l: 1/1 (l+ z2), B =..t 111 {~3 +P), _·_- · -C=.~m (~2 +i) we know that A.M. > G,M,
D .. :t.m;vz, .- • s .. :i:m~,• · F"'t'm..:r,i,, · · 2
,., >,2·; • :> ·k:i . t 2) or y2 + 112 > 2yz
§, 1.:t So:mtr ShrrJ_>,l~ Proposldon's,: . . .. :I
Prop, l, if~. B; -'C •Jenrm tht nioin~nrs. and D, E, F (h·e produ~ts of. iitcrria .or ! m<:i + i 2) > ?.:!: myz
a/Jout thrf~...mu111a11y_p,;p~1J.d/cu/(lr o.x,u, th~ Sl:/m of anj '.two of:them {s
greater tlfa'n th, (hlrd:- · · ·. · .. ,.:•: ·_,: , ... ,•., ' . ''"f ' 01 . I.e. A>' 2D ...
SirI)ilrlt!y ·B > 2E and C > 2F.
We have, ·Am l:m,(t + h 8·,. Im (i~,f".?,); q~;,m_ (r_ +l) . · j _•fyl~MENTS .OF INERTIA_ IN SOME. SIMPLE CASES.
then A+. (- C•Iiry_lr;.+_ti) + r.,~ (.t~ +F)- r..~ (} .;2) + · § lA. Moment of lnertla of a uniform rod of length 2a
=2Im;.;+vt, , , ·. (l) Abi:l~r a.fine i/irough,a,1 end and perpendicular 10,1/ie n,d;
:, .1+B>9: ·1. •• . .,. •.. , Lct:M .be the mass of it rod- AD of leogM 2n, then mass of the rod
Prop,Jf•. Th~' ,Nll71 :of;ttie .mom~1w1 of)~er;(lHfbO~f ,ar:iY. three_ re.ctang.~/ar per unit l~ng'lh = p,. Ml1a. .. · ·
qxe.r 'n1~din~: at a given point_ ls, ~!i'~ys c.~nstarit ·and 'is_ equal to tw(ce tha Consider nn element PQ of breadth 6.i: at a distance x from the end A,
moment of :inertia about that point. ·· . . . M . ' 1
We-: hav~ ·. · .· • . · · .. Mass of the element. PQ., 2a fu = ~m.
.I
A+ B'+ C= Im tf +'h+·i·m·(i2 +x11, +! m(J--:+,./) M.l. o:· this clement PQ ·about the llne LM passing through the end A und
=2:t· m (.2 +/ + z ) =22: ml( ,
2 perp,ondicular to· the rod AB
= 2 (Mior the body _about the given point) = 2x om: ~ rox,
•., r=--l(x2+y2'tzl) ;.,. distance o'f'.thc bss m·at (i,:i:z) from·ihe glvc11
M.'i. ~f ll;~ rod Ali about lM :,:,,. _.
pof_n! .o :u origin, .............
j

· ::·:'.'Jll~.~~-0~~~~~-'. ~~:.· •,~.:,:·,;-;•r>'.r~~~


:~••,·.:~~.;;;}Zi!t~!i:~::> . ', ,. ·.'
,-, ...,.,., ·-'~*~"'•~~~.~.,;u;,-:; • ,-, ···,,, '.!.:, ·~· ..
., _., ....... ,, •·····"··~·•,~;r,·n·,,.x~ .. ~.-,.-,'

liiWttlW/iifi i£iUH~fU§,~iml-i-WI;';:;,i.i~)};4~:i.'.~;;(i,~l~~~;~~!tr{~;~i:t~;JMfJ~{:~:;:~~; :.;,: ,; ~,;:·;::~:,. ~1,\i(:aJiit\h':'t~~):(~·~w.•q,,~!?<1.!,\t'I!.' u,:.t,t{t;I; .~~1•:;


., ..
·,;
i.t
',
,Jvu.t ~;
.... , ·~ ... ,, "
l
'-'-''f'!,1/~l,\,,J, II/ Hlfltl!

., :,'"'•.\~~' t
. . ' L f,
Mmm•ms ,,ml, Ptodil,'ts of /i1crtia· !i ~
"'f.. ;,r -·- .r·d.r"· -·
· n, 2/1
,\1 M [ I
, · ]"" · 4 . ,
- x:I. = -·J l\·Ja·
2a J . .
· · 1... ·, . ., -1· ;x X .., ·
.
I
i
'.
•.
. . ll . .. . •·-·-,-. ,._,,,., ·~•......... -·-·::io .n\' th~ r~c1a11~lc ABCD u\>out: OX.
r
!I
I Ii) Ai/MIi (l Ii.it(! 1itro;,_~h.1lic ll!lt/;11~ 1mi111 A --,:-... R..~,: ,.: .. - • - - .
W,·
.,; ' " ;
·. Mil·
."f. . " l
! i,,,,l
. ,·.1/l(i v,·rj,en,lu:11/(lr. '? the. 1·0~/, . . . ; I . ' .. _,. __ ds :•-»•" [ .1· l t1
"'l l
6a QC/. -II •'
· L,~I I.M he !he line_ pn$~111z !hrough.·M ..
ol' !he r<Jcrnnglo .ABCD ahLllll ·OY= ~ l\-lt1l
:',~
th~ 11n,:J<Jlc point C uml pcrpendic.ul,1r'·10 '•
the rud AB, ·· · :~
Con;;ider ~n elcmcn1 PQ
6.r ill :i distance x
C.
fro~, IL
~· Alitcr.- C\1YmJcr: an ~f~mcnwry' ,1r~11: 0:,6,1· ,11 _ll poi11t (,1·, ,1•)· or l!w l.i\nina.
. l\t,,,:l i:r .11\-·
"
. . 1"" .: .. ' . . ~ICt\ =;
. M
pox61·., -~-11~\'0\' = GIii .
.
l
1-
i ~
F
),ms, 1:f the el¢men1 . · . '·
'j
!, X. •• • ox M.1. ur 11\i~ eki1im111ai·)•
" mn~s nbout OX::...1·2'.0111 .;,'
' 4,u..,
,ef.{ 6.r61(
' r,;
· ,vi • • r----•-r--·-r::,,,.,.-10
A.____ .... _,-·--:'"'"·/f'1:;r ~
p() ., -· ux"" C,1>1
-: 2a (·: p==Ml2a) . . M.!. or tl1c rcctongular lnminn ABCD ilbout OX t
,\,l.(,
.LM
of 1h~ cl~mcn1 PQ abo\11 the line
. M =J M I'";
··--
4ah • . . •
dxd\' "' - M
4c1b
f'-a [l) • ,-III dx
I"
•, i
/ 2
l
-11

, ;'v/ f' '~


."' .,r•. o,r. ::: -M
- • 1. II~ d:,: = -M b··:., [ x] " = l ·Mu 1. ~
211 ,111/1 •' ,,I 6/1 ll ,\ ·• ~
~
,1° . M.I. or 1l;c rod All about LM . (ii 1 ,1/Jm11 11 //1/l' r/1,:Qugh~ its ~e111rr a11i/y,ape11dici1/ar Ill /1s 1i1(1111'. [
= 1 .!.-x
'vf dx=- -x·1 2 M['I I' ='-M11
I 2 1 · t.~t ON l'•) 1:1c li11c through the c~.n11·e O ancl pcrp¢ndiculnr to 1be plan<:
,,f llitl 1·v~1:11111 u\,1r" l.rniinn A/lCD. · '· t : !
I'
2a 2a 3 · :J ,;
C,111:1,,k·•·, .,:1 •:lcm~nl~r>·· nr~-n 5s5.1• nt II riol111 (x, ,r) of the lam inn. ~
§ ·l.5 Moment of rn;_~tia of n rect~ngul:ir,lamlnu,(Meerul TDC 96 (l3l')J
(i) Ahou1 a line. through its c~nm a11d° para Iles to a :ide.. '. ' . ' ld-'SS ,ii' lh•.\ cillll;,lll!:ll')' nr~n "'p8,vo,1• ...- ~?it 6,1'0_1' .. 0111: I
I
~
~
Lei M be the muss of a rectangular' lamina ABCD ·such 1hal rii,·,1:rnc~ 1,1' iilb: "1,:,11;n1:1h, ,\l'!;'ll l'rom ON= \1 (i + r~),
l\8=2Cl Md !IC=2b:. . .. j
:. M.L ,ii' 1hi$ ~km,111turf 11·rns$ ubou1 ON .
lvlass ricr ul\h area M , , , M !
rectang Ie = p = --· = ON· 0111·= (.,· + r .. ), - 6.,61•.
. . 4ab · 4ab ·
(•
Let OX ~nd t1Y be th~ lines pu.rallel to the .sides AB and BC of ihe Hen~~- M,1, 111' tho rcctongular lamina obout ON
rectlingle th1·011gh it:; c~nu·c C, . · .
M , '
Consid~r an ~l~rnentnry rnip.PQRS nr brc~ull1 ~I a distance .r from ox 4-~b (.r + .r·_)
r-
0 ahd pnrnllel 10 BC . .
Mass ol the strip N ~y 2 (1M). 1, l /r') dx
'
M
PQRS"" p, 2i16x
M .. - ....., .. ,...... ".-Y ...... \
\ ;. · P,t··c . ,\

:: ··- 2uox = -2a o.r = lirn.


• \ : . l• &Y M J +,.4J a) ,.
4a/,;
\; ,l
= ·,foj, ':i
M.l. of !he strip nbout .. ·b:·· ;y_ •. l<.:
ox c \b 2om. [ ' '
"".~[ (a\+ b1):
:I .
_J
(see § 1.4 (ii)
1:1, n~1".1I ON= f (a 2 ,. t>\=J Mei!/ f, ';!}'
A _:

:::!/; l M
-- • 1· = lS - Mb'
~ ' SR 8 Notc, · .:
~
,I • 211 V,
- V•
• 1J
• ,, '
•••M,L ::.mu! Ol'+M,I. nbmn OX. . . . . .,
§ 1.6, Momeni or ln .~rtiu of,~ C:ir,11lar,,yfrc;:,(M,e.e/4.( T,[J.c;, 95. ;~
(i) Abo/II a //1'11!1t'l1'1(:. · . .. . . · --, \;''.;)· .
[i
~-.. ;;
r:
I:
.,.,.,.r,• ·•: ,,.~·· ':"'·•··•·,:•,. •···•r: 1•~f1:j::~,~:~li"'•"-.:1:••;•.;.-... (r~1>~N·•nri1111-~•'IHi{l i~
·"'.
i!
r1:,
~1
!·:
•·.•,•;,.·,·, .. \:::::-':;~: •' :::::::'.::;'.;· . <. "':····:;:;.''.:1>:···•· ' ......
·<~:::·:·· ·;:,,.,.,;•

-----~-~-.....,.,,.....,.,.,-n:rr--r,<. ~~· q.,lh.lRi.'X\i<h\%\.r ,·t!• i,,,~tb,..>.i,;UP;IU;i?.1.~?;1"ii


' I
, • ~.,.1.•,• ••:G• ,:, .-,.::: 4:,0, •' ,, , \' • ·.i I ' ' l •:: ,'1 ! ,·,•,•Jiri!,~!~!i:i~'., ;:,:(•;•,• ,•.;:, ;·•t~ !n: ~•-!;.>!·~•;;,-1iS;,:~:.'.•t:~.' ,'.:~-:.:~,~~~).:;;_: ~;:"• .' \: '.·'. '.•!°:•··•; ' \ ' ·1'• _'? 1 '. , 1
:,: '1:' -1!:> '.~•:, ;,,.;:·. '.\fl:\,.~:•:•~/ 1:i; ·};,';c;,;1,/',\C~:.,;i(:cJ::/Ji:,\/~•;til;•A,;,"c,:,:,,5.o,\:<i!e<>:>::,~~;ti,i;s;,;""'•""k'1<<,l,11,,,,:"'°""l.'ri..,~<i•

·•-•'>!..,~·1,~.1.;~,,:H,O.:l..l1;.*,••••';.,,,,,.,.',

.;.,,.:,i: ,.,;;,.,.,:.,__.,,._~,,1,._:,.:.i,:,~,Li•••• .. ••••"•..-J,..,,,~, ,.-.....,....,,._


6 .',,,,.:,,,,,
........
. . ,., 'j
i
, Dynan1frs of l?/6/dlfody
.I.

" 4't/r1.be , Monr~nts


'

i ' mass ' p,§r, ' uni'.t:


o·r ihe circu!tir Prod~~rs o/ lriertia
i ,. 'P =M~na, , .or ·th:~· . /,;,,r~e, , 2
"/
I ·,. (r sin,$) ,: Sm= ,1sin 1 0; pr 6,06r = p,-3 sin 2 S 6e6r,
., ! ,.c~k~i~~,r ' ~ry cl,~ Hence M,I. of the circulllJ· pl~1e about OX
P..Q·~,lr~•. ~Qr ,l~c"·w.rtct, ' ' 1 ... , '' .· 1'in•[, rr
~r.~~··~·;§:1;;.' ' " ' ' '. ,·, ' ,: /":, ~. ;I =:,
r;"'t<I
· I pr.l sin ~ dBdr"' p ; ~: r .1o' si,n 2 8 d,~, 1·
D1s'th'n'~e-.:*r :,l)is., elcme111
.'i
afaii,~fo· 1P-=·~,11.,. cl ·si'n ~:. :: I 2
4
= !, pa • ', 0" lI (I - f coti'aj de'..':t'pd{[' e'..:•1 sin 2il
,', · MJ.:' o'f I ittlf··' eierninf ab6ut ' t I i

·di~rN!.i,f Ai1, ,. · ,.... ,.. ,· · - ! ·4 '2 . .::. I M - IM' ·2


-g-·Pa. tt-,:;j°"na2 .1ta4 -i a
· .. ·;•ji\ '.\3;~!', 6re·= i2 s/h~ ·~: )i:q:58 =·ga 3·sfo 2 9·; Sa.' (·: p = M!na2)
',. ,'>/ • ' •" • # ~, j, • :~\ ,:,~:, •, .•: ' I
(ii) About aline thr~ugh the cemre aiui perpe114icu/ar to Its pla11e.
·Hehe·e M.!,. ·or the
:)l'/ie of
Leitti,i•tfiloie'.
'ON''6~ .the Iii)~ tliroug'~ the· c~:'~tre iian() perpendicular to the pl one
·
. n·b·ou'r'liifc '-titlinerefia" :-
·
1
211
1,,•; •·•
.
I_ •,: \ • ; . ,
... ,
"'
. 2/t ,·, ·F-:f.t of :the: elem~~!ll!¥. a,rea
abo,ut ON
=l:.= P.p~ siq 2
-~ d,9 r. f pa 3.J · (1 -90s 2~) d.8 ,,, = 'Or .O(li hf . pr5er pl ~9Br. . · =
·.
1·: .. , .,. ,, J2n 1 M
H'ence._Mtd.f;tne· :cifcular]).ate about 0
1
=-2 ,?a.,•.i"9 --, sin
"
32
11 · .:,'.,;:- - ~ ,a , . re
.r.p
•·. 0' ' ,;,ta
2n r'1 j.
f .
= 0 . ,J Op,. d9dr = p
J2~ [ · . 4j/9
'f.
. F
•~;>l.

.. , ~zy:~; .,
,'!,

. . ' 2 . l,
(·: p>=Ml2lt<2) O
I 4 :
, I
.
M . I
· (U) 1oa.u1 ·q lin.c ·fl/roucl1,. tht• c~Mrs arid perp.endicula,•. 10 its 'p/a11e. = -!I pa, ,, [ ,'e J3ll. .. -4 , -·
n,i2 •a •Zr<= lI Ma
4 2 (·,. p ., M/na2)
~"~h~ wire,
orLett,hcQ.N·~!r.c:q1¥' 'l,!p;~ ,t/J,f6,i.tg)i th.i
. centre O a~d ~i:rp~~?!cular 10 the plane § l.8,.1'1-iomcnt of In~~ii oC an ell!ptJc disc: (Meen,it TDC 95 (P)l
,',· hU of-'llie clcmentllry arc· P.Q abo.ut ON Let ii.be: 1Jie
per unit, area of the disc·
,m,i::
of an elliptic disc of u:es '2a and, 2b, !hen ll1:Us
~-q ~ Pq 3 ~. · ·• ·
oit'.ql,=?-:. p~ ' ' '

=
Heilce 'M.1. or ·thc''wlrc nbout 'ON · . . =.p= :::b,
• fo~·.P,;;_~q ·fh ·~L9:1i" .. 0~/ 3
('," p =
Consider an elementary area Sx
=plix13y=om,
at the point (x, )·), 1hen its mass
'
Sy
tl1;1,, Mori'rent 'or Incrtln or n ·c,rculnr. J)l31e, .-, M.J; of. the elemeni.ary mass about OX
(i) •A6ou1' ii iliamti~r. i · 2
= y 6111 >.-2 ~fu-6;•, = .
Let M'be the milss 6r a clrcular plate 0
=f
Henec. mom~nt of inertia of .the elli'ptfo disc about OX '
or centr~ O and rndlup a; 1h~n niass f)' '
pd.tdy
-<I tY
area of lhe · plate ,. p .; Mirra 2, 5x
1
Consider an . ~lcmi:ntar~ ntea
=p(~,. d;,:
r6e6r at the point P (r, 9) of the plate
·referred to the centre Q ~s ~le pole and r...,,. . A'( ·1 db. 1
·X - - ) A~

~
OX as·t~e lnitlal li'nc, . , =' i2 p ,.)'-' d;,: YJII,

·· Mass of the element,. p. rS.86,,. Sm.


Distance · of lhts eiemer\( from
0 '0'
OX= 0M=rsine,
,:, . ·M.i,' df ihc· ~lctn'.?hLob·out 1
ox.
• ! " -....... _,

~~~~l,,!~':!: -~~~'!~r;?~t:1~~~<~\~n~l)>rn ~t1;.~t5>-::-:~tt·t~~~~1 n·:·(~~;~~1~1aM~t~f:-I1r-.;,; ❖;:,~~;.-;'.,!Hfm❖,mfowr.u~:m:miff~!-tm~-· ;~. 1f i-:) 1~;;{~ ;,;,·i1;tr:• ·: ,. , ···:·' '', :w. · :::,:;--'!'trmr0:;;;i'.~!'~>r:.~-!l~'.>'•~:,,
~_,,•,~'/-"'-'"".:•!\-i> ••-•. ~0,.;,,.,.• b>-\. ".-~•-· U..../,H,•. .·, .- ···~·, -,~,...~•.• .. "''.' .,L'..'-1·1•,'•·"

:-. ;-;~?-:.: ~r.;~;;~:~~:!f4;:::;:·•. --' \' ... ::., ;. :,:,,~"oti:1!<!•~·•;~9'-N','•,•;4.;r;.;,,;1;,;,,•1:, ,,,:!.'i-: '••,' .,.,
' .
► .

,,~· '·
of Rigid EtHJy 1 Mo111r::111s 1111d 9
!
.l
:,, i p /)' ' (I - ~·')·Vi d.r
!..... : ' Equ~tiirn of..'cllc
. ;c--
1s - ' ! ) Let O he IIW :entnl
pi1ral J·c l<~rip,•J,
1
i

l'
. 1/l I .,J '
\l'ti.\lllll<:
Ullll
=i p/,.' ·"•;
1\ I .. sin' ,1 en, 01/F.I, P11t1iny x"' (I sin fl
e . M•\'., ·,_ ,vi
- p - -.-•-. ·O - - • l
I · Z11, 2b,}c,• .,8i.1il,· ,
. (~-:. !;,,
:::,:
(/ J·'•',: Od0" ~ r,h~ n , 2 co," OdB Let OX,Ol\ OZ lg the axes thrqtli!h 1h~:~c11•.rc und
.rc,·tangulor .r,riralkloplped.
<11' lhc -
,:
,n ___n;, riti ::
:... ~ ;:,11 ..:..:_._ l:l
·
! ph·1u ..l. ra1 = . \. Cur.si~,11:ar. elementary vulum..: 6.1 oy llt · 91.'-' tli.c Ill th~ ~
r• {3) r,oint I' (x, ,1•.;:);'\hen its 1r,us$ ::

/1. ,
I : ,1 • I(, ; ,1

,,, •. ..!.":!.. . "1;i 11 = 111 Mo2 (•: p = M/it(i/;) "fl 6.c 6y ot"' 6111, , ~· .--··-"••"-•"··""' il
rtulJ
,l
Di~lnnce of 1hc poi11l P (x, y;::) l'rom . · · f!,
Stin:lari)' lvl, I. of lh.; nt\ elliptic di.,c abo~t the 111in~r a~is BB' d 0a 2. ox · ' 2 2 '
I,\ ✓(.y .... ),· f "
· •
2.r. .
'
: l. / , ' ' :J
'
t···- ,.7··.,· .
1
:._ M.1. of tlkelcr.1cntn,ry volume ..• · . X

1
i'.lnu M,!. nf 1he disc uhout the line ON through the centre O and perp~ndkular
In 11; plan¢ · ol· mas_s 8111 at P about OX r\?' y ,i: 7-a- - . .
.:.: M.I. about QA+ M.I. ub0ut OB = p (y2 + i 2) o~ oy Si. '..~.:..,·--·--·--:.,. ·•·
"'"Ml/+!. Ma 2 ,.., 1 M (a~+ b'), . H,mc·c M,1. of the rr.cuu.1gulur ,'.,
4 _ , J ,1 , .
flnrallolopipcd ah11ui OX (which is ::
*·1.ii'; Mom,mt of ,lnertia of a uniform trinn·gulat-· tamiria aJiou't :oi'ie · parnlld th 2t;J, . ·
,idc,,. · . ., . . . (M'i:e'rut TD.Q 95 (P"i) ~f:;
" r f.
l ,, (''
Le 1. M be tHe .mil,~· ~nd h "'AL, th¢ he}3l,1 pf u 1ria11·gular '!amina\A:BC. • J 2 2
p,(y, + : ) dx dy dz
l.c1 PQ be un ~lemenmry strip purnllcl to I.he' Dll~o DC, of breadth ~-(nnd
_,, .i, _,.. ' ,,
:~
nl ,1.di.<lan.,,• ,i• i'r<im \he vr.n~·x A·or lhe lriangle.
From ~imilJr iriangl,,s Al'Q and t\l/C, we ltnvo
.
:
A . .
/1 :: I ' f' [
_,,
)'l'•• + !~ .,1
i., tlxd>·= P f l
..,b
'
2 (n:
• . ,
•1
+ ,),.:·) d,r
• •
tfr
,.::
;i
~ tl
~

/ x

,
I

y-~ ~;.f>· .]b~1, d~ ~ 1,p t'..,, '1:(li•r


, ' +•,,i'b)·tlx.
.'
,\/:\!, "' PQl8C, , 3 l~
JJQ u. n.s/h 1 ·-.1,,ht".rC. !JC_:.: u.
on,·,, of I.he el~ni~n,ury Strip.PQ
lll~l'S
,
I i -~
, "'pPQ /h., P•(o,://1) 0.1 = .1.e. lie (li + c2) (,r)P ,.;, ~-be ib~ + c2), 2u
3 ' ~u 3 ' '
tvt, I, ar the, ~1omen1ary Slriµ .lhou1 liC ·
= \x •. 1!) 2 0111 := 2!! (ii -,x) 2 ~ll.t.
= :!. · ....A:!- · be '(b1+ cZ), 2a ·: p =_M,..;
3 · 8alic · 8<1bc
.. h .. a·---~ c, =3 M(o 2 + c2).
M. I. ol' 1he tri~ngle ABC about RC L ti
,,,r I:,.
::: 'QE r
h (h •· x)·'
0 .
=(palh.).
;:dx' ·f"'
0
(h· .r -· 2/tx' dx· ~,
the
centre O ancl. pnrallcl: · 10 2h nn<l 2c arc [!
2
""(po/I,) 1· ! h :r? •· l hx3 ·r l i , ]'OI""..!.ll pah 3 = !o
·1
3M (c 1 + a2) <\.~~ f ',:ti (a 2 + b1). respcctiv~ly, •, ~·
, ) J l Note : Fbr cu~·;· ~f siie 2~, ·2b = zc'k 2a. · , ..:, , · ":, : • ::
•' .
. , [·: M= ofo.ABC=;o,,(icih)J :.. · M:r: of a cub·e about. a line throu~h i·lR centre and parallel to one edge' ~;
§ 1.10 Moment of inert.ill oi" a/ectangular paralMoplped abo11t axis an =~.vta 2. · • r::.
through its cent.re ,md l)ar~llel to. one of Its edges. . , , · . r'1.11. M.r of'a sphcrjcal· sl!cll (, i,c. l1QilOW: spher~) ~l;_siut (l!~moier. • Ii
·· . . 1 ·. · · :·•;; :· ·•... (r;fc,crut,, TDC ~nJ ji

I~
:)
A .1~hcric>ll shell (!, c. hc•llow, .~phcro) rrl r~dais a Id C<irmeci. by the ~
rcvolul,on of·a scr.1i-~irc·:!lu1' an: or rudius a ubou,-its ui;lm\llC!'. ;.-,,.:;

_ 1 ·!1.,,4:
1
,?:""Pt · J

!
~
ij\S:f1~~:~,~ ,.:·.~:~~~:-:'1~ri!~~~~~•~~;tm~rt~~?mf.ifflrn't~~~Jn'mt~~,1!!~~~!~~i-m~,1~1o.~~vw~:~:ffl rl~••.~•~lH',;,~~w~;~i;·f.~~;·c.jp,·;1~~1~fltfHt}tl'a?flffffl~~Niill,lt'!,,~~ ..:-:. ,I ,.,.. ,ff,.
1
•:-t•,M,,,r,., P7' ,r ~l~j

~
' :. :?,~_:::-: :•·::";·
• •,_::,',~'•;:,:V,•'.•'' ··,,:,,,, .. :··,· ·. I',~'.;:!~~;;'.
If"~
·.~
------------~-~.~r.~~
.'
:::

• i'
·, '· :, i'.•!,.,:,i;,.-.J<;••, ::.:'.·'.!:::- ,:_it.:

:::i
::,.,,,,,i~,~....:.<-=,...... ~....... _. ...........................,.... ~~~
-~;
·t ' , ............. _ _,, _ _ -......_,.,, _ _ 1o'I 'It '11!"1'··•. • .. •-'-'-"-·"···"'""'"· .. ····•·"-'""'...... ".··"'""'•'L~, ...,.1.,'.,.: •• ..-1,.1~~1. •••••• , ...,.. ~,,• .1.,1,..,""·'l ..v\•~t,:l,i,1,u-.. •.1u,,.,,
:/
..
: ."'11 ~ 1 ~
I 10
.
• .
~ors,dcr · 11n (:'lcme~tnry· ar~
... ',. ·.D)·11:)11i~~\ or Riglc'.
·
~ft . ¥omeryt{ and Products ·of ltienia
.' '. ~ .; ' ,• . 11
.~ ' ···#.P:•r1§~·,. .,,t\t,Mh:e, .-P:O)i;l,1·, 1 /:l;1:0f!: ,'tit~ .. :., .. : : · ~ C'j"lre ;of 1¥ ring nnd ocrpc.ildi~ular .to its plane)
il ' 1 ~-M)l~.c1r~1;>l~r-~:rf, ~.cJtc.uJ.,tr.lqg pf.tP.9if1~: ~ . : · =IPM •
2
:. r
~111::: ,J sin 2 .Et. P2rtP •Sin eaea;.
~: · PM, ~ a.s.itl g will be . fromca by' 11)e ·
= 4 l
3
2, Pl.' j~i)j 869.Sr: . . _ , · (see * 1,6)
lj • re.vol.\).'tio'n· of thi~ nrc PQ nbou\ !lie (,.,

': diarti~S~r A~.. .. . A .-. M.j. ;Of :ihe sphere_ abou1 the di.ameler AB
f8·=O.r-., "0
:t : 1, I n
' i/
H
:._•M.~ts .:~f.:..;fii$ ,.,~lc111ci\tl\.t'.y,, ;Tipg ,.;,\, ._......
. = Bni :' p. 2r- RM. aoe. · ,.\. E31 ..
=
4
! . 2~pr sinJ 0d8d.r = 21tp fa 5 sinJ 9d9
.
fO
M
2it .)=·!Mal.
·:i1 =·P ., 2"'•~._a.. ~_10, " ~ · 2.sin,·
•· e· •·a ·o"'l'·=·l"•:'tta · a·-•11•e·.•. , .. =-.~-.as.
,5 •, · 3
(!n
l
•:;1ta l ·

J wh.ei:e p"" 4:~ 2, M is t!ie .mll.$5 of .tire she ti. 1. § 1.13. M.I. of an elllpso!d.
' . .:mi. . (Nfcerut TDC, 93)
/ · • M.l:. (>5. t?/~"Aem_en\:iry -~l~,g ~bp_~t AB • ·· · · . LeJ the' equa·i,011 o(ihe ·eJlips_oid be
,?· l''. z2
\:,1
j (8•. l!P;t !/if.~J~h.-
t!1e ~en_t~c Of ti\~ ri1g nnd perpendicufor to its plMe) -+·-+,=-I.
b2' c-
j = P.M,f; oi_it 12 a1 sin 2 ~. p 2n:_a2, sin 9·60. . ·i, ··· a2

ii 1
(see ··§ \·,6) I '· ,. ··, ' ·i Consider an elcrilelltnry v,olume z
! .~ Z~/)'~~
· • • • • :

~l,ry 'fl6.8 · .
3
. . ., · ox a:, sta1·1~e point P ex.>•, zJ !rnre
elliJisdid lo lh.e positive octahl. C
i MJ. .qf
·,::· shell nb"out lhe. dinmeter ·A'B'·-''"•·::'' .
1~·e
.•.: Mii$s . of ... ltlis"'':·:~ clement
. II . II :, •·• !•

,::
=:if·(l ·ztipaf
..
sin 3 8d0
.
= 2nP,a1 f 0
(I - 0).sJn-8d0·:· :,., p&.r !S.Yoi .
wheri p ,.;; Mass
-
p.er unit v,oJ ume X
,,: IM; .... ' JM 'A
·,;,; =- 2y:ifa1 f\ j
I - . 12) dt P..utth)g cos

9 = 11 s~ "ihnt .'.. sin'·£l'd9
.
=.dt
l• I
l
:: ~ ,= 41tabc • M is the mass
• . '
.• I .
f~
:: -- - 2..,.,,· .JiL. 4
,.,·.,.. 11 . [1- l ,1r·
. -- { M;,J
. M·,
of ihe ellopsoid.
::,. · 41Jfjf ' 1 • ., 1 • • Distanc•e· of'l.he point P(x, y, .:) from
., =·,.·r:-,;(:
Y(y'2_ + ,..2) •
ii
§ Pt~) M,ih of'ii·-$,6)i4•~-l:!1!:er:e.~½'ut·a·4.~.~.~v~.ri . 0 11, ... ·,

· . ·· t1,\~M1!~ rp,.c ~~; ?4<PJ, 97J :. M.I: of this cilernentary volume about OX
A·:sc)j~ spl)~re qC ~dil)s a is: (,~red by :1M re.v~,lliti9·n: of 11 ~erni-cirt:ular 2
= (j, + z2) p 6x 6.:. oy
nrea :or rlitlhls a ·i!b<iui its clianiere1. · .-. i-,LL ·of.1he ellipscid about OX
.~.o.o.~r~:~r ·~n·.e !cf.i:i~p1ry.nr.ea ·,6a6~ al. the point 'f (r, l!J) of.the semi ;c ircµlar
=8 JfJ(>..l + t 2>p d.x d;, di, i
a:,c4:• W~}lp:!1}!~ ct,:rn:eilt )~ r,evol.v,ed, · x~
where, -;; + , + c2 s; 1
a)lout-/h'c tliaiiicter AB; aclrc\l!~r ring 0
a• b· 1c·
the integr_ation being exter,ded pver positive octant of the ellipsoid.
ii(: r,~~iij~; ... h,if·i-~H::f .: . an~ .
, x' = u, '--bv' = ,·, ,2 ·= w
Sr'PSS•S.~tiOi) r.fi,0qf. is f~.zm:~~t r• Putting 2
Mass. cit •.l)il~ ele'rnonlnry ring a 2 2C
I(
r=:6~ =·p. 2ft. r· i.i:n li. ~~f.l/3{:'. i.e. x:;:; a~½, \1 = bv 1•1• t = cw Vi
.,.a.
... sin
= p2A~ M e596, . so that di; i:: fau ·½ d1,1, dy = 1bv' 11\ d1•, dz I" Jc,{½ dw, we have
~.h:er.o P. = ~3 ·• !;f.: ls·:.ihe ry:i~ss M.1.'of tne ellip~oid about OX (i,e. lhe .axis 2a) ·

·P.f,i..:.
~11a
f i.rr,;} ~·.!. or ii;l!vdlt" e.iihi~
'H~=--Pi• ..,,, LI,•• .• 111......... m., •• r
.
= 8. at:.:•? ff f (b 2v+ c2w) u•vi v·V,)V•Vi d11 !fv dw
1
rtilt···*,stS.\iJ £4·#,(!!-: l!n:'=" !n~i:>HiH:.-111:~;
.,, .......... -~.
,:h~.1-~,;

i1:-m~;~)7!:~~,:{"r,~\ii;,HRtm~~,:ir«,$Ri'SRl::..\it•~i!'•ft~nt~f1.: :·?,r:;-?':' r;;;~,;:, ~ :: i~t:i1!~17ftfflf~~tr:,;~1~~r:r~.,,:.,;i::1:,; :.(;)gi'rlftt~ 1~{':tJWW.UlP.Ut!~X ~i\ ! ,:11:.;:; .:rn·!';<•r. {f".:?,., '-~ t, •• : " 1 •:.:v::.~fl':i-:~~i"'iWH'f!"Hai~.:-;~i;:-;:·,Ht>!?;.i:·,;~,;1';,NU,ffUHmHffltm?tt'<~}ffH~;~mH+NS&tnS,l·
·, ... ,,,, .... ,;.., ',',,.':,;:,, ..

f)y1l/lllliCS of Rigid /Jody


5, Circular plate of ra<li11s II and moss M,
= (ll1r (i) Ab•~\11 Jfa dinmelcr ·
a '
,.
I 1,·· f 1,·· 1
I I do d11• rJ J
r·' ., -1 -1 1 ,ii) Ab<>ul a line lhrough the centre ond
l
,· '•,
· .. 1;. ,..
I\'·· <fr 4-
'~ du d1· d1r']
--·~-..l!::!l~lill~~J~~---~-
. ;
whcrn u + 1• •I- 11· :,: I 6,. Elliptic disc or axes 2a und 2b ·and muss M,

--·-Ji)
.
:•· f'i'.-1 rif) rrt, , nil rn rm 1c
. (i)_ About _the nxis 2a ~ Mh"
"'ui;,· P ··-•-·'-·•·-"--·- + t'' _ _,___
. By Dirichlc\'s thcorcn,, , . (ii) Ar.out the axis 2/i
· · rn , i + -1 + i 1 . rn + + f + I j · (iii)' About a line through the, centre an,j ¼Mal

I pcrpendiculnr to its plane .. . M(11 2_____


lJ_.t..!......_ + &2) •
I/re\ ✓ n, 11. . Spherical s,ncll 6f r::.dius- a and mas~ M. - - ·
!.!: (Iii:, · :::::.•. (/,~ + c1). __ _:_ .. __ ,,_ = l '·
+ c·),
' 41:u/1(' !, "• l, ✓ rt · l
. ~ ! !. About a' oiammr .. ·1 'f1,i~ ··-- 3 f
~ !. 1.:, Rcl\:rcncc Tahk
The 1m,mcn1~ <>J incrii~ or some ~,~ndrinJ rigj,\ bmfas considered in §. . 8. Solid sphi:rc of r. adills a und mass M. I .✓Ma2. . t'
r..rw ~ 1.13.:ir~ gii-,,n in 1heJoil,.i"111~ 1nhl~. The students ~re udyi~cd 10 :t· About a diamct~.r 1
r~m,·mh:;r nll 1hc;~ as 1hC)' will he u~,ci frcyu~ntly, . · 1
9. Ellip~olil of~ls 2a; 2b, 7.f .nnd ma.ss M,. j
1 1
i I

i M(b• _+ c2),
r;;·;~:;~b;;;:;_·~==--:_ ____··-··-- ' IM,!. ! , About the ~xls -2a. . ,
Routh's Ruic, All ,the nbove,M,!. ·may ne remembered with the help of
·. ,
I
i
:. !. · Unirorm thin rod.of length 2a nnd M. m11~s .
the following Routh's Rule. ·
~ -li.iw ilwnuih 1h,1 1i1hldle ,,,,;,,, ;nnd '·
M r. about an axis or S)lrni.netr)I 1:
i1s · l ~1a 2 · ).
Su1r 1:
( 1ii). Ah<wl ,1 liM 1hr11,ugh 11nc ,•11// 1md j ; Mb~ j3I :: Mass X of S~U!lrcs o(,r.:.!]:cndi9ulnr3xis
3, 4 llr5 .
,.1:
· f)~l'p~nd1~ul.1r In ll~ h:l!j!lh r ' [ ii,:
T:·1e denominator is 3, ~ tir 5 n~1:1)rding ns the bmly is rcctn11gul.or (inclui:ling
•••-- •• '•••••• . "-"•• """ ••- . --••--••-••••--•••ao-•-•-••••,.i-•"-•"••
l . ••••-••••
. •.,t,.
rn:l); ellir,tic~I (inclu.Jing circular) or el.lipsoid (including sphere).
i:
::
52. Hcci:ini,:ulnr plak of .~idc., 2<-1, 1b :1n<l m~ss l\'L
(ij · ,\limll u line thr0111tl1 lhc 1:~111rJ ,ind 1iar~ll~I ! Mf;2 EXAMPLES' ';t,{
1,0 tl1t! sid'.': 2" ·" '
i'
Ex, l. Fin/ the M,J. of the arc of a circle abo~t
(ii). Al111u1 line J the CC11ll'\\ ,\l\d pur:dlcl (i) the diameter blsecri11g rhc ora i
111 lhc side 2/i 3Ma' (ii) an, axis thmugh 1hc te11rte, pe171a11dic,ilar to itJ pla1te !
(iii)
(iii) a11 .a.xis '
1M(a' ~· ~•) ro irs plwii ~ A

···--- ---·-
R cc(anr:lllQr r,ar'nllelopipe<l of ~dg~s ?,~. "J.b, 2c n'nd
--·-----·-- Sol. Let OB be the diametci·
bi$CCting the circul~r arc ABC ,:, j·
mab M. . , )- !l
sc blending an nnglc 2a nt \he : " . 'l ,, lL
o-i??i':F
. ~. - .. .... .... 'd __ ___ ___ ..,_ .. __' _____ , •---,---··--·'"
. ' c~htro 0. Lei a bo. the rndius u S
:,, th: ..·~IH,, M the ar,. , • :,11
, , _..,,, .,. ,. ..,
Consider an elcmentnry nrc
(i)
Clrcul.ir •·inr.: uf rudiu.~ a rind nm.~~ ·'.,1,
Ah1H11 i1,, dialll,\t~-r , 1: .

{ Mn2 ' PQ = 11 ·oa ~t·t.he point P oflhc l:
'C\TC, ,,(!
!ii) Ah(llll o line lhmu~h
.
...,'·"" . . ..,. . :'.'.~)~~:,'.:.~.:.=~
ih,1 ~~1\lrc ~11J
. -~,.,-.........
'(/• ,,... !
~I ,
!ts Mass om = pn 68. _ j<
l~
,,il
,t
j
i: •"•
:r ~·
-:.
(,,
?:??::· rnmmm~i·.;;,;~~. .,ii:-::~~-~~~~·::mr.n,:1rsy:1::~nit1:::;r:--~rm:~r~,~-~-~•~\'"••·~:~~~,
••. '·' ·:·· .... ,., ,~,.,•,: •,.'.1 i'.·'.{ ~}:-:: '.~i'•'.·'•~:•!•'••·."~{";";,::••:~•·~P?.!t!ifffl~!{~rt~~~~~fl'i~ff~'tf~r~1~• ..-·.~' "~~"'!•'l'1f""(' ~-~-·-~:t?t"i~~i,,~, ..~""t'tf•~· !t~~tn:f 1' 1 H\~,:l:

.,,:;:•{:,,;•~;'.,!:.:,•, .. •, ;\, · -,1 •.,, ···• ";:·:::i;~i.~~;.,"';:·::::· •::~ :,·,.,;., ... ,:,~}::•;:-:~~;-..:, •~~ :·:-i:\'.:;:,;•·,:..;•x.,,.,.,.._ .,,,::,11'.•',..;•>'\•,~·,,..· · . •'• :•.~: ',~:::·•· .1: :~ \: ··,,:-?:•!:.-::=;;:,. "·.. "\~··;::!: ::~:=:· .. ..'l,:,•.. 1

-.·, .., , , r. •••.....}!'i;'i:i'.7;,l'i~·.• ,:,J.'~~.9}•1•.-.i!❖~


,,.;. ►.r,w;

f(
·,~.,
•::❖: .•. ·•.•. , , ·77;"7":T~•~,,
' ~
·•:.;,,;,:-,:·
. . -;.,':!, ; .. '.\}!;•::, '. ,,,,, .,,
~-j
<{',;

........... ,, •• _, ,_,,., •• - . , . , . , . .. :..:.....u.,,,, ..:i:;.;;.::,,._,., ••• ,:.:.~_;.,1:'..__,;.•1:•,~-.J::U.l"li•


~::!, ...,,.:~ •. ~, !..,:,;,, ·~ •• ~ .. ~, .. :::.-..•-';.· -:1~1..:u~.~
"" ., ,,1., s;l.£..JJtl,,.l/, , ·•· ,.,.,,., •• ,,,,,,\,,,. •• •"n""'

1:: 14· •· i ·•,·\:r- \'\ • l',.i :,_'<;. ~·


p;J
!~~;: -D)•namfas- of Rigid-Body
• i I: :••,1.tP.i,' t,.;:,_i; ;.,.- -~.:.,.f{•·•. : ·1,, ~,,.~ ; Momcnis a1id Produc:s of lnerlia
~} ;,i.,herc P·=:.'·ma'ss per unit lcng\h of th~- ore ' ,;• ,. --·:.. l5
J.. ..!1_, M•ts ;he mnss of !he ci'r~-;'ABC ·.4, · '::,;1:•, ',' •," ' I .. , '/
,
Consider nn elementar)' arc E'Q =i:i'M
·•,.ii,

;,,'g;;;,," ;r / ;rom,,i,.,,.;,,,,gs,r '11!1;• ,,,;,~ /' ,: ,,"'. , , ,,


ai lhe point P the wlre. or
,', Its moss·= 8m = pa a8 .,·.,9:: '.
u{~lhe
:,•, M.1.
2 arc
clomcntnry nbou1 OB· . ,:. , ,, .
2 where p' = m~ss li!.;
:\: PM3 , ~hi2 f.'{O ~in_ 9) pa o9 ,:'/ ,., · ••' .
.per unil length = 1la
i.. pa sln ' 9oe. · · .' ·, ' ·!: ,' ', · ,·:: .,.: P.l.1 tlf . th;s elemen1ary moss about
,•. 'i.ii.r, ofih~ ·a;c Ale ntiout tne· di:atneier":OB' OA and OB= PN. Pl. Sm
;._;

fi
= r pn
-a.
3 2
sin 8 j9 = ipa3 r. (i ~- cos'.
-o.
'r8).d0 .. ·
'
= a sin e(a+ a cos 9) pa o9
= pa' (sin 0. + sin 9cos 9) 89 0 N
:1
'u
:1
:= 1paj [ a~1sin 2e t =t {£;.~~ [.fo- si~ 2!r.- Pl of the wlrdbo1.11 OA and OB
1

=J>~ (_s/n e+si~--~ cos_ 3


0) d9 = pa 3 r~:c~s e.+ fsin I 29
:\ ·Ma2 · · ·
:i "'1ci"" (a- sin ci cos ex), . . .. . . . · 2Ma 2 .. ,
~
t, (Ii)' Disuinee of. the point P from ON, an ai,Js throogh th.e centre and
= {2)=--·
. 1t
pcrpendiculor.,io.' 1rj~.. pl:iii,e .of tfi~ arc =OP,=·;,;, , . ·
li ... · "M.'l. of the elcrnentnry m.iss. 6m al P about.ON , , Ex. 3. ShO'W that,:(~ ~,!. of a. um\c(i~cu/a; Jla111i110 abow atan~fj:_i
' "' a2 • 6ni = pa 3 60 · parallel ra /he boun.d,-'./1? d1a111~1er is Ma
4 - 3n where a 1s 11!, radws
;, M,r. of the nrc ABC nbo:.it' ON=
.
r· •CX
p,i3.d8::: pa 3 [9]~-~.'
' • .
and M •is the mass of lamina. [Mee rut TDC 90 (P) 96 (l'))
s·o!, L,et lN pe the tangen1 parallel to tho bounding diameter BC_ or a
semf •cl~c/i'lar lamioa of r.adi~s a an.d mass M. ·. · · .
cxa n1 /20. ... M.,:62,
=-2·M · Co:ns-lder an eiementafy ma rM or at the point P of 1he lamina; then ·
ltt mas's 15m = pr 6~ 6r; · ·•. · · · ~
(iii) Distance of the point p l'ronj. SL."tt~ 'llXif through the middle point B Wh'erc ;p ;!, Mass ~r unii arr.ta
of 1he arc ABC ahd Pi\[P~n~Jcuj,nt' to _lts plane . . .M, ?.M ·
=PB= ✓(OP + 08 - 2{)P, OB cos 8) =.✓(a 2 +a2 -·2a 2 co~ 8).
2 2
= !rra2'= 11a 2
I . L
"'aY[2( I - cos 8JJ =412,2 sln 2 f 0) = 2a sln i e
i

Ii Dista~¢e br the point


P from lN = PT
r
:. M.I. of the clcm~h1ary mass om at P about Bl= PB 2 , o~ =J(A'.; OA - OK= a - cos 8
= (2a sin {9P, pa_ q0·~· 4a 3 p tin 2f9 69, .. ., M.i. dr lhe elementary' !nass _6m at t
P abou1 LN · · ·. · A
M.l. of ihe ~;/ABC'iibouf-'Pl1= f 4a 3 p.sin 2 f0. de· = pf- '. 5~; = (a ~; cos 8) 2 , pr ae· 6r 1.:
. ,-0
,', M.1. of the. lnmina allou1 lN 1· i~

., 2a3
· 2Ma2
pf-a (I -. ~oi 0) d9,. 2a 3 . 2:a. [S - sin 9)~ ~

,. r r
9:::-:...½
2
,~o (a·- r :os 8) prd0 dr I
;
I"
l!l
1
1:,,
ri,
ex -(o.-s1ncx).
=-· . 1 ,;
=p J,•½ f0 (a r - 2;:i,2 cos 8 + r3 co's
2 2
I
9) 'de dr
Ex, 2. Find 1he product of inertia of apemicircu/ar wire ,about die.meter
and tangent ar irs ex1remi'ty. · · · · ·
Sol. Lei M be .the milss, a the radius nnd. b11 the diameter of a
semi-circular nrc, L_et OB b: the tangent a1»tho..~11.1~~mity O.
., p Ji [£.: ,;·~ }ar' cos 8 + ¾r. cos
-½ 2 '
4
·
2 e I
r20
d8

I J.ti:

~~m~~!-'.11.~fr~~tJ~~\i~~,t~r:w~n-:ti;f.~t'::J~-~rrtsP.~;-t·1~~"':'"'?~'.'·~mtfi~~~~-::· ,.}'.P.fN•:/:-·:t::,~N.ts!~~fflM\~-g-r,1001Mrit~mr-~·:nj?Hn-! ~,\_ 1-•: ~,1!1·;· ~~-, , · : ·· ·:-:· n:-:: ,: :3 ,._~; i' ·: : q'.:.'tt!:: .;1Hrr1H~1:: f1.'l};-tr:>:tf, q:,., ·.: ;,, .-_.: :-· •i: -:' 1H1Nftt,i'M'9n{9,i--.H1'3~~~~t},,'i\~1,,;™RH:-!n-~uwm,Hl!'lH,mJ«1"l)(lf.1r~Hrnc, :;,,;.t-1;,;urwm.
\I'•',',' J}~•:,. I'•'. ,:l't'.•: \'J, '/,,', >_>,,,
,,•,1;;..;:.i;.;". .:::1:•; .<·::·

=µ <:t>S 8+ co,l e'j d9


J =1:, 2.p.2.5sx = f py 3sx
0) dO '
:, M.I. of the whole aren OABO about OX= .,
. f" t dx
,,' 0
= .
:!) ": ,Ill) (j,
··, ;. !
r·('.) ru l
_.,__. I
"l pf.' (4ax)Jl2,d,: =~ a 312 p ! lt~I.! ,:d (f p a 11.! h:v:!) ah=! Mah.
' ' ·,, I' 2r (2) 'J I .o· , l S S l · . l

" 2. a·,, ·f 'll - , -,. · ! · -4


·1
n']J = Mt1!
· ,(s s }
I -~ - -n
J
· Ex. S. Filld rhe MJ, of the area of Jhe lem11iscate r 2 = a2 c.os 20
l, ,,
(i) abo111 ils axis• [Meerut TDC 96 1 Rohilkhand 83]
L . , ii \ .t (ii) abow a line through. 1he origi11 in its pla,N and perperuiiculat to Its ·
K~. ~. Shri,v tlim 1l1a Mi 'of parabolic arec.(of !arm rcctw11 axis. (Meer)Jt 90 .(~)]
"if b,r "" ordir11:i1e m dis1n11ce h fro111 1he \'e,;1c.r 'is Mt, 2 1ibo1.il tlie f (iii) about a line through /he origin a11<f perpendicular to lts plane, .
Cl/ iii~ \'<'!'IV,\' (Ill(/ f M (1/i µ/;(}11i' J/1e mis.. [i\-!c'~rut TDC 90, n (P)] Sol. The loop 'o( the lemoiscate is formed between Ei = -·rc/4 a'nd
Sol. Let 1h~ cqunuor, or llic pnrn,bcln Cl' lu1us ·
e = Jt/4: The curve is ns sho.wn in the fig. ,.,
'rc,·wm .1.1 oc ,.• = -le1.~.
, ' Consldet an eler:nentary area r li9 or at the point P (r, 0) dr Ille· cu·rvc,'
then i1.s mass 1im = pr 60 or
I.el (),I Ii i1~ the por11nn <11' pnrnhola ,', i:he mass of the whole area is given by .
,u1 off ·b)' ,11, <JJ'.Winn1,;: at ·,1
ll\~ \'Cl!t'.X.
Ii fro,n
M=2 Jr✓4 '' f a✓/cos 29],1:de dr = p J- 1114lt/'4 'a2 CQS. 29 da • '
Con,,idcJ' nn 8 a -1114 r ,. 0
stnp PQf/S of
O,\', r:1rnllcl IQ x.h = pa 2 [ 1sin 2e ]~iJ4 = pa2. ' ... (t)
. ~
1\ilJs~ of lh~ Wip. li111 =p. 2,r o.t', where 8"11/4 ·
'

(i) M.I. of elenienl.ary mass T,


p is die nrn,s per t:hit nicn. X om at P a.bout the axls OX
:. M = i\foss or the portion OABO of the = PN1 • lim =(r sin 8) 2 p,'ae Br
p □ rnholD
,I, = pr1 sin• 9 69 Or,
-=J 0 p 2vd.r
• ,·, M:I. of the .lemniscate nbout OX

.,: 3p J,)
J,
d,1·= 4p Vu.
=2 J
ltl'4 '
ec-,v4 · "
'
f
a✓ccos 28 ~,3 sin 2 9 'de dr ·.
1114 e... ,,;14
4
.\'r•w, 11,c Jis1:u1~,: ,11 '"'Cl')' p,:,in1 nr 1hc ,t1:ip r,·rJm Dy; the tangent .ii th.: = 2p J 2
, -)a cos 2e·~in 2 e de
vencx. •~ "'t ,. - ltl'4 ' .
21'ln4 J' r,14 .
•·. M.1. or·,1,c ,trip ~b,iu1 Oy=xJ6111=p2.v2yo.,·:: = 2 • ==- l cos·• 29 ( l - cos 20) de
4 0 l •
M.I. (If !he whnl~ :ir1'Q OA (/0 nbo111
.
o,,"' r' 2p ,r'-1' di'
. J() ' ' =~ pa
4
J"'eos 2
1(i '-COS I) dt, ' Putting 29=1, so dS -..l1 di
., 2p J,~ ,(" 2,'(a.v) d.v = ,Jpc,Vi
' r.· ·.r~ll d..;., ! pu.11.l
' h?l.l ·.. 4[o .

= ~ pa f f
tv.? . ]
(l . (J I
cos 2 t dt - >t/2c'os 3 r dr ·
., ~. (-~ p(/11,: lr'l.!l hi ;.,, •,I, · 0 · 0 ·

l
I

r(f) r(!). r(:'.) r(f}


]1,
' ,\ f
1 I lt 2
=' D~ ,n,,j -- 2t1jJ
om 1
I
M.L 01· th\· sll'ip f>QR soltHII OX= Mq' [ l"i from (!)

"' ~~ (l't -!/1). >t


·I

'
I
: \.:;
f
It' ~- ' ••

, ·11 l t , • . l
I

'i.';i:i,:i;;:., '' .. ,...... ,,,' .~ ... !.. "' '":~l!fi~m~;~1!'.:·t:fY.~X~~:·:~::,~:1rm1:1;::·:;r,;m;:;1>!,~,,,1~c\~~':':~·m·,11;,.. ~,,,.,,,,,,,.,..,,,, ,.,,,,•.,,,,tr:'~i1••:-••;..",~'A,"":'~?l',:''.;,~~1'ffHt9,~f:i-l!tt~µ..Ht~~~t1f;'IJ!)'~~:""""'""'-:'~\'IO,:"~}~~~~~W;..ll'fflll''~''"'~"i~~~~~~1
I '
i • '
" • i " •

' "\ 1.' :::~ ~I'.~• '• • t;-::,.<:;;: .. \,,,,.,. · , ·'•5!.l'.~f!.H'·' ·-· :,., ..
· ':,, :::i:::::::~ ,,;,·::,:!,:"• ·•:-:::•t:'.i'.':!;- ,,.,:::,:::'!!·?:·. . ·.''.>:r.!1::;•.

--------=-==cm-T..~-~-~..-..=··~~
:::?.::·::;::::-: -;~!};~·~ t·:·:.;
~~~--: ~~ -~----------~--
·: . .

; :: ~;;!:,.'.. : ;:: ; : "


;4
fJ!t::;4
••• , ... , ..... ··~···.. ,,, ... - ..... ..-.....~ •. i...:.:J;...,~ • .,..,,.:,.:.•.·.:.,,:.:, ,:..,i~:.::.\L.!.o:J.:o~
~iJ ~;
tA
,;,j )
rn . ... ~,. ·~ ... ..,, ... ,_,,, ,;,:,,'
~H . :j ~
. Dynamics of 1?t'gi'd
I~
(.~,•i (ii) Distance of.the point P (r, eJ from OY s Jir,e thrllugh the origin ln (he Momems and Prod11cis o/ !nm/a
or the lemnlsca<c and perpendicular 10 its axis ,. PL= r cos 9, · 19,
r? of om
at P nbout OY ,l . =,1~. p411.r-t·S.r = ~pttx4ox. _
i
;;f
'• :1 ,. PI). 5m .. ,.:; .cos 2 (l p2 ce 5r = p,3 co~Z e BG Br. '
:. M,l o( the ·iemniscate nbout OY M.!: of 1he given hollow sphere abou1 a clinmetcr
;J
,J
:~i
~u
,/I
=2
·
2
t'!:, j tt'l(~oilep,-' cos
e.Q•n/4 , .. o
l!/4
.
.
28

.
·
d8 dr
= J/,)
"!pttirf.r = ~pttl (a 5 -
. . •'
3M
l

=.!tt . ..:......_._:_, .
__ . ( as - bs)
b5} ,.
i
t:\
:~
.. Yd J · aA cos 2 '28 cos 2 ede
4 .~,V4.
ii . 4rc (al -bl) ,
;, 2M i;S -·!JS
4
i~
,\ =~ 2a'\f l'.1 f co~ 2 29 (l + coqe) d8 "'s: a3 -l;r · .
:} 0
• 1tl'.l. . . E~.j1.)Shaw thatJ/ie. M/, o/_a parahoto(d of revof1.1/(an abo111 i1s ,ix/,·
.. ½Ma 2 • ½ J0 co~Z t {l + cos t) d1, is M/3 >;·the square of the radius a/ //s bas'e,. , -
·- ·_ (rc
=~ Mal •4'+3
2) . Putting 28':: 1. Sr,il.! Lei the prabolorp of revolution be gcn~ratcd by lhe revolution cir
!he area! bc>11ncled hy i~e parabola y2 =: 4ru\ dnd X•/\~is :ibou1 the nxi, ox.
as in case {iJ l.e\ b bd the radius of iis. base, · · ·~.
2 I
= -Ji Ma (3rc + 8). -~ :. tori the point A, .: '
'y=A.C=i b
(iii) Lei OT be the lin~ th.rough the origin and prepeno1cu1ar
to 1he p_!ane
of lhc. lemriiscate. · 4ax, ,i,:.,

Distance of O~l al p from or .. OP= r bi:


,.::--=DC.
411
.... M.!. or 6m ril ·P about ot·.. OP2 • Sm= l. pro Oor., p,J 'oe' Br ~o
· :. 'M.!. of the lemn!scnte aboui OT
: ;. 2 J ,v4 J n'(cos 20~,3 d8 ; = -~4 Jri/4 ci4 C6')S2 28 d8
nrau ih O)I al the point· .P
.9•-ft/4 , .. o , -rv4 (;:, >') of: th~ area OACO, . · .
4 By 'the rev61u1ion of thii.--
"1 pa4'. 2J0l'/ f (I +c(ls·4e)·'d8

"'i Ma 2[e+¾sin. 4el.kl1114_=fnMa
.
2, area ox 9>' about OX, a ~ircufi'J'
ring of; radius y and area or .'a
Ex, 6.'F'/11d rhe·M,J. ofa lio//ow sphere about a-d/(lmcrer, its extemal cross•s~tion 6-l' 6v is formed.
<Jnd inrema( radii' be int a and b, mpwive(y; (Meerut ,TDC .. 90(P)J Mass 0(1tiis cle~eriiary
Sol. I( M ls/the mass of the- given hollow sphere, then mass pert unit 6111 ;. p2-rcy ·fu· 6y.
volume . t- .. . · ._. ' . .
where! p (s the mass per unii Volurne.
p•-· 3M 3
(i1t(l
J
SM l:lf----,
..; !nb )• 41t (a 3 - b3)
.·. Mnss of the paraboloid oi revolution
h~... · .Jc""-':> . - . .'1., vl<1a,i · . :
r:,
) ) I .
Consider . a concentric ·.spherical M= f:•: •t· )'" 0
J: p 2rcy di' dy,. ~np,. !
I
fO . [;-21\,
.Jot
.d.t .
shell of rndius'. x (s.t. b < x < a) and 1 ''"' n b4
thickness &x · = ;,.ia ·
4a.rrfx=4rtpa,[ix 2 ;·~
... (I)
Mm of lhis elementary · shell
., am"' p. 4tt.t1ox . ~ ............"....... ,. Now M.!. of 1he elementary rin~ pr mass 0111 nhou\ OX /n
M.t. of thts shell nbout a diameter centre and perpeQtlicula; 10 Hs plnnci , irs
2
= f.r2. Sm = _y2 tim "'y . p 2rty /3.r D)' = :!np ,\'l .OX O,\'
M.J. of !he parabolo/() of 're\'Olu1k,n ~bout OX
..... ..
., ,~

},'.,'.~

~~JW:ti~~8~~~UW?~mN~!U~mE1ffltffl~l~UL1~R!;{t;,!~ni'!1fm:mrrr~.lf~llmm{~l5lU~HM1$~!{'i\t~i·¼r',A~fflfi1Htis-!!rnrtf{9ttlmir-Hlf~~fr\,'\,-:•::l!W'f:.~.~:~;;:;:·;
, , .. .-.:, .. , ,•..•: ,,•., •• ,•,<..l.t;-. .i;,: (1~,'.·'.I\ ·., ,•, ·'' ,,,
.• ~·.
, . ";. '

.,

20 D)·11amics of Rigid Bod)I


Moml!111s and Producrs of /11err/a ll
l>i'vir-:1.111.\} • ,lytCtJtl/,1,1/ 1 ":,• I
" J , · 2rtp .,..1 d., d,r = _c. 16,n,· d.,
0 \" o .: . :· · 4 O. 5
, 'n ~& = - \I/~
n )
· s
-fa- ."'. 3 cos e"' 3I .cos 30J,y
z:: .c:::
Tt a [ ·
=
~., I Ja)1= -24a
,
I ,
D" 6 1 1, 4 ·
= 81Cp<!• , '-• // 2
·• ., \. 8a · ,no·a~ [ 8
= j M. ,sc1uure of !he radius of the &a,~)., . .= f-fo 3 ,;. 3 cos ex~ j ens Jo: ·1 J .

Ex. 8. Fmm o 1111ifon\, sphere 1;f radius a, spherical secror of ~cr'1icc,/ !las l
angle !a i.r rc.11101-ed. Show: 1ha1 rhe M./. of the re111ai11,ler of mass M abo111 "30. p [8.;.9 cos o:-(4 cos· o: - 3 cos et)) i
1/te i.Jxi.i of n·m111e1r\' is ' ,t ·. .::na 5 · ·3 .
.. . ! Ma 2 ( I ; cos et) (2.., cos, cii. = JO'" p (2 + 3 cos ex - cos et]
ScJ. Lei the spherical sc<:1or OABCO ·of ver\ic:ll angle 2a Lie removed
from the :,ph•:ru of rnd1us a and centre O. This may be gcneia1ed hy the
2na~
= - -,,
.
1., 2na 3 1+ cos a)
f
M , (. .t · 2
·..-----~ . I -liCOS 0:) (2 + COS 0: -cos 0:)
.
.•
,ffrom(l)l
rt'-'olut1011 ()f th~ area OA DEO of · / .
111,1 circle of radius a ar,d centre at = jna 2 ( I + cos o:) (2 - cos 0:)
0 :lbout the diameter EB. · Eli, 9. fi11d1he M./. of a risht solid cone of most M, hci'glit'/1 and
Consi,ler an elemen1ary area radius of w/rose bate is a, about' its a.xjs. ,. · · • · ·
rl$0or at 1he roint! f' of this oreo. Sol. Let O· be lhe vertex of the rig'lil solid' cone of.mass.M, hei'ght Ir
By the revolution or I.his elementary and radiµs lo{ .wnose bose is a. I( a is the sel'ni•vertical antle; ond. p the
urea, abou1 F.!J\ · a .circ'IJlar. ring of density ohhe cone, !hen · · ",.
radius· f'N = r sin ·ti nnd area of M = fnpJ; 3 ta:1 2 ci. .,.(l )·
cror.s:ser.1inn roO or i. fanned. E
M~ss cf 1hls.. ~lc~erHary ring, 6. Con.sider an ele.memery disc PQ or A
thickness· llx, parallel lo the base AB ~nd
Sm·-= p . 2rtr iin e. ,·51:JS, at a distance x from: the vertex .0,
sine Mor, ,', M?SS of ihe disc, Q
.-. M~ss of the remainder I. 2 2
()iij = pn.x flln o:oi. '
M"" f n
~
fn r:sQ
2n pr" sin 9d8dr~ 2npa
3
3
fa sin 8p0··, M,I. of'this elemenlary disc about axis
OD: l . , . ,. 6
i
= omCP'- = (pru 2 tan 2 oox) x2 tan 2 a.=½ p;u~ ton 4 o:6x.
1
Z.!:£ 3 ·_··--:::-·_3_M_
= - 3 a ( 1 "'cos o:_) :, P = 2iu:.i 3 ( 1 + cos a) .. . .. (I)
:. M .r. of the cone Qboui axis OD.
Now M.l, of lhe elemcn,1ary ring about EB, the line the centre and
lO il~ p-l~rr.c =Ji,0 lp1t x4 tan 4 a dx =p.E..
,l ' . (0 ·
h' tan 4 o: =··J..·M /12 tan2 o:,
Ill · Crom (I)
.= PN~ . Om=? sin 2 S : 2rcpl'.' sin SSSor"' 2rcpr4 sin 3. 0 58 .Br 2
.·. M.I of the remainder abou'1 EB (the exis of symmetry). = ~ Ma • (·.- tnn cf= IV/J)

:.: J
j a 2n pr4 sin 3 8 d9dt:=.i rep a5 f Ex, 10. Find the M.I, of a truncated cone about its a.xi's, radii .,1,;
0ut:t
tr
1'1.1'0 . ~ .·
11
Cl
~in 3 edO ,.
I
of lu ends being a and b.. ~ [Mcerut TVC 93 (DP)J·
Sol, Lei ABC'D be the truncarcd conq w/rh the vertex ~I O and o/'
= i rrp as j rr J. (3 sin 0- sin 30) d0 scmi-vcrricr,/ angfo a. Also l~t O(B"' 'ti one/ 0 2 C"" a,
J t:t ,/ ·• .
·~ :·' ' Consider on a!emcnrary 1vip pcrpencjicular to tM 'axis at 11 :dlst,111()/J ;1·
from O 11nd of.Jhkknc.ss l , ar. . '. . .
!rs 'Moss = o;, =pc (.1: Ion a).iar.
--..•
, '.:.

,t
.. :;:~;;~':!=~ ' .

1::;;(::·~ .· ... ,,:· ... ·-·~:::+:'.:•:: ',',• ;;;;: :;• '. ·:· 'i ;~;. -. ::::::}• ,· ,::,' ,;,;',, > '' :·:·•::?:::;~:~;.:

~ - - - - - - - · --..-,w-,.;,t. ,J,iJ .lji#>!,%*i,(ib'.l•i\i',,.,l ... iJ!~'.'77TlW~;y~

·,/1~:~;,:>:.:·
,: .,/,/'-~-i~:~ :'. ,'.·::•'.·J< .·
•. '"',
··<1· I ',;' ~ ,i, \,: t ~ '' · '

,·i
~\1
ii
:(.,
- - ~ - - - - - , ; ..... ~ : •• 1;.;.:1&!.~!,\,.;:,,, ..•.• ,,

·:.:
..-~ 22 Dytwmics ti Ribld Bo~ly Montcnrs nnd Producu of ,;,enia 23
:-\""♦

t .
:

l:' M ls the total mm of the truncatid


"·:1 cone then '· =½µa:1o2c1 Jj j 1
uH . v - 1 'w_H au dv dw, u.-t I'+ w:;; I.
~I -· · fa COi 0.
M= · pru-2 1an 2 C( dx l 2l2 ~ f(t) f(!) f(I) . ,,
a "'g µa ~ ci f(l + I + I + l)' By Dtrichlet's theorem
': 00(= b col ex, 00 2 = a col o: l . :. : .
- ~ 11·,,2bl~l \
- 48 '; · ; . .
= l pn. ,~:n2 .ex (a 3 - b~) cot:I o:
Now M.{l:, .~r, the elementary mass 6m a1, P, about 0X
t
I, .•. , .
,.. i p~ co\ CX (al 1'b 3). -cc-· rc,;·~~·c, I = (l .;ii) .,sm
\ • · · · 3M tnn et . , ·:· Di;1J:nc~ or P(x, }',Z) from, OX is '1(.)• 2 1-x2)
!
l
~ .. ·.··· ,' •••:•~-~•rl(~l-b~). . ... (It .
= ~>;" (5: 2 +i
·fu: s;, 6z .
2) ...
! 11 · ·' · :._' .' No:,-:- .M.l,·•tif 1hc ciemotnru:y dist:· ab<>ut• ;, M;r._9(ihe:·octan.i·~_r the enipsoid about:OX
1 ·i .. \/; .o,io;, .•a)llpc.· thrciugh.'the ·centre and ,V .' , ·. ·•. .'. . . · .
i
i,1 _· . 2 2 ·
:·.,/ Jl.qt•(.l'~ + z3).~ dy·d:.; 1· ·· whe~c. "'1 +.~!f: \ ~ 1·
}
r
·j
·: · -~eqicrii:11cµ1~r 10
·.......
.· .. :L:, ·, ·,. -- ..-,, ·6" , 1
,!"·.i''."',l~na-,-, 1>1"'i 'u. ptt-c'- \un 2aoi- I .: ..t
' , .. .. . . ... · ·""·' . a. b c .
.n .oeing ,l!Xf~nocd over t.hc p0$ilive. octant.
"'L,.,

Ii '
· ·.: · :::t:p·1t..;4.1an4-a6.r. ·
'' . ' ' .... ,
~ . au~: 'I =:b'.fa, ·t. =cwlf', so that dx;,. f au·'-1 du et<t
,,.»

·' ,.,_. :,:MJ. oC_the•tnm~ated con~ about its axis 0 10 2• M.!. ='f µ,~z2b2~2 ,Jf.J ~'/2 v~ ,)n c u·Vi ~-•11.:1v-½ dududw
(b 2v 1- 2w)
• · ,. ·. Jd ='faco1a··
: ·· ¾p~4 tan· 4 a. d.r::,- 10l pn (a 5 - b5' ) .cot 5 a; lan 4 a where u + 1· + 1<· S l
• ',1;1>b,co1Cl·' · · ·. ·
I
;:·1' 3M'.1~n·Cl· .. 3M.·aS..:bS :: 8
" . •, . =·-,, 7 " ~ 1t (aS - bS) col a:,-' ......,.__c._,' from
••• •• • . . .
1
J0· 11.{al_.":',/i~)
: .. ; ... .:•,- ·"t ,, . I_O aLb, vl-l·wl"'(d11 d~ d,v+_c< JJf ul•l v1.- 1: wH du dv dw]
·. ;:,.;. ; ':)~~..:p: .(n) Fil)4'tha,M;f,;~i'f!,q_i,;i th~ x-ax(s of rh~ porlfo.~'.,~[:llfpfyif'. ..
·.,x~i.a~.-+ ·irfb~ + t2tc 2 =· 1; whkfl'lics ill (he.positive octa'n(,' supposi{l&:11.;,Haw
. · .• . :tpk'v.a.l~.~i,e lien.tl~,'ro,.b~ . P ";" ~~- · . · · . . ,: ·.-. '.'...:. ; .>\. ·. . l
"'s.iut
\k-~.;[ "i,2 ,•t(l)'r-'(2) qi)
.. :.::: ·.: ·.'f'{·!-!'.)-+,2-t-·IJ+c
. 1 f'(l) f'(l.) f'(~)
r(l+l+l+2)
) .
~,,:/: ,,'. :,:.:,'S~.l.,i:~~i(er:,5gi6t §-1 .1~1.on· page· 11). · . • ... :. : ; ·. ;, ·, . . · ~;.: • :.' •.: t' '. ,.._ . '
.. ·.,· "· .. ··:.Oon'sJ.clcr·-nn·e1emc.tru-y•,101u·mc ·ox·6)"8z at ,the polnt'F(x;-y.,z)·\.wb<mi,,-',:
·.-·,:·;··.i·:·;:P:~·Wt;z:·/;·.: · /'. ·· . · · . · . . . ·, •1.-,:: .·.--::·~-:·: ... ··.:,·r:::;.- -~ .,iJ,\y(b~+C:)·,t;.'=¼M(b_2+c2). By Dirichlet's theorem
: ,: _..: j,.,:·;;,: ·:'lyi~ss 'd(t~isl~lement = pox oy-6i
,,.·, .~.·,,tM,;...M11ss•.or'th<i' = Bx oy oz,·. ... .
, . octant·"''fff. · ·µ.x;,,z µ.xY,z .. . .
C,
(i:l) ;Sh_p,w rhat. the M.,{ cf an -ellipsoid of ma.rs M d~d semi-a.r.e.r a,. b.,
wii/1,°jegard ·to.. a' .ditimetra/ plane who.re cJ.ireciian-ccsine.r referred tn
: :, dx d>' dt,: · · . .'·
y _,.. -_;,:::·:,-'.·1 ··. ,.'. i ·.•. • . . • , xl . .2 . ·z2 .i,' princip_ai pia:,;/s ~re_ (I,,;;, _n)Js, 1M ta2P+ t/~2 + c2ii2}.
• ... •• • • •
, . . . , . •· : • 1· .
. where
. - z + L1/ + - 2 :;; l . Sol, . °Fro,rn·§ .}:12, on ~age'('! I), the momcnu of lnerLla of ihe
• .': • ' . .. • •• ' • . ·, (I . (; with '.reg_nrcl.t'~.;1h~ pt,ir,cipal axes a;c ,
·. •;1r1e)r.itegta'tion ·•b~lr\g, e'xtended ovor the . 2 2
lM. . . ... t } . ' • t lM(a
s (b2~\l):-iM(i-2·.+r}) s . +b ')..
··: ·,·'?Ji.· . :i,_'~; t,_ '
:: .,. )'J,!t~l~,jt-:{"·U"'l·-11, . 2_,-,\1
..... ::' · ... BY, prbp>r,i\<1(§' l:3 ·on page (2)', ~e momenl.s'Of inenia with regard
. . . ," .... i, ·· ,,a : b " . . , 10 pnp·dp~1:t;1.ari'ef·are
.,, ·. ·: .. ·, f. . . l IA ·V
}.::i:1".'ix-:=·au~;,)• =bvli. Z·"' cw", so that d.x = au l du etc: i .l 'i &or,i,Mc2
M;; 2.';f .. .
:;:'"H._:n'1h
;,..•; ' _.. ·2'.J.J· J'•uVl vYi wV I I I
1

~I
u l"i,
.1
w Vi du dv,.dw, .... , ...
·. . .
l ·. ,,:,''l. ,
;,· M.I~'6f_".lh~iei!Ns.6id ii~out the cliam·eml plane whose d.c's referred 10
,Al. . .. · · . . .. principd.J;,-tiia.11ef'~e.i.J; m,n is :,·.•;
. . . cw her¢ :\1 :::i-.,v • t . . •

:", ,., ' ':.-· .:

I '
~~,'..~-:\l~l::);!ffl',l!l:(<::).(Ul!mll!lJ!!ll(l;lls!l'!iS;t\\-:':C·111i•'f,';'rn:lr1ll!llffl!Smlf•lll,llm:,'.·~ ;\~! ){,,f!fllmffli;jll!f!ffl'.'{l"!llmlW-'ll'm!ti';(:cei;;:;H~. "·''.? :· '"i :· ·
I
IHUtti~UrlWU~i iililUMl~tU~tiUi.il:1 ~i~n~;n.;; i::;,~{U:~ H:-~;1niH{~3h:i1~*:~~ •:; ·: .. ;,.~..1~ ,,~.:~i~) I •}!{~~).l}K~t:t~.{J~H.·, h~li~--·· "·1 ·: - '"~ ., ,.

.>

r/ Rigid Mo111e,L 25
't<I , D)'l1amirs lJoci,\' a11d Pmd"W'' of,focrlia
\
•'· Mei .-11 + 1 Mb 1 , m2 + l Mc 1 , n; "! M (a 2i2
} ~ : l, . ,· :i
+i/'m 2 + c2n2).
'
:, M,l. o.f l'ri\l- r~l!longular
pn;al!clnpiped, obout ·the edge
~ l:lS, Theorem of·l'arnllcLAx!s 1 ••
OA ~ M.l. oi lhe rectangular ~-----,,-,--, 0
The 11:11mrn1s p11d products of inel'/iruibom (V.'Cs 1/11:ougll the c·e111re of
'paralldnpiped about a ,pnrallel
gra\·it) · or~· .~i1·,·•1, tu)iiid :he_ 1111m1~nts oiir] pmdll(:ts of i11anic; abow parcille/
ax.is OX' .through .i\.~ C. a.
a.~e.,. :G' + M.I. of tmal ma~s M al r( J n ff :iJ(H I
Let f.i, i-, z) b~ \he
C. G. 'Ci' about'OA.
coordinates of the cen\re tif
M , '
gravi1y G of the body referre~ "3 (b· + C')
10 _the rectangular axes. OX,' e
·+ M (perpendicular distance y,'B
OY, OZ through n· fiMd point I

0. Le\ GX;, Cir'', GZ'. be lhe


axes through G parallel .lo .lhe
Z\.
?I
• l . .
mp(x,y,z) .
. . (x',y',i')
of
=·f
6 from· OA)l
(b + c ) ~ M (b +·c } = M (b + c }. .
2 2 2 2 2 1
1 /
axes OX, OY, OZ re~pec!ive!y,
~- Aliler. Consider• ah 'elemeM Ji~ oy 61: al \he pc>int

;o,
If (x, }', tl and (x ', y ', z ') P wliose conordin'o.tes
) Cl {x,y,z) X'
are 1hc coordinates or a p~niclc referred to the rectangular axls along edges OA, OB, .OC' o.re (x, y, z),
of moss m a1 p· rcfemd ·10 the
i V' :, M.I. o.f this clemc.nl about OA · •
coordinate axe, OX, OY, OZ X = (pSx Sy 13z), (.)'2 t 2), + '
parallel axes 'GX',GY', :. ~.I'. of the rectangular pnrallelopipe_d ~bout _OA
GZ 'respectively, the'n
X "'X + ,\'', .)' = )' + ,1' ',.: "'Z + Z ',
= f"'o
24
Pb>l=o pc p <i + z2) dx dy dz =j_ M (b 2 + c2)
1.= o " 3
e

M.l oi the body _abou1 OX .1 ,i,} '


=r:m (.)·' + t') =l:'.ni { G' + )' ') + (1+ z ') 21 · ·i "~
· =--·
Sabe
... r:,ncv•l+:'2)+(,y'l_+z')I:111+2)',r.my'+2U:m,' I: . ,(!)
6t :c
I

Now r~fomd l.i CX ', GY ', GZ' as nxes the coordinates: are Ex: 13, Find the M.I. of a right circular cylinder about
(0,0,0) · (il i1.r axi.s, [Mmut TDC 96]
nr tni,r '= O. Similarly :l:my' 0, lmt' = 0. = (ii) a Slaight line through its. C. · Ci. and perpendicular 10 its axis.
Sol. Let.a be the radius, ,h the height and. M the mnss of a right circular
From (l), M.L of the hody aloul OX cylinder, If p is lhe density of the ~yli'r.der-thcn M"' pr:a4h. •
= tm (y •l ,+: z '1) + M G2 + z2}'.·· . , Consider an elem.e~tary disc, of bread1h· Bx, perpendicular to t~e axis
= M.l, of the body about GX' + M.l, of 1hc C about OX. 101al mass M at 0 10 2 an~!' at a .distnnce ·x from lh_e centre·· o( gravity O o( cylinder,
Also, Product of lntcnia {P.l) of the bo,dy abou\ OX and OY : .-, Mnss of the disc om= p, ita 2 ox.
,. tni [Cx + x 'LG+ y 'll
"' r:nu )' ; +::;,: :;;r.m +·ni,i°y ; + :;;r.mx; ·M.L o( the .disc nboul 0 10 2 = ai-:0~1 = a2 , p11a'ox • fpM 4ox, t !
=tm.x'y'+M:Xy .-. M.l. of the crlinder ~bout 0 10 2
.,. P.I. ubou1 GX' and OY' ,. P,J. of the tr\lal mass M al G abouli OX 11nd ·~ Y1
0)' = J.y ll PRa· 4 dx. = LI pita 4h = .!.l Ma 2
. I 1
EXAMPLES (·: M = p1tii 2h}
Ex. 12, Fi11d 1/i~ M.l. of a rec1a,1gular_par(ll/elopipcd abo:,/ :ait edge. (ii) Let OL ·be ,he line through ,he C, G, 'O' and perpendicular .lo tho·
Sol. Let ia·, ?.b, 2c ~c the lengths oi th<:. edges of a· re~t~ngulur •3X:, of the cylinder, . t f·

pnrnllcl.opiped of mass M. ' · M.l. or the cl_cmen1:iiy dis,; about Ol.

' .. , • .,_ -.~~.-, ..:, i:1~·


~'
/! ~~~?/?:::•'' '1•' \'.'' ,,•' .• ., . •'! ,·,:;: >; <(l •;.·. (l.•:·~w~1;n~'7"t;ro~: i:;"''.,1.'.~• 1::1. ~:~!G::-: . ,:::·\::r►::t:~'. i'. I:;·7·,•:•:~nt:•]"'.•f.:(:~:".~1-??~!\'.~•:' ' ·-'• ,,: ''.'l'. ', ',;·(:•: ,•,I,·:,:' ''"·, •i:,··'.'' . ,. h .' ·.i:~.'1;~:!f,W•~:~~~l W~&i;'.};'+i~"'¾~f)HfiM:l?l~~~--.;!,;,;u, ,··:~! •11~m~1i,-', 1JH•1:?• ~•'ltNN,'1,l,¥,.;,'-',....~ ...~t3.'it~t''r;io~~(1)1~:.Qtfi,1.\;:

~11
,I .,:. :r~ ~:; ,.,, ' : : !, ~.-.'' ,::i···
; :: :; ;;t:~'

iii
- - - - - - - - , .,.....,......,.1,.:-·n.,,..• ,-,,~ ,.,,lk,(WJ:i¼IJ7,~r:r~~--;:ro:~n7.r:l~S'E'fil
. · . · ~. ""'......_:_.__..:

' l

: '(s_: ·.. . ;,•,i, :


·: ~' 1: ,:1 :,;,; :--~'':<·' '::,::•:•,!,
,.,,.,,.,,,t,,,, •·.,

,. .. ______ ...._.,_,.,.,....., ......-..... .....--'--...


, :.ll'.:,11:i..1i:-:u!.:..•!l!.1::,~11t111,,:,;.:.~,1.'"ll•:.t..•.u.""'t~• ,.i..t,1.o,,1 :..'\\1,~,·~1,1,r,,U:t.W~\•.V--...-•~-~-rtli!""-~('11U•ho'•"11 ,w ,. ",, ,, ., • •·

:j,
'l
.ii
~
26
'.
--
:~, ~
•;
!
Dy11amks of Rigid Body MOIIICIIIS am/ Pmducrs of /11erria 27

·1 1•. . 1
"'M,l. of the cllse nbou1 t~c p~ru11e1 ,k tan 2 a (3 tnn 2 Ct+ 2) = M/11 (3 tunl a+ l}, (!'ront ( I )l
I "'66 20

f
I line EF through Its C. 0. 0 3 + M.r.
of tthe 101nl M m 0,,, about OL
20
3. !C + J) = 20
. 1,2
M (3(/2 + 2/i;)
2
""fa om + xl/51/1,. (1CJ' + x2) Sm ~· t' . · (·: rnn Cl =c//li),
= <1 e12 + x2) pna2&. • A_, ____, ~-.,,.- Dd i ~x.' 15. A solid° l1ody of denj•iry p iJ /11 tile s/ic1p~ of thf' snlirl /onn<'d'

0
:. M.J. of !he cylinder about• OL {~ a /i by .(ii.I> revolmiun of th~ cardJod r" at I + co~. 8), 9/,0111 the i11i1ir1/ li11e; slio11•
~ 1 o ~------- ·111►-M'-··-Hlo,· t!ia) \ts ;M. I. ah.011: (1 .straig/11 liue tlirvug/i thi pole· cine/ 11rr/1Mdir11/ar to
111
I .,
"' -~·i (~a~.+~~) pna d~· , . ~ .' ~., ,c .. '.i/1itia/li11e:is. ::i
52 ·npa5. · · . i
· . -!OS · . . . . .
· .' l [ · .
= ,-..,1t11 t• i:t.r 3] ½ D : .c . Sol;,Lct ()X bo·(he .initfol line (~xi,. of'lhc ,cardiod)
. +!) x ·½ } • . · 81\d Ol' 1hdinc
perpendic~lar to' 'it •lhr<;ugh, the !Wl~ O. ·
·· :..¾ p'n"a2;;,{a1 ~} Ji1} .. ¼'•M ~a2 + {;,2). ·., , , . .Consicler no clelric.n·tnl')' ,area r&eor at the 'poi111 P(r, ti;;,.11°)1!.5$
· . r-:x_, 14, Prove tl1d1 th~ M./, of"· U(1/fo,nn i,{gl1t ,c{ic:i:.tar 'so')irl cone: of
·1114,rs ~1, hei1:1ht .. 'h, a11d /)(}se•r(ld/u.i a, ·nbo"(a·, ,d1'a'mcler of ·its "base is
"'" of'th~ clcn1~nttlr:,, rirl~"or rndiu~ Pr.
oll1nincd by die ·· rcvot,nion ol' I
M ', :l ·2. ' · · :: . ,. '· ' 'elernc.nt ro8or nbout OX,
· 20 _(·~
0
,. + '2~ ·'· [Meeti!LTD.C, 9.2(P.i;:'°9.:l(P);; 95(BP)J
. . 8111 ,., P. , 2:1(?1. • r886r .
·. 'Sol, Let ·o b,a tne vcnox oC a,righl ctrcL!J~·coti~·;~(mass M;·bMgfi( Ii =:2rt~r.sln e. r/l86r'
,D(ld -~nsc-rndi11s a, bx ls tho' semi-'vci't(cal,nngl<(nti~,.~, 1fy:i .density orthe · d:foi,',2 sin IHHtQr, .
conc·,,.rhen· ,: ·. · · · · .. ,. · · ·:, ·:_-::· ' · ' . whe~e P, is Hie'nfass per u·niJ vol1:me )';
· M=i_ nhttnn 2 .ap.' ' ...{l} · · · · l ,, ofl~<dolid fonned by 111.c revolution
Cosidcr. ;in e.lcmen1nry disc: PQ of I.. ·of lh'e conllod aoout the initial tine
O,t;• • , , .
I \
: . tllickne~! Bi,' parallel to the 'bnse,:4,8 Ai':d
'• ,• . Rt tld!~'tt\~.cd ,X' ,from. the ~er:t'cx :o: ~ - · ' I Jvif of· this elonieninry ring :tboul
. OY.
·· ·:. Mass-<i(thc' disc· , x;,J, · = ltsi · M.l. · ahou1 the dl~mcter·
.. '. ', '•:_.~i ' .
. •= om.=·piµ-;tan a.fu-1. . . • . · • ~tll/'... ,.·\',.1 j
!, • •• ••• ···, '• ' '
i '
\· PQ ~ ~U. of mas~ Sni nt centre L ahou1. 0 l'
::l:,M:I.'-.pLl~e-'. disc 1bo~! lh~ ~iametcr , · · · ~ -,:~ 1 . ':;: 1'f,m, Pl~+ Sm: Ol~ ~ (1 PL2+.0l") om .
· ,AB· of01h'e •ba~e ·of t):i'e cone . . .
'!l.(l ?- sin 2 9 +? cos 2 6) 2~pr2 sin 8 890;
' :, . ~-:lts :•M..r."•,11,b~Qt :pa~aller' diameter PQ. ' . .·: .. .:• ,._:: :.-.: :f: \ . l . • ,,. ..
of:the ·disc· +-M,l, ~f the-total mas$. 5m ,at'.·oentre:;C· nbbuh(\\.B. ·. ~ !·
. = r.p (sin 2 9 + 2 cos 2 ~l r4 slo 0, li01ir l'
:, .,,1qm,:' CP~;; &,7. t:D'.2~ piu2 iarl 2 er. ti..;: 1a~t~:\(h'{~?i'8i. ' = np (t +·cos' ei ~4 dii (i°&eor.
. ..,,,:,1~.f. ·ohhe"-cono."obou\·.!he diam<:t!l1'.'of 1tie,,9i\;e' ·::>,··,.'i M.l. of 1hc soild of re,volu!lqn· about OY
:•,;o r\·14~•1~~:. ·cx1f; 2 tiln 2 ~ + (h- x2)1 J ax. .•. . '. . . "'I".: f o(l•,· 0161np(l ,;.cos 2 G\r4 s,in'8d8dr
. (} ' .. . ' . . '
0 ru
n
O
,
,

'.,,J,:.p·,nanf a'
' .. 4·
fQ\;~
-~
1an 2 a+ 41/? - 8.'i.x 3' +·4i) ilx
' . •" I
"'l 11pa 5 J '(\ + cus 2 0) ( l + cos 0)~ si11 8 d0 ,
,\: . () .
. . !' · ·. 2 '[ l. · 5 . ,
. =.-:-::•pre.tan, a.
4 s • s .41/
11 .ian· a+;:/\ - ,ll + 1,' sJ· I- =.:-,f. itpaS j o { I+ (1- 1)2) . ,s dt I + cos 9 "'I
'.'14 . ·: : "':: ' 1 . .. 6 ' i;,•, L : ' 2
f

.... ,,.,,, ..,.. __ ,_, .. ,, ,,


illllJm~;,c<•;i: -:1•;:;;1>;'i<i/l!illtilllHBM1·i•;;•: ·.• ;._,•••> ;:.,,.. •:• ;:. ·. ,.,.,,, .. ,, •. , . . . . . . . . . . . . . . . ..
Wti8l1}~t i:Hf I'· :,f ! fni\bl~~tiliDlUlibfr ,: H: ~.~f; t,,f,!fo·, •-~ ,.( ~'!:!:'i ~!~/;~t•j~11,.,~!!(1ti:,'h;; ! • ,'.-:, · ·J :, ;; ';, ,,,; , .• ;\,' '~~: ,:h!~{1$~t~t'I\' ~i•ti,'l ;-;{u} :tnf."t 1:!tJ ,~.•:· ,,; u.;,r: , ·.
1 1

., >
I
i
'-'.' 11(/1//{('S OJ 1(/ 1/W qoay j
1 :
1 1 I 2 , I( I a 2x'
=:.. 6111. (l PQ) + 6111 ._~N" =l·---, + :....111)·)0111
r2 · ,
• . < f .; { 2r~ - 21 6 .+ 17) dt, · i t 3· · l .. ·. 12 fi• . /i2 .
"' f
.
1q,1r 1
(l

]l .; l .nn,,.'· (35~.::. l= -352 r:pa~


.,
I 1,
.
"'2:...(a
1
2
+ l:MD 2) Ra o,r
. . •. ' ' \
!
= l np,,.1 Ir , /:. - : r7 + !1 18 · •· .
1\ •
i2h 2 . h !'
i L1 , · ii i I' ' 21 · . 105 . ' · ; l
F..~. ·1Ii .. J,','111/ 11,q M.I. of a .1ria11g/a' A Oa/)0111 a' pcrpe.1;c/ic11/ar JO, the · "' ...££!..3 [a' + I 2 , .! .(2b2 ..: 2c 2 - a 2)] , ,.:i 6.~ · I
i 1211 ·' l
I· /!lane 1/im11)/li A. . · · (Meeru't TDC ·9o(P,),; 97]
I Sol. Lei Al.·lic o ·1;n~ \hrough A und_ pci·;,c~diculnr to.lhe plane pf; \he, .i. ~ (3b2 +.3c 2 - ~ 2) .i:3 ox..
1ri11ngl~ A/JC of mass Mand dc/~15i!y p. I · • .6'1 . i
t.~1 the heigh! ot' lhc lrinnglc,•· Alf,. h. .:. M.I: of ihe ,:), ABC about Al,
;, ,\,f"' •]

=: j poll
pHC, At
, .. ( l)
f h na
"'. o-=-
6// 3
·, · ' · nah · 2
(31,· + 3c 2 a 2) x3 dx =
24
'2 2
=
(3& + 3c - a )

·

CQnsiuor on ele111en1nry . $trip .l..'..\ ., 12 (3b2 + 3cl -a•),
',j
/ (from {I)]
('(2. or 1hicknc,s o.,
01 a dis1ance .x ', A ·'!' I

from. A and r~rnl le I 10 13 C, Li)l th~ i § 1,16. Moment and Procluct or Inertl::i or a Pl11no lfamlnn 11b.out a
mcninn AO nnd :!he pcrpc~d1culAr L~~ . :
AE 1i1ce1 /'Q .i1 ,v an1JK rcRpec1ivcly. .If the mome/1/s a1;d prod11cts of inenla 'of a plant lamina allottt two·
P. perpendicular axes in its plane· ara g/vqn, 10 find the moment and product
Cl9arl)' N will be th~ middle point
of NJ; . throuch /heir point of i11tersection.
of inertia a&ou1 a11y perpend/f,dar lines
Froll\ "s1mil:1r 1n.111glcs APQ and Let .~ and 8 he the moments of inertia and F the product of fncrti'a
AllC. we hnvc S- ~ ~ · of a plane lamina aboul the pcrpcndicul~r axes OX and OY in its plane.
PQ AK PQ X . ~ Consider an· clemer.t of m~ss m of
-BC = t\£
- or - a. = -h or. PQ = -·
h the lamina al the ppint P whose
Also from similnr triangle ANK ond ADE, we _have 'i co-ordinates are (.x, y) wilh rlference 10
il:f.,. ~ or il:f. =:.It. or AN.,!. A[.J•
I the axes OX and OY. Y'\ y
At;) AE AD It I ,., A-= t mi, B =: t ni.i:2 a11d F"" t nv:y. · '
In t:,,ADE::, we have
A0.2 "AE' + DE2 cAE' +(BE.;. BD)' Lei OX' ·and OY' be the
= (A£2 + BE 2) + 8D 2 - 2 BE·. 8D perpendicular axes in the pIanc of the
lamina and i,1clincd nt a.ii' angle a: 10 OX
·r•; \1:1•• ~l<x,y,z
K\ ·•ji( ,,,(X'y'z'. . .. -
X'
+
= AB 2 q BCP- 2. AB coSB. ½BC
~nd oYresi:,ectivcly. li (X ', y ') are lhe
\ y
\
I \Y~--•
••: .. N
n 2 + c2 - b' a co,ordinatos of the ·point P with
'o ·x' +-----X
=< ,.1 + ! .,2 _, -------;:'--:--'-
• •~ •· •2ac · rMcrence to these axe~; then
I or AD1 =; (2/Jl + 2cl - a') ... (2) x '= Pl( =·x cos o:+ y sin C4
Nnw, moss of 1he elementary strip PQ, and y'::: PN = ,Y cos a: -x ,in o:,
:. M.!. of the lamina a:iout OX'
OHi =rPQox = p h Om. ax ' = r mPN2 = I: my ' 2 = z111 (y cos o: -;,; sin c;f
II:r,J M.I. of strip PQ abdui th'e line AL.. . f 2 (r
111.x2) sli1'- a..\ 2 ~E nuy) sin 0: COSS 0:
l,:l
::: (l: my2) co\ 0: +
= M.L of slrip PQ about the line parnllcl 10 AL through its C. G. /N' t · =A cos 2 o:+ B sin2 o:-F$in ia. . .
: M.l. of rnm 8:11 ill N nbm11 AL i: ... ::: !I, ...( t)
'' Also P.l. ~f 1hc lamin~_riboutO\'. and OY' '····
;ie,. 'I
.... ~.'•
!
i ;

I.
. \ '
·:•[•:-···:·~-·· ··: ··:~:·il~?:~!:!'··-••H~~~t"!:!·!~!~mt~~e.~1.:"'nr~umnIT";flm!~~t'!:~:t'mtrm:r.im:1;-rr;r.:;m:rrrr.:;:;~n~·~trri.•?~fri' 1
~'."'.' ;~,:1-::r":~'f·J•~p:~~.•Htr~:~'·:···:·:: 1"l' 1 '''•.:~:'·~·:•·i::::-:1!:!!m•~!~~:,f-;~1111!i~~~um~~.,.,,Aff~,'=t-l~•~rr~Hnm-w..,,.l:·.
1
.

•,:••.'.i!'.·
•'•) ·~::-::·•:• ··:.:;::t::t··· ,..-,.,~'.G;?·

-~-,-----~--~-- .. , ~ ~ . ; , ~ 1 ; ; ~

.. , .,. .••.-r,-
m
',;<'•.•'

i
l'i
~!
·'(,j
\~
I
I
... _.,., ..... ~, ···~·~• ..... ...._.___..'.:,.;...~;.i,4\.,:.;~!;'.;~, . J;:,;.::-.1',.•1:>J.~'!.t,•~~.
(I :; ,•,, ... ' -~~;i:,~1... ;,•{,•:<:•.'."

.~. ...-~
•,:
;lO
Dynamlcs of ~.i~ld Body
J.
,,.\,l,,'t,,,,,lt;i,'
~•-.'-1•-i·l•• •,

t
!1•l ; :·-,: • Moments a11d Producis of lnert,'a 'l
~ll
='I: mPN, PK=- I: 1111•' x'
:>l
\l .
\~
""t Ill(~· i:US cc - X si~ 0:) (x cos 0: + .I' sin r,.I
2· · 2 ' ... . EXAMPLES
;:,, "'(i; "')' - t 111.r ) sin a ~o. a+ (l: 111.r,1·) (cos 2 o: - sin 2 o:) ·· 1
=1(.1-ll)sin20:+Fco~2a. ·, ' I·, li:x, ;11_;. Show that M,/. of a rec·iunfi./i: of nmu M 'ancl ;-frf<,.,. 2(1, 2/l
iiii ... {Z) : i >:' 2M a21/ ·
.§ 1.17, M.!i ot n Ilor!y about a Lin~ .. about a ii1agonal ls -:;- ~
I . , 2
~ " + b
;1 Given the· mom~nts a11d protlllrfr
n11itua/(r pc:11)mdicular (IXeJ, to /1111I rhr i11cri}a of ti /Jody about tlwc . Dli)u.:) that ,iii CC/IC <;J' a sq1wre, .
~ meeting point, abo1ir a,1)' line. t/irr.-ug/i their : · L~t ABCD be n rcctansle of mnss 0 nnd AD:: 2t1, BC'= 21,
!. Lei OX, 01', OZ ~ three mutually· perpendicular' :ixes. Cr.,nsictcr an:
clement of mass m 'of the body at · n1,S
M.!.
of rectnngle obout OX= A= t Mb 2,
.
otrectRngleab<1utOY:280:1M(1 2.
'
:,,.
M
o· .. ~
.

1 the point Pex,•i', z) then . ,


1 P,!. of !he ,rectangle about OX a1itl ./ 0
A= M,L ntiout OX
: 'Z 0 Y= F 0. = . (B)• syn11nctry) . , :. /
= I: m'.(.l+ z2) m'p(x,y,,)
.B = M;J, nbout OY ,, I If di~gonal; AC make nn ang)e 9 with AB, tppn .r-.i .••• ~ · •.•• ..
/ '1 co~e._Af!.._
.. 7 t!ll'(t -tx2}2
'' '
'
. · - AC;-
2a ~·.·. -~.:..-
✓(4a 2 +_4b 2)'- ✓(a 2 + I}}
., / ' : \
· r,;, \ 1
--<
''
'C =--M.l. ii.bout OZ ,'
, ,
BC b'" . ) \ ~
,. r.. in '. (x2 + y2) . Qnd s1.n .9 = -AB = -✓ ,~ , \\'.')
· ·· (a~:,+ b ) a, ---;---'
',V"'

D"' P.l. nbout OY nnd oz· X ,', M.!, :of,t\le rectnngle about ACj •x
=:i::m'yz
E= I?.!. iloout OZ nnd ox. ·y = ,1 cos' 8 'I- lJ2sin 2
8 - F' sin 29 (sec equation (I 1: § l. I,$)
=•tii1• U . I ' : i a · l
a b2 b2 2M
=-Mb? ·,--+-Ma
~ , : . a2 + b2 3 ·
-•--o,.-
a2 + b2
· 2-
3 a2 + b2
2
P = P,r. ~bout OX · an. d Ol'
I=tm•~. , Deciu.ction. For a squnrc,. 2b = 2a,
.'. M;r.: o~ squ(l[e about AC ·
· !..ct OA be a line throuah l~e point 0, (meeting point of the axes),,
anti /, m, n Its dlrectlon cosines,,' · 2M: ;'• ,;i4 l. 2
Ir Pl is 1hc perpendicular from P on OA, then "'3'.'. 7'7
ll +a
"'3 Ma,
2 2
PL = 6?2- OL = (.l~ i-1- z1r- (tx+ my+ nz)2 Ex. 18. SA ow tAot rAe M.1. of an elllpll'c area of ma.rs Mand semi-aus
I ab
2 2
=x (! - / ) +y2 (l -~1
2 2 I .
2
) +. z' (l :... n2) - 2inn?t - 2nlxz - 2/nuy. ·
. · . . .
a and b abo111 a diam·erer of length 2r is M
2 2
?'
=x 1 2
(1111 +11 ) -1- ,- {f.. + P.) ·+ z2 (1 2 +m2) - 2mnyz..: ~n,hz - 21~:.' , 4
s~j, tet PP' be 1he di.a meter of length 2r of an
f
2 2
I
o:: (>-2 + z ) /t+ (t +' x2) ;1

2
+ (.i2 +,y2) n2 - 2111ll)'t - 1.n/xz - 2fnvo•
(•··
' [2.m
' 2 +n 2 = l)
Mano (semi,axes a nncJ b. Equ.~tion of the ellipse is
x2 .i :, '1,.,
nrea or ma~s ,.
I

••. M,I. of the bo(jy about OA .


2
"':Z 111 'Pl = t2 ~;,, '(y' + i 2) + m2 l: m '(z2 + xl)
+ n• !. m '(x2 + )'') - 2mn I: 111 ')'Z - 2nl .t m 'xz - :Uni :t m 'xy
I
-:;+ 2""'.
a• ,b. .
Jf 1P e,make
(r CC,S
1
an angle 0 with OX, then co-ordinn1es of• P n.re
:
r sin G). . . . .
l
... ( l)
I
I"
2 2
"'A/ ,;, Bm + Cn 2 - 2Dm11 -:2£11/- 2Flnr, ... (I) .Sin,·e f' lje on equaiion (I). I
Note,§ l.l6. is a .~pecia! case of§ 1,1?. :;!
for II plnne !aminn n = 0, /=cos ex and 111 = cos (90' - o:; = sin cr..
,•, <r?fi cos 2 ~ -;1- (r'lb 2) sin2 8 = 1· I ,!)
' I 1 •. a•bl
Putting n = 0 in (I), we .&ct the· M:1. of the lu,ina abou·1 OA, or ti ~os 9 + a 2 sin 2 9 ,(2) ""7· ..
1".-\ cos 2 cc+ 13 sin 2 et-Fsln 2a, . .
". '""•~, ... _,

r
~~1tr:wm~mmtr:m111niun:· i:~1~ufmmm•mtr-v.:.mirn-rm:t~ ;:!·:.rrr: ;;t : :tHr.i~Ut~Hm1tnf !'- ~ri 7-f ~r; <: :1 t::: i{P:11u.,1t·:1t1HnHtm~m."ln>: ·!:; :~~-: i'i; !'.! :·. ~ f~ :;:;.::::. ·· 1
:, : :::!~!~;;:;:!;t:•.:t~:::~'.'. -::: "::;=".":: ··, •,'' •:•.•;'.;-._,;;,~;?,ffi!~~?}~,":t~.'•,=~~i,-~,:;..i~!:••t;..~,1~;:,-~.'.;•,·,(,:fJ~~"S~!li'.~-i:?-•;-..,..,~.-
.¼0111e11rs a11d,Prodr!c1s of li1enio . 33
N, ..... , i\U ,:,f 1!ic elli:ise ,1bou( OX= A "'I Mb_2, · • . . ' , 6.
;111J M.I. ,,r the cllir·sc i\botlt 01' =B-=:>11\111l, ,1.r• + 2/i.ry +· by- + - b ." = 0. ,;,(2)
{I - 11•
. • :, ' ., I i
r.1. 11r 111~ ~\lip,~ nboul OX ;ind oy, '-"here t, =abc + 2fgh - (If· - bg· - ch·. (By neom.rt?)
"· 1: "'(). •. lly i,,·111mni;-•) • ' , .c, '
,', M.I. ,,r lh<! clli1is~ JbOUl 1hc oinm~tcr P.P' ~ g , . •1 ?ulli~g 1· = 0 in (2). we hnvc .r·., - ~
I · . a(ali - Ji•)
= ,~ co,z \; .. Usin 1 8 - F sin 28, ,•. If r is the lcns1h of 1he se111i•diumcter of the ell ipso lo che. axis
"' Ml>' ,o,J 0 ~ ! Mr.r 2 sit,' 9 - 0 <)f :r; then
' I fl 'i
= M U,' co;l + ,1 2 sin 2 G)e •·I ' t,
(" ".' - a(ab - hi) • .. ,(3)
,\•/ n!J>: :S.ow, the equadon (2) of the ellipse can be wnnen as -•
::7. -,-::··
Ex. 19. If kt illlU J. 2 be the radii of 8Yrarion
I ' -- a .2 _ · 2h
_ , __ J ~ 1 I
b ,,..,
....(4)
c,•I-' C
I
/:'
J,I I
·I
ohu,11 r11·u ,·u11111gar,• dian1e1er,· 1/ien . ..,,here c ' "' M(at., •· /
• J_+J.,,/J.+J.\ Which.is of the ~tandard fonn Ax 2 + 2H-0• +; By2 = I.
. I kj kr la 2
2
v) . Th~ squares or the lengths of. the se1Jl-nxes of the ellipse, ore given. by
'Sol. Le( OP== r 1 und OQ == r 2 l)e two conj1i'gate" semi•diamete.pi. of an . value~ R2 in !he equd1ion . '
· IJminq. of m?,ss _M and semi-axes a." : I
( . t · \(.
A--/ I \
8--l=H·
,

.
of the ellipse about OP
,; M a2b2
1 II !' R' l Ri)
) •
. " 2
=M.l(r=-;7· (See Ex; 18)
or(. J:.. - ~L )(_·.Q.., J.. i=
l c' R2 (·c' Rl) (-.Ji.)•
c'
I 41'f ,. , · I 4,t
·-:; = -·;-; · S11nilnrly, - =-- • a +-
d · (- b ) •-.+--=O
l ab - h2 · .
ki 11·/J· .)?, . a'b' or-+
R4 CI •. R2 C •2 ' ' ' .... (5)

....I,. ·• -:I ,.. -:;-;


4 ( ,
'"i + '1')· .=; -,--
.. 4 ,
(a· + If ~ and P arc the lengths of.semi-axes of ellipse then J/r.t 2, 1,13 2 are 1he
ki ' k•l ,,·i,· /1'./,2 roois of (i), · ·
(·: ri +
I
~.' + b2 • ~Y 1rorerty) i ;I 1\ ab-hr.
c'l . ,::.2 ,,..z
,·. ,..-·,=-oro:--.., =--,=--········-··-
.,..4 (l/<1 2 +
·;,,,1. of d11 c/li/)tic area of rn<lsS M mid I o:2 J3· . c '2 . ab.- Ji· (a/; - h 2)3
:. From ex. 18, M.l. of the ellipse ~bout the diameter

·
Ex. 20, S/toll' thm the
+ 211xy + Ii/ •I- 2V +2//+ c = 0, abo1i1 a diameter parallelto, rhe o.xis
. -uM6 . .
2
=Mn~p ~M_t:. 2 2.:..:..[_a(ali-11
4 ?- · 4 (ab - h )3
2

6 .. ·
)]=-·-4(abaMc,,·,·
- h·)2
••.
1!( X il' ·•---n ' E.x. 21. SliQw 1ha1 1/ie M.I. of an ~/lipse ofmass Mand' semi-axes n
. 4(r1/1 ~ /()
whl'i·e ,\ = abc + i/gh - cii" - bg 2 - and b abora a 1ai12e111 is -~ Mp 2, where p /; the p.erpendicular from 1he
Sol. faiuation of the ellipse b ~cntre 011 the /angenl,
ax' .,.2/i.i,1· + byi + 2.q.t + 1/i·• c = 0: I ... (I) xz v2
.Shil'(ing lhc nrig\n 10 1hc,cc11\rc of the ellipse, 1hc e(luation of th~ ellipse ·Sol. Let the equation of an ellipse pc ~ + - "'
2
I.
1 a• b
biCOl\1;.lS ... Equa,ion c,f· I.he langent lo !l:i'c elHpsc. is
[ ) "m.r + ✓caZm 2 + b2). . . ·.. .'. ·f ...( I l
it where· 111:: tUli 0, ,if tangent is inclin~d o.t M 8 co th~ ·~xis or x. ....
I
I
!.
I 0 ~-.

' •~l)!H;'," 'J ·~'.' '.:;: •'. ., ' ,, .. ;,;i:~i{,;; ., . -: :


1
~:1 1.3{>>!: • , :~:~:: I: .
"';, ; .. •:. >.':-:;::••. ':=:~~::::·:: :· ··:.:;·:;:•;:}'' .. ,;;~:;;; .
. ~

--------=--===·"·"·""·' ~~;r;;:,"m:'?7)
'. .. . .. . .. .
'•-~~----··-· - ·-·

' l

·'·:-~'.";."
,;-.;.;;(,',:,, ·.'··> .
,,,•;~>,•,\ .
n

... ,,. '"'"••·~••--••"',.••--d• ~,,,.,..,•~·-1,,:,.~~:,.~!l:!l.~~ • •:.1.:.!;.~J.:.:u."..r.;'.1.JJ.l.i..:',Z,.:. .. ,~.,•r • .', .. ,. ,• .:.\;.tu.:::,:,{!:.::"'· 1.-:11: • ,i.:. 1,1 ·: 1~'.l:i~'..·,n:,,1 ·,•:(.(11t-;.1.~- ... •':::, '.:,·::H.H.f!tli ., ....... ,, ... ,....

34
'
'
-~
. ~\-: If
perpendicular from
p · is lhe
tpe
tan
O 1
-~- ?,•
Dynamics of Rigid Body
l
Mcmenrs

~ol. Let.M be t~. mass of.a


Produ,·n of Inertia

rig:11lcl'rcyla.r done o: :icigbt h nnd


:s5

""':::~• 0)'" t h , ' " ' " ' ~ radius of :A•hose base is a. lf o: is '
:Y I!,
- ..J(a2m2 + b'l .. ', . . the sem1-Hqicar angl-!: and P. the
j
p - ✓( I :t m2) ··j's · ? o •• ~ .' density of: tpe :cone, then
M -_ ;1 pr. ,·13'1ta~·2 a.
., ✓(al 1an2~e+ b2) ~A ... (I) ·
:I
✓(l·+ tan 2.9) O · Take Jh:~: vertex or 1hc cone· as
~1,,: l 2 · ,.. the origin,:x-a.x!is along the axis OD
,, .I , • o
... 'l(a sm· 8 + b cqs 9). 1... (2)
,;~ of the cone and· y-axis perpendiculnr
~; M.t. of.lhe eltipse aboul lhc diame1e.r PQ which Is parallel to the tangeni
;;
=•A err? e + B sin 2 B - F ~in 29 . I
10 OD. .. ·a
£i The slant side OA make an angle
: .-
.
= l,I Mb'l coi
'
ti+ J. Mp 1 sin 2 0•~.o
4
,vith OX. :a
4
a'
:=!.M,(b1 cos 2 9 + sin 2 0) = l Mp2. from (2).
'
:. M.l. of the cone about·OA =A cos2 a+ B sin 2 Ct.- F sin 2a, ... (2)
where, A= l'4,J. of the·corie Jbou\ ·ox, B = M.t. of the cone about OY
. :, M.l. of the ellipse about the. tangent · •.;, I and F=P.I. oflhe cor.e about OX and OY. .,,
=· Its M.I ..about the parallel. line through .;:,a. '0' Now coosider an e}em,;ntary disc PQ p1l!"a!Jel 10 the base 1\8 of Ille ·7
+ M.!. of mnss. M at O abou·. uie t_angent cone, of thickness 1h .and at·a distance x from 0.
'=¼ Mi +Mp2 =¼Mp2. Mm of (/ii~ e/ementMy disc p11.i tan 2 o:&. · =om=
Ex. 22, Show that the sum of the moments oj, i'nerria of an elliptic (i) M.l. of i~e: e!emen·.ary disc about
,• I< c· ,,i
area abour an:,, rwo, perpendicult;r tangenrs i.r always the same. . O""' I 4 4 0
ivm, . ""jPID, tan aux r;
§ol. M.t. of no elliptic nrea about a tangent inc lif1ed a\ an angle e t,o
the mnjor axis ' •I :, A"' M.L,.of the c-:;ne about OX'= I h} pi, x~ tan 4 o: dx
= ¾Mp2 (See last Bx. 2i)
= .L "t;h 5 tan 4·o: = 3,11 h2 tan2 o: '
· .. ¾M(a 2 sin2 9 + b2 co~ 2 9). , 10,., . : 10 ' from (l)
Replacing 9 by 9 + rc/2, the M.!. of the' elliptic area about n perpendicll11lf 3 ; :,
tangent . · • · ·
"'ToMa',,. ' l\lll Cl=
{I
i~.
= !4 M(a 2 cos. 2 e. + b2 .sln 2 G). · ! (ii) M,l. 'ofih~ elem~ntary disc about
= Its M.l. ab'ou!
parallel diameter PQ
OY
:. Sum of the mor;icnts' o( iner1ia about any twd• perpendlcuiar tangent I + M,!. o'f ITUlSS om et C about OY
2 sin( e+ bi' cos 2 9) + ! M(a 2 cos 2 9 + t., 2 sin 2 9)
= l.. M(a
' • • 4 ' . ' ' = ¼0/11 • c.fl:
+om. OC2 -= (¾ ;X' t~n• o: +x2) , pr. J tan 2 a 6x
"'tM(a2+b2) ... ! • +
= l (ian 2 o: 4) pn x4 ,an 2 o: o,t · \•
which is always the same as it ilindepcodent· of a. . ·1 :. B = M.l. of the cone about OY
Ex, 23, Show that rhe M.!. of a right so/Id cone_ whou /ieight is h , I
, .· ·,M2 6h2· 2· ·. =J II.~ O.ri-4) p;ix 4 inn 2 adx
~ r wIrose base is· a, 1s
· W + a a'b our as lant s1'd e;.a/! d
., a · - -i- O4
/. v
an d rartus 2 ·h +a I . I ~ ·, • . ' ~ 3 ML'
"'10 pn 1'; qnnr.€1· t -=:,1~an C( =20 r( a + 4), from ( Ii
Mf. \ (h 2 + 4a 2) abour a line 1/irough the centre of gravity of rhe cdhe '.I . ! 0
( 80 ·
per11ehdi;ular 10 /1s axis. · (Mcerut TDC 93 1:P), 96 (BP)J
,. ..:.. M((1l + 41i2)
20 ·• ' ·, ran a= -h ·
,. i
of: the c-cne nbout OX _and OY=0, By symmetry ahout
,.,, -
. .......... .
(iii) f= P,L:
. ' .
OX.

1
!J':•:'li~('5!'t~~l'lf,'.'.('i:?,c¾l(,\)',~•;,t;(i'.l')V,o,~~m:m~,!':!ffi!tc\:--"''.cc'l(,~'o';',,'·(,' '.\)\\'limnlffl!.'< ;i<cl'l< '>\•' ,<: !1•.J\Hf<IIHSI/W,!lll!t<fll'l!Hl)clll' i: :'' ,,,. ,· . '• :: :;; ','.'.'. '.'·' .'. ·

., .. , .... .... ••'


•·•..' f···• ,•,::,:··\< ;:;"i,".!•~/ :•'<~:"'·~1K,•··A•,••~•1; :,·,·,n .. ,J,· •···"

.H, rnv111cnrJ q11u ·1.,vun~i,> VJ 111c1,1w ~ I


Dy11(1niics of Rigi<1 iJ,~d.,· ><••·~'"<-"

It ~.. .. r (2) r (tl 4 4 2 . ,


Al~o co~a=-:=
OD
\1r'OD·+AD·)
OA
OD
') ')-=--')-·-:;-
✓(l,·+w)
= f0 • 2p11 a4 •~in 3 9d8 = 2prw 4 ·---,--a-=
2f (f)
-J pr,:a =-
.. 3
Ma•
'
from
AD a (ii) M.I. of.the ~t¢m~nlnry 1fos ·abou1 O.Y
,in o. = 0A
= ,.'(lil + aZ) ,:: tis M .!. aboul parn!let diameter through L
rm,n 12) 1\1.1, of ihe cone. nbout sl,nm. sklc + M.l. or·o111 ~r L ahout OY

J , 1, 2 · 3 ·, , a: ' JMu 2 6h 2 + u2 = &111 PL 2-+ om OL 2 =


1 (f ~
2 siri2 0 +(a-' a cos 2pna 2 sin 9 oe
= -- M11· · - ; - - . , +. - /vi ((I· + 4/i·) -,--, = - - . -,- -
11) 1i·,..11· 20 h'+1r 20 Ji•+a'. = pr:a4 fsin 2 e+ 2 ( l - cos El)') sin 956
Scn111d /'art. Ler G/.; be il \in~ thl'Ouuh the C.G. G 01 ihc "pna° [sin 2 9 +.2 + 2-cos' 0 - 4 cos 0) sin 060
to its ux1, OD. " •
= pna~ (3 + cos 2 6 - 4 cos 6) si~ eoe
nf the cone about O.Y = M.l. of the cone about p~rallel line B = M.I. of the shell about OY .•
C.G. C + M;f. of total inass M at
M.I. of the cone about .the line Ol f0
= \l'pna (3 + cos 2 6; 4 cos 6) sin 0d8
4
.
'I
• "'M.l. or the cone about OY'- M.I. of total inm Mat G. about t'
= t, M(a' + 4il 2) - M. OC 2 = t,; M(a 2 + 4'i1 2) - M . =- f 0p +? ·_ 41) d1
p1w 4 1 Putting co\ 6 = 1.
3M 2 .. 2 I· ' .·
=
80 (Ii + 4a ). . = j np04 = t Ma 4, .. . , .· ' fro:n
Ex. 2<1, Show 1h111 for a thin hemispherti:(,/ s,hil/1 of mass Mand rod111s And C = M.t. of the shell nbout OZ= B ={ Ma2. ,(By Summetry)
11, _the M.J. t1,?ou1 t1ny line 1hrv11gh ilfe l'er/t>,d,'j Ma 2. · (iii) Sinr.c. the COl)l'dinates of C

., S111. A hemispherical shell with I •. C.O. C (aA, O,.O)


o.ro
:, . D = P.l. of the shell aboul OY and OZ
v~ncx ut the nrigi n O is 11er\er3ted
by_ th¢' revolution ~r the arc .OA of
quadrant
a:
.
OA8·of the circle. or radius A
zl = I'.!. o'f the 8hel1 about lines through C. o., '0' 'parallel 10 OY nnd OZ +
P.l. of Lhe total mass Mat G about OY and OZ.
,,.o+M.O.O=Q
· · ,,!
If Pis ,he density or the shell, then 3 (Since shell is symmetrical at;ou1 lines through G, pnrallcl to OY nr.d OZ)
~;
M=21tpa 2 . ,.,(I) .
j
e=O=F. i
!' X
· Take the x•axis along the
"'" ·j
are the direction cosines of any line Lllrriugh the vertex 0, !h~.n ·
symmetrical radius 08 of the shell
I the shell about' this· line
!'
und axc,,OY and OZ pcrpen_d1eulnr y + Cn 2 - 2Dmti ~ Wnl - 2Flm '

to OX.
, ,.. _,. + ~ Ma 2 , 111 2 + ! Ma~n 2 =tMa 2 112 + m2 '1- n2)-= t Ma2. l,
ill' Conshfor nn ~kmcn\iiry arc 11 50 n\ j
~ii the point P or the arc!OA. § i. 18.'Theorem !. A cloud curve revolves ·ro1md an)• line OX. in irs
The mass or ,he elemen,ary ring obtaineci' by the '<>f Lhis
owll plane which do,es no/ inrarsect it, Show 1/ial th~ M,/, of tl,4 solid •Of r·
[\ .
elementary nrc a &e at P about ox: revolution so formed about OX is equal 10 M (u 2 + 3k 2), wl.erir M i's 1{1c
=om= p . 2r.Pl, 1109-= 2prca2 sin a·oe, mass of rhe 'solid generate<~ a is the distiwce Jiom OX of tlie cu111rl! ·'t of
Pl.~ a $in F.I)
(i) M,.J. of the elmentary ring about OX 1 ·: 1he cruve, and ,k is the radius of gyra1ion ofihe c_urve abo111 a Ji11c 1/1ni11gh
C parallel 10 OX. ·
.:: &m, Pl 2 = 2p:ta 2 sin 060 q2 sin 2 9.,;, 2pn a4 6 66 iin" Prof. Let C be Lhe centre of the clostd curve which revolve t·:iund
A = M,I. of the shell apout OX nny line OX in· ils ow~ plnn~ which does·•'nol 'intersc<.:L it Olvon th111 the
distan·cc of C from OX, CC 1 ;= a. · · ·.:: · · ·:'
' .. f

~- '•
l

., •.~;•,:::;::;:' • :,n:., ,. ..:~\•<•1·,,.::.:.:..·,:~:::•.-~r1:.:.:z,:-.:,.•f.y:~,:~'•:q::-R~f.{;}~ffi~lW(~r~.P.U~t\U:rmHt~P.'4.-.""4J/\'t~ ..w~.;ir-~•tf-"4''":•'-·•t!-C~·~'.,!! .. ~t•,r!"•!•:•,··'" ....·vt,-:•~•;~in:(nlo(,~').;.\o:,v:;

'•!,~•., ;, . ,~~{'.: ,;.,-. ·,,:··:·) ·',•:!:;;:r!:t:::,'' ,;.,·,,.,;,.·, ''•::·:?:;,::.•· ....... ,,.:·.'4~~~r~::".:• .,,,~.,,
•} ;, ' ·' ,· s,: ;,.,:~•!~"i~\i'.i:, ,;,~{:::~:•;: :'.,;1;.
., '.,·.' '•'.'.!~'." :'.(~\~!\' ,, ... ·~· :: 1:, p·'.;j\~j(\ ":i,:, ,.,,

'ii
,'.·1)'..'•
~ ;~:·~~~'IHtt~,l·~•-····•< ,, i

"'7t;'~'T1,.~.~:m1t'!jr,;z:~ffi:i'-•>.~!;•N<~-~~:p~y~

---._ ·•,;_ t"\,;.,·" ',... ' - ·-

;:·.·:::·:7:-r:f::·
-------
• l

• ' . ' ,.~ '1 ,, •'


·:,,·,,'.' ,;,,:,i,.:-.1n' '·

:-•,•--~,"Ht~A!Nt11t>1.,.;,1;,~ ......... -.,,.""".;..:,. ..\;....,,._ __ , _,,_.,. .•• ~,. ,. ............ ~~•.!..~..;...:~,;;._:_;~.~.,~;,;,.).,,', ,,.,.:,.,, .. ,,.,-:,,.,; ... ,. •. ,• .,,-.,v•-.t,,-•-1",•.'\•~,,.1,..,.. h"'""f'!'~"';_~~....::...:.U.:.1!:.:..;..,;_.:,,,,;.:.;...::..,,;.,:,.;.:......:::.,,_.;i,:,..utQUL!!;;..~ ,,.:, •,~,.•... ,,. .. ~;t:,',.)i,!~.•
,,:

½f
-~i 38 •Dynamics of Rigid! Bod),
·.-➔• •
Mn111e11(s and Products of lnerria ,\9
' i
'i lf ·M ls che m~ss of the .iolid of
-..{is
,}., revolution fromed about OX, thc(I, 1heroem, 1he mass
by Pappus Theorem, 'We have
M = 2rca pS, where S is the nre,.a of ·
of revolution is given
/ n1 y,&Q
1
,!__l,
the closed swface. ' :1:>
•'iii
.i Con&ider an element, HJ is the :.radius of gyration or the Os'~ :~
rM6r at P(r, S) taking C 'os the' ar of the curve about OX then its ~ :a :r.,
pole and the line CA pru-allel 10, OX M.l. a:bout CA (a line parallel to OX) O O'\a-rsln/J !C~~•"'."x
ns the initinl line. For this element . ',., is pl?., ·
I :s
r606r at P there will be e.n equal· a-sin : . :a :~ Co~slder an eiemen\.~s at P (r, 9) of the ate taklng C as centr~ and
0----or -t,-?7
9 ,"'
,element for the snrne value· of 8 at ' CA as i.nitial line. For- th.is element os a1 P (r, 8) on !he nrc ther will
Q in the opposite direction. \ an cqua{ arc cs for lhe same value o( 9 in opposite dlrcc1iot1 n1 Q on
The tli~1nnces of P nnd Q. from OX arc given by art, i i • ·, ., '·
PP'= a+ nine ttnd QQ' =a - r sin S. • we have. PP I= a+ r sine and QQ I= a - r sin e,
Now, the nrcn·of the closed curve· :. M,I; ,of :1he are of ifi'~ curve about CA,
S:. 2 rdOdr JI ...( ! ) =;2lfJJ(r sin e)2. pdt .
the intrgcrntlon being 1nken to cover the upper.half of the area. .. ·2p} rJ,s1h2 e d.r ... (2)
, ,•, M.l. of the nren S nbour CA ls Sp!< the in/eiP"alibn being taken to cover the upper half or the arc,
on<l S pk 2 .; 2ff (r sin e)1. prd9dr · Now, 1,/4,l, :or the surface of revolution about OX .
2
-= J[2~ (a; r sin 0). (a+ r sin 8) + 21t (a - r sin 9): ia - r
the inlrgrntlon belng tnken to cover the upper' hu!f of the area, 1

"= 2 p JJ ,.:i sin 2 ed0dr . · · ' ... (2) "'J4rtp (a 3 + 3a? sin2 9) ds
:. M.I. of \he soll<l of revolution aboul OX, J
"4rtp~ 3 ds + I2npa Jr2 · ds
=Jj [2n (a+ r tin 0) , (a+ r sin 8)1. + 2li (a - r sin 0). (a - r sin 0)2) . prd9 dr : '.21tpa>/ + 6Mplk1 ( B.r. ( J) :md (2)1
= 211:pa/.(al -t- 3kl) = M (a2 + 3k~)., (·: M = 2rtapl)
., Jf 41tp (a3 + 3a?- sin' 0) 'rdSdr E:XAMPLES
w4npa2 JJrd6dr+ l21tp a JJ,.:i sln 2 edSdr. .. Ex; ·2s. The a 01.11 Its axis, of a solid n.bber ryre, of mass M
= 4n p a:I, S +A 1tpa , Spk~ , · '·
[By (l) and (2)) anf circular cross-section of rOJiius a is (M/4) (4b 2 -t- Ja 2), where b (j rhe
= 2npaS (a 2 + 31:2) = M (a2 +'31?). ·: M= 2npaS. raJius df,the cur,,e, .,
Theorem' J!, ,4~ closed cii,rve revolves round any line OX iis own in · S61i Lpt OX _be. the axi.s .?f 1he solid ty~e of mass M and circular
plane ·which .does not il'lledect ii. Show that /he M,I, of the surface of cross-~e~tiqn of radius a, Sci!'id tyre is obtained .by the revolution of the
circle ;of ra'dius a and centre C about OX, where CC .,,·b.
revolution so 'formed about ox is equal ro, M (a2 + 31: 2), where M:is .the I.at q. qe /the line through C, parJ\llel to OX, .· ·
man of tlze ;rurface senerated, a Is the di.rtatice from OX oft/1,: ce/ui-e C I Then ~;.I. pf the ci.rculat area of mass M' (sny) about ,CA
~
of rhe curye and k is the, radius of g')'rat/on &f the an: of the curve: about
a /inc through C parallel re OX. · · ' M'k 2 *l.M 1a2• :. k':la 2,
' .. ◄ I
Pror, Let 11bc lhc lenglh of \he arc of. 1he closed. curve, 1h·c~ From ~~edrcm i''of § I.! 7, M,I. of the solid tyre about ox
1=2fds. ... (l) I . = M (/j2 ~ J'k2) · her~ n is to b
the inlegration tnkcn to cover' the upper hnlf of the nrc = M 2 ~ a2) = (M/4) (4b 2 + 3a 2).
(b 1

l
: \-:, ~!-!:

, .•..,_,·,·':"•z
,, ..__.,,,._ ..... ., ... ,...--,,•; , ..

n,V:r·.-••
,, .,-,,•,•.,,. :,._v.,".•.;.,.
. ' ,·.' ,·:•.~:;.:1 ,:,: ..

iUlifHW\';it~);.( .;~aH;~!iif!nllUHUmlittU~}~.Hif;H~~w1~ ;;~~~ !Hit~~ ;{,-1) }~~t;mus:~;;~; ;} ;'. ;,', ,• '. ,' '. '1: :·;'' -:, : ;., .• ;,:~)~fo:t,;\ i!~tiUlU1!;~1!) n~I:'; :· I~•; l'•.: )HJ ?H; '.'' ), • ,·'' \ ,.. ~ ,." , ',

j
lI
".:,,·,,;•
•~11.
E.~. 26. The M./. abm11 !rs {l.ri.1 a.f o /wl!aw r,rre, a/ma.ts M a11d c!irf,/iar
, , , ! . 'I •
. .. (' '4"'
;i rcal;q>
- · .
. ') 1,2 + ,2
.-
\'
''corss-sec1/011 a/ rndi11s a is (M/2) (::b· + :,a"), 11'11c1·e b is 1l,_e 1:c.d/11~ a/ th~ 5
cw,·e. i Let the c'llipsoid de~rcase 1ndefinitel:,, sm;ill in si1.e.
Spl. [{~!'er i'igure of Ins, Ex, ~~. !. :. M,l. of the enclc•sed ell.1psoidal shell
Herc 1he ho Ilow 1yre .is obtuincd by 1I)~ revolu1ion ,o'f the ore io'.f th~
circle of radius a nnd centre C nbou1 OX, w,hcre CC'= b.
M.l. -of, lhe nrc of mass M' (sny) or the cir~le about CA,
:
11
=d { .:i rwbcp · - 5 -r
1;2 + r 2 'I

Since the shell is lxlunde<l by "imilDr nnd similarly situnled cnncc1Hri<:


. :.,(\)
M'k 2 ={M'a 2, . ,'. .i:~ a2. ellipsoids, therefore if a', b ',CI arc th'e semi-nm of the simil~r cllipsold,
,·. From.Theorem JI of§ 1.18, M.!, of theihollow tyre about OX !hen we have ·
a .c . i
= M (/J' + 3k2), ,ii, .here a is equal to b
;,= c'
= M (b 2 + ~ a 2) = (M/2.) (2b 2 + 3a 2). b, CI .•
l.l~. tvl.J. by the Method of l;)iffcrentiatlol'),
§ .•,"b=·-;a::,p(I ond c=;---;c,=qa.
. a .a
If y is a funclion of x arid 6x, 6y are small increments in the. v·alues :. From (I), M.I. c,f the ellipscidal shell
,
,/
Qf;,; and )' respectively, then w,:, 'know that · __ · I

'fu_~. ~- . ' ' =d[


'+ 2 a5 ]
J4 rtppq ·. ~
Lim
O.~ -) 0
< - ~- 1.e.
w-
0 - appro~1mntely, f c
= 1rtpp.q . (.p 2 + q 2) a•da.
uX uX B ,
... (2)
or Si"'~ ox, But ,he m11ss of the ellipsoid·"' ltabcp r "'I ippqa 3
f'or .:,x.ample , , ;, Mass of the elli;isoida\ ·shell
(i) Area of a circle, A "'-itr2· thcri 'b M = d (3nppqa•') = 4nppqa 2da.
d '\ d O r,;.·1a E: c' · c:: :x
oA = ( dr A j· or= dr (1tr2) , Sr= (2it,) or . · · 'I. 2.-c Hence from (2). we have
M.l. of,lhe ellipsoklal shell 1
= (Circumre{cncc of a circle of radius r) x thickness •Or. M M
(i1) Volume or sphere, V = 1~i-1, then . ·, = 3 (p2 +·q•) a2 = 3 (b2 + c')
rl
6'I ;a ( rlr \I
Ij · or= drJ q1tr)1 . Sr= (4itr)> or. • I: § 1.20, M.l, of Hetrogeneous Bodies.
The method nf di fforetltiation can be used in finding the· M.I. of a
= (Surface of
the spherical shell. of radius r) x thickne.ss or, , hetrogenous body ·whose boundary is :i surface of uniform density.'ror lhis
This T\'ethQd of differentiation can be used in finding the momp~ts of proceed as folloV{s : . .
inertia in some cases. For this see the following examples. · I ; (i): Find the M.L o: homogenous ~olid body, of density p.
• , I ;
(ii) Express this M.!. in terms of .a single parameter o: (soy) i.e. M,I.
EXA~1PLES
i: ;:: p¢(o.). '
(iii) Then by differentiation, the tvi.L of n shell which .is considered to be
Ex. 27. Show thar rhe ·M.I. of {1 1hin homoge11eous e//ipso~dal :shell made of a \ayor of uniform density ~·
/bo1111ded by similr,r n11d similarly ii1ua1ed concentric ellipsoids)\a/!pi,1 an ., p,j), (c,) da.
<l.<CS is (M/3) (I,+ where M i,· ihe of ti)e she,'/, ; ·,nass (iv) Replace p by '.he vnriable de1isit;; cr,
Sol, We know thnt thr, M.1. of.an ellipsoid of ucnsity p ar.d se.rp.i-axis ,M Th~s thc,M,I. of1ll1e givf n hetroge11eous body is give~ by
a, b, c about ,,'.axis is equal to: · .1 · ·
M.l. = j p¢' !o.) do:. . . ' :
For· illustration see the following exa:,,ples.
.,
..-:.
~ .... , ;

·•· ·,:::':;:,; :,·,., ,. :'.,.,,:··· ,.... ,,., .. ,,.~,., ,., ..y,·'.•~,l!m:';~~•~".'...,'":'.··•·"':"·"tlmi!:r':'. '::i:il-~l,f'.;::··~··:'":«T:'?'(~·~"·''~·:•'f-'"'·''"'•·'.,.-'"'"·'•~:r•''i:.•w'''"''"("''"'~~>\li!m:t~~\¥1,\«!H!l'l'•i!-l'~ill'"'"!"'!'•''·"":"..,..,"',_ "'''.'l''•~-:,--...,,.,...m..,.......,;,..,.,,

I
.,,,,.,. . : : '. ~ ;l ; '.: l ' .. ,,:_v,;;~:~;·,,;., ;,::;_;•::;;:•, .. . Ii ,;;;:::;; ,,•,,;,,::: ,'.:,._ .· ::•:::;•::::: •' ··!:r:::·:~··. .:'' :,::.,:~•:,::

~-.-~----~~~rr~'"n"':T~,~;"j'\"r~l
--••• ~,•••-•.v~•
, __ .

' I

,, '.,::,.~.;,; . .',:,J;r.,'.
•. ,•,/ ,,.•~~·:- ,,
·'
.. . -:,;,,:;'.

·... ::.,.,:•,•,,'UJ.:A/.l!.,,S1,.. . .. .,.,.,,. ••:.,••·',.i·,rnf\,•;


, •. , .. ••.• ;..;, ~:, .:{:, f:, ,';.,. •,• :, ,, ,,.•:,;~.~/,,•,: •, ,"·tt>l'~~•:\' •::'

j I'

42 Dynamics o/ Rigid an~ Prcrfocl.$ of Inertia 4'.l


..-~ .
-~
: :-, llXAMfLES
\

Replacing p by a= M and. Ihen !n!cgra1ing, \he. m~ss of the he1rogencous


'

E,x. 28, Show 1/iai rhe M.J, of a h.,,,.,,g~nous e/Upsoid abo11.1_ rhe major 1.ilip~oiq Is given b)'
axis is % 2 2
M (b + c ). the strata of ~ni/«;11 deylo• 1:ein'g J/mil~r com:ellfr•·c M 'F j "41ti.ap,;-a'da = n")..pqa 4 ,
",o
and. the
e//(OSO/d.J' a/ons tile l"l(;j<.fr axis \101') 1/ng as the distance Hene:e from ( l ), M,L of lhe hctrogenecus· ellipsoid
, ~ ' . l
from the· ce111re. . =iM + q·) a·== IM {b +
Sol. (i) We know that the M,l. of nr,,eJlipsoid cf density p and semi-axes
a, b, c about x. nxis is equal to . . . · Ex. 29. The Mot. of a he1roge11e,ms el/,ipu ab'out minor axis, 1/i~ .1·1mru
of tniiform densiry bel11g confocal id/ipse~ and .'aensiry c1/011g miflor a.tis,
4 · J b2 + c2 val)•ing as the distcm'ce- from the cenrrc
( 31tab.cp ·_ -:S- • , · 3M 4a 5 + c5- Sa3<~2
Also the 'mass or the ell\psoid'=J r..ab.."'!', . . , I ;ao • 2a3 + c3 .:. iac2
(ii) Since the boundary surfaces are si:nilar conccn'trlc elllpsoid, therefdre, : Sol. For coiif6.::·a1 ellipses,' we havr
if a', b ', c' are the scmi-~xcs of the ·.~imilar ellipsoid then we have a2e2 ?' a2 - I?= Co11stant.. ·1,0,,>¥

:T;ld11g a2 - b1 =-c~ lhe eua1ion of 1hc confocQI ellipse is

, b' c'
. a
a' -=---
i,
b '·
C
c' · ---r----i,..
i : ):.2i = l, ,,,.,, . '2
where. a "'b + c .
2 ~ . . .
·
, -~
~- ... ( I)
1.e. t, =;;a .. pa uncl c.,;;, a= qa. l! ,+c : b .. . . · . .,
! The M.l. of homoge'nous ellipse\ er unifonn densit:,, p nbout minM oxis· Is
... M.t. of tho clllpsold about X•nxls . , i '. a1 bi + cl ..·
2
. .. , .
4 b2+ cl 4 g2'+ gl (pnbp) : - = 1prtb (b 2 :+c 2)~rt, . .
4 =pnb..J(li + c ) • -
=-ttabcp • --o:-rr.ppq· ,,S, 4
3~ S 3 .5 'Diff~rc~\l~ting, the M,l. oi ~n elliptic str1110._or'1lniform density p
(iii) Differ.er.tinting the above M,I .. the M.!. of a shell of uniform·dens'it:,, = d{l pnh (b 2 + c2 )11l1 ··
·" ' "•, . ..
,,l + ,,a
=f P!t (:1-(bl + c2)3'::;t- b,½ (b2 + c2) Vl.2fil db
p ( '1 5)
"' d 3 ttppq • ~ a = i4 ~p✓(b
.
1
+ c2}.(4b
'•
2+c 2) db.
'
=:. ! 1tppq (p 2 + q'·) a4da.. · :Since 1h-:i densit)' varies ns the distance from the centre. lhcrcfore
)

(iv) ::iince th~ densily 'varies as the distance frc,m the ·centre • p by tJ; and integrating the M.!. o,f the hetrogeneou~ ellipse nbout
:. a"' NJ. '
. Replnci11g p by o "f).a and then lnteg:1dng the M.!, of the hetrogenous
o.llipsoid !l~out 1h{ major axis · ·
= J~ 3ii n'>-apq ci + 1h<ida
Q
1 fCl>4(t,2 + c2;~n. bdb-'l fObc2(/1l· ~ltl.)1121,eu,·]
'
= -4 1'fA.pq (;p' + qlj· J a aSda "' -2 rtt-pq (pl+
· t:/2) • a6 I ...:c ii =I.! M, 1' ! ,:b2 + c2_ ,512 - c~(l,2 +
,r ' 1,,' .
c~,~a "f
Jo
3 · D 9
· Also 'the mnss of the ellipsoid = f rub.:p = fnppqa 3, ~l
'
h [! l(o• +c2)sa._·cs)
f . ' .,
_ c2 ((t,2.,. ,2)3,1 _ c'))
. •' .
.', Mm of the ellipsoidiil shell =d{jr:ppqa 3) = l>1 nA
I
!!.S (n5.:: cS)• _ •c2 , .
, .. (2)

= 41tppqa2da, the m:!.si · of lhe' ellipse =npb~ = pnb../(b 2 c2 +i


or the. ellipllc sfr.ita· of uniform density p
i(l:,· ,.
. l ..

, ........ ,--,
. ···----··
, , ~~r, ..·,·~•••••."v.~.'.'·'·•· ,:,-,·.·.-.· .~ ·.• ·.-•.-~-.';J,.~.,~•.•~•-•.i.,. ,'

,.

mo~:en1s· ana l"maucrs of,. luerria


'•,'=
+ ~s
"'P1i (IJ' + ('ir 1
".2bi dh By solid geometry, we can find thl'cc mutu.illy perpcndicul~r diumcters,
,.,,-' + ,·•· ' :;u~h ihat with these diomctcrs .is c0ordin.i1c 11,(~~,:!li~ cq1i'atin11 of1hc ellipsoid
=p/i· , ., 1/l1. is tr,,nsfqrmcu inlo the !'onn•
.J(/J"jTt'"j
A 1x'+8 1y4 +C 1l1=Mtl•. ..(2)
P hr ),/J ~1id 1hc 1
of 1hc hc1mgcncous, clliri,r ·r:·.c priiduct of in~rlb with rc~pcc1 to 1hcsc 1ww .axes will v!tnish.
M= { "h= + ('::
· ..:~--;-dh 7i'w:.,: rhree new a.i:~s Cini wllcd Iha princlpnl nxcs of lhl! ltod,1· al 1//e
✓ ll
. ur + ,·-i poi111 O. l\11d a pla11e 1/iro11gh c111y Mo of.1lir.l'e axe.r is (a/fod a prlnclpol
= n,.[ ( bdl! - c:
plane of 111,, body. .i ' '.
Hence Jiir ·e,'UI)'" body /here esfrts /JI CFfl',I' ('!)/JI/ 0,, a ·.rel of tl1re~
= !'ii-. [ i mutu~·/1,r /.>crpendicu/ar axes, which t1rc Iii~ 1hrce pr/11,ip(J/' dia111~/~J'S of
- ,...' , I(b 2- + ,.., ) ]"
o th,: ,?I0/1/eJlrl ~llip.spld a/ q, _rnch !hat the pmduc1s of /11crtld of. Iha body
"n)., [j aboui them taken rwo at d time vanish, ,
.,. c1i,V:: -c-'J - ,·'{(bl+ ~J)II.! _; rl]
Note. When the three principal moments c,r inc11in at nriy p'oinl O arc the
= rci. [i (a·1 - c·1) - .-: (<1 - ,:JI, same, the ellipsoid b,ccomcs o sphere. In this cnse every diameter is a
Hence lh1111 f.~ft !he ~1.1. of !Ile hctrngcneous principal uiamelcr and all
radii vector.~ ttr'\)' lhc same,
,\-1 ~ fi,~ - ,· 1• r 2 i·,?- ct)
1 § 1.22, Momenta! Ellipse, I ,.
-..
1 ' • i ~ I Let' OX and OY be two mutually pc;pendicular axes nnd OQ n line
' ;-1,r -c·)-,·-fo-c) th;ought 0, all in the plane of a lamina, Then M, !. of t~c plnnc l~mlna,
,I
J,1/ 4n· +c -5,rc-
1\·,,· a!:ou I OQ given by is .. C

=·io. ~aT+ cl -3(/cl. .A cos 2 fl-2Fsin 8cos 8+ B sin 2 0,


§ 1.21. MomentaJ EJJipsold. where A, Bdenote th~ moments of inertia about OX, OY nna P 1he rroduct
, of ir.ertia ab0u1 OX .ind O Y. it
The M.f. of a body about a line OQ whose direction cosines are
I, m, 11 is given by Let P be a point on OQ su~h that the M.L or the lamina about OQ I
'I
2
A/ + 8111' + C11' - 20111n - 2£n/ - 2F/m, '
I.
ITT?Y be invc;scly proportional (O·OP. ' ' .,•;
wh-~rc A, 8. C, D, £, F arc the moments and products of inertia of the oody i.e. A cos 2
e- 2F sin El co~ 9 + 8 sin~ 8 .. ~ j
~
.ibou1 ,tile. nx~s. .. ·•.,. . . : ' OP4 " ;
4
Lei /J be J poim on OQ such 1h01
be inl'crsct;· rrnponion.il 10 opi
1he M.1. of the oody ot,out OQ
' ·
or A cos~ 0 - 2F ;in 0 cos 0 + lJ sin 2 0 = M~ where OP= r,
' . . ,..,. i
I'
. 2 ' ..., I 2
1.,,. Al + 1/111· + 1.,11• -
'I •
2Dm11 - 2t.nl - 2-F!m« - ,
. . .
or A,.1 cos 0 - 2Fr ~OS 0 r sin 0 + n? ~in 2 13 "'Mk 4
' OP- oi A.~• - 2F xy + Bi•;.= Mk4 . . ... (I)
i
or A/·+ Bm" a• c,,l ... 2011111 -- 2£11/ - 2Flm =,Mt, Sin:e A _and B arc cs,;cnllally rositi-ve,, thcn:forc equation (I) represent an !
ell r~c. This is ,;ailed II l1]0me11tal ellipse of the lam/nc1 nt \
I <
Note. The scc1ion of lhc morncntnl c!lfp~oid at Oby the plane of the lamina l
where OP= t is ;he. momcntnl ellipse. ·
or A/ ,2 ~ f1111 /· + C,, ,.2 - 2Omr,nr ,. 2£//r,/r - ZF/r,n1r = Mk 4
2 2 2
,'
' l . C, .
' ,.j.
or A.r- + IJ., · + z· • :!.Dyz ,- 2£1,x -)F~)'., Mk ,
, .
,.,(I) EXAMPl.,ES !:·
•;
Since A, 8, C .ire emn1inlly positive, therefore cquorion (t) represent :~n Ex. :rn. Find the mom~n,ui e/1/psoid,at ~·11y point O of a ma1erial .r1raigh1 i;
ellipsoid, This is r.o//ad 1h11 momenrnl ellipsoid (Jj 1'1~ body C11 0. , rod l8 of ma.rs Mand leng11i•2a, · ·' .. . ,,. ·, ~
::
S,)l. Let G he the ,cntrc .of gravity or a miltcriul itriifgh, rod AB of ,:
1,
ma.\i ,\·/ and lcn&th 2a, [:,cl· 0 be 'J' point on 1ti~ 0 r.pd $,l~;'QO n <:, ;
··.;·,,: 1
I,
~
/(
--
'•
if
.
:•,,t/,.·, 1
• • •• •., "· • ~·•?\ ::~:~nmm~·~·/;~:~~~~.~~,···.::·:~·:: .;,,,,' ·:;:1H'n~n~'!•' 1)~:~~~~!'"iH-'l\lKffflf?ffi;J?,t;,'l~t.t:"ft:.r:N{••N~>•s~.CW!i~!-;!t}i,,.l\o~·11~t:,,!-!''41oU'1~'1"'r)o·•}W~,~-lmtr,.,~~,.~ ..
. ·~

',;;'.:::~i>:••: '.:.:','.,;· ,:'.-'·f,.,; •,:;;::~::::: :, -·,. •.. "•!:::•::':>> ' :·':'(·:r:v~~--·


I
( ••••• C)\j\ iOJA)t:flfiA,1,,,,.,.,_~fS'.:l'i', .•.:r;1,•A&S!W1t~,\~!~l:ilt

'. ,. ;;:'.', ~?: ~· ,: '


' -- ___ ....._~--•+ ., . ·•

' '
' ' .,,:, ,;,;,;,:~:, :::,:.

:.,·-"'"""'' .... """"""~;,,~~'"'.HH-i1ilu...:...,:.,~...-~~..;~:-...:..~•··; ....,.. :· t·•'mn&mnl:U:.-.:.•1•. ·.:t~,t;l;: 1~.-:an.~,.r~:!1:r-: 1'. '. .'.' ,
1
.-;:; :. •• .,
1

C'!.Vf,1•1•":°r!-:'<":11',>;.tr~.:,:t:•,. ;l•.·J,,,!;, ·,a .. 1,~,


. .!t~ ·; :· :, '

..-~ 46
. ·~ "' Dynamics of Rigid Mome11ts cm~ Prochtcts of 'lnerlia
[!
'!
47
• Consider the nxis OX along the rod
and a:i:is OY perpendicular to the rod:
.,. Ex. ,32, :Sh()w that the equatio11 of r/ie momenta/ elllpsoitf at I/re dmer
:. I\= M.r. of the-rod nbout OX= o: of a cube oj side ;,a referred Jo irs principal axes ls ···
8 = M.I, or the rod about OY = M,,l. or . ' . 2x-' ·+ 11 (yl + t 2) = C,
the rod about.parallel axis GY' + M.I. o,. tvhere C ,'s· co11stan1. ,
of mass M·a( G aliout OY 5.iot, Let d be the centre.of gravity
= !l Ma 2 + .M~2';/,.' M.(!l a2 + c2) of a 'fJbe of side 2a. Let O be a corner
• of the :cube ·at which· we have to
.
Similarly C = M.I. of the rod about
.
OZ=
. .
M (ll a2 + c2J. detennlne .'t~e equadon of the momenta!
The coordinates of the C,0, '0' of the rod. are (c, 0, 0):
:. D=O=E=F. · ,· .th~ line OX through G a's the
Hence equation of the rnomentnl ellipsoid at O·ls : axis of x anq two mutually.Rcrpendicular
2 lines OY ~11$1 OZ-through O as the axis
Ax + Bi +.C:2 -2Dy2t-2E?X-:2Fxy = Const. · ·· of y and ~: , · ' · .. ·
or O + M.(! a2 + c2)
J • )
l+
M (~ 2 + c2) z2 = Const.
' Tne coordinat~s of G. referred lo OX, OY. OZ as a.xis arc (av,t 0. 0)
or- M (fa 2 + c2) r,y 2 +z2) = Const. a11d the prLducts of inertia of.the cube about any two 11\Ututilly perpcndiculru- .',''
lines throbgh G is zero. ,... . • ,
or ,ii 2+ z2 "'con.~1 I I• • t • • '

·- •' .-, \h~ propuct of inte~!ia abou1 the axes OX, 0 Y, OZ taken in pairs is•"
· ·Ex. 31, Show t/Jllt the momenta! at 1he c'entre pf an elliptic zero. TMus bx, Oi', oz are the principal axes of the mo111cn1al ellipsoid at
x2
plate is -:;- +..l'.:22 + z.2 ( - 2 =-consr. l +,I) ' : I J O. ' . , I
· a• b a b- · Since the• M.! . of the cube about. any axi.1 (pru-allel to an· edge) through
Sol. Let-M be the mass of an G=l Ma 1 .: . . ,
plnte of scmi-a,xes ~ ~nd b. L_ct the .ax~s ) l : :;. ' • • ') '

OX and OY be taken along-the major and :. A = M.I .. about OX=! A'J"·t: B'm 2 + C'n 2 = Ma2 1
y
minor axes of the clllp1ic plate ln its pll\ne
• A' = B' = C = i ,',,fnl.
',, ~

and the axes OZ perpendi~ular • its to B=M.I. aliout OY=M.I. about parallel axis through G . ,
plane. Then '. · ·
, · + M.r. of total mass Mat G about Or'
A =M.I. of the plate about OX . ·A 2
, , i·
= 1r Ma +.M.00 2 = l Ma 1 + M(a\/3) 2 ,d! Ma 2, I
;
""¾Mb 2 /
1 • , l ·,

8 = M.I. or the plate 6bout OY = ¾Ma 2 C='M.1, about OZ= il·Ma


, j l
2,
I.
and D = 0 i;' E -=. F,
C"' M.I. of the plale ·abo111 oz. '
= ~ M (a 2 +.b~) · •
Henc·e of the mo~entnl e'llipsoid O is ai
1 Ax2+ Bi:+. .,- 2Dmn - 2Enl - 2rlm = Const. 1f
and since pl~te is symmetrical about OX ond OY ,r
,·, D=O=E=F. ¥ Ma 2i + f Ma 2z2 =Corsi. ,
.•, Equa 1ion of the momen1al ~llipsoid at O is
+ z') = C, where C i~ a co11sran1.
Ax 2·+ B:;2 + ·cz2 - 2Dmn - 2Enl - 2F/m ·,. Const. Ex. ~i- Sirow rhat the momemal el/lpsoid ti/ the cenrre_ of a11
Of l Mb 2x2 + l Ma 2,,l + l M (a 2 + b2) z2 ,. CoMt.
-;- c J;.r ·+ (c t-a ) i + (11 1 + /}),~=conn
2 2
1'2 is 2 2
" .. " ,1

.. .2 2 2 1~ ' . .Sol, !Th• c9ua1io11 or nn ellipsoid. rcro,Tc<l \0 lhc pri11,ipol o.,u.; i,


or •\· + ~- + zt
·ab a.
+2
b
. j'" Cons1.
1
.......
·,,_,_,;,_,:,:tr,:.r.~,.,: ,',•;-,,.,,_, •.
. ·~:(, '·.- ..

~;-; H/U111r:Jt/ ~ r IV/lllt'I.\' llj /J/e/'11/1 4\l


()j
i\
. \ ' ,. "•:/.'/,• 0111'+ C(? 017'.:= g<l.l ,tos 2,0 + a' sin.) 9).2,ltCI.) p Cl)~ 860 I
+~ 5
.,, ,\ " ,\ L l.-,1ho,11 0.'( .. :) M(/i~ +
+27,' i
I
., ~~,.~ (~o, 1 0 + 2 ,;Ill~)~'" fJ68 =,Jtpu 2 ( I + ~it\~ fJ)
.'. ,·I "M.L ol' tile hc1:1lsph~1·lcnl shell nb,1u1 OX
;M 8{\6: I
\
'
f
. 1., . .

H:: :VI.I. nlmur


' . .
Or'·=!' M/c,.l +u') .= n1.!1tpt1• (I+ ~ir, 2 B) cos Gd9 ·
0 '
.
('" ,\1.1. :1huu1 OZ::!' + . JI . . .
= TCpl1 4 .
.
(I.+ 12) dr, sin O= 1
;111(.) () '" 0 = E =· F, 0
,•,11~au,1n ,':I' 1hc ll1tllMl\~al cllip,m~ .i\l. lhc ~en1rc ,,f the. r, + i-13•]· 1·., :!npcf .. i Mal,
= 1tpe1 4 L
is 11 = 2rca 2p '
,,..
+Ill'"+
.. . \ ,\',
c,· -
..., 21,)11111.- it,,1, 2F/111 = uonst.
., .,· .,
'

1 .•/llr - c·) x· +} M(c 2 + w) y· + l ,V(a· +•b·) ~ .. = con~t.


h
, ,\ (J

B = M.1. of the hemi•spherical snell about p Y


;i . l
'j

.,•"
.. "I . •
or 1/J" •
"' 'I ; ,;

C") ,\·~+le· + u·) _v~. + (<I"


., 'l.. . "\. .;
= [\, M,!. about pornlli:l di.imeter through.C
E.~. J-L Slww t1wi ·.1/i~·-11.1m111'111t11 · + M.I. of lotol mass M Ill C,about OY
.
t!i(' c11rn/11r. a poi.11 _<w· !edge ·of = !t,,fo•' +Ma·=
'
1Ma2 • . /
1

r-1'.r. of the ele1~~~tary ring {about OZ


Snl.. Let O htt n . = Its M.J. about BC+ M,I. of its mass 0111• al l nboi.r. OZ
point' on th~ th.c ;edge
· of 1hc circulnr bose of
! = PL2 om + OC2 om= (a2 cos 2 9 +a2) 2rta£p cos 808
i -~ 2rra 4p (cos 3 8 +cos 0) off I
u ,dhin hemiHh~ricnl
~ = M.I. of the hemispherical shell about. OZ
~h~II of rudiu, <7! arid
M. T:tkc·thc .:.xis·
-1110,.~
OX 11/ong rhc tliomcter
z ·
= t rt12 4
211:a •p(cos' _9 + cos 9) d9,. 2rca p, 4 [ r(2) r(l)
2 r(f/ ~ (sin _e 'o •
.
r<I.! j
.,

OA of hnxc ;,f the 2


,.,11a (f+ l)=jMa 2.
~h~II. nxi,
pcrpcn,liculnr 10 OX·
OY ,
,
li:.ic1·dinates· of C.G. 'O' of the shell are (a, 0, a/2)
.
.[
\ltrnui;h O in rhc pl,ine D= P.I. of 1110 ~hell :,hout.OY, OZ
.i:
of the bMc ond oxis A = P.I. of tile shell about lln<1~ parallel. lo OY, OZ through C + I',!. of mass
M ~1 G about OY,.pz . ' . f
OZ r~rpcndkulnr 10 Y,
1:
il,c boic. The thin = O + M.O.a/2 = 0. I·

hcn11.,phcricnl ,hcl I Similarly E = P.I. c,f the _shell aboul OZ, OX


"0 + M.a1?..a = 1l Ma 2
:~/,
rnJiu~ 11 is n.hlilincd hy the revolution of nrc OB.of the qua:lrarit
of rndiu., a 11ho1J1 the.line CB which is ri;,rnllel to OZ ontl nt.~ anu·f'=P.I. of the shell obout OX anJ.OY=O+M.a.O=O, i
from it;_ . · (I
Hence lhe equation of momcnlal ellipsoid at O is i·I
Co~~ider an clcrncnt· of nrc q oe at P By the revolutio;i A., ~ B,v + Ci" - 2D>•;:. - 2F:u: - 2F.i.·y = const.
2 2
ilb\)l.ll CB (/ circ\Jlur ring of radius Pl= (J ~OS A and
obtnined. or 1,Ma',l + !\ Ma 2.i.2 + !l Ma 2~2 - 0- 2)l Ma'u- O·• c-insl.
!
,, ,
!
Ma~s .of this clemcn1nry ring 1 or t~ + S(l•~ + i2) - 3u = const.
=on," p , 21t (I COS' 0, G/0~ = 2r.a"f1 COS 0 09. ·Ex. 35, Show 1ha1 /lie. momenta/, ellipsoid ar a point vn 1iic rim of a •
M,I. of lhis clenicntory ring ubout OA . /1e111is,·,hdre is .2.r 2 .,. 7(y2 + z2) -p Xt,= Con.rt,• I

= Its M.l. about PQ + M.l, ot' mos,1-om a\ ,cntre L· nbuut O;\.


. . . .;
! f

I
".:~:-:-r:!;···:~·~·":~:;:~~0~~!•~!'T~'\:"1~~!i~~ffr.~~~.~~~:mrm:;~rn':,':;tr:-:.::1::~~~1·,~~-,;:1~~-,r.·~1:~·,.,,.,J...,·l~,~·.,½."l:'f1•'.l-:Y,l'-f'1••• 1•~(>~,.1r;··-·••... f ~ : r : m H t i J ! i ~ { l ' ~ ~ ~ i ~ 1 ~ ~ ~ ~ , . . , , ~ , , . . _ ~ ~ 1 , s u , ;
""•";,.,

•',',"• ,<:,' . ':-":::::·.::·· ·';:;:,'::'.:'.' ,;;:::1:,••,

••*·---~.Y~--:--;~~~;·;•'"':=-•--;,..-~ .• 1.,1,. : i:,1.um_rs:'(a,-~.fw~;-;-:, :,1fr.,1 :~:S! ,}·:~r:.ITT:~1:r~

. :;,:~~::~;:: .
--'-~~-~---·-

·t.•'• .· •;.~11 l ~I,,;, •

w
w
~

•'
~
:,)!,,
·,,:J
.so •.,' ;, " '., ........... ,,.,....... .,,~-·.0'"1""""'""-=>,:..::w.~.. ".."'"'"-'•~~·-·"-·•--....... ,...~'."':,:::"·"'''"''""'•"""' ,,,, ..,.

~I1
:;
i
~

...,
, -- ~
~
l
!'
.;
Sol, Let O be • ,
point 011 the rim of a
'-')'11anucs of Ricidi Body
Mo111cnii iaj,i-1
!

I ,I. of l11er1ia
i ,,
;: hemisphere of radius a,
,; hs PJ. 1a~ou\ lines
SI

I~
, I
and !)'ass M.' If p is the
ut Ci ulmurOY nntl 07. o. pnrnllcl (O O,Y a11d oz + P.l. \~.r mnss M.
n density
=O+M,9,fq=0 .
"' M=
~ Similarly' E = ?.I, of he11ii$pherc ahilut OZ n11d OX
~ Take lhe axis OX I•
I = 0 + lvr: iIt a, a=.Kl .Ma 2 I
~long the diameter OA • t .1
! of the. circular base F '" P.l. of heinisphare, about OX nnd OY = 0 + M. a. O =0. !
~
l axis OY perpendicular X Henc,e;the ·cquntiorr of momentul el,lipsoid 0 is a,
A., + By, ~
2 2 2
to OX through O lo the
plane of tho base and 22
Ci.
- 2D.,·: -:2£~ - '2Fxy = const. ·
nxls OZ perpendicular or i "J;"O'~·~ +?
Ma ,i· + 1iv{a~z 2 - 0 - 2, { Ma 2u· - 0 = cost. e
to the 'liasc. or 2.t :+:7(\·2'+ z') - 7 xt= const,

Consider nn elementary sirip PQ of thickne.~s 8!;, parall~l to 'i:x.:36;· Prove th()r rhe eq11aiio11 of the 1110111e11tal cl/ipsoid'a/'a poi111,
nnd at .a distance ~ rrom C, then : • , · base
011 t/ia. c!irc11/ar adge oj i{so/fr! cone is •· 1 ; ·

Mass of this clcmcntll1'y disc, om


= p1rPd,i;~ "'pn(a 2 .:.. ~2):oc;. I /c:ia2+ 2h 2) x 2 + (;!3a 2 2
,f, :ia ) ,v 2 .:. 26a 2z2 - I 0ah.tz = CO/ISi,

of the e)erncntnry
wherei hi ii· i!ie hel~ht and, a the radius of tire base.
M.I.
nt l, about ox: disc about 0~"' lts M.l, ~bout, PQ'+ M.I. ,of mass lim ' ,sb1.J leti o b~ a poiiii
2 c1n th~ 'cijcutar edge· of a
'"1PL Sm+ Cl.~Sm.,, f! (n2 -c;2) + c;2J. p,r (a~ •• ~2)· QS solid · c,biie of mass M,
f
.. fnp (a4 ,f. 2,a2~2 - :ls 4) ~. ' ' semi-vertical: angle a:,
1/
~

·, ,•, A = M.I. of the hcmispt1ere nbout OX


' h and radius bm ·or z
a, If pl i$. hS den.~ity, then
~ s
:. J ~1tP.(a --2a ~•-3~ )d~--npa ·-~Mel
n I 4 2 4" . 4 2 ,. M =l 1tpl1 3 til& 2 a.
o' 4 · · · , - 1s ~s ·· ' ) ' '
Tnl<e:th~ ax/$ OX
. '

8 = M.r. of the hemisphere; about OY ·


the diameter OB of the
= ltR M.I.
abou/ ,the line 'through C (diameter or base) and poral!cl to nxis O'V: perpendicular- lo
2
=fMa ·+Ma~=fMa· 2, , OY+ ·· ·M.l, · or total mass· Mat C
· about
· OY OB in thdplane of the btise. a.
and .OZ:p~rpendicul.i.r to the l'!.'
Also M.I. of the elementary disc about OZ bnsc .. ; ! •
= Its M.I.2 oboui 2CB+ M.1. of its mass Ill l ab~ut oz om Co,niider an
= l.Pl Sm+ 9C om=
p: (~ 2 '-s
2) + a 2J prt (a 2 - i; ) ds
2 disc PQ P,~~llcl to the base, ·
n1 n llisdnce ~ from 11,c vcriex A and of thickness
, ' ' .... 2 " "
:: f 4
Pit (3a - 4a1 ~2 + S4) cfs ' 1
,', C = M. r. o~ tile hemisphere about OZ :, Mnis ior/this eie;.nentary disc, Siu = pnPl 28~ .
= Plrs 2;tdn~ ,Cl'. ds,
,. J0"lpn (3a
l
4
'
- 4a 2~2 + ~4) d~ = .!! p1ta 4 = 1 Ma 2,
!! . ' .
M.f. ·or this, elementary disc about· OX
!1s M,t. about PQ + M.f. of its mass on1 at l about OX,
'"I '"I

Coordinates of the C,O, '0' of the hemisphere, are (a, O, j a), 2


:. D = P.1, of the hemisphere about OY and .OZ
2
s) i
= l Pl 6~1 + Cl p11; = (i,; /OJISCt + (Ii - 2 pr.,;' tnn 2 ad,;,
:, k='M.L. of the cone about OX

:: I I~ ~l l~nl CX + (/i-~) 2] plt~l tan 2 cu:fS


,:,1,.::.. ,,
. ~ q. ' " . ' . ,.~•. ~ i:..;~. ,~.;:.:(f)~~Q::.!,ti't~!J.,,,.' ....." <; .. ' ~ •• ,;,,1.-.;: ..t;.;. ,' ••' ·, ,.,,, ·-.

11 Molna11ts and Product~ ·of lllsr.tia 53


"f:·:· = Pll [7 S4 lon 2 Ct+ Ii' c, 2 - 2h ~, + ~4 1.d~
CJ.
..
, ·1 1 , . 1 l 1
:.:. prr//" 1iin· a. - 1311· CJ. •k- 1=-
Ex, J7. Ifs a Ax 2 + Bi+ C;;" - 2D.rz -·2E.x - 2F~y co11sra11r, be th~ = ~ .
L 20 1 .. ~o J 60 °' (J 0: + 2) of 1/ie m0me111,1I. e_//ipsoid at 1he centr~ of grav,ir;i• 0. of ,;i bod,r
10 any rec1r.111g1t/ar a.~cs 1/lrortg/1 0, Jhen prove t!iat mo111e111al
.t
\
l "I ,' , 1 -, h
:o ,IJ/1• (J 1an·a +. 2) = -·
= ...!..
w M (3a· .·+ (·: ton o: "' ellipsoid at the point (p, q; r) is '
8 "":S..1.1. of lhe cone nbou1 O Y S + M [(qi "". r)') 2 + (1;1' - pi) 2 - qx) 2] = con.sf.
= !is M.I. about line parallel 10· 0 Y through C U:.e. diami:ter· of bnse) where M is the 11iass .of tile 'bod)•,
-0 M.!. uf 101~1 mnss Mat C about OY . t' . .
+
Sol. Since Sa Ax,2 +By'+ Cz 2 - 20)'Z ~ il:tx - 2F.:q = coostunl is the I·
··I ,, , . . 2 .. J.\ ' .. ' . , equation of the momenta! ellipsoid at the centte of.gravlty'O of the body
= 20 M (3a' + 2h·) + Mh =
20 M (2Jcr + 2h·), .
referred to the rectangular nxes at 0, lheref~re A, 8, C life Ille m<Jments
Noi• M.I. of the elemen1ary disc abou1.0z ... and D,. E, F nr~ the products of inertia of th( body about the rectnngular
= lts M.L about AC+ M.I. of iis mass ·sm at l about OZ axes througn O. · ·
= ! Pi}om .;_ OC2om = (f ~• tan 2' o: + n2) pnc,2 Ian• tide, Let.A',8',C' be the.moments and D',E;,F.1 the prodlli't.i: of inertia
of the body about the ,p_arallel rectangular axe~ through (p, ,q, r), _If M is
·=pit (¾~ 4 1on 2 a -1- a 2~ 2) d~. the ;mass of tho body, then' · · .· ,.
.-. ·ca:
M.I. of th~ cone about OZ A'~ M,l., ,about x-axis 1hrough. C,O, 0 + M,!. 9f mass M, at O ab9Jt the
J " . (t1;4 ton·' ·oc + 11•~
.. o;.p11
' :2 ).1nn. 2 0c,I(';
\
ax.is parallel to x•axit through (p, q, r) '_ Ii
·, j ! '=A.+M(q2+r2), .' , , r.;
l;
~ pror1 :{ .l..
10
Ii' 1on 2 oc •+ l u'
· J · C(
i Similarly, 8 '=.B + M (? + p2), C' = C + M (.p'·+ q2 ) i:
D '= D + Mqr, E' = E + Mrp, F' ='F + Mpq. ff
·1 ., , , ''13 2 ...,
"'jo' M i3h· 1.in· et + IO a')= ID M(I , (·: 1a11 et= a/11) Hence the equation of the mbmcntal ellipsoid al (.p, q, r) is .,:)
A i +Bi+ C'z 2.- 20 'y~ -2f.i:x-2F'X)' = const,
1
. •.:
111~ coonJina1es ul' C.G. O of the cone nre (a. 0, h/4).
or [A +.M(q2+r2)Jx2+ (B+M(? +p2)]y2 + [C+M (p2+q2)J z2
.-, D"' P.1. of the ·cone ubou1 0 Y nnd OZ
·- 2(D + Mqr) yz - 2 (E + Mrp) Z,l' - 2 (F + Mpq) X)' = canst.
= P.l. of the ~one about lines through C parnl\cl to OY ;ind
OZ+ P.I. of the mnss M nt O about OY and OZ or (Ax 2 + By2 + Cz 2 - 2Dyz - .2Ev: - 2F:q) ~
f
= 0 + M , 0 . h/4 = 0. 2
+ M [(q2z + ?y2 - 2q0•z) + (?x~ + p_ 2z1 - 2prxz) · tI•
£ = P.f. of .the cone about oz·and ·ox + (p2i + q2:i - 2pqxy)J = const. ~

= 0 -1- M · ~ · a "'.!. Mah ' or S + M ((q{- ,j-)2 + (rx - pz) 2 t (.py- qx)2J = c9nst. H
4 4
nnd F = P.1. rif thc'.'cone abou1 ox·and O}' = 0-.H4. a. 0 = 0
·1 § l.23, Equimomental Bodies. .
-Two systems or bodies are ·said to. be equin10111ental k/11e1ica//y (()r or t~
-~•
Hence the cquutiun of the momenta\ ellipsoid at O is dynamica/1)') equivale111 when mo111en1s, r;J_/1d products of 1!1enia of ona J)'slant
2 2
Ax + By" + C: - 2Dy_i - 25;:.x :· 2FJ.)• = cunstan1.
I·, 22·1. 2 ,,
or QOd)' about all. OJ:es .arc .each e,quiit: 10 Il1e /HOfllen/J a11d• producis of
inertia ofJlie oiher •)•stem or body abc:iut. rhe sanie axes, · '
·t
· or -20 M (3a· 'I- 2h ) ;.· .,. -20 M (23a + 2/i·) )'
13 2 2 I · .
.. Tho nccessnry and sufficient c:ondltlons, for two system( to be
equimomentnl annhat : ·
~t
+ -10 Ma . z - 0 - 2 · -4 Mahr,x - 0 =·constnnt. . 1
(!? the centre gravlt1 or lheftwo systems is the same poiriti
~
1
2 2
or (3a ·t 2h ) xl -1-·m,1 .,. 211 2? y2 + 26a 2z2 -10(/hh =.canst.
2 (11) both the systems have the same mass: and · .
)
(iii) the two systems have the ·sam; principal axes and same principal

·I
moments about the centre. 9£ i;rnvity, . . :t,, . ••·,

..
·1•·
' ~··" . ·'
. '
':1• ' --..•
, '~;.

i'
i;.
f!
'~(•; ;;;:·::;: : ,, .. ~l •:•'•;·~·' '. , . . ", (::.;":: '.!' :•·1:::•: !:·r~i~::!:~?~~?~-~~·~: •:·::;~m:m;r :·:~;~~1:-::::-::-:t;~.~1Jf:;~f~./!~...:~'fJ;?~•J~r: ~'.•'i~'?"'•:-··: ·::•,' ·~:·'~":-~:~: ·~ :•·; :·: t' :\~~l~mm.~~l~~:,ir;::-:~1·~~~~~~f1Sit?.fl'~4~~·:.,,'!.;.~,,",\f ~~iW•'•>':---"t~;!;"t~~~~-.u.i:~•~v,.~1~:-:m:,·~)MUttMl~,JH; . •
1

' :'. ~; ,; :(i'.::, . <i:'.: ·.'_i:·,;.,:·:,.' ·,'::_.::·•.::.?;: ::•.:<.·::· ., ': :~~:.:~· :: .


.,\;! ;\tt,'1• • '~·

tr,.~~fTJ'1{Bl{iUl'nlIJWfflmlfflf:llm.rn!S'i
- - - - -....~l'", ...

. . , ::;·:~· :·:·: ::: .


•~:1_,:•;1.,:,, r ,, ,_.;,;:,·
h1

·11' ··• ""•••--•-• .. •·•---....,..,,_,_.,~~.:..:..~.;.:.Ul.:t,.J.!.'..:.1.;Jlwi1Jl:t•J}

n ''i''""'''•'"-C<<:;,,i~,_bU'<%,:"-~•.,w,:,, .... ;,,: ,


.1•:,,~H!.l:SU!th., : , .:..:, 1~~.',,,;,. ".,v .•~ .• ,,,,; ..._.,,,,,.u;,1t~•.•1, ,'\•••
}:i 54
!J '
~ ...
·~ ..
;11 of Rigid Body
;:, Momems and of /1ter1ia
ti § 1.21!, Tiu momc:nts a11q pr~~ucts of inertia of a ·u111'foii11 rria;;gle 55
'I (lny lines are the .•rame a.s ·the 1iwme,,v and producrs cf inertia of
~
,I
par/ides, placed al rl,e middle points of 1/ie sides, each eq11a/ to
of the mass of the triangle. ' ' .
= ((1', +'. 12ND'J ./Sx.
;i .-. M.I. of.'tl1e A/IC .ibout AN
Let AD be the median of a trihngle ABC of .mns·s M. Let AN be : .trinngle
1 perpendicular on·BC from A, AK perpendicular 'to
lrinnglc ABC nnd AN= h•
AN in' the plcne 6f =
i
Io -'lt-•,(/1"
t, 0111 : ;
1211)! '· ·
. . .
i
'l-

.', M=f BC ,AN p ahp =f I 1 ; :, : M


= ~ (ci2·+. I2ND 2) =-~ [a 2 + - BN'/J
Consider an elementary 48 · · 24
strip PQ pp.rnlle! t() BC o:· • I,

thickness OJ/ and


.x from A.
From· similar triangles APQ
nt n dlstance •. V T K_
= l ~;12:(
a
2
f-czos$ JJ
• ABC, we hnve
l:Q ~ I\L ...
.~Mr a2)12;(::.-c ii2 +.c2_:-b2 )21•·
~4 t • · 2 2ac
BC AN
N D(W3)....:...._ = M a2•./2.2 (&2 - c2)•]·,r M' ln,' + 3(1,2 -
AL · xa 24 a · 24(1•
,:. l'Q=- · BC=·-
1\N h .1nd. P.l. oC: the triangle AB@- about AK and IIN
Txox,
'••I
._.,,.,.,
Now mn$S of the strip PQ = pPQox = = J 11 (Al. LM) ,ra'.x = J_,l.x · !. ND )Qh.E. xa'.x = l pah 2 , ND
(l ' ,0 1I, l
:, M.I. of d1e strip nbou1 AK zd1 M/:. NO= l Mh (SD- 8 ) = J.l Mh (a/2 - C COS 8)
. 1 ·1

= Its M,I. nbout PQ + M.1. of its mass om at ils C.G, (i.e. middle point ;or I ( a' a 2 "-c 2 -h 2 ] ·I Mh
PQ) about AK . i =-Mhl--c·
2 , 2 2ac· .
.· =-·-(b2-c2)
· 4 a · ·
... rn
=0 + x 5m =3/; ox.
2
x3 Now we shall -:,,nsidcr a system bf three particle.s cnch of rirnss M/J
ui the :middle points D, E, Fol' the sides of the t:.ABC' and fi11d !heir
:, 'M,I. of the ti.ABC: ttbout M:, = f/~!?.. .t3dx products of inertia nh(1.111 AK nnd AN, ·
pariicles each Of mass· M/3 a1 D, F., f about ·,i I{
= ¾pah3 "'f Mh2,
1
.. ,/ I J
""I
.M. [ 2 rli 2 J .
Also M.I. o( the silp P(J about AN
,. M.J. of the strip about parallel line through its C.O. M (middle pciot of
M · 2· • · ;
y ov E.7~ ~¥ I h'l ·.··,
T3 FS 2 =-S:
h T-,l 2) T(- )
2 M1i· ... (,; ~1
RC) + M.I .. of Its moss om ot M aoou1 AN M.l, of ,!he three particles eaoh of mass M/3 a< D, !!., F about AN
« !!J. DN2 ~ !!J. E'J 2 - !.':f.. F/J 2
.. . ·... !1'(/'J2 j, .. 3 : 3 ('
.
3.
=t (½ PQ)2 om+ ~Ai2 om =b·
7 •

Bui from similar triongles ALM nno AND, we have


~ + LM 2 · Pf- ,\'OX = r :
1(8~"' BN) + (1 CN) 2 + (f B.l\'J"l
2 I·'

bM Al !. . _!,_ ,
M [( r, , ' -~ 2]·
ND= AN= h " LM - h ,\D. = -·
3
--
·2 I
C COS [1
.
J + •·4I .(/, ~OS (.- )", +' ,4-I (r' COS 8).
.·. M.I. of the strip PQ nbour ·AN M , .,
2
I a /2 ND
2 ria ?] = 12 !~~ 'i- 2~· cos B) + b cos 2 C + c 2 cos 2 BJ
1 2

.
"' [ - · -
3 41i2
+ - .
1i2 .
r =xox
h M
- 12 C + c cc,s lJ - 2c cos + (b' cos 2 C + c2 cos 2

~~::·~~·&m~:iRgm,~mrmnr,umrn1~::~~r::1fm~m:~t:~a•nnnVRfM!'{rn~~mmn~r~1·~J}8i1m.•nnHttmnn111tiffttnmt1r1Ii·l:.?{m~1n~:-:~ •~t:'.~: ~ ;_:: ;~ .~


.... ;,,_,.',,,· ,,.,:#t ::,,·,, ,,

f J,\ 11(1111/l',\' IJJ l<tgut l.f11rt., JV/(.}l!/1:llJ;; (,J//ll I f\Jl.-4\«.. •.i ''J. ''..'.":_'_'..'."

Let LM he nny \inc. ll11'0Ugl1 ll1C


"' :/.i i{i, ~"' C ~ {' co, {J ): + (/1 <.:!:, (' - (' <.:<lS
,If
"-c~ 1,11 "!'
. .,
c - c ,;<1, Br+ r,c d1, 11 ~o,
..
+ ~/11 '"·' II w, Ci .

'
I vertex A und ,1n lh~ plnnc i of the
triinglc A/fC. Lei pund y he. 1hc l. T _ ......~~-#{,M
. ~ ·~

i; ., , ' , 1 , 1.) l l
! ~
1 · • , I ' j'
IIi ,fatanccs or the vcriicc, /J ond C
,he line LM, i.e. BT= Ba11d
..-;- ·. \
~: F
, - ', \1
\ 1 , e'•
ll h· ---·--c ·----,
M!) · 11· + Ii' - C o· + C' - h· ti" + c· - . £1:t,.!!t.:::~ CK,.,. 'f. . , .,. Jrn/3\ / '1llJ ':
:::-11 +be·----
6 2ab 2ac .. ) , · Uu: ! 2ilii:
; .:
I • I ,I' '
:, ' Perpcnd1cu llr' v1swnccs o
1· B ---Lo'{r;i;jj"''
=---, I ,vf r -. "I , '1 , ·•i , "'1·
c·t I I: 1
4(1;• - c)' + a - (11· - D.,E, F l'rom~M nrc as fol!<f.vs ' C
2..la"..
A1 r . "I
, ·
J1.
i
OM= CP +)');EN+ CK=½ y and fl P" ½BTt \ p, t
= ":i".i-;;:i La·• J(b· - ... (5) :. M,l. or the triangle ABC ahoul L,M · ')
= Sum of M.!. of' mosses ml:!. each ut D, e. £: aboul LM .•
Jll(!
AN
=, ,\ .MD
P.1.

I
q( 1hc three pani,lc, ~u~h o!' niass M/3

M ,JI
,,
A,,+, "' . Ar I - )M Fti . A H.
a1 0, E.. i: abi\ul and
8
,.

1:

Ii
= !!:. · OM'+ !3l ; EN2 + !:l · FP 1
.3
·mr1t
3 · 3
t,1,1m2•n
'I
·,I .!
!
=,3 L4 (~ +Yt + :r:Y- .+ 4 ~- = 6 (~ + 'f + ;,Y ),
0
. I • . , . r
= !:'3~ I
I .
r ON, Ii+!, CN · f!2 - ! 8N, IJ.]
. I
= -·· Mh (-1( 130 - ·8N) + CN - !1N) = -•
2
(
=
a
Mn['· 4 • - + CN - 5!!N
1
]
Mh (4DN
· .
+· CN - BN) r
. Ex, j9, If o.: P, y be lhe dis1ance.r of the· verticas of a u11ifor111 1riung11/ar
la111i11a ofmass m jmm any line In .(ts plane>' prove that file M,/, abi;,111
I
'
I'
;:,
· 12
I•' f
12
<I
2
. ] •. . ., . UM n
is¼ m (a.2 + 2 + y- + ~y+ ye1+ o.0), ;·
~
= 12 Mh I 4 •
2+:)co$ ,c - s c co, a .1 deduce rlid1. if h be 1/!e d/swncc of tlie:ccmre of /11crtia of tile tr/'lmgl" •:\

I~ a o·1 + c'' - /1·l 7


the liM. (hrm M.f, abo~t rlii;· line i.~ ·
I a·l + I>'' - c'I
=- ,11h1 ~ ,._ +-h ---_--Sc· ----j ;1 111( c:t.2 + ~l + y2 +
l2 . 2 2(1/, 2(1(' ' '
Mh • , • So~ Let ARC be lhc
=Ja- .(b 2 -c') ....(6) ,,,
. lamina of mnss 111 nnd
l
From (l).'(2), (3J an<J (4), (5), (6), ii is dear \hat _1hc-mom~nt~ apd!p.todu~l~ AL, BM, CN tlic pcrpcndiculm ~
of inertia ·or 1he 6.ABC of mass'M about AK and AN.arc \he $aine:ns th,isc from A, ll, C on· a line TK in
·o·f lhree particles each of mass M/3 pio6ed al .the middle point~ :of the t•, its pl~nc, 1hen
l

;'\
sides. . · . .. · · Al= o:, BM·.,,~. CN= y.
Note, Also· the lwo sys\em~ have. the sume mo,s M and the sanje cenlrc tf OP, EQ, FR arc the !:
'
'\1
of gravity. · . ; perpendiculurs from lhe middle (:
J:
Hcncc.lhc trinngle of mass .M ir..cquimomcntiil to three particle~ each or poin't~ D, F, F of sides o; TK, .,I 1:
mnss M/3 plnccd ut tl:l~ middle poinls of the side.~. lhcri t!
t'. I i
DP= (BM+ Cf,/).=½(~+ y) L. f' 0 N K :;.
EXAMPLES 'i' M A i~
i1
Ex, 38. Oblilin the -n10111ent _of i11erii<i for- <J 1riang11/ar )am/na ,ABC EQ = t (AL+ CN) = 'i (<l + y), :i
a//0111 a s1raisn1 li11e through A (nr any vartex) i,i tli'e plane of the!ttiangle, ::
;FR= ½(AI,+ BM)=½ (o. + ~!· ;;
. Sol, Let ill be the mas, or lhc lrinnglc ABC, tl,en. thcj lrlohg:lc is Slncc lhc triangle is cquimorncntnl to the lhrcc particles ·each of ma,s ,111/3 t
cq\limomcntal lO·:lhe three punicles cn~h or ma~, ml,3 placed n\ lh'c ·
D. £, F of. ils sides, · · · · · I
.,placed nt the middle poin\s {), E, F of the triangle',
i :. M.l. ()f the 6.ABCnboutfK' i .
-= Sum of M.L <lr masses ~m ench at D, E, ~bout TK .,::
.,, f: ''.;

1':•

~-..
\:
' I I I ' . ' . i:
.........,:••1:l'Hi~~,1\•, •• /;•,.~•Y,\!11.\'•'''";'"')l.'"'('\;,'-:•tr•~~f'll~'t<J~)Htt'~~:-:-•,♦l\",'\:~••'.'!""'<J':":~~J~--.•-l\i;.~t,imlf•"i.H(\'"-'i'-"'>l'\llil""'i½J"'-"••'.•'•l'l•··"'~!~," ........... rs:1,,r,t:'r,~,~~~,R,Tii~i.:t~i:,.~1,1~~-.""'11~~~~··""~1,~N::tM~,,..,~1••J•
. .. :~~~!it••;:.· , , • ,; •• ·•., ,·,-.1tl~.L:1r,,-1:,,.,,,,;,•.,1;.. !,'li•\•:11l-!11>;!n,,~::,1M~,..,~~••'.•,··•.,,";,,:,;t,;1:,,.:,.,:,:·1;1,\·· .. , ; , , , · . , · . . . •.-... · · ~" ~-· ,,, , .. ,. , , · , , • , "'" , • • •
0

,·,·.1;,;:;;:,,· "',;,,I;,: Ii~(•,, ·<:.,;:H'~••i;,;,. ..-.;; '.-~~;.:,,:;: ,, ',•:;:.; :::.;:•:;,.;: . ::;\it:(' '\::\:::::· ·:::::::::

..., J.>o.;,ll)XU.-.il'h,,,,,J,..~rJA:!W~._?,ij~i;,":Wi:f

:'.".0!{,:::::::-·.
__._·.-~--~~·-·_·__ -_;_,, __._·._.__

., ,~ "'., .... ,;. ;. •,, ,;•-;n;(1:,,:):,; .,::~~:,'.,',



' ' \ i ~:

' ,~i,~-.:::-,~,, .;.;. ,-.,.


fl~::.:
;.,,
1;,;
~.:; '''"'"""'••"'1"••..•••••-"•""l•,•,• ...• .......... <l>t',... .._'>\h•'I,',/,..,,.
··~•--.. ~ ......."'.:..,.:.',.1; .. i.: ...• ·.1:,~; •..:.~,,:.;;.,;:,:.c..•.·.:,,:, ...
·;:..t ..;,.,,,, ~ .. N :,:...:• .::.::.\1\,, :i11.J::!,'.•,:,•
• •. :· •• :!i:11-:..1~~
. ,1,u1.:,',

..-~ Sli'
DJncum'cs of Rigid Body

+( 1JcEQi +(1J(F(?)'
~11.1
·( r)·
m ,., , m l
(DP)
, m I
2

2
.
Mo111m1~

1hen ma.s's qf
, I
Products
.
of lllmilJ
:\9

,._,_ (..,+y)·+- ·-(CC-rY)• ;-- ·- (~ + P) MBD = mass or ilBCD = M/2,


3 4 . ·. 3 4 3,4
fm (cc2 + p2 + y2 + Py+ yo: -i- o:p) ' NoY, th~ MBD i$ cquin1orcic11tal to 1hree
I...(I) pnnicles ~nch of rnass equnl to on~ .lhird
Deducti~b.. If '1 is th~ distance or the c~ntre of inertia of th'e the injass of the lriangle
J ABC.frori1 TK, !hen /i "'1 (CX - p+ y). A1D,, i.~.;M~D is equimomentol to the (m/6
. l . ,
:. From (I), M.i, or the .1ABC about TK thf~e· ; particle encl\ of mass IL_ , "
"'nm 2 2
(2x + 2p + 2-? + 2py + 2yo: flet~) r
3(· M):='!:/.1
1

1
I / jl' '.
at i'ts !he middle points A ~J,_.,, "

P, o and;s
-~f its sides.
"'t,il fo:2 + p2 +-? +(o: + p + -:-J2J,.. ~m-(o:2 + p2 + r"+ 91t2j. Simi)ai;I)/ ;1(\e t:>.BCD is equimomcntal 10 1h,·c~. particles c:,ch Ill mass
Ex,
1
40, .Show /h(,{f a u,11iform triangular lamina of mass m is 1(~¥)1::/¼ ,'4 nt the middle poims Q, Rand ()_ t)f ii$ sidc,1.
eq11l1 1011/Mtal With jhree /Jllrticles, ench of mass mil i placed 0/ !he (,/l/!;11/ar Hence:ihe pnrallelogra.~ ABCD of mass M J~ c~uimomcntal w tic pnrtklc,
poi111s a11d n /Jarti,:IIJ t1/ mau. lm />laced (,// the ci:'ntre or inertia of the
triangle. . ' . .' I •. ·, I each o;f m.. ss idt6 at th·e. miclcilt p11ims /', Q; R, S o( rhe sides 111,d riar1icle
of nln~s' f /4 + ! M = 1"1.1 al O (the roint of interscclion u( Ilic cii:,gonalJ),
. Sol,. (Refer fig_ of E.>:. 39), . . Ex.;42. Particles eq,,al 10 OIIN/uarter of the 111C1Ss of ,11, ,,l/{i>1f.c
tr l'X, p, y are the d!srunccs of the ve11iccs A,,B, C of triangl~ ABC from a area are p/1Jced at Ille middle points of the chords joining the ~.,1rr111i1h•.t
llne TK in ils p!nne, !hen
of r111.i· pair of conjugate diameters. Pro"e 1/iar thne four porrirles -are
M.r. of the trfangle ABC 11bou1 TK I ,, r.q11imo1i1en1a/ to the elliplic orea ·
"'~111 (cx2 + ,02 + 'fl + py +yo:+ exp) (see Ex. 39) Sol, Lel POP' and QOQ r
·i"lle C,0. of the masses m•'l2 ench at the -points A,!), C i!nd a particle lie ih.e co~juga1e diameters of
of mnss jm placed ~t !he centre pf inerua of the. trinn.gle is the. same poi11i an elliptic' area of mass m. If
ns the C.d. of the· triangular IAll)ina, qi is \he eccentric angle 'of P
thep ef~chtric angle of Q is
Also, ~uni of the masses of th~· four pnnicles.
.i .!.m + J.m +J.n, + 1111.::: tn3ss .:,( the ti.ABC.
tl ~
(1112 t ~). :
t2 12 it , , :. ;: Coon:linates
i I
of P ore
nnu M:Z. of the four pnrtic!es about the line TK .. ~ (a ~os qi, b sin qi) and
2 + -1.111
"',!.111
= .Lm
IJ
, Al}+ --
(r1. ➔•
2 2
1'2m . Bt?/
i
, CN2 + 1mh2

2)
1,
0 +i-l9h
fl
i'
coordiriath of Q are
(a c,1s·($Jtta.), b sin(<!>+ tta.)
A'
A
-~
II
or i(- asip<!>, b cos¢). 8-0
rs lm
11
2
(a -t P2 + yl li"'Py
'
+yo:
...
➔ C(PJ, '
· Coordinates · I')/'
h=l(ci+P+y) p' ~re; (·•'a ,<ls qi, - b siu q>) and .
,,, M.I. of the MBC about the iirlc TX. ) ·.
Hence the triangufor'lumina nrd the four p3rJiclci nrei.equimomentnl.
that of QI are (a sin<!>, - b cos Q), t
!f (~ 1,i,i: 11 , (x,, h), (x 3, Y)), (" 4, y4 ) are the courdinatc$ of 1h~ n11udlc
rtx, 41, AllCD (s ·a. uni/or~, paral/r.logram of i/rass M. Al the middle §;
poim.r of 1/1c fa11r sides ar~ Jll<1ccd particles each of ma·u M/6 and ar th~ /.,, M. N, K. of Chords PQ, OP'. P 'Q '.Q 'P r~sr.:cclively, then J£
inter,tt>i;tion Q/ th~ dia(io,uifi. t1 rmr1ir:J, o/·mass Ml.3, sho,v that the.re five x1 =.1~ (,;o, <!> - sin$). y 1 "' 1b (sin$+ cos qi) ·
{l(lrricles u11d the pom/laiogram are cqulmomcn1a/ sys1im1.r, .
x2 = ~ {c (cos~.- sin qi), >-2 =fb (co'~ qi - sin$)
·
Sol, Let Al/Cl) h~ n unifunn pnrollclogrom, or moss M, and
'P, Q, fl, S the middle rminls of il~ .~ides . •J = 1,aOin (¥ - ·cos(¥), )'3 ,:.,-ib (sin t + coi ¢)
~!\cl x4 = fa. (si~ .t + cos ·<J)),. y' 4 ,;= fb. (sin ;J).- cc,is $).
........ «,,.,. .,
_{''~·r v•

~:~·m~~l']~❖t~::·1v.:t~! t:i :;•:r-m:::n111m11m1tm rmr .11~ :\:t ,r: ~r 1 : l~ :~~ ;:·:;::~rruu1r11rnmmfffl1H11?r }'lHl~!ifH't»n1nH:1n,11mn11mu"mir1 ir:H:•ff~r1r:·,: :::~·:::: ir.:! .: :i'i' i. ·::i :::n \; ::·: :: :.:t :~:·1:.:~m·~·mt·1,:%: :~t1.'«f.'.~~~~M}~H;.~J..~~;:;.;:: t:~:,: :;::-:•r..:::,Si:¥•~Nm~:N::t mP!:f•'.iKJ•:~ i:: 1:-~1t-J,:l-:::-1:.,;,,.. ~~zmm.--tfl·::.,.:::i:.:.:,s"stm:1R1111
•.!,-.',I c •,,' ••. ~~•'.i:,.\,;",;n'.,'.h'•' ,c,'•,~•·~i,' '.' .'_,,;,,;-:,,:.;.:, ,
' .', . ~:-., ,,; :
!nlfflW•/:,•;::ec: ,;'li,C"i,HUlM,mwm1•~,;,, ......... ,, •. ,."·•··";".,., .. ,, .. · .. • . . . .• . . . " ... .
~ii~~!{~:~ n!U: ii ifr~f;gmnna~mgm; m~~tH·)~ H! 1 ~i!•;·1 ~},fo5~.~~1~:irr:;~l tJ:a:t~-!}h:,.!b, ,.; ·.• ,: ,,. ,1;_,: ,;, ,t i,:~•;,~i,;,_,~tMs~tX~aJHtJWm ;!/)'~) m.'t ,c~·; ,-1•,>,-,n,

(:'.l Dy11m11h's oj Ri1-;(d. Body


·:;;•r 1
ol"ii1n.,s (M/11) nl the, middle r111in1s nl' its si<lc1,,
II' 1.i' ,V) ,1r~ the_' ~1umlin;il¢, ol' i'nur · p;,1·1icl~s ~ach · 111'. mos~
11114 :11 /.,·M .. N. K 1ll,•1i
.~" ; i.r 1 = .I',
,

i.t'. C.Ci.
l

11(
,.
•· .r: + .rJ l
.

1l1~ f11ur·p:1nidc., 1,
. .
"'0 ;i°1id i" !.
.
.

:ll
,I
r,· 1 + •:., + ,·, + ,·J) =0.
' • • "
.

O ll'hi,;h i~ aim -111¢ C.G. 11[ 1h~•cllip1ic


' • '
:. ;M,J. nf the 1rinn¥lc OliC, ;1hn111 OX·
M' M ( ,. \l
::--·•O+--•
'.111 : • J/1 .4
l''
-J +---·I i.1
311 4 )
\l Mc!
=--=A 1.
2411
:. 'M·.!. or !lie 1rinngk·Of.l , uhout OY.

1:1111111:1.
AIR1, !vi.I ,,( 1h~ fnur r:inidc, :\I
• ,
L. ,\·/. ,\', K, 11hou.1 the 1n"jor
'
,l'.xisl i " ~M(.cot 1! )'M(
JI/
le
I II
+ -- !c cot
311 l
fII.,~M(
J + -J/1 \ !r1 C\11 .!:11 )"
=1 .:!!
~-
r\ ~
I
- \ ; .,. , ..\ +
. " •. '
,-3} '~
I
Mc , re I c n
=-•·cnt•-=R 1 •• O'l=-co\-
m 1 ~ '\ : 2 ' 2 8n II 2 1;
"' 4 ·-; 1,· ll>in lb+ ms¢)·+ (~ns <l> - ~in(!)) ~ (si~_·<i> .. cos¢)
~nd P.I. M !he trinnglc OBC abtul OX and 9Y . • ·,
+ (sin it,- ~OS$)'] = O.= F 1 (',· t:.OBC i~ symmc1ricnl Aboul OX)
= {1111i1 = M,L nf 111,:: ulliptic nl'~il nhnut mnjor ~xi,. Lei OP be .i line inclined nt an nnglc crni OX, 'lhcn M,!, of
Si111dnrlr M.L 1)/' 1hc rnur ·panic le, .11 'L. t-/N,K nbout the minor axis t;OEJC about OP · l
::: •;IIW:::: ~LI ill' 1hc dlip1ic ·,,re~ ah1:ut n1.inor n.~is. 2
=A 1 cos" l).+8 1 sin ~-2F 1 ain 2a
and P.I. lli' th, l'nur p~niclc, M L. M, N, K ~hotit OX, OY M , ,I ,• ( Mc2 , 11 I . , . ' ... (l)
= .-- c·Jcos· a+ - co.t' - ism- a
'.' ~111 (.1·1Y1- x2."2 + ,1\1'_; + ,r,1r4) == 0 . . .( 2411 . 811 . 11 ) '
::. P.I. of the clliritic :,rco abnu1 OX nnd 0.(. ,,· ; The .I. of thp other triangle~ ahout OP are obtain~(} hy rcplnciniJ,
·Tln1s 1hc four roniclcs c~·~h nf 1nuss 111/4 at l, M. N, K have tlle[xarne. a by a+ 2n/,1, o: + 4 !1111,....... in (I), ko,ces~i vcly, then
mn,,, s.11iic C.O. nnd lhc ~nh1e p!'in.::ipul moments a., thnt of the ellip(id area, M:.L of lhe polygon abnut OP .
Hcn.;c the pnrticlc~ ore' c4uirnomcntn! _lo lhs> elliptic area. , : 1 : : ½1c' l.. cos' .
•"~-- O'- +
. '. .
co~· (o: + 21\'/11) + co.~ 2 (ex
. f 471/u) + ...... 11 tcrrt1s.
J
Ex. ,/.1, Sh111v •1/1111 1l1<' M.I. of a ragulnr pol)'go11 of II sides ,abo1i1 any
r; . .
. 1 1. 1 . . Mc
2
2
.rll'l!lg II I/IC I l/'Ul/8 1I IIS CCII/re IS-;;--,-. I
+ cos
.
(271/11)
O
. I
. 't
,
) ' w11ere II IS I IC ,u;m 6 er
l: ,
!,:.
l C

+ ~ ' • ~oi 2 1! sin 2 o: + ,i~i


8
' .(

O'-
• I \

+ 32!. 1.;. ,in 2 (o: + 471/n) + , , , rr terms


]

~ 4Jt )l
44 - cos (<.AAI . ' ' 11 1
of sides a11d c is rhc /c11;,:1/1 of each .rldc,, · 1
'
SQI, Let ABCD ...... A he o. regular polygon of II sides each' ofi 'length
:;;2n· · 2 ! c:s 2o: l +
"' .24
! [ '.'
I+
. {
I + en~ l20: + "ii;' J+ """ n terms
·. ]'
c. Let O be the centre or the polygon · · ' · -0 2
l { (
~
nnd lines OX (hisccting BC) ~nd OY + -Mc
- · col", -11: ·:;
811 . n ,
· 1 [ I - cos 20:
.
l+ l - cos
.
4it )~ + ·... n ·1ermd
2o: + ,_.
II ,
· ]
• ·
(pcrpundiculn'r li'l OX) be •tllk·c.n. in
its plane ns lhc axe, ()J X nnd y Mc 2

t
Mc' rt
rc,pcc,ivcly. ==-{ II +S) +-·-cot 2 -( 11-Si ,,.(2)
481! I 611 II
tr Mis 1h·c nrnss of the pnlygon where S = cos 2Cl + cos (fo + 47t/11) + cos (2Cl + 67i/n) + ,., .. , n. terms
(Min),/
0 ··
then ii cun b~ divided intn 11 .isosclc,
itln ,,•~ .• cou_2o: +(n'" I) 2rt/n L. sin 2rt _
1ri;1nglc, cnch or mass M/11. '
nr · i,:11.~dc~ \irnngle . -+- ' sin (211.111) ·· ·
"
1110» 0 '•. ,Cl• M.l. or 1he polygon about OP
OIJC,. MIi/, '"
(Ml~!•' 1 2
Alm L80X = LCOX = ¼ L.BOC Mc ,l,fr: · (
cot2 -7l ) . (
= ll (2rcl11) ~ Ml,
.
' A/ I
I 1
=-·-.·n+-•
4811
= Mc.: •~in~ (!1/n) + 3 cod~.] ..
16n /l'
'II
;
.
· f

Nqw lh~ trinnglc OB(; is ·1


I 4S . sln 2 (!1/n)f- ·
Ji~:

.II
cquininml'ntnl 10 lhr~c pnrti\;ks cnch

;- I
l
\
~-..
•.,"·,

.·,;,.:;,,::,'.i.'.,r;,;,:,:•:ir,;"•':"'. ., . ''.•~~~,:~ i:•: ! 11 ,.,.,~_,, '',' ,;1,; i •, ,,. . ·: '.:,: '.:\i !!:;~:> ,:,:: \ :\ '.>,:'\·'' ;• •' ~' '.• '~' ., :: ('1-? ,:,! <J,:• '".:'~! :~•::~~-. ,,.," '' ;:,:,' .·i'·:, ' ~;·•.:,.'.,;.: ,'••·· -•·1, • ,.. : ... , ·,:.::,::--;: ;:,. ', \~<-:: ·'.·· ,::.,. "•:·.~•:M:,:::····· l .\.),,,: '' • ·: •• ;!,• •~!:::~~, . .,, . <\

~h~.....,~-.htii,,VA•.a),?.';'.'";'.'"Jl'a~.; f;.'c'h·..,,,;'!,~

~·-------~·- :-:::::<:'.:~-- • . .... . . ,;1 ',',, \'.•~·,i,


,•,,•;,.';:.,;.{,,:••
·,•I' ( ~.-.; ('
... ,..:'-:>

iV1
u
/'

,,,.w, _ _ _ _ ........_,, .. ,~'.,. • '••.','\',q'', '•,.;,,.,.,


___ ,,;,;..-,;.~,.:...::i...:Y:..:a•~"·. ,,;: .:: ,·.',::.•UU:U\.I:·
·:;:! .. ,;·,;::;11:,

62
Dynamics of Rigid Bod,1· Mome,!rs ai,d Product.r of lllutia 6)
-··
= Mc
48
2
l·'" (I - cos) (2M1) -t- 3( I + cn.1) (2iv11 •]
I - cos (~f,'11). .
.' ', :
.tnd,:iliou\yF=
(
,
l I
)'·-2 ·
, ·: "4 CF
::, !1..£:. ' 2 -t- cos (27i/11) ' the monicntul ellipse "I G will pass through,.P, Q .111d R. Al,o
24 I - cos (2M1) ! GD jiSjll/.: ~h1111ctcr ,1r 1he ellipse nnd bisec1 EF; .·, 1hc Ulngen1 111 P will
Ex. 44. Silo..,, 1/wt rherc Is a momenta/ c/1/psa O( the centre uf /11~rtia (1c p:ll'idlcl'. to El-' which is pnrnllcl 10 BC, Hc1H:c 8C ii 1;rngcn1 In 1'1~
of u wit/arm trirmgh• 11'/11(-/i tOHtlte:S the s/dc,r of the iril111gle m the ml'ddle momei1thl ellipse 1H P, Sin1ilnrly the sides CA on<J t\8 ,1rc !(1ngcn!~ ,,,, the.
~~~ ' mor{i~ntbJ ellipse nt £ MU F respectively,
§ l.2.5, l'rindpnl Axes. · .
Sol. Let ABC be n triangle. of moss M. 1--et G, be .its C.G. and
t>, E, fi',lhe middle points oflt~ sides, · :rJ f,ni whether a 9/ve11 stmighr line is at ai1.1• poim of /1.r len91h a
N~w !he momen1~1 ctlip.se n1 the prin'cipahaxis of a 111a1eria/ S)'Slt!/11, A11d if the line is .a princ,'pa/ oxis, then
centre of i:nerlio O will pos.~ 10 (4el&rinine 1hc 01/1er /WO p11ncipa/ axes, '
D, Eond F if the mo1nen1s.of interio ILct the given straight line
of the rrinnglc ABC about OZ be' lakcn as the axis' or z .nu
origin, z!
GD, GE and OF :ire cqunl to
MJr.4 Mk 4 Mk 4
Lei the :t'wi, perpendicµl;ir line,
Y' ~ _,.Q.l,..,,... .
I
-•-und- . OX and OY, pcrpclldiclilaho OZ ....~a\/::· -:-Jb" ·· · •·
OD2 OE2 OP" be ln~c!,.,as the oxci ol'·.r·:and ,. •• , I ..,,.
Lei the IYIBC be rcploccd by rcspaq,i'.vcl:y, . . . ,,•. ~; ': --.... X'
three particles eoch or 'mnss · 1M
pl~ccd a1 the mltldle points D,, E, F';
Then M,l. of the triangle ABC oboul AD
Now !Jr.t ifle li11c OZ be 1he
pri~ci'r,nl Jxi~ of the sysicrn n1
', ,vh;er~ 00 '·:: It. Lei.O 'X'
't~'
. /0.
/v ' '
:·-~-x
= (M/3). EN2 + (M/3) p7:i"' i l'f' ((1 c sin 8AD) 2 -t- (7 b sin CAD/J An angle 9 to ·n line
and O 'Y •'bti the
d.l.M[c2~!n 2 BAD-t-b 2 sln 2 aAD],. .:.. (1) nxe~. ,
ll . • I.
Bui in 1riunglc~ BAD and Cf-'!?, we have• ; o porticlc of mnss m nt the poinl P ol' !he material Nystr.m .
sin BAD=·~ and sin CAD ~!l!Lf.· and {.r ', y ',' z ') be the coordinates or 1hc poiiH P with referent~ ·
w.2 AD ' '. a/2 . AD sets •,1r uxe~ OX, 0 Y, OZ ~nd O 'X:'. o 'Y '. O 'Z'
· 11 sin B ·. a sin C then i,.,,e ha.ve .
2 •:;:;s-ands1nCAD= 2 ·
0

,•. sinBAD= AD· XI "'X co·s f) + )' .iin e, :,/"' - ,x sip a-t- ,l' cos ,€1, Z I "'· f:.. h

,·. from (I), we lfove . We know that the necessary and sufficieri't condition$ ror · the oxes
M.I. or
the t:JiHC about AD 0 'X 1 , 0 'Y'; 0 'Z '.to be 1hc·principal axeseif lhe s9stem nre that the product; .
1 of inertia of 1he system ,with referencedo these ates taken two a1 a tim~
::: ..L M [ la 2c2 sin 2 8 ~kh 2 sin 2 C] · - · Yanish •i.e. · ·
tl 4 - l ' I ' A D2 G '
I:my'z'' = 0, !mz 'x'., 0 and !mx 'y '"' 0.
:.l.M(ti +A 2.} -I· = - - · -I •
2 (Mt}) V{e have, i:my 't. I"' i:111 (-x sine -t )' co~ 8) (z- /i) '
ll AD' 6 AD~ =f(r171};t) cos 8 ,_ (I:mxt) sin O+ h(Em~·) .sirr 8. - h(T.m,r) cps e
2
( MA )'1 . · =D~o~·e.-Esin9+Mh(.hin9- 9) .. . ...(!)
GD=:f:AD
; J !!1ltt I:m.r - · km\'
= 54 'GD2 ' ' ( Mt:,2 )· I ) ·:' x: 7 . ·!:m M- ')' = "7,i" '
0

'"'

Similarly M.I. or tho, tr/angle nbout OE= s'4"" · GE~ r,be'.')·.; !m (z - h) (x cos 9 + )'.~in 0) . '
.. (r.;,z~ si_h e 1- (rmu> cos a.. 1i (t111x) eris e - 1i crmy) si11 e
... ,, . ·-~·
I

:1-::,;1",',''.
• ","_ • '✓,~•".:• ,~,•, l~s .•, ·-•~•.-.'\I ,.,.~~ • ·.. •. · ,·. -._., "•~·· '> • ,· :'1'i•~16•~•,,~•· ·,.

c. ::~,\-: :;;,,::,:},.:;•,, ~~~;;:~~)::;;~;;r~;t:{):,,, :, ; '/,:;·;~;t.:;\:~;~:,;-.~.tit;'i'(~:,:~•:i.i':•:, }'., ,'! ·.' ,.

. , , , " ...... ' " ' , • .,,,,, • ~. (', j ~· , . , , , . •.• ... ,.. ..... ,u.J 1,.,11,i.> i ,v,••an.i..i 1,1) 111!;1 H!.J \J.J

"'/);in Ei +£~·use - ,\1/i.(,r~•,,s O+,i'sin 8) .. /2) Hence if an axis passes through 1he C.G. of a body amJ is-a prln:ipal
,,nu !.ii1.r 'y '·= E111 (.! cos e+)' ,sin OJ(-.. ,· xin e,.+ ,I' CO$ 8) axis al a1iy po'int ofits l~ngth, t:1en ii is a princ}r11I _axis or ali points of
= Iri::,,,,.: 1 - t r,11.r'H sin eciix 0 ·+ (!:111.1:,·) [ens:" 0.-:·-~111 1 :'ll its. length. . . _ .
+.:. ~l - Em {i + ,1' 2)1 ~i;i 20 + (!,;1,1)') ~ns 29 ._'· Cor, l , Through each poim i11 rhe plane of a /cuirina, 1/iere exis1 a pCJir
;: +(.-1 •· ill ,in 2/l + /.' CM 29 of prindpal a:,;es of 1/ie lamina.
' ... (3) · Lei a line through any point O of the lamina nod perpendicular· 10 ils
':'{,;,,,. i::111,1· ',1 '= O,;i( ¾ (A - H) ~in 20 + :/."cn120 = 0 plane be taken as the ax.is of t, In this cnso ! (t coClrdinate of the C. 0,
2F'
or lun 2€1"' · - · - or 0 = - ln11- • - - I
• 1 I( 21' \
,..,(q)
ofithc body) = 0,. .', 0 0 ==
E. Thus eq. '(6) is salisticd for e'very point 0
ll-A 2 I U-,\ I in· the plane of the lamina. Also from (7); Ii= 0, ··, .·
I \ J
r\ht :::111.1 ': '"' 0. und l:n1: 'x ••~ 0.
1
! Thus Nxis (the iine perpendicular to th~ plane. of lhe limina) is a
prjn~ipal nxis of the lnm.ina 'at· Lhe point O wher~ lt in tepee ts the I.am Ina
I) c,1, 11 - I.: "n l:J + Mil (x.si11 0 - y,:us 0) = 0
~n(J I) 0 + f, C<)S O- Mh (x CQS 9 + l ~i11 0) = 0 an~ the ot~cr•two principal axes will be thc'.axes through O in 'thct place
)111

f. ,in fl - D c:o; 0 D sin O+ e Cll$ 0


on the lominu.
I
· Mh - ·-·--
.l' ,in e- ,. e
_ - - •. _ - - .• - ..- ·
1·0s x cos + ,, ,1n e e EXAMPLES · {

1..nu, ,,fl 1 = --£ sin D 8- ~o,


8 /) sin + F, cos 0
- -------
0 Ex, 45. (a). The 1e'11g1hs AB and AD of 1h~ sides of a rdcwnglc ABCD
. :i' sin 8 - y cos G - xcos 8 + 5' sin e arc 2a and 2b: show rhat the im:/ina1io11 ro 1,,8 'of 011t bf the principal
_ (£sine - D cos 0) sine+ (D's in e + £ cos.8)cos e _ g_ • , -1 3ab
axes <ii A 1s 2 lan - -- •
~ (x sin G- 5' cos cos 8) sin 9 + (x cos e + y si.n 81 cos e - x 2 2
. 2 (a ,- b ~
;\lsri _141, = ~,in.§1-'}cos d"' ~ sin 9 + ~c.os e.. (b) Find 1/ie pri'ncipa: aics a1 a comer 'of a square.
X Sin 9 - ,\' ,os 9 X COS 8 +,\'Sin 9 Sol. (a) Let AB and AD be
= _(.f.::ULfl_::_P cos.Ql( - cos 01.t.(D sine+£ cos fl)~~= Q taken as the axes of x · and. y

...
(,~in 0-yco, 0) (- ~os 6) + (xcus e +ysin 6) sin !l
c D
Mh = : = ·.::•., )'
TI1us,1hc condilion 11la1 the
syw:m nt some poinl of its lengt_h is lh,H
y

QXi~ oz mny
·

be the principal
I
:
.
J
.
' ...

·0( lhe
(5)
respectively and z oxis, a line
ih./ough the comer A nnd
pi.rpendlc1,1lar. to the plane of
the rectangle.
The.n A= M.l. of the rectangle
.
4.Y1 ,y

•, :o
'

\,,J·2b •G:b ,.... .'J<'


J
.g"' g)' I . , ,:,(6) about AB ,j -·1·1\'' .x
,\' '
= M.I. of the rect,anglc about A 2a B
r\nd ii' condi1ion (6) i_$ ~01isflcd 1hen 1hc po,ni o ·" ·where lhe lin~ OZ axis parallel to AB through
is lh~ nrincipal .ixis i, given bx C.'O. 'O'.
00' = h = .t.= J2.. .;. MJ. of whole mass M at G about AB .
A(; M"i' .. .(1)
Cor, 1. If a11 mis 'passes 1hm11gh the C.G, o/b bc,r/;. C1111i i., a pri11cipol
:lMb 2 +M~"=!Mf.{
) J l '
f .
nxis 111C/11,\' poii11 of /rs /r.11gh1, Ihm /1 is a pri11dp,1/ a.ti,· a1 all poi111s of Similarly B - ,M.1'. of the reclanile. about AO
iis length ':;:} Ma'+ Mai.= Ma 2• f ,
nxi.1 be u principnl axi.1 «t 0, then P = E = Q, :. frnm
Let 7.
get Ii= 0. W~ich implies th.il lhcr_t is no s11ch nther point as Q
=
and F P.I. of the rectangle abc,ut AB and AD
z-axis is a principal axis ot O and pm~s through th? C,G. of;
=: P.1. of the rectangle· aoout. axes piyallcl lo' AX, AY c.d. ·a··
t P. !. of whole niass Mat G nboui AB and AD ·
then 'i = O, y = 0 and D "' E = 0, · nnd fr.om (7), ,;ic see that h = 0 + M. J. b = Mqb , . 1 . I
1ndcterminnle. \ ffthe princi'pal axis nt ti .ls inclinp_}t an a11glc A(J, 1n:en
' -:·.

. ..
i
•·'..
i IU
''" ,,. ,,;~:,:~:'.} I '• :;~',,,,,, .... •·1•
....:illiim;:,:":N".'!~""""":''".''::!!!!rnr.:'l,"~i;,!!Jl!:J!"~'i11!:-'11m~~-tl:'i'1 ,·~"'f"l' ,. . ,..... , ,. •"·""" ·-

. ' ~~ ::-5 '.:,;'. . '' ;~ ·,:; '~ .. ;' ' ' ;}:! .'.{~i~i: :,,; , •·:·;;F~~:;. :,. :-:;:.: ·::'.•·· :;•=:::; .. t .. ·'.:~~;~~:,.~!}~ .::-:, :'.;:::,:· ·. ..' ..,~:,T:f:r:::·:~:

·.• tt.~.:W.PktlMCHR,, ,•., ,,1.,,~• •+J,!,\◊,<,.i;,.•,-!\.~t~

.·:.:~r:I!~•-:
'~··:·· ___ .... :

, ...,,.,,,.,,, ·,. ,. : :• . ~·:: ... :. '

··-1
I~•

~;/
r::,,:
,........ --~..,..-..v..~ ~ _ . ~ , . • • 1 1 ,.. , Ii ..:..,.:.;.1u 1•1m•··· ·uar:naounr·?.!.~·~1.:!.l.tl!u::,.1.~r.:l,.llLCt.JJ.1.;...... o!t...-•~'~•· , .... ~, ... : .. ,,, .... ,,~J~•·..,.;;,."·""'~~··•:~ ... M.,.,,...:.i..:.:..:..:,..::.......:.;AJ,t:::,;.-.•..t!':;,n;1J,i:,1,a1,~;:•lD<:' .-: :1,:.~~.-.1,•,:•,..~u.~ ... ·, ...... ;~: .•• 1.......... , .. . ,, .::,,i,,>1,• ••
~,•.
)~,·,
rJ.,.,
}
..-~ 66 D)'namics of Rig<'d Body- Mc1111e111s cmd Products of l11er:tla 67
:):,,,
:~; ton 28 = _Y::..,..., 2 Mab =~ Sol, Lei O be 1h~ middle
f,:
i.',1 l1-A :!.l M (a 2 - b2) 2 (a2 - bl) roint of 0~' where Ill> ~nt.l AE
·.1
from A AIY
,', 0 "'! tan·I _j!!E.._,
.
F
1he mcdion
l 2 (,i2-b 2)
Ly! 1hc lines OX
(b) Procec:d as in (a). Herc 2b = 2a ; nlong and A
.~ (m,:ii
(m/3) •. .
:, e ... ltar1
l
1 ~.,.!!.
4 , :
io BC be laken / :p': ·, ,Q'
n, the a~e:S ;or Jrefcre.ice. . , B O E. (1n/3)c""' X
Ex. 46, /\ uniform rectangular pia1e' who.re }ides.ta;e of lengths 2a, 2~
Lei ".Pjund Q. be the middle
has a. portio11 cut ow in -rite /orm of a !CJUarc whose centre is the cenirf
points . or; AB . and_ .AC
of the. rectantle and whose ·mass i:r half the mass .of 1/ie plate. Show rhdt rcsp~~1i°ve!y, 1he ?Q is pnrallel to /JC and is bi~emtl n1 1hc R where
t!ie axes. of greatest and least M.I. at a comer of the rec1a11g/e· make angles 11le mcdion•/\li' mee1s OY,
e, 11t + e wilrl' a side, where . · lf mi's 1h( mass or-the•A ~BC 1hcn it can he.· repl~ced by Ihm: por1iclcs
20 ...t_!!L, c~ch of 1n~ss rn/3 .t\t the middie 'points m/3 NI lh~ m.lddle points E.P. Q
I 5 a2 _ t,2 · 1h0 side~ pf, 1'1e triangle. :+i:f.' ' ! ·· ·

Sol, Let, M \:xi the m11ss of lh~ · ·'"'· P.!: of tile, ~ ABC about (??' and 0Y · . .. · ··
•. recUtngle ABCD · · or · · sides
. AB,.; 2a, AD= '2b and let '2c be the side o
of the squore PQRS cut out from it with
its centre nt, the centre of the rectangle
sue~ thl\l the mass ,of square
· ·,. o ·

2B I
·. ·
=·fM.
Gl ,~
· 2c
C ·
= P.l.'of rn~ms m/3 each"at E, P and Q about OX and OY
= !f· OE. o>f OQ': QQ.'+}' (.:_OP'). PP'
-= (1111'.1)1 PQ.(QQ '-PP')
= 0, i·
::, OP' =OQ'= ~ PQ
': Pl''·= Q(J ',
A= M.t. of the rcmnining portion 11bout · p •· '. TIIU~ l~e 1P:L pf the tri~ngle ·vanishes fbout BC and rcrpendlculnr In R<;
AB · ' . A 2a 8
m 0. fie~ci: SC is the principal •axis of the trinngle ABC a1. 0, ·
= M:l: of ihe rectangle about A~ - M.I. E:x:. 4aj Sf;ow that al rlie centre af a quadrunr of ail ellipse, th,• pri11tiplr
of the square about AB , . 111. //Sp
. : 1nne
, are enc
. 1me. d at• an a11g
. · Ic i1 tan .1 (4.-· -· -
ab- ) m .r 11e 1rns. .
2 2 v,t1s
.., (l, Mt/l + Mb 1)- (lJ (iM) c'1 + (!1 M) b2] = l6 M (5b - c ) ,. rt o 2 -b
2
.
similarly ·
1.
·· · · . . '

2 (Mmul TDC 92, 93(llP)) ·


B "'M.I. of itic reiµnining po!'tion about AD,. L M (Sa - (h
' • I 6 ' ' Sol. .Lei OAR be the quadrant o,f an ellipse
F = P.I. of th'e remaining portion ~bout AB and AD ' •.r2 · , v2
,;; (0 +Mab):. (O,+½ Mab)= Mab. i -+"-=1
a2 t,2
:, .IC the -P,rinclpnl .axis in. the plane of the rectangle· :at :0 maxe angles Le1 ox oi'. t,e an ~l~~entru-y\ati:rn ·ut the P(,T, ,I') or !he
e and ½n + A to the sides AB; ·t~en ·._ . Then A= t-fi, of 1hc quadrant abqut OX. ''( 'I . •
M4'(V-i, ...
tan29=---
.
2F '· Mab .
2 .2
6 ab
2
8 -A . lI"; M (Sa - 5b ) ,5 a •.:. b 2 =-·--· • f
= r l

. ;·=O •:n•O.
· 1·' . 2
· · P." dxify. '
·
a
!

f E:1:. 47, ABC Is a tr/angular ~rea and AD ·11 'perRendlcular to BC and


AE ls a median, 0 ls. ihe middle point o/ D'E, show that BC Is a 'principal . " ir'i(i
., QQ: 1.-1 c;i2 ~.. ;:-.')~- dx:.
3a· o. ·, : ,1•,,:

rm's 1;/ rhe triangle at 0. . _~., fltJi.. ,N1,cqs.,4 eda .. pb3 a~ I''Ct) r(t)
Q..__ __._ A. ·x
I _
·
- , ~ r(l
3a·'• I I1 f, I · 3a· _' .
3
,.

J, ·.:·:;t:::: •;' 'HJUH~!~: ::;:.:;;~~?;;~' ·,


, , I'. ·::.• '. • ·,";";',,",~'I,> ,J,,11,Uu0:'>;,1"f:'.,~;f1.",' ~•~•,' 1
.', ,, '

1
WlilfiUil:JftiW.:(JWUU&ijllili~afttmffUth;i$f:,;~;;~~~;.;(!~!~f:~~;{":~~fi}:l~;~t~t:~1H:j~,f.b';;l:; '': ,, : -~:; f;.; i<1:;1~!~}1't/•~~(1·•:ws~H~thif>hi•,n;c;: >:. 1
! )' :n·.:~1 ,.\ .,,, •r ,, •~ ..

r
. "'"·.--....•-•-,•---.-..u1~~ff41'l;";H~T:W"!l'tc&IJdWl>......to'•!...~;~~.;!.,',;..,:U~,:,J;,,,.:;,~j~~/!,, '· ,,J,•, ' , ;,, ,, .. ,.. , .'., ...; .~..... ;,;,,-...:';l •..:. .• ;.;.:,...,-:;~ ~;1:,;,1Jl,.,,k&,1.it11{1, ::!:11.:.:..U ll::i, •,; /, •i.;,, I!.! '..: ..!:i~,. H:..'.t.i.'Cti.ttl~~ :~, ..,,.,:.:.1,,..,.,/,., .,;.,.,'4,lt .. •1Y'N uH,~tUt)ft\ ••·•~•· • ,.•At\
'~
i
~
Ii

'
;j ·~a O,r1J<111d,:f 4 /ligid llody
tt/r,mmts w1d Pmd11c1:, oflile/'ilti
.
.1 .. ·.· . .
l
!;.
= 76 pri1i,i" =¼'M62. ·: M (11rn,~ of _ I?. nab,·
- 4 j 1 -\
;=:-: tan [
- 'l
.
,
8 lt/J I.I!\$
'l ,.
+ (er .. Sb·) 1.~n 2 qi
]

,• ' Ii" M.I. oi' th~ guudrnnt 'nbnut .0 Y , i • (.~w - /r)


{b/41J\o1(a: ... :r:} \ . /) E~. SO, Slu/1\' 1'1<11 "' mi ~.,1r~111ir;,· rU' rhf bowidiu,q dia111Q.t~r af (1
=
'.
f of,.
.1
11

= ,c; o
pr d.ul,1· = p -
a
f ·,?·'1(~• - .r) d.dI
,J
1,t •.,

·
·'l ,
1'
· ',-(•111i,dnn1/r,·,. /am/1111 ,Ji~ P!'iJltipal o.l'is 111r1/.:N t'III wt,~le ➔ tnn· 1 {8,f.',,rc) 10
=.!.Mei'.· . , . . I • • .
,. l . . ' . .
s ,; u sin 0) . ·1he r/iWIW/~I'. . ' . ·
P.1. u(\l\e quadrani nbou1 OX and O Y
f == ! ·: Sol. Lei 1hc nxis of .~ .nntl 1• be 4aken .alon~ 1h~ dinn1e1er OA :rnd
"
tb/<1)v(i. ,.l1 . ... . ~ ,\fob f. . p·crpcndkulnr 10 OA u1 0 i'.o thi: p~lne of th~ lnmina.
= ; f
.r=O pO ·
pxyd,,d,\'.:: !1 p
0
X (a 1 -·:. 2) d.,· = -
21i
. '.:·, 8quu1ion o_r the semi•dn:ulnr ,
: Ul\lJA~ is r:: 2a COS 0. (
If 111¢ ri-rincipal axes are inclined at :i.n :inglc a to OX a1i'd OY. then
:.et· pr69or be the mm or on
2F .
1an:l.0::::--= 4ab. ., . ,•, 8=l·nn ;.I 1' -•---.
4 ab ) · · demenl~ry arcn at P, . I
· 2
_8-,/\ ··n(a -bi) 1' <l'-b'
l' .
:, A= M.1:·"or the laminn obo'ut OX • 11'
Ix, 49, Find rhe pri11c:ipal axes of w1 el/ip.'ic: arecl at ,any poi111 of its : Jllll ·1l<100,8 ...,., '
/;01111di11g arc, . ·· ·-= .. , (r sin 6)·. p,dGdr
, •, · 0 • 0 , PO.·. ,
!iol. Let P(a cos$, b sin¢) ~-e a ' ' :

;ot,
point 011 the nrc an ·elliptic ;ircaor ::: l (2a) 4 p J··,i:,'"sin 3 8 cos◄ 8d0
bounded by the ellipse ' ! ·.' 0
' "! '"'!,
; 4r(~)r(t)
1 4 0 ICOS0M A
£.+t'.:.=I. := 4pa 2r(4) = ilrrpa
a2 b'
Consider PX' and PY' a~es
:
y
~ !
yl·, · la= M.I. of the lnminn about OY
pnrnllcl lo ihe axes oi the ellipse. I
;,. f f
iv.l ;?o C'OS 8,.,
(r co·s 0)·"} prd0dr = l (2a)"') p
'f ,:os b 0//8
,vl
Then . 0 0 ... ' 0
A= M.!. oi the. elliptic area abput ~
'
4 n:i n1I ) s 4
l, '

4pa · 2r(4) =
PX' 8 rcpa •
= l Ml)+ M (PM)' .,rnd f= P.!. of the lamina about OX nnd Ol'.

=M(lb 2 +ii2sin'$).
. ' . f0 f0
= Ill.!·. :!i, ~os e 8) ., ( r sin
\I' cos . e·) , prd·adr
8 = M.I. of the elliptic area ~bout PY'
= { Ma
. .
2 + M (PN') 2 = M (l a2 + a2 c,$ 2 9;.

~nd F=P.l. of the clli'ptic area ab-fat PX'.an.d PY'


=~' p (2u) 4 J0,;,2os:I 8 si~ 9//0 = lp
,\
4
11 ,

.-. Ir ,he prin'~-ip~I .axis mrrke nn nnglc 0' to OX. n1 0 ,hen


= 0 + M.PM.PN =Mob cos¢ sin¢. ...
.', lf the principal axes at f> make ~n angle 0 wfth OX and O Y then ton2El'= =_£
! -A 31i
tnn 20 = _1f_,,, 2M ab)c~s ~sin¢"
B -A M <¼ a1 + a' cos 2 ,j))- M 1; 2 + b' sin'<!)) i {f i .'. 0'=f ton· 1( f; I
E~. 51. S/i11w 1J(a1 l11e pri11dp~/ (l.tcs auhc 11oric of ,1 /ic;/f,/oop ofth,•

0 - l 1 •.1[ · 8 al; rnn i' · · -j l i. ,1eiwii.\1a,1e r1 = a 2tos 2€1 al'e. 1'11cli1i'~<110 1/u.' i11i1i11/' J/,,,- a1·0"11h,.r
.,. : an · (a 2 - I;') scc 2 <j> + 4,,l ;_ <lli tor.2 $ I I I -1 I .d Ii ' I • 1 :1
2 tnn 2 nn 2 + 2 Inn 2 '· [l'vlec1·ut. TDC 93, !IJ€P)J --..
S,11 .. The .equ~1ion ol' the lcm11i$Coto is : \\~.
. . ....

:;:;::f::~=
:·:::::-·~:·:•·
·,-:-:;.:,,
,::_·::; ·;;;:,,,.,
::·,·-'.•'''', "!:'.·,,:\'.
,, ,. •,1'( ,.1-,, f, ;i/.l,tti':ill1:1',9:!,t1;,• ;,:.11!,-~,!~~:.':~-:1!-1(1:1~:t:~~
,.',1,•,,,,,,;,.. •: .. •·;; .. :·

ou,_ • '""•-U·•• -~ ...... , .....


20, • . . . . . . . . . 4 .... . 1,~·
;,;_ ··a~ Consider on ,,v4 l
•' ,-,~ lcmeht 01 arcn
. oeor nt p (r,
[ - i cos 3
2e 1 = ~8 pa 4

~l ,. Mns& nr the YI
t :, If 1he principal axis at O mah an angle i)i to OX then' ·
~
':' pr 60
area ' ; ·i
· l ian- 1 -
qi=
l 8- ~ .l
8
2F- "'! tan-· 1 { ---.;..___
(37t+ 8) - (31t - 8)
} ;,d
· tan· 1 !
l I
.r

:. A= . TI1: oJhcr principal 3tis beinJ at r~g~ angle's 10 .this principal axis will
loop of the be 1ncline.d lo .OX at angle rt/2 +¾tan l . . .. . . · . . .
about OX ... 1
• 11,/4 , 11/4 ·
fo~o J,.o
a-/(eoi 28PM2' pr<i8dr., f I IT'i(eos 20µ sin'0; prd'edr
·o~o ,.o
l!:,x, 52. A,wire Is {11 1he form of-a .reml,clrck of radius a,
at an end' of (t.r 11arne.ier rhe prlnc/pa,l a.xeJ in lt.r p/arie are to
J1114 the diameter at ang(es · I
= p J0 l4 ' \
l °}1-.l(cos 28) 2 ·
s\n 0d0 = ~pa
4
0
cos 2 19.sin 2 9dfl · 1 J. tan- 1i
2 n
and 1! + l tnn·d. •
2 2 it
'iv4
= ~S pa f0 cos Sol, Let C be the c~ntre ~n~ QA the dinmeter of a seml-circi1lar wire of
4 2
29 (I - cos 29) dO
radius a, Let the axis OX and
;, ..L
!.6
pa 4
fO 4cos 2 r -.
cos 3 1) dt
29., I:.
OY be taken along and per•
pendicular to the diameter OA.
- ..L
- ·16 pa-
4 rl _re}). 2re2>
f(f) - r(2) ref) 1
2 nt) j
•,
Putti~·g
"'· .
Consider an e'lcm.ento.ry arc
ao9 at P, then Its mass,
e
om:::: pa69.
. M
. =~ pa
16 ·
4
l--,2'. ) . . =-
1
·
rt
4 3
4
na (31t _ 8)
[92
.,i where p = ---
(n:£1)
a
I.
:. A= M.I. of the wire about
r'l1f • M<m-v. ,,;i,;

. 8 = M.l, of half loop of !.he lcmniscate about OY ox . 0' . 8 rf ,- UA X

= J0 lt/4Jrr-/(cos2Q) 2
0
PN , prd9dr,.,

JIV4Ja-./(col28)
0 0 ,
' .
r1 cos 2,9, p,,:d9dr
, · :: f0 . pad9"' J na 2 sin 2 9 . pad9 = f paj f0· - cos 29} d9
·4. ( 1114
= 1Pa ) 0 cos• 29.
' •
0d0 "'ipa
f 4
-~
r:14 ,.
•cos 2 29 (l + cos 29) d9
.
0
9 - l sin 29 1n m Lpn:a3,. l
,

I 'l _b l .. l
4 •
'! B=
.~ ~ (31t + 8), (As above)
of the wire about OY ·
' '

'. =I"0 , pade = _f n(a+ a.cos 9) 2 '. pad9 '


· '=nodf n/4F,.Jo-./(eos
P.I, of hnH loop or the lemnlscnte abou( OX, QY
28), ,
O

0
,.d . · PM. PN, prd9dr . • f0.
= pa 3 r.(l + 2 cos 9 + cos 2 .0) d9
'
f.
!V4 JcrJ(co1 20) ·
., f · . r sin 0, rcos 0. p'rd6dr (I + cos 29) J d9 l'
0 .
4 n
"'~pa fO111ios 2 20, co~ 9 sin 9d9 .. ;Pa f~ 111~os 2 29·, sin 29 d0 4 ,d
l (3 ... 4cos 9 + cos 29) d9 ·
) .

f
''

1
,.. :.:;1,::,:,•:: .... ,, '.';'.•ii:--:·~:'"-~';'•l'."·''' \i\~f!:~:'\~!:'':'•,:•;:,::n.:n:r,-;:;;:,1~;::m:r:·:r.1:'1'.:!:',~f,i•,1,,,.1;i~!'"i''J';'~••i·!l,•~:•1i'.~l•::•,,·:·'(",'''"''•~•'.;~'"~\11g1r,m.,,11<i~"'-'il<~-,..",11-,:,,.,...,..,,,l!llll_,,...,..,,,,i

~g~~C,-"•M(,!M!~i!~~?~~~t!ffllft.U.r!'.1N!l:~:;"f"t'.9:;":t"'f~~-!fi:·~:.:·?~~rn;Jf~~S!t~f~}!ff•ff:t~.i:-~:=•:-±{?~.~ \Hi~S~rl!l!~.f~~t:M:i~P.am•tH \l t <',?'~f,.1~ ~ ~ 1r~r:r-: :~ ! l: '. •;


·· ..·.,_;_,:;

VJ, 'l.!~.'-1,!~ ,.:.,


/' •'!Vlll\,>l\l;J \,<111\,f j IVUH\.111'

{ pi [ 39 + 4 sin 0 + ! si~ 29 D = P.I. rihou1 0 Y, OZ= 0,


E =. P.l.' about OZ, OX= Mah, and r I
"lnp,i-1
l . . . . 11 F ... P.I. aQoUt OX, 0 Y = O.
;
❖• I
nnd
=J 0
f = PJ. of the wi_re about OX ond OY ·
" NI , PN, paae: = fO"l'i sin. 9 , (a+ d co,s 9) , pade
Here D~O~nd F=O,
0, Clthcr two
axes is
be ihc prin~lp,il nxis r,:
u;,r,i5
axes w111 1:-c in· the .n rlanc. Ir one or thc~e prin'c'ir:,!
ai an angle 8 to OX in xt plane, \hcri ' ·
OY will
I
'
i Il
= f0 "cm 8 + i sln 2El) d9 = pa,.I,3 [ - cos e -f cos 28 ]n0 2E . . ½Mah
1an 29 = - - = - - - - - ' - - - - - - =
. !Oat1'
.- ... ( I l
f
f'
I
I . . C - A ll·Ma2 - M (Jul+ 2h') 23tiZ.: 2h i, ~
II
. 2
= 2p11 3= -·I'( i'vla 2,
I
.'
'
· ,, · IO ,20 ( . · · · · •
~
I
'' I
Thd other principal axis will
be perpcnrli~u\a: to this prlncipol)xis
. :. ff 1hr. pr;ncipal _ij'xis ot· 0 mak~ a'n
2F
·

lan• 1 - - ·= l-ian~I
, . (
. :,
:;ltmGI
2 V
910 OX, then 2nJ Part, If one of 1he principal axis pass through the C.G,
con:e, then · .,. !;
I
I
8 = J. ----- I
1an:e =
h
=-'
/ {
.. : [J - A ,.
1({- ~) Ma2 1 ' 4a t
111c 01h~r principnl his· being 01 right angli.s to.th.ls principal
.· d' · ox· Ul _ong 1e n +· 1 tan ~1 it·
4
~c I ... lan 28 = l tan 2e
I - tan 0 l6a 2 • oi 2
"'~ , .. ,(1) t
111c
Iinc 10
2 2 . .;, , :, From (1) and (2), we have· .
h
. Eli. S3. find lhe f!rincipa/ a:w of a rig hr' cir{Ular cone _al ~ 'point on
1/:c circiw1ferP.11ce of the·base, and show that one o/1hem will pass 1hro1igh
IOah - -.......- ~
23a2 - 2h 2 - l •~~
e
1/.1· C, 0. if //;e vertical ·angle ()/ the cone is 2Jan- 1 !, ·~· ·
0 " l - h1) = 4(23a2 - t
Sol. Let O be a poi nt on er h = 2a. )!
the 1.ircumeference· of the base !I
OC ·a 1~-
of n right circular cone of mass· tan o:=-=;:::.i,
· AC n l
M, height Ii and semi•Yertical )-

angle a. Toke the oxi5 O.Y. along i.e. venic.le angle of the cone =:lo;= Z 1an- 1 !'
1hc i.linmu1er 08 of tile base, zl Ex. 54, ff 1he verlica/' angl, of _the cone .is 90' the point Ill which ~
t-;:
a~is · 9 )' fl~rpendicui111· to, 08 generator is a prin_cipal axis d,\,/des the -generator in the.· ratio• 3 ::7. ~
:,,.
ft
nnd in the pl~ne of the base ond So!, Let h be tl)e height of a cone b
il.~1$ OZ pcrpcndicul11r lo the of ~enieal angle 90'. !'
bn.~c cf the cone. , Let the generator · AB· be the Y d
(r
I !
Then fro,n Ex, J6 on pngc 5 I, principol axis of !he co.ne al the poiM I r..
we h~v~ · 0. ,Consider the ~9clion or the cone .

F
A= l\l.l. 01 ll)e

~~ ()al+ 2h~)
cone .ibout ox y
through the gene,fator 'AB. and the· axis
AD. Take OX and OY, the axis of x and
axis of y, perpendicular to AD an·d
i1·
~
· B = M.l. of the cone 11bout to AD respectively in.this iection ~
~
1)

OY= ~(~}a'+ 2//1) anrJ OZ the ~-a.xis perpendicular((') this t::::::::..· .1a-=:.:,; :}
sectivn of the cone. ' B ~•c ii
1:
C"' M.1. ol' the cone nbou1 O? = :6 Since the cone is symrne1rical abflul OZ.
,', D = 0 := f.. I'
:'..
·,.
1:
::
::~ . ,,
ij

:f ' ~-..
i
i,
$:
,;;:;:::_., :\·-.;::'ri7.,11l}i?:~!.'1.~J:~'1~AH.~'tt;i\H~~~;_fl,l~1r~~1~~••Nff~:... ,,\\,1,,1rJ1~1mt!'l'i-u~."""".,...,..,P\,~Cf'MN1ti-,1n•u/<!.
' l I •

,:,::;>:: ·, .. :;·:::::·::
: >.:·,•~· ':; :r •:~~'•'.. ,

·•----··-~·~~• Ct >th;t:,,Clt••N;?,'r."':'/'t:r~,~,~Rff,~¾•')7:~

' '
J'
, ,. ',:,:,11?::;:,.):·: ·•1•,
'
'.: '•;: i-•;• ·:~:f•, :··
,, l
r_.1 •
1
~;j
!l~:1:~
f) , ................ , .. .._...,.,,.,,;,;•. ., ~•.-,•~•,:r,nl~H,•l".''.'1 ,, ,

~l ..-·· :;
14
of Rigid Body
f"·

~~,_:::
'·, ~
;~ i
Prod:1t1s of Inertia
75
·I
:it
,',·OZ is .a principal aids Bl 0. The other l wo· princ,ip:d al\es at O a\e tl\e
~J., :1~11orn10, .A!J nnd the line through O and pen,endiculAr ro ,genera1or AB in.
.I. ·
.. 3 7
10. AR= io AB.
lhc nbove scc1ion of the cone, ·1 I
AO= AB nnd OB:: A6'.-AO.= AB-
Con~idcr iln e.!cmon1n.ry circula~ disc or width 6~· ·a1 a distance· .x fro:n
the veitex· A and pcrpend'icular\o !,he Mis AD, i.e. 'AN"' .r. .
Ao.~i. ..
,'. Rndlui of !he disc "PN., x tan 45',. .r. oq:
7 . .
lvfo~; ~r the cicmentary disc, -6111 "'p~ 28x,.
0

· Ex,: 55. The lrng'/h, of the 'a.>:/s of. ci sci/id parcibvla of revolt1ticm is
·equa(101'1~ lai11s-recr({m ojrhe·ge11era,1ing parabola. Pmv1: that 011e
M.I .. this ·disc about OX"' f 2 .Bi,! + MNJ.
or PN cm axis di a point In the drcular rim meeu Iha axis of revo/111/011 Ill 011
,=' (1x2 ~-(IIM - ,ii I PIU' 2ox. ! tan·1 1
4 l'
·
• ,·. A., M.I. of th~ cone' obou1 OX Sol. Let the length of L.R •
h . .
of th~ parabola be 4a. .
,. I0 {l'x2
' '
+ (AM - x) 2.) Pttx 2 dx · ,•,( L~ngtH of the axis
Ab =i 4;' $nd equation o( 1hc y
= np f0h(ii~ 2AM . 3 + AM 2 • x2) dx .
.t paraboia is
'.
y2 = 4qx. ~ . ,.,(I)'
"' np (f . fh 5 - 2AM . lh 4 + AM2 • , Let O be a point in the
,,.· J_ 1tph 3 (3h 2 - 6h . AM+ 4AM2).
r· circ~lar rim and ox:,tbY'
X
l2 . . · · . ar. s/p:l!r~Hel 10 AXandAY.
Also· M.1, of. the elementary disd ·about OY 0 ihe pdncipal axis at 0
. tf'
2
'"ifN ~11i~01f2am_;({,i,l+AM2)n:px26x,
1 ·,• OM =AM 0 is i ,inclihed a1 . an angle
;o~
e io '! (i.e. to the axis of
,',fl= J'j.I. q( .l,~!1 -~~p_e ~~O~I pr_ . revo;1u;1io,n AX), then B

1
=( <-!l.;. A,lff_) ,n~.:x2,t0' = :.:(~,<1.r4 +·A¥i x,~) dx · e;= 2/ tanJ1i .-:a:-;
2F
. ' '",
. {I)
'' ' '
J' ·4 ·'2 I i, •.l 3 . · ~
'3)
nr:r ~
="•,.' (
,> ', .,_,_,,. ',,,
. 2'
I/ -; .h. = ;.,., .rr:p,11 . .(3/i -r lo AM,,.
)-.0,:Ii, "+'·/f.M,. . , • ,
:consider an elementary strip PQ of width 6x at a distance .r from i\
and .i>e,rP,endicular to AX, then .its mass . . .·
Slncl · r,k? p_r/rF'iPal .j~i~.' ~l ~a_ke an nngl~ AOX =-~s•. !o, OX, 8m _; 'p11P!4Zox == prri ox,. ··
· .' · .2F ·
,•. From ta~ ~a· .. q:-;;r- ~;e have · . where (};'y) are co'or.pin'ates of the point P.
M.t bf this• elementary .disc about OX'
. 2F' ) if' . ,. !l , Ph-iom -t- ODll\m
r.an 90' = _.._ orr·.., = e· . A or B - A = 0 or A = 8,, . ,
B-A · - •
= +'0D 2) p1ty25x
,', TI"I (3h2'., ~h.trtj + . I
~o ~P 111 2
Ph = +(4a:,2) pre/ox I•
t (3h 2 -·6MM')
or ...,_ ! · 3/i 2
• · =..... l
l~ ,• .. ·· ~,9 . (·: a10,A'=AD,.4a,y=OD 0D 2 = 4a. 411 or OD= 4ai
or
2
5(~h - 6Mt.f),= ~Ii~ o,r '911 2 =101.iAM or f.M = / h, · /7 (~ch +I 6a2) ~ppax&.r y' = 4m:,
From similar 1r)nngles and !4,pO,
0 ,4aM.!. of the
A',+,. solid about OX'

d,Q. ti&.. 6B.


j' = J ;'(2a.r +
·O '
4pn. <J.wi;•
Ali "' itD "' h· "'fO'

l 1·

I
ili~~rA:!~~~~Q·;~m,rrm·\'lt~;1r~n~~1rmrm1::1n1t;1~}·!r~;1~J;:J1-.~if.tf•!t1·f~.!1~m;1fi:fml:'$~i~~i t~-~•, iHffiEX-~n1un:munw·mummi•H,TP.~}-:, 1 ' :t•·;f !i 1:- ~,
)f\~~~t;-; ~Ji·,,.~ .. ,',;,~.::\''' '" 1;:!;;.'.!~;,;, . ; -~ ~J. ..~ ~' .~£,; ',

:: ; ;: ;,.; ~; ;) :( {:~i~;HmlUii.H;r~ I'..(/ ·!~Wei-: rH?H:i.fl:;",.;,;i .i~m: :, i•,:. ·: ,'~··'

'
f
u.111u1111cs oJ, Higtrl Body
~,,
Mome111s ana l'roaiIcIs. oJ I11ar1w
.. ,....... 77 ~
r. i..1 ' I
"Jn()11
.
= .~~..~,;:.~
lI !11 '-·J ..... 11\a• .
pn,/.i..
and the,cquµtion of the norninl
/,Nat l is
.r-2a=-f(x-a) · .,,(3)
· · !
.' ,1
Al,v M.I. of' the' dcmc111.ir)· uist nhnul 01"
or \'+A' - 3d = O.
· Consider an claincnt fa'llr , i
tI
"r /J.tl~u/1/ + MD 201:1 + [fy2 ... (4c, -.,:) 2! pny"o:r at the point P(.r: j·} of lhe
lamina, then

)
,,
2
= [tl,l' ,.. ( 4". -
.~) I 4 llp11,r~.,· = ( I 6n 2 - ,,,,. + .r2) 4 npa.ro.r
PM= leng(h of per-pcndicular
.·, •fl"' .1'·U. or ,he ,oliu ubou1 0 Y' 1
from Pon tani:ent I.T given by
:::
6a: - ?ti),'.,. .r 2) (2) ~ f
.i
I
~(/+-!~ ,.Y-fi-~ 1'I
f .r I. = 1x 64 x g;i:pn
·. 5..· , I
. , .. : 1,, " . , i I
"'..;;rp,,: Kw.1- iJ ,1" .i. ·1
and PK ,I, length ,of I
!'
:. ' . .!() '
perpendicular from P on the .
fl.I. nf ihe blcmcntnry ()isc obou1 OX', PY'
nonnal lN given by (3)
= 0 + OV. ,l•ID: 6111 =~a, (411 -x) pnyf~.r I•
::: 4a. (<lu ·- x) pn. 4axox, _ )' + x- 3a, t
- ✓2 '
. = f. .-... F::: .P.J. of the solid abl)UI OX', O'I''
P.I. of the element ~bout LT and lN ,
i
~(i ,
p ll ,I 6a· (4a •· x) u/x
. '
1 !
0 •
PM, pg .. Sm= (l:2..=..!!.)(y+x-3a
~2 ., . ✓2 . )'pli:6~ ·
i
. . 4a . . , ' . • ·.
::. IC>()nn~r i.a.,
. •
2
f ~. a =f
l]· X l6X32p1t<1 5, If the tangent and normal at !, are the pnnc1P,al nxcs, then the P,I. ol
the !amin-a al:,out ·these ·will bo zero. .
,, ·Frn111 · (Ii, we h~ve i.e. P.!. or the. lamina about l,T and' lN
b, flv(ax) ()·-✓ -a1(~x-3aJ · pdxdJ= 0 ~~
I
e,.,, i 11in1 _ . . _1>< loXJ2pM5
1 I -I l . I 0 y=-2.J{ux) 2 ) 2 • · ,. f
' '*
- - - - - = - t a n (-) numcric~lly.
x 64 x 8 - f x 64xJ,2) P7ta 5 1 ; I /
or. 2 .
2fbf2~aj
, (/-4oy+(Ja~+2ax-x 2)) dxd)'=O
• .
!i I
Ex, Sn, A uniform lamina is bounded bj• a para/Jolie arc, cf fatii;s 'rcc//1,n 0 - 2~(/l.\) . ' I
4t1, a11d a double ordinate. 01 a distance b." from the. vortex, If
b."' 3a(7 + 4'/7), show lhC/1 twd .of the pri11c)pal cv:~.r c,1 the end of, q la111s or J i .\i- 2ay2
0 \ l
+ (J.a 2 + 2ax
.
-:-x 2) y} •2~(a.t)
~v(u.x) dx = O l
i
'
re,:111111 11re !he 1a11gen1 and normd/ 1/1ae,
2 Sol. Let th~
1• "' 411x
equation of
· '

.
lhc 1
1nrnbof:i b~ ·
. I
1
or 2J0 1f a.d(ru:)+2(3a +2ax..:x2) ✓(ax) rd.x=O:2
i

·.. , Coori.Jin:,1~~ ol' thc•tnd I. o( L,R, I.I.' nri: (ti, 211),


D·1·r· · · r· 1l Wt', gel _,.;:; d,, · -2a •
! ... (I) or f 0b(~a\12 .r:vi; 6aSl2 )'2 + 4a312 x312 - 2a 112 xsa) dx = ~
• I ~l'~ll11,u111g
d.r )' 16 ala bsa + 4asa
or [ TI . b~a + ja~a bsa ·-~ ,a bm.] = 0
.·. At L(a, :·
!6:. "'_2E, = I, ;:
tb: 2a . 5
Equ,11/011 of thll. iu111Jct11 /.,(at l i~ or 11 ~b + 4a 2 + ab ,. b2 = 0 , 1 .
3 f .
';
;

)' ... 2.u " I • (.r •· a) or )' •· x - n,,; 0 1


or b - l!ab - 7a 2 = O
\. . , l
,... (2) !:

I
T I'
. ~

•. ..
1 ;
:
';

I f
i
f
,

·,· ·· ·•:1r;! !t:•~-:.:,•,••1:-·•·,••,:••'"';•;,:·


~,:,'.~•1/:,,,,•.~'-.:_;..,.. , :"''<'•'• ,, '\•1mr,ra-t;1,1""'J.:"'nill~;,:.~-o.+,Y~'"l.~~'~l~n~t~~••·•~~1~-,liN'~'"'·:'4~i'.J;.'£1•,u;,;:,:
•,•.,,·,,•ltt,.titJ1•,'••,;••,,,•1,,,,,,.•1\•,;•)uf,•,'•'l,,,,f~',,,,.,l_,_f,,•,,i/1,•',•r•;,;'"t,,_, 't
...,:-.,,•:,,•i,
·,'
-"•:••~:,1•
- '
··•: ... ,,:,~
'
.• '"':.,,:1,,,•,•,.,,,,,.'\
•l"ff,;'l1'f'~':l·~•~•·~.,r,,wr,.:sm,i'l;'+~UiJ'tii~lt{i'!J~i~U'lf-l~l"i
,,, ,, ' • • • • ,, ,, • ,,,.,.,,, .. ~
..K~,.X~"'':O.·Wn'>c.11v1011~11'1,M~C\.~~i.,·'

.,,;,.:•.
•is•i; r, \ {~ ',>;dr,,•
. .: :~": f ;; :~ ''.' : : '"'.. ~' \ I '~ ~; ,'.; :·:,::. ' ·;:::;::~;\'· :, ·: ,:,'.•' :;:;;::,,:

··----~--···--··~,..•~--,, ,1 s,twa>';l'~~r;-.~-ry;~r~r.1r;r"J;"1"

:-~:~;I~:_::-.;:.: :- ~ . ----~·~:v~:'.•,,
.._ 20~.•'.:·:'r..;~;;i.:•:~;,·: ... ' -~·; ~.';.. ·:. - .. ·....

· ' : • 1." -~ , '.•; I

I
tli/
'l;

r:
Id
:>! ;·,~:i~i.•1\\ ,,'\,•j,£•~~'.l..!:•
78 I'
,'(11",~;i>.i•

i~ ..:~~:.-.. ::·
I
· D)·11a111lcs of Ri,~id Body . '•Prodt1c1s
:
~~
Mome111s
:I: ✓
•:
196 ' of lner1ia
,,~:'1' .!.1 - (/2 + 28(/2} .79
.3 a
()l'b=---------=- 9 , I ( -±--'17
14 8 ) u =P.!. of the squnre OABC nbou1 ex', GY'
i/
i~ :)
·)

'.~i:
2
or b --~ (7 + 4Y1), Lcuvin~ - ve •
2 J 3

=0-f
& · · :i / ::.(P. L of three pnrticlcs each of m:m m/.l n( P, Q and if:)

;;',.~: i H01\co I/' r,·+J


(7 + 4✓7),
.us
not be n9g~tive.
Ill
((ci-a) c +c (c-b) +(c-u) (c.-b)I.
"•)
"'':' 3 :.. ~ (a +.b) ,. + k 2];
f 1hcn 11\c princlr,ol o>;e~ DI 'L ore lhe inn.gent~ Md r1orinol !here.
1
Ex, 57, .A w1f/or111 .rqucire lo111/11u· i.r· bG>1rndea\ b)• tlla. a.xF of.!' and)'
.·. Ir !h~· principal a:,ris nt the. centre
axis of x; t,hqn .
O .i.s inclined a1 nn angle 9 to tho
·::.
Ufld the lines · x"' 2c, ;i = 2c, a11_d a· ,•omer i.r cut off by the line .
1
· 2Fi ' - (m/3) [ab - 2r. (11 + b) + 3c2l
x,11 + y/b = 2. Slu.r,v th(lt tlic prini:t'pal a.reJ·. M· the ce111re. of the square arc · tan 20 = -::---r- = ''""'-'--'--:!'--'-'""--..... -
/11cU11ed to the axis· of X 'at a11g/e.s··gi've11 by: ' B -A ; (in/3) (2 (c - b)2 - 2(:c - (/)2)'
. ' 2
Ian .28 ,. ab,. 2(a + b) c + 3<::_
_ at, - 26 (~ ~i b) 3c 2 , . + ,
- (a-b)' (a!+b-2c)
(C-b)(a+b-2c)
Sol. Let 0/\BC, be the squnrc
lcmlno or m~ss M bounded by lire y
Ex, ;S~. f_h~w
rhw_ one of tha principal oxes a point on ihe circular, 11
r/111 of 1/ie:s~/iii hemisphere, is inclined a1 an a,igle· tan· 1 •10 the rc,dius
axe_s und 1he lines x !. 2c, )' = 2c, Y'
the point. , .. , 1
line .:!'.+l'.=2 i.e• .i.+L=
a .b · 2a 2b er . I ~ L~1 'C be th~ centre an'if
OA 1he dhirriet~r of the circular
cur off in1ercept~ Ob= 2a and or radius
0£ =.2b on the oxes. Lei m be the.
rirn of a he(ni~phere zl
a and nrnss; M,.
Take OX and
· mass oPthe triangular lomlna ODE

.W
X' Or' 11ic·~~is of;; and y ~long
cut ·orr from the squnre. The 1rlangle and perpendicular to 0,:'I in 1.he
ODE CM be replocod by three;
r>~rilcles dni:h of mass nv.) n1 th~,
.middle polntq P, Q: R of its sides ..
Co11slder lhe llnes GX', Gr" r.hrough G and paral!e! the sides of lhe
-~ plane of 1he, circular rim of tfio ..
hemisphere ,and OZ the z.:1xi~
µerpendicul~r to this plane,
As· in Ex; 35 on page 49 we
'y'/ C -~A X

squnrc as lhe n·ew axes cif. reference. Wi!h reference 10 these new axes the have
.coort!inotes· or ·p it're (-(c-:a),-cJ,Q.are(-c,-(c-b)J, R are
[-(c-a),-(c-b)], , · Ii= lvl.i. of; th~ hemisphere ahout ox= f Ma 2, 8 = ~ Ma 2, C:: j Ma 2
,'. A = M.I. or the rem'aining area about OX' D = P,!1 l\b~~t;OY, OZ= 0, E"': Ma 2 and F= 0, ,
=M.I; or square OABC about OX' - M.I. of a.ODE about .GK' Since D = b:d, F. .', )'•axis OY•i.1 the· principal axis.n1 the point 0 and the
= M.I, of square OAfJC about G~ '-(M.l. or three parlkles each or mass other two P,rin'clpal axes a1 P lie in ;;z plane, If one of these principal nxcs
nJ.3 at P, Q and R) • 1 · make an :an'gl~ 9 to OX, ihen .

=} Mc -f [c + (c--b)2+ (c - b) 2 ] , : ~£
j•
2 2 ¾Ma2 3
I. ,' tan 20 =.:....,... =- - - - = -
Sirnilnr!y . · C:- A (1 - l) Ma2 4
, l S

B = M.f..of the remnining area about GY. 1 2 tan 0 · • 3


or---.;:.:·-
I - 1an2. 0 4
= J.3 Mc2 - !!![
·3 (c - a)2 + i:2)2 + (c - a)2 J, 2
or 3 ta'n 9\ 8 t:i.n 0 - 3 ::·O or (3 inn 0 - l) (trin e + 3)= O
ond F=- P.1. of the remaining oren ~bout OX', OY'
,·, Ian d =lore= tan- 1 (l)) ·' • tan '8 ,,; _ 3 = 8 > TT/2
' ,, ' .
·-··-•«,.,

:~.::,. .

J~ff'I~!fQRW~mr:nr1im::m~~~~mim:mmuft;P:t1m~tr0ur1t~~::1:1~:e;,~nmsmmtmH·ti~-nn,i~tt-:imt::~{-<>f~t1tm1~~ma:HtH?iUSJm?J.Q!1r5,1t·1'. q:~,a::}~J'f1v:,n~1~: ~"; ,, ,.. ., •. ·


n--, ,. ~t-, .- , , ·o-~~~t1;,,:,,,,.,' , ,,·.•.,·., .. , • 11,:..:.,-.-,,,,. ·.,:£, .". ·., :.

l.J)IJU,'lf/~,'.) VJ l\l~!U Lil.JU)


and Products ,of,fn,cr,tla SI

"rhi,,1·. i,. ir,td,·nl:;i,iblc. Toking the principal axes as the coordinate ax~s. equation (!) reduce to
r;,, SY. Slio1,; 1l1a1 011c of !he l"'i11ci11al (1.,·cs ar an:: the fcnn
of 1/ie circ11/ar h,m· of'ti rhi11 hci1dspl1crical shell is
rtil! w the mdi'11.1 tliroul!h 1/i~ poi111. ., ,.
A tx~ + 8 'yl = Constant. •..(2)
Equaling the invariants* .of (I) (2) we have
Sol. Let OA ht ·1he dimnctcr ol' the circulor bisc of' a thin A, +8 1 =A+B '' ... (;i).
shell or rndill., a and mnss M. Tnke OX, OY, OZ (he ll)(es ' •••.(4)
arid A 'Fi' =l,B-Pi . ·.,
in 11\~ la:,1 t:.x, 58. I 2
:. A ' - B ',I ✓ ((A'+ 8 ')2 - 4A 'B q = ✓ {(A+ 8) 2 - 4(AB-' F ))
As in Ex . . Ja on poge a8. we h~ve
or'A'-B.';;•✓ ((A-B) 2 +4f2) · · '; · ... (5)
A = ;'Mac. 8=Z Ma 2 ,'c; =,,\1•1. Ma 2. and F =. O.
li'Yf,1", 'D = 0, .£ ,,::dl··
J , .\ Ad,jiag and subtracting (3) and (5) wi:. have
Since {) :::.,0 = F, :. 0 Y is the principal axis al O and th~ other
1 ' = { fA + B +·✓ ((A - aj2 + 4172Jl ,•
1

:ixc, ni O will lie in l~ · plnnc. !f one· of the,se principal axes make . . . . I


,in nn,;k e10 OX, then
2£- = l..tan ~ 1____· Mu;;_,_2
and 8'
1i (A+ 8- ✓ ((A-B)2+ ~F2))]
i.e: (he principal momMts at O are equal to·
Ii
,
e= t t;-·
•= !
! .;!!. '
C-A i (1-l)Ma' i
. J
8 {[A+B± ✓ /A.-B/+4F2 )l. . .
§ 1.26. P,rlnclpnl Moments 1
l
.
:u E1', 61, Show 1h01 for a thin hemisphe~/cal so/Id of,radlus a and niass
· Mo111c111s "f inar/a of o body about /rs principal axes at;a11~• M, doe 'principal momen(,s of ir.utia the centre of gravil)I are at
CQ/lcd i1s prl11cipal 111omc111s 01 1/1111 poi111. ... ! l 11.,·._g_ i l ' 2 ·
' TI1c· equ~tion of 1he cllipsold at an:,,' po.int Is given by ·: · i 320 Ma • 320 Ma • 3 Ma '
Ax 2 + By2 + Cz 2 - 2Dyz - 2Eu - 2FX)• =Mk('" , . Sol. Let G.llc the centre of. ·c
.TnkinG the pri~cipal nxes .. s the coordina,e a)(es·cquation (I) gravh:- of a hemispherical solio ,
{he form ' of raiill.f a and ,mass. f.1: lf C z
A'x2 ❖ 8'y2+C'i'=·Mk' . : , ; is, the cditrc and CD the central
Where A ', 8 ', C' !re the principal momems and are. the values of kin· radius of the hemisphere. then • i
C0=3al8. · . i
A-A.
H
cubic equation
fl
8. - 'A.
Ci
F.
I =.0.
Take CiX and CiY the axes
thr•Jugh G and ,parallel to. the
y

G F C-;\. plane base be tak~n as the axis


of x and y respectively and GZ ·~
.TI,is cubi~ equation in ),, is calleci' the· 1cducti~/cub:c·.'
the rentral radius as the ·z.axis
.. El<AMPLES . . · · then. .
Ex, .60, ·r; A and B be tlte. mo:nenri· of in.min ~/ a u//if:o, !,i .lamina · A;= M.I. about· GX = M,I.
abo1t1 parpe.ndil:ular ems OX a11d OY, lying in:'its plane, aliU /.·be rhe about AB -M.. CO 2
pmliuct of i11erri<:1 o/1he fami110 obow rhese /Ines, shaw r~a/ th'e lp~incipal
moi11c11ts 01. 0 are equ!1/ to : ·, j! l .='.'25 Mii2-M ( .83 a )~ =:320
83
MJ,. '
J[A+B± ✓ ((A-B) 2 +4F2 JJ
Sol, Here we consider the uniform lamina, so there
ellipse at .0 w,hose cquail~n is .given b:,, ·
• i ·j !
Bt•M.l.aboutCiY=fMa2 -M ( 3Ba
'. ',

f , ·, =-.-.Ma
..:,20 .
83 2,
. .
·..
.

.
Ax• + By2 -. Zflxy"' Constiirit ·· . .. ([) • r.nvarlanu I lf by change' ll-'<t$ ;Vitnout th: chnn;c or origin a.< +bl+ 2/u:y tnnirorm to 1

,·)., a'x' ~ b'/ • 2h' ;;y ,\hen a'+ b'.,~ a +.b 111\d f b' - h' 1 " ub -1i?:.Jnu1 a+ b'and ab - h1 1.11' e:1llod
1hc in•1i.r\wi1 or tho sy,i,n,, .:::,f· ··
VIJ'l\~

::~(;: :!:l::•

.., ·.::!:'.::;:',' ··:.,···~e:;•, . ,.:-::::··!!·:: ', . '·"-:·.;·,::-::-. ,' , ;:?::{• I

'••,•r,•:,1~•, •,•;~;~~·:J; g:: ;: , .'• , .::,:, ·, , ;': l ::.~ '•!:'.~ ~'.~ '. .1 f~'. I ,j ~~ ,: .
.·.•;Ji'~'.iY•!\••; .. • · ,., .. '

-------==·..,.~,.m,m,11n,111t1etmu(mtlfJ'WUl'lBmTtHffi~
,·-:-r---~~~ .··.··;-:;::·:~;
:.!.'.::;??:::.:··
~ ~ > J... - - - - ·

.i-::-,'.\ \:· ,; ~j ' •

... ,, .. ;,.:··.·
,,,,,,,,•,<···
.. ,I

w , . _ . , _. . . . ., . . . . . . . . ...,.._~,,,m,r:.'l'\;'1,-'\."lc,-"°'IIJ"M.."I..._,._..,_..,..,,,..~..:•.., · ' ,,.,, ··.·,..,.; ...!;.-.::.J,!l:;,;,;:.-::,• :,,a;:;IJ};,_1,, ·

it'
;;
..-~ ,
~i 82 {?)•namici of Rigid
Mom~11.1{a~d Pradum of Inertia
... 1
;,
C=M,t ~bou, CZ=f Ma 2.
: ; l. I ;
83
:, O:=,P,l. about OY,OZ=OandF=P.I. about OX,OY=O.
.No'I{ cccrdinates of C are (0, o, -'3.k), ,, OY/is' .the principal axis al 0.
. D = P.l, 'abo111 C:Y, OZ ,
= P.l. about parallel !i~e~ CB, CE- P.!, of M at C about GY, OZ ~ M = M~ss of the half cone
= i. (! pith' tan 2 ·ex),
· "'.° 7.M._D. (- Jata)::: 0, \ 2 l ; . : .
2,
S1m1lnrly, E"' O, F::: 0, D::: 0, £!. =· F. 8 7[Pr\°cip~I moment about OY}
,', CX, GY, GZ are the principal axes nl G. =~ : _pnfi5 (tan2 a -1- 4) tan 2 a ~D
20 ":'.l'-'
Hence 83 Ma 2, 83 Ma 2, 2 Ma 2 are, the
· , ·
principal 111ome11ts. · · (see Ex. 23 on page 34)
320 320 5
Ex, 62. Show 'that for a t~in hem/spherical .rhe/1 of radius a and 1~ass J; : :
= -:Mh 2:•(4 -t- tan 2 Ct)
M, the principal monrtnJS of ln,ertia at the centre of iravizy are ·' 0
S-z5 22 1· ' ~ : ![ ; t )
"' - Mh 2 i l + - 1an 2··o: ...
,
12 Ma., 12 Ma , 5 Ma, . , . , \
~/i ,:
S : ' 4 ' )(.
Sol,·(Refer 'figure of El\; 6l)i · .. · ; .
1 M.
: A= [a(bout O, __:;. M) .I. about
Let G be thl C.0. of the hemlspherlcul shell of radius a and mass; 'OY=-Mh~ 1+-inn2 et 1
··· Here C,O. = a/2, Taking the axes of ,'f, j,, ~ in Ex,. 61\ coordinates ~.f C as . : s, •:· 4 .
arc· (0, 0, -a/2), .
C= M!IJ : about

. ,', A,. Ma2 .,.. M.CG 2·= Ma 2 - M i ·( f J2 "'l2
•· 5
Ma~.
· • •ttt"
OZ:i- ' - pith s t.an4]
2 . 10
3 Mli•'tan•
et. =- '. a.
lO
2 2 ' : I
Siri;ii!arly, B= i52 Ma ,C=1Ma andD=0=·4=F, , \
(see Ex. 9 011 page 21 J
•:· D=O=E=F,· :, the lines OX, GY, OZ are \he principal axes ai G. E = P.l.. about OX, OZ ,. •.

Thus·.the principal moments a( 0 are · = 2 J m· J.a f h sec·8prd8dr. r SHI, 9d9. rcos er sin.a
.
cos qi
5 1 S 2 2 l $~~ a,~o r=e . .
12 Ma •Ti.Ma , 3~a,
Ex, 63, A 1111/fcrm sol/cf' circular cone of semi-vertical angie o: 'and
= 2p r J rr12.
,t:;O .9,.0
a ihs sec 5 9 . sin 2 9 cos 8 cos tj)d4> d@
.
height h (nut in half b:, a pldne through Its a.xis: Show that. the principal I'
mom.mu.· of ln11rt/a at the vertex, for ·one · of the halves :are = ~ !i5 {1Y2 J a tan' 8 sec 2 e cos~ d,1) de
., !· 9=0 S=O • ..
i Mh 2 (1+~ tan 2 t) a~d ; 0 Mh 2{ l +¾ i'an 2 ci) : . =~Y h~ol 3 tan
2 i t r l 3 la 2 5 tt/l
6 .cos ¢d<ti=n-lz tan\i:t• [ sin.I!> lo
. r l i ] , . 1
· :1:.1.Mh 2
· ' I •
2
64
-lil(1--1an

2
a)
1
+(-)1an
,
a_ • 4i ! ji · · ·
· 10 . . 4 3 9n =~,Mh ,tan Ct . ,
. .
· 'Sol; Lel OACBDO be the half cone of mas\ 'M, MCBD lls semi-cirdu!ar
If the 1ptincipal aKis (other than OY) mak'e an angle e to OZ, then
base nnd OAB Its .trlnngular face. :Take the-z.a>Js OZ along OC, ,nxi~ OY "": ;' 2E . .
tan .. o·=-=
(8/Si;) Mha tan et . (&131t) tan o:·
A-C iMh 2 (l+l1an 2 o:)--l.Mh 2 tan 2 a =.. !-ltan 1 cx
perpendicular to OC In the plane of the triangular face and x-axis: OX •; S ~ , 10 , I ·. ,
perpendiculnr to thls triangular face, i . ,.,:,., ~
~n6u-·
. (81.3n) tan er.
.
Since hat( cone Is symmetrical aboui .zx plnne which is perpendicular • .• ✓((641.ln 2 ) 1an 2 a.+ (! - f tan 2 a.J2J
to :.Q Y. i ..

;i,'l!''~tr.~:)l!,1l'lN?rl•lft~1.!'lm:r~!l.mmmmi:imr:::;rmmm\!l'Si!,immnm,;i:i:ti,,mmm'1'l"''~''•lll'infflil'!Hll'!111lfHf•tmmia-mw->1(•ll,<>i:r1,i'.<0' "ii,,,.,:· •. : :.

., ..... ,...,, . -~ ...


,
,,;•r;.:.,_. •.•:!•n,~~,.

·1
ll. (x.,y,)
i
' ) + ·--3--
+,
'\I+ ;;l X3 .1'1 1'2 +._1•,,
,,. . I -.(I /4) tan 2 Cl !
nn" cos 28 = . •
2) tan·2 o:.,. ( I - ! , , (
\1[(641911 tan· ext!
' '

Hence th.~ other principa·1 inom~nt


.• , ,l

,, ,1rc thc.coordin:11c$ of C.G.,


x 1 +;1·~ +.rJ = 0
E
i:·
= ( ,· i ',;: U.,. ;-\ SI 11 ~ 0 - 2£ SI n 0 C(.)$ 0
\ YI + Y2 + ,1'_1 "0
=; C {I .,. \:os 20) .,. ~ A {1.- cu:; 20) -; E sin 29 ... (I) 8 D C
' ' (

;;
= .,_ 3 , , i · TI1us (x 1+ ., 2 + x,)" = 0 (X,.Y1) {x,,y,)
:o o: (l + cos 20) + : OMli· ( I + 7tnri• o:) ( I - cos 2_0)
' j ;
+ x} ·+ x~ = - 2 (,t 1x 2 + .,2.1·., + . ... (2)
/
1
" . - ..i. Mhi ton i:x sin 2A Si milnrly ,1·1 + ,1·i + .l'J "',- 2 0'1>"2( ,1'2.l'J + )'JYI J ... 0)
3 , , , 3 l , , . 2"'
= ..:- Ml:· ( I .,. • tnn· r() - -◊ Mh ( I - - .tan Cl) cos ,,
Sit · T.
Now, Bel = ,.,.; (,l',1 - =
+ U'; - ,l'z)l .•
· IO ' I ' . CAl = t,l = (,1·1 -s.,)l + (1'1 - .\'j);
4 ' ; .,
u sin 26 nnd,A82,., ,2 = (,1·2 -.r,)2 + 0·2 - ,1·1)2

= i'~
' 1
Mli ( I .,. ¾tan a) -
2 3
10 Mn' ( I - ¾rnn
2
a).
- -.5
1l
Mhl; tan
'
'i
'./' : '
I
•••• w
.,
.' . a•+!,•+
,
r') ,. ., , ,
= 2 (.l'j + "2 + ~1) + 2(l'j + ,\'z -t J'l)
, , ' l

· O -¼tan 2 I b) I ,
t X3.l.'1) - 2(i'1Y2 + YiYi + ,l'.1.1'1)
- 2(.t 1A'2 + "'2-t,
, , •
✓((6419it2.) tan 2 o: + (;I ',~ t~n 2 o:) 2] "2 (x7.,. xi+ x3) + 2(,·f + .rl + Y3) + (x7 -t •1 +~j) + (d + Y! + Y3) 1
4 M" 2 ·•. -(8/3rc) tan <X• ! ; '
-- " tan o: .. 1 2 , ' 2 2
Sn v[(64t9n 2) tan Ct+ (l ·- Hin o:) j or, a2 + b2 + c7 = j (xr + •12+ •J +YT+ Yi+ Yi)' ... (4)
= Mh 2 ( I + 14 tan 2 o:) - - Mh·
3 , (I - f 2
tan o:) 2 + (6419it2) tan 2 o:
.,
The 1rinnglc ABC- mny be replaced by three panicles each of muss
10 10 -1 tan 2 of] ✓[(64191t') tan 2 ex+ (I M/3 placed 01 the rni'ddle points D. E, F of the sides whose coordinnles are
3 '1 3 ' ' ' i 1 l l (l:_2_'
( Xi +x, Y2 + !',1
l-2-•-2-J\_2_•-2-p x,
(x.1 +x1 )'3 + )'t +X2 >'1 +.1'2)
= io Mh· ( I + ! tan o:)-io Mh 2 ✓[_! _-¾ tan 2 a/+ (641:tn 2) tan' N·
2 nd -2-
Replacing e b>' 9 + 7!12, the ocher principal moment is rcspec1ively. '
,:a C sin' 0 + ,'\ cos 2 8 + 2E cos 8 sin 8 . .. :, A = Principal moment aboui x axis.
3 · 3· ' . 2 2
= 10 M'1 (l + ¾tan o:) + f.11t 2 ✓[( I-:\. tani. o:) + (64t9it 2) tan•
2 2
~l•; . _:(.rz+•)).
10 +!.M (x:i+x
- - ) +!.M (xi+x,Y
1
=lM -.-- --)
Ex. 64'. Prove that ihe principal radii of g):rarion at the oj a ¢.9. ·'. 2 ·' 2 ~ 2 .,
1riang/e are the roo1s of rhe equation I i i ! U = ~ [2(Xf + A'i + .,j) + 2(.r 1x~ +-1'2"';\ -r.·
c.l . !, .,
Hz, +.i:j).,
. ·4 a2+b'+c2 . ' '
X - 36 _,l + lQ8 = O , ' . ; .. M (.rj Using {2)
where 6 is the 'a',ea of ihe trtangle, ) :i , 8 = Principal moment nbout ,\'•axis.
Sol. Let ABC be the lrinnglc of mass M. Taking the centre 6f!gfavily +Yi+ Y~)
of the triangle C as .the origin' Md the principal axes through, ~s laxes. q
I.ct (x1, Y1l, (x, 1:r2), (x3, Y3) ,_ be l,he coordinates. of the ,Ai 8, C vertices t
A + s = M (XT + --~ +.-:5 + YT +,Yi + y5)
I
respectively. or A+ 8 =-&, M (a 2+ Ii +''c 2) Usl°ng
Since C.G. 'G' takln as is r I
i
·!
Since x, y· axes Jhrough ..G are principul axes,
' ':' ..... ,'
•.
'
l
,. ...... Ir
l
\
i
'I
Ij
',; H~~ ?~~': '.~·': .·, ·: · :.-, ·,' :,;:-:; ;::~i1~ ,;. ,,;.',..; •,•·· ., :\:f:!~~:};:,;,,,:·· . ;,;:-:::-~-:-.:::1.::,;.::= :: ··~<;::::r::-.::.,.. <:::·::·::..- . . ::·::::~~: i :. ,.•,~n:·::·::·:' '·., :~~·:~1::::~:;:··~··

••·" ,. ,11;.r.e;:,;:-;.;:,.1.t.,~~l:rrn-.1~,~.~~:,:-);'f

.. ' ',' -. ~·.-:•,·;·~-·


• t\•~ ·.!, . '
::;:\.:,; v'.:,''.,,•

(1•Hnh~~~Nti•M1"""~""""'·'-•""'"'"'" "1Wo "!l'!ltl!l,l':-t'·'·.:.::~~~-•tt-;i!...({•:Um,.·1:, I',, I~.


\:
:; I
~
,l
86 p~inamtcs of Rigid Bod).' ./ Momei11s Prod,icts oj Inertia 87
s
:1
:;
!
,', ~l·(::o:~ x)'(')':~~,;•
or 3 2
3
2 · ~ ( X3; x\J( )13 ·; Yt )
d - ' + nl.a + 111,2a
. ,11.0
:1.n ~= ;m+m+m ~a.
!Z
\ · i.e. coor~i~otes of G nre ({a, 3a, a},
'
+3l ___2_
M ( x1 -t-X2
Y1: Y2 J"'o <l Let GX 'i GY',OZ' he the nxes
to BA, 8,Y:on~ BC.
3
or _(x2 +.x~) 012 + >•3) + (x +x 1) (l'3 + )'il +(.i: 1 + X2) (.;. 1 =O In. reference to these nxes thruugh
N
M
or (-x1H-Y1)+(-:-..i:2H->12)+(-x,)(-y3)=0 Using o: G the coo~di ryotes of - a
or x1y 1+ X2)' 2 + xly3 = 0 ...(5),, f-
L are (a :a:r.i, 0 - a/3, 0 - a) D
i.e (ia; -· 3~•,..: a), B a L A X
Also .AB = 1~ M
11
2
(Xi +A + ~) (y1 + ~ +.y5) M a~e!(d~.!a' O-!a a-a)
. 2 . ·2 ' . 2 2 I \! :· i ,
y
=m· M !<x1y1+xv', +xvJl. + (,t 1y2 -x2)' 1) · + (x,Y3 - x1y 2) + , { '1 , • I, 0
i.e. J-111,;\<'• ), .. .
NNow ~ = area of the triangle ABC · · f 1' ; ·I -.I · • . I i
ond 1ore:(O,-ia,a-,a,~a_-a) i.e. (-r7,l'a,a)
:· =f (xi <Y2- Y3) +,½ <Y3 - ;11) +·x, <Y1-: Y2ll :. A1 = M.): of 1he three rods abou1 CX'
or 2e. =x 1 (y2 +·;1 1 + ;y2) -t- x2 (-·y 1 - .,2 ..; y 1) + (- x 1 - x2) 'Jli - y 2) Using (I)
::,)111.l. ·of A'B + M.I. of Jici+ M.l. of CD about CX'
73 {xJ'.)'2·-x2)'1) orx 1)'2-x 2y 1 "'34. .,t
=[m.H.:la) 2 +(-a/)J · ;~
Similarly, X2)'3 -'x3y 2 = f 4. and X:;)'1 -XJY3 .. ½4, l '. •, .
1 1 + [fn1a2 +111, f (-1a)2 + 02)) + [rmal + (j-a)l + a2)J = ,p 111 a'2,
:. AB,. '~ M', [0 + (1 t.)2 + {fti.)2 + (f ;:))2].; tfo~2 ... (6) 0 1 = M.h of the !\tree rog's ribout CY'
U k 1 and kz are the principal radii of gyration; then A =MJ:1 and B = Mk1 ·= !lmo
J
2 + m I (- a) 2 + (~)2') + jln1af + 111
.. ' . ,1 ,\
[0 +(-!-a)')')
.)
' '
· 1
~ l . -t-m(~2+(-fo)")::
:. ki + ~ = M (A + B) = (a2 + b2 + c2), [from
36 C1 = M.l:. pf the three rods abou1 OZ 1
AS t. 2
an v,,, k21 ' k2=-=-,
'.l t-r{J, l08 .
= [l111a:
J ;
2 +~, i<Ja) 2 + (..: la)fl l + [~1 (H•a) 2+ (-la) 2 l)
. . l . . .J J . .
+. (k/)2) l = 2mal.
:. ki and~ arc the r~ots of the equation II
i '
! .
+ (!n1a2
)
+ 1/1 1(-la)•
) )

x4 ~ (k1 + ~) x2 + (¾7 , k1) = 0 1 =


, :. ,
.'
a:bob1 GY', OZ'= I:
=nl (-la) (-a) +Ill (-!a). O+m (la) (a) =iua2
rj I ,\ .) , ,I
l 2 . 2 1 2 1 1

or x4 -

o , .
36 (a + b''+c ) + 108 ti. = O. £ 1 = P'.L:abotit OZ'. OX'= l: 11
?

Ex. '55_, Three rods AB, BC, CD,


C I • •
each of mass m. an.d length 2a a're · = 111 (-.a) (ta)+ 111 (C•) (-fa)+ m,a(-1a) =: - 111a-, ,
such t,har each i} .perp,enditu/ar 10 the other two. Show rhar the principal .and F1 =PJ.'about GX'·, _Gl''=.!11.ix 1y1 \
momenu of f~ertla at the cen1r1 of mass are ma 2, ~ ,;.d2 and 4ma 2, =111 (ja) (-fa) +m fa) (--j<:i) +m (-fa)
·so.I. Let BY'bc n llne pnrallel to CD, Toking BA, Bl', BC as lhe,a,m Hence· tl)e,,momental c1/ips<2id a1 0 Is.
o( x, Y,. t respectl\lely, the coordinates of middle points L, M, N otfrods
AB, BC, CD are (a, O, 0), (0, 0, a) and (0, a, 2a) respectively, . A,.?+ e,,
2 + ciZ2-2D1)'Z- 2E,t.1"- 2F1X)' '; :l_11,k4 ·

lf (x, Si, 1) arc the coordin•ates or !he C;G; 'Ci' ·of the rods AB, BC, CD or .!!!ta2x2 + J!!ma 2
J ; j l
i'·~ "';,. . . , .:\'.:· 2ma2)':~...
+ 2nia 2z2 .- -t, 2ma 2u
-~-•:,;.
-t, 11i1r.i' X)' = 3mk 4
·) .
e~cll of mass m, then . · . or +ma 2 [·(Ox2 + !Oy" + 6r2 - 6:;t + 6,x + 2.))'),;. 3mk\ .. ,(I)
- m.a+m.O+m.0 I - m.O+m.O+m.a I tfl
.f ,. m+m+m "')a' Y == m+m+m "'3a
gt,~ 1,.
f:
i'
.\

~?:'\::l!t;f.f:;m:;:; ;f;"f~t~HS<;;.;;:-;~,. f:7~:::: '.~'. •1;,::~:;;:~~~'°'tru;f'.'.:;.l,'lf.f{;'lj,llrl:ii;l~;?,i4Htl.!i/i.. J > l t - t ~ -


,
.,,,_,;,_.:,:,,~.,...,.. ...,,.,,,;•,.;,,_., ' •' -, . ·..•.~/-~.. .'' . .-/-:.i; ·:-~•·,.~--

origin, prnve tho.t ir on·~ ·oi ih\! prfodp;,[ at 1hc roi111 (.\\ ;:} ,nnkl!S :mile O v..-'Hh
Reducing !Ox2 + IOy' + 6~ 2 - 6yt + 6tx + 2,t)' by means Qf the
OXC\

• h " 811/1.n• - i:1I •1.!J,1• .fi' • 1') (I' ,J2' • 1·) -, (<'l - ,l:lJ
· cubk 1? - (~ + b + c) >. 2 +.(ab+ be+ ca~ f ~ i2-h2) 1. · ;.• •ox.is, I en 10n o = , 1 ,

ilb {(x· - .,.- ,1- ,r - /J'il - rd


\

i~ {nl
. r.·
•· r).'. . - 21,i"vl •· r)'I
~ ........... \.

- (abc + 2/gh - al-


bg 2 - c!i 2) = O, we hp~e 9. Thij prindpal :ixci: ;:,1, the ~cn1ru or gravil)' b1:int; the ax~$ 'ol' rdi.?rcn~c. ,\b11.1in lh1.* ~qua11 Jn
2 of the cllip,old 01 the poin! (f', 'I• r) 011d ihow that th~ pfincipol momc111s 111' i11e11i;i a.
i,l - 26A + 2011'.-396 =0. ' I,
aru root~ or
or p; - j) (1'. - l 1J'(). - 12) = 0 :, 1,. = J, I !. 12.
Hence 1h~ equaiion of the momcn1al\ ellipsoid (I) referred to
~xes··through G tokes the form
piincipnl
(/-A)IM
/>q
I)'
--,l- ,.: pq
(I - 0)/ M • ,-' • l'l
(Jl' a-<.VM••1 •</"
l'/1
(fl' '
1•
\
l
=:I{)

p?+ 1iy2+12;2}=3mk~ where I. M. A, 8 have I here u,ual meanings.


or nw 2I 1 + !fma 2y2 + 4ma 2 i•i 10, Find the M.I. or• quadrant of th• clltptic an! .tlt,} + l1 1 " I or 1~01, M ubuu1 111,: r1
1hrounh its centre •nd P<'1'en<liculor lo iu pion¢, ihe <lensitr 01 ~,,y p<>int is pr~~'"""·"'I
H~nce . 1he ptincipul mory,ents at the centre of are 10 xy. • .• [Me,cr.ul 90 (S)I f>C
IL Find lhe M,L or ihc iolld ccncr;)lion b)' 1hc rcvoimion ur fhu f1:lrnt!n10 ,v • 41t.\' ah9v1
ma 2 . nu, 2 nnd 4n1C2~. the , -oxi, rrom x o O 10 x ah about x •"-'IS !Mee rut TOC 9J(I')] (lllnt. Seu !!>, ; I
ll, Find the M.I. of ,n ellipsoid abou1 lhe ._;, or 1.. (Mecrut )rl,)C 94, 94(1')1
}:
i::;
1.
EXE_RC\SE ON CHAPTER I .
Show <hut th• moment or lnenia or the par1 or the ON;a "O{ parabola.cu1· off by •n~
,'
1(,1ns,f,1,1 ;:
,:
ordino« •t a dimnce X r~nl\ l~e vrnex Is (3n) M>, 2 1bou1 the inngont a! 1pe vertex, ,.
!!
Jnd (I/$) Myl obov1 the princip~I d!ameier whm y is the ordinate correspo~din& 10 x 13. Find the M.I. of the cordoid r" a ( I + cos 8) .or density p, dho111 lh~ lnitl~I lint. jl
{Hint. Se.: Ex. 4 on page IS). ·· (Mmul TDC 9C•I 'ii.
2.., The principal am ·•1 the ~ntr~ of gravity being the axe~-of reference; p{oJle tha111he
momenta! ellipsoid ~t the poinl (p, q. r) ii ' : I · :;
'(AIM+ q1 +_hx1 + (BIM + ,i + p 2) yl + (C//.i +p 2_+ ql) if I
I: 1
::·
~.2qr:n-2rpu-2pq.ry.:ccnstant, , ; 1
when «{erred lo iii ~nlrc of gravity :is origin. ! ! I . i
J, Show 1hai • unifo1T11 rod,. or ,na.s, m, is kinotlcally ,:,qvivalenl to three ·par(ie\ei' rigidly
conrc"cd and ,ituate_d. one al eoch end of ihe roo ond one ,1 iu m,idd(c point, the
~:·:.:
1!14ms of the pMicles being nv6,rn/6,ln,/J, , ! : . i
4. Show 1ha1 any lamina I, dyn;,mically <-quiv•l•n1 to •h•..,hr•• pMietes; ••~hi one-•hird
of the mu, o/ the lnmi11a; placed 01 the comers or a m·aximum 1riangle•inscribe<l in lh•
el!lpse, whose equation referml to th< fl!nelpal.,, a:<_e, at th~· cent(e ~r ,tne11i0 is
!ii:
,?ta+ /IA• 2, where mA and ni8 arc the pnnelpal _moment,. of ine11i~ abpu,c OX nnd !jt;
OY, nnd m II the· m'-U. , . · B
s. Show thn.t there Is momcnfai clHp.se Df ,U\ an~u!u point or 11 t'ri.:uigular area
1he oposltc side ,u in middle: poinl tmd bisec1s the adj~nt .sidu
touches
ii
6, Flnd 1he ptinclp:i.J ll:X;CS 0,1 a tamer point or .solid c·u~ · ~
[Hint. In Ex. 32 o~ p•gc 47, D =i ~ F ~ O, :, op 1, o~• princlpol .xis 'a1. 0. Other
e ~
7.
!WOpnnclpol DX<! .~~\.1~r0Ugh O •nd al riQhl ans.le, 10 OG).
Two p•nicles, caett· of mn,s m >re placed 01 the ex1remllies or
<iliptie area or mass M. Prove th01 princip,.t axes oi any point or 1he clmimlcrence or
the ellipse will be ih,; tangent ilnd norm.ii 10 lhe ellipse, if
•. .
the mino( axis o( an ' ~f;
t·::
' m 5 ,i
i:i =t t -2,1.
8,
Auniform l~ina bounded b)"the ellipse b1! +al/·"' alb1 hos :111 e111p11cMlt · Ii
c, a) in il whoie m.1jor a.>.11 lies in the line· x "y, the centre b.:ing at · · ·' i:::
~"I""
:,
)·.

•; •;r.:
., l!'.
{',
~-.
:_"..:,

i'
. . j . ,!i
,,,.,,,.,, ... ., .........;tul!•,i!,,·:,.
,\.. ~l•e·"·'~~.::!
· .1. ~. !~$"'l't"J\tUf'Ua-~•Hl<i1f~i'l'·~{\"l)ll.fl?'ii'P~:U•.'t•"'•"'l'f~~,·l"~"W~•"~~l'i't''\~l~~~."tf-')irir:H~·'H•J\!•xli:,;•;t'll.(<~:¥\1[
••. ;, 1 t~ H:,l!i, ,1 ,1';,.,;,u:u1,,,.J.,,., ..,·,1···• .1 •\\ :1>s;;!ti1!.. .....,,~1.1):·\11:,;,1;;u;,1.,1~·.1 ..u,,1~ •• ..
i . .,,~'.,·, ... A .. , .. ,. , ,
4,/<:":• ,.\., ...,.,, •• ,.,,,r·•·1•·:•1n'ffU••1'11('"L..,;l~W"f"11~~:NlllN~~c.nl!ffl++fitJ!~J't~tu-Nt:i.f~t'Ml~~ffHHMl't+f.JN.t>,.\iii
'I.. •, " , • '" ,,,1, ;ui ..., 1,•1• .w , r.<'l\'i71u111wil'WTl~ • , ... , ., , , .

,.,~\ \} l ! '.''i' •,•;:~r;w~::, • ,,, :·•i,' i;:;:~;:,~: ,.,,\•~;~::.:;. ,; .' ,· ::.::•;:,~>;, :::::•:t:,:! ,·•-:·• ·:':"":1;;:;:;~:·

•>a•i .... •h;!nS!.>11,().1,~.,1~~.... ,.u.,~~l$l,,m;•;.~;.\!{~

. ' ',,,.,,,.,,.
·•:;~;~-- ~:·.~~;-;:;,,.,,.;.•,
~- .. • ;,,
..,..,,;:,.\::,. :, :~.!;,,'{~;,.

,,,,,,,-,if; ..

. -·----- .. ,. , _,...: .............. ::..,J,u,,~: . .., • , .~ .. ,~ ... ,.. , ..:, •··~ ...... '···, ...... ,,. ••1'""''"'"')iil•,-. 1,;....:..,• .s~.:.:..:.'...~1.:,.~-~•• :;,.J",;:t:>u!.::1~: :, ,; .''.; ,,' •·:: ~· :.:.~1:.;.hW'!.:1. ... , .•. , •.•, •• "" ,,,,,1•,'..o, .~• ,,,t,,.,.i,HJ!h,

:t ·! •I
·~:i:
D' Alcmei:t;s Prir1dpie 91
.'•j
-~
:,..\-:_ ·.i
,/
ln cnse a b,ody Is lic<l to a string then the tcrsion ,111 the string i, (In
1

:t
\ 2 impres·s~d force on the body:
Eff~ct\ve l'oms. ·. , . [Mceruit 9S(!JP))
:1! '(,

'!~
D' Aler:nbert's
'.
Ptinciple Tire eff.ec1i~e forN /M a partiC'le is dejined (1! 1l1e p;·()d11a of irs 111ws
m and. i/t accelermion f., rr n· par1iele of muss m is ,itu3\eu :11 ihc po\n\
ii
t} in time 1, then the eli~~live forces on lhis p:irticli: at this time I nrc
n §2,h Mo!loii or n !;article, . ,
•m$; ,,,11f parallel 10 the axes,
~j § 2t4. ti• A!emb~rt's Principle.
The mol\on. ol' :1 p:irtkle h determined by Newton's second· law or
"!
i Iit {)l'O/IOrllorwl 10 tlte app,llcd force i11 'Ihm dirc•ctio11'. From ihis la,v, we
· motion, which .cil'ile., thnt '1/ic rnie of chiiirge ofmo1~ri111i111 i11 any direcrioir
,Th~ ;r~vers~d effective ror,ees at each poinhor the bo<ly :ind the'
lmp;rersru ;(extern~!)' rorc.~s Ull the systcr\l ore in equilibrium.
'• . i, ! . · (Mee rut 8S, 86,. 87, 88, 90, TDC 9 l, 9 l(P),
deduce the formula ·P = mf, when~ f is th\nccelcrritfon of th~ particie of
mnss n, in the direction of the npplicd fore.-. P. :
1 i
1•: : • !12, !}l(l'),. ?3, 93(Pl, 94, 91(1'), 9S(P)1J
• I Let:(.,,,,)!, z) be \he .~oordinMes ·of a pru-licle·'of m~ss m, · of a rigitl body
!f (.~. >•• t) be lhe eoordiriutes or ct movi~g particte of mctss 111, at any ·
x;
time I and Y, Z be the componen\s or the ;'::,rc·es ~nrnllel to ihc nxes, 1hen
whi~h; i,s in motion, 'at any lime /, If f the r~sultorn of is'
accelera,liohs x', f' i:' tl)en the. efrective for.Ce on tile panic le is
by Newton's second law or rnot·.on the equations or motion of the the resultant of the' imp1essed forces nnq R the resultant of the
nrc I · forces· ·(mu'tual actio11s·~·on lhe panicte. Then· by Newton's second
, . -· =X, = Y, 111i' = Z.' ro'rces\ inf is the resultiint of F and R. Thus - inf (revcr~ed cffcclivc force),
§ .:fa. Motion of a Rigid llody. F andR are In equilibrium, This holds good fpr every particle of the bouy.
A r!gid body is nn nsscn1b)oge of pnr1icles rigidly connected togetlicr :rp~s t (- mf';..t. F and t R nre in equilibrium, the sumnrntio11 extending
such thut the di.1tance between nr.y two co11sti1uen1 p:trticles dqes not ch11nge LO all ;I~~ po.rliclcs of the botly, · . • ,
on· account of the effect of forc:s,. But, J~e internal actions 11nd rcnctions or di ffcrunt P,arikl~s nf a body
Fot'I\ rigid body we assume thnt are in,equiliorium i.e. r.R·= 0, \hcreforc t (- ni/} nnd·tF ar~•in cquilitiriuin,, ·
the ~ction between its two parti,clcs act n!orig the str(lil\h\ ine l~e'tie; 1/ie reversed effee:11,,,e fo~ce's aciing al eacii pii~tic/e uf tlit body
I (1/ld t?e, irt'fpressed (external) forces 011 the system arc in equUibrium,
the action nn re~ction betwc·?n ~,c· two ~•.inicles nrc.qunl and opposite. Y~ctcr Method : . .
ln considering the motion of a ·ngid ·body, we write the equation of niotion Consider a dgid body fr, mo1fon. At time 1; let r be 111c posi\ivn vccltir
of the particles of the bodJ aecorcling 10 11:e equations in § 2.1. But here of a Ipardcle. of mass III ancl 11 and R the external :ind i111crnnl force~
··~··
the el\ternnl forces :ictirig on n particle of the body Include, together with resp~Ct\V,el)' acting on i · · ·
the applied fores, the Ul)known :nntr forces acting ·due io the 'nction of the By he':"lon's second.
rest or the body on it. I . . . · d2r ..
O' Alembert proposed a method which enables us to obtain all the m~=F+ff
necessary equations without writin~ dowr, the equaticris of 1,notion ·qf nil.
dr2
. . ... 'd2r ' \.1
nnd w.lthoul 1consi~cri[lg 1he ur.kno:_,,,n inner forces. This important f.
or f.+R-m:---,==0·
is based Qn .the forllowing 'ruli! which is a natural consequence . dr .·
of Newton's third law of motion. ·
The i~ternal (Wions and reaction:i of all)' sysrem of rigid b.odies /11 ·
morioit are /11 equilibr/11m M1oi;,11s1 thems,eh>~s.
i.e. tl;e for;es F, R, - 111 !1 a~ting o~ a pnrticle of mass 11i nre in
Now;appl,ying t~~ fame arg~:ncnt of every par1ic:e of;ihe rigid body-,,1110
§ Z.3. ~ef'lnltlons·, , . ,
Impressed forces, . . . (fyleerut 9.S(IJP)J forcf ,ti:, tR.and t (- m~;} are in eq~ilibrium, whm 1hc summadon
The extema/ forces acting o'n a· body ore calied 'impressed for4es'. !
For example, the weight of the body is the impressed force on the b~u:,-. ex1erids to all pnrticles.
"'·'• '• .•.n1".. ;,J.. "·,•,~•,,,_. ··~ ,., -,., ,· • :,,.:,.h'./,,\',., ·.,. ·._, :.::,:v.• ·,.:.:..,:.,

,., ,,,,e11wcrI s /"r111c1plc 93


l"Ji.1, 11!~ 1111,:,rnal_' li•l'~C,; ;i~1ing on ih~ t>r>ll)' lilr111 fl:!i1·s ol' cqu~I mid ...............
,11~ for,~,·.', ER =-0.
E_quati.Qn! (I), (2), (3) st Me thnl 1/ic sum;· n/ tlif ct1I1ImI,•Nt.r,
:.I l1u., -ti!~ l11r,-~,
, . . (--- · d'r) . cqu1i1bnum.
· ...
1.. l·,,111\J .l: -_111 - , llfl.'. Ill_ -10 tlic c1>orcli11al11· ,1scs, of thr ~Jj,•,·1iw Jorn•.,· is
· , _ dt)· 1 _ SI//IIS
:Jorcn_:
ojrhe Clllllpu11ei1_/$ /W!'(!(fo/·u,
·· o ·
/lif ,\'(//11(' me;·
··
i,(',
d"r
l:F+ r.l-:111--:;- :,Q .
' '
·-.
.
_ Equations (4), (S), (6) sthle· lhnl 11,,i ·s11111s nf 1l1e mo111c111.1·.alw,11 1!1e
_ dr _ , __ I . ci.re;· of ,:i>orc!ina~1· of 1qe effecri,e forrr., <11'<! rrspec1frcl,1· «1111(1/ ui 1he
/-te1u·,, ti"• 1·,•1'('f'sf'(/ ej]c·on·,, Jon·,,s (J('/n)g at e(Jc/t JU1rt1c!c:
11J1d 1l1e 1111111'<',\J<'rl 1,·.n1..:rn,1n J(Jrc,;.r on tire .!_\'sr,•m are in s1Ims of 1/}c m,me11Is ·a bow 1/ic Ja,ne ,ucs of rite exr~r11(1/ (/111pNs.vccl) forces
,-..:01,,. Th~ nh,,,·c I)' r'\kmh,•n',; rl'in~i11lc ,·ctlui:c, 1hc prol
1n the prnhkn1 .. 1 si:nis·,. Thui -i1c 111nrk iill ihe ·cx1crnnl l'or~cs
The ¢qua1ions (I), (2) an(i 0). c~n_ :;;e written us
I
4 (l:111.i·) = I: X,
,,,1

furi:cs i/1 Of)posiic dirccrions ~nd then


:1n(J lllilfk Ill~· cli'~i:lr\'~
prot>lcm a;: ,1 pml1kn1 ol' s1~1h;, by cqun1ing 10.•zcro the
,d
di (E = r Y and Ji (l: 111i) = rz 1
ull th~~c 1111,,·, t111v.-n 111u1uillly r~rpc11dicular dlrqct/ons and shows thin rhe rate of change;! linear 1110/llll(!ltiill of rhe symm
about ,t11lahlc_ rnints. . _ -i11 an)i direction is equal to /,\e toial exremal force iry 1h41 direc1io1t.
'§ 2.S. Genera! Equations of motion of~ body, The equations (4), (5) and (6) can be written os ,I
ro dcdua !hi! gc,!e,;a/ rq1w1ir)IJs of ,1w1im1 of a rigid bady Ji'(1/i1 'D d .1 . · d - ,: , 1
Alc111her1', 1iri11ciple. ' di (rm () { - , ) ' ) l = I: (yZ, - t Y), dr I!: 111 (U - xi)} = :i::_ (i:X "'." xZ)
(,"v!ecrut 89 ; TDC 92, 92 (P), 94, 94 (P), 95 (P); 96 ;) d ( . . 1· ~ . X ., •
and - :i:: m (.i:y- yx) = .. (x~ - y )
Let X, )'. Z tic 1hc co111ponc111s, pnr~llcl ··-to th~ axes, of thtj ch:rnal dt . I

ltiicc nc11ng on ~ p;miclc nr mn,: 111 who~c coordino1es arc (x. y, ,) ril :1in1e Which shows that rhe· ra:e ef ~1:ange, of angular momewum (mome,11
1, referred lo ,my ~el ui' rc,;tungul~r .i~cs. Then reversed cffccli~c ;(~recs of mome111um) abou_t any given axis, is equal ta !he total 111ome111 of all the
., lo !he ilXCS on lhe pa11iclc Ill ;ire - 111~·. - 1i'iV, -111z'_ ifhus: lhc •ex1er4'1/ forces about the a.xi!. . ·
. re,\IIIMI •A C.i~nrnl. l'orces and lhc reversed effoc1ive forces ac1i11g: ol/ !he 'Vecto\- Method Cons_lder n rigid body in molion. Al time I let r .be the
1
pur1ic1c 111 pJr~lle/ 10 1he n~cs arc X - IJL\.-, Y- 111i.' ond Z- 1117,' ,espce1i ❖ely. vector or a: particle-of mass m nnd F the cxtemal force octlng on
13y I)' Alcmhcn's principle lhc_ force$ whose c".mponent~, i '.I' are. I.

X - nu', Y - mj,'. Z -. ,,,~· ac1ini; a1 ,he particle m nt (x, )', z) toge\hc( ':"ilh ·n,cn by D' Alcmbcrl's ~rincielo, we hnve
simil'ur l'orccs ·acting at each other .r.or1icle M 1hc body, form a-sysi~m in
equilibrium. .. · I : !. F + !. ( - m d2r
d? = 0 I
Hence, n~ in ,1a1ics the six conditions equi-li.br_iurn • are of d 2r
1
r ex - 111.i:') = o. r (
Y - 1115•') = mi')= o. rcz - o or Im dt2 = F.
l: (y (Z-111i l-z (Y-111j.')j =O, l; (z (X- n1.i·'l-x(Z-111i')) = 0
r
ond (.1· ( )' - 111j:) -y (X - 111.i-')):::: 0-. - Taking eross product by r, we have
wh,•ru 111~ ,umrn,ui1)i:i is extended 10 all 1hc pariiclcs or the hody, d2r ·
Thc,c si.1 ~qun1in11s can be written us :E mr x dt " !: r x F ... (2)
I: nix' = r.x ... (I) r. 111j,' = I: Y ... (2)
Equations (I) and (2) are in gencrd vector equo.tions of motio11 or. a rigid
r,;1i'=t.Z ... (3) l:m(l-k'-t,i,')=l:(vZ-tY) body.
and
[ Ill xn "' [(zX -
(z.x' -
I:m(.rf-yx)=E(xY-yX)
XZ)
:Deducrio11 of general equatio,,s of mation In scaler form.·
111cse cqunlion:; (I) 10 (6) arc the i;cncr.[ equations of tnot;on of :a • To .de~ucc_ the general eqlf
tions of tnoti,,n of a rigid bo_dy, we substitute
' ' :the following In (1 ), (2), , .
r =xi+ yj + zk ar,o F =Xi+ Yj + Zk
iwhere (x, y, ( 1 are. i~e carteslnn •:.oo;d!n~tes: o,(· lhe_ pal'l.icle m and X. Y, Z
;arc the comp,:,nent~ ,?f foicc;I:,fiinrallel to .the.?xef r~sp.fc1ivel)I,

·:.:::::i;;t::: , I •• l, "f• •~ ... ,,, ., .. ~ .':··:; :j;~5~;.~?;1r:::·w0~?;~1.::,r.~~lP.fl?Rt~H.:~~·w~,;.;.;:!•'"~.f4Uf-NF:~~~),'-,"f4',~',(,,"(~~j~~,-~,(ff"""...·'~}'~-,~1Rf~'l~JH""H(•ftJ1

·, ~ ',·. :: :~ '. •~;: •"' ,I~:::• .. ,; ···;:-::?.:;·· :r··• ·~'ff;'.:''

,._ _ _ _ _ _ _ _ ,,......,..,.r,~-~-, ~ · .. / ,, .,.n,c.,.,.!-)),4''#\U)-.i-O:iU11~!.,,... ,,,,,:tA:L1P.!,u1.1S!<!-•M!~)l.di

::?·
·:,:·
·,. ~~::: ?i.::;.
:;;,:•,:·,,

. . ' , . .
t}
f1--·-·--···-"""7~-··.·. .·.~•-"--,···-·-·----·-"'' ". ---~~-""'·"="·'· .... ,. --""""'··'·"'. .-..........,...,-..~····..............
,::, : •Ir
..-··
'{• I
l '
:• 94 , \ i
Dynamics o/Rigid $oil;• ' D' Al:111be,1's Pri11clple 9S
',.._·,
Substhuting In (I l and (2), we get 1 , d'r
.i:: 1,1 (x'i +/J + i°k) = t CXi·+ Yi+ iki tm..,-;-=-F.
t/1·
... (I)

and t !111 (~i: >:J, +tk) ~.(.i-'i ~ j,'j;; f~) l =. ~ (.d +:,J: + ~k)x (Xi+ YJ + Zk)
or Z nr I(J•z - Z)') I+ (u- - xt) J+ (.ry• - yx) k J : · i t
If ~~ the positkin vacwr of the c~ntre of ine\tia ~f the body, then we
hav~; · · ·
:~ .. ,t ((yZ - zh i + (zX-xZ) J + (xY- )•)Q kl ... m · ii _tmr tmr -
·::" Equnt!ng,,coeffic!ei11s of l,J, k en the two sides cf ,eq,uatiO!lS (3) and (4), !·•. r=--=-- or I:111 r=M r
1 • I:m M
:,
~ii we get tHe 'six equations of motion of the rigid body in cartesian t\mn,l ,J.'

: ; i. d2r d2r
·-~~' § 2.6. Linear Momenrum, · •.. . · 1 •·• tni7=M 2 : ... (2)
The Un ear momentum in a given 'qirec(/on is equal to tl,e ·prod1<ci hf . . d dt
l1 the whole· mass of the body a11d the resolved part· of the velocity oj Its (1 i nnd (2 ), we h'a ve

1' centre oj grav/1)• In that direction. . '


Let (i,y;!) be the 'coordinates of the centre·of gravlt)· ofa body of
• • · 1
!:
: •
cPr·.
' ..Md}= 'i. F,
·~·, .• •• ' • • •
.
• •
'
',
' ,'
.
i,:;-C.l)
,1;1:; ·~,.

I. mass M, then we !)ave ·


1 = ~ ~ !. m.~ ·, ; !. m;.. M . ·\
•: Which is the vec.to'r form ·Of the equation of motion of n pMll~le .of
mas,s 'M :p!hcecl ·~; ·ti)~".centre or
i'nerlla of the body and nctee upon J,f the
1. r.m M ' - '· eite,rr.al ·forces,!. F..• ::. .
l
i r. m.x = Mi, Similarly, r. my = Mj and I: m{,:; M'i. · · · Deduction· of rhe.·eq'ualio11s ·of morion of the cmr're of i11Crri11 ;;, scnl11r
• I Differentiating theie relation w.r.l; 't', we· get form. . ' ·. ' '
l
• i .. t mx=/ix, tmJ1 = /,(j, nnd tmi. =M!. 1
S?bstitutin&. r =xi+ _Yj + tk and F = Xi+ Yj + Zk ill (3) l1\d equating
Hence the result. · ·· ~ · the ·c~,efficients of j,J, k from the two sides we cnn get the. equations of
§ 2,7, Motion or the Centre of Inertia, mo1\J,n,Qf the ce.ntre of inertia in scalar 'corm. 1 ,
, To show that the centre of inertia of a body mO'.•es as if al/ the mass Not'e,i• T~1¢ proposition discussed in § 2.7 is .called the principle of
oJ rte body were co({ected at i't and if aJI the external forces acties on the ,·on~e'(wl1ion of mo1io11 · of 1ransfa1io~. From ·1.his it f()l\ow, tha1 tile 11101iun
body were actl11g on it /11 direclio.M parallel to '10,re iM.'r'lrich 1Jiey act.'. of .d:.o. is Independent of rl:ltation. . .
If (1 :ii, 1) be the coordlna\es of the c~ntrc cfinertia of a body of
1 § 2:s. Motion Relative to the Centre of Inertia,
mass M, then as in § 2.6, we 'have ;r,; show that the motion or a body about Its centre of lnenla. is ·
r. mt., Mi, t my~ My, t mz = !.f'i. : the sam·e !IS lt would be lf the centre of inertia were fixed and the
Diffcrentinllng twlc 7 ~,r,t. 'f:, we .~et . . .. . :. I· · same: farces acted on the body. .
tpu .. u;,tmy =My snd!.m;: =M'i, · ,.,(1) . (Mcerut 91 (S) ; TDC 94(R), %(Pl;)
But from the gcne,r.ill equations of motion of a .body, we get (see·§ 2.5)
· • t ~· io t X, t i,r.iy' = t Yand r. mi:'= t Z.
:. From ·(l) and .(2)1 w~ get .. .. . .
...(2) j !Lit· (x, y, i) be. th;:
coordinate·$ of the ~entre of
gravit•y (cer.ire of irienia) 0 ·of f
I •

. Mi .."IX, My f I Y and M'i ., t Z.


These ere tho cquotlons of rnotlon of a particle of mass M placed at
the i body referred 10 · . the
rectangular , axes OX, Oi', OZ
'IZ ,IZ' , mp (x,y.2)
(X',y',Z')

I! ~J.f)x·
the centre.of lnenf~ of the body, nnd acted on by forces.:tX, :tY, :tz parallel through :a fixed point 0. 'Let ,
to the original directions of the forces acting on tl\~ 'diffoten! polnts tt\e of cx,\o oz' 'I\ be the exes
1:!ody,
!
· .
' '
inroukh G parallel to lhe axes
'This proves the the.orem, . · · · 1 ox\ OY,,OZ re~pectively.. , ·yr
Vector method: CcnsjQcr ~ figi9 body in mptiot ~i tim,e t let r be !~e
X
, !(\~, y, Z) and (x:, y', z')
'positiiO!J :,,ector of a particle m of !he bqdy and F :be. externsl force acti~g a,e )tlj~ -~oordinates of a particle .
; on it. Then the equatiOQ of motion oi the body is ! of tn~ss; 111· al P referred to the
coordioa'ie axes OX. ()Y (,7 .-.A

' ' ' ' .. .. ,. '"'


~
• • • . · . . ••••••" •••H•• .• \ " . '

•"-"·"'••r.,o,• ,,.,.,' ••· ,r;,,··~•·•-.. . .. ·~······•·••'·"-' . •. , • ~·.•,-,.'I,_,,.,.,.,,,.,,

f~·mJ}t1f:\!f~;::;:,.::; ;"''·)\!)tWlffi~~~~nnim· /;i}'~.-. ::_;:,.;,;'.~, -~•;:1;,""" ''W)V ·, ' !' ··n .. , , . .,. , .·, 1 •,., • .,
~'4,t,W:1::/.'~ i l.1(t' ,.....::JiH!~~~}ffiU~~~li~: J~b~i t~ ~h )~:<n.-:.~:RPM~.•tl'iv~,:~!: l,J~~n; :, ••: ,.; ., '·.',y11), ,·, ,1;,:};'.~"J.'¼~(~H;:;,:,itt;P.)ZSVh";t~ ;.'t)~-t ~1 ~;.:!-,,~~.-..~•.~H;(•~.:..:-1·1-..~~,
'. ,,
l
\
!:•

.
~ -.•···· .. :·~ .. "_J ·:·~··· __.., •• , '1
\
.,. ~·-~~f
1wiill~I ;1~~, GX '., GY ':CZ' r~~pcctiv~ly; then
D' Alembert's
'
Pl'inci'pl~..
97
I
\'
r I
x=y+-•.~'•,'.:ir.~ .. :~,-.'·~r:+:5',:_,
• \ \' \' :=I\' + \'
-· \" -~ \ ' - ..
' , . ..=Fl:F+t'r 1,(&,. ..,(1),
i•
::·~: '
;II:\ ~l~l\:•1d~r· lhl! C~J\lnll·o·n· I. i' jj; :~ . .
iv). \~:hich bcc'omes :;:. (;:z ;_ · N9w position ve~tor or' \he centre of inertia G of lhe btidy referred to O
as: origin is o.
E11:i•',-•·'1(': +~·•1-(?:+;')(~+j: . . : .., . • 2 ' i:
~:
= !(i- + 1 'l r (i,.. I.' 1 r'I z- ... ,' . ,..1., mr, : 0, so t hat ,.,
" 0 , u. ~
,.. n1 d r'. = ,,.
01' L 1111 \' ·~··, -
.
+ if i
Ill,,)' t mf''+t !, 111\' 1
• '
I i
-!.· - •• , i ... ,
, m 1dr t;?
, .., - zy t 111 - zt 111)' - S: !. mt A~o the. equatio,n of motion of the centre of inertia is
= r, (\' 'z ;., t' + z'"': zr, Y, n n:
. I ' ... (l)
Now rcforrcd to ' ox ', (J y'; oz I as axes (h!) coordinates I or :G are
' 2
1,J d : = r, F.
'
i
i!
:~

(\l. o, Oi ..
· dr ":~
'I'" , • ! "From (I), we• have .• i:;

tm = () or I: mx = o.
: ·,• .:::.!.!!:~ 1
d '
2
(d r
:t'mr'x-:;•+ rx ~ . M 1+ 0 + 0 =rx:E. f+ :& r'x F,
2
\ ~
I
~
Simil~rly. t Ill)',= 0, !, 1;,z,. = o. dr dr )· . I
. i: 111,i:• = 0, i: mj:• = 0, t mf' = o.
Al,<, front ~ 2.7. we have M f = t X, t Y, M ,.,l: Z. Mi" f
or
· dr
.t mr''x d'~' + rx,!, 'F = ri< DF +Ir' X F ;
· · •
Ii
\'
(:
i11u.,.rrwneqn.(ll,."t-·c·get., . · · di.r' I•

t m (r' f I - l)'°.') + n M )' M"' t (y 'Z "." z, Y),: )'}: z '- i" t y or I:mr'><-:r=tr'xF. -l;
·ur t 111 {r •~·· -:')-'') + y!, -H Y=t(y'Z-z'Y)+'ytZ-D:Y dr .'
. Which is the vector equation of motion of a rigid body when th~ ceptre
...(2) '
' I f:
·;-:
or · ·(111 (>• 'i;'' -"· z'/') =· i: (y 'Z - ·t'Y), !'t-::
Similarly, we get tile Olher t1tio equations as . or inertia is regarded as a flXOld. point. ·
DcducrlotJ of lhe cr.mespondlng' eqliations in scalar fom1. :1
·• l:. m tl.X:.' - x'i'') = l:. (z'X - x 'Z) tt
.1nd . tm(x'f'-,\'',i'')=t(x'Y-)''X), lf (x, y, z) and (x ', y ', i ') are the cartesiM coordinates of the partich:s :i
:i
Bui these equations nre the same as .would have been obtaine~ if we m referred lo the rectangular axes through the flxed ·point O and d10 parul!el ~1
hnd reg.in.led the centre or gravity ns ·fixed point, i ~xe's through the centre of inertia (; respectively, then we ha've · . \:
·Hence 1he :prop:osition.
· : . r =i xi +yj + tk and r' = X 'i + y 'j + z 'k, ):
r;.
V~ctor m~tho<l, Consider a rigid body in motion. At time 1, :let.? he the Le 1: (x, y, z) b the coordinates of G referred to the axes through· 0, then r:
po;iuon. vc~tor ol' the cen\r~ or inertia ,G .. of· a rigid body o(miiss[M. L'el r = I +.y j + k. · x z :;.
p.u-licle or the body ar\d-r hs ro.~ition ve~tbt /cferred
be the IMSS <if Alsp if X, Y, Z are. the componen1., of external force F paralel 10 the axes,
Ill U then :;.
10 the Cixed origin O and r' ·11s position vector. referred to the cent.re or
ineni~ F, ··
1 Sub~lituting in (2), we have ,
, F =XI+ Y j + Z k. !ii
,Ji

d2r d2F d2r' l: m: ((x 'l +y 'j + z 'k) x (x':i + y"j + z''k))
~;
-
r=r•1-r',sothat dt' =~+-;;: ; !l
{.;

The moment vec!or equation of the rigid body is


. = t ((x 'i + y 'J + z 'k) X (Xi + YJ + Zk)) {'
:I
or !m ((y -Z')'')l+(z'i''-:c'i'')J+(:c',i,''-y'x'')k) ;t
l ::
;,
! mr X = ~ r X F, . = E ((y 'Z-z 'Y) i + (l'X-x 'Z)j + (x'Y-/;X) kl.
I the coefficients of i, j, k from the two sides we.shall. 'get the '·

or r, · (i'r r'J} . · :
(r + r ')
,..
X -:;- .. -
·dr
2
d ,-
dr
~ r. ( (i' + r ') X Fl '
·
I
f
of motion of the. body in scalar form referred to .the centre of
inertia as fixed poiriL • : · . · . I t
1:,
Note I. The propositiion discussed in :§ 2.8 is called the p,:inciple of
or ' :.. d2 r · - d1 r' d2 r;; , conseNation of motion of rotation, From this it foHo.ws .tl\S,t. the motion
!mr'x +rx-·t-m+rtm-.-. +-ttnr
.·. · dr- ,· d? · d? I : roun\J the centre of inertia_.is ,indepefldent of its•rtfo'tfon•:o_f ffahslatfon • lf
~i,,
.. ' .' ;~..-\-~~·~=;~. : ; 1 . ),:,~;·.~-.: ';·;'' ·•: :;~t:0r~>-..,.··.·,~:.' ~~fl·
l
ii!: -~
I
I ' t : . .~:
-:!~-.
: :: 1 ~,;,::;,:,~., ,,., ',._ ' ,, ••·;~.mm~rt,.. Y.'>'!~1~~~0j':},~,~~,t~fff~'.~~~m~~~~4''";;:~°;1':::~f:(;~:1·f;~+(.;\'•l\~ifr.:1,,:~~••·,:-!, :·, ;, '•.~\ ' · ·· »:, •· ••" ,,va ~r,,.J( ,,~•w,~,
•,,,,ih.,•, \'•,i\.,, . , ~ l e lC' \ta\'.1&1,,.IT\V'lt,"tr~~,.);!'J~~'H\'7(W/'n;tn-f
•!~)~;'4\MJIJJ'\,4•~•~1' " ' i .. 1r.(Jl:'IJ-:,, , ,.,:'1. Ul)!Qf'1l-'>t\~fl>r"~N•M~~·~u~11J1n•(11n•'"''i,U\\1,'i~l
,, rr,, ,, . .,...,__,...,11"1 ,,•n i,,,orr• , •

,,:,:"; ' ',: .. : ~ -: ,:··:: (' ' :, •,'•:; :: ~: ,. ·r~•.:•~t~('.;~~ .. ·•'"' .'~,•·•_;

-----=--n,r• -;,•:··--!"<;'"t:'-:O:,-, ,) ., .,.,.,,,1,...,xs:v;.ii;,,1'l•!>1Hl,·\',!t-•.~oo;.t:"t;~;;;,~~•~;m~


,;,;p;~;•,;,;•.•,

_ _ _ _ _ __,...,.. ........ ~~l'lflll.'IIUll'W~,-·"°""""":"'""\,-.•~••~:,, ..,,,,,,.', .,,, ••• "•""-'"••--·,,,,.,.:.,~i) .


, ..
)..)1•,:,
:;j>.,•U,1•#)•, •· \ ,•<1",tt•,Y';.J.'.'.i• .'' ,_ ·••• -~•- ,,!.,~u.i:1,.v, • , ,.,,,.,,,,,,,,~·-~•,·><·••<'•lo'• • •h~l•,'(~fl'\► \'"11'','•·• . ••

:J
;
:~
:; -1 j.'

·:.,..; D: Alem~;n:t- Prt'!ti/)lf•


1~
'(
98 D,1•11a:11/cs of Rigid ! ! 1 · 99
•/
;e.
!,:
Note 2, 'rhc two propl'lsi!lons· 'dl~cus~:d !11 1 2.7 nnd.'2 ..3 tog~ther prO:ve M,O+ni:.~c· M.0+·111.a · ma ·
" M'+'
·' in, I· "' M + m •.=-,--M + m =xt (sa~),
\ ('.' AO=BG•a)
.
,:• 1hi: principle oi the independence of tlk mo1i1,n of trrinslation and
.·EXAMI>LES I.ct A 'ff.' oe: tile posilioin of 1ho bonrd 'when the man reaches tlir oilier
end[! bf;tlic' boo rd. lf the b<inrd .11ips tllrnugh a dist~nce AA':= x (bn~kwn,·ds)
Ex,: 1. A rod revo.[v/ng on· a 111100th horiio111ai plane abo11t 011~ end ,durin!J., t~e;time the IMn wal.ks frorn A to D, ihen in thif l'Osiiion the ,fornncc
}: wl1iclilsjiud, breaks into (\VO part,s, what is the' subseqwml motio~ of lh; or c.q. :o{!th'e system from A (townrds 8) · ·
.. two pqr/s, ·· · '
'•
-:, _M,A8''.+h1.AC' _M:(2.a-x)-t-m(a-xJ .
,);
,i $cl. Let the .rod AB revolving aboui - M:+ Iii -· M + 111 "'~2 (say)
flii the end A on a srnoo.th horiiontal p!_an.e ' Since the .po~ition of the C.Q., 'O' of the system remains
qreak into .tw~ parts ~C a0<l C~. Clearly ---d.,-·-~,6--"---;;s : .rJ==x2: . . .
r~ the part AC will continue to :orate nbout A· or ma: ~ M (2a - x) + m (a - xi ·. ·
-·)~ A ·With the same angular _velocity. • , Mtm M+.J11
~ ~ part CB at the instant of breaking
acquires Uie same an·gulilr veloci:y ai:d ,
or mcJ::: 2i!M1-t- ma - (M +m)
x· or .,t"' 2aM/{m + M)'
. l. Its centre" of gravity D has a linenr ;;:elocit.yj':!-,Jence this part CB will ,fly
. · \.!n,ich Is the requli~d distance." •· ·
l'Ex. 3, A C"ii'cular !Joard.J,i:;placed on a ;-1nooth horlz"om,i! plane 1md a
l . oft along the tingent line (the. direction ,of''llnear velocity) ·at D to :the ~ns- rou.nd1 the edge of ir at a uniform rate, wliM is tlie motion of the
i olrcle wiih A as ceritre'and'AD .as rad:us', Also, since the motion .of~ body ,¢ board. ' :. • ,, • . • • ·••

about its centr.e oi inertia Is the sarne as 'if'1he ce.ntre of inertia wa~ fixed
!i and I.he same• forces acted on \he oOdy, lhe '!,an CB will continue
at.out D with the same angular velocity, 1 · ' •
Sot. U1 M be the mass· and O the centre of the bourJ. !C initiolly the
bo;;1 Is a1 the! point A on. the edge.. of tlie board then. the C.O, '0' of the
system will lie .on the radius "OA: such·thal
Hi'nco the pnrt CB wlll move. a..ong. the
tangent. at' D .to the, circle with
00 -.:,M.O+m.a : . . ~
A as centre and AD as radlus with the velocity a'oquired by its. centr4' of· J l.: . - .. M+m• -M+m'
gravi.ty at the instant ·of brea~ing ar1d this part wilt also go on ·rotaiing S{n~~ ;th~ external forces, weight. of
1
about D with the same nngLiliv velocity'. . ' ll1e board arid the boract
----"\ .
· •
• Ex/4 A rough uniform:./Joard, of 1)1ass m and,/ength 2a, res/J ~n a
Jmooth horhontal plane ar.d a man of mass M wa//q Ofl It froth Qne ;end
downwards: p~d the reoction of the
smooth ;horiz.onal plane act· verticR!ly
I
/ ..
·10 the other. Find th; dislan.ce through which the board moves in this rime. therefore there is no extemnl
· (Mee.rut TDC 91(P), 9l(S). 9Z(P), 93(P); 93(BP), 1'..,;·cc' in :oie horizontal · !
o·----.-jA!m)
a , . 1
Sol, Here ftl1e external · the motion. 111us by D'
forces ,;re (i), thefwclghlll or Lhe principle Ille C.G. '0' of the sys1em will ~ /
remain at: rest. Hence a~ 1he boy runs

,;;;;;;f =,
tioar;d and th·o mnn actln~ A ·' a,
vertically. downwarq~· a~d. ~ll) • ~~. r und tha edge or !he board with iJnlform
spend, 111~ centre O of tile bo(\J'd will iJescribe a circle Clr' rutJius
°\'
the reaouon of the hor1zcntal · 6, (m}
plane ac!ing vc.tiicatly upwards., A . · , OG = mal(tri; + in) round tile centre l\! G.
Thus thcro aro. no ex tema l , Ex.: 4, ,Find the motion of the rod OAB, with rwo masse;- 111 and 111 1
(o~es ln the horlzontal dlrcctlon, 1herefore bt.D' Alember1's principl~, the attache~ ,~· i/ at A and B respectively, '",)nen it move;- ro/./nd 1/!e 1·erticnt a,s
C.O. of the. system wi\l remain at rest. As ·a maitcr of fact as !he;man fl cof icq/ /pci;du/unr•with "niform angul11r velocity, the' adgle 9 wliir-h I/if
moves forwlll'd, the b\1o.rd sll'ps backwards, keeping th<! poistlon of \C,O. rod ma/)e,I Plilh the vertical .b.ting cp11:ta111, · , , ...

or the system unchanged, . . • I Sol.\ ~et! OA.8 be the r,>d with• two· masses 111 nnd n1' at1aJ1ced nt
A and B re,spectively such thnt OA = a and 08 = b. When the rod OAB moves
· Let AB be the position of the board when the:mnn of mass M ls at,A.
Distance of C.O. of the system frcm .t..,(t<?.wards B) 1. round 1ttel vertical es a ~onicar pendulem with uniform angular velocity, ·
: ! l

,,,, .. _., ...... ,


',.
., !VI
I
11/\1 D)'nllmic,, of Rigid Body
M < •
\
making constant angle 8. with Za v.( . x st n i:x , along MP.
y
the vertical the masses Now by D' Alemb·cn's.principle 6 \
Ill and m' move· in circles" on I u 1he reversed effective forces :i:.<~ !
. ho~izon1al planes. with radii · 9 ... acting at different points of the rod, ~(M/~a)6XPMlll' l
11 ~in 0 and b sin 9 and centres , .' lG and the external forces, -weight mg ( ,E,F.)
\
01 Mand N respectively. The ' ::~1 ,.,, noel i·ection at O nre .in.equilibrium. i
' . . h ' lb I '. • •• A ma.!l.\tle.~
mQ11on ahQlll I e verl!ca e ng /: •• ·;; (A,E.F.) ·1 To avoid reaction at O. toking l
with unifonn angular velocity, /
.< -~'
. moment about· 0, we .get · , ..
i A
l:
\he effective forces are e'ntirc[.~
in wards. let~ be the a9gle thnc
ihe plane through OAS ·makes
:'. !'211 •• m'slne,l!
<~:::. · ·..: :: : i(R.E.F.)
j · -I ..
~(~·5x.w 2
• sino:.}oM-Mg.NG=O '.iz Mg

.•
wiih a fixed venical ·piano z m'g , 1
. 'l•}vt
through OZ, then the .only orJ I;w-x1 sino:cosi:xdx ·I
·!. 0
.effective forces 011 the particles .: /
nrc·ma sin 8 ~2 and m 'b sin 0 ~• along A,1,1-oiid BN':respectively,: t ~ Mg,, a•sin o:= 0, . ( •; .OM ::;,x cos Ct.)
fly D' Alembert's principle the external (orces, ·weigl:its mg;_m 'g anri or,~ w2 • {½ (2a) 3}, ~in o: cos o: - Mg a siri. ~ ·':: 0
th~· reaction at 0, and the reversed eW:ctive . forces ~,a sine ,j,2, nlo~g
· MA and ni 'b sin 0 $' along NB will k.ee.p ~c.:tod equilibriti~.'· · i~ ~r
. ,•-(4a· 2 ;
Mgp sm Ct _ag w cos a.-.
1) =0 · .,
To avoid reaction al'O, toking momcrit abou'rilw point 0, we get :. eilher sin o: = 0 i.e. Ct= 0 .
ma sin 9 ~2 , OM+ m '. b sin O$2 • ON - mg , MA - ni 'g . Nil= 0 4a 2 . • . ... 32
or -- {I) cosi:x- 1 =0,1.e.cosa.=7
or 'ima sin 9. a cos 9 + m 'b sin 6 . b cos 9)' ~2 = g (m~; sin 0 + m 'h _sip 9) 3g . 4aw
or $. ·• = __(ma+m'b)g
,.___...,.;.J..,....__ ( '.' sin 9 io 0)
fma• + Ill 'b2 ) cos e : 'I ' .
. . H~nce, the rod is inclined
. .
~t an ao~le zero or cos~ 1
· 4aw
(2½JI ·
, , Which "!{.ill determine the motion of the rod. j ·; ! i 3 . • 3
Ndtc. It" w~ < 1£. then cos Cl.> I, ,', in this case cos O:,. ~ gives an
~\Ji¥ Ex, S,4. uniform rod O A;· of lengrh 2a, free ro turn apoht) /Is end• I
; 4a, . 4aw
(!,I, revolves, with llni/orm angu1ar velocity•W. aboa.r l~e venical: Or "through t lm~ossible value. of o: i.e.' when w2 < 18. , then· o: = 0 is the only -p~ssible
0, and is Inclined at a constant angle o: ro ·oz, ..s~fJW thdt ihe: ~alue of ·!
' 4a .
1 2
1~ is e/rher tero or cos- (3g/4aw ).: . : vnlbe of o:. ,
(Meeru.tTDC 92, 94(P), 95(BPJ l Roh'Uk.hand 83] j Ex, 6 A rod, of le11gri1 2a, revolves with uniforn1 ang1dar vc(oclrj• (I.)'
Sol. Let the rod OA of length 2a and mas~ M revolve 11/il~ uniform abq111 a ve_rltf,:a/ axis through a .rmooth joi111far 011e ex1remiry of the' rod
•nngular velocity w a9,Q.ut the ·vertical OZ .thljOugh 0, mnl<.i,?~ ai ~onstant so 1ha1 ·11 • 1escribeJ a c'Cf1.e of ,.r~mi-vertica/ angle Ct, ;/iow that
;;nglc o: to OZ. Let P.Q =-ox be an element of the rod at a distance .x from ,i =3g/(4a cos o:J.
0, The mass of the element PQ ls ~ Sx. . ProYc also 1'1a1 direclion of reactloii al tfte hinge makes with 1he::'11e11/ca(
an angle ian- 1 (¾ tan ex) · ·
This eleme11t PQ will make a circle in the horizontal plane with radius·
PM(= x sin o:) and centre at M. Since the rod revolve with unifon:n angular Sol, Refer ~igure o last E~. 5, I
Proceedin; as in lasl Ex, 5, we get
·,·clodi)', the only effecti~e force .on this eleme~t. is M 13x-. PM ,:w2 along · 3g . 2 ,, 3g '
. I .. 2a cosa=--, ,.e,(1) = ~
. ft,,f. 4aw 2 4a cos ex
'11w.l uw r,eva.ed effecti~e)or.ce the clement PQ .is ·
.:;f: ·_ •/' '•
,.

.:;~~:;/;: '._,:;. ··,.

<,;·,;_,-;,; ' '! ~:.'.'. ~: i'' ':: ,:·:~·!~~: :·


·',:·:;,:.1:: ~ ~ '.'' ~- ' :
{ ~ ,, , ., ;'. i.-, •l~-'.·:, .. ,

. ,\:,:;c;::';~:·'..·;;:r,
', :01_;:,·~ .\'..-.:.•,.. : .i,., :"•:·'. ;?: .. ~:{i,,_:ii:.',";1,,,,~...,:, "·
:-.•::-.,:···.,·

f:"-"':d.t~~~~~~ . .~,.,"""'"""'"'..;_'-,.,b,.,.,.~. ,., ..:.;~.;)~~i!:.!t..f:..!•!:.:;.:.:, ·-~:-~•,\/::tr,;~:,:: '.!::;::.uuw,: ... : : .:; • ·, .... ~, ; :.. .~ •• ...:.·"~~-~--:!-:i::..i!'!~nt!!1J:~;:.:.:i1 •::!.\.l.!ti.t)i!:.. ,!!.:O,)IJt"..lt"ti ,i.~ ,\, .t.::..i.,~::i~a.s.1,W.uw.......t-..,,•.,.,.,_.._,.~;•,~,•,,,~;.s,~~""""'"tl"'(~••r-.w-'•.,· 1••·•" rnn ,,

~ YI I
.. .•
;
,
,'"~:
102 , : bynam1~J of,RiJ;d
~ I,. I IHt. 1/11~

Second Pan : ,
I( X nnd Y 111e the borlwntal ·a.nd ~icnkal componer\ts of 1he r 1.
at the hlogc O. a.~ shown In the rtgu.'.e; !hen resolving.the force$
and Yefticaily ~ gi::1 ,: ,' q_.,
M . 2'i M '·
X " I 3;j ax. PM.<til :: IO
., ~ cJ (½(2..) } sin ex= Maw sin ex
~ w2 !l: ;in Cl. d.t· (·: PM.=.x sin cx 1
I
I/
•~~r~':, ~ 1
(Mil) 6XC¢ xslnO

i
2 2 I I Cl K :;.,~(R,&,F,)

· and r .. Mg. ·
If the reacli~n a1O mak(e J°" angl)e ~ Willi the Vertical,, lhch
X Maw sin ex a f 3 . , . ~
iv:\::.,:.
• mg

y)···· ... -
l;in e""y"' Mg =8 ~a
ltn 0: [sub,st1tu11ng from <ql ' Mg \
!,lg
•I ( \ l ,
or , G=tan 14 t~no. , ., .
1
li,x, 7. '.fwo uniform sp)1ms, ehi;:h ,of mGss.M a~d radius a, are firmi,>'
i· '"'
P'' .
~-~
flxl!d to tile ends of two uniform thin rods, each of mass m and {erig1h /,.
.. and ,the other ends of the rods are free!> hin;sfd ro :a. point O. The, wh.o/c ~·:::
,"," system revolves as In the Oovemcr of a steam Eng,na, about a vcr11ci1 -11111. K0 1 - ~1g NC"' O
l{M through O wlth the angular velocity w. Show that 1vJr'.en the .motion 1:
steady. th<t rods are Inclined ta t1re' ve'nlcal at an ·angle e, givan by t/j;t or ~in 8 con 0.tx-,.Mw 2 (a+/) 2 sin e
equation ' I I

• ~J (I+ a) + ½ml : - 1118 2 sine-'- Mg (a ... /) ~ill e= ()


cos e .,.;:.o_ . - - .- 2- - - 2
w~ .',f (/ + a) + ½m/2 , [ui. M(a+l) ).qos 9-g {1mr+ M (a+ iJJ'I sin 8 =0
Sol, Let O A, 0 B·b( two r,~s, each of length ·1 and mass M n11ached :: Either sin 9 = 0, i.e. 0 = O•.which is inadmissible.
freely ·10 a poinr O. Let C an,d ·P _be tt:e centres of .two spheres ench of (}1,iP + M (a+ 1)2 )co~ 9 - g ifml+ M (a.+l)l ~ 0
mass M and .radius a !jtleohed to the other ends of the two rods. When the .l, , I
motion ls steady, le~ 8' be the ino'.ination of the rods to ,the vertical. , _ ,8_ M (a + ()1+ 2 111I
Consider the motion or one of the spheres, say the sphere with centre· al or COS. 9 - . I ,1
, : ., 1 w2 M(a + I) 2 •~ J ml·
.
C. Let 6 x be an e.JcmeM P Q of the rod at ,p such that OP =.x, then· mass
of .the element is {ml/) ox. •
1'1ie reversed effective force at 'the element 6.x at P is
. ' ,. : , "/2 ~.~. 8. A rod of lengtli 2a, is .tttspemled 11ri11g ,,if /e11,~1/1 I, 11r.1~r",d
·" 10 C/111!, end, if 1//e string' and rod rrl'ol1·e abow 1l1r 1•~rrira/ ll'itll t1m1im11
by~
76x.w .PM= ff OX uh~in 9 alone M/:J.
I •
2

a11g1rh,i'.' vrtor:lty, an<( th~ir im:li11mio11s 10 rhe ~frlical ·1u and~ mpec1/1·rl,", e
frla( ' :i., •, ·. I

And the rtverscd effective fo:ce i)n the sphere is '· 31 (4 ran 9 -· 3 tan q,) sin q,,
ii -=· ' -·- -
Mu{l CH= Mw'- (a+ 't) sin 8 along ON. I ! , 1
' {lan;p-•t~fl B) sill 1:1
The ext,ernal forces on the rod OA and sphere with centre at C are ;,
wel!ih1s ·ms nnd the Mg a.nd,rea.::tion a: O. . · !
[Me~rut, 86, BB, 90; T,D,C. :>l,
To avoid, rea<:thm at 0, tukiiig moment abou( ti.· we, ·get 9:;(P), 94, 94(P), 96, K.~npur 81, 83; Raj. 113]

,7
, :t 8x·(l)i,t sin a.OM+ Mw2 (a -t I) Ain e. ON·
',,
I

!
.... ... ··-
,

............. , ........ , .. ·
',•,-•.• .. •.• ,,•.,,., 'c:-.·.~•.

-~~t~{. :; ;::} '. ~;:, .. ·,';;;, ~;;U,i,,1\·ii!~l-.mJHU•);{.\"~~:~h1m;:.•:•< ,~y;pcr:, q~; 11•:·-~---•','\'' .... ,, .
Sol. Let the rod AB of length 4a .
oncJ ma,s m be suspended by a +3 sin ~ l . cos e
siting OA ot' length I. Let 0 and. ifl or ~·l eu2 sin 0 + 4a sin
be the inclinations of the string and
the rod to the 1·cnical. res~ectively. Dividing (ll r g tan$
(2), v,e ge; ·f
C1:nsidcr an element ii tan e =~~
g
(I sin €)+a sin$)
PQ (= 1h;) of the rod a1 a distance
x from A, then mass of this element )( .. or w•
= g tan.8/(1 sin e + a sin qi),
is (Miu:,) ox. ' •.•. ;'!. _,=,. P (M/2a)llX(l)
1
PMj
Substituting in (3), we get
,\s the rod revolve with uniform 1 8 tan e (3) sin e +4a sin IP)
K', •• a\( r, (A.E~F.) · • ' "
g tan.., - - -.
, angulur velocity w, about th!. t -3 (lsin,8+asin¢) ..
~ertiud OZ. the, element ox ·wili'
8
or a
3 la'n ¢ (I sin .0;, sin¢)= tan_0 ¢31 sine t sin¢) la
radius PM in or 3/ sin 0 (tan lji - tan$)= sin §(4'U,fn 0 - 3'a tan l)i)"
z Mg or • 1(.,(4tan0-3tan~)sin~ . ,
.. ,;r · a (tan qi - tan 0) sin e . ; ·
cten,cn• &x IS ' , , .}/ Ex. 9, A plank of mass M is ihilia/ly at rest along a Urie nf g.reate,il
lv/ '\ M 1
·, , . ( ~ff.slope of a smoot/J plane inclined at cu1 angle c;t. 10 tlie lwrltri'11, and a ma11
2a &x.,,i·. PM "'2a 6.(.~i·. (I .s,n e-+- x sin <I>),. alo11g MP.
t5 of ,na.s.r M', starting front the upper end, walks down th/!. 1>fa11k so t/wr it
n,e ,p,ternal ·forces acdng 011 the rod l!le (i} tension. T ·at does 1101 move, show that he gels ro 'the othe; end i11 time

✓ ll (M +~ gasin..0: }• where a. is 1he length ;a/ tlic plane.


A01. an<l (ii) \ts weight. M8 acting vertically 'down wards at its 1

point (i. I
~
Resc,lving honzantally and vertically the forces acting on the
get ·
·[l\lieerut, 84, 85, 871 89 1 TDC 94(n), 97; Konupr 82;)
,Sol. Let the plank AB of m~ss M and length a rc'st along the I.inc of
Tsin 6"' J: !;
.a
Bx w2 (I sin 6 +.oin $) grca1csl slope of a smooth pfanc inclined al nri angle ex <o the ,horizon., A
man of mass M' starts moving down th_e plank from the upper end A,
or Tsin e = 2Ma w2 Jola (I sin 8+.i: sin qi) dx Let the man move down the pluo~ through _a ,distance AP .. x In lime r,
S,ipce. the plank does not move,
therefore if x is the distance of
or T sin 9 = ~ CJl2 [ Ix sin 9 + ½~. si~ ¢] the C, 0, of the ,plan~ and the
mari from A in th\s position,
or T sin e= Mro2 (I sin a+ a si'n ¢). . • I... (:1) then
and ' Tcos 8 = Mg, . . • · · ',i,(;2)
Now taklng moment about A 13( all the forces ,acti~g Ari the rod t)B,I
gC<
re ix= M .AO+M'.. AP .,_·.M, (a/2.)+M';;
I 'M+M' ·
'
M+M'
'
. ' . .- i
'
-Mg .x~+r, ~ oiw2 (1 s!ne+~sin_1ti) ,M:t=O~ : Differentiating twice w. r. I.,
'/', ;WC gel·. X
' · 2 2a - · . · I
,:c-,;, Mea sin qi= ~ro .f0 (I sin 8+:csin ¢)xcos ifld¢ · .. M' I
' 4Q• ' :i"'M+M':r.',i ... (l)

= ~ ,CJi U~ a+ f 2
sin
3
x si~ ¢ I cos ¢ ·
Now the total· weight (M + M') 8 will act vertically downwards at the
C. G. of the system. . ,
.-, The equation of moti<1:i of the C. d. of the-system is given by
I 1:
.: ~. :"-~·
·.:,1·e:;;~,!:J'_.,
' -·
','l!!t•· ... •• .,, ,., '~···' "''. ~ ~:;:mmmf!;~:~m~~-~·~.~\'!(;'~~~~;l~W,tf~i~~f!!~f'.~:t·~v:.~:·iw.:-ut~~·,?.~•~~;,~r~; Vl~:~; .~••:\' .. 11 ·:•'\Y~ll;?}·~f~l~l'frHf!.\~.~HRW.HP,'HHf~mfi-:tftf-,1~~•'1,,11~'1•'·1•1N(Vti'fJ,...f~~"•"'!t,:•~:,,;?•'-~H~~':"'-'0 i..•,,s"'l'<i~f•~~i"'il/UJ.f«l~"'J};i

, ..·. ,,,,,,,,
.' ', ,''
'. ·,:,:;-,•,•·
' ';: '.:':;'.I~!''',-, ' '••. ,, ,·-.". ,··h :·:~:~;':"·. ,;'.'.'.~:ti.::·' ,·,;::~'. ·;,:•

------=-~"Jf71"'r."'.:Q;r.Q~,",l;T'.i'.:;f.

.-.~-~:i~it~~ih~i :.
·----
,;:;,

,._•,;;i.:.::,' ',?·:~:!:

. •·••• ••••••---.-=-•~•a_,,,WlH!~O~,"""""'""••••;,.:,,.,.,, •.,,,. •••·• .;.. ., .. ,,, ;'-'c;4-.·,.;,.,r;\'. •.C,,-.:.;,:.:..,_.,C, .;JJ,:C~;tc ;:., :.•><:·•;;-;;,: •,. I; :;,•,· •. \ ., ·,•i'H•;;,i,,!\ .. ,'.

I !, i .\''11•:1•

Ir~ . D;-,wmics of Ri11id


-~ i{'
: :~,
~

CM+ M') ,',:' = (M!+ M').~ si11 a. ./(2) D':Aui&~rr:r Principli!


' ,j . 107
Fwm II) 1111J (21, w~ l/Ut ' ,: I
§ :do' Ah Important Rule•
M' .\-'= (M + M')_ g ,I~ a, The effec/ of 1111 impu/n Oil, u body i·enwim tlir same c1·c11 il lit(• ./ill(//•
wu g,•1 M' .;. ·= (M + ~ -N forces acriug sim11/ta11eo11s/y 011 ii /He 1w11lc1:ied.
0 a .1 + c 1, let· i be the impulse due 10 011 ilnpulxi vc\ l\1rcc F w~ich n~ts ror n time
Our When D, ,i· = 0 T. If 1,1is 1 the finite force octin.!1 shnultnncously •on the hody, th~n
t ==
c\ = 0. : . t '/'

M' .1: =. (M. + M') g ~in <l , t. .


; ; ni(v 2 -v 1 J=f O Fd1+f 0 fdr=,l-~fT
J111.:g1·,t1i11i; :i!l~i11, we .\\fl M'j = M. + M' ,: sin CJ.,½-, 2 +,• ,, T-, Oas T-. .0 .-. I" r11 Cv2 - 1·1,•
2 sli\iw·s that the finite force/ ucIing on .1.l.16 body rnay he ncgl~ctccJ
when / = 0, ,1· = 0. ,', C, ::i 0, the c4l!Qtions .
M' ,1' = (M + M'J g sin a . ¾i2. .· . . Eqtlu!i~ns of Motion unde,· Jmpulx'lvc Force.~.

.
rn·
. t"'
✓{ -
.(M ,.,M')
+
2 M' .1·
·g~na
.'
} ••
.
. .
'
11,11am(11e lhe _genaul rq1w1io11s of 11101iu11 of 11 syi·,~111 11/.'ll'tl 011 b,1·
of impulsei at a 1i111e. ·· [Mccrut·TDC 96
v, wand u'. v'. fv', be tile 1·clm:ilit:, [lllriilkl l<l the:, ll1'~, r,:xpcc1ivcly
Putting ,t =AR .. a, il;u ii111c lo rc:icli th.: 01hcr, nnd B o'r ,11c r,l:mk is hy
; and after the nction· of impulsive forces on lhc rrnrt1<:lc

✓{(M-,.f~·/~rn.a(
X'. Y', Z', arc':the resolved pnns of the totnl ilnpuJ,c on "'
,' = .~xcs, thM "'
►·•."' 't

~ 2.9 lmpuls~ of n Force, l: n, (u' = u} = l: f 0 Xdt,,, !:X'


• Tlw i1,1pillse 11f ,1 /'urn a(1111g 011 a part/de /11 1.111y- interval of lime is l: mv' - I: nw = I:X' ...(I)
to be the 1.•l11111ge 111 ·111Q111e11t11m produced. : · 1 , i:: niv' - ! niv = l'.:Y' ... (2j
Tiius due lo n·forcc F, 1r.1hc. vQ(oci\y of o p;rticle or mass m chtingcs ! mw' - I: mw = i:Z' . ' ... (:l)
from v., lo V2 in time /, then 'the impul~c / fs fivcn bf l ; the change itt momentum parallel tu an>• of the w:cs Is eq11C1I to 1/,a
/,.11,v2 -'111~1 =m(v2-v 1) . ·, • ' ·
t<1/a/ impulse of th!_ external forces parallel in 1/ie currcspo11d,'1(~ axis:

.. m J i:Jv.,
f 11 •. 12 •• _-dv
-/71-d dt ·
.
i : ,:HMce the change in 17JOmcntum pa~al/el 10 any of the ax.e1 ll/.ttic whole
nias;, M, supposed collec1ed al the centre of inertia and moving with it, 1,
· r I t
I 1 e~ual itO 1/ie Impulse of the 'external force parallel to tlie·r.nrrcspond/118 w:ls.
1 .Agai~ we have the equation · ·
i · . dv
JI .F,dt since. F-=m-dI i; ·
I ' ,
• ~m(.yi:·-zj,')=I:m(>•Z.-zY)
d
I . , .
T//11.1 the l111p11fa·~ of tlie Jrrte
F °is the ~rl' '
, . ,i dt ! m
·- · (Yk - t>·) = r (yZ - m
of the force,
Now let the force F.J11crcci.~e indefinitely and the lnl~g.raling this, WC have
10
, . .

n very smal) qunntity such ·,hot the time integral


I.

.•.
, (12 - t 1) decrease
f F di rcmain's 1inite.
l1
2
! '· '
6'(-:-
,I
, Zd1'~ e f0.' Yd1Jl
'
· Such n fore<! Is -cnlled impulsive force. ~ince the time interval 1 is, so that the body hos not I1101•cd during
Note, The impulsive force can be me!ISured.·by lhe change in momentum 1lliis Jnterval, we may take t_-y, 4, hs cormnnt~. 111us the nllovc ,:quation
produced. · ·bCi:omcs · ·
I .
l:111 {y(1✓ -w)-t (v'.-v)) =·t(yZ' -ty')
'·· :!:m ()>w' - ,!V'.) - I: ~1 (.)'"'..: t~l = l: (yZ' - 1. Y') ... (4)

,1
:I

!!'~~~?;i~~.t,·1~t{t~W•I11~B~rl1~m8frffitl!Uttt,m4;,,:'1'.t,~~::·;~:'Ht·t·'.ff;~f•::;1:i1'?t~!rn<tU~am~t~t:~,}!M~~aHtliffM~H~ffi!tffillt~fflffffi~;wn~1s>:·11··:>.rr~!tt:t'l ·n.: ,;,,"n..


, H>t kfs-('J> ··~t" •~l'.;·r~.;1;,-,, ",",, ... ;.,:-·•·.
·.:i;;;1 ,;,;·;,;,:,·<•.'gt,;H,,t~tttl!1Uai6•,H1,~,a,.:\1;,••·•~,, •.. .-.:1•,,•,",.,-, . • "···· .... ·~-- ··-·

'·, i D'. A,l~mbett 1{J>·ri(1cip'//F' .... ···• 169


l.: ,;, (.n·' •~ ,1·u·; .. ::: ,11 (.rv ,. y11)
r:
II/ (r.u' .- .m') •. r.
/I/ (w -·,\'(I,))"' i: (:X'.:. ~·Z').
=t.(xY' - . I
I
-1;f.tr ~
Ex. 11, CII//IIOnof 1/1(1,S M, 'rcsti11g 011 a ,0;1:gli horiw111a/ ,pfctM of'
i
l/1•11ce !ii(' cl1m1g~ i11 1/u: 1111m1e11i of 1110111~11111111 a/1,<ml all)' of the. I .cy;vcd'flffici1m1 offric1io11 µ,Js /iMd \\'/1/1 s11ch 11 c/wrga 1ha1 tile' relmiw w,/ocil)'
at
1s ~qm1i. 10 1/i~ 1J1orne111 alw111 1ha1 1Jxir of 1/1~ i111p11/ses of:1/tq
forces.
·vector method, Let l onu I' ~c the rc,ul1~11t ex1crnnl Md in!Ornnl
1I of; ,1;~ b,11/- aud ca1111011 the mome111 whc11 it, leaves rhe ca1111011 is 11,
Sliow 1/101 ihe cannon ·1vil1 recoil a •disram:a
1

·,)n the r,~r11clc or


inn,s i11 ~i P. Al$o lci'-thc velocity of' 111 ( 11111 )z I
l'nim v 1 10 ,·, then. : . . . · , . . iii+ /1)
: 2µg'
a/011g the plane, m being 1he nloass df the· ball. .
Impulses = chnnsc in momentum [Meerut 1:0:c:,!)~(R)J . i
l:.. I'= 111 (v, - v1l Sol. Let / be the impulse between l~e cannon nnd ·the·bal\,.:lf v.•!s th~
or :!: l ;, :i:: I' ~ :E-111v 2 - .l:iHv 1 velocity o.( the bnH .ind V the velocity of -connon In .epROS!te ,.dl~cction.,
Flul r.I' = 0. by Newton's third l~w
:, we gel, t I= t mv 2 ~· !.111v1
· i, ·"· 1he 1010/ cxremal impulse applied 10 the sys1em of
... , then the relative velocity of the ball an·d cannon at the rnonicnl'l:he b~\l
leaves the 1nnnon. is
. v+V=u (given)
. ·
/
, ·

i~ ·tI Al~o since, imp~jsi; ::.i change in .mom~.nlum ·


cq11a/ 10 Ilic clumge of linear nl0/l/e11111111 produced. (for lh(r,ball)t
· · .,.· /:am (v-0)
Now le, OP= r: then from (I), we get arid

/=M.(V-0) • (fer (he-cannon)
l:r X (I;, I')= tr X nl (vz - Vo mv=MV or v=!!f::!.. ,
111
... (~)
ur !:r X Ii., I, r X 111v 2 - !. r X mv 1 ·• (Since tr x e
fiom (2), in (1 ), we gel
i. e. Iha. IOIIJ/ vq,•/or s1i111 of the 1110111enis of Iha dtema/.'i'1111111/ses MV .~. ..
WI)' (!oi11T O 1s uqual 11> 1/Ju· i11crca.re /11 1hc ang11/ar 111on1en1un1 ·-;;;-;, V= u or V (M + m)·=-mu·
al,ow 1he :,a111a (!oi111,
. ; .. (3).
or V=mul(M+m)
EXAMPLE~ If t~e c~nn/ln moY~S:lhrou.s.~,~.distance .. j•

E,:, l O, T,vo perso11s (ll'e situa1ed 011 a pcrfcct/1• smoolh horizon la/ plane . x in the direction .opposite to:iho::d,ircc(ion

;.·:.;_+
al a di.,!am:e /! from each (!f/ter. Ona of 1/te /!USOIIJ, of //1,<JSS M tliro,wi, a · of m.otion of Lhe qall in time 1, then on the
LJ(l/1 of mau n1 reaches ltim ln 1.ime r,' prove 1i/a1
1cwards the .otlt~rwhich .J. r~u~h p'.~ne, for the cannon the equation of fl.
II!& j1r,i pcrso11 will begin 10 slide alo11g
Iha pla~e wirh v.~./ocit)' nu;JJ'{M°t), motion 1s .. . µA.'
·' Sol, Let / be the impul.~e bctwi:\cn the b.11 an\J the flrs1·'j:,crson. If the Mx =-µR=-µMg , .
n.rsi r•irnun 1hrnwi.a ball with the ve.focily ·u on~ begins to slide along
r,lonc ·~:111, ,vclochy Y, .
then since, 1rnpul~c = change in momcnlum
. .
·or· x' =-µg
Multiplying boih sides by
get ·
and Integrating,
·
2x ~
.

.', I= .M (li.·,.O)
.f
(for the first x1=-2µgx+ C
and I= m (u - (for when x=Oi-x= V (Starling, velocity cf \he.cannon)
:, mU..=Mv .•. c= v2 .
Sin~c lhc h~ll rcmihos the ~ccond person in lime (,
••• '" =i=~ut . ,l> . .·. .?.= v•-.2µgx ..
From ·(2). u = 11/1, Substituting in
I

we get
the cannori•coffies w rest . 0,
' 2 . '
.x=
,'
: .. Q~V-2µgx:
·=-
,, ma
Mt . ·x·---·~·
,:v~ :-:·(,.mu:· .) .1
2 ,.--....
or
':'" :lP,:$ 7· i\(ijfin ·' -2µg;'
which i, the feq\lired"dlsiah e?'''· : \.,'.
j
,
'
.
• • '
· 1'i
; I
. . ., I . . ' • ... •
• lI
,,,_.,,,,, ••••)'''f'>"'4:ttmt!!?\~••i~,.?:rt:~~:ri::;r:::-~•~::.~:n~~~~::~~~~~J:1fflffi!!'r.t:1~ij~,:~~r~~•;r:f"••. 1~~i"tlV,l'-'.~~;:•:~-11.~!_l~~(•~•::•:•:~,i:••;;f•r:,;':~r:m:rnft1~~:P.Hffi~~~.lfl'(JJ1~Jlm,t~~fl'•.-.•++utt,!t~.:.U\~ILI.A~~~:

:\'ti~~.~~;.;,, 1H·•:,\>:~ ,· :, ~;.-: •,:,:u': ' ::~:}'j~;'.--~~:, ,,;• ' .. ;·.;.; .:·::·::::~:::·. . ::-•."'·\~-:!'.:!:r~:··

·-------·•--:,~"\'"T~f~.-.~i,'~/!•'?,l;;;;"?

:,:;0~:::~::r:
·---- ' '
---~
'

·:,,.;,,.'
~.·
·, ',' .. '
ri
li
l:\;
,, ,. , "'• •• ,_,,.. • ••·• ....... ,..,,u,.._,._._,._•., ,, . , ........... t.'.,, , .. , .,;..,,•• ~l:,,;~:..!;U!J,,t.!.:l.•;i,I,:~::, ,::{!~;~!(,\'.,.:' ·._ ,1!41;::;; .
l!
•·• •• f• ,:.:~••--~•-"- ,...::.:,,,6_:.,_.:.:,,.\:.r.J1,, ll,.:,,::, ~
.:-.. oJ.!."1..'. •,:~, •• , ,;.: •• , , .... ,. , , • . . . - ~ •• " • "•"' , .. , •~• ,, ~•,. 11:
I
f;-
1·,' l-10
I,!111
~:.,i: '' Dy11qh1lcs of Rigid· Bo,dy
r,i
ij
&:~
tl MISCELLANEOUS BXAMPLES. ·
·6 . .· D' Alembert's Princi;>le I 11
. ; ; I.
tJ j 1/,in tlrr:11/ar: disc ofmrJss Mand ratlit1s a, can /urn fr~e/y • ; ; I ,
:1
1 a nut n thin axis OA, wMl;/1 ls puprmd/c,dar 10 it.r plamt and p'asses thro11gt1 o~ ¥~ ,1 1iin ll ':w· l:8111 LP. OL
I
! t1 i1oim O of ilS cirr:11iiljfrl/llce, 'T/·/C! ,:i,;/s· OA /.! ·compelfod io move in' a =;w2,1P,1. or the uls~ nht1ut OL nnd the li~rizont:1I fow lhrou~I\ O!
,I /iorlio111ai plane with a11g11/ar vc/o',·ity' C(I. abo1it la end 'A. Sl,ow rl,at 1/,e ='vi.~ ',(.RI, ul"th..: c.lis~ nbout_ the pnr::1Hcl lin~s. tllr9u~l1 C ..G. '(.''.+ i'.I. 111'
l11cl/11ati"o11
1 e 10 the verr/ca/. of the rad.ii1s of the'. disc th.ra11gh o ls wh9if rnns~ iw.u1 ,C.C. 'C' ubout the hori7.()llU\I nnJ vc1·ticul lii1es 1h1;ol1gll
cos·. ~i.ar,l),
lr,n/ess a}< (g/a) ·and itfrm, 9.is ~e•ra.,, ;,... 0); :, i '
·soi: Let·:.C'·b'o·:rne; centrc·or the lhin ' . '
:d ui2 ~O!+ M.CT,OT)_ = w2 Ma £In 8.a cos 0
circu_1a·r: disc.or' mass M. which qa_n iu~
abow.a "ihin· ·lionzontnl axis· ~A per~ or i .· : · sin 9 (!: - aw 2 cos 8) = O
·pcndjdi'fnr;tO)ts-plane llrtd passing through· which gives, either sin 9 = 0 i.e. 9 = 0,
a point• O of its ~ircumference. Whcn 1he or /g/.., ciw2 cos 8;,. 0, i:a:
cos 8 = g/aw 2 or' 9 = cos~ 1 (g/ (l(.,h
lxis OA' iurns ;horizont~lly round A, thc- lf ,w2~ (g/a), cos 8 > I, which is 'not . possible and hence in this cusc
disc. wl!! be•raised in ils own vertical.pfo.ne. 9 = 6 is, the only p.o,ssiblc value. .·
Let the· radius OC turn· through nn Qnglc • . Ex. 13. A 11li1.1 !teavy disc can rum /reel)• abo111 an axis ir i1.1· own
rJ to the venic~I in ~mo 1. f;lane, _and this a.;;l~.rrvo/v~; horlzomol(i• with 11nifon:1 angular ~:;:10,:ity a.
Consider an element ·of mass om of ro about a fixed pmm 011 Itself. Show rliat the 111c/111aI1011 0 vf 1he,..zJiw1,· !!i
. lhc disc ill. I'. .L'ct 'pN and Pl be I.he
perpendicular; frc-m P oi:i the venica!s . ·'
'
'
TMg oi the .disc 10 //1~/ver(icol .is give1i bJ• cos 0 =(ghf;k 2r.i) •~:here_./,:' ,:v the
di'.sf'a11ci'·r>f lhe centra of. inenia of the disc from .the a.ris wirl k is rh~
through, A nnd O ·respectively, Thon
obviously the tJ. PNL wlil be in the rJd'.'us: of g):ration of Ifie disc aboat the a.tis, If < rlilk 1. 11m,·e 111111 :lie, w'
plane of the disc is. verrical. ·
horizoi:itlll piano nnd Nl wltl be' pnrnllc1 1
• / _sb1. Let C .bi.: lhil. c ntre or a thin heavy disc ,or mas1 M which cun
ctibe 0 cf rel~ of rndlus PN wllll consinnt a~gu!or ve,loclly co about the· vcrticai
7
lurn. about an axis OX. in its own plune,.When· thc oin~ revolves· honwntolly
through A. Thus the reversed nffectlve··rorce on .the· ~l~menl Sm at P along : with O uniform nogula.r velocity (I) about a'fj;,;cd point O on il~elf, 11,e.disc
NP ls Bin ._/jP ,!!).2, . . .. . .. i i,rn; lµrn abo111 qx. Lei (he, lhe lncHnai!i,i. of the pl.ane or the, disc I_O tl1e

But NP= Nl.,-i, U, · , .o ' vertil:'al al time .1, If CB Is the perpcndicularJr6tn C 011 OX, then
NP•
.-: om (1)2 Smro2'fli,. on1 Ct?2 '£1,, . i Ii:.·; .' : CB='1(giveo) ., ' ' ' . : '·'
: ConsJder an element of mass Sn1 of th~ disc .at P. Let PN be ihe pcrpendict:lnrs
Thus the reve,tse'd cffoc1l~e 'force omfJi NP nli;,~g NP is equivalen\: 10 (roin: P on the veriical throu.gh' 0 n_nd PA perpendicular on OX. Lei Pl be
forces Smw~ NL 11lon·g NL and 8mw 2 LP along LP. The external ·forces on lhe perpendicular froin P on th_e ·
the disc nrc lls weight Mg nct1ng vcrti.cnly downwards at it centre C and :Vertical lhrough A, lh!ln obviously
the. renctlon at O. • · · · tlie t\ PNL will be ·in a hoti2.ontal
D-y, D' Alembert's •principl~.· rnversed effective forces and the external 'plan~ an(/ NL will be paraH·el to ~ 'B
forces kcl)p lhe ,system In equilibrium. . .. · ,qA,; Also N ,--,-\ 90""'·
·L PAL= e= L CBD, where CD is iij
To avoid reac'lion nt 0, we take the moment aboiit the axis OA.
1
·.Tiic forces·61iiw Nl nlong NL ncis pnrallel 10
'OA ·vo.n•ishes.
OA·, h~nco hs rn;mentlab.out , ; perpendicular from Con the .,lenicl'II
: through B.
:9o?·\ : .
~-;, !·
i
J !

,•,. Taking 'mo.men1· of all the rori::es about OA, we l\o.ve ' Now P will describe a circle of .-/
Mg.CT=!.omC() 2 ll'. OL+Q . rn.dius PM·' with constant Mgular I

velocity w nbout. the:·. verticnl +iv,g

l
lhrc;,ugh ;he fix.ed point O.

'

"tP.m~;~~r~~1tr&!J]frnt~~~S!lfJ~mnm~rm~~r:t·r~~~rn~~!~1rfi~~;mnmmHfli~onw.mHf'}H{f~~~1*i~1i~P.m~tm1,.t:t~H,nu1~tt~t>·;·?c7~;rr;,~-m.t:·:f?::; ~ •:' ; ;; ~ )- : . . '."'.;: f•i:: t: '. ': ~~'~ t';~ :·t~Jr. ~tm ! :::;~!~;!; )'M{i}'~J(-\,\ ~~(iRf?') 1H~)~ ~ (~~{t~ .~:-;,.I:.;;:•:) ,t}~'i:-~~:~9.n,H1?:{;" :-a< rt?a q-;, : ~ I ':-~~~) g,; !f· ;\' 'i'l!m1lfflfflfflifflfflffl ,:~, ~ti' f! Vin ;~-p
:,,> '.,:,1:~:rr;~;11H~t1:,}U!HHH'.~(,\~,\?!t<!~:•;rn,:r,;;.;;nt;n:'U"' ' •. ,.,,,, :lo<

~ i 2' '
'Dy11am(cs of Body \\
Thus the rever.~ed effeciive on ihc element Sm at P a_loni N.P_ is ~:
:,;
0/ll; /jf, ,;;

f3Ul FiJ:>·= [p, i


011,w' m.,"' NL+ 0111(1) 2
Thus ·1he reversed erfective fotce 011;002Nl along NP is
ihc i'circes 0/11W 2 :Nl alpng Nl .and 6111w2 LP
i:
on _the disc nicl_ its weight '"".g nqtlng ver1ic~Hr
.I iiI'
C :ind .the reachort at the ax1s·· O~; · . f
. By ·O' :Alembert's principle, reversed effective forces .. and the
forces keep the systeni in equilibrium. ..,; .
.•
f,,
,,;
To avoid reaction on the axis OX, take the''moment about lhe axi; OX., I ~i
/ !:
1l1e force b111w2 Nl along Nl is. parallel
OX vanish~s.
10 OX, hence its moment ab~ul
· .,,~~
,a_k,moment of aU forces about OX, we have ,.•
,.,.•
:,
· · Mg.OC='l6m,shP,Al+O · :~.
1'.

~
()r···: . Mgh sin p = ol' :t oni. AP sin 9 .·Al' cos 9.
.= w2 sin 0 cos 0 :E AP2 , om ~

~ <1:i· sin 8 cos e (M,l. of th_e disc abqut


"" w~ sin a cos a . Mi? ·
or sine - w2k2 co, 9):: 0, i1l
wh;•;h e
1;1ve~ either ~in 9 = 0, i.e. = 0, or gh - w2k2 cos 0 ;..~. :1
cos 0 = (ghlk"r.i}), ~~
.1
f:
Now when w2 < ghli?, cos 8 > I, which is not possib.le and ht;;nce u
en.~~ 0 ~ O is the only pos;;ible value, i.e. when.<,o2 <(gh/K'), the
li
th~. disc i~ vertical. ·~ .. t~i
~;;
EXERCISE ON CHAPTERrII , I ,:1
i;;
1. Stnte D' Akml:l<:.!1'$ principle ond op ply .i\ ·lo prove th•II ih~ ·mmi~M 1ronsl.1iob ond or :11
,~~f. ,
rotnt_ton of o rigid hoJy ,~n ho regarded o, indef<Cndr.111 dr"eaeh other.
[H<nL $cc ~ U ~nd § ;.&], · m
l. ,, i:sh1 iod 0,1,B c1111 tum fre,ly 1n o vcmc,I pllllle about u smooth fil,:,&
two heavy p011icl•s or moms m nn'd 111' !lie ,miched 10 the rod al,\ nnd S
ii. Fi11d th~ motion,
I .
I 3, A plank or ·mus M ', ond
~ .<mou1h vlaM ,nelined 01
2a i~ i,11tiolly ot rcM along~ line or sreo1es( s'1~pjor
rt lo the hotiion lllld n mQn, qf mess M sio,:\~g ifr9m,
·i:w.
end Vf)k> down pion.I<, so that It 'doc,1 not move, show 1h,11 ";' g~ts, 10 ~i:·.
end in •time, i~~ .
.. _ ~Ma
{ (M • M 'l ~ 11n «
}Ill 'I
• I

. i!: •
/\i ,
(!llnt, Sc~ !!x, 9 on page \051, ... , . l
I· i,
! I
\ I ,:.---;·
.
~
!/i,
;:' ,:,
\.:.
..........
I
::
I \
' ·1
:, • , , I
.-.~~\'
,':•~•',t•~
•i(~.:

,. .,. ,.. ''"'"c'.•!W,U!!!'.C'-a'i,:,•7::·c;•Ct'.:::),,,,_=cc"'~C:,~;,ii.,,.,,,.,,..,,,,,,':,,,.,,.,c,:•~•,.,J.,.;,..,....,..---:-""''_'.,...,.....,..,~.:,.. ~J


';••!·.«,:.,, ;.·,:\•.>,.•:
'.-; ·+:i~~i-i';.'/ :• ~,::~:.,~.,;•,>, ·;:.- .. ,
. ,.,,,,:· • -:~\$~! ii 1i :<' ,'

---------,====="'·""""·.~:xr?.n.·1\:,';i':1J:<!i:.,$h,17,~-!i!;:Y.
. · .. •·
s~! ,:'

I
.,':.·,,.•:, .

...

", .... ···•....... ~~;;,

8.,,.,,
r,1·.·.·· · . , .;'·
:
-. . ', ,: ](. J.M[·
·:_).' .:·s.'•,
li'
~ •; ;f .. I .
. . IIHS!11U1tQf'll,\1\i£ijA!ICAllC•t•m•
~ •• !' .. INSTITUTE FOR IASIIFoS ax.,-:;,',NAflOl\l' ';i.
r·•
' ~~t 8i~r~-1~~?g~g:
1

'('I
&
j • I .

!-. .-!1, ·1c'no!MAt1cs ·


(Equations otContinuity)
1.1. Dellnltlons an.d Basic Concepts
·r. l, Hydrodynamics I Hyd.rodj'llamics is ·.that branch c;f mathematics whfoh · •i
d'eals with the motl<10 offlulde or that or bodies in fluids,
I I 2. Flu Id : By fluid we mean a substa.nce which is c11pable offlowlng, Actual
I' fl.uids are divided into two categories : (i) !iq\uds, (Ii) gaaesi We regard liqu_ids a_s
incomptmible flu,lds for all practical purposes and gases as compresa!ble fluids.
I : · Actunl ~ulds hnvo live physic'eJ properties: 'de;\sity, volume, temperature, pressure

J' ,
ll-,

.'
~&UXD. and viscos.lty. . ·-~-
. 3, Shen.ring stress: Two tn,es or forces act OD a fluid eloment,.One oithem ls ·•
e .

bod:,, force Md the. o~her lij ,urface force. The .body force is proportionp.l to the mass
i '1JYNPXICS-.;%- of the body·on which it acts whHe the surface force act.I!
o,i the boui;idlllj' ,of the body il.nd so it is proportional to ('I
i t!ie surface area. . ·
! Suppose F is a ·surface force acting on an
el¢mentary s1.1.rface area _aS at the point P of surface
'\
S.. Let Fi arid F2 be resolved parts of Fin the directions
'
· of;tangent and nol1llal atP. The normaJ force per W1it
, 'i I aiea is called'riorm:al stress and is also calledpre~.sura.
I The tangentia.l forte per wtlt. area ls called shearing
' sttess. Hence-Fi'!s' a kind ofsbeari'ng stress nnd F 2 ls ..
a horinal 'stress. · Flit, 1 ·
· 1. Perfect Fluid : A fluid ls sall t-0 b~ perfei:t If it doae not exort .o.ny shearing:.
stress, however small. the. following l\aYe·t.he so.me meaning: 'p.orfeot, frictionless,
inviscous, nonviscous l!Jld !deal, ( ·
Frolll the definition of shea.rihg stress 111\d body forco It Is clear:;t}:iat body fore.a
per unit area at every point of surface of a perfccl fluid nets along the:n.orniol to th.o
surface at that point. ·
,\;..\
. o, Difference between Porf~ct fluid and Roal fluid : Actual fluid or roal
11uJd is viscous and compressible, Thama,in dHrerenco betwoen. rnl ll1,1Jd.and perfect
fluld is thi,t '11:r••• aero .. OJl)I plane aurihco of perfect fluid l~ alwhys normal to the
surface, whU~ie ls not true In case orrsal fluld, In caso ofviaoous flu'icl, both shearine:
str.ess and normal stress.ex,ist, 1. ,: . : .:·
6. Viscosity: Viscoslty is !Jiat property of;eal-fluld-as·li result of which they
o!Ter s'ome resistance to shearing, i.e., sliding movement of one particle past or nbr
. (11)

<;~::::•::~::•,' ·,·;:;~f::~:t· ·"'.0•~;:~;:r-t . .


-.;n~t• ''
..,.\':,'.'·
' '<\( ~: /t'.;, •· '' .:,: ~~ ;, ; '
. 1::;;:~•:,w:: ..··· · ... ;;:\:ii;:'";:,:·' ;~:::{;'-::•'/

----~•,..,.... ,. , ,,.,,,., ,,;r,,m,m;p,H,,,.tJi.:WJ.iiM.'ll>RvWn·S!,Hb.~•:l,•'.·•~.~❖-;,!1,\'.•:t;'.2;

': :-:::!,'.~•:::~:-:.:•
~.::l
:-:•·!·.'>

t
. ,, ____ ,_ _ _ .. ..__..,ta,,tr,;i'°~"'v"'"''""'"'.,i ....~;... ,.;..,_;_.-,,,.,,,, ,' ,.;.,, . .,.;:,.;;;,:~•.;...,..u~...,1,•~·~tM~~~........-:~1:;,:~.:.:..!.:,~•.:.,:;..~w.•!.,!t,~.:.t•,,, .... ,•,.-,, ... ,., ,,,,:~v,:\v<'1i, •• ··:• .,.,.,

. . 'i . . ' ··~ ''"·


i,t
j
2' · ~!-!~:R::1.1!'3~ \?~UAr10Ns 0F_c9,-i~1Nu1~ . • __ ·.:.............,... -· - .... : ., , ..., ...... · _. _' ,-...4L~ ·,
) another pnrtiol{v,ise~~;~y is als~ known aa Internal ... passes ,through ~hls point. Sln,ce_t~e .f~in.~ Is jixed nnd so x; yI fi>:.liill'!l,:~(l.@r;i:i,tl!\ni;
frlc.tion offlu.!d, 1,Jfknow'n'!luids have thls "property
1n varying degree, Visco~lty· of glycerine and oil is
i ·,.,.y.a,rj9p.l,e,s0Jis!lJlO(Ji9..~isrnibjl.~.if,,.)!,~ ••.;.·C,!'.;/liM¥.lli!jl,9 Wll;l!il\5li<r id,' nii1 lrp(~ , •'.'
· . f.ern:irk: ln Sulorlen melhoq w~ sludythe million of evel")lllul.dpJ!IU.clei'lta-UxeclAolnl:.wh~tea,s
largu ln coropnrison to viscosity ofwn~er or gases.·
' · 7,. Velocity: L...et a flui({ 1tarticle be at P·at, any o
; 'I i , • ,., •, !~I •,f.
MlatJtn"•thnrlwA.ttugvthampllAnOf9'(Stll!IU!rlnarl!CIG1il8Y"l).(nn\nt
·t'(:.'d.i ,J!i>f.l/7, r- .;;np -11- ~1 r.,nn:r1 .. 1;,-;.1.r1 1'71 ~ , : •. ~.1J1,.JH/1 ~ J':!,H!;~. 1.L ·1,nHrl''. J~.I
,: ule s me)f.od_ corresponos lo Loca1tlme.. rate o change ,(\d,, a,grang an.rn.e\h.oll
Hence

timo Is, OP"


rand rit ~ime t +ol, let it be at Q; where Cl;\ tr · · corresp'onds to lndivldu.~I time re.le of cha~ge. ... · · -·1 " • "·,' '··' ' 1 "", '· •
,. /t~, 1 M\' , 'if1, 1,
6Q = r + 6r. ' ,. ·
Ex. Explain
' ' -~ (..~.,.
the differed1:ne·lt1i~eii; E!iie,;liih ·1-~ri.4 "L'd'grangian meth'od.s in
1.,1i

'1·"' es~=.,®17 {5p J :r +or-r "or.: , hydrodynamics, , . ,,,,.,:,. 1 bo, fr.•.•,! .. ,,:(•·'··":
~.til.;1111~.~ t·hf.! . ·
. tus ~t seco,no~ pro~uc_e ,incremer.it- PQ =or in r. If St ➔ o, or..., 0, then 1.3. Local and lpdlvldual tima r~le of.ch,l'!nge ,. ,.:,, '. ... •·.. " . . • .l
-il~t~awaJil.~fdf!ll1d'~ doniedia¥J~i) tangen_t at p :o the curve.
.,., __ .,, ....__,,,,_..,.,....,...,,.._ ·. J-r•-:..""'8r-".~~-...--•~•-•-u·,....,,-"'""·,._ ..,,...,._,__ _,._,,,,...... -.,.
Consider a flu.id motion a§~oc-lii.e4•iitb\Scal~r'pii;l,iit function ,p (r. t), Keeping~
Wedcfino , •-·... Lt,.- ·, · !,he point p· {r) fixE:Q, the chan~e ~~,~ ls ~(-'. · ,,,.,- , ¢·\) • ~ '.i· · · .•
" dt Iii,_. O St .i!qo::in,:;;.) ~,(r.111~ \,)(11, l)tlC-l.!lnih,(1 , I".!' ,: . . , Hr;-t'.;+'s'lf,,i~·t~ ,~. '. .; ·.. r •".'.,:. : · 1111.::!/,, : ·. ,·:

•f:•:•n ...,,i,:i.!,'•·•~f''
.
d"1t•1~J·.1:l 1:,(~ l)l ;;si,·~11) >,l•rntff~\ 1 ~~)1,,,JrJ•,;L".r.t,•1,f·f. .l.
le ,0e r ,dt 3 '· e ns.,;i~rtxe, ,~r:t~-,~-.Pt...1,,,!18~T;) f,/~1Jli, i!} ."u}Hr.1'/,':' ·1d~ ,·u;·.,J 1.lh;H}
·, its rate of~hang~ is ❖•;•·.
., · Hin
c ;~1/' .; i•},:
+
(r, '/.>+ .6t):- flt\ t2
.,1.,. .,. ~'.F,tt"IJ,: "l'
"!1 .,1:(1:;-::i;;:• •11 tl!:t,·lf-r ·.1':'1fe,!rh'hn~
H :.,.~~,}~:. ~;•1,
Lfl,:.:n ·•;''i: .,::.',;,:',,,i} ,9,:•\~:fo.,, {i,t~·t-:•·~n1::H:t.t11~ ;;!1 .i
·.<,1 : hi';~;{'•} .~:
t .: · a1 :-i o : · /Jt ..
is. fu<ed .hence ¥i Is
i,,•,\i;l·tlr, ::·'f:'.: ;(i,1\1•!i
· ' 11 aJ1 Fli.linF-tow>'•n'crow·n1:l,Y-•§u''i'fifll'ii', 11 i.,.,;,1 •.,.. ,., '!,, ,,;: ·-:1:,,•i:1 ·,1,:;:., ..... ,,_.i .,,,,.,,:., Sin~e P (ri
1I . u a. t(!·
>rn'd''rti i:~··or"ilb'wi ·1.;1.<;1n~ ·lit~;,%1t.iu17bsu1f.raH 1s'· :is:~'afi':iiYtf''ii't'1ffli11~ t'a\rli\it!· t
• ,(1, • " I'
1
,.,., • Keeping ;Ii~· particle f\Xe~, cha.ryg~ ip• ii; \.. ,. · .

,•
. .

11
We al:~· ~~'~i: : :; '.;: (,; '~:;~;l~: ,i~:t'.~:i:):\:'~'i'~• ,;"l!':[Zi;l::::::,;}t:~r;;~;i;:;;;
:,,;;.,,,,)
..~"l'~'t".
d 'I'
en~ "'y. _n~~UJ 'V.~. 001. y •.,ar,en ,d
~•••h.ti_~gupl~ 9~t,llf.4,.~9r1111~l t~ctgr,P/,1:-°~;;P.?if.t:f.:.:hu:, \
1

. . ,11,fm,1,•i.!1£''1/:,,/l .. ; ,1lr•·,·.,_/,1•..<,.'f.k,,'.!. .id l,, •,1'1/1'')1;,11,,f,; ~,!' :L,


.i:ne•Eh.lJ ♦ace• .: 1;·11,t'i :~•.ij;dt:·~!~
,i,:i ,.,._ '.,:,,,,,,,\::,'
'.':Iii

.' ~a)~
: ·
· ·: ·;·•.,·,·.1l'\ :;,;h t$1(!-",tt§.r-; A,t·RO,.n!9,·.CfiifJ.u,;n,.,, :,, ,..,. ,,..,.i ,~,
ofcha"nge ls

··I'.-.·.: ·, 1;:}~.
· . ,
;.,♦; !:ii(~;"l~J.) !!.i 1·i-~m1J.c.:·/1)r•~·M~5+ §~,-~._"4':0t/14;': t~r~~t)·::ili! ·,.id1.;.-;•1:
:ri),1,/

IJ~~/!\,~v>. ·\.~{:; ~~i., .) . ~:)


·1 .:~:~: •-~-!ff~,r.i-1:; i:.:·:•.

f~llgd, i,~rf(u{d~ril,timi~ i'ate,fif c;h~{lge.'3. i,. o•.;


, '·'

.
.,:•;.
::/;J
i
~ . 1.~,.J'.)1'e:.11~.!d.ro..ot1Q.!f'may t:fH.tUdl~:d·,_b¥r~o,d!f1~r~nt,rn~1tio.ds.,,.. · :.· ;",1 ,,, ,:1
·, "'ib'·ethom 'M';•::,:i•·:
" Since ~" ~ (M1ro ~:(?i1,.>:1~d):.1
,,,'( ' ~· .. roe.,f.h Q,d, Y":.1_;('
(1).. -..,a~.;i11g10;n ,. .." }11'El u,Jl~r,uim '.,::' ' , ,,. !•.1' ..l ,:·, {11,', :;_!::

1/ Lng'rnngidn ritcthod ?'ln thb ,mct.hbd;,any!plli-t!iifo1 oft.the,fruid, Js1se\eetcd•·· ' ~L-.


at= ~-d ,£.td
ax'!+tij .Y+.~ z
and its 'moUoirii' 0lud~ed, H~nCC\WD1tl~term\ne-:tl'ie :liistory.:of,e;vb')•.'ilu,id,p:iir.Li ~le:,,,, b . 'd'mg b~. d,t,..' I .
.,DIVl ., ,, . ,,; ::, · . i; , , ·. .
Lch.fiuld pnrlitle bii"initlail7:'ll"l't~!pVm:t1(a'/"//i c'}f~eti.:tiHiplul(dr,ti•llliJ'"t\ M1'.
· . . . ~ Qt ·2:b ·c'.!f 1iff, -~ :.l:fr '.1& :· fil! . £1 £1 ~2! ·
..
t),\I same fluiJ:1.. r,,_,rttclribe at (xF_y,';°:i)t'{~•ll•'o~lo'&i!it_Ni~:l)J/¥.'~fffllhft:ib'tlS\!f:>f!";But>'
shice pnrtlples· w"hlch have l~!ti"1l.x ,different !)b,sitl,9ns ~CC)l}'l)' .~if;ete~~I\PPSlrJOn~ 1! a ::.a ' 74 a 'd +, .-d +, =>.U~ +u~ +wa +,
•d ~ ~.,.ff•~·-~~?\ ,J ~~\ \t.f .!-~. f~ ~f. ·: , ·vy . z vt i~:·
.• a{l11r,.t~i-,,;n~l;!on,.!e, al1DV>!~<l1 ,hOl\.W~l:\~ ~®;,.d/n,11\~ _qf,~li~. llll.\1}~!!1\,,.P,.fo'(xj!",.z)
• d~R~'ndi.OA(Cct.. ~1,_cr~.1ro.~WhY~ilJ11' )t11H~ :l.•,f! _<,·.lt~J{ lt,''ll, t:r,,:!rr;: •.!~i1·,JIM.flllt 'h'/•:.•tiftt! t·t:i·:J·~
[F~r . i!:,. q ... ui'>i-\,J + wi.:1;, t$4':J .m:a+ ~ &!. . ]
.dr:-:,d(.1h µ,m::•fa, JIIHH\'1:.,(~,\':,1,n ~~~ )<! ,£i(,-.,:J'Ji.:...'1~~c;1 :)'H.1, :,-'-~ ,,.,1, .: ,·1 ',1 :•,,,
~ .·Qt (("· a . a ,.
·
it'
/ _:. ,.Ii (a, b, c, t), :r-=/2 (a, b, c; e) •. ..h~.,!,\'.s,,('a',tt/1c1t)v,,.•~ .,,,.:,.1:i·,.,.,.,,.
i,:•:rn'tb;(r et":. ,: :~··rc·Jh'i'lr' 1·•1l·i!-i&il"1 ;1t•·::~i\J'·kbWiih1~lJ1 '"tJMelb,1::~ so
,
qr,
. d.t ~ o< + ~-~+J 3.:i + "aza )·$ ] • (i
• ~··•: ."('1,'l'tl":s rd,<d. i·,•.•i, -~ ·~•al'1•:I•·.,,, ;;;1•tt l'I. 'npltthf, (,...f:r,L•!">J(,,J'>''rt;t/ l.i.-'ti'.•·a? ,;
~•. k.
lftf.,,,<tJW. n .~t-.'11,>t .·.:-. t
_..,,
o;titl\, .. .,IL\i ,.,,I..,.;, 7.f>,':'\'h)'j, •. }:,.\ 1•:,}a· P::9,Y.l\lr,.,, /(,,. !l.fJ,i:;:1,:;;,:·y·•J•;,',••.: ,,., t·'t',I'· l:b1,.,,/ ')/•:,
tli a we 1can a~sume t at 11rs~ _a_o seco_n<1.,Qr11e1:-1,part1"" oori~at.Y~~. -,y,r., 1. _a,,., s, .t.:
~;l,..
\ t",-',. ... ,,..
,,., >t'h '"
...,.,, ,., :·,'l d:J(i....,..\;"''f·;~•~,1.•1•,1f•,1.11.'1•,~l,},:h
,•., ,.,., ..,.}'' ( •'' ' l - ..
,, • 1 ~:1u<:•·,t11!1;!1::1·•••1;
"

8 ·si f1oi;n. ortent ofn elerat!o f,a l\uid. article are~ " Z ":Yieri(""'.'. .,ll ..:h :,,.
.'ttt,J f ·'}';~ ,Bu i,;.,1P:..i"',":'ij51.:r1. i,. ~,n ~lHJ) .-U,\fl~ .i;.l.'.ll,',i~J,'1 J-:d:;<~:-~1.!:)Yr:i.•)rt1:'.,·rn11~n
1
I

1
2
,:.\
.\• ·t,':::::Xt:i'..,::;:::i;,~i:;~~-:;:.ri·:·.:,:· ::: ,::::::::::1~ ;·::ri :;-:.~~:·::·;:.::::: ._:iC~i-.:i: ,. ;11 ,,

:\t::!'.fi'.'.::·.:_:.i·_:;:'.; ·.~t;:.,~i:~1a·~ 5'.'.fl;it%:;:,;:i~:~};~i\;:,':tt:},·~:;:;i::,;:~:.::~i,,,:,::i.;:,j:


1 'Tli'is'ii'~li~ t'~l'.ttioll'~/:&/t1~n'•tl:Hi'tw~'tlm.;1i'a.ie•."'"'; ,,_,.,: ,., ,,i,Ji: •1::.;
N~li'!'~'imi_larly, for a ~ect.or i'unction,lt c11n b11 JtoJli~;,tnat1 ,,: ; .. ' ;·
•'

, . 2, Eulerian. method: In this method, any P,olnt !i"!ll,g,l!¼,!A,c-S!ll!!~~.q£o.~P,\~4~YJ,,


· ', a _l1i,1Id,ls,11ele.c.t~~,~:,,.d.-.~.EI-. <?P.MfY§:,t.lva, ~lj~~~;.lf· .~\"\~ !,tn..~;,Af.i~A'j ,!J,~i.~,,~~.i\~e J\uid ·:.! ·; :· "
'
. .dr ar '
-,-=:-+(q. "')f.
.. G ·• :.~_;'· ·0 . \\~: .,t;, :1t
.,,,. ' •,:. ,:,
'i .
1 \,-;q~' •t•. ,tt!N 1 ··, !• :. t1 t~,: ,.,,p1 1 '11, .:r n ''.t~i,:np-; ,.:."'.H'." fo .. ~,. 11;~1: .-·: ,~., ;1!". -:: i ~~;, •:H,1.{~.:;·t · ::·ti ,1~;:; ••,:;Y\i. 1
1
1
4~ •~
b~,t!.i:,1~~l1:-~•1 r ,{,}i:r~i:',1\1 1,~.!i~·;t:t•1J,1 r· ./1 ,11 r:·. L:
i;. . i
, I

l· \'
•~f"•h., .. "~--

:.,:: .

., .. ... , ....... .
,.
•• , •.• , ......,'t!!,_-' ,•:~~~:·-,··. ,,,••, . ·,~;- •••

: •:~:.,:;::~::::::;~;~;;_:/!::•~~:1:,1ti~<•:.1•,';,:o ,:t, i:.': > • '. :. ,T}. :d •: '. "i ',('

··-,,., ....,-.,..., '·,.


.....:;:;~ '' · FLUlO DYNAMICS Ii KINEMATICS (EQUATIONS OF CONTINUITY) 3
1..4. AceeleraUQn -.· ,. . .. . · ! Rotll,tlor,1el'lil~{!on ;Aflbid rootio~:i~ to be rota,tlo~~l ffW • curl q ~ 0 at ~·aid
· · · To e~plJitthe m~tliod of diife~iii:i'tiatiorl. foll:ilwint the fluid· ahd .to obmin nil av~ry time nnd• at every point. . . . . ' ,. ..
expr,esslon for a«eleration. ·. ' , . . . . 5.. :tr-rotational mfftion ,: A fluid motion !s said to be !rrotationo.l If
Consider.'~, scalar., function ·.$ (r, t) associ:ated with ·flu.id. motion, TI\en· , . ·:, ·W = ·~i q"' O'at every point and at every time.
$ (r, t) ::; ij) ·(x, )I, z, t)., . . ,· , t.6. Deli.nltlons ·01 some euJes .
' . ' .· ' ·£1· . ~ ·: E! .
a~= d:x + iJy dy, + cJz dz+, at d_t.
·
1. Streaixi line . . .. . (Agra 2004, 'I(anpur 20iJO)
A stream line _or line ofOow Is a curve: ~.t. the tangent at any point ff It, at any
Dividing by dt iiod t11J:ing instant ofthneicqincides w.ith the gimtlon of the rootioi, . ofth"o fluid'at that point,
. • •~ .. d .... d:z, .. It me~os that direction of tangent and direction of vel()'City a(e pa;rallel, i.e., q Is
.;:= di =u, ~=11, fli·=w, ~arallel to cir and so q x <tr= O, • ' • •
~ ~- £1 I .d.,: fb'. ck ' dr. r de . . r sin 'a dw . . ,
o;, u. +i!i·'
ay v +-~
: T'-1
· we obta,ln ·dt .. az tl! + at · ·! ~s . ~ 'ii:" v ,.w or :1i~ _l_.~ "'_..L.J._ £! ..
Ta.kine: ' q~_rd+vftwk, 41 ~ ' v.;,_
~t "iJt_+q·.
. · . , . . or . , r 1~i_:1_a ' dw I . aa · .
These are the requlred,d11Terontial equations orastrean'ihtlO, Stream Jines form
dpubly lolinite set at.any time t, Here
or 41 o +· q ' V ]',$, ·
dt " [ at · · · q:suihj+wk,''
.d ;,. , 2. Stream·tube: The st.ream lines dra->m though encl-\ point ofn closed curve
This.= dt'"at+q,V enclose a tubula:r s\.\.l'facc in. the fluid·whlch is co.lied stroam tube or tube of flow, A
tube of!low orinfm.itesimal cross section Is c'alled siraGm (Uamont. . •
The operntor .!i..dt is cnlled 'Differenl!ation {ollo~ing the fluid', 3. Pnt,b lines · , (Kanpur 3000)
·· {Agra 2004)
So°:"etimes we ~so write gtn J,laee of i, Acceleration
1
a ii defiued as total . A path li.ne is a curve which a particular flu.id particle describes during its
motfon. The differential equations of path lines a.re
derivative(MaterieJ der,ivath1e) ofq w.r.t: t..Then
. s§. r~+q
·a .. ] , a a. a .. ' a )
· ¥t = r,i + 11j + wk, i.e.,
a= dt" V q"(fl+ua;+va;-+.Ulaz q. dx _ s!,i: _ dz'-
dt - w.dt - u, dt, - v,
Equating the coefficients ofi,J, k from both sides, ·
a a a· Path lines form a triply !nfinlte set.
Cl!,. ( ai + U + V 'ay° + W & JU, 1
4, l)iffcrence between strellln llnes nnd path lines
8 ,J' 1a 'a , . \ The tangents to tho stream lines
give the directions of velocities oi!luid particles
' ot + U -ax + u -oy + W -i)z ,\ V'
C12 " ( -
' al;variaµs points at a given time, while taogent.e to the path llnos ~ve the.directions
of\velodties of a giveu fluid pe.rticfo at various times, Thnt Is to say, s~ream lines
a .a · a . a.,
a3,; ( ,+·u,::+vr,-1-w, 1w.
., ·; show how· each fluid particle is movj.ng at n 'given instnnt whereas'tho .path. lines
1
Ot oz _,.
Q;t V.)'
1
, l
sh1ow how a given Ouid particle is moving at ench instant. ln steady Oow, stream;·.
wh~re a 1; a 2 , a3 are ~ompone~ts ot the ~ccelerat:ion. along tli'e axls,; lines d,i not-vaiy. witp,tla,ie and coincig~ iwith path Jines, • · '"
1.5. Kinds of Moti~n..- . , ; Strenk lines: A str,eak Uno Is a lfriii on whlch lie nll lhoso fluid elctnents that
l, Stfeam llne (:t'amlni:lr) motion·:.A flufd motion is saJd to be stream·linc· 'nt'some.earlier !ns.tnnt passed through a particular polnt !n ep11co.- :., ,
motton if the trncks i:1£ a fluid particle form parts of regular cu yes, (Kanpur 2001) : A st'rcak Uno Is deflnod u the locus of different partlclcs pnulnii:-'tfrough fixed
~(j 2, Turbulont.m.otlon: A fluid.rriotioti !s said to be turbulent if the paths are ~~ '
widely_irregular, . ·· . : (Kanpur 2001) 1.7. Veloe,lty potential
i.t,l s. Steady motion 1 A fl1,1id motion is said to be steady if the i:oodition at
point in the fluid at any, th:110 remains' the same fer all time. That·is t~ say, a . Suppose q = u! + uj_+ wk is velocit:t at any point P (x,y, z). Also suppose the
~§ express/on i. d:x + u dy +:w dz ls an exact diJTerential, say -
motion is said to be steady if · • (Kanpur 2000)
. . ~;, £!!_' •, .· Then . ;; -: d~ =; uf: + vdy .

'
· · ~. , . ,, .... '· · · ~ . ,f
whete'p,p, q. denote dei{~(~y;_t;~l~t~.a~clo~~ti)jspcc~i~eli or .:. ( +~:dy+ ~ :i- tar/:1'.;'<i dx -hYa'y + w rfz
!-'.'cix .. :.....
·' ....
jll, ,,,.... ,.,,,,,-,,., ..,.
,_,/(.',\ ,')· , I ,• .

~
.,I,,
~l1
1
PH :.:h::•i::--,', ,'r,' .'..:1•~•.-;;.~~~;:~~~r::~!i:J-;~~~.~~/'\~~'i~~~)~~f''"'l-,~Jp ..7•~~!'''"'•~•~-;: :•f~ '1,''!~1;;:::~ 0 ♦•,\~~ ·..,:-::--\,'~~~-~,'!lo:~~½•"•Y.·
'~1iij •j,'<:, •••: ;, ..

·. ,·,',•(:-•:,:,','•"•'•·
~11;1 ,·:~:r'.::::···
-~. ,;.;:·): ,.,;»;:.::~~.-;. ,•,:-;:,:··::. .-·
~~~ ..~:;.:;;;j~•:;;-:
.:._______ .,,....,..,.,.,;..; .. v.--- -•-. ---·~-

'('.,,-,•::,
·.::'., ,:.::>.-: ::. (!.(';\~,;,... ;:, ,.,.-y,;.,,.,;,,.,. :,oi~,:"·'"""'''f"'£,;l:~c<sl'1,:..o:,,;,>1:'f.,,;,tl,i>i~ii<ii;i.ilii.,,u'<><,..,..,,,..".""'"""'""'~-"~J~.,i:1\;;,;.;i,,::,,,.,a:.,.:.;.;.;
' ·:·: ,,;-;, ,:,,:-.~-::.• ~;•:•~-~.; ::.:-;:·.,: i :.. ~ ,;,, .: . ,; '. (;'.;...
·· •·'' •''" I,',,, .,.,a,•!·t\'l':•:>{<<'.-'. •~'.,<.:•.,.•

, ,,,1,,, ,,\l.!.A,.'1&...1i,.<>-·n-.,.;-¥>U•'>\hH;,,,l/\'nL'~N':•,•l•s\1N!:ld"-,''-'••• •,,,~ .. • ,, , , 1 ,,·\•::;. : ,. M:$1;::•


1 ,.',,,1tt••:.,.,. ..,: .••~u.Ua: ..... , , ....._. ..•.,

.. it ..... ..... ,..........:..... ,-.. ~ . -........ ,:::'~>.·~:•:·l'.:_:_:L:~•.I~'-'..'.::~k~!.•t9~:~t./"l~~s 1 ·KINEMATICS 1eou/4T10Ne OF CONTINUl'M


i ' •, . '
. i j k
•'• .,:.ff<l'l9r.• :•; ,·, ·. 1 , : : ,', ;Jh$.:~:.,(~,:;Yll';t~:I),,-. ;-f n:,;•, {(JU'\ hi;t!t !, · ,t~n.t, ii.:"•'.' : !~:r,-1;) /1',}f,1})' 1
".''\"'•
~ ··§! . ~.,,·,,,."., ·,:, 'h;J,·, . ,...,,. .,, . ,,,. ·.-:1··......._ · ~l+rll+t;lt;, ~a tljia a;a
.'rhls ,. .;, •,,,;
• u=.- 8 , ., u.=-·., ,w,.,,,.,., , ,
1':11:Qyt,, \1il.,';1 ,-9z
,::.:'r->1 ,:Hl;: :tip/',;,, ::I
,1):-.,,,•,.
1l•H,i•'i.'H' lvt,;h)•.:.:,(l(J'!".11 ,d
•, ; ..
II/
ufi~~+J;:::1+,~·•~:.::,~.:·::,: ,:·,.
. .U IJ

Hence · •q'J.: ~.-:'.;;·,J '.i~'.•lli':J,,: '; ! Equnting the. coefficients of!,j, k·on botl, s!dea,
1

\r,~".':•-,.,._.,., ·,:' '••:.,\'~ ... , . ,;,\i( I or +J iiy + k oz }",-Vq,. ,>"'': ·•· 1 "':":··,;\ ,. ilw . av
C,'!' ( ~-OZ
)> TJ=
( au cw )'.. ·t;a< ( -
OZ -a:;
d.)' '
iJ1.1. )
·~, :.:, ':~ :,· · 'i .,. .. "!'' •~ • ~ 1::1;1, ·:·:: •! ·! · ,'t ~·~ 1:1' "''.Ii'~\.!;
!•:! •:~•n\;•·:.;; t}~':aij ·r· • ~ n \. :: 1:·,;,r, :•.. r'·, ·l 1- I

· ..;,, /,.,; lnt~gr,;itJon:;of,,-, .3'/' !'!,0.:deci,a,tes,;}),at·ll>. ~101-1i,/;1: (x;:;ii,- i:),U:w,1."1 ,, ..:;., ·:., . ,,.•: •. ,.; ! : i A fluid motion is said to. be irrotntlonal · if ~ = 0, rr = 0, ~ "'0 ··otherwise
,• ;,j ,;.l- I,,,-: ,h1n•:;1,,f! ;;:~· \ /,I/:#• ;,,: qq)J:i!hl!: t,1.tt, ~1--r'1.:1 1.11'/i.'.
0

F\, n1;:1:.- 1 ;",i::, ~Ii;{; :•,:,1,,', 1" :} 'rotational,


where / 1 (x,y, ii) ls a· const~nt o( lntegrat:on. T)us. q(lu,a,ti~,n ,.~l~o,~~clar~s _th,at : ,2, Vortex line: Vortex line is a curv.e suph thlrt tangent at each point of it at
4t::¢1(x,:,,,i),:.:~:,.!i'. 1:·'..·~ :di{"t·.\ ••~·:· .:,;., ·/1~,'·.i"~\: . ,,,,, ' ,any i'nstant ortito.e is tn the directi.on oivorticity v~ctor at that point. It means thot
q = - V~is the·r'equireih-el~i'ioriJ¾ere $iJ defined as veloc'itxpol~nlia[b'~ velocity
function. Th'~ ne,'ga'fi've'sigr{fo":the 'equat[im q" - 'v$ is a convention, lt ensures that . :w is parallel to dr. J'h!s ~ W x dr ~ 0 ~
D' 'eqµa\.ions of avortex line.
t t =·~ = I these are the differential ·
,i'f!R)Y, tnk.~.~:fro:m:.hlghe.H.~,·!~'i;V~r.ll:9h~.~~lei~i,ii: w,i\:, !, \:il'\ :.,p,.:,.l ,,; ~ . ' . . (Agra. 2001)
Theorelll 1, To show that surfaces exis~::,ylikb,.~P.t;stni~ H.!l.~S · 3, Vortex t.ube: The vortex lioes drawn through each point of.a closed curve
.I I
I
the velocity potential exists.. .~h,. ,. :•,. ,..,,,., ,: · · ,E:nc!ose a tubular.space .in the fluid' knqwn air vortex tube. A,.vortcx. tulie of
:inlio.itesini:il cross,sectioJ1. is called a vortex filament or siroply
I ~ ,. , ,. , lr:.oer ;_'l'_l:li,,,/li.~~i:tR.\la,!..e.m1.j1\l9,l)J; ,9(,s\rna.m..l!!:~S:l:r~ irix~&1l:>x,,,,, ..,!,'~ ; · · 4. Beltranlhllow: Afluid motion lualdto be .Beltra.nic flow Is .Paral.lel to·
•II .t\ :v.1!~ 'r,,; ')•.'!1.iJ ·,.\, ,,du,f nus•,·~ i.iJf,U :I1ib t1;i1 ··,1~lim{ 11;h,rlJJ.! i;
~:-:!!l1.1, ~i.ti,b,t
,Wi i.e., itq x W!.;o, In ihls case -q.·fa.called Beltr~ic _vecitor: ...
.B:11 .\\j ;\(II_ ~~:1~.n\•,. Pr,St7µ,lr .;r1:1tl·"lJ'" t•!~,::~'\ tmn f..i::,JJ11iliii ~~H •:,.~an 'j,J
T-ha· su.Haces which. cut (l) ort.hogonnlly are) g{ven.by, ,,_.,..,; -:ti,,:r. ·.1:. BOUJ'.ldary'surtlace '
;:,f·;fr~: u:.in!' ••,•·Jh 1>}·.,i~•fq~hi.t14-r~ddfWdt'4 X 1bft!f1 "'i··w·, I~ J.J,i :;n-;j , The contact betweeo tb,f.fluid and. the surface
1
' ' . ;Jt;: t~•l'.,n r, ··::11 ·r:; i:.1n~:tr.',i.•J::l!J J.i.,:l1rd•1•:-il)(\"1 •; will be'm,iintai,i'ed H the fluid· and s'url'ace have the .
·'!'he necessary and sufficient coodit£on for the,W{istence of(2} is . Isaµie 'Velocity al'ong the norm aJ _to tho aun:ace. Let.•
admit a solution of the t)IPe .. ,,. . ,.,r,:i ·· i~' ... i a,., ·'.·,',' .i . :I: be an a.rbltrlll'y pojnt. on the boundary ·su'rf,aco ·
' /(X,J11~ "C · .. : ::; t ,., (3) iF' (r, t,,.
Ow here the fluid velocity is q ana velocity
~
c buing constant oflnlegratioii.' '.:.· ,·., " \:. ,:,.' ., ;_.;, I 1
dfthe surface ls -q.- Since norm'al ci;mponen~. or
. 1 . ',;elocicy ofi'luid"" rioti:nal compooerit of the velciclt:y
The necessaey Afid sufficient condition•.fp,i.;t\h,e ,exi1>~~.nee:<ifi(?:) is,~ha~ ,.(( . of surface, · • '· · : Fig. 8
u·( il!L .:.vaui.~rl,&'/12!!!:;~ iau(J)\·[Jff-'.£!:t.!~~~J!ijI ,;. ,.,.;,,,,,:,1·::., i
., · ( J . This ==> q . n =. u , n ... (l) .
.-.Oln ::Ot'iii;,,, 1,' ~•:CXi;i ,0:1' •,i:l:,.pi\,-aY:r.1axi'.J.111,l:i·,J #.\:i'l~!;d~.' ,,.:·; ... 4
,•.,J:\,,i'rt:· .. ;,1r•''..: -.
' : Since VF' is normal to the su:face F (r, t) = 0, Hence~ and VF both are parallel
1

.,
':'.!"._;,,If -we'•sh'ow ·thati (4'J·iil' s,£i§Ci~<f W:h'o·Ji'ev~I' lvel6ti t)'·1io~el'itiil.l:existsfii1f.; 'w ne n
1
• ~l"\i1 i
.. <£!·'•-~-••.'•'•"'•£t'._-;~•: .. . i!l:t i 2!t.,1.h' •:t.•Pl'IJj'l ,•J(
; 'i'i.1-lf.! )):!::.i
liJ.l{ ' "H!:l'•I~)
. :'1 . . . . 1,;:
;;.>,11.J.:,:.·.); 1,u
··vec~r· N,ow 0) takos :the form , ·· .
r~.,
i;;

-:,~x··••!!.~ - .::..:,, •.. W,~ 'i'q~p-Pi~xe~u~t.wiv. q:,rs.. "1%"1 '· ::,:, .:... ;• ·,. '. .. .: :·
t. 1,1,)'\' · · q. VF,= u VF, · · ... (2)
t1'.•",J ': .r/ ;! ,:i:r71:., ,:}, , :·r.:~.-.~ .... :, .~b·,2~..:~, .:jl~'!'.''tr•\ ·:.~i:•lr:.:-i:r :);'a::_,t•w,;•:i.:2 '11''•/ •.. r:~":_-. . L~t P (r, I) move to a ·point Q (r + or, t + St) in time ot, Since Q also lies on
s
L l;t of(,j) = u ( -
·,1; ,,.; ... :,.:.. , .. ,.,: .... \i~ 0:t;
..n
+·&,),.ji'!J,(·"1~\4-/J~)-!tw(-.lf'~ ¥ ' ~ 1)
Rr :·•?~1~:11J
•-;S: 1,~ -Re:P-1:. ,.. ,:·.,,1~X.,?!: ;zy ,%; -J:"
,F (r, I)= 0, Hei:ice F (r + 6r,.t + ot) = 0, By Taylor'3 theorem, namely
' . • . 'N_ 21.:'
·(:I. u .,0, +'II ::Ori,,IM ,',O,o: ..O·=:~.H.{il•Of (,1) •.,, nu, .i ·, ··· :,., ,.. , , ......
!. . f(x +h,y :+k)=f(x,y,>t+(haX,tk ~
"-· )+ ......
,
~· 1 ►:,;},1H'!"rrcef(~)·is·sat,!s~edt·u: 1 ,.{:·•.n:,:t•'.:"•p_,:.•:·• f ·,,:(: "i. :;:.1H:;·,:.,,.1h ,,,;i •r:t1: =~:in.:-.:.
· ' 1,8, Some deflnftfons ' . . . i . ··:1•::,,
'We get F'~r~_t) +( 6r. VF+ 6t :)= 0.' Also F {r, t) .,_ 0.
1,' Vort.lolty Vector: If q ba th~·velocicy vector, tliii'At_w.jk~url"',:j·'<ll:,l<&t'i:ilis Hence 6r •· ="'
& ' aF° , aF . d:r. -·~ "'F
vr + 3/ = 0 o, al= - dr. ,,. = - u •. v ,
,'Oalhl<hvortlcit~ ,Vtloto;. :ll'He ;o:,ati!,emnficl/l.tla1L1111,-b, ,Mil-n ~'.!1hbi:nP§OD ;,.•R;4therford.
""'Ooi'dsteln etc, follow'--the:,alel'initfon••W!,,.10,nr-l:,q:.-w.Hetea1i' ·:a.iM1'off1l·Robei1:soo.;ck. For u ls the veioi:!ty of the.surface.
follow the de!lnition W .. ,½ curJ.lq,·Ir.w.eiwrltu.rW, .. \v.·.(I';, TJ, ,) Jh»ri W . CUT~·,j' gives Now(~) becomes · ·
. • 'J.'. \ l;\: q;, ::\· :•, ,. { ~"\\ t:? , •t: ';\, ;_~1\, . '•!• ~ i aF
q,VF'":-:ai .or
a . ] F=O
- .. [ aj+,q,r,7 dJi' =0
I .
or dt
f

., .,,.,,, .. ,....
. ~ ., .~~. ,.
.,,,,. ~\·· ,·,,•,•!;•~\t;,:,\/_.;·'.t, '.·~,-'.·'•''
·;.:.·.,:,.<•:,.::,
·•••••• n•,,Hl•·•·•·"•''

,. . ... '" -~~, '1:-


11
. ,, '··~:< ·FLUID DYNAMICS KINEMATICS (EOUATION~ OF CONTINl,/lTY) ·U
or ~q~ivalently ···(
.
f •~ f + v -a: + ~ -¥-. JF:= o.
<1/
1,1
vX
0
:)', C' <1Z, .
£Q
ut '
+ V ; (pq}" 0. ... (3~

.This ls the required ci6ndjtidn 1or the si.,rtace t<> b·~ a posslble form ofboundary · This is Eulerian ,equation of continuity,
surface, J[the surface is a rigid surfa,c~, then the .condition becoi;nes
· 'aF aP oF
u ax .;,v,<1.Y,+ w oz= o,
·
6
a
,a~.+ q. vp + pv. q .. o
.
,,
Remark: ,Normal componenl orvqlocl:y ror the boundary
!. or [fi + q. 'r/ j p + p'v. q = 0'
. VF ~F1a1 , , ,aF "' (4)
·cq:n"q,jVF)•-IVF) ·,,.·Tt•'l,VF~O , ~+P,V, qmO,
or
'· i)F/ol i1. aF ,,
• - , - : : ; - - - ~ \.v.hore Fx~aotc, . . This ls an alternate form or(:3)', .• .
(r;;+fr,+P.i)"•· X' I : ' ' [Equol:ion (3) !s Ill.so cn,11ed .equation oJ moss of conservutlonl,
1.1 o, equation of c:on\lnuUy ·: , ; 1
.1 ' . . , . d
. The.rate given volume m,ist ~~ balanced by~
or ,zen·eratlon'of mass wlthli:i 11 , Deductions: (l) 10 pr1vo dt (log Pl + V',q " 0, /
1
equa:nct 01,:twar,dlloiv of mas;; from the volume, This ~ount.s;toJa:~ng thatmaUer ·
ls ne1t.her created n11r destroyed. · . , ,· , DMding(4l by P. and writing
. 1 dn d ·.
1.11'. Equa.tlc>n•of Continuity by Euler's m~thod · : · -p ==- (log Pl,
dt dt . ,
Or.. Determine equation· of co'ntinuity by vector approach for, a hon•homqgcneous
incomp,res~ible fluid, . , ... ·., (Kanpur'Iioqs, Meer-rlt2003i we get the required result, ,' .
,,. Consfder,a f1Xed surfac·e S, cnclosiDg.a ,tolulli'e Vin the region occupied by a · (Ii) To write cartesian 'forot of the eq~ation or continuity, Wo knol"
moving fluid, Let n be a unit outward n1mnal v~tor drawn on the surface element d·a · a o a a
dS, wh'ere fluid velocity ls q and fluid .density fs p. Inwar~ n·otm.al velocity is di = -gj + q,, V ;=at+ u a; ~ v ~ + w ~ •
- 11.q.lMass of the !}uid entering across the sunace S in unit 'time if. . N9w;(4),is red.uccd to
· (. rt"•"~+
a a · v a;a.... !'!'aaz ) P + p ( ax+ av .aw
au a;+ .,.. ) ..· o.
Jp(-n.~)~S=-f11.pqdS=-rJv v.;~~;dV
S . · ·"S··
"' {1)
This is'the cartosian fornl,
The ~ass ~rthe fluid within . .the volume Vis 5/ dV,
(iil) Suppose the fluid Is lncomprossible so that
d ,
0

;/}- 0, Then t4) => pV, q :- 0 .,. V,q a 0


Rate of 11enerat!on of the fluid with'ln the volume is i I ., 0. , ,.. ~ '+ £!:, + f!£.
l . axay~z
, tif p~~~J if dv, ·, " ; ! : .,. (2) This is the equation or continuity In this case.
, V , , V I : Note dn this case q is solenoidal vector. For a vector r is snid to be solenol'l!al
IFor pat av) .
a (d..V) =p a (·Ti, = p. 0 = 0, as voIllme .is constan: t ~·~·t·
. . t'1me1., Here ' vector if V,f = 0, '
; (iv) Let the motion be irrot~tipnal and incompressiblo: Then there exists
local time tate of change has been taken because the surfac,c is stationary, Equation
velocity potenUaJ $ s.t, q " - V~. ·
of continuity gives · • · · ; '. l . I rt£ .
Here also dt = 0, 'Now (4) becomes
~dV= -{. 'v, (pq) du !on'eq~ating (JJ to (2)]
0 + pV, ( ~ Vq,) = 0 or v'2$ "0
- ~ . 02,., 02...
or ·+:::..:,:,+ ::..;r, = 0,
or ~ + v , (pq.) }fo = 0 ax 2 ,: ey 2 cz 2
• , .
TJ,is is the equati6h of continuity this in c,~se, .
Since S ls arbitrary and so Vis arbitrary. Hence integrand o(the last int~gral Note': This dedu~tion .!an also be expressed ,;1~ : S4a,U/ thc.t the equc.lion of
vani.~hes, · · · continuity reduces"lo [;ap'ldce'.;, ~ftiatioii wheii".l~.e. l/i;,u,icf,fs incompressible and t.. "~::
I · iriotationa/, · · '.Jf)c;::•·'°~ ...,,::. ·· ' "•.::~,, ' ·. · '" ·" · " · · ~.,.,.
\

,· •,••,•J1'•'• ,·?:··::,
',,·.·,:.·
,'';,;.':t:~:"•
,·:-=::·;,
.. ,:t,:1-·:; ·:;•::{·..-'.:'.
,;::M?•ili•: :.:I:: i: : ~

·. :·:-:;•::-:·::. :.:,\·- -
< ,o . " ' , \ . . ' , \ ' :,;M~: ;, ·,:-::::,

!i;'~"'-"""'"7"~·-""·"---~--........ .....-..... .:•••'Ill'' '< ...... ,.,,.,,,,,,,,.❖,,,,,•..,,...euu,,xwr-i.o.,:;;.; ;,,::, ._, •.. ::ll., "'' ,\~.;..,. ,~. . .,. .
Q ..
1.. ,·--../l<l'h~~'"'""-....:..~u.il:l:iHi!.!;'C.:.H. ~, ••'1;·, ~,.~,.~ !"'' 1:',' '•,.~••.:..t.":,!.l'.t:1,••-. ,.,.;~ ... ,-H...... t .... w.irlli""Y'.'.'1.• 1•,l'r'\Hlnhi11,,,.,

~ '
!'• jl 1 ~INEMATICS (e'OUATIONS, OF CONTINUl'l"Y)_

........... ~;) .,I J._ ' ' • ' . f'LUIO YNAMICS


Also x =x (a,-b., c; t),· y =y (a, b, c, t), z = z (a, b, c, t)
', .''\:
M Suppose.the motion !~'symmetrical: au a ( dt
aa"'aa d \ ax ) . · .
d.x ) ~di. 0:1 d ( ili: )
;j;; . S1m1lnrly,;;-J"'d7 oa etc,
~ In this easo velocity has tnly one compone'nt, say u,
-~ · · d a· a · · ·· a
i
1. . , dJ

Then we have 'Ji .. '§i',+ u a;;_as q "u, V" a,;. Firstly, we shall determ-lne_Ji,
i , Now (4) becomes '' .k .k: il:i:
(
!
J .
a ·a ) · iJu..
( at + u, a; p + ~ ~ "' 0,
a , . . ~
o(a,o,c)". ca. El.ob
\ aa ob
1~ ~
~
i:10
(vi) For steady motion I _In this case 71.= o. No;,, equ~tion (3) ~omes az 'cz az..
::
1l I.I. '?;.;,(f?q),. O,
~- ~ . ~
. .
aa·; ·.ob,, ac·
;Ju· ou au ' :;( 'ax ' ax ' ax ?,; ax c!x
} t or equlyalently, . ox -I' oy t .:az • "O. a; .Ft "§; · aa° ilb ac' al ob, ~
dt - ~
!Y..-\
This ls Euler's equation of continuity- for steady· motion, ; El. · ~ + ilu '•Z!; · $!: I + .!bi. Z¥. ~
1 a . . ,• . im · ·;i1, ·ac ·<Ja , ob.:, clc. <'l ob ac
:~
i
-I~ Problem, Wrlte full fqrm for the operaffr usid for differentiation following the ,fluid oz. oz ' iii
~
'cit . ·&z ' in
aa · ob •if
'i;UI aw ow
·~
\.
~·~-
i
I
motion and give equation of continuity,
Solution :
a' 11 · a ,a a
di" ae•+ u,a°x +v. ~ +w iii, .
.· ' (Agra .2004)

·1 br:
. dJ ., .
.<·.. -=Ji +J2.+J3
· · • dt
'
, i)a i)b

,.. (l), ~ay


,,?a Tb 3c

j I
!
1: =operator of differentl.ation following o·uid; motion,, ·1;
Now J 1 e~pressible as
. ?E. £1'.. ~
Equatioo ofcontinuityis
?,a ca ~a
+p:V,q"O OU £1'.. Sz'
Ji': \ ab ob
. .1',12. Equation of continuity by Lagrange's method
Lot initially a fluid particlo b'e: (o, a~ Urae t ~ 10 , when its volume is at o, c) iiu
ac
i!;l
ilo
t
.
<lo
: r·
dV0 and· don•lt.l' ls Pd, Aft.or ii 19:·p~e' ,bf time°!, let the ·sume fluid particle' be at £1;1; ex+ au b. +'£1;1; az £1.. ~
.(x,y, •)when !tij \!olum<! ls dVand densityp,Sin·ce the i:nnss oflluid:elemont remains aaaa i;lycla i).;:oa ·aa cla
lnvnriant during its motion, H.enco ·. . au i!x au El. au dZ El. . az
Po dV0 .,. pd\r . or da db·dc = p ax dy dz . = I ilb ilb + i)y fib + ilz ilb ab ob
~ ox au ou au oz 2l £!
or Po da db de = p ~ ((x, Yb, z)) da db de ill: ac + ily fc + ilz ac ~c ac
· ' o tlt , C , ,
. . · h c(.x,v,z) ' ., OU £J&xr..!l + ~ ~ ~
or' p;/=~o ... (1) were J".i!(a,b,c) ox o(a, b, c) i!y o(a, b, c) o{a, b, c)
is the reguired' equation of contlnu!ty. ' , au au au, ' ' 'i!u . '
(l} · · ·
=J ,.+ O,+,.-;;- • 0 =.J -a
;i., •
Remn~k: T~is :111c1e can al~o be expressed as: By considering the co~tancyol mass of a
· · lin/111 volume of the liU!d, obtain the equation of continuity.
1.13. Equivalence between·Eulerlan'and Lagrangian forms ol equations
of continuity
:.-. J1 = J: : - ~· I•. •' X
!For a qete;mi~ant :anishes if any two ofits c~lum.ns are i~entical)
(Meeru/ 1993, Garllwal. 2001)
Let initially a Ouid particle ·be at (a,·b, at time t = t0; ~·hen its volume 'is c) Similarly, J2=Jt.'. :J3=Jt:,
· ow ) ~ J V. q
a;o~. .:diz.
dV0 and density i_s p0. A/'ter a lapse ·or.time I, let the s~me fl~d pl>J'ticl~ be at
· (x,y, z} w-hen its v~lume is iiV a.rid deosit;;,.-isp. The velocity ooi:npbnents·in the two
systema iu-e connected by the equations: : ·: ,:· . .· i ·
J
i;
. dJ ( Bu
(l) be~~mes di"' J i)x
,.,.
+.
' _,_..

.,•'• u 0 X,~.i.Yttf•Z,q=:-f:li+vj-1-wk,, · ~

•.----------- I ""•,,,_ "• '


",,' <.•;l _;-~, '·

, ,; i•'.1h,;,,;.:~i',,;;,:i$~,1\',:';,~J;~~,t':i~t•~m;~w-aut~>'<1 ;; t~_•: o•fl->" 1. ,r,-• .·

"'•'''"'"N•Sl
I i.

: 1Flu10 0YNAM1cs
#'
!l[ ~INEMATICS (E<lUATIONS OF·C:O~TiNUITY) ,

t\:JV,q . "' {2) Using ·1~e fact !J1~~·tne rne ~l.~~ent In cartesian CO•Ordinntcs Is giv'en' by.
, Step 1,: LagTangiB.!) equation or.eontln\l.ity 1- ,1 ds2 =d¥. 2 +dy2+d.z2,weget • · ·

I!
:,
~
~ pJ"' Po
, d
il7t7.+
a.
= di (pJ)'~ 0
, .,
d

p J V.q c10, by,(2)


= 7ff J + P dJ
dt = 0
co•ori:llnates
2
ds "' (h1 da 1)~ + (h 2 da 2)2 + (h 3 da 3) 2•
1.1 B. Equ.mtlon·of ·continuity In generalised orthogonal c:uAni'lnear
, , ,
I
c.,,-------,,c•
, •. I Let'p bo th~·fluid density at a cw-vlu~ear t.t--1------r,
!f# + p y,1q point ~ (a 1, a 2 , o.~) enclosed , by a small
, Diviliing by J,
= .Eulerian eq1,1atlon ofcontinu5iy,
s o.
paral!elop.!ped wlth edg~s of lengths h 1 da 1, •
·1<. c.o
,, ,. h3 '"'-3•
.i.. L et 9 , 92, 9 .b e "· J ·t· ·
w,e ve oc, y
.
q3
.' 1

q1 .,
M OA. 1, 1 0 ,
OB •h2da2 •
OC • hi
. dQ3
1
I
2 2 1 3 I
Step n,\E.ulerian equation orcontin!J.ity
components along pA, OB, OC respectively. , ql o A
=. sfa · ·.'"
dt +pV.q-O
!i.Q'
=>. dt +.p J dt -O,b.y(Z)
lrfd._ . Mass, or the fluid that p11sses in unit time 8 -----,...-~
81
' ' ' a~ross the face OBLC · · · r(: 4
E!.Q dJ d .
i: \. l
11
= { di+ p.dt = O = dt (pJ) = o,. · "density, area. normal velocity
·1 ~ p(h 2 da 2 • lt3 daj). q1', .,
integr11ting we get = Po, say, e c p q 1 h,i'h 3 da 2 da 3 .
. =. Lag:r,mgfan equation oi contwuity. I . . ,of(a 1, d;; a3), say, ' · · ·
, 1.1•7\ ·Generalised Orthogonal curvilinear co-~rdfnat~s ·
Ma~s of the fluid that passes io .unit time across the face •
.. Suppose : ( 1 (.r'; Y., z) ':' a1.f2 (x,y, z)."' a 2Ja'(x,y, z) = aJ, ·a,re·; •!1:Jie .three
C'MB'A :f(aj +&:z1!a2;a3)
· lqdcpcnde:o,t ort,hogonal rru:oilies or surfaces, where. (x, y, z) , !IT~ cartesian
1 co-ordinates of a point:. the surfaces a 1 = eonst.; •a2 ·., · const..- a3 = c:orlst,1]form an
QT'thogona!'system lo. which (a1, a2, a:i) may 9e use,d as the artha,on:a\ curvilinear
"'f(a1, a2, <l3) + oal. Ji.,
~l , . . .
co•ord/nates of a point in the: apace, The relation between th~ two co•ordinates Now the l\l(cess of flow in ov;rilow OU( from the fzt~es OBLC .wd ME'AO' in \/,nit
(x, y,'z) a:nd (a 1, a 2, a 3) Cll.!} also ~e expr-ess.ed by the r~lations : ' t~e ' ·
' ..El
.r =x (a 1, a 2, '¾),, y =Y (a 1, a 2,a 3),, a= z (a 1,,a 2, n 3), =r-(r+oa;~ ila1)
' ox' .
'' ' a.r' "": c/:1:',
d:i: "' •.,-.da 1· + -a., da2 + , - da 3
oa1, a2~ ..... oa3 , =- 0a1,r·
~,·
, va I
dy~ ~oYCf da1 + J;t..
,I , "a2
da2 + f-"da.3"
a3 . ·
I,
= - &?1, -
~ '
(p.ql h2 h3) da2. da 3
. , 0al .
·ot d. · · az d. . oz, d .
dz = ,,-- a + .,,.- a + ;;- a . a .
oaz 2 ocz.a · 3 1
oa1 = - -:;- (pq1 h2 h3) da 1·• da 2 . da 3
oa1
Squa:ihg and adding these equations columo•wise, we obtain: i.
, Similarly, the excess or flow In over tT6w out from tho fnces CLMC' ~nd
dx 2 + dy2 + a:zZ~ (li dai + (h da )2 + (h~ d~3)2 + coeff.
1 2 2
• - . . . , , It
daz
,
6tda; O~B'A; .OCC'A and LMB'B are roJp~ctlvely ..
.. + coefl', of da 2 tia 3 +:coeff. of da3 da 1 ,I a
- ,.- (pa 3 h 1 h 2) ii1J1 da2 da3
\ and -·,-
c1 - .·
(Pq2 h I h3) do. 1 do.2 da3,
a~ 2 a· '2 ' l!z' i ' ' '' ' I ' ' ""3 ' vaz
where, /i2,,(-)
l ,aal
+(!!i..·)
~al
+(-)
', ilaj
etc. Rate,ofinc:rement in mass ofthe:nuitl within \he paraJlcl~plpe~ .,
By Orthog~naJ property; the terms Col'ltaini,ng d~l daz, daz da3;da3 clal vanish, = ~JPh 1_da1,, i2 da2, h~_d°,'~L,.
!{en~e·, ·
~p:'> ' :.-,' '
a.i,2 ~ + (h~-~.~~) + <~s .</'~3)2. \ '
= ai-'· h 1 h 2 hfda 1 , da2. . ...,.

.,,,,,.
L_, ' ' '
,..,,.,,,,,,,.,M,~!'t!\i:~'~'.''"'''.':~,,'.:!l'.'~~n~:·:;;·:,·,7,•,mmm1:::1i;';:117.:".'.'l~l','~~'.ll'•!t~~~!t\!':m!\r.'ll(l>:~~!,1-li''~J•\ff,>'\'eij;➔'"''ll'!/,;;,.,,,,,,,,!,~•;ll;,<1 •''<~1iS';<'W'"'''"Xij!m!i'll!ftl'lll-!~!':',<','1,i\iliffillli>1•;.-••·•!l"'1'1f...A_Q'I,...'""'"'~"'·'-·"~'--··-·......,•.,,,

,:::,:•~:('.•.~.\··· ',,'·'•'• ";':;,r'.:'.;::.


, ,, •,i:~;i1:;._-::·,,:
' .., : ;, ,;~-;,: . 1;·:~ :: '.;-;
'it>r:;,~H1•_::, TM"""ff;tJr.i(\IVf.•,1.;,,,,'./. • .J,· 11 & 1 , ~ , ' . 7 ) ;

---~ :.::~»;::· :
,,·,r:,;1.-;.:•,•

,',,

,;i
~ ' ~ " " ' l'ilW1i W ~ ...~-1-.itoo~, ......... - · ....... --~ww~-'\."•>!t.•Ml'l'ttn:11n-ts•n:i,t~:h•t!,;1ol11lt.l,,,,.•~--• .. ~...:....:•. .:. •• :.. ,,.'.:.,"1,,, .. 1,,,, ... ~~..:.~..,_;_,...;.,...,H .... ~~'.trl~J..~!.IJ,:,:.r,L!!,~•..... ..!.!!!J.;:,::.:.;,1,:,: .. ,~!i~~:utoto!u'..:.~, ..·.:.-·.:.:~,w-•-...-·.. ,;,111i-""~'dUOHHJl;•:~ .. :~ ... 1.. ,n,•1• .. ~·
k.l ' ~

r.~ s..
.;
!
FLUID DYNAMICS . k1NEMAT!C!!I @OUAT!ONS OF CONT!NU!ry)
.;r. ">.'f'

\ij ..:~,:-~
:
Equation of contin.uity says that
z
t:1 fo(;!'easa In mass J. total excess of flow ln oYcr no..., ouc a·,--...:..----,,B':
(; o · a: ·· ·: . a· •
i.e., ¥1 h1 h2 h3 da1 da2 jaa " -[-:;- (PQ1 h2 ho)+,;-- (pq2 h1 h3) Cf-+-----,'C' 1,
v , . va1 CO2 .

+ iJ: 3 (r 3 h 1 h2)] da 1 da2 da3


w

or £2..+ [ _l_ (pql h2


ot oa I
a
+-;-(pg2h1 h3)+ . ] ,-h-1--1-
(pq3hil12) 1 0
VA U· N
oa2. : 112t3 O p (X, p) Q (~ + O~, y + O)', ,. + 6i)
This is the required equation )(

I,
Deductions: (i) n\l;:t,angular Cartesian co-ordi!)~te-s: · ,
ds ~ dx .+ d;y 2 + ck. 2 " (h 1 da 1) 2 t (h2 da':1) 2 + (ho da~f
l 2 2
F!g,G
Hence ·h 1 .,h 2:=h 3 •1, a 1 .;x,,a2">'• a3·=·z.
lo this c~so the equation ofconlinuity becomes · Mass of the fluid tbat passes
'
in unit time across .theI fac~ QA'.13'0' .
'' ' . 21:. I
. -Qa~o
ct +tax (pq_i) + aya(P<12l.
. ;,, a',;
a (pq3) ] = o ", . ,. =f(x + w:,;y,z) "'f+ ca:, i!x
i •. • I Now the ·excess oi':Oow ln flow out from t.he tace APCin1.nd (;/A'B'C' in u~inlme
SphorlcaJ co•ord!nates : , · ·
Hero ds 2 "(drf + (rdG) 2 .j. (r sin (i dw) 2• ef-('i+ 6x :•~)a -
, ""'
&,: , ~~ -·~, f,(pu
, vX , .,,.,
&};; Sz).
,,
Then li10:l, h2"r;··.'h 3 •rs!n!l, a1 :r,. a 2 .,e, a 3 ~w.
In thb casa Ule eq~ati~n otcontinulty bo~mos ; .
i : . • =_ a~;..;l . Bx Sy a;, ·
a-:;2 .+
,,1
1 [. 11 •
1 • r . r B1n 0 ;-
:a
· ·, r, rsin 8)I+ :;-
,,r (pql
· • . .a • ]
".8 ,(pq· 2 • l. r · 8) vro (pq 3 . l . r) = 0 1
1n +, :, : SimUatly, the ei:cess oCflow in over flow out from r.he faces CC'B'B, PQA'A and·
AA'B'B, CC'QP Is re~ectively
££ 1 a c . 2) a a · a
, "'2, Mrr + -
·~ 1 'if ( •
or "ror
1 1 " r8 Pll2 sin ) + -
. rsnvo.
8 .-o -a ·(pq3)" 0 ·
rsmow . - ;- (pw) • 6.t 6y & and · -: ;- (pv} ox o;;
~ ' ', ~ '
oz.
(lii) Cy!lnd)'.ica(c1>-ordinales, Here we have Rate ofi6'crenie,ntin mass of the fluid v,,ithln th~ parrulelopipca
ds 2 " (dr,2 +(rd0) 2 + · a . -~ .
= re. (P ox . oy .&l = ~1 • &: . &Y . oz
Then hi :o l, h2,;, r, h3 ~-l,a 1 =r, a 2 = 9,d 2 .. z. ! r:
Equation of conlinu.ity says that · ·
The.equation of~ontinuit)' is Jncrease.iri xnas~ = total .oxcess of flow in ov.cr flow oiit i,e;,
I - + -1[a, (Mir)+~a·· a ]
(M2l + ;- (pq3 1•) "0 , ~·&toy oz., - Bxor.&[ f; (pu) ~ ! (pu) + f. (pw)]
J '
1 . .., .. , ,; or ,. 'vi r.,Z

a ·
6r 1
/ .Qe 1
,cit +-;:
. 1a ·
(Pql r) + ';:'.'~e (Pq:i) + az (P'12) = 0 or
I
-~t
•U
~:t- VX
(pu) +

(pv)'+ _i!iJ
Z,,
(piu) "'C,

. 1.19•.,Equa}Jori ol continuity in carte's.fan, c_q•prdina!es j . l i'


;1, · equation of~pntiiiuity. · .
· . . 1 , • • . : . • (o/arhwal 2002, 2oqs>
'l i • ,duid is itcompressible, then (1) becomes
au· aw ]. .
· Let p denote fluid den~ity at P.(x,y, z) enclosed by a srnnll :pnraJJeloplped with ·
. edg:es '.of lengths ox, 5,y, !\t .. 'Let u,'u, II\ be '.'('elo,lty components .along AA',·AP, AB
respectively. Mass of tho fluJd that passes in unit tlme across tl\e face Af'Cll
rau
0+,P iJt+&,i,+az "O
or : i!u + -~ + $£. .; 0
. " density , area , nomiiil veloclty · ii:,:. i)y . c!z. ,
'i
= p, 6;y. u· 0 f(~ 1 ;y, z)', say. oz; i This is the equation ofcontinu.ity:fo this case,
. O,'.
(i!J The equation (1) is al~o ~x·pre~sl'ble°·as ;

.:.1.
'" ..... ,
, ~, ·J ,,•.;-_•,"J;~:<.l'·,'{ll<f:/.--{.••;:,c'"z"\':',; ;.•-•

.... , ..-... ,..;.


ii
'ti, I l I l KINEMATICs'(EOl,JATIONS QF ~NTINUITY)_ • ' ,q,
r
'/• I · LfLl.liD DYNAMICS .•q•;,
'
.:·If . . .· i Ii
I (i~~.!;·+·v.f+w:* )Ptf( t ~~+-~ )\'~ ,. ff ,/2 sin 9 Sr; 68 • Bw,

[
.. (ii/) If velocity has one coi:nponect u, say, then (i).libcomes, By equa~Oll of continuity ~.
.hi
-11, . . .£e +m!.= o. an .. . . . [. i1 . . '
ft r 2 sin 8 or.08 6(1)"' - a'; (pu r2} , sin e+rail (pv sin 0) + r ~ _Br 6a ow
. a a ,~w' ·]:. .
at er. ,
This equ,a.lfon is uery important for further sl?Jcy. Simplifying this we get { . . · ·
1.20. Equatlcin of po.nilnulty In spherical polar co•ordlnatesi 1 a (pu r2> + -\-"
£12 i· -2, a ,pv sin. ei· + -.-"
1 -aa a (pw >'."o.
1 -i)
To derive the equ~Hon dr conservation of mass in spherical co,ordinates.
al r or · r5 uv r sin u w
·· (Kanpur 11./B2i Garh11J<1! 2001) This.is th~ re_quircd equation of cont!ni,tity,, .•
Let p de.note fluid density .at a pcint Jl · ·: B' Problem 1. Each particle of a ma•~ of liquid moves in a plan.a through "a:i:iJ of z; fi.nd ·.
p (r, fl, w) encl9sed · by' a sin,all
parallelopiped with edges of "lengths c
. ' '
; c•1 · i , •
I ,r
the ~quaUan. continuity, .
; Solution: Prove as in above Article 1.20 that: ,
. / ,· : .
l . .
or, r _Se, r sine ow. Len,, v,.w- be v~lociey I AA I 7a ~ 1 a < "> 1· a , .. a ( ·.
coroponenls · along· M',AP,AB
respectively. Mass of the flu:id that passes A
"'
u
.; I !
: j ·!
8
= '""
96
"' 4
y·' . ·
;z o) 1
i/t + a, pur + ~ a8 /)V Sin + T;i;;a 'ai;i pw " 0
l

in unit tim~ across the face APCB ls V . A' I l particles move along the axis orz and lience u, = Q'.,
is
i_ .

density. area·. nomia.l Velocity p . :Q • 'Equotion,ofcontinulty


"fl . (r" S8.r sin e.·ow) , 1t (r. ~. r.,) ~1 I, · I
I '
l : ..£e lil 2 1·-a•
' . ., ' g. V'
iii + ~ ai'. (pu'.. )_+ r sine fo" (,pv sin O) "O.
= p r~inin 8 .S0,6w = f (r, e, w), say,
.. . : i
Mass of the fluia· t.hat passes !n u:cit time a~os.s''the face A'~da1 is Prqblem 2, llollU!geneous liquid moves so that the path of any particle Plies in the
. . pf_ i: plane POX; where OX is .f...xed axis, .
f(r+or, 8,t!l)=/+or·. ar. ·
Prave that if OP = r, LPOX" 8, µ " cos 8, the eqaatlon. of co11tinuil} is
Now excess of flow in over flow out'fyom the facesAPCB,A'QC!,B'. in· unit time
cf-(f+6r.f; )'•-6r,ff,:
·,., . i)t i;: (r 2
q,) - /µ (r:a sin 8) = 0,

where q,, q9 arc the components of velocity alontf and prependicular to OP in. the plane
= - Sr . ¾ r u sin 8 oe ..Sw)
(p-
2
POX. . .
a -.
= - 6r , F (pu , r' 69 . ~ sin 9 ow).
Solution :'Here ):!lotion lies in .ty,plane.
. . r . I . . .. ·•... . . , Hence w" 0. Prove os a Article 1,20 that
Similarly, the excess oflfow in over flow out from the facesAPQA', CC'B'B and
M'.~'B, PQC'C are, respectively.- . . '
. £e.;
o/ .!. _ar
l...(pur
r2
2) +-~-l.. (Pu sin 8) + - 1-
r sin·e ao • r
~" 0
sin 8 ilw
-r sin 9
.
·
ow.~
rsm volJl
"a"-
(pw. r 6e. Sr):
:
Put · w" 0,. p =- const .. so t~11-_t.~: O. t
and - r $a • r ae (p~· . .Sr-. r .sin e 6w) · we se.t
il ·
'
Total excess of flow In over flow out. ~ar
lil 1 l
(pu r) + rsin O aij (/W sin 8) a 0

= - Sr .'·ir
(pu.r 2 ;in a SU01) .:. ooi -l;; (pw r 68 _6r) - se:, fa- (pv;r sin aSr 6w) a,:a (Ur 2) + sin1 a OOa (rV SID. 9)" 0 ... (l)
a< 2 .· a. .
=- [ ar: pu r ) :,,:~in ~ 1'. r oa (AQS!ll 0~

,i.
<1 }
r 'ai;i (pw) : _6r, 68 ,:OW u
o . Bµt µ=cose,~ dµ=-,sln&d9
· •· a · 1 a
Rate offocremen t h{rii~ss of the fluid with!!) _tho paraUei'opiped_:. . . }f ~1,1:~_- J,ln 6 iffl , , \:i,
. .:iii{J&i~lr oe ·• ''" A ~,;,,. . : · ·
Also u "q,, .v "q 0, WH!(theso values (ll becom'es) ~,
I

L ..,,•,:···~;:--:·, ..
. ,.,, .,.,. '' ·,•,1·,•~~:?:~:1.\~:·:'

;•~;·;::::::;:•:, '::.::··'.:-:/•'·' . ·;:?:~1::::.::·•


,''.
,,,:/,:;,,{)"•,,• .. ',, ' . :,.,~·::·.-~-. <,,-.~: ,' ~~ !: :;::;
!;•H'.t~·:;fli;•-:,~•::-~:•>-~{•, ;:,._~:•;;::.;;•:• •. i

=-====tfTl:r'f.........-~f•"'m"'.~."'j,t!,;:•~;'.ixil'J{!f:'y:

·:~0:!:;:·;,
,,•;•,1'.I,'.)•,·
·, .•,:.'.~:;: .
... ,, . ,,.
~!i
itl'·'·
ti
~l
•t.".:I.A.1ht.lfHJ.tk.',,fl:IHt,•nt,11t......,_,......;t.,.i;:~J..1:.., '-',,.... j.•• ._.,:,; _ _ ,., ~ .............,.,.:~~~:.:.:.~..:....:..;:.. ..!.!!1.;:S..:!(L\!,.;~:~Wi.!i.!l.. ,: .•...:•.'., ·•~',w;, .........; . , ; , ~ • r l e f f l , ~ l b . ~ ~ ~• ..l;.!t.l~IJ!;!. •.~:·, •:, ·•:,JHl!HWt••., .. , , ., , .... ,,, ••• ,,~, ,,-.,!"' ,J~t":j •~f".l1Hr-:,~h-•1:,,, .,. :. ,,,,' :,.;

;J AO FLUID DYNAMICS
J(_; .,_~ 1/ . ,;-a (r2r;,) ,..,
a (rqe sin 9) = o. KINEMATICS (EOUATIONS 0/f CONT!NUIJY)
,{•f;
):J ::-, : ur , vµ ,,
;/· 1.22. Certain Symmetrleal lorms or-equatlons·of con~nuity
ll,..I. i
·-1 1.21, Equa1ion of continuity In cylfndrical co-ordinates·· (Garhwo.12000J · I, SP,h.ericaJ..S~metcy ·
Let p denote lluid dedslty at a point P (r, e, z) B~------..., B'
en.closed by a small paral!elop,iped with edges o_flengths' c
fir, r oG, oz,
Let u, v, w pe velocity componel),tS along '
)j' .--L1 , The .motion is symmetrical about the centre of the sphere and velocity q has
only one component a.long the radius,:. Also q =q (r, t). We consider two consecutive .
i ·sJih~res ol'radii rand r_-t- /Jr. Mass-ofthe lluid which passes in unit time acro~s the
AA', AP, AB, rospectively. Mass of the fl,uid that passes : iti,.,er sphere is density. area. normal velocity ·
•{!
in unit time across the, face APCB is · tw =
p. 4itr. q = f(r, t), say. '
d~nsity, are~. l'.lonnril veloclt:,i
=p.ro9o•u I~ 1A'
Y!l
I7 Mass of.the Quid tha_t passes acros's the outei sphere in uri:il time

.i: = r (r, a, z}, s_ay.. '


Mass of the fluid ~Hilt passes !n unit
i
the
/>(t, 8, 1) Q =f(r +Br,!) =f+,Br :l.
iI Fiu,1
The excess of.flow in•over now out fro~ 'these two faces
face A'QC'B' is
I N. . iJ
r-.(r+ or., ar)= - or. f
/(rt- or, 8, 2!) "f +or; i• .- I:
=
. f) .. ·.· . ,' .
, I
time
Now excoss-offlow hi over now ,ut from l.he facesAPC8 and A'QC'8' in unit
· · · ••. I •
i
"' - or, (p 41ll7_ q) "
4it.or;'
.
- fr 2
(~r ' q)
Rate oflncrement ln•the mass oftha fhud wit.!ihi the spheres ,, ,,..
,. r- ((+ Sr . g/.·);.
or.~ .
- Br·. J
r;.ir,
= - dr j..
or ·
(pu r 68 , llz). ,, ,,. = ff£ (411 ;,2Sr. pj ! · ;;,

. ' Sfmllarly, tho excess of flow ll 'ov~r flow o'urfrom the. fgces M'B'B, PQC'C ancl ££ '.
• · PM'q, CC'B'B aro, respectively. •· : , . . . : . · ~ cit • 4itr 6r.
·• a ·· , · a, · By the def. of equation of continuity .
~ r ~~ .'r6e (pu • or.&) ·and - & az (pw 6r . r 60). · a a
Hence total excess of flow In over flow out
j '.
_e 2
a, 4it r or= - .4it. Sr,. -or (pq..r 2)
-~
" - [ ~r ,1 \PU roe.,
.

f
¾
.


:
6B) + oe . fo" (pv or. M 1- 6~
'
'i

!
t (pw or. roi!)] 1 :
'.
or ~ + J. .! {Pqr 2) =0 .'.
iit ,:;. er- ... . ... (1) .
1J • : ·a · ii ' · this is the ~equired.equation ofcootinuity.
. ":-[a,: \Pur) +aii (PV) +a, (pw),, r] 6r. 00. oz.! Dedu~tlons I (I), Ifthe·flulcl Is incompressible, then the last becomes
Rate ;flncre~eniin mass of tho tluld.withln tho pru-o,lleiopiped
. a· .... o+~¾(r2q)=O or t(r2q)=O
"~ (p or. r 0~ . oz)
,, Integrating, r2q. = const . .,. f (I) ·· or •r2~ "'f(t);

. "~7' r 6r 6e &. (UJ Problem: Thep~rticle, of fluid mou~symme,tricallyin. space with regard
to .f&:ed sphere, ~hc:iJ t,hat.etjuation of continuity is · · ·
j:ly eque.t!on of continuity, . i2E. . gp_· ;i). 2
· ... a · [a a a a1 .+ u ar i= (ru) •.O,
sgf r oit 68 & ., - a,: (pu r) + a8 (pu) + a, (pw). r liB oz Thls follows Jr,,ni'equnt!on (l)~d the~e replace q by u.
(Ko.npur20Q~)

or
im
iJt -t-
1a ta a
7, a, (pu r) + r ;ie (pv) +;j; (PW)= 0,
l: 2. Cyluidrlcal S.Ytnmetr.y: In tl:\lscasevelocltyq at any poia_~is perpendicular
:o a fixed axis and !s a function ofr'a.od t only, where r is perpendi.cular distance of
1· \~~ point from the ax.is. Consldedwo ,consecutive cylinders of radii r and r + or
•,•
I This is the required equation of continuity.
i ~ou:nded by the pla:ies at unit distan·ce _apart; FIQw across tne inner surface
I,'
I '
. , . ·. ' . . ,
= p, 2.ll~,::-q :" ((r.;,n say.
!.
! .
j
l ., .: .,, ... ··~·--···
I
l

. ~ . . :I
~5!~l,IW,;r:~1·i')!'!l'<l:!1ll('lmlffl,l'iffll!.li'!llITTffilH:ll:'l'.1.!l':M:'l"c.l'.l'.::lr.U!fflfflm~ll:'im111_,~ii\;1/\/>;1m11mtn!!m!'!!ll!1lllimi'll1i:l\Hfl',ji\!<i';!l:;'1!1,.-t·::e•,::,::::·

·••"'-'•'-"'"'-'""""",.,_,,,_ ___,_,-~---- .....


, '"-•·~- -··· , ........., I "'••-H'"·•-··· ~ --~>• ,~- .... ---- , '·" •ru, .... ..,,.. ""'-"'~-u~.Vl-l'.~~~.... "--'"-'•"... <:,"t•~,•.~,.......;_"1.,-'"(._"<,.,...._....... "" • ., _______ •• • • • :. _. •• ::.·•• ,,. ·-~-:.. ::..:.,.-_;.. ~-,.;,:;,,;,:,,·::.11.~"!ol,.)tf.~"ll!l~.llo~'!.,..... ',!,j_..._..;,.,.:;_;.,.:,;,.,,..,.,..c..,1..11>01~.S"'~~'S"tl-~ti..~l.\'.t).;,,'1,'.:.,'t.'1,..'ck"1..-,:,,_.._:.,;...~.._.~,.. -,,,.,....;.::..~......:-.M -··~ ...

,v

, FLUID DYNAMIC,S KINEMATICS (EOUATIONSOF CONTINUITY) ~

T!'!ow ncross outer This => + ,


-~ =dt;d:;:=-0,·· "

y .
I
0

+or, t):;f+or:.~:- ~'- lntegnting, log:,:" log (l + t) + log a,


:. vr
Excess of flow tn over flow out logy= t - log b," "c.
, . ' ii" a, or i x= a(1 + i"), ()I.lb)~ e1, z = a
= (:I-or r-( fr)= - lir, fr= - 21l or, or! y"' baH•la) • 11 , z., c, \
Rato of,incremen~ in t~°emass_ofth~ Ill.ti: con_tained fn th~ I 'These l~_9,equatio1:1s represe11t path Hnes. i
A.ti•·
\
=1tt (P, 2nr ,,Sr)= Ti, 2nr 6r fr~ble~Determine the streamlines an"d the path of the pqrlicf,es i
· :,- u=x/(l+t),u=y/(l+t), w=il(l+t), . (' (1.A.S,WM) !
def. of equntion of continuity, -Solution : The equatiqn ofthe streamlines are given by ~ •• _••.,,,_,.,,,_ ..._., ..
·££ . : . ·a
dx _ rJi _dt. . .
at , 21tr or= - 2n 6r, fr (prq}
a +-1 -(prq)=
d
u - u, - IL\ I
~ 0 ~ -..-~,,,_d_z_
or
at r ar or
x/(1 + I) y/(1 + I) z/(1 + I) I
This.is the required cquation'ofcontin\lit:,r. or · d.xlx d.y/y _= rJz1z·
Deduction.: Wben pis constant, thon tho last gives (i) (i!) "(iii)
1 . a , a \
By intogratiog (i) aod (ii), we havo
o+;,Pa,:(qr)=O or ~frq)=C C
Jog ;r: =logy+ log'A, A is integration constant.
Intewating ;q = const, =f(t)· or rq-, 1•1• . ,
= x=Ay ... (l)
~olved problo:iis related to stream lines arid possibl~ li°q¥id By integrating (i) anp (iii), we hav~
P"ro,blem l, Filid the streClm lines and paths of the particles for the '.two dimensional log ;r: = log z + log B, B ls ·8Jl Integration constant,
uelocity field: : : : ,,, :,; ~ Bz, , •• , (2)
, !Hence ihe streamlines are given by the intersection of (1) and· (2), The
u = l : t .," u "y, w,; O, dlffereotial equation of path lines is given' .
So_lutlon: We have . dr
q=dt
U" + , v =.y, w = 0.
~ 9':!:,_ 2 - J s& "..L dz _ -~
Step l. To de,lerroioo stream lines. dt - l + t dt 1 +t ' dt - 1 + t
Stream lines are the.solution of dx
=> -:: z-
dt dz dt
-~--
x. +I' y l+t' z' l+t
d<4z=
u () w Integnting, we get
log x" log'(l +t) + log a I
Putti~g the value·f!...:Lll d.x = =
• .. X y = log (1·+ '.t) + logb
, I
log z = log ( 1 + t) + log c·
This => (lli)r:1.,:,.&
X
:!:i.,,~
y ' y 0 => x = a (1 +t), y .. b (1 + 1), z ~ c (1 + t)
·\ =>. (l + t) log x =logy+ log Cl, dz= 0 Thefie give required path liMs.
~ xl+t :::ay, z'cb Ans,
Problem 3, The velocity q in 1hree•dimet1Slorat flow field. for an incompressible fluid.
These two, -~quations re')lre·sent stream lines. is giuen by · ' ..
Step II, To:dei.emiine path lines. · , q = ZXl:-yJ.-::k . ·.,: .
. Path ll~caare the &fluti_ons of. . . ' i Df!lermine the equation~_.o(Ch11;_stre11m lines pass(ns: Ihrou,i!l:ihe point q, 1, 1).
~~~ ';:ff.:t._
;~,t'dt·:·J1-,,,-Uidt -':I, di,:·,,.
1:.,.6 : 1 ,":::J/'· . ··'.,.~·.,:·1.!·.· ' · ..;,'.">/\ :...., .
:.'""':..,
"·•
·-".'', ., ' '.". :··... : . ··•· '

:} .:~ ;~-;~;,.: . •:\•?::--:-,


. •; ~~ 11 ~,}~\. '. ; .. ,_:; ' .
, - , , " : ' ~ · ',:, .• , • ,X ,;,,•)SS9\.P~"•:~111:.!~ I 1,!.~~;z;')'.ii;:!',:'.\~

--... ~.-;---.,.-., ... ~ . ·;


, ,:J~-. _.'

i.:l
I
0\;::i ,, , ........ .,_.. ., .. ,, .... ,._........,_.__.....__u:,.,.!.f,,:;. !,.. ~,:,',.:,,J.;.:,!:::i:,:,/: ,.:.$1.t>;r,,~; ., ... ,,. ;, __ :,.,.,__;; .,,~ ......... ~,.:..... ~-:--;,j'_-,,~.;i~nu,::~~1·~-•-
. ._·;:.,~.!:;;,~••.'m,G.:~;;;. , , •~""~~!t:,,,;:1::,;:1•:•t

r,: FLUID DYNAMICS : KINEMATICS (EQUA"TIONS OF CONTINUITY) t"!, . ' -----1&


,;:1'2-
al"" i ' .
I
·:,
Solution: The equut!ons·ofstream lines are given by
~
f!:!..,! ti.,.., s.-,.·o '" (1, 2, ;~)
... dx I
--a
dz ~ -dx; z ldv:
=- dz
~=i.-. i
dt t' dt · "'· dt '
u Ii w Z;; -y -z By Integrating (l),. we have

From (i) and (ii), we have


j (i) (ii) (iii) :
:=7 .,. '.ogx-=logt+fogA .,._l:x=At, .. ,m

'E',:.!ix..' ~ +~=O Let (:r:o,Yo, zo) ba eho coordioates of the chosen !lu!d particle at time t = 10, the,?
2x1-Y ;,: Y
By Integrating, we_obte.in x 0 ,.At 0 ,a,A.,:!!
to '
log x + 2 log yO=,log A
or From (4), we have
Xo
x =- t.
:c;,,~·•_A, where A l_s an !nteg,-ation 10
.
From (i} ond (iii)/ w'e-hnve
ri.:r. d.. ~ 2!.lz· · By intagr~ting (2), we have
-=---=- ~ -+-=O . • fb_
2x; -z X , z· =dt

By int.egrati~g, ,~e have
or logy" t +.log B ~ y =Be''·
. .i:z 2, = B, where .Bis an [nteg-ration constarit. ·,,!".

>' '=Yo, ~ lo .,. 11 "Yo i 10


i.
I
j At ,·., (5)
At the point (l, ~.1),A z l = B
Hence the requi~ed stream·Uoes are- I From we have ., ,.
i-t .
. :ey~ ,. l . ahd . :xz2 " l.
.
i
.. I ~
Ans,, 'i Y "Yoe:• o.
Problem 4, Find the 1equation of the siream Unes for the flow, · By Integrating (3), we have .
I
. q= - I (3.,..2) .:, J (6.r) i L cl.:
=
' ' . .
dt. 0 ~ z = c i.e., z is Independent oi t::::. z =z0 ,
·at the polnt (1; 1), ·
Hence the path lines are given by
Solution: T.he equntioos ofstreamlln·e ar~ given by
d:x._tl. ,.. "(x0lto) I,'/" Yo e1 - 10, z = z 0 • ... (6)
' u - ti ..• . A~
Here q .. -i(sy 2) u=--3y2,v;,-6x· L~t .the fluid pal"tlclj? (%0,Yo,zo) pass throllgh a fixed pohit (x1,Y1,;1'1) at an
d:i: d .. Instant of time t "T, where 10 s T ~ t, Theo-th, re!atioa· (6;.reduces to
Ot - - -• ., .5!::l. => or 2:c d:i: dy
.., Sy• - 6X . . _%1"'"'0.o
( // ) Z',Y1"Yoe
. · T- Io,z1·'"•0
·
By jnlegrating, we have .
·or. x 0 = (-x1IT)i 0,y0 =;; 1 e'o·1',z0 .,z 1 ,,, (7)
·x~ =: ½y3 + c, •wher~ cis an integration constant. · · where Tis the parameter, Sµbstituti.ng the-relation (7) into (6), we h'lve
lhe· point O, c=l3 =:> 3x2 =y3+2, Ans. . · ,: = (,:1/TJ t,. Y"YJ ~t-T,z ;oz1,
·. which gives the e4~atJon of streak lii:ies pe.ssifg through ti:ie point (:r:1,Y1, •1),
which determina6' the _e·quation ofthe st,ea.o:j lines.for the fl~w field.
: Problem 6. ·The velocity c~~pon'i!fls In a. two-dim.Msional flow field {or on~·
Pr'oble111 ll, The velocity field ,:it a point in fluid is given as ; I incompressible fluid are giuen by . . . ,•
1
: q " (X/ I,.)', ()), ' .'
Obtain path lin~s.and strea~ lineh. · (Meerut 2002) ' · u = r! c9sh y and 'i.J ·= - e:" sinh ;,.
Solution: Here q =(xlt,7 1 0). Determine the equation o{!he streamUnes for this flow.
Tho di!forcnli:al _equatlons or path llnes arc given by Solution: Tho equatiqns of the litrea~ lines are given by
d. d:x.· d'.·· .. ..
I q=-=
dt
dr (iv.
i+=J•-k=
dt . dt
dz =& => --.-=--+,-
u u ·. e:" cosh.Y .•r s1rihx::
or dx+cothydy"O
.
i n
l. -•·-.... ,,.._,_,
I 'b,

li:"<lltil!J~~'.~'(i~\1\'l!f/~"lY,~l$!lmRlmmmrmnr,1m"1Ji7-:•m;;.iali!r.mii;:,~~i!lW1:~1l!!<\\',\\(<'.•l\:($\llffll>m'lliffllil!'!ll9.i!tWl-!f,:,1,;,,~,,·,<\l':,t;c'i> -~--·,,,:·

.... "... ... ~.,, ... , ..


.~fl~U•.:.fu,~•,.::;,r<,~V'•'>""-'!, "'~,

l
l
l

I
..............,
\FLUID,OVNAMICS 'KINEMArlCS (EQUATIONS OF CONl'.INUITV)
.~l':·
, By integrating, we h'ave,, . ,
Intc!lTating
·'•x 2
,,z:,., (1 ' + Alo)
. C
;y" ,
, X f Joi/~~ y"o, }Og C => ; sil;ili j = ce-x 2
·..,:,,,here iog c !~ an integTa·u~,:; codstant. · Ans, .or· .:,Z .. 2 (1'+ At0)y +·c ' ... (1)
Pi::oblem 7; Obtdlri the i1fea1?1, lln.e;,of a {low ,
. 'u =:&i I/ =--y.. t
X5'=Z(l +At0)y 0 +c ... (~)
(1)- (~) gives ,I f . . \
Or,' If Iha ueloaity q iii giue~ by ·
' ' q.. ,:d -yj, ' · x2 --~= 2 (l +Alo) (y-yo) Alls,
I!
'determine' the equation ofth¼weam lin.e~. II. To find path lin~s which pass through (x0, Yo) at tirno I" t0.
Soluj:foo : · q .;_, iu + ju + wk Equations of path lines stc .i = u, .)I'= u. ·• ·
Herewehavo · u=x, u=-yi, w=b, . or
dx.,, .
di ' l +Al, di -
4x_ X
Stream lines are given oy
1
1 Ii
:::; dx = (l + Al) di , ... (3)'
di,.fl.:t.=dz
ll ·V W . dy=xdl '" (4)
i:Ix_Jtt._~ Integrating (3), we get
or
;c >Y- 0 .: x=I +1~+c 1 "' (5)
! • 2
_sf1..
dxr--y• -~
=>

or
Integrating "these equations,
··log~+ .logy= log c, z = ,; 1
. x:,,i=c,
,,:-0

z:c 1
,
(i.e., dz=

'
0) Put

(5)- (6) g{vea


~ i:::t:r0 ,

· ,. A z
t = t0 .

x 0 ,. t0 + t 0 +er 2
• '" (6) '

'
: · A 2
Stream lines are given by .xy "c, z = c 1, X - XQ = (I - lo) + 2 (t 2 - lo) ••• (7)

Problem 8. Consider the uelocity field giuen b:, Using (7) in (4 ),


. q"!(l+Al)i+,rj.
=
Find the equation. of stream line al I t 0pc.s:ing through the point (xo, y0).'Also dy =[ Xo·+ (I - lo) + A (1 2 - /~) ] dt z
obtain the equation of pa/Ji line of a /lltid element which r.omes to 't.xo, Yol at t = le, . 12 A ( t.3 2 )
'Show that, if A= 0 (i.e., steady {1ow), the sirwn.l/nes and path lines coincida.
Integrating, y =
, x0t + -2 - 101 + - - - 10 1 + c2
2, 3
... (8).

Solution : · q =(1 + Al) I + ,tj ., ,. . PuWog, y '= Yo, ,t = 10, we get


and q = ur+ uj;+ wk , Yo t5 . 3

This·.= u'" l + Al,


r. 'fo determine stream lines,
u = .r, ·. ;,, = O. ,;

,;
'I
(8) - (9) gives
0
2' A (' 1
:tolo + - -lo·+-
2
)
!Q _ lo3 '+ C2
, 2 3 ..
,., (9)
::
I . ' ; 3 3·
These lines arc glvcn by
y - Yo: "o (I - lo)+ l (I 2 • lo)
2
- lo (t :- lo)+ A [ ( -3-
2 I - lo ) ,2
- ro,(I - lo) ) 2
dx=~=i!!_
u' u w
Stream line~ at time I= 10 are given by
i
; I or, y-y0 =(1-t 0\ x 0 + 2 ·r
1 (t+t )-to+
0
· A
2
i( 15 +t32 + tto). ~~ 1··]
~aflt.
II or . 1
y - Yo= (t - t,0) 0 + 2 ['x• i,i 2
.c+
10) + . lt + tt 0 - 21&1
J ... (10)
8

! {n two dimensional m·otlon,


l + Alo !<
Required path lines are givon by (7) and
m. Let.A =O, ):. , _r .
or X dx d;y
To show that path liri'cs aod s~ream lines are cofncidont .

·\:/n-tr: ......,.. ·:·.::~filt,m]~'

;;,:~·::?· ·.- ::, ·!:: :··· .. ·· · · ··· · •~· ::r r:: ·-~·

;.,,;,,·. . ,;r:':t;;~r . ::,: ~ ':: :~:• ' .


-----~,...,•·r•-.~-:<~-TT"'~~~~~!{.·•~l),",:.t;1:~~1!,!>;)\~l;:;]:;;i:'1°

.:{::;;:::;:r.
q
,,,,,,i.•..'1'.•'

.~,..,,,,.,,~,._~,,:,,i1r1'.'"'""-":;;;";;,;,. ,, ...,.. :,. .:. .....;,;, . ;. ,, . ,.;;:;~~.u~-~·•-n(i,-¼i!.lllltti1,-,,,,,,t."''''''''''''•'··'····:... :. ,.:.,::·,,,,;·.:."-';'t';~iu::""""'-"-",;;w;"'-""'"""''\l!:•:::::,:.,.,::· :,,.,.:,:,,1· '·•''-'~"1.. ,.:., ·-·"··--•~.'-••··-·~::.:~"'~~~~.. .,..,,. .. ,,..... ,__ ,'
1 , • I I r , , · ,
.;!
t'
1-Lf .! . . FLUIDDYNAMICS I
i !
I
K;NEMATICS(EOUATIONSOFCONTINUITY) · .
~
. ,

jl: By step I, stream lines , Step· II. To determine $tream !me~.


1 :i
1/·
,..z_ .ro"
2 2(l+Af)fo
o v
· • · ·dz d.u d.z
.Stream l!f\es are the solutions 0(7"'
· I·
v
.=:w .:Putting the valu.es,
;!I,\ or x2-·1
. tO
~ 2, ()' -Yo) "' (ll)
=
.,_ .
. d .w:
.fil!..
.,_ +· :to.Y
..,_ X u.,:; J'
+Z .d2 y· .d·y. + Z dz
· \; ' By step !1, path lines ~r~ g:lven ~J' · 3x2 _ r2 ~ 3xy "a;;"' x (3r2- r2) · "'3x (y2 +z2) '
'!J-Yo"'(t -to>[ Xo+½ (t-lo) +i 11 2 t'/to-'21~11, ±1...=.sk.
1
I by(lO) 'rh'15 (;!)
' • \ ~ 3xz 3.ty
and 'x-.ro=(l-lo}+~(r-15), by(7)
1
and xd.r+ydy+zdz,,,.ydy+zdz
'" (3)
, . ·2(x2+y2 +z2) 3(,r2+z2)
' : ( ' 1
This = \Ji·- Yo" (I- lo) Xo +,2
' ; .
- (~ -
. . (2).,,:,, ST:,,~= 0, int~gratlng this fog,; 7 l~ga
nnd ' !x-::xo•t-10 ! · :ar · · ,y;. a.:: · ,, , (4), this is II pla.rie through Ox.
'< ( js ' •."
Ellmlnatl_ng I .,.1 0 ~om the lruit two equntfons, I I '.J. 1ntegrat1nt ca>. we set :
. ; . [ ,l . ]
.,y;
Yo" (x-xo}. xo+ 2 \x -xo)
1
. . l log (x2 +.,..2 +z2) ,.1 log {y2 +z2) l lo<> b
.2 .. a . . .
+ r•
:,' ! · ·or
2
·or 2-<Y-·Yol., x ·- .t~, . •{x2 + y2 +~~)3 "b (y2 +z2)2. . ... (5)
I
wh!clt is the same ils equation (ll),Jfonce s\rcnic Jines and\)ath lines are ·Problem velocity of an iltcompr.elsible fiuid ~t the (x; y, z) is given
J?roblom O. Prove tha.t.liqu{d. ~oUo~ is possible when ueloci;y'at (x, y, t) i~ 11iue11 by by
(l~ -'r2 ~ :iz2.:.~2)

' 3,...,,
u .. - - 6 - t ti a 7 ' w :11:l '
'
,r•
. 2 2 i .
where ,.,.. lilt' X + +
,r .•
r, ; c~
..'l, ~c
·<!l
\ ( f rs ' r5 .

and the ~lream linu are the intersection of the surfaces, {x 2 + y 2 ... z 2 ) 3 = c (y 2 + z ) ,
2 2
prove that the· liquid motion· is possible a~d the velocity potencial Is cos e;r2, 1/so
I
l>y the planes pas,ing through Ox, · l (Kattpiu· 1993,Agra 92) 'determine lhe $/reani lines. ·
: ~ Solut!oo t StepL To prove that!he liqukl motloIJis possible, For this we have
3z 2 - , 2
to show that the equation of cootinulty namely
, 3xz
Solut ion :·Given u=-,
. 3vz
~== ,· w =---
.. r5 · · r6
r" ·
·
., ,au>£!!.·+ cw= 0 g
2 h . or x
"' (J)
OX fly 02 Since r 2 =x2 tz ' ence ax= r etc.
is sa tisflcd.
2 2 .2 ·2 -or ·x· i!r v. <lz z . Step l. To prove that the liquid motion is possible. For thls we have to
r c;i: +y. ·+Z co-=- -=,:. -=- ,.t!:;at-.the eguatlon-0£ contini.rlty' · . · ' ' . ..
·.i . . : ax .r'' i;/y· r ' i!r r
. . CU. QU QW .
i. , E§x ~- ~l r~ - 5r3x (3x2 - r2) .fu! k
' " 'r}O ' i)y,. r!O
sr3 >-2,, i
a;+a;+1i.""o
,./ il• sati~fied,
£!!' ~ •Sz (rs ~ 5,.3x2) '£!!. ~ la (;s _ -~r3..2)
1
· : -dW ;. (r 6 -5r 3 z 2) , i
ilz · ·. . . i, ax r10.. , •,i)j, . ,.10 7 ,

Thls • co
·cl.I. : 3x 2 ·2 au &:
-6.r),•,-=-(f -5y),
2 2 \' . .
. -i=-(3r
ax , r? <1.l' · r7
r
io {(6z - 2z) r.6 - 5,-3 (3z2 - ,.2} zl
.
7
~
·a Si (r 2 - 5:r), '' This=> ilu illi '
, + "-· +
3z O ~ 2 2 ·
,!, To {Zr - Sr' (r •- z )l +
1 5 J s ·
(9zr - 15r z ) = 0.
. r I • v✓ 10
OU . au aw ' .
"~

Hence the result.


r r
t,·,
"" il.r +a;_+ ;i; = O. Hence the result, 1,~
• . . B l'f
Sfep II. '.1:he_~how that~"' to~?,•' .. ,(

l ~
1 .

•I'•"•••••
.........,.____ .,
,

...,~ i ~LUIO DYNAMICS· KINEMATIC~EQUATlONS OF_CONTINUl1'Y) ~.'.":'.), l:


···.•·a" ]1. 1: .,
~t~.~~7 ay
£1.•.§!.
~jl.; az ~~ ".~11.~ ~.u dr,1~ d: .
hr .
! ·Tho (5) a.;1d,
x•.;t,·(,.2·+yZ.j.,z2)3....
(6) i3qifatl:oiia re1fre.sent .stream Hnea..
.
,
~- (Iii).
, Ms: ~-
t!
. ~-:-fra:i;:HL,f+3yz<h+(3z?-r2)~1. ~J' i>r..oblom.i/.S,\ow t!idt' if,velo~iiy pote11/ial ofdn lrrorotiori'al fluid. m~t/011 ·lj equ~l
I'
. .
."'~ i lsz' {;,: rk'.+ Y r/Y 't.,/cizi - :· dz];
·,
2
I.: '
'· I :
'
•:,:;;;d;il'I
~i~Q·\o-4 (x~+y~
,.. ·· ·. , . .
+z 2r·a,l2'. ~-1~11- 1 {y/x), the llnrs Qffl.ow iie M the serleiQfji111 &ur/ac~~.
2
7, x +y2 + z2 ,= J<:'~? (x2 + ,r2f
3, (Agra 2001,· 200,f; Kanpur 3002)
1!
i
~

\'J, · : Solution.:' Spherical co-ordinates are , · . . , .


. rl .[ . , .( ,,z )' 2. ·...] '.. , . .
=J} Sz.d 1} .~r .dz ,. . ; X =r ~in 8 COS (I),.)' = r Sip S!n (J) 1 ; e H COS 1)', ,.
. ! ..
· .3z . . ·dz. ( • +
$'=.A (;,:2 J2 +z2)*3/2 z tan•! X.
;,:
.•
,-3'. r. cos eta.n· 1 (tan(!))
'
.-.=--dr+-·=d -3
• r4 ·· r~ r , "A .
. ·.. z ·r·cos·a · cos8 · ··... · · ·
$=A r- 2 w • cos e. giv!fh by
Integrating,$"-.." ~ . = -.- 2- , neglecting constant oflntegration. or Lines ~mow are
r' r. · r '~ ... '~
i:J.x.,.d::i.= dz
. Aliter. ~ -3xz ·, U V W,
/3;,: =-u.=7
r de
, Integralin(i w.r.t, ;,:;
.
orequivaJently, =-~=
dr
_l.£!
r sin 8 do>
= __l_.£t

~ =_ ~
. 2.
f (.2x} (x2~+ _y2
'
., i,2)" G/2 d:r.
., .
i
or
dr
or r ao
rd8
rsin6 aw'
rsln8dw
! 2 A ~ = .!~wstne "'-- 1- ~
=(.;. :· )(-/) (,:2 + .)'2 +~2)-3/2 ,3 r r2- r s fll 8 • rZ
I d.r . rd8 rs!n 2 0dw
,. •. . z rcosO .. cos'e., or ~ =~ . . - cos e
or
.
'+'=
(:? +:,,2 + :2)3/2 =-=--=.......,.,..
r3
r3 r2\ ': (1) . (2) (3)
:f:;'.·~itl~Yf

on neglecting cons.tant oflntegration, i /


lly (l) and (2), £!.r = 2sm
7°508 de.
Step rn:
Strea::n lines nre the solutlons..of :
~_!!J. _d. Integrating, log r = 2 log sin e+log K
•l,-u-w' O.. , . ( .x2 + ,,2 )
Putting the values ofrespective terms, ~!' r=Ksm 2 6=K r2
M ,..El. .,..:..!k_,. X d.x+ y dy +z dz: or r 3 =K(;,: 2 +l)
3n .3Jz. 3,2-,2 .. 3, (;,:2 +y2+z2)-r2z ·: (:r.2 + yz +z2)u2,. K (.x2 + y2)
(1) .. (2) · (3) i . (4)
or
¢r
,
x2 +':;2 + z2,. 1{'2,13 (.i:2 + :,,2}2/3,
Taking the ratios.(1) arid.(2),'
. d.;,:
;,:
,._s:L.,,
.)' Strea~ line,9 lie on this surf~ce.' · . ~rove·d, ·

~
Integration yields th~· res\l.lt ~ .12, ;iuen u ~ -:2ylr 2, V" c2;;,2, w where r de Mies .d(sta.11ce from z
3 0,
log;,:=logy+loga o.r .,=ay. ... (5) a,xis, Find• the surfaces which or•. ort~og011al lo, strlam lines,, tho liquid befrigl
. By (1) IUld (4), /it;>mogeneous. ·
d.x _ xd;,: + ydy +zdz : Solutioo I Step I': To show that liquid motion !e posslble, we have to show that

0
3:,; - ' . zr2 t~e equation of continuity ( !~ +t.+ t~ "0) ls satisfied, . .
or 4d.x .; ·~ ( ZX, d.x ; Zx ty \2zdz ). ·Here au. . av a,; 2c 2v :( 2c 2x J:'.
+y.+z . -+-+-a=...<.., - - -3-. +O "O
X ;,:
ax ~ oz ·
.'V'.3 r r ·r
Io.~egrating; 4 log .i: '7 ~ Jpg c? + :i +z 2) :i: Jog b · -· ·· as,~. 10:i+ y 2. Henc.,e result I. ·
·;-J·~:\:····· . { .:~}--
i~.

~ -:--:;;,:t1',,'.i'•'}')).,,l~".:{•·,1>;+~•·r,···~~'),{im~;Z~'iiff,-,'¾h1ffU:i!:\~~~~~~mJffl:'~~•t•'•"'•t''• •·i:'"f•"'"•.,..,~,.
:~ :' ~~: :. '.};:'

·:::·•::::·: "!::.:-::?-: :, :~!: ;: :;·.


·.::<:•,:~;;:. ::<·:
• •<01'.>.:~i'·''
- - - - - - ~ - = - c : . " " 7 1 ' ' > - r i " " " I .... ,,,n ,n.,Q,o;;v;v;,:-,1;:~~~-~;"'!:':i~:?.(,"::~

-------- -T
·:-:,·::•,':•,,,
..
,, ,,.. ,, ' ',;,...~.:1:.i,1,<:• 1
'.•

1!i'H"111'!!Wl!Wli~--i.:w..,~.---~,.~,..:;,.,. ·•1;~· .. '·•l''l!QIMIUCll' v'.,~~:::.;:tcl.ll/l,!.;.,,~... ;,,:..,•• ,.;;·······~'""~"~.. "'""'-+~¥'~•-~il:"IU'.:'';,mi!M"':i'~''"'·:,:•:J,:r.i!,:•>:!i:!>1~1"-":.;.,~.z......~ft:.,,w<,•tN"'"''."""'.'''''~;· ;,,;,_,,,,:

;ii :" :lb, : FLVIO'bYNAMICS ~INEMAT1C:S (50UATI_ONS OFCONTINVITYJ ..... • ,! ;,


,~ ! '. . ·, . .. . .. . .; . , . , . ., ,. , . '
-~
• • . • • , I
:J Step ll : The surfaces orthogonal to st;eam lines are tho solutions o ! : : Solutia:,_: Seep l. 'lb show,tbatllquld motion is pl>sslb!e1 we have to show that
·~..
' !
;J u~ +!Jcl:t + wd~ .= O \ theiequntion of continuity.cu + 2.\/. + aw .. O1.natis!ied. ·
:I . CT .· c'l:,: . . ; · ctz . i)y az , .
'l' i,e,, ~ l'.b+il=-dy+Odz=O : ou ou · ow ·
\· ·r2 ; r2 ' · . Here a;+ a;+ a:;"
0 +9+.,0 =0., Henc, the result!,
;)
.
Cir - 3/
~ + = 0, i·oteg-ratln~ this log;= log a
. .
,
:~m~
: _Step II. To 1
show that the surfaces orth'ogonal t~:strealn linesar~ planes throu gh·

,or y,. a..:, ;: ,. a ;or: Ans. : The :re,quired surfaces,.a:re solutions of


*} . ; . ~ l udx+11dy+wd;:cO, i.e.,
~
:
This surface is orthogonal to stream lines.
.
Problem 13, Show that
. :.
I
.

Ij
,,; j_ + d · .,_
-u.1y.,.. ~ y+o.w:• 0

7- . =O,,.
'd:,; fil ,

I
i:,_,z , (xZ'-,r2):• . 'or :
I .,
f . .
.
u .. - ~ , v=
, (xA +l)2
; · ·
. (:-2 +y2)2
!.
, w=-L..:
, x2 + 2 :
· Y !
.

·
.
I ''I
: .' . • x
. lj1tegrattng log- =.log a
· Y
or
x
- =.a or x.r- ay,
are the velodty compOlllittts ofa possible liquid motion. ls,his moilon irrotatiottal?
·· 1 1 · ' ·' (Garht•al 2004)
·I i\
\ . · l / th · h · ,
I ' "t.1'f JS a P a \, _roug Z•a:t!S,
. . y
. . . : .
·1 Solution I Step I: Tq show tha~ the motion is possible, we have to show that : : Step rn. Tl? !f0'w ·tba!_y!)locity potential 41 does not exist,
·. (du 01.1 dW · )· , , · 'By def., ~- d~•-(u<k+vil:1+·wi:a)
the equ atlon of contlnulty 3,, + ~~:a;,. 0 JS sa~1sfied. . · - ,. _ (- toy d:e + tCX cly + 0 dz) ·
H ere .fu:. - ~ I ( 2+~2)·2_,,( 2+ 2-.2,:2) or cf,f>.,lll;>'d.:c-ux,:d:y'.,_M<k+Ndy_,say.
<1X"' ,:2 2 1 % ' " .. "' ;;. y l · ilM ' oN . ' . · aM " · .
.Here - = (J) • - = - (J), Hence , - "'gJf_
6'' 2 - a:i?) ' .' '' ' ·.ax . 'ay i1), ex .
( +:)' ) •
~- ~ l ' / ' ' ,,
' .. · ' '
(x~ +y2)3 • . • . .·· ,, _Therefore tho equatio11. ls not· exact so . that d~ ., o:,yd:r:· - <uxdy can not be
i, • i · · · . , , ait:ew~tad·so that$ does not exist. · · ,. .
-11 "-r-T7 [- 2;y (t' +yZJ2 - (xi - y2) .2 (x2 +y2)2y) l i • ' • . . .. ' • , • • . ;. • . . . . . .
~
(
+y) W , (;:
· . · !Il~?.lern _15; In !he...sleacly motion of homogene~u: liquid If the .~urf~ce~ f 1 ., 01 ,
,. _ J,L_ (3::c2-y2)
t 2 23
/z -;-. ~z, define the stream lines, Pf'T/VB that the most generdl udlues of the uelocllv
·' .
. . v< + :y ) , : )c,;,mpone/lls u, v, w· are '
· · .
· •
' . ·'
aau~~+~~"l 2~X\ 3 i(Sx2-y2)+(y 2-:3x~)-f0):0,
1
! !' F(J ")'oi/1.fv F(f ')'rli/1,l2),·F u ,_,o(fi,{'J)
\
X VJ Ul, '(i:-'l-;>) : ' ' '' ' I: ;· ' l•/2 :il(y,z) ' ·. l'? a(z,:i:). =~1••~ a{x,,Y). ' '
Hence th:e result I. , . ... . . , ... . : : •: ~.olutlon: Since the mob op issleady, beoce stream lines are Independent of i.
Step 11. To test the natu.re of the rnot10~. 'fhe:mot1on will be lrrotational ;f ;Tp~~efore'fl and/2 arc functions oh,y,·z only,
, auoy "';j;:
av ' 0, 1:a;-
au ; OW ow ~u
rJy "'O, qx - i)z =;_o
' !
j :. ,
:
fi,; a1,
' ' '
f2"• a2 ~ df1 "'0, d/2 = 0 ~
' ,,

J. ~!L 2.\/. ,,:.,. 4z (x2 _ 3y2)_ ~ {3,x2-x2).., 0 i . ~fi_& + 'ofi d.y + 'i!fi ch .,
.ay ax •(%2 ,+ :?)2 . /,:(2 + y2)2 ax. , . 9): ,az \· 0
~-£.!£. ~~-~ .. o · •
a.. a, . <x2 + 1 (x2 +.f>2
aw d/.1 · ~ ~ . O·
2,~ . .,
. ... ..
·
1lfz d:t+.f&dy+ o/2 dz =O
i1x
dx
.ii>' u.z
, -dr . di:.
<ix - az ,._ (.>:2 +y2)2 + ci2•:;.y2)~ = ' .J 'cJfi 012 _ °d/1 ~ , . 'il/2 "fi-~i!ii "'.i,fi il/2,_ of1 iJf2
Heace ~oUon !s !rrotatlonal. . , j iry oz ~z cy <lz ir.r a,: oz _a:i: By oy •c):,:
Pro.bllll!f'l4, Given u = - wy, u = 1llX, w =0; show that the surfaces intersecting the
stream lines orthogonally 'e:x(st a.rid are the planes through z-o.J:is, although the
· Qr, .1 . !ft ,,,~z
I ' 2 3
... (l)
1/eloc1t:y potential daes not exist.. ·

L .i

... ······· ..

, .. ~•·•-·-· ., .,. , . ', . '


.. , .... -~•.·s,~
' ·~ '~·"
• "· 't ·~ . Fl:UIO.OYNAMICS . KINEMATICS (F.OUATIQNS OF CONTINUITY)
a
.. •. cikh) . 0<11,@ · a,if1J2l Putting: the values of respective terms,
where · J i " ~ ) , J2=., ,( .. ), .J.3·•=:,;---{
. vy,Z.
• )·
· .. v z,x . o x,.y
But the str~ara Jines are given Gy· vkt 2~2 +wkt 222z - 226
u2x .+.
224
4.1:2, + 2kt [(l)2
b + (;·)2
- 0
) a/it ·b c akt c
191;=~.' ... (2)
I o~ · 2x' (
2TI · 2x )' . 2k ..2 (
u.-7 +~r.Y u+ lt ) +7t:
2k 2 (
w+7: ) •O
·
a~ t . b, .
. , and '(~)·
e
0 n compii-nng
. . ' (1' , ! u . J;
IJ' w = F , 'soy.
= Js Ji= .
i' ·· Hence (2) is 'satislied If wo take
i
~ ='11F, u =J;F, w =J3Ji', :·· ; ;ff "' (3) u-2x=O
t "+l=o
>' V t I
U) .. £ ..
t
o,
To detennine:the nature or Ji'. , • . .•
· In order to mnk'e lhe 1/qQid. motion pos~ible, the vcloci~y ,CO:mponents must i.e., if . u=~ ·w,._l:'. w=-!
t ' t ' I
satisfy. the equatioJl or cantiliuity, mi,mely. · ,'
It will be ajusti!icable step if the equation qf continuity 1
kax +~ ,. !!!!. =o h+~+~ t
ey i!z ' ' a,. ·ey a.",o ' l
•(i!J1 iWz .aJ3) ( cF cP i!F). •· i
F ']';" +ay°" +.~ + _Jq; +J2 <tJI +J~ az = 0
is satisfied.
This=>
· · I ·
Tx +];" 4.•=az-. = 0.
l3y thil prope~ty of Jacobian'.
0J1 ,.aJz 0J3 .
l
11
'
Herc
.
au av aw 2 l l ·.
•-+~+-=-----,,
,o~ i)j,1 0:1 t. t t 0'· .i
aif1j2l aF . aif1J;l aF · a !f1)iJ rJJi'• 0
Hence ---.----;.----=
ti (y, Z) UX g(z, x) oy o(x,y)" az
I· Hence c;i'i~ a possible fo;rii, ofbounda:Y s~rface,
Sib:iila!' Problexn : S}J.ow thai the eliipsoid. lt
c1F oF aJi' •
. , x2 . (. z2 )· i, .
2 :. .211 + kr' 2 +'2 ':'•l
·a; ay' in' 1
a ,k r b c . }
is~ possible form qf liou11dary surface.
or ..1
'a/1 oft · oft 1"' 0 or a(F,/J,/iJ ... o. I · -ii
'ii.~,' i!;y ' at 'a (,:,j,,z}· 2
Pr.oblem i7, Show that , •!..2 tan 2 t + x.; coe2 I .. 1 = O
· • 2. ·
... (1),
ofz · ?Jf2 iifz • . . a b" ·:
'i ax ' a;y az· i' •'is! possible form of boundary surface a11d /ifld an expression for.f!ormiil uelocity,
I
f:
Thi~ prov;es that F..'1,/2 are
not independ-enb: · · · (K,mpu,:·11004:Jlfeen,t 2003; Gharu,a!S002)
I
~
Therefore F = F if1, fz), Now (3) proves thilreq~ed result. 'SplutioO:: To 'show that F = 0 is a possible form of boundary surfnce, wo have §,.
to sho'w that ·'
~
·$olved'probiems rotated to bounda~ surface; a;;, · oF'
u-+v-+w·-+-=0
aF aF
Probiem 16. Show that the uariable ellipsoi~· · a;,; a, oz a1 · .. ; (2)'
_Putting tCe valueJ ofvarl<fus temfa, we got
·. ·x
~ + kr
2
·°'[(X.)z (z z,]
b' + c. = l . T ,·2 2
u ~ .t.an 2 t tu,~ cot2 I+ w,O +( 2.:, tan t suo2 I - ll'.: cot I cosec 2 I ) a 0
is a possible form (o'r ilie bou11dar;y surface ofa liquid. at an;y t(m~ I: a• ·b• · a• b2 ;,
' 2 2
• : (Kanpur 1990) or· ~ t•
z, ( + x sec I ) +,el·2 co.,z t ( u,_ y cosec
t t 1 ) ,. ,,v. i:·
Solution : Let i i· ' a.2 an ' u tan I b I co
('i
-iit F (x I yI zI t)' =• - ·x2-
ll2 k2' t4
+kt2 [( :tb )2 +(\ -z )2] - l "i 0 ;i i 1 ... (1)
Thus (2) will ba satis!ied ifwa tab
I i/
x sec 2 I • cosec2 I
':~,,
:!"!''
To sAow that F ..•0 is a possible ron:n ofbouridary surface! i~ ~$it;ough to prove·
C :
u+ tan·e =O, u-.)!cot7=0, :
iI
t that •/: ;,.:,;, .. : ..;·. .
aF,.:>i-aF:"' aF · aF
: •· i
.1 1
u::.~.,;, .. ~ , ' t { t.
"ji
u ·a!::1'U~7W-:fJ\!! 7iz +Tt ... (2) ~, .... ,·t.''. .:~",., sm t cos t..: . :.c'"
· sin l,~os .. '•

~
r'
!! --..
I: :·,·•···•1.,.,
;
·Ip. .,:,: .

. . i.,.rt"'O~l.l):,,ffi~,:.1p.1,•,·•1•·1~---h~•1'!'1'•r,
·,,:,,,:,\/,,. .
• : : ' . ' 1· '
0
.
I ·-·,-1KlRt'Ui,"\~~;Til;;•n~,.:.1,,~µ,.,-i:,:•,·,,r •l\~1-~~il{C\f{.•-i-cNi;i;:-<.;1,,~·1;~~;;.;:lmli3v4\"llj,lll'l\lJJl,l,(U:r.~m.r4,f~~~~1-.~•'IWll-M~f!1t:J~~'l"l'il1'J'HN""""""""..,.~"'"'rlHm-rQH'U1rH,s.;-:~t'
............ ~,itiftj!:'1'!1;::""t1·rt;";'!"',1t·-..,.,·:-:·"':"':'"•... ~ ...
-.,·.,\ ·, ,~;.• 1/,\•.!1'. · .. ~:-•,.,:,-;;n1,,n,.t .~,.,, . .-1,1 ......--1 ...,.1 ,!•,1•,f H},'!h-i'.:n1 j;f/j;~ hl\i"/;:· .,,iv;.;;.~.-i: .. -,.'•'',., • ... • .\" .-'• , • .,.. , .... , ... ,, .. ~- ., • .- fl" ··- -· ;H\IJ(! ,,r; ~~;11·,;,us•·~,-1, .. i'/♦ UH.IIUN/.ol .. ,,-.rr,..., !H' ' i,·r:., ' · ., •• · " ,, ' ' ,' '
t
:j
t.; ~

I
~l !i',: (;: :: '.~ •; ' , :1 i~-:-:· :?. :i · •:;;,;;:;w;-.. '.>;,-, ....,:,. ·-. ~.;:;;:,;-:•;;'.; :~,,; :, :'.•';_:'.:,:. ':·::,:;:-,::.::· '<·::·-::::~:· .';";:::;<: .

..... . . . . . - - . ~ ~ - ~ 7 . i w m , ' i ~ . ; . , .~fo ,¢1: .{,,r ,,~)~;,-m~st

~-:~~:·:·:::::~:'.~.
,:·.. -.·, ... :~ .:;.:....:.,:,·,.:~~l~:i~<~~·~
·. ' •,' .. ,; ·- .

•· ••~•HH<'t!'

i{i
'1-.,~
r.:t
. . ····-··-I
.· t. .'. . . . . . -...,.....-
:' ' ,wtw.•~1r•·, ·,i ◄ 1~•1•~~~.;,i.il.U~•• ..,••-"••".,:.;.1,...,,:,,_1~ ... ·•~n.!ilkW'!tl,tJ'rtU~l~:~,;r;"'",...1'i•••"''''.i

}1
1~ !I: :1.~ 'FLIJIO PYNAM\CS ttiNEMAT!CS (EOUAT.IONS 01' COt./TlNUfTY) .

}f ;robl_em 19. Show th.al all nicessary and suffi.ciint COllditio~s c.an be-satisfied by a.
~i ..-·· If; This will be a j ustifi11bla
I
step if the equation of con.tinuity,lnamely
I I
·v~/od,y potential o[lhefoi-m ¢ = ru:2 + l'l,y2 + y.: 2,and the bo1i~ding surface ofthe form
ax + ££
!; 9.!:!,. oz = 0 is satisfied.
oy + i)w i ·
f
. ·1 ! : Ji'=_ax4 .+b;y4,:i-cz4 -X(t)"O,
Now
. ' ' '.
cu +~+:OW ,._·...;...J.-·-1---:--1--'+o=O.
a:; ~y .oz . sin t C1Js t sm t cost
'
. wrd-e X (t) Cs a given function oftim~ and <1., ~. y, a, b, c arc iu{tab'!e fur.i.ctions of the
. tinie.
· Solution : Let
'
· ~ ,. ~ + P.i + yzz
· · ·
••. (1)
Hence (l) ls o po~si~le foml'ofboundary 'surface. F (x, y, ,i,I) ~ a.x4 + by4 +c,z4 -X (l)"' 0, ... (2)
: -rJFli!t i Step I. .To prove that¢ satisfies all th~. nec(!Ssai-y conditions (i.e;, equation of.
Second Parl. Normal velocity':' . IVFI cont:inuity) , ··
-(·2: tan t t- 2! cot I cosec2
sec 2 t) , or.
'fi2+ &2+ iti"'O·,.
· a• · 6" : ex d,)I . az
f (( 2,: tan t . + ~ 2 )2 ( z
1:,'i. co.t t
)2 ]11'2 . Putting the vaiues oii-espective terms, ·
02 · 2<1. + 2fj'+ 2y ;,, 0 } or o. + P + y " O.
, I
... _ ~1, 2,. tan t se,!, :- a2t cot t cosec.:.!l Ans, 'I'he ,·eloq~ty potentlo.i I!> has to snt)sjy thl~ coriditlon. .
",' (q 4.,:2 tan•. 1 •r a' y 2 ?ot• 1)1'2 · . Step lI, _To "prove .f.. = O satisfies· the condi_tlon of boundary S}Jrfaco,_ We k.oO'w
fuat · .· . ·.
Problem 18, DeterminJ th• r,cstriction 011 /i,/2 ;{3 l( i
x2 : 2 ... ,2:2 ,'
., u=-!1!,
ar ·v=-·~ • . w=-'.£1
By ;oz
7' fi (1) +. ~ f2 (t) + 3 /3 (I),. 1
.a .•,·: c , b This ,; ·: u = - 2cu, v "' - 2P,,, w =:., 2yz.
Is a possible form of boi<hdary su.riace of a liquid. (Garhwal 2003) F' = 0 will be a bountlaty surface ifit satisfies the condition,
I .
,;c2, ·y ., ·z2
F"'-;:; fi (t} + • ( (!)+-:;fa(/) - 1 "0
. oF oF i'JF aF '
Sol utlon : Let
· a•· b•
2
· c•
... (1) U 'j; + V d,)' + W Jz + af'"' 0,
'I'o·show thatF::: 0 is a possible form 9fboundnry s11rface, we have to prove that Putting the values ofre3pect!ve terms,
(F"' 0) sntlsfies the condition - 2u:c. 4ilx 3 ".' 2pj , ,;by3 _: 2't:.4cz 3 +~·1a• + y 4b' + z4c' - X' (t) = 0
· -+u.-+v-+w-=0
ar · • aF oF aF • 'or . x4 (a' -8tlil)
.,., . +.,..4 (b'- I 8j3b) +z' (c' -Bye) -X' (t) = 0
... (2) · .... (3)
il1 .i.lx . . cz . az
Since (2) a.nd (3) both have to hold for.all points (x,y,z) on th~ surface hence
Putting the va!~es of respective terms, they should be identical, Comp.aring, we. get ·
~- ,l z2 2x ,& ·z,. · a'-8~ ~ c'-Byo~ X'(t)
2 f{ + 2 fz' + 2 (3' + ll 2a fi + V b2 f2 +•C
W 2 fa = 0
a a c . --;;-= b :" · a ,. X(t)
2x (. ,:fi'. ) _& · , zi3' \
( ' . Y/2' ) 2z . (4}. (5) (S) (7)
or 2fi
0
u+v
. l
+ I, 2f2, u+-;,;-
,"'12
+zfa(
C ,
w+z:r3 1=0. B)' (4) and (7),
da
dt- ~aa,. X. dt a dX

' ' fi' • (2'


·lfwe take u + x zr = 0, u + y-;;;,- "'0, w + , f{" 0, then (2) Is satisl]ed. This
.' z • , ,
,cir =$1ldt+'dXx·
I I l ~12 , 212 . a .
wlll boa Justifiable stop if \.he values of u; v, ui.satisf); the equation of contln ~ity.

!
. .
i
+k aw· o
'Qj'+ az "',.
. Integrating, log a= lo~ X +J Scutt.
the volues,:
:1[1i·· f2' r{]
' Similarly, log b'.= log,X + f 8/3 cit, . by (5} nnd(7)
-- -+-+- =O
'. 2_ /1 ' /2 f3 . )

lntegrnting, log f; fd2 = log c'. or' ft f2 fa = c, Ans.


I aod . log c -~ !og~+:J_sy . b:,, (6} and (7), ·

\ '
.. ,. I
DI, ;..._-.

','.,I~•'.': ,;'.:~l"'fl{(r,.it\•;;,;_.,'.,~>,<'.•:~• ,:',',I;!•::,;•,'

'I"l',i.•,7.' •~••,;,1';",1"\',"J',.•,'

_.,., ..~;.-,' •, - .
,,,•.•.'•••M'•H• -'' I,
• ,,,..,.,.,. · u "-"•'"" ·.Ar .. .,~..,•• , ••..,,. .. 1,.vr,,,,._,.,,._, .. ,,,,.. . , ., . , ... , ~~; · .._,._.,,., ~_,,· .,. ,...,..,....,.. · •.,.:-.· , ~ . ., •.•< ,-.-.,., ,; ~ •• ,_, ~~-~~• •,. ~~

\'; ., ;;:.~;;, .. ~":t{J;~~;:; !~i l.):ii'nn-1.:P1.<1J;-•v•,,--::r'l."'".!{·, ,}~;~1•,1,.,,,.,,.-. ,v,, 1 •H·, -,, ..~,. . ... , . •

~- ···~- .I
·?1 ' ·! iLUIO OYNM,i'1cs KINEMATICS Jl:OUATIONS QF CONTINUITY) "''""I"'
·J:J~: ,,
·~,t.cx+p+y;,.Q :!
TI1e surface F = 0 wlll have to sotisfy thc~e cQn9ilions for tlie ·possible form or u=-½,7- u=-if
boll,lldary S\lrface. ·., ,. · · This will be aj)lstifiaple step lfthe equation•.ofcontinuity'
Proble1-1 20, Proue that a. surface of the form
, ~
au
, ·a:;+Jyta;=O
ii aw
4
w: + by4 +•ci,. ?' (If, o is satisfied, Putting the values,
is a possible form of /)o'undary surface of a homogeneous liq1dd at 'ii;:ne t, the uelocily
pot;,mtial of the liquid motion being · : l L. 1 f
-z·r-2:~+ 0 =0
• . ~: r~ - y) x2 + (Y.,. Cl) Y2 t (Cl - ~) z2 i

.
,
where X, ex, l), y are gh-en (unctions of time and ?J, b, c aresuita.bl~(unctlons of time.
. . . ' . ' rkanpur 2ooaJ
or
( ~ '
· !ote"°ating, log($= log const. or(~= const. which ls
!if+!!!=~
. ;;fvon. Hel)cc (l) is a
Solutlo1;1: Proceed as above, ! pQssible form of boundary surface.
Her"1,equation of continuity "" (P -y) y :_er.+ ex - l) "0 i + I '.:
Condition of boundary surface => I , Sci,Jvec! Problems relate~i' to iiqua·tlon of continuity :
/ ~·.
I I Prbble,m 22,A m~ss offluid is in motion.so that the lines o(moiion lie on the surface !
loga=S
f (~-.y)dt+lagX,
•,
j of d,o-axial cylinders; show that the equation. of continuity is, ·· . ,
i!a a1 iJ ,·
''at + r as {PUa) + az (pu,) "', 0,
''
'
l
l,, ,j'
J
log b = B .<Y ~ rx).'dt ~ !og;., u8, v, are uelocitle, p~r;endicula'r and panaUe/ to z,
. , . (: t ·;

Kanpur 2~00) (M~en,/ UNJ3;

f
, ,
. Solution: Oonslc:!er. a poil:lt P whose cylindrical co•ordlnatos,are.(r,.8, z), With 1
log c"' B (ex.,-!,) dt ; log X. Pas ceDtrf, construct a parallelopipi!d with edges oflengths dr, rd~, dz, Since lines
of motion Re on the surface ofthe cylinders hence the fluid lies on the surface of tho
• 1~~bble1n 21, $how that cylioders, It mell,lls that there is no veiocity in the direction of' dr. Equation· of.
cootinuity gives
Y('' ,fl v2,
'' , -2 ( (t) +"z ~(I)"' 1, a ,
·, a b · . , at (p dr. r de. dz)
'•" where f (t) ,p (t) =co.,st. is a pqssible form of the boundary surfa·ce o(a-0iquia'., . a · a· · · a · ·
· .. , \ ;1 , (Kanpur 1993) = -[ dr a; (p. o,..r de. dz) +rde ;:ae (pu8 , dr.. dz)+ d2 a; (pv, dr ,r, dB).]
' 2 2 , ,,, , . : .
,., (l) £Q 1[a . a
Solution:Let F=¾f(l):+~4>(1)-1=0·.
· a .b ·
. , •
, I I
er at+;: ae(PU~)+raz(PV,) 0
To prove F = O.is a possible form
of boundary surface. For~hif have lo prove ~e or i!!l.
at +;:1 a0ci 1i?.Ue ) + az'
·ci < l O .
P,ll, "
that ' .. I . ,
u i!F' + u oF + w oF t oF = o. 1
... (2) Ii Pro~lem 23.I(euery particle moueso11 tlie ~µr(aceo(asphere, prouethat the equatlon. · ]i
. ~" i!,, oz at :. ofco~tinulty is '
Putting
. , .., .
the vnlues, 2. 2 _
i ,
i
1
1
11
I!
·
·
a cos a,+ asa (PW cos 8) +' aia ,pw' co~ 8) • O,
Tt' · ',
I':i
u ~ ( + u ~ ~ +,w,O + ~ f ': + ~ <i>' =
0
.! \ i,
or
2x· r
-f(u+!L....
2 2
,, ) ·.?rt ( .,;..,
J:3_)
+ ·2 v+ 2$· "O,
· r;
·
w, w'· /he angular velocities
,
. of any ent in latitu'cls and /Q11gitucw respectively,
. · .·
. air
p being the density, a,.q, the latitude and ·/ongitude respectluely pf a11 · e!amen.t a;d
.
· .. (Ga'rhwa/2001)
1:.
1:
,a '· f b · : Solutioll : Step ·I, To determlnb the .equation o( continuity ln aphcrlcal l:
j: If take 11 + =. t;:." o u + l. !: ,. o, thee 'the condition (2) will' pe satisfied. · CO•ordioates, Consider an arbitrary poin~ whose polai• co,or~ln»tes are (r, 9, 9), With i
., · we · 2 ·f ' 2 ~ ' ·· . . • , · Pas centre, construct a parallelopipod wHh edges of l~n.gth, dr,"i' ({(l, r sin od$, •:
1 Her~ w~ get · f '.,: ;)
1
1
I
i,
. I , I i·,
';
~- '•

l,
I. ,
,,
,
' '
, '.
, ... · '
L'
i ,
, . ,
,

, , · •V r,• ~~ ',W
'

AV,f~~:..:•.'ff",:"'f1!~:....•""" •"'"~*!"''l!'(/'J)ttfJ.it•.'1•'1•f1
.

i;
,, ..... ,.,, .......;,;:~!f:!!i;:,. •: ..'.·,.. ,:~•_, ·:·, .... r .. :: ·,;:. ,·::,\iil\l!lm.';';;;O::·;:,; ·~1r::::r~•:".f.!m!3JW.ll"fll:!!w..!l!:'1:!'t:~'1lr.fl~-JP!f5f/'l:''.'1r.:1!'m1:·"•':', i•:·, .,.. ,·,~·,:·,,1,•r,•7•·T ·:1."•::ir,,!%\;;,;~:;1:;~~~,:~~"!!ll~'!f'.'l!ll!111,r.>/f•t .; ~, 'fl .. · • - . ,. ,

;,:,•,,, .-., ,,,_.:, ... ·;•:_.:~;-:'.~'.:. ',.: <:~:;:::,::~::,•·· ... ,.. ,,,
•-t{,' ,,,,,,,.'

:,'.,:aijl{i;tf;;fi-~;·,,T•;• •!(•,, '.1,, •:\\:l~•; ::1\.-~'''!' ,;-:,,· ·~. J 1 ··<1:i:1 !l!i';;:f·\! ·~· ;, . ;, , : ..:;,.'•!-, 1· .,,1.>:.C!--J>'.1'.tt.S,·•·:-,,1;:,;:.:;t•;::::·: .•.-~\·

>i ii <J..Ulr"f'l"P"';!,i,,,{t,.,,•.r,l ,.,,,t,1';;_~-{':};:_,

·,;:
~. '
I,:
: ·:·::~·:x:::·;·: . ; .-:·
·•~ :· ,,;•. , .;.;r:,·",;'_,,: ."

i::i
f:j
1•>:,,~ttt,~J«i.ti,i:,)~,•O~•.•,:w,r,.\!l;,M;~;.;;u(~·.·u.. ~-.v.•••;, ,.. :~. •.• ... ,, ....... ~~i'.:4-.e::;,. ~!!.1.'-~ ~·_:, • :.1 ,.~::.. J:>~:·:.:m1:;1.:t!. .. , . ,.... , , •. "., .... ,.,,, }f',.:,,r,,-.:c,;r•f~ ~'lei'.11r.:.,w.~~V1lf-()~¼t..iWJiH.at:..; .. ,- ......... ..:• ., , ...... , •... ~ :1~t~.,, .•• ~ ........ , "' .,•, ,s,•.~•
~:
~:, ii :
~;~
/<

....... :l..C 'FLUID DYNAMICS I klNEM/\TICS l§'OUATIONS OF CONT1NU!!'i'.) _ , '·, . ~

·.~ i'
I Let q 1, q2 , q3 be velocity component~ atPalongdr, r, da,r kine d$, respectlvely.
Th• equatlon ofcontin\l!ty gives
i1
i ·, ~otutton I Step I. To ,derive the~;uation ofcon~!nuHyin e~herical co-ordinates.
(Here write Stoti l of Problem .23). · . .
lI a . .
·
: ; 'step II, To detcmiJne t.he equation· of continuity In the required case. It !s given
iii (p dr , r dG . r si.n B ."J$) or
t!ia't lines offlow lie on tbe .surfnces cones and hence velocity p'erpendicular to th~

I
•I
"' -[ dr fr (pdi ,
,
r de', r sin Od·~) + r de r
.
ta (pq2, cir. r sin ed~)
a' . ]
surl'ace is zero so that q2 "0, Now (Ubecomes

£Q .ll. 2 .....Ll.
(}/ + ,:2 i!r {Mir ) + r sin 8 04> (M 3l - O.
-
!· + r Stn ed$ r sin 0a¢ (pqJ. dr. r de)
l(eo1acm1? q 1 by q,, q3 by q., and q, by w,
Simplifying, we· get
la 2
fr
~ + r 2 8~~ G [ .~in 8 (pq 1 i-2) + r fe (pq 2 , sin (pq 3) I= O sy or
+ ;::z a,: (pr <1,,) ,.
iz.e. a 2
{pq,..,) = O

cosec €1 a
a (pq1 r2)-+ rsiii'"ii i!t + a,: (pq,) +;:Pf/,+ -r-a;;; (pq.,) = O,
or ?.a
a1 / 21 a,.
r
1
asa <pq2.
. • e) 1 a< J
sin +~a¢ pq3 " 0
·m
. ~ J'

This is tho equation of continuity 1-n spherical co-ordinates. P,dblem 25, I/the lines of motion are curves 011 the sui-(aces of spheres all touchi1t!;]
ihe :ry•plo.ne at the origin 0, the equation of COl'ltinuity is ,,
Stop II. To deterinloo the equation of continuit:Y, in ·
the r~qulred cnse. : , · ij :· ..
· . ~:Sm
· · e PE. ~- · o2JQlD.
at t ~q, + sm oe +PU (l + 2 eos ")" = o'· ,.·· .
lt Is glvon that fllild p!JJ'tlclC/S ~ove on the surface of
ephoro, honce q 1 ,. 0. ! · .. ib(lere r is th.e radius CP of Me of the spheres, e th.! angl~ PCO, u thi'u~lcicily Ill the
plan,e PCO, u the perpendicular uelocily, and <l) the inc:Unaliofl oft'he•plane PCO lo a
, To got tho oquaUp:n or cont111ufty in present' coso, 1~0 rl.Xed plane through: z,axis. · .· \. •
bavo to replace e by ,90 - 8 in equation ( I) and de by ·j. ;Solutlon: We consider any two consecutl'1e spheres with ~ Q
d (901-, &) • - dO. 1
' . I cen t'res C and C '. . .
For OP line makes ari angle so-,, e ;.,:ilhz-a.xis. L.c_t (;P·:'r, C'Q .. r+or, LPC0,»8,'.
l'lg, 8
· ·. (l:. W, ,i, !Cll' , Then .. CC'=or, · OQ;:,CP+PQ .. r+PQ

. . ·.. d . •
"q2" r(l" gives q2_,. r·dt (90 ,- 9) = - r8 ~ - rw . b2 i, c2 _ 02
• cosA .. c
'\ 20
"q3., r si~; 0 $" gives ► 1·

a2 = b2 + c2 ~ 2bc cos A
r'lfJ "'r sin (90 .:. 0) f"' (r coi 8) w' i :Applying this .forrriul~ in aC'CQ, Fig,D
Putting theseValues In (1), 2
.1la o· • . l •. (' i'!
ilt + -f: r sin (90 ~ El) : - aa' p -
I ( ).... ·1) . ii .
cos' 8 . ,+ r sin (90 - e, ~
i
cos v
" , o Ol'
. · c·q "C'c + CQ ~ 2c·c.cQ cos (rt - e)
2 2
2 2
+ (r +'PQ) 2 + 2Sr (i- + PQ) cos e
l"tll (pr w) "
i. i (r + 6r) rl (or)
· Qe 1· a 1 a • • Neglecting Pt:i,
or ) ell + cos8 oe:(~Cll cos 8) + cos9 ~ (P cos 8 w')"' 0 · r or ~ r Sr cos e =PQ (r + or cos 0)
' a • ·a: c1 : • ·or
PQ = r or (1 - ,cos 8] (r + &r cos 0r 1
or 717 cos O+'ae :(pw cos OJ + a¢ (P. cos e w')" o
·1,., l:,r .
1
'ibis l>s the reqyjr!ld cqua.t'lon of continulty,
, , I '
.Problem 24, lftli'e lines ofm.ot~on are eurues on the sur(ace:o(cones havlnc 1/,eir
• ' :
= Br (1 - cos 6) ( ~ +;'cos a
''
vertkef al the origin and the o:cu of z for com.0011 (Uis, prov,e that the equation of
.= or (l - cos 6) ( l - ·or
- cos 0· )
'· r
: con/In u1't:; I, o' :. l3r, (1 - COS 9)
• 2 '• . ' 2
C

;t
I £1?:+l..(p
at a,- q,)+ f:'!lr+~.E..(rv,)-0:
r r aw ..,., ~ ";· ne'!lleCtlnr;, lir and its higher powers.·
PQ.,(1-cos t~
. (Meeni! 2002, Gurhw.al 2000) i,;
~~
l,•
·Lt -
~1
'

,.ii
. ~:

' .. ....
·,·~-~··, ·, ,,.,-,.' •~ ....._~ ..
, •'•, \,.,.-:~.,.,"-...,~-'1.J..;>'•i7, ,•:.r.•:~-~~•.·.·,·' '• •

'i, ..... •

1
, :· 1FLUID pYNAMICS
,I I KIN.EMATICS (EQUATIONS'OF CONT!NUIT'I)
-~
Sin.ce the lines of flow He.o.n the suif~c~s of the spheres, hJahc:e velocity along
PQ is zero. Now we. consider a· parallel.opi.ped with edges oflengt};is (1 - cos 0) Sr, i.e., .£.
r oO, r sfn o~, ~he velo~1FJes alo.ng these· eleinents .~re . o, u; u r~spectively. The a, (pw··ds) = - ds}: (pq W)
D3, ·
Q~
equatbn of continuity gives · · or
a . . . . ·.. !.!12!:ll
iii
+1-
as (pw q) = 0
~ (p {l - cos 8) dr, rde • r ai.n ,e dq,J lflir, IO
Prooli:im )!8, A m.asf offluid moves In such a ;,,ay thal each partii:te:desc,:ibes·a circle
(1 - cos O} dr . (l '~ c:s S) or (p • o . r d8'. r 'sfo 8 in one plane a.bout a fixed axis: show that tit~ eguation of continuity l.s
.. . ·. a . •. ~+~(pw)"'O .
l
•·I + rda r ao [pu (1- cos~) rlr.. r sin 0. d•lf • . I •
where w is the angular uelociey'ofa pariicle whose azlmuthala.ngle is 0 at time t,
+.r sfo 0 d<t, ---:-2--
, (pv • (1 ;- bos 8) dr. r d6))
rs1n 8 ,;,¢) · · • ,
Solutlqo I Consider a point P whoso polnr co•ordJnahs oro tr,'•aj. Let there ba
an elementary area r 58 lir,-whon this a.re~ is' rovolvod a&out 0/then .It describes ii.
or f I
+ .· 1
r2sin 9 (l -, cos 8):
· · [r fo tpu (l - cos 8) sin e} + r (1 - i:~s; e(
.: . ;,. ,
i (fv) = O
,
J elrelo so that velocity OP.vanishes, B;y equatfon ofeontinu!t;y,.
.r I
' .

... · a £e. 1 a. 1pu.( · a·i +~


a cpuJ = o. ·r
i)t + (l· -.cos,S) 08 · 1 - cos~'') sin .,
or ram r6tHr 1
. @. .
ae pu) +
i) i) . . . .
or r .M 8 i)t + SJ-l). 8

For (1-cos 6) cos 8 +sin2 a= (1-cos 9)-[cos 8 +1 +cos.a].


!
i)~ (pu) + p~ (1.+ 2 eos 8) = 0.

·~
~J
(9P
C

Problem.26. The partiples of a fluid move s:,mmetrica,/ly in spacf regard to a 0 Q

fi:xe.1, c:,mlre; p°roue that _the equat/9n of contin.uity is · Flg.'ll


• £2.+,.·£!!.+.E.l.(, 2 u•·~o. -- a (p roe llr) ~ - [ llr a,:a (p , Q. • r M) +; 68. ra8
a (P q, 6r) ]
a1 · i!r ~ i!r ' ;ii
iuh.ere u· i.,,' the velocity at a dista~ce r, (Mccrut ]91)2)
Solution, Here first pro~e : or ~:¾[ O+~(pwr) J=o. Forq=rw
£E.
at +.11- (p qr 2) = 0 a a·
r2 or
... (1)
or ft a8 (PW)= 0,
(This is equo.tion
,. . •.: Cose 1, Page 29).
(l) of Article. 1:~2 ,
Put q "'IL in (1); then· . Problem 20. Shaw thal in the moli~n of a (1.utd if! two dimeM£ons-ifthe current
£2 .£.. (P u,.;>) ~0 ,I (x, y) are e:<pressib/e i,i terms of initial ca•ordinate, (a, b) and.the time,
co;ordim;,te,
at + ..!.
r2 .. ar . I I thSn the moti.dn. is irrotatloh.al if "
,;,, 22 (i-Zu) £2 + p ~ (ur~) ]= 0 .l l '. .. ·a r±, "'1 + ~ o
ot + ..!.[
r2 . i!r i!r , ' o.{a, b) o(a, b) r. , (Meerul 2003)::
~ · Qe. + u £2 + ~ 1_ (ur 2) = 0. .I i ·Solutlo.6 : Let u, ·u be velocity components parallel to the a.xis .of x Md y,
re~ pectively, Then ·
at or
Prol:>Jcm 27. lfw is I.he area.
r2 'ilr

of cross secliori ofo. stream


.I
I
proue that the X "< U, :/ = IJ, oa "' au
OU ox dil h .
CX a;-+ i)y Oa
.I
equaticn of coritinu.ity is · Observe that
a a .
at (PW); & (ptoq) = 0
a (x, x) ~ .W....u .. ~ ~ ~ "
i)
o(a, b) o(a, b) a(a, b) . • a(f", b)
OU
bu
~ ri; + -~
au I .~
ob ii i
Solu tfon : Consider v;ol=,e bounded by the· cross•secti~ns through points P ·
ani Q where Q wh~re-Q is .at:a distance ds from P. Mass or the 11uid within the ~- ~ 2-)!
oa ob / ila ~ ·I
' .•.. ', ''" . .
I •

of
volurno " pw ds. By def, co:ntlnulty, rate of generation of mass
' =.excess of~o,w_i~ ~ver·now this volume. r . ,-,, ;
!I
i
g . ,
I I \•., ' ,l• l•':: \I '
~~~
, •l
~ tt"N'ft-~:•., -••n-. ......,.... ._...~..,nu;4L&.1.n.in_l!=--•=u=•••"jllli"""'-'•,.,,t,)"
!
;'••\I'• ,,,, • ••)' -.1;..••1
, ..,,tfl,~,"1'i
,,,,,,, •)/•
, n1'>-<L!1•:,,
;,,,_" ··;"t"-..,.•,, ; ,ii•~• 1:,,1••.1.f~"'"""'"'t
, • • •,,,},.,11uJ;,;.,I,)
"•~~r:" ' - "!"'"'"'"'":"i""•~M-f'•"1!:-,,,.,?Wrn.."'!-.;i~~1"-)-.,
,',•11•'n,;,~• ... \\,~iill•l;(<41,,, ~• ~.,;,;.ti;,}; ,1 ~ .... •~lVK•'lt,,f~~'~Tl,~:J/~11,;.
~<", t• ;>, , r,'.. ,\';-,, ,,"ilh...,~:,;,o\J\ol~"'--.:...,.
, , " • ,,:~.;g3:~~n,
~, :-.•~::,; ta.J.H"~a~
cr:;n,nrrr. ~,.,,·,~;nliHrt, • ;v.u .. ,. , ;·•r: "'·...,-..-.,., ~.. .,.~----:-<ll"l"'"""J"O"l'"""'"":T"''l't'!I . . , , . ~ •~

·,;.:\:,:1t:s;,_..: .. ,.,;,;,,:, -:;; .. ',!:•: ~~;' ' ·«:·•::i;,;· ,;,•;:,:,,'.·•,' '.: ·:: '~):,.;' .·.•:r::-·::::::'.·

I> 11MUJ.¥.¢!h5.,,.t.. '.f,\,!$1,,,J.f.,,\l..-.~<5',~"°~

~ . ~_ ....
";'"'.- ... :--·
:'.:?£::;:?:::·
.· . ' .. .'.:. : .. :. :
.~.--.,!.,..;~-:...- ..

,.,,~,;:,;~;:_.;:.,,,.. ,, ... ,,~~···•'"" .;_.~:~·~:::~


.. •~,-:; ;,, ' .,,, ···•'" ,.~•'

,-:,,;:,:,

r~l'..,\\'l~t'lJtfWr.c-#\"'l't~•,_,,'f<"O~".o.....~~:otA.❖,..,,..:..,, ,.,_._\;.,.:,,s~. , ., . ,. ;.', ..~~~1. ,. ,:::, • ..:;, ,.,'(1.:. .,/,;.;;.•.•··~::•.!,..; ... ;:;::.:;.:.iu.-:.-. ,, ..• ,,.,.. " .•.••'. ,.,, .,,~, ..... ......,..... .,,.tt•,,t'Jt.,.,~~r;,;.,1;-~.....J.1-\i:....,,:, .. , ~ . ; . . . :·~.::~~i:::u:: .. :~u..-n::!,,: ......... ,... , ... ,,.,t..••i!\•"•'"
ii;
KIN!!MAT!CS (l!OUATIONS OP CONTINUITY) 1:f{
. ·-~~ FLUID DYNAMICS

.,au~-
oa i!b ill, iia iib
cu+av·£>!.~fu:fu!
~a <lo
::·(<l.·•.x11.)2 + (~)2
ii),
2 . ill a et i> .· . [;(~'l
+ q ,;, q: oy ~ (Ol + ax 1),: (~) + 2 oxoy ~ +,·(6)2
. ax
]. 2 ·

,,: ax ( OU ox +'QU .£t.) - i):, ( OU a. + au 2l.) . .· ..:2[(11)2+··(.2:1)21 . (7)


, . k~ ~·• Ba ~~ ~;~ ax i,y a,:..2 "'
.
, ,
+£.X.(.?Ji.
ob ax
+fui:R.l'.)-k(.fu!~:.fu!.£.X.)
ii), aa a,a ilx ilb cy ab
Squaring and adding (3) and m: .
.; .' q2Lr(~ax )2 +( £!1.ay )2}=( 211 0)2 +(.l:!.·)2 }"'(~ )·2[( h:)2 +( ~ )2 l·
a:c ;:ir(
oia'u:(E!2t._£:Ih.)+E!:'.(fu:.Q:f_Bl:E£) : 02
· ay'( iJb aa aa /lb ax <lb .aa oa ,oil L a,.2 . a,, il:t ay ax a,y . al •
·( au au) i(.w:l ' · 1 · • + 2 ?i~[ .-21·+ i!:i a?t, ,J·.
", a;- ay o(a,b) ' iJx oy ox ay 1zyz o,~ ily
~ 0 'iff £!/. - OU = b
OX oy ; B.ut ~
ax2 =- .&
&i 2 • H~nce the last gives :}
or iffmotiol\ is irrotation al. . ·
1 . . ., . r?j

'
Problem :JO, lf q is lhe resultant velocity at My point of a fluid which is mouing
. q:
:2[(.&o:c )2..• +(. ££oy .)2] ~ (~o;,; )2[ (fi
a.-:2 ~2) +( cb> ax )2 ]+. ( ~
11 )2 r( .ft. )2 ~ (. ~
cb> L il;i; ii:>' ax2
)2] m
•irrotalionnlly in two dimerisionf, prove that ·
t._f~.2:.t.'
(ful.)2 (~)2 2 . .,, .. . +2~~[ ··~2
<ix · + oy,· "''l q 'v (Kanpur 2002) ax iJ,y fl;;: oy 11%2 o.x i:!y J
. Solution
~ontlnufty fi
Since motion ls· /.rTotatiol)al, theref~re ~ exist~. Equation of

9 2$ c
,
D or ·
..
7
v-2,i, =,;),,
a2,,. .
+ =0
.
, .. (1)
I

Using(2),
~[(~J +(~ JJ[(~f +(tt

n:Y.
.
J [,(t1)2.+(A)2·].
2
q .. - y'u(ves q2 ! ( ~ f +( ~ )2 ". (2) 11zr.12i)2l+.(2i)2'= ,q
l ox · a,, . a"2 . ii), ax
~· Dlfferentiating (2) P/lrtial!y w,v, x and>' respectively, or (fq,)2.+(~)2
ax ii:>'
=(~)l!
ox2 ~(..fi.)2
. a.r ax .
.. :~L~d ~11
1· ' q;ax ~ ox a.x2 +. iJ;>,'o.x oy "' (3) Using this In (7),'

. (~ )+c~ r+q 92q= 2[(~J2 +l(t n.


l,
'. laq· a~ a · 3$ a 24i 2$
9: ;zy;,,, ax ay a; +.a_;; 0)/2 · ... (4)

Aeaindlfferenlintink (3,) w.r.t.;,; and ~4) w,r.t. y, we get or. . q 'v29 "' ( £i)2.+ (. 'Qg_ )2 Proved. \Ii
j ox ' oz
I ({&)2+q~=(£:f)2
i!x 9x2 a.x2
+g_t~+(J:()2 +,ql.J:L
ax QX3 ilx ii), ay 'iJx2 d.)' "' (5) Problem 31. The velocity components for.a two•dimensiorial fluid syst~m can oc •
given in the Eulerian sy~tem o.r u "2x + 2y. +. 31, 11 ~ x +y + ½t. Find the displacement
I (Ell.)2 +; &=(A ,2 +,Q!...tl..+(~)2 +~~ '" (6)
, . a:,, q a; ax oy ) . ax oi ox a.l ay a:,,3 ·of a fluid partia/.e in the Lagrangian. SJflem.. . · . (Kanpur iooo, 2005)
Adding~) end (6), Solution: . · u,. if+ 2y + 3t "' (1)
1,. !~ + ~:! l + ~ ,i f ~ +' ~ l
· . · · . u,i•X'+j·+'½t'
)2 ·+· 'l· v.2Q ·,.· £!
..
,.,(2)
( .e.il. ).2+ (
,i:,, a,, ~ a,.2 a,,2 ilx il.~ \ i)x2 . · o;,2 ! 1
• d.x ' , . .'!!:t, d · .
"."Ji• u"'»",Dy"' dt • .7iimD,
. ' +
l(~)2
. ~-~z
('s&)2 l 2.hi!., rly
+ 1,y2 . t
u "'.D:c
By (l) and: (2), ·. (.0 :..2) ~-:iy,; 3t
,

., .. \3)
b." - ~ , we ge~ i . :, .. . . . '., (D - l)f- x·~ ! t

' . .. ,. ,...2. · ... ... (4)


Using (1) an~ noting that'
. .·. · 2
. a.>:. O.Y2
,, 1.
,, ............ -.. '· '
.r l''

\
I .·>:,:, · :,•,:::,,::, ,1:g1;•1;;,;,;mt!'Nl.iS!fll.lHi•!t!;Jmm:1<>ll<•;.i!••'::,:,·,:,,·,):t<>,mH:<"l•!'"'H<"'1'•''''"'"'"''''"''""'"Mfflffl!:1ffltt<>tm>t<t<>'..'T''''U""""'

. '
gom'1.',~~mJtJW'!,t'm!'f:"ells!\;lfflllllllr.;}lrl!/;;.1r11:.r1mnw:1•, i:-:\l!!lfflffll:lf.U!! l!mnmimm:!!Hl{i~lfflt!\l!l~lffiRll!Ml!l!Hll\tl<li\1\ i',(\,'Il>1Hf •'· •,•,:,• ··: ,, ·: : · ·
1

·••• •. ~, .• -=-• ...... ,,..-.-.,,

,,,•,n••-.·•·•'
. f,~:,•r-,•~n•\'s'.(,:> '•;.,''.>,.\I
...•'.,q•,'··•· '-"·'' .... , •• ,,.., .• -1,,,, •. ,_

;).~;:,;0,;1~"\~ISt,~1,~>~P',~•;i•,·. ,.,•.,.•.,

•''
'-;t,·
FLUIO OYNAMICS
KIN EMA TICS (~QUA TIONS OF CONTINUITY) ·
~6
Operationg (4} by D ~ 2, . I·
Solving thes~ two, w/get
(D-·2) (D:.: i)y- (D.;;; Z)x ':½ {D "- 2) t 0

c1 .J 2.Yo - xo) - .l.


. or <D 2 :.:.3v +·2,)- <D-- ~l x = ½(l·-qii .:. (5) \ 3 · 54
(ill+ (5) gives · ' · ··("'o .. Yo) 7
c2;o -3-. +54 .
(D
2
- 3D t 2) .Y - 2.Y " ½+ 2.! r Putlirig these value~ In (7).and (8), we.gee.tho required expressions:
or {1,?2 - 3D) y = ½+ 2t , il· [2 ,i 7·]· 7."7 t 2 ·'·
-3 (xo +Yol +-27 e31 - -27'- -9 LI+ -3
x = - (xo - 2Jol +
·3
Awtillary equaHon ls gi~ei\ by
1 · [ (, ~o +Yo 7 ] ·' 7 ' , 12 7 · ·
. . m2 -3m,.·o;' this ·=> m"0,3 yc-(2yo+xo)+ - - + - , 21 --t'--.-- Ans,
3 .. · 3· 54 18 8 54
C.F·.. = ci, eOI + C2 ~31 = c1 :I' C2 e31
P.-~blem 32. The ~~loci Iles at a rolnt in a flu.Id in Iha E~laria11 efflatam. a,.~ gluer1 by·
P . 1 2.'l
~ - (. -+2t
1 ) ~ -1- ( 1 -D- )-1 ( -+21_
1. , ') u +,: + y + z + t, 11 m 2 (x + y + z) + I, iu • 3 (x + y + .•) + t. Find th/J d~p'lacein•nt af a.
1) - 3D 2 · · 3D 3 2 , flu.id 'particle in th• Liigrangian system. Also determlna the va/ocUy d/.tila fl.v.id
particle at (_xo Yo zr,): . , ; (d'ii.,.·1;.,;a1 200())
,. - 3D D ...... )( 2
1 (· l+ 3 l +. Zt ) , · Solution : The velo 1!ty ~oroponent.s raay' be expr&ssod in terms of, tho
d_isplacement as
l I r(
) ·1 . ] I [7 1·;
= .;. 3D L 2 + 2' ' +3 (Z) = : 3D 6 +21 ; d:.< .
u= dt =x,_+y+z +I, .... (1)
1( 6
=- 3 7 I+ 12 ) ·
u= ~ ~ 2 (x +y +z) +t, ' ... (2)
.y ,;C,F. + P.l. gives
dz
Y =c1 +c2i!31_½( ¾1+c2) ... (6)
w =,di =3 (x+y +z) +t, ... (3)
·The differential equat.ion.s can be written in form of operator as
Dy = 3c 2 .~ 1 - ½( ¾+ 21 ) ... (6') . (D - l)·x -;; = z + t .. •.• (4)
-:,:.;l, .. - 2x + (D - 2) y a 2z + t
By (4), Dy
2 . .\ '1 - 3x :.. 3;; + (D - 3) z = t . ·
• .. (6)
, .. (6)
.I :Multiplying (4) liy (D - 2) and adding to (5), we have

or x=Dy,;y-
2t [(D - l) (D - 2) .- 2) x .= (D -: 2) z+ :?.z + (D - 2) t + C
Using (6} a;:id (6'), . (D2·-3D):i:=Dz.+l-t.. ·· .... (7)
1 ( 7 + 21· ) - '( c + Cz e31 - 71 - 1 ,
:,: "3c' 2 e3 1 - 3 - 1.2 t Multiplyi,t- (4) by t and (5fby (D -, l) and ~dding, we have
6 1 18 3 · ilD-1) (D - 2)·-21;,i ~ (D,,, i) (2.; ... 1) + 2. + 21
or ·, •
·•·,:=-c1+2c2e
31 1 'I r .,
-18-91+3,- ... (7) or (.D2 -: 3D) y ~ W:i:·; 1 + t . ... (8)
. 7. t2 Multi-plying (6) by (D 2 -3D), we have
,~ = Cl + C2 e - ·-18 t - -3·
By($),' 31 ... (8)
(D 2 - 3D) (D - 3) z = 3 (I:l- 3D) x + 3 (D 2 - SD) y + (D" - SD) I
Initial conditions a·(e x =x0, y =;;0'at I= 0. From (7) and (8), we have
I I
Puttin~in (7) and (8), we.get
2
(D - 3D) (D - 3) z = (Dz a +, l - I) + 3 (2Dz + 1 +I) +{D 2 - 3D) t
. ' ' 7 2~· I ·
D, (D - 6) z =3. •.. (9)
Xo = - C1 + 2c2 -18 ' Yo= C1 +Cz
·T I I .. The. solution o( the ~:.r[<~?~tl_~'. ~t~tion (9_) i~ ~:Y.~~ bY. . ,!.
I <-,<'•• ··"Y••:4 .,i. Bl+ Ce 6'-..,:¾,12, ·
I I
... (10)

•:
.--..
G • ~

,:,J\::•:t;~, ',:,·::,;:·:,;..:. ',::,;.:~,:~•-:,, ,' '•' . ,:, :' -·' ·;- ;, ' ~~; ;: '; :• ::•::·:::: ··. ·~:::·:. . ,: ~~•:-r;, ····:·;r1?l:~;;::::~:r~,,, ,.....~·-~:
r~~

- - - - - - - · "-·~,,,...--.-.~~. ,,,,,,,.n . . ,,,r..1»!>ios:th-'•'·'•·••·•••·fr(L1',·Jl·•.-,❖.•:,f,:..:.!,~~


::? ,
,,·,,
~::-:~;:;-:·'.-z,::;:·:
,• ·,,,,,.,,~·· '~:\,!,',

rJM'!,~~~~,-~.;,,._,..,,.,,.,:..,...,~-.__.:.h,,..,.,,:.,..,._,.,,"t,~ ............. ~1t-lt,i~)11t~~l~M11.11,,u ... h,•l••·•••1'J••,.,••,,~,·~,,;n,,,.1,.,, . ..,,~~".l'lJ!l'.•,n~UW1>.l'~~c/,..;~w .. W.•J.of,.,U.t.:t• .. '' i.,.~ll..l!lu:., ........ ,f,•• .. 1 ,.,, ,\7!Ht\H,',·!.•

i . l . . .

"
!
:)

j
i
\t,
!l4

From the equaUons:(5) and (8), we have


(D;-:-2)y-2i:=2.u11,
: •
. '
:
I • FLUID DYNAMICS I . ~KjNEMATICS (EOUAilONJ OF CONilW11)')
j
i
; 1.
i
I
:to +)'o +.-o -(C1 + C2 + C)" Bi,
Ci+C2 +c .. sc 1,
I ,.:::

I
... (22)
; -3yi+(D;3)z=2,HI, "I . ·. B B l
1
! Solving the equations, we have : ' I 1+ 2 + Q + ,"' 2'
/ .
· i.q2-6D)y~.2Dx·.+l-t j (11) 2 (~0+.Yo+zo)-(C1+C2+C)"'B2,
(D2 - SD) z = 3Dx + l + 1. (12) 2 (C1 + C2 + C) = SC2, ... (23) ·
2 (8 1 +·B2 + B) + l ':' 0,
From (1), we have ! · ,·
: (D-i)x=y+;-t-1 : 3(.:ro+.YJHo)-3.(C1+C2+C)=.B,. j ·

- 5D)y + (D :.. 5D) z ~ (D


(D - l) (D2 - 5D) x ,.'tD2 =
2 2
or - 5D) t . ·· + Cz .+ C) 60, .
· . . .3 (C1 .. , (24)
or (D-l)(D2 -5D)i=2D:i:+l-1+3Dx+l+t-:5 . · ·
3(B1+B2+.13)+1"-l, ·
2
or (D~ ...' 6D2)x=- 3 l · :.. (13) • From these three sets, we obtain ·
The solution orthe diff~rcnt!~I equation becomes
. · l . ·• , . ·
. ,
:
,: ;
,
C = l ( . . , .' · C _l( · ·
1 s xo +.> o +zo + 12 , 2 - - . xo + Yo +z0 + -
1 )
,
·l) ·
St+_1 1-2 , . ('14) . · , 3 . . 12
X., A l + B I I + C l I! 4 • .. , , ·. . 1· . ,' . "
,,

!'1-oceedlog m the s11me manner,y,e haye·
: . ,. ·
I ·
·.. · C=·-(ro•+y 0 +z 0
.,. ·..:·. 2. . ; 12
+.l.). ' "•~
.
• ,
Y:·~.A2+ .21+ 2e
B C St I
, ...
(15)
Also
:,;
,:•·B 1 •-
l
, i l
B 2 ,;,--, 8 --.
Thus the equatlona: (10), (14) imd (15) deter.mlne the displacement of a fluid . · , , ' ·
12 . 4 1 ·, . .
:,
?e,rt.lcle, ; . .· . : i • ! we g:tbst1~utmg these _values Jn the rflat1ons (lfi), (17), e.nc! (18) and simplifying,
Let x "x0, y ".>;o, ~ "zo when.I'= to" O ! / ·
The relatlocs (14), (15) and (10) give:· ' · ' X ~ ¾Xo -½ io :-{zo + ¾( Xo -f)'o + zo + ~~ ) e6' - 11 I + ¼12 - 7; •
Xo·= J\1 +.C1, Yo= A2 + C2,. ?o =A -t: a ... l 2 , l l . . · 1· 1 · ·l
~- . y =.-3~cr.+ 3 Ye -jzg+ 8 ( xo_+ Yo +.•O +•rr)eet -61 - 36'
Thus x.a;;~-C1 +B1l+C1e 61
+¼tZ., ... (16) · l '•.. l · l' l( . 1) 5.1 l l 2- l
1•..*o-C2+B2t+C2• 81,' ... (17)
z•-2.x~-'z_Yo+jfZ0+2 xotYoHo+~2 e ~4t-4t -zi,
, ' ' . 1' . d~terrnir.,es the displacern~nt of a fluid partjcle in Lagrangian 1escriptioi;i.
·,.z'"' z0 .- (j + Bt + Ce81 -
4 t2. . (18) Let u1, u1, w1 be th~ components oHhe velocity in the Lagrangian d~scription, •
Substituting tl\ese v'alucs in (1), (2) and (3), we obtain We have · I · ·
' 61 1 ' '' : ' i · -ox- '. . 1),1 l l
B1 +!6C 1e +2't=,r0 +y0 +z 0 -(01 +C2 +C)+(E 1 +B 2 +13)t u1 =ai. =1• xo +Yo+ zo + 12 _e -12 + 2 r.,

/
J' •
'+ (Ci + C2 + CJ e61 +t
' ..
ui.=ta·."'2(. .
xo+Yo+•o+·12)eo1_6,
l 1·

'B 2 + GC2 e61 = 2 (~ 0 ·+j0 +z0 )- 2 (C1 + C2 +CJ+ 2 (8 1 + B2;t-B) t ! ,: cz ( ' · , 1)11
SI . . ·. ·wi~..~=3 xo+Yo''to+fa ,- 4 - t..
+ 2 (C1 +C2 ., C) e + t, ... (20)
, 1


1
: :
2 i•'
' i '' ' ' ' ' ': ' Thus the velocity oi''i:he Ou!d i:ia.rticle Js given by
B *ac, • .. 2 t,. il'(x0 +Yo +zi,)- 3 (C1 + 0 2+C) + 3 (B1 -1:;e2 ~B) 1
0
. '.
.ql,,;u1l,+u1J+w1k
,'
· Ans.
+ 3 (0 1 + 0 2 + C) e 6' + t. ... (21) · 1 Problei:n 33, The ue/oc!t:i,. compMents of fl.ow in. b;t!imJrical ·co'.ordir,a/e$ are
2
Eqllntlng t.he coefficients·of t, e81 nnd the constant t.crin, we have ~(r2z cos 8, rz sin 8, z t), Deter.ll' ln.e Ille components of accelerat1011 qf a izu'id particle.
; ; Solu,tioii : Let c,, v, w Qe vefo.city components ln .i:lJnddsa.l. co•brdlnatos
'/,- .~ z), We know that 7
1\\

' 1 i
)::~:·.' .
1· ••

I· LI
~ ........... .,,,,., lI
FL!,)10 DYNAMICS !<IN'EMATICS (EOVATIONS OF CONTINUi!\') ,!lf. I .. t
l.~1. -.£. lJ . ~ .ow-!'q=; l(o/,. j (0) + k (0) =·0
Given
. ·vx ar '· izy
11 =, 2z cos 8, u = rz siii e, w =z 21
r 38 ' ·Motion is irrotational.,
II. Con.sider fluid motion given qy
·
I
I
I.
Let ai, a2, Cl3 be compoti~nts of acceleration, · u = ay, u "o. w "0, (a "·o)
• •

a'"ial +Jartka3 · · '


' 1
.. , i j If t
.curl ·q= · -a -a -a.
' 'd. .· '( ;/_. .,., )
11 " di q '.' ai + q, V Q
ax Oji qz . i i
ay O 0
..; · a . · · ~, · a u a a
~ ,,_ + (q V)=-+1:-+-- +w- "f (0) - J (0) + k (0 - a). '~ .
I
di at ' ct ilr r ilB ca; q = - nk " 0,
!!..a
dt ill
,E. +r 2z cos 0 l_ +z sin·aJ..
r/r cl8
+; ;,?..
2
c:,
... (l)
CW'!
· HMcG xnot!on hi not inotationril.
~}\♦

a1=du_JC;, ~.~4£.~~-~..... e_:


\Consequently 1:notlon Is rotational. '!
dt r ' • dt r • • · dt Prohlem 30. lfuelocily d.(stril,u~n .is i
q = I (A.x 2yl) +j (Bizt) +·k (Czt½
:!
a 2 • a a ·? a ) ; . . ,.
( at+ r .z _cos ~ J; +z sin 8 ae tz"'( ~- (rz cos 01 ~
? .
i•
l
a 1= a~e
whei-e A, B, C cons Ian.ts, then find acceleration and uorticfty components,
=0 + (r 2z cos 8) (Zrz cos· 8) + (f si!) 8) (- r2z siri 9) C03 8) ; \. •
Soluti n: Let q = ui + uj -i; wk.
·
1
(Garhwal.2001 1 Kanpr,r 2001)
rz 2 sin 2 6
u =Ax 2 yt, · u =· Eizt, w = C:t 2
= rz 2 [2r2 cos 2 {J - r siri 2 0 - sin2 8 ~ r! :os e]
')'hen
I. Let a =.ia 1 + ja 2 + ka3 denote acceleration. Then
r:lv UV
az=,,-+- ·fi.g du
'd.i r . , . . , n = d.t , a1 = di etc,
i! 2 - a . ,, c1 2 a )·. , ~ . 2 12 . o
= ( ai + r z cos ea,: +•sin" a'8 + z '. & · rz sin "+,z r: s1~" cos 0 a a '.J.,-+u-+u--+w-
a a a a
-=-+q
dt iJt ' ot i!y i!y ilz
"O + (r 2,teos 9) (z sin D) + (z sin 8) (r:i cos 8) + (z2t)'r sin'O) d a. 2 a •·2 ·a 2 a
• ~ ,Zr~ &in Ei c~s e -=-+Ax
d.1 a1 yt-+Ey zt-+ Czt -
a:.: . . a:r . a~ ... (1)

. =.z 2 sine f2r 2 cos e +·r co.s 8;. r/l . . 'du. .


d a .a ..... o 2 1 "i "di gives .
aa,"' <!-~ at + r.2;: cos €1 ~ + z. sin e. ~ .;:- i:2t 'az }(z l( · ·a· ·a : a u
ai +Ax 2Yt a:;; + ~/zt a; + Czt 2 az') (Al: 2:yt) ,
m (
a 1 =(
a z2 + 0 + 0 +z 21· (~I) ~ z 2 (1 + 2.12] . 2
,. Ax .)' + (Ax 2yl) (2Axyt) +·(By 2zt) (Axt)+ (Czt 2) (0)
Finally, .
a"i = rz 2 [2,i. cos 2 0 - r,. sin 2 6- sin 2 e.. rl cos aJ
! "Ax~y 11 + 2;\xytf.4!By~t 2) . · . •.. (2)

Ans.
l a2 "' z2 sine
a3 = z2 {l + 2lt 2]
12r2-cos e+ r cos.e + r!)

Problem 34. Giue e:r~~p/es o(irrotational and rote/ion.al /1.ows. ' i !


. Solution, I. Cconsid_er fluid motion given by u.=. kx, u = 0, w = o,'-(1, =O)
i: az" di

a 2 =(
du
w1
'th, d , . .; l
dif
ven uy ( )

~ +A.l: 2yt 'ff;+ B,izt


2
f + t) (B.?z;)
"Biz + (Ax yl) (0) + (Bizl) (2Byzt) + (Czi~) (B;lt)
Cze 2

Then q = ihx = B;iz [1 + 2B.)'z't2 1i Cl 8] ;,. (3)


I j k dw , d ·, b ·
.a3,. di·WJ!h dt 1nve11 y (1), .,., .,.
I l. _·. •.£. -7-
u(~\~;t l+ 1Jy2zt ¾+C~,2t)c~?.
curl q x

4\:it; iJ;, ........ *

, ,.• . ··:-:-.~•;;,:: ~. '


:~:=·,\:;::.'
,:-:•,;,•,1·. .;; :•~;.::'. ''
,-.-n~~-:n~~;:7,:-ITfITTY!'';':1D,~?,3:•:!;,,;.;,\ ">;

~- .. ~ -. . :.:-~:7:-:~~:::::;::;':: .
'/,\,.,.'..:.:·
,;,,,v:,,

!'.;~frt•Jt1,tt:fnitx!ffl,t;:~~·m:,:,:t?r'...V~1r.. ,..~ ........


,
_,......,._,,1 . . ,.,... .,:.:.:;u:.:~.1.:;,.u.t..1ltH'lf~.?i.;,:;,.~1 -•.:.1:.!!1:.4::•;i:-.\:tiUl~'i:::• ...• , ...; . , ~-, "' .:..... ····~~..... ,.,,f..c'ld•r~~.lot,:-·'tt"')'Jl'J(t,WJ(~'\~,,...,.;,,,.,1...:..,,~~..., ... _,,, ...... L :••·-~:· ,:.t~_..·.l::Jy.1.11: .... ·:· ._. ... ;..... ,~.,. ,, ..·,\v~ •;,,:· ''1fH~IHIII/'..'

It
.. .•
,
;\
::, !i.6
i I
Fl.UIO DYNAMICS k11sk~1~n;:s (EQUATIONS oF coNT1Nu1rv1
., a 3 "' 2Czl + (Ax2y) (0) + (By 2zi) (0) + (Czt2) (Ct 2) . ( ., d:,; . x· ]
:,I d¢,=k 2 ~ ' . . . - - d x
·i . ,,. 0 112 + c, 2i , . , 1'.4~ . ·:i:2+.l x2+,2
·.i 1
j Ana, Accelerotlon component~ ar" given by (2), and (4). ,. Ji2 (Md,, ~Ndy), soy
Il, Let W" cu.rl q, Then W !.s vorticity vector.
', l
oM·
-■
1- · t' ·-2v' a ,.2_,.2
a;; . x2 +} . ,(x2 + y2)2 .J; (.1:2_+·;,2)2
l j k .~ - -

.w .. I l.a.r2
1. l.
ib-
l.
oz Yi.,,~[ (x2+ b-2.,:2·]~..b:.. _£M:
. ·r lu- ;,t B;,2:t Cd 2 ·OX . (,:2 + li - (.:r2 +y2)2 ,- il:,,
l Vorticity coinponehtf'nr.c .
"i (O-B;,21)-J (0- 0) +.k (0-At 2t)
Henc~ M dx .,: l$ exact. Ther 7fore ifs solution of given by

h:y ~+Jody +C .. l!..:x. tan" 1 ( =-)+ q


ls 0

Problem 313. Test wh ether the mot/OIi specified by


· - By~ I, 0, - tl.x21,
l
Ans. ':i:-+y
Hence <I> e,dst.s anrl is siven b_y
q, "')
y ,;,

1
i
. . h2 j l .
q = . (..- -y_) '(,1, ='const.)
.
$=h 2 tan- 1 (
, u•/.
~-·,+ C Ans.
, ,,:2+;,2., ·' '
1·; ··/s· a possible motion f9r. an in.compre$sibl'e fluid. lfso, determire the equa1io1ts of Pr6bJem 37, The uelocity' ueclor in t.he{low-field.is given by
t ! ;, 'q .,:; i (Az - By) +J(Bx -
I
i' s{ream lines, Also tell wh~the~ the q\otion l-s of the potential kind and if it determi.nes Cz) + k (Cy -Ax)
!I . the ue/ocily potential, · , ' I (Mecrr1I 2002) ' l.vJ,.hr, A, B. C a,:e' non:zero constants,
. , .· ..::.1.0:t.. . h 2x ..
; 'Determine the equations o(the vort~>: tines. (Meerut 1999)
j Solution: Here u" 2 2·., . u " ~ , w" 0.
X +y . :r +,Y i i Solo tioo : Let W ,,)~ ·+ J11 + k~ be the vorticity vector. Then W = curl c:j
I
•f r. Equ'atlon or~·ontl~ulty for lnccimpre~sibla fluid is. '','
~- ·
i . .
.
,
w'=
· . l j · k ·. 1
I . . ·.eR+mL+clw,. 0
ax. ay az " u o, : . .
a
a;.. ii;a ' . a
~
.! Az -B>· · !Jx :-: Oz Cy-Ax
But ilu + /)u + ~w ,. ~ _~ + O _, ·•,,., · "i(C+C)-jc-A-A)+k(.B+B)'
ilx cl;, , i)z (x2 +y~)2 . (x2 + y2)2
:() . This.~ ~=2C, ·'IJ"',2A, ~=2B· .
\',me:< lines are given by .
oi
Hence equBtion continuity /s aatislfed.
II, Str~am lines ari given by
. 'd:t:d cl:
· ,. dx
l"~~ =T
dv dz·

- ... ::l,,_ : Putting the values, .


u t1 0
. 's!.L & ~-E.!.
i
i
or I
I
dx ~·=>:
_ h2.Y ·
+y2Jdy
2
, . 1i2x
~ dz
o
.
2C - 2A - 28
't Qr •ef.!. = ·i!z ,. Ii!.
~ . x dx +;, dy = O, dz =b i C A B ,
~ • i4-·y.~;,a 2, z=b. ""' Adx°-Cdy=:O, Bd:,,-X.dz=O
Hence stream lines: ire circles whose centres lie on Z•a.'l:is.f lnteerating,
.m, To trisl the oxiatence·of velocity poten\inL Ax-' Cy°= c/ By -'A. =:c2
. - d.<I> " u dx '+ u dy
~
+w. dz . d
'Vortex lines are givo'n by th,se eq\lations,
~ -hzy..,.......... +;i,2,:: ......;!:t_
,.2 + ,,·•. :c2 + .)'2 , I:'ro'blei:n S8. $how·. that,~·= (x - I) (Y - t) repreSQnts' ,the •ue/ocity potential o/. '"'
!itcompreu/blc
cµrue$ ·
two dimension.<il
·
f1u1'ri;. S.11o;v th_o.t'the
·
sire.am lines 11t, time to.re the
I. .

',., ,,._. .' -~·


~

,I.
I
1\,. . .. . i
!~. L FL\.JIDJ)Y_!JAMICS .itNEMATICS (EOl,l,/ITIQNS OF CONTINUIT'()
'l,r • "'
. (,: - t)Z.;. (1 - t) 2 "l:Oll~ta.11.t . . .. l
:
I
;
i
I
Integ;-11tir;g, l_~g (,: - y) .:, I+ log c
·and /hat the P,O.th:s o(fiuid par-tic/es ~OV'1 the equaliollS, ., ,,r x-y = c e1
.... (4}
. lo;r(,: - Yf• ½!(x, +y) -·a (x..: yf 1 + b, . l \
(2)+ (3; =>. d,: + dyf' [21 ·- (,: +y)) di
P11tx +y,; it,. d,: +dy = du, 'then (5) gives
... (6)

where a a11d b are co11std11ts. du. ~i


·so.lution: Given . $ = (x-1) (y-1) .• , (l) , dt + u "., .,. (6)
+· 'jo, show that 't.he liqi.pd motion :1$,.;ll?.~sible, : !tt.
It i_s of the.. typc·dx +Py .. Q whose· solut:ion ts
..
. ai ::..:
g·=r.-i· ·
·
.&
~' .. y-t,
.&·2, =0 ..=> -52 + ~ . I ye
f
f Par .
Jl'd.. ,.~~Q•,:dx .•
=o.
o,:. 2 .. 0, <b-' . ·•
. 2 .= 0
c),: c)J,.
:Hence solu~ion of(6) ls
=> · V2~ =· o, wh!ch ls ilia equation or continulty.

flci~
Hence (1) represents velocity potential of an incomprcsslbfo'two dimensional
. . ' J
r
u e' " k + 21 e' dt :
Il. To determine stream lines.
u e'. = k +2 (t - 1) ~1
11=-~=-i;)'-t) u. =·k e•t + 2 (t - •l)
~ =-
-~·
.= - (x. - t)
Or:' (.l; - k'c
+y) =:c-y ,(·x•v)
+ 2 !og .........._ - 2, by (4)
i)y C
I
Stream lln~s are given by or. l~g (,c -yi".. ½((; +y)- kc (.x -y)" 1) + l + lo~·o t·
dx=ftt.
u ,, TaJdng •
1 +loge= b, he = a,
2 we get
_fE;__---EL_
- (y,'." t) - - (,: - t) log (x -y) = ½[(x + y) - a (x - y)" 1) ;- b
or (,: .. t) dx .. (y - t) dy.
,:2 .. v2 , This represents path lines.·
Intei,ratlng, ·
2 - b: ~ 2 - ty +ltonst.
-~r x2 - 2ix = yi - 21.r + canst,
~r (,:.;.I);., (y- 1)2 + const. · ·
2 2
(x - 1) - (y - t) = con st:
or
i•,hich represents stream lin~s.
III. To.determine path lines,
11:c
-=u=-.::t=-(y-1)
a,, ·
dt ox
d N I
E:1.d =v=-~=-(x-1) \
I <TJ , I·

=> dx = (t - y) dt , I
. ., (2) l• 1·

dy = (I - x) di .: i ... (3) ',


Upon subtr-action, dx.,--, dy_ = (:.; - y) d! \: ,. =·'' ;

,. .,, '" l .I
-I
' ,dz-dY;;.·a.t:: !\ t· .:·•.· •!

I
1
,.'"•·

;\/, "·•'°i ··:~r;:(1::~l:',"tV'.l!:~~t~'!'•:~~t~il·.,1. •'•,7;•:;:,,1Uin ..:i:.\fr(~,:·;::;J~·~~·;77-::,-i~:,y;i;~i~mmrrmm~~'~:W~".'.~'~J!.:tFHltf~.1:-:irt!~•~~h~~·~~1..:.:.~~:,.~·p,:;~l~.,,'N,f!,;.~·1Sfi','l'"v'.,.(J'.)ifRfrolfrt1ll(ld~if'IWff:"fflffil'1'1'n~:'m~"~;"".;-:::1~~'f'l•-;~~~~~•~r,


1

,,:1.a:~Pi:(i~~•,;1. . : ;. "· .. ;~·- . : •:.:~ i~ '.\ {: . :' ·-~ ,:-: ~;:,:, ;, ' .
,,., ,.,-,,
·"•·' ,"
.'''.{'i:···· •1·1•\'f" -~
,_,:,,,:
• ··-~-,

... , ,, ,. , .•wtu,.9,,, 1.v.1:-•:n~K•TI·R;,W,;;~:;.:;

·.''.-'}:;:,:1~-:,:·;.:·
·••',•,·,•.• ....
, .. \;; ;~ -:.,.;•:;.';-;-..-~~i!:•W{,'JRt"jSliti$~1<,.;,!?,\s·!Hi''•: ;;1,:.:~:•-', '!n"r.:•:, '" . , ,-.

, ! :,:,;:{' M,
'••

Av.
.... .,,.,.
3,1, .. Ax
~g
~:,.
:5
1}
j: or .::;t:.r::::-u=~ .:.;:.=-v=-- :.
I,,I ' ax ' x-i +•y2' O)' _x2 t y2' t'
·2t;,__w=O .· ·

I
.... (4, 5, .6)
t
!, ·! l oz ' "· ' ' '
' which shows I.hat <p· is independent o,f z,11\enc~
. . ¢ = ¢ (x,y), . .
!.
Integrating the.relaHon (4), we have .
✓ ~. 21), I
, A~(:i
q= , 2 +
-ll , '
"Determin.e. whether. the motiOl'I specified by
(A·':" coltJ't.) ,' , or
· ,p (x,y) =A tan- 1 (i/y) +f(.y)
1

: =f' (y) -Axl{x2 + y2). I.

i.s a possible motion for an il;compre/sible fiui.d. If ~o, · detennine the the relation (5), we get
equarions of the' streamlines. Also; show that the motion. is of pote11rial · f' (y) =0 :;, f (y) =c?mstant. -' ?

ldnd. Fi.n.d th·~ 11eTocit'j pptential


Solution. We knoW that
:. ·3
·
.. Therefore
''-· :1,4,
¢ (x, y) = A tan- I (x/y).
Show that the wlocity'potenrlal • .l
Ans •.
I.
.. V.q = Q; .,,(1)
; ;.J\
1
'tp = - a ~2 +
' '
2z2) r- ' , '
,i:'
or •A {- axa(-L.)· ii(x.·)·}•::
+ i" + ,ty z'Lt: J2. "1 Q,; i i
xi '
': \V~
~ .' . ' 2 I · .·
' s ·. 'fies. the Laplace equation.· Also det.er.mi11e the ftfeamli/11/s, j!
Solution. Let rp l:ie the ve\oci1f .potential for the velocity fielr.l q ~
2.fy, ' ' 2l.)t }' : II i then , · , /}
or ,. A {(xi + y2)•.,: <x~. + yl)l ~ 01 j )
. (J = - V<j, ..-.½ a'v (x:i + y2 ,:. 2z2) ~
1:
which. is e:,,iden,t, Thus· the equation of continulty fo.r an:i~con\pre.ss!ble
fluid ls satisfied and hence it i.s a possible motion for :an..iffc9mpressible
q.;. - ½a (2xi ·+ 2yj - 4zk), · 11
rn.tid, .., . : . , ' Taki11g 'div~rgence of both the sides, wo huye
·The equation oi the streamlines afe . '!,12,p .. - V, q. . . . · ;. y . I.
~
)

!!!.=~=dz li or' = -½a V• (2xl + 2yj -4zk) = 0


V2¢ ~\
~
U V 'W . or 'f/2,P = ·-la (2 + 2 - 4) = O ' - ~ L
cft__._.· pi - dz: 2 ' ,'
Hence Laplace ·equation b ~atisfled,
•.
o . ·~ z [.
or -Ayl(xZ +jl} "'.' Atlv;2 +yZ) ,- 0 (!;=·
or ·x<U'+ydy=O,dz=.0. ·. · · . by.
The equation o.r strt>;·;mli~es ~re given
..,-.--
rH
By integrating, we have· ~
. · · · ( dxlu f dylv ;= dzlw y• . / · . u
x2 + y2 = constant, l"' constant. . ... (i, 3) or , 1/x/(- ax) = !tyl(- ay) = dzl(2az) I
Thus the streamlines are circles. whose centres are on Z-axis, their (i) ' (ii) (iii) 1
pla~e.$ tJelrg perpbndlcuiar to the axis, . : i": From (Ii) and. (Hi), we have . l
t~
. · , J. . k
I I, 1·: +y I . . - logy = ½lt.iS,.~ - log C,
nv _. ·alft; a1az I 1 '
a1ai: ! . where C !5 an integration consfant.
H
g ,
Aain V ,-q-. 'Ax __&._ \ I or 1 . ylz = C, ·
\ : - (x2+·y2) (x2 +y2) · O
1 I~ t[t;t)~:, x which represents a cubic hyperbola. Proved; ·j;
•h
or ra { Ax
Vxq=k.Lax x2_+Y2j+ ">'lx2+y2J
l a f~fJ ·
Ex. 25, Show that
U =: - ~ V = (;c2 - yZ) Z 1 W = ....J::.._ 1 ~
j.
. or V x.4
f yl - xl .
~kA. l(~2. + yl)l + ,(x2 +f)Z
x2 - yi 1= 0\. Y'
. . (;:Z + y2)2 ' i (X' + y2)2 ,:Z + y2
are ihe velocity compcme11ts of a possible liquid moli0/1, Ji thlr motion ]i'
irotational ? r I
now so
Tous tho·.
,p (x,y, z) su:ch'that
q= -V¢
--1s,· of potentiaf kind;
·
we .can determine
f
f
~
.. ,,:::::!!!:":'" "'••"·. "'iif~.lm,,'·:·...

·:,~H:::i<:;:<,' -\~:::1-~:J:; . ,,«,,•


!:::·r::·

,-,.•-n~~""l'~•t',.V,·,~l-1$i,J.1!hr,•,•,,:~~;>.":".f,:-7:"

-. ,':".'½;:, .·
,, ,> •. ,,, \ ," •,·,•,<.,•.~,·-~1.1(\•,•.v;J•,• ', ......... -;:,:::.,::,
I
!
;•
u
f:f
l~l
~:; 1
~
- ~. -~-----....-,.. ~..:....:.,.,·,, ··, , ,, ,-......,..~._lit.,,,..,._,,_,~.....,."·'"-......' '" .,:c., ,"" ... ,.,,~"""'"""""',,.,..,_""''"''""""""''.,__,~,"-'.!.·,:.::..e.\:,:..:u1r,,,mc:,. ·,.., ,:, ., •:; ;,.. ,,.:....,.,,,., '"~"'"'"'"""'·'·",.,,,,, .:, ,, . .

I
·~ i ' : .
.,-- ii
i; ''"'!~~- -------,.--,..:..-__:_~~=:-- PLUID O'YNA~tb KINEMA:r'ICS OF 1"HElFLOW"FlELD

•'
: '.
~ , Sol~tlon, The condition.for the possible , motion is given. .aw · av au · aw , av au
1 by where ~ = -'- - - '!} = - - .,- C= - - - = 0·
.i9' oz' az 'iJx' ax~ ..
i!.'.!.+E::'.+ ilw =0. .1 The magnitude. and direction cosines ofit~ direction are given by
1 .ax (!y· az. ·
'!'"ft'tr
~
3x2 _ y2 : ; , yl _ 3x2 · · Q=y~2~'!}2+~t and.
,: "" . 2yz,(x2+y2)3-ll2Yf·(xz+y2)3+.o=O, . Differenti~ting, '!} partially with. regard to.z and't with regard .toy
:;
which• ls,an Identity. Renee (u, ii, w) lire the velocity components 6f a• and subtracttns, }'le: have ·
, . I • '
possible l!qµl<l motion, · : ·, · · e (oui_ @W) _ a (8Y _ ftu) ,.. o
az. &j :iJX fJy DX ~
.. ,
i Again the condition for !rr9_tatlonal motion is
av aw · aw:·, au
0 ---= ---= · au av · .
Oand,--.,.,..= 0
.
.,.
'
~:!!_2 I 1a211 _ 1... (av + aw) = 0 => a2u + e2u + a2u -.,, o,
oi e-J ' ~ ,. •oz ; rJ'j iJx az t eyl ax ay az ax2 8).'2 ; oz2
!· So ---= '
av "{!w ' x2 ':"' y2 ·,x2 - y•
_..;.__...,___=0
oz <iy cr2r + y2)2· (,:t2 + i>2 ' ·
Hence the v~loc[tyicomponents.sa.tisfy Laplace Equation..
, Ex, 28.1 F/ndi the vorticity components of a fluid •particle when
. Ans.
~ - £!!.' ·;,,, - 2zy + ' 2xy =0 velocity <ilstribt{tipn!is · . · .- ; ·
ax oz: . ,(.x2_,+ y2)1 . (xi+ y2)2 , . • j . .q ,=) I (k1X?JI) ....+. J (k2)' 2.!.t) + k (k3U~),
anQ
·
•.. ·
I ,
au _ !!:_
(!y iJx
=la. Gr?-(3)':r2y2)2
-xl) ~ la (3yl -x2) = O,
..,; (x2 + y2)2 · , .
1 :
where k 1,. k 2,;k.i 1;1re:constams. . •
' • ''
.
Solution.; :The vor_tlcity C,Pmponents, t, 't/, t ·ar~ glven by
,Th\ls.'v x q = 0,;.. 'that the motion is lrrotational. P11oved, : ' ' aw - av. '.'>"· '
.. Ex:,~20, Find the necessary and sufficient c_ondition thatyor:a li11es .:i ~t :=. -ily ~z = -
-· k, y2t
2 . '
r may' be at fight angles to the stre<1.mlines. · ·
f" .~QJutlon. The equations of the streamlines and the vortex lines ' • au aw
/ . 't]=---=
ax 0
.<1'
1 :are· given by
; ' ,:lz:
av au
- = - k 1x2t
dx-~=dz·:
.U V W ,•;'r'= -ax -
'. ' I fJy , .• .
· Ans

dx dv .; dz · Ex, 29.: P,etennine the equat,1ons of the vortex lines when the velocity
and T="ifi;=-r· 2) vector of th~ JlowJfold is given by. . ..
The equation (1) and (2) ate at right angles. It follows that •
q1'=iI ll(Az
l, •
- By)+ j · (Bx -' Cz) +.\ (Gy- Ax),
. ,u~+v11+w~=0 . ' ' . . where A, B,: Li are1co1,1stants. . ·
~· .U (aw
---
. av) +··,. (au aw) i-W
·V - - - -
. (av ·iJ11)
- - - : - =,
·o' .. Soluti~n!.. 1:TP,7 vortic:ity cpmponents are gi11en
~•az iry: az iJx. ax ' I : .. ali, ev · · ,
In or<l~r that u dx + v dy + w d:. may tie a p'erfect clifferen't.ial, we I I = -·
'1 '
'Dy .,. -az = C + C = ,2C
.I
have 1· , . ' · :
1 l!u
L! dx + v dy
, •
+ w. d:. r= ). dqn:•).
.
(E!li/X iJ.x +J(I'./))I dy + ~/Jz £1¾). ;:t/ =
'fJw
iiz·-ex" =A·+A = 2A,

.
....., .
Ii m).2!}_
Dx' (!y'•.·ez'
V ,.p_.Qf.
.
W .,;)..£f. ,.. ,.
'; ,=
av·
"a; -
/Ju "B ·"'
ay = +.~ = 2B. \· ~.

l}ich .detcrm!nes the necessary and suffl~l~rit c9nclitlein. · Ans, The equations of the vortex lines are

@ . Ex. 27, In an IMompressilile fluid the vorticity at every point is


ons!'iint-ln magn/ruae and direction; 'prov_e that the compo'mnts ofvelociry
u, v, w ara the so/ur/ons of Laplace equation.
·· Solutlon, Let. £i be t~e vorticity: at any point in an incompressible
nuld 1hcn . ·· ·
.Q=~i+!]J+sk
·· .:
:;,
! dv

xC
dz

i(l)
=t=-r
E. = !11. = !!:.
2A
(ii)
2B
(iii)

ff!W:~~:~r~~:~:{fi{~m::?t1f.!}ira.mu~NS1!'EUWf.J't!~ti'>.i:~ ::J:·:. ~•1;:; '.[it; !:~1!~U!Hlffll1R)J~~1~(~~ t~m.. n;;t,;;!~mut~·m,f{:pmf!H~~NtilffH~~.:; 1~Hlr. 1 \/~;·!~~'ff! :u:' ;,; ' .
. -- ,,_ .. _,.,, '~--_,.,..,.,.,,.,,~ ..."""'-·"'£ . . . . ." "'' .,. ,.. ............. ,,,_,,., ·.\, -.,•.
~

...

~ v-d~-.~·,-.;·:u = (!! dx +:
10
From (i),_ and '(!i), we hav~ .
Ax ~ Cy = kp · .I ... (!)
or~ ll dx d~ +~!dz.+ ~~ dt) . '. :. ' ...
.From (ii) and (iii), we have: . . 1 • , . +). (2P...cp: + ~!:.ay + 2l!.r1z + PE.di)
· By -Al. =. k 2, where k 1 and k 2 are lntegrat(oh ~onstants .... (2) , . . . ,,. ax ay az at .
Equating coefficient ofdx, dy, dz and dt, we h.~v.e
Hence the vortex lines (l.) and (2) atitlle s~r~fg~qines. Ans,
Ex.30. Investigate the 11ature of t,he liquid »lo/(of rven by = ~ + l §E_ . ·= ~ + ). §E_ ,
ax-'by , .::J::±bx · : U iJx 1ax'v oY'. ft/
u= x•, + y2 ,v= x•• +--:;,w=O.
y•
I!
· i 1 t. w= ae +).~
oz ' oz'
0=2£.+;.§E.,
ot
.I
Also, determine /he ve'/ocity poirnlial.
..~ · ax - by 6)1 + bx
I:
.•J''
The components of spin ,arei
of
,
Solut!on, .Here u
· x2 + y2 , ·v. = x•, +·y•, ·, w - 1, 0,
= 2$ = E~ - av = !.
il)I az
(ae + ;. !!i:!:..)
ay az
- i. (iJfJ + A !!i:!:..)
qz . az ifl ifl
au_ a (x 2 + y2) '-·2x (ax..,. by) __' a :(y• -x"),+.2bxy
ox - . · (i2·+ Y'? - · i(x2 + y2;i => 2~ = ;. ..£k + a~ EE. - A a2µ - ~.'EE.
ay Dz fJy.. Dz • · i'ly Dz az ay
· av _ a.(x2 + y2) ·- 2y (ay + bx) _ a (x2 - y2) - 2hxy . ax a __ o.l. EH.
a ·r1 ·
~""_El:;.
,ffy - (x2 +. yZ)Z · • -, : i (x2 + Jzyz · "'· ii>' f)Z. oz ay
... . .!!::. +av= 0
,ix ay .
·
,Thus, the iiquid motion satisfies the contlni.iity, equation. hence il
is ·a possible motion. , · ·
'
L. ' 1: :~ \·
2~ "' 21!:. 21!:. ',
ffi ,oz
• Let' 0 be the vorticity then ·· .. , •: , · a:.t a.l. ' a.l. a.i.·
I -

·
9. = l ~ + j.,, + k ~.
~ aw uv. O
!
i1
Similarly , 2rr=· I Tz •ax
!!i:!:.. ·j/!,_ and 2~ =
ax. --;;j.
!!i:!:.. , Ji!:..
where· s =- - - = ,
. . fly· Sz , I az .ax i)xl)., .ft/
au aw ' I
i
·1 a). aA aA . :t r A, I
·
= :- ,- - "' 0, '
'I'/
ilz · ax . : .I
I I
Therefore 2 (~ - + 7J - + ~ -) = Ax ).Y A,,= q
OX iJy OZ µ~ µy µ,
iJv a,, ' I I
.
.
I: = - - - = 0.
• , oX .ft;.
, ', I ,·
I ' • '· =- $A,.. + 17).Y + ~A, = 0
' ·11 follows th.al the.nal1tte Qf the Uquid mQtio~ i~ rrrotationl. Similarly tµ,, + r;µy + $µ, =;, 0
Let </> be the velocity potentiali then ' ' : I ' !t follows that the vortex lines lie <>n the surfac(}S
d,P
'{)rl>

' oX
dx +
' ft/
==
Et. dy = .-, u dx.. ·:.. V dy '' '' ' ' -0
j/?:-tJ~
). "' const and µ ,., canst, ii.ns,
Ex. 32, if the velorfty of an incompressible fluid, at the poinl (,!:, Y., z)
... di/> = - , {atx2 -+y2
by dx + !Q' + qx d } ~is given by 3.a/1S, 3yz!rS, (3z• - ,2)/.,s, prove 1hat. the liquid 11101/0,n ls

~:
i,
... d,. "' _ {a (x dx + y dy)
x• + :12
x2

+ y2 Y
£kg~}:
x• + y 2 ..
.
y~ p_ossible _and t~at th\ velocitY, potenllal is cos 0/r.Z. Also, determ11f. the
stream Imes. ·
. Solution, '!'he ~ondltlon for the possible liquid ·motion l's'
·,,

I
'f' · · · ,
/Ju + :£;'..,+ aw,.,,,o
"" , cp=-½arog(xi+yi)+btan-1(~)·, \ Answer. /)X c/y /)Z • ,
Ex. ol. If udx+vdy+1vdz=d0+.l.dµ !wh,are J..,8,µ are 3xz au . 3z lfa i!r 3z 1~2z
·~:i . ftt11ctions of x, y" z and t, prove that the vonex li/1es: ar bn.r lime are the u=--;r.,. ox,""·;r-·-;r·ij;"" 15, --,r-
/i11es of intersection of the suifaces A = ,co11st. andµ ;7 con~I. 3z. 1'5x2z.1 . 3z , t.Sr2z , 6z l S.i:3 · 3z ~
I'I': -·---+-- +---+-=v

,
Solution. · We know that- · :i · or ~ I ~ f .~ ~ ~
~Ii ll dx .+ v dy' + 111 dz = df/ + Xrj.µ
' ... . . ,.. ,.
· i

.. -1-

...--. ...

".. ·::r~.;r:;:;•; ,: ·,:••· "'. :-1·~· ......,.....,.• ~~~.s~.t'\fl~~~'ffl'i~~·.:y,.Jffl~:'~'-:::~~·;-:~:·:;r~·.!t;r:1·r.►;r,-r::;.•!l'li't,-::.1;1•.n.-,,:~•;·;fi;~~-.:.1.i.~:·~l~-..s;..\]'~~~;i~~-.~~~fflil1-mHJ9'~~~~~nm~.i-;{~t-·h.... ..,fll~•"h:'-::--'" ...:7"'!~~r•:-7 ►- ''""",":"t""\li~:f\"tfi-'?l::~MS-,\..\}

1
11.\

Hff~FHi~~~-f~~HJ(r:1.,}M:l•~•:1 ·: ~i:;:~J'f:H•"''-· '.• ,1,;,;.•{• .: e,, ·, .,,, ·,\~;,,,,;,.:,;n: .,,:::H~~a:r'.\'. ,,.,,) -: :• . ;,; ;q;,:--;~;~t!~};~ ,'.!,:,::::,;,?:,:, .. :,; ',: ·~:-.;.,;:-:;;;,t!'.:•'•: ,,:;! ·!: ·::;·:,·), . ·· -: '!~l:-~:•ti ·•:·, : ·· , ,· ,, '•·< ,;,,.,.;;, v, ~ :.: • · . • · '.'. ~~::, :·:!': ~: !"' 1 •: ~ '.;~; ,, '. ~'.:·:,,..~r,;~~f'.~:-~·
lfl
----- ~------ ~~~'T.'f'T'~,t:;,--:;;--e1

,:,:.::•::.::;:.,:
--•-""""......--~

. ',,-:•,•:>',•'•:•,' ',·,'','

··."'°'"""""'~"""""""'"'........__...,,,,.,,.,."', ... _.. _'!o_,~,.-..._,. . ,# ...... ~,~~""'f;:tlm &1w:::t&:A , ,.u., ... ,~ ... 11-~"...i.;,.,...._. ...........~ •..•••.:,.~.:,.::.. ,,. ..... ·.....,,..,.,.~n4~~~~"";'""uu~,1,;.... t ~ .• ..:.. , . .;.::.·... :•.. 1• .-~-.... .:...~.:~ruut:.1:.. .. . ,:,:.., ... ; ....,,.,,, ,·.·. ,N,/f.
,'
~ ••:, , , ••• :•h•-\,-~!'-··,:•

~l FLUID DYNAMICS l<!N_E_l\1_AT!CS OF TI-!E FLOW F!!liD ·


, . I
i: ' '
0"'
••'*
', or . lli _ 15z (xLf. yz +z~) "" t5z _ .lli_ = planes rhro'ugh Z•axis, alt/tough lite velocity potential does not exist.
·.,
_tS r1 :. rS · r5 0'
Discuss;11e;JJa_tv.re of flow. .. . . . .. , · 1 •
which' ls an identity. Hence (u, v, w) are tJie velocity i;Ompo,nents or a So}u.tlb,n, .The mo1!0.n will be possible if !t sat1sf1es, the equation
'' possible liquid morion. .; : · of con11n?iEY,, ;that is., ·
'.'; If'¢, be the velocity potential, they · . . !, • ~+~+.£!'.:'.=o
d¢"' (D,P/{!x) dx + (8¢,/;jy) d)j + '.(q¢/8z) dz j1!•' ax fJy oz.• .
or. d,P ;: ·- (u dx + v dy + w dz)' : ' which is trpe :rrom the given r.elation. Hence the motion is a possible
l . . one. 1 I :
or ·· drp ,., - 5 {3xz dz +. 3yz dy + t3z2 - r2) dz}
r The differdnlfal equation to the lines of flow are
.·• 1
dx = !!1. = a =- A = .&. = dz
i ' '
.9.r 4¢> 7 ~ ;s. {3z (xdx + ydy +zdz) ~ r2 dz}
n
~'.!
.,
dA.:,.. ~· ~ d (x2 + y2 + z2}· + dz or J · !· ·
. V .U
W .:.. W)I
x·dx + y ~y = 0 and 4z "'0
WX 0'
,:1 • Or
::1 . 'I' 2 /'• ' ";:, ' By fo(egrating, we '!i'ave . . :
>!
· d¢""
· 3zd¼2) +-:T-=
dz 3i- 2l'dr dz .. d::;
· (z) . · ; .· · x2 + y2 =-co.nst., and .z = con.st. .
or · . - ·-2 r · ... -•;-,~+=
2, . r r-· ,-. Theis~rfaces which•.c)ltthe stream lines orthogonally.are
By integrating, we have , ·•· ' · .. · µcu:+vqj"-+:wdz=O
;;.,. _ !. = r cos f) _ £fill
' or, :! - (J;y~f,"fl.>xdy= 0
;; 'I' ~ ,,s.... r3 - ,2 ' . . , By.i:n1egrating, we· have .. . . •.
\i constant. of.lntegra'tion vanishes. . . · . Proved. 1 . ii');·; d:lx-dyly;,,0.., log(xly)=logc,
•1; · · The equijt!Ons to the streamlines are given by where c is .an1 arbllrary constant.
}! dx_,. 1l•.,, '1! . :I . x cy,
ThereJo~e = which represen 1s a plane through Z•axis and cuts
/.I V W tne str~afi line orthogonally. . . . . •
' dx dy dz . T~e. ✓-~loclty potential will exist Ii u dx + 1• dy + w dz is a. perfect
or "3xz/r3 .. 3yz/r3 -. (3.z2 - A)J~3 different(al: ;But u dx +· v,dy + w dz Is not a perfect <lifferential,
· dx" .!!J:._ dz ' , l:,x~+~y+zdz theretqre, the surfaces intersacting streamline~ orihogonally exist and
or 1
~xz = 3yz "' 3z2 - (xl +y2 '+zi) · 2z (x2p +·ziJ are the Rlanes through Z-axis, atlhough the velocitypotential does not
(i) ·.{ii) (ili) .' ! . ; (iv) exist, Further
From (i) and (il) 1 we nave
dx· s!i. ·
·,.... "!,
X y..
.
·: .
.
,;, logx = logy + log c ,;, x,.. ·cy,
' . . .
V xq = l fJlilx
.,.(I))'
j
aloy iJ/ozl
(.I)):
k

0'
= 2.wk.

Frain (i) and. (iv},;.we have . Hence the flow Is not of the potential kil;ld, It shows that a rigi<l
. ' dx X dx + y ~~ + z dz . body rotating about Z-axis Willt constant angular "''elqclty wk ·gives· the
3x "'
B:{.lntegrarlng, we hav! ·
1 2 (x2 ;- + z•)' ·y
1
· ; :
0 .. -
same IYF!e of.~ot!on. ,, ·
\
AOf,

· '2
' 3log1 =· 2l Iog· (x2· + y. 2 :+ z)·2 + togD, !
where,D is an :irbimuy constant, ! .
· · \~'- . . . x213 = .o (x2 + y2 + z2)ll2, : . ...(Z)
ir1.i(Jfhus .c~ equa\ion (l)'and 1{2) rep~esems the stream lines. Ans.;
· .,7 if Ex, 3¥, Fr/ran incompressible fluid; u =· - ,wy, \' i" (IJX, 1y. = O, thow ' '·• '

~· 'V' .thai the J'UrfaceJ'


' '
inre,sectlng
. the streamlin,es. orrhogona/(y ex1Jr
.
and are the '

-.. ·.• ··~·. ~,.:., -~·•· --••· ·r., ••· ••'-•; ...,, .. _,
' ' .,, ~·:<'-'1'\"-','· •
., .•.-.•.)_I ;..;11:,)'.;::\~t\ '1 n~•.•~".,'\ I,'-.•·-~

-
........... ......
-
t
;
EQUATION'OF MOTION
~'.
. !l
'• ~
. By Newton's second law ofinotion, ·

2
~
~
'
,,, rate of changa of,roome,tum =lotal applied force
! ff
... ··J:ffe·i:;dV=J(Fp-Vp)dV, by (2) and (3) -t
' ' ' : ~
i j t
'
i 0~ f[~p-.Fp+Vp }tv.=O t
~:
i/1
I •' i I
,. I SinceS
v~ishes,
Is arb{trary and SQ Vis ~rbitraryso lhat the integrand inhe last lntegTal·.
··
' ·
½
1:
if
, . I ·.t: . .•
d ;
EOUATIOH.SJ OF MOTION
I ' f---,-
i{,,
0 j
7ip-Fp+Vp=p
49. = F _ l '3p , '
"' (4)
II
Tbeore"i;n 1, ~uhii-'a equntioo of motioo: To deriiie:k/,ljr's·Dynamical . p . . / • dt · ~
equations. · · (Kanpur2o'op,j2002;Meerul woij This equation ls known !!S Euler's equation ofmotion. lfv,ie write. t
~;
Proof: Let a closed surface S enclos!ng'n·volume V of a 'n'o~;viscous Ouid be q = q (U, u, w), F =.F (X; Y, Z) t
moving.with the fluid so that S contains' the same numbe'r of flu.id particles at any
time t. Coosider a pointP Inside S. Let p l;>e the fuid density, qth'e fluid velocity and
then the ~artesian equiv~lent of(4) ls
d . 1. ·a a· a) ~
(ul+ uj+ wk) = (IX,.+ JY + kZ) - PC1 a:; + J ~ + k a:; ~
,;,
dV the elementary vohime enclosing P. Since the mass p dV remains unchanged .
dt ~

tbro\1ghoot the motlon so that ·, ;, · •. · · i~


. d ··,... This · => du = X ~ l £.e.. !!}!. = Y .1. l 2£. dw = z - .! £.e
p ox ' di p /lz: ~
de (pdV)~O
The on tire morn en turn M of the volume Vis
... (1)
with
di
d a - a -a
de= & +Ua;'+ V;;;; +wa:;.
, ·p 0)1 ' di
a
c
~-
~
i(
~
Deduction : (i) To derive syryimct.rical form. :::•
a
~
M= JqpdV
V Here we have q = u, V = a; ,
or >
.roomcntwn = mass x velocity . '
di aia ti/. a,:a . F :.'F'.
d

-dMfd
= .[ _g p dV + -d (p dV)·q] Now(~) .becomes ,
dt . dt - ., ·. . a.t ! !

Jdtr./_g, ,, i:)t er .. FP£.e..


( l..+ul..)u ' p c)r
using (l), dM ~
dt dV. !I '" (2) (Ii) .To derive Lnmb,'s bydrodycamicnl equation, By (4),

~ t q • V) q ., F ~ ¼Vp,
••. . ' ! I '

Let·n be th.e unit outwru-d normal vector Ort.the surf11-ce :el~~:ent.dS, Suppose ( •.• (5)
F ls the e:z:t~rnal force per ur,it mass acting on the fluid and p the: pressure at any a· · 1
point on.the elem~n.t·ds. Total s'urface force is · , '· : 't)~ .:'S+(q ,.V) q=•F.:..-vp

V
·1;
p dV'•.'J p (-l:l) dS
S usin~ this in {5),
ol
V (q.q)
· -'. · p
~ 2 [q x curl q.,. (q,V) ql
I

(For pr~ss11re acts along inward normal]


£S + v(.! q2 )- q x cu.rl q ,..F_ l Vp
= f V
FpdV·+ JV-VpdV, ~Y 'Gauss Theorem
' '
~
writlog W = curl q, we obtain
2
• ,
p

=a1
~" + V,. ( -1 q2) + W. X 'q =F •' -1 Vp
· 2 ., . 1 .p . . 1
"' (3) . Th_is ,is _know_h as Lf!:mb..'s /IJ•~rcr~ynamical}?4a.lion .._Thff!tief aduantci.ge of the
1s 1ha1 11 is muai:wnl undv a chrmge·or co•ordma(f~ysle111.: ·:: .
I
',

•/68)
" . ' '
~-

.,. :'.'.:::~1g~_,:-' : ;,;:~• :~i,,:1·· , ,: '.15;,,,~!, .: · ,. ' .;:,~;->,::, .,;,•(:>:;•:;-,?,'(, ,.·,:::,;;,~i>:i••'•;, ,· .:. ,'•>,::: '•'.:•r~w:::t';;>,

.~Ji:t:.bGfl\t;;Ac,,,~~~:;:~

-· ·.::·::;-.:,~.;;;,:
,,,\,,,,,,, .. ,,
.. : c:: ~i~,; ').If.: ,;~ ~;:t ~: : '
,

.. ,,,','(,•·.· :,:,,.1,, ,

if""~•.,_-..1(....:,,.,;;.i....:..'-••-•-•.;•••• ..•..·••~·-""''"''J(--""'''';"'~•·'••• . •. ,... ,.;;,;,. ,.c .. !· ...;,..,C,,,;,:.,:,_;.u;<.•-•-••·•••'<1""""-'=>'."~,,:,':.'.!,;,:,,::i:,:,.,:~=..... J....... -•mHuM,.,._.,::::""""'~N~l>•O•\,-,,>,;;",.N.)<

:1
ll FLUID DYNAMICS : EQUATION OF MOTION ' '.t
;I I '
;i (Hf) Euler's c·quntlon ln.cyllndricti.l co-0:rdh:iatc,s. '2.1, Definition
iJ•l .J:!uJor's oquntlon of motion Is · ' 1
; : The 11elocity q Is called Beltran.l vector.If q is p'arallel to W, i.e,,.:!' q x W "O.
-:~ d '. D 1' · : ! Def. A fluid is said to be barotroplcifp m f (p), .
.!i!.9.r!:!Sl.='F--'ilp ... (1)
:,,.~ •dt .. dt. p 2.2. Def. Conservative fl~.ld of fore.a :
· Let (q,, q 0, q,) be the velQcity components ru:id (F,, Fe, F,l components of 1
; In a conservative field of force, tne w.ork done by a force Fin taking a unit mass .
'i: ox.tarn al force fn r, 8, z d!Jectlo~s. Thei., we' know that frdrn a point A to a point 13 is indepehd~nt of'the path, i.e:. · ,
;;. Q[=(Dq, _ _tl: Dqa+ ~,q8 Dq,)
'rJ F,;r=f F.dr,=-n·,
0

µ,
:! Dt , Dt r ' Dt · r ' Dt
I
f, 7*• f)·
J\CB' ao:, .ADD ·
:l
:i I
· F .. ,(F,,F8,F,),
· Vp= ( a 1 ~ a· He~en is a scllla:r function and ls known as.potential function. It can bo P.ro11ed
l
'i I,. Substituting in (l) acd: equating the coeffici'ent of i, J, k, ':l'e obtain Euler's
equations of motion in cylindrical coordinates as: ·
.F .. -V.Q.
D 2 , •. Theorei:n '. 2. Pressure equation (Bernoulli's equatioo· f!Jr unsteady
..!l.!:.;.fJJ..,p •. H.e j1 · motion), Wh~n velocity potential exists and forces are oonservative and ◊~
Dt r r p iJr · • derivable frolll a potential n. the equations of motion can always be
,.' •Dqii+~:..,F _122.. integrated and tli@. solution is . . ' c o
... (2)
Dt r ·. a p ril8
./J . · . . ,. ' J·a·~ o l
.f!e - ~ + - + .Q,. F
'
+J.!,.,.F _b~ r(; . P at 2
q2 (I)

:Dt' • Mc: . • A
•D· a ·11 a i'la 1l a 1
(Kanpur 2001, 2004; Garhu"C.l '2()01) Flf, 2,1
where nt .. 1ii'"·· ' ai.+7 ae +q, az · · r '" (3)
'l
'Proof: Existence of velocity potential'::;> the motion ls mot~tfo:o.al and
-v'~. . . .
:(iv) Euler's cquntl,ons ofmotlot1 ip gpherlcnl polar c4ordlnetes.:
, . · • : (Gt1rliwol Sll)Ol) i Forces are c~nservative =Fr. - '9.Q, .
Euler's equation or~otlbn ls } ·· , ·· · p d .· , . dP 1 . iilP
. . i£S .. Qg .. F -A ',?p
.
, dt· DI. p· ... m
Let J,
P = 0 &p , then -d" - so that Vp =I:-;;-
.. p p,
.
<IX'

Let (q,, qa, q~) be the; vel~city components a.tld (F,, F 0, Ii'~} be the compoZJenw. of i • or. ..,,,,. .,., dP a,; .,. i
yr:,_,-=.,"'-.==-
l ..,
vp ,or
..,,, l
vr=.-Vp
on
external force in r, e,,~ .dfrectlons; Then we know that . .. . · dp ox · p ilx p . p
By Euler's equation, . .
Qg . (Dq,' qi-q~_Dq8 q~~ote q,q 8 ~ q8 q~cote) d l . 3
Dt, " .Pt ;- r ' Dl - r · + -r-' DI + . r - ' (
,7tr;.F-PVp. or w+(q.V}q .. -'lQ-VP
. : '
F"(Fr,F8,F.),
(ae' iQe ,1· £2.)
t;p,;: ;, , - , , - . - ,~ • or. . f<-V~) +v:cn ,i.P) ~(q: 9) q = 0:
, • . .ar . r or r &II\ 8 a~
S':/.fistltuting in (1) ~d· equating the' coefficients of i,j, k we obtain Euler's
equations of [!!Olion in sphel'ical ,polar coordinates as: or V(~ ~- +'2 +P) +½;i- q x curl q =0
• Dqr q~+qr ·1 ~ For V (~: q) = 2 (q .x cu~t' q +(q •. \7) q] ·~ J
m·- r "F, -;; o'r
...... . 2 .
Dq 8 _:q, cote +fil.!!.,;F .. lBL "• (2)
.. v( o;p+½q(-in.;o, .
Dt · ·r · I" 8 p r1le !For cu~i q '.,, '1ix q '." V ~ (- V,P) =- curl grad 1/)., 0). . ,
£21 Multiplying sc'alarly by i:ir and rioting th,:i dr_, V = d, we get ' · ;{
"";_qaq~ cot a =_Ft-p' rsin O· 01> l
l' : .' .' l . l' . $!,
· DI •'
. . i· . ·~(Q+P+'l92-~.. )=o ,:•'
;,
where 12. "' L+· . 1. + ~ .1.: +-2:L.. J.. ,
Dt. ·at q, ilr ·r·-.clO · r sin 8 oq,
. ·t ' I I I ,
/,
,. , 2
, ..,. .
. vt .
. ·..,•_.'.• ,., .. ~:
::
·::

·: :··. ~
lil'f«1S$!l!lM!:".ll'l;•.s~~)J~!'ltl'llfil~,~1:rc•:,:{,t':,•i11.!'.-r,,:s.'.!,~!!ffll'!t'ttl!,!m~"1f',,,,·.,,;c-1;11mmmr.w~11i-.i;im;!"!ni•·•>'.·,,,:·:,··· 1· ,,1"'·?•:,,·,~:
...
,,.,.,, . ,
I :

'' 'I- ku10 0YNAM1cs EQUATION OF MOTION

0
~ft:'~\ .
. I11tegrating, n,: P+½r? -~t F (I)
.Putting the values,
. [!.+B(x2-l)'o ,._l(12P..:i:-j~·)
where.F (t}ls a eoostant oflntegratfon. ot . (x2 + y2)2 ax +~1..](ul~vj
·(x2 + y2)2 cly ·) p ax ily .·
• o"' • · .As u, u are !ndependent.or1; by.assumption,.
or _O+
f 'p+ 2
,
dn
'
l q~-'fta.F(t)
I ,

! .•
... (1) . ·. . iiu au
ai;::? =Ti, Hence \he
I
ast gives


The equation ls knpwn aslJ.er11011Zli's equal ion for urw;leady irixitqlional motion.
This.ls also k!'IOWn a.s pressure ~quation. .. : · (Kanpur2005) __
B_ [ (:r:2 _yi) 1.. + 2.xy 1..] (i,d + u,f) ,. _ l ( 1?.11 + j 22.:)·
lf fluid is incompressible ill'en (l) ~ · · ' (.r2+y2)2- · iJ.x Oji 1 • p ax oy,
' , 1'·1 • I . . 2 2
J J· dpc"-, -,t,
0

. ,.,, ri i • om'. F.
. dn l· This=> _ _!op·.. _B_[tx2-_y2)l.-1-2xyl.]B(x l •.. (1)
••+"-+-q•-.::;.,;., .(1).For-=-=- h •
: p OX (X2 t y2)2 . ~ . O.)I (,:2 + )')2 ;
P .2 · a1 . .. . P . P P
Peduotion :·Supp1ise the' motion is stead:,,. il d!
an ;
1£2.
- -P ·,zy
.,_. ""+y
:8 .
'2-, (2)2
[<" 2 - Y·2)"a. + 2..-Y ;;:-:c ]( ~ · ...
z 2)~ ... (2)
.~,)' .,: +,Y
Then~~O.Now(l)beco1nes· . J .ii j ....::.:L l . (..-2
2x (3y2 - %2)
... (3)'
·1 ~ut
~ J·(x2 + y2)2 "
n if SYl 1 ·q2 = F (I)-= C = absolute constant
p .+-
2 . .. ·1
I
; 1. ( ·x2 - /- }
-ty2)3

= ~ 2y (3x -yZ)
2
... (4)
I.
c/j' (:,:2 +y2)l \ (:tf' + y2)3
.!
or .
O+
I ·d 1
$!..+-i=C,
.p
'
2
:
a l~.1~2xcy2-x2l
ax l (x2 +y)2 - (x2 +-ila .. C"' (Q)

This is known as Bernoulli's equation for steady motion. '.K,anpur 2000) a 1· ~ ) . 2x(x2 ... )'2)
"' (6)
Ii p = constaht, then .. . ·ay (x2 + y2)2 " ·(.>12 + .,:,2)3
. 0. + ~ + ½q2 "' coost. . Writing (1) with the help of(3) iUld (4),

Ex, Der~e Ber11011/li's.eq11allori f;r unsteady. motion, of an. incomprcs~lble fluid and £2. " ~ [(_...2 '.. y2) X (3y2 _ ,<2) _ 2xy2 (3~2 _ y2)]
hence'derlue expression for $lead;; motion, : (Ga_rhwal 2004)
ax c:? + i>e
Solution: Here w)'.ite the 11b·ove prooC and !t.s deduction complete. · Ee_,. 2pB~x
or ax (~2 ., :,,2)3
(7)
Problen:i l, Show th.cit the velocity field ,
.· r , . ~ - . E(x2.cy2) -~ . ':: Wrlting (2) with the help _oC(5) and (6),
. 0"' O -,1f'
9) f'
. v .
· · .u (x, y)" 2 2 2 '
; {.r +y )
sn :sfies the equation of motion fo~.an inviscid iJ;._com.presslble llofu,
u (x, y) -
.
2.
(x +Y_)
2 2 '. w "Q
. .
· :
. ,
pr· termine 1/ie
.' , . ;~ ,. ·;~B~
-~Y tx•+.r).5
[(1,:2 _ y2) y (>,2 _ x2) + 2.,:2>' ;x2,. y2)j
,
P,res$ure associatqd with lhi-li uc/ocily field. B i.o ~n•tal}t, . 1 \ ! .i
or £E.., 282y (>:2'.,,:,~2)
Solution: Euler,fo!;!Jn~!on oCmoion ·;,, nbsence orexten,al-•f?icesJisl ! • • .. (8)
, .,, .• • • I !
' ,i .
ay. •(.>12 +.fl'
~~(.l,;,p I : '
Difforentiating (7) and (8).partinll:,, w.r,t. y a_nd x we find that
dt p ; :

or,
a ) 1
( a,:+ q . 'v q = --;; 'vp
. l ox.= ft
oyfl_ at ay (Pr(jvc it)
.
But motion is two dimensional l\S w = 0 and q = ui + vj This prchs that velocity fi~ld satisves·lhe equatJon of motioA.
dp El!. a dy
a ck + :a'. 1
,3 +i.l.+ul.)q=-l(1M+jM) G -
clx i!y
( 'ft ox cly p ax . . oy
f,•
--..
. : l
"· "';;Jllr,''r"''·'"' ,,-····•h'
1\ "" ),r<, .. ,,.i, ·. ,..,,,
, "'t:!l"~;\"<•.'til'l!l>!~'~'"'''"'~">:J:<:M'~"~·•;w,~J·.c:,,<:•:-.,:v"''•i''
,,.-; ,·, "., .....,,.,;}.,-' ....... ,.,, ..·,·•.··.l''(f'" •. . · .. ,.·:., .. ,,,.,.,,,,,..,
· · ...n.,r,~"'"•~..,.-·«'-"'"'fmll~~!HHl'!''ff
·,, .'• ., .. ,,.,,, ,,. · •• .• ,, .. ,,... · "• · ... •
P.llm:W->«<-..;-..1:.,,,_""""'"W"l~~)W,W,
·' '' . . ' •. '· "
___
.
,,,,i,....:ii;N•\h~• M'•
',.' ' ·

. '

,;.;::;:::'.:;.:. ·\•,\•;:.::,;:;::• ·,· · ;, .,,, ·, •,·;,,,;<•-r~:;t:::~:::•·· .-;1';:.;.. ··'


:: ::, ~ .
.;:;] .. '.{~~; :t' ....... ,:-,;'.',~ :.,. ·::,:·•;;,::. .
-~'-6'.t•~:;. ··,!',•:•::·•_:
SiitHi\i ► JhiJ,:;;<1}.J),SIJ,VJi,,~M:O~:,;:\. ..j

.--.-.--"":"7""~--- .
.. ,,,. ,,.,,, . :'< :.-~~ \ ,:

•···-·----·:···-•=_,.,,,,,,,.·.<::ca•~="'""t""w...,.~. .,;:.,_. ,,,...,._,'"..,,,,.'.>;""''''•t·"."' ~..._.,,,.,.,,,,.;,.."".,:''J""·'""''''-""''•'/•:.wcr.~r.=-••••;·"'-'''""'"''"'"''''•'·.,::•,wcn,i.1:•~,·,,, ...


0
)'$' FlUIO 0,YNAMICS ka0ATION Q_F MOTION

Using (7) and (8), . : : Using (4) into (1) and (3), we ha\le
1tA ' ·nx m:) +(A sm-sn
' n:e -.l 11Z)
I
o , d ": 2 2
'
. . \ '(A
. cos-cr,s-
ff1' ,i: )·- '( --sin-cos-
dp = Zp B2 [ J...J_ _ X_(x ~): l d.y_] I . 2a-2a· 2a.2a 2a 2a 2a
1 (xZ+y2)3 (;f+y2)◄: I
~ 2pB 2.IMcl.x+N,dyJ,,say ... (S)
M
x ( ,-]aCOS l'Et·
2a s n]a m) =--plzeilx ,,
ioM' ~ aN j or, -M.2 [ ru: nx. • nz · rue . ~ • 2 -ffl!
cos-sin-cos•-+cos-s,n-sin ] 1 on
=-,ff"-
'..a~·- (x2+y2)◄ '""a; Za ·2a 2a 2a .2<1 2a "2a pox
M clx + N dy Is e;,:l\ct.

~s l
.\
, or

M.2 ~ !IX 1 N{.'
;;- cos;;- sin- =- , .
..a ,.,:i 2a p o:t: ,
... (5) •

f
.
(M+clx~N.(d;)=J
·;· : (x
tdxz
+y) 3
Ody
. , I • . i:i.nd
,
I 1•
· 2a
.
2a 2a 2a
'
· . (A cos:!!::. eos&)( 7tA cos E. sin· )+ (A sin & sin&)
2i:i
'
2a 2a ·
iti
' •

J'2x {x~ + I m ,,.-=


r l!· or;l; :! -7tA cos--, sm -2a cosa·2a-. +.··.2a .2a, 2i:i .2a
:
1 ,•. ( 1tA • m: .) l ?Jn
.,.. 2l y2)" 3 dx
·
+c ;=> - 2
1
2 2 +c i x .---sm--cos-
. p il:i '
... · 4('7+.Y)
:· i 2[ · 1fZ
cos -
, 7fZ on
sin -2a =.,. - =
rt:< ·, ·. ru: ,
sin -
!tZ , 2' ,11:,: ] l.
1n view ofthls. (9) b?co~es, 2.0:"" 2a· .2,:i · pi)z'
2pB 2 · ' ~2 .. nz 1tZ ' l an ·
p=--.,........_,,.+c' or .·-.-~os-sln-=-.-=. ... (6)
. 4 (x.2 +})2 l I • t•::Za 2a . 2a . p az . ..c,;"~
, ,.•
·_:-pB2. • : I : The equali~ns (5). and (6) show ~hat the velocity con1p·onents satisfy the
·· t' or, Ans. ,dqualions of µiotion: ··. .
. 2(x +y,)
p: -
·
. 2 2 2: + C1
,' . . . ... a . ~

; Aga•in, ·· ·1 ·dp = 22. dx +'V?. dz


This is the require~ expression for press·ure. ". . . . .
ax 2' '
oz '' . '.
'Prohlont 2. Th.a particl, velocit;t for a fluid motion referred. lo rectangufor. axis is
ciuen by tl,scomponent,i'. · ' · oi : tip ~ !.!..2.d.: [ cos .!!! si~ & i1:t. ~ cos 2!! s!n 1!! dz.]:
· . 2a 2a 2a 2a 2a
u•Jic~sru:.coe!;:,1100,wc~sin""' sin:nz , i . : By lo tegTating, we have · .
. 2a .- 2a 2a .I
:: . .· · -~~½p1. r~os :-co~:l•·Q!
2 2
.
..
'
· where A la a conalt::n1; Sliow lhal thla I, apo,,sible motion i:>fan'inco111pressil>I• fluid·
, undor no body forces in an_ ,ilt{fn·/te fi,;ed rig(d tube, - as:,: s a, Os~ S 2a, AUf'
the preusure auoclciltfl wilh this uelocily field, , · ·
1
1,
fi.nd ! where C is
. : .
M• int~patlon
· ·.
constant, Th.ls gives th1ilreqruled pressure dlstrlbu'tion,
· · · · .Ans.
"''' Solution : The 0(lUat!ons of motion ror a· tWO•dlroensional steady, 1Jr,iscld,
lncompreulble !'low und~·r no body. force, In cart.esian toordinat'es, are {li•<.e!l. by . : fl'o,blem-3, Petefmine the p_ressure, if tlie velocit) field q, .. O; g 9 =Ar+ B, g, =·O,
. 'au . ;Ju au
l c),.; .. ' . '1 ' ' ., . . 2 ·. . . .
u-+·v-+w-=--=
·ox j a,, oz pox' (i~t~/ies the'eqw;.lion. of motion p ~ .. :!J-,
where A and Bare arbitrary c~n.stanis.
i , , .• . r r ..
1 on

1
I
i 2
,. o---=
- Pay ;Solutioo·: '2=p.!(Ar+!!)
dr r r
aw i!w ilw . l on
.u-+u•-+w-=--·=. £!.e.,. p(A2r. +_If )'w 1)'.
•• a~ ily ·az pi)z'
· , .. (1, Z, 3)
' dr · ,J- : r
u
qere A cps-
., n,.:-cos-llZ
1.1=
' 0
,. u=., . A sm-
w=. ' ru: sin-
' !t.:I . , (4).. By integrating, we havo
! 2a. 2a .. ,. . 2a .2a I _ l ' !32 .
Ftom equntion (2),: it .follow:. that the pressure p is inµependent of i.e.,
p = p (:,:, z).. . . . . . .
p=p ( -A 2r2 --+2AB
, 2- . 2r 2 . ·
r )+ C,
Cis·4h integration constant·. Ans,
I

I:
, . ~, '"·~., .........

. ,. ' ,~,--·~, ___ .,,''


i .~
I I FLUID DYNAMICS ............. 1 .. , .....

' 1::f)UATION OF MOTION


~- :·
Problon-: 4. F~r <;>n. lnvl~~•d incomprmlble, :#eacl:t flow wi'th ;egligiible b·or:J:t forces, I
• " EE. f!.JL:
i r r1 .p, Du f
velocity components in sph~r.ical po/aJ'" coor;r#nates are given. by ; , · By (3)., . a ,.
. ,:I
r .
• rir*V
· r"
. ( 1.--:'·
,
· R3) ~ose, · ~.;;y
" . ·2r
· . '( 1-1:7
·. Ra )·s/nfi,;
, ,
· ··

u~ =-0. Show that it is q possible doludon ofrnom~ntu~ equaticmd,.((.e., equations of


PuUi11g
·.
"
lho' values
.·- iJ " -1
l M
.
! (.n
.
) sin
\I'. 1 + -
3
'

e l3
.
3
' ( 1 - --
+ \l'.D R ) cos o .,.__
1

motion) R anifV are co·~stan Is. '· · · · p ar r , 2r3 1'3 . i


Sofot!on: Write u,.; u; ''e :..'v, u~" w. Th~n With .0 given by (2), simplifying-; wr get . . ; I

. ( 11/J ). . . ' l: 22 ~ 3~R3 (. l t R3 ) sln2 8 - s·v2R3 ( l - ·,R3) to•2i9 l;


u., V 1- 31 cose, v- -y: 1( -1 +.3,RS )·. a!n e,. w=
. .
o., t P er ' 2r4 2r3 • .• r4 r 3, I ·
.... (6)
i;
, r .-,1 .. 2r · ;
SirollaTly (4) giv~s , :'.
. To ~h9w that vel~clty componen~ satlsf,y"equatlon ofmomentunt, Wf have'to
3 3 1·
show that \.he veJocity.coo:iJ?onen!.s satlsfy Euler's equation ofTI1oti~n.
_£I!.=-
la 8v2R ( 1- -R ) sine cos e • 3v2R 3 ( 1 +.-·
3 3
. R 3 )''sine cos 8 ... (7)·
+-- ~li .
gs·=F-lvp · · · P iJa 2r . r . 2r 3 2r~ · ·
cit p (Calculate it) · ·· · / · l·
or
a' q, V} q = F- -l lip
['I+ Differentiating (6) partially w.r.t, eand simplifying, we gel I
.r.
p·, . : : ' " ~
-pl 1.:2_
, =(· -9V.r2R3 . ·e . 8
.I
' <J ' ,$
. ~" 9v2R6) sin cos ' ... (8)
BY.' assumption, q is independent oft,¥}~ Oand body for~e j i~ ~egliJb!e. o<r ar 4- - -2r
- 1- ,,
. o, '. ! I '
Df!Te1entiating (7) parUall/ w.r.t. rand simplifying, we get
' l , -. i . ' ( ) I
,-. (q,.V)q,.--Vp · : :. .., I 3 5 Il
' p ' ' \ ; 1 1 h.. c ( 9v2R ': 9 li2R ).· sin a' cos a • ..: (9)
' a, ae '
aro + r.uaW+7si'iie· 1l¢.o ,,::
,W i I
p r4 . 2r7
Write D ,. u ! Since (8) and (9l'are identical hence equation of mo\ion is sntislied. I·
;:
Putting Ure vali.i'es ofu, u, w, , I:
Pr~blem 5.rhe velocity componenls . i•
. • r( f - ?
D .:' V L
:R3 ) · iJ l( R3 ): ' ii ].
cos' 0 ai= - ';: 1 t 2,3 s!n e aii ... (2)
.
ur(r,8)
·,
=:-:( 1-~ )case,
' 2
i;/-.
·..Spherical poll\r equiv;.,ient o,f {l) is , . ~,.
, u 0 (r, 8) =•
V ( 1 7'?".
a2) sine_
.Du' - {vi+r w2) ., '_ 1.p ,ar
~
... !,,.
satisfy equations' of motion for a two dimensional Inviscid incompras~lblaflowr"Flttd
Dv + uv _ w cot e = _ l l .il.e
· r, r p r ila
I:
the pressure associated with uelocily./leld. V ana a ara aonsla11ts. ·
Solutipn: Euler's equntion ofmotio~ in absence of external forces ,is

i(·

uw uw ' ' '


. Dw :t - .+ .,- cot e i -,, EE.
-.-. ,,. d 1 . : ~!

r r p r sin" v,. · £g ,. - - 'vp . I · (1) !'.


dt p • · ... I:
'!
Since w=0, hnce the above equations become i a ·
· . u2 ; · · 1 on ' I d
7t"'ai + q. 'v, l
1:
Du--=--=
r ! ... (3)
But u; ~ u·, u6 ,.·u are ln~epen,nt o,ft, !:
,:;<~.:
p;,,,
'1w · 1 a..
.Dv+-m--,.=
ir' ... (4) esat '" 0•
r • pr ~a I
I ... •\

a,
I
0 ,;._,1_·_1_~ I ... (5) · Now (1) becomes

(5), => ~ =0 => p = f (~. 8),


. p rs!n,8
(q. V) q "• - Vp

'Write u, ➔u, u0 = v1 u{."' 1'.'• Th~!\-


p
1 '
... (Z)
tIi,,
~i
;: .......,.

....-.,...
l,.i
" ... , "' ,..,,:•.~;· ;~j:··- ':''.!l:m1:ll:?.~lf~tf~~~~~,.,..'.":">{"~r.1~!r•~-:-•"'~~~~~i~

•·,v,,•, ···:::• ..:·,:·!,•


,, ;: ':'. ·:-,;::;,,;<
' \ ·,1 ·~ '··

,, , . ~ ~ • . - - • y - - - i - - • ~ ~ m ~ . ,1.,1,,.1 ..:,'.,'f..•·"•'N,.,:n.:.•·1ii:
·,~ .-:: ;,
,,,'.'!<t''.,•··, .. ·! :.-; :· •.:,,::ir,·~!

M
~

•ft·
~~
j
:ii
•••••..•••••Wf'ff'1"1\.~ ..1•'Jl\tt1<N-.i!,.-Uult,'<'t"""...,._, ........ J.. ...,,,.,.,,,.,..,,~-•••_,~ ............ ..i.i1,1'.1,•,USG·U~'l'l.l;~r1~f:i':l~lft0tt1l?tl.~+N.-tf·:·.,.... ,,r?'
l
I·. ::;.e:i-
,,..,

~LUIO OYNAMICS
, .,,.:,,,, .,;, .. ,i:,, :..,, ,1~\~fl~•Ht";'t•\1..-.JJ:~fl!,t.,il.1_;.;,.li.:_l):.,\, , , ·,, ., ,, :::1utmu:. ,.)f{iH;.;,1,\•

~
r
., J Problem 4, F'or~n inuisclk lnr:ompreuil;le, steady flow with negli9ible body for,-es,
~OUAllON OF MO;ION
' /
•:'.'
.r ;' .~y' (3),. 2a-~
il ,.
'•
.
:~ 1: - uelocicy components in sphrrico.l polt:1.r ccord./nates•ard «lven by
, R3 , . . Ra
1 · or,. r µ,Du
-t,, '!
u, .. v(1-r,r
)~osa, ~o"-•V( 1·+2r3)•1ne;
• , .
i! : :' Putting the values
•~,
~
il
_, ut =. 0, Show iht:1.I it ls ~ po',slb/~ solution of mo_m,~/\IUm equati_ons of "_,
I • ; '

_HE.al/v(1+A-)slne
, . ; i 3

VD ( l
n3 .
-'-;,r ) cos e .
--
' } motion) R. and V are constants, · , pi')r_r 2r3 .
1 Solu_tion: Write _u, =;u, uo = v, -u~ = w,:Th~n, '. .: i \1/ith D given'lly (2); simplifying,.we get
f 2!!.,.. 3'y,!!R3 ( 1+ R3 ) sin2 a·- sv2.n3 (. ~ - Rs ) coiJ a
1
· 'R~
}13 .
, , ' ' ... (6)
u=Y(1:._r3}os91 v=:.V(l+ r3)sintl, w=O, p or ·z,-4 , z;.3 _ , r4 ;,3 , .
To show that velocity_col'l\pon~nts satisfy equation -of Jil'neiitum, we
'show that the velocity cooipQnents· sotlsfy Euler's equation ~fmotion.
2
to l !' '
_Sinillarly (4) glv~s

' l ~" s01R33 ( 1 - R:) &in e cos 6 + 3


, _· ,1 , '
\ftt (it )sin a cos 8' ... (7i
d. , ,
II
I

,,. '' £1.9,.,,F_l,;,p


di , p
P"" 2r
.(Calculate it) - .
_r ·
_
2r .

or a q ' ,;,, ] q,. F - p1 Vp·


lr iii+ ' Diff~rent!atll)g (6) partially w.r.t.: Ganq simplil)'ing, W? get.

lly assumption, q Is Independent oft,


:
l · ,- · . , .
(q , 9) q
.,
l'
9p
a
Te"' ,
0 and body Force F Is

... (1)
l

,
:
, 1 ,:i' . ., 9u2 ,,3 -9;·,i;,Re) . -
paear'
_.J£E.,.cl~-~-
· ◄·
'

_ : Diff~r,entiaUnt(7] part,ially w.r.t,


,. ,.,
2 7" - -
~;'..
sin 6cos 9
,

r nl)d sln:iplifying, W? ~et


. '
·... (8)

l a 12' ,,.(fil:::B.:- .~1i2.R9 ) sine cos e ·-


a.;. - 2
p ·- 9: i. •
, '., p or ae r4 , 2r7 _ ·, , _ .:. (Vi
write . '
Dr,r;l.+.!/.1..+-Ll.. · l
r)r · r 06 ,r Bin 8 o,P / 's'ince (8) ~ni (9) are idan\ical hence equation o'r motion is satisfied.

lI PutUng the values


"r(,
D"'vl·
·
otL, ulw,
1--
3 'r
R
iJ
3
) cos·e--- R0 ) sine..,..
1.--
3 ar.. r
·
1(-
'

,2r
a]
~,fl
. '" (2)
ii. j'
i " '.
l?robl em~. The velocir.;, components
-· I · - • · ·2
.

·,. --~'.'(r,8)=-V(~~;r)cos~,
-
: • ' •

I_ II. - Spherical polar equlviilent of(l)'ls ·


_, "lv 2 t ~ 2 ) ___
Du-
-, r , par
a- 1
;i m .• I
:
.
·
I
;
:.
.i
1
I · •,
_u 8 (1-;e)~.v(1+~)s_!ne·
2 .-' ···:·

satisf.; equations o{~~lion ior'a l~o d/me~sion~l inviscid Incompressible flow. Find
• Dv + u~ :. w 'cot e ., . . 11 gp_ l~e pressure associated wit~ uelqcity field. Vanda are_conatants. ·_ - . .
'r r·,,prae
:,Sbiutlon: Euler's equation of'moti'on in absence of.externa·l forcl!s is
'[)w +1!!E. +~ ·cot _a ., .-1-~- 22 I 'd . l
/ · r r, prsin8o$ gg=--'Jp ... (1)
di_,. p
Slribe w,. 0, hnco the above equations become . da ·
: , , v2 1-an · - m - t q, V
Du--=--= "' (3) , _ _ . di . ai -1,,
' r par '. D,ut "~ .,·u.,"1i6 "'.U are.independenfoft,!
, I.Du'+ .!!!I. " - .:U.e ·-,, ' £.9, 0 '
_,. pr ae . "' QI= •
· a
o.. ----!!l?.
1 1
"' (5) · N~w.(l) becomes ' '
_ pr~ns-
,a -
(5) =~ "0 ·= p "((r, 8), / (q. V) q p Vp '" {2)
'-'r ~ u, u'o ~ v,. Uz = w, Then
, I

mffi!.imlllT;~ml:llt',-~!<f.;m,l/!U!'!f!'l'!'ftmlmi'lllrJ!:ill(,t,:;;,,,:-m;-;:n11';~mm:nmwi~:1,l!-!(!;,!W•l-:NHllffll!'1l'llWlffll!tllm!mt,;if!/l'i''i\;1i1<>>l'.""''

w,., .......,••. .•• ,..-.. .....•~·· .I.


~ ..•• ,01•.1.TT•Vh?-~s c,·,
'I\
l "'!
FLUJO DYNAMICS '
EQUATION O~ MOTION "" , "
'
,·;£s, ...:i:::'•

I . ,- .·
·a u a a
..

'
. . a2
1.1=-V(1--;-)~os8, v=V(1+ 2 tsin.a,w=O.
r ·J ., \ .
·.. a2 . . ' Putting the values from· (9) and 00),
.d l
- .1/..ffl:!..2 = [·- ( 1 - £..2 ) cos 2 a- -(
2if.!a rl
2
3 l +
r ·
1
r
2
£..)
r2
s!n2 O] dr
~
"' .

Wrlte D = u a,: + ;: i3a + w a; ,, !;lut,w ,. O_ 2


·a v· a· , , . f+_ 11;z (1 - ;'i a2.) sin ocos e )ae·
l ( 1 + ;z
a ) + ;3
D= ua,:t;: c!B. "Mdr +Nd& i ... (13)
Putting the vafoe,s of u, i>, _we ~et, · .. '-•·:MaN ·. ~
It ~an be seen that' 0, ., - . (Proye it) : .
· · a ( i aP· ). · · a ]2
D· .. v·. [ -...( 1---
· a ') cQse-+ 8 0r
I I ! . • V • '+
-+.-. s'!ne- ... (3)
. · , . r2 .. i>r,, , r • r2 ae Hepce M,dr ➔,Nd8 exact. $Qlution'ot63)_ !~ ~Yen by
iJ
Cylindrka.l e~uivelent o'r'(2) is · · ·· i I
r[(.l._a 2
e-(l+ 02
·),sin2 :e]dr
·

11 2 -: . '!'an,
_-=
i.,-::£._p~
z\f.!a2. , ,.a ,.s )cos 2
r.2 _,5 . ,/
Du. - ...., .....
,I'.
'I)~ ~!ill·=_ .1. £[/.
per
·._:,'(4)

... (5)'
I·I .
='GOS~ 8 (- -2 + L
1 2 • .l
)'- s!n 2 9 ( ,- :__2 _ L 4 )·
··2

• r pr ~a · 2r 4r 4 , 2r 4r •
Dw=-l~
. . p -az '
... (6) ·P =-- zv2a2 [ --cose+-
P
1
2
a ] +c '
· 2r2 · 4r4 :

But w = 0 =1 Dw,., O ~
~
az = 0 .
=> p = p (r, a)' Betnoulli's Theorem 3. Bernoulli's e~uation for steady wotion I ll'U) the ',{

f.Jrcc~ are conservati,e (ii) motion is ~taady (!ii),deqsity pis a function of.pressure
Pt.JtLing the vaJues in (4) and (5), . , p only, then the equation of.moti\in is . . l
~ I

l 'an:
t'
2) 2
a cos0--
. if.! ( · a )2
l+- sin 2 9 ·.. --=, E.ll + -21 q 2 +· n = C, C being·absolute constant.
-VD ( 1-,·-
, r2 ,r- r2 .par,
... (7)
' J p ' .
if.!( l - a2_)sin 0 ·cos O=·- -:-
i
· ( a2)
VD _ l + 2 si_n ~ - - 4
1'ik;-
ae· ... (8) , , (Kanpur 2002, 2003; Meeru.t 1992)
' r . r .r . p_r . -Pr1:1of : Step L For.ces are conservative =1 F" - vn. Motton ls steady
. Slm)llifylng (7) with the help of (3), •. . ; I ·1 =o t, = ?• density. l~ a ~nctlon of r,rcssur·e p :_only=,. there exis\~ a rel11tloo. of th\l
1-~ ··2V2 a2 (. a 2 )·· 2• _t:r\i'2 a2 -1., a 2 )· 2\ ., '
--~ =-.-
P. vr r3 - .:1-. •·
r-
cos e-~l1+"'1' sln ,8
r r . ,
••• (9)
tYJ'lO
: fp
P'=
d
!:fE. so the.t
l ·
,:;p·,.- '<lp,
$impli!'yinv (8) with the help or(3)·, . . ;_ o ,P p Iiti
-lt= at(i-a
·

P . r,.
~
2
r.
2

.
2
2
', .
.)si,n8cQs8+
2
Z 2' ·
at
r
(i+~)s!necof
'2
· , .. (10) By Euler's equation, 4g=-VO-'i?P
dt '."
!.h
· Differentiating (9i w.r.t'a. and simplifying, .· •, 1 :
' , I or ~+(q, t;i)q=-P(n+Pl or 'v' (Q +P) + (q, 'v') q =: 0.
. 1 h_ ·. 8V2a2 , , .
.. Pao ar =T sin ecos e
, .. (11)
But 'v' (q, q) = 2 [q x curl q + (q . 'v') ql \l,,
!:,
Differentiating (10) partially w:r.t. r, · v (rl. + P) +lvq 2
- q x curl ci,. 0 ;;,,
2
'a p__ s •v2 2 .
'ar ae = ? .
·
a, sm 8 cos 0
' ' ...
c12i
or ".( Q ;-P+½q 2 )=qxcuri'q. ... (1}
it),
>
Evidently R.H.S; of(ll) ao.d (1~) a.re equal: This proves that-the friv~n velocity
components satisfy eqµ!\tions ofmo(!on. ·. i ! · • ! Ste-p II. Multiplying().) sc.alarly b)'. <j. and noting i,:
II, To find pi'essure-p. , " . . : i: : q,(qxcurlq)=>(lp<q).cul-lq=O, ., . •~;'

' d.p = ,
I l'
For~xq=0,we~btainq,~(nl~p+H 2
:
)=o. '
'/;
),
I ' I ' •'

''""" ,.,. " ,.,,.,:'.:!::::'.!':':•,.•, :~''.". :•"·•:·:•: ... ,·: '''.':::0 :!~S·.::-:,'c:':',7,~;,;<'.:'.:cr.mlilJtm''.'!li~~~~~'t-;1~r)~'liJW!::;<1;;,~·«•·":1<"'c~:"•"'!''""':'~•. »;,,,JW!lm!"~"l~••;;,,-,,,1:n:,;_:,rm,r;;:;~11~•: '. : '· '" ·1~-:·~r-··,-,;-~~,:,.,!M,-.,,..,._..._~"."~""''1

,,,,.,,.,,,. '.,.-,_::::;:,::..
:::.•:::,;, .;_::·::-.::·•:::· •',•,•.·:.•
;,~j,tt,W••;\',•• ,.,,.·' ·:, ·':·::~;'.·:;1;;,•', ' ';·,: ~ •: :~ •: '' ',:. i :; '; \ :, :- '. ';. ~:•::, ~

:->"l'<:;'t"""..,~.~W~:'~:7i7f,fT,i;f';~;~~~:;~~'.,""7,
_ __ _..,_ ,__...,.:...-.,.: ___ ~--~-

:,:•:1~: . ..,;!~iH~:~'.i:,: .. : .. ·,e•~·>•- ... -.


,•.•l'tf:;,',•:•;1,1}~;,s'.. . ,\•',;<',>,: ;,
,·, ~-·· ., ..,,.. . .,, ..,,,. .,,.,,.•,· ••,: •·:t•·· -.,,.,. ,, ... ·;•..~j';•,•;·:-

r1~\
,•1

,,! .. ···~···--.. - .. --.:.,•,·~•~L".;.:. ~.::~.;~~:,_·,.u:'.:.:-~~:.. ,.. r.,•·.:.:...·.... ,.: •..•. ,~ ..... b-•,~ 1:,,,.,.,,.i.,iM-t.,,,!!'Jo(~~~~,;w.;~~U!!:::1:" < .·•, :;;a:•,;t-i~;-:::·,tH.WUZlk .............,;;,. ...:,.,...;h,..~~w'..... ~v.:n~!t-.lH,¼~!H':t~·v .,.~,-,,!; ,,,,

'~;, !ii
g ")! FLUID DYNAMICS ; ~01.(ATION' OF' MOTION.
1,
n:
i .!.
c~
...,... ·:i Tlie solutlon of this ls n + P + ¾-q 2 " consf... C
PJ,=Po
-;-~-
.,, (1)
;_
..
.,' i· A
, IJ
;her~
..
J ~ aa ((x,a, Xbi,cz)) , The. components of acceleration ate
.
\t ij• or, n+ J~-+ jq 2
=C. · · : .?.roved, ·,,:t= a2:, , a/l
i
.. ~- •·
2a1 :t= •2 ; z"""'i·
:~. Theorem 4, lf the !l)otion tJf nn ldeal lluid, for which density .is a fJction of ~t ai .
pressure p only, is'steady an~ the external forces are conservative, then th:te exists Let the external.forces be conservative so that F = - v'.Q,
f a,family of surfaces whlch contain the stream lines .,and vort<!X lin'd. But Euler's equation of motion,
L
. I • • •
'i
dn F 1 · · .1.., ·
:LI. • Proof: Step I. _'v ( 0 + P+.½ q2 J= q x curl _q, ... (1)
!
.
·
=
~ = - -p 'vp ". - 'vQ - -p vp,

l ;, Hero write step I ofTh'eorem 3, 1


, i'ts Cartesian ~quivalent,is
I. n:. Write W" curl ·! I : · i an· . 1 ea.
!l 'step
. :Then v'( .0 + p'•+ ½~ ) " q xW. 2
q :' vorticity vector.·,
.. ' (2)
I,
· ·· ;,i2.,,
at2 =-a:;-.;;-.a,,.

.j ~="'£9.:~.L~
Write c;, ( ~ P + ½;,2)" N.
-t- a1 2 a,, P ay
! Then · N = q x W, a2z·:·_ Efl _ .k~
Thus ,,,. N, q;., 0 = i:,','. W, (For n (bx c) = 0, l/'.any two of al2 - . i)z . p ~ ,,.,:I'
' ,;,JI.~ .

,a, b, c aroequall Multiplying these equations by


"" N ls perpendicular to both q and W. ,, a..., .i!:L k.
Als'o N perpendicular to the famiiy' of surfaces aa' · aa.' c!a
. i:espectlvely and then adding columnwise.
n +P + ½q2 " ~o~st. = C.
...~ ; , ild:c + 22 .fu:: .j. il2z k ~ _2fL 1 , ~ ,
!For '<J/is perpendicular everywhere to/= const.J , iltz aa or' iJo 012 iia oa p oa .,, (2)
This leads to the conclusion lhat q and W both are tangenti!l to the surface i ! :~eP,lacing a by b·~d, c respectively, we get tw~ more equations
11 :
Q + p +-:,2 q2 C,,
,··
. '
,
.

l
=
.q,
' N

'
I
~\1\illllililm/1/lnmii,,_ w
I ,
! .
,•
I I
1 1
,
iJ :c ox '1J v :i., il z oz •an l iJn
2
--+;;.....~+--=----=
2
a12 ilb a12 ilb at2 ab · ab Pab
2
"' (3)
, .. It ma ans, that the supaccs Q +P + q2 " C . I 2· 2 ·. . 2 ,
·
2 ·
h ax+ a 21. + a r az,,, _iln _ 1 M . </4>
: i:ontl'lfni sttoa~ Jines ~nd vortex lines.· , a/l ac at2 oc a12 ilc . cc ' p oc "'
:· _RomnHc: : The ·iibovJ theorem can also be ~ P l 2 i j .. The equations (I), (2), (3) and (4) togetherr~present L~gra,age's hydrodynamical
.. restated' n,j!'!'ollows: : · • · + •2q • "",""· .) eq·ua t.ions. . ··
· .'.lb pr6ve that for sieady motion of an !nvls!cal . Fig, 2.2 I , Theorem 6. Helmholtz vorticity equ,i.tion, If the. external forces are
isotropic fluid conservative and density is a function pressure p only, then· ·or
=l!(P)),; J_d~ + 2 ·d(W)
p = (p'
1
l q 2
+ Q =const. W/' '<J ) q, .
(p di (GarhwaJ 2003, Nanpur 2000)
.Proof: F is ~onservative ~ F = - -7Q
over a su.rfnce containing tlie stream lines and vortex lines. Comment on the nature
of this constant, ·i . p is A functfon ofp only,,,. thre~ exists '1: rel ati.on 'of the type
Theorem 5, Lagrange's equatio:n of motion. To obtairJLagTange's equation•
,o{ motion,
· •. Proof: Let initially a fluid part(cle be at (a,
· · (Kanpur 2003, Garhwal 2000)
b, c) at'time t = 10, when lts volume j: p= f.
p
p
£E..
p
'

'
i is dVo and densfty is Po, After a lapse or titne I, let he S!l!'lle nuicl porticlc UC nt ,,, <v P= Ii aP "'ri diie., r .! i fJ.E.
(x,y, z) when its volume is dVand density is p. T'ne equation of continuity is ax dpax Pax
\1:

:-: . '~):"
~:;~~w~;,r;,,m;;1:i~r;1;;,;:,,;Jr.<Sr«lf.mfflllll\~l'.,';l(l,!f!m·~i;i;;,1m1~,\g\1llll!r.W\!lmll'.ll~!;;0-,~9H!llllml!ll\>;lffl',ffl!llll.l~ft!•ll4;1t'.i;/;•·o<!>.i\H/','1:i
jri!f1j1!;!)1/:t·:,:;ll!'!,:,wixmiJ!MllfUl!i'!i'·:,.:;.,i''••'"•·.''\)'.·;•,
im~ ~~nH,1> r,, .;fb-~~ tm.dw«SUHO.ttfffitfnH~}\:•:, •,;,"",
\:fh:.! {k~f ,.. .,, ,, '"
()~~i,~tN\<uij,:1,•,"
f.\,; ,:{,~~:,, ,:,,: :..,.,., ..... "'1..1{~\Hi:ttJHHHUl\~!t<~
:i' n). 1r11 ,... " . " '.H,tl?;:-,
",•·.,in'
.. ·, ;; ,:;)J~~o '.!f ,., ,,., '

.....,., ...
.,'· '":?;:•:(!· , . FLUID DYNAMICS EOUATIONOF MOTION l;,o. ,':
1 .. wlth two similar equation$.
=:> VP=.:-'vp,:
·.. p
By•Euier's equations ofciotion.
do· 1 ·
: Ifwe write, Q =n +
'
J/
C p
1 then the.last bhcomes
==F--V",
£9.
dt ·
'
p ,.
a2x ~· +-i:t ~ + i12z az .: - £i ... (1)
or . dt + {q . v') q,, - vn - VP . . 012 aa at2 oa a12 oa - oa
1· . Similarly we hav.e i
But 'v (q.q) = 21[q x curl q f. (qi, 'v} q] , · · ·· 2
.£9. 1 · • u. ax· ,
~£l'. a2.z az .£2 .•
cit + 2.v'q 2 - q nurl q= - 'v (n + P) at? 'ob ~ a12 ~b / at2 ob "'-: ob• "' ~2)
.,
ltu + v .( n. + p + J. i;,2 )··~ q ~ w. a .,, £! + . ~ + a .: ~ . , _ N
2 2
or ·l ,., (3)
. at . 2, • i1t2 ac c!c' a,2 ac ~c
/
Taking curl of both sides and·noti'ng t,hat cwl grad" O, we obtain . ax , fb:. •
a a . ·x•;=-;,=u, >''" .,u, z==,=w,
:1.,
aw Put
' curl .!!.!I.,. - curl q ;= - .. cu:rl (q x W} "t ilt cit
cit . at · 01 . Now (2) and (3) ·are expressiblo as .
3
or ~"' q (V.W}- W (V.q) + (W.v'} q - (q.V) W,
., •
au 2! ~ e!l ~ + clw dt _ aQ
at ab at ao at ab " Qb "' (4)
v',W = div curl .q «·O and equation or conthniity is I au ax. av aw ·az ~ ~
.:. (5)
£2+p(v:·q)=O ai.•a;;·+ at. +Tt'a,;'"'"' ac ·
dt ·· . . Eliminating Q between (4).and (5), we have
aw WE!.fl. · · a · · · ··
Hence ai=O+p dt +(W.'ii)q-(q.v')W \ .a;;L,H,S,of(4l=~L.H.S:of(5).

[~ 1+q.v]w=.ll'.'+(W.V)q \. li.e. ftax +2!!. o2x +·a'1,u ·e:1. iiu.E.)'.._ a2w az a~ a~z
or
. clW ri.Q. i l ac a1 ab at ab ilc iic ot ob + ai ab oc + cit itc ab + :ot ob clc .,
or di=-;;- di + (W.V) q, I ·= a2u £:1:. + OU + o2u Jb:. + au h_ a2w £!. aw a2z
ldW_}Y.E!.fl.-cwv l ,! ab,atac a1 obolilc utaca&+a1ia1:ac+a,·abac
or p di p2 dC ' )q p · or ( au _ ft
2
)+( £ll. _ o2ii £i.) + ( a2w £! _ _
1J!1 E..(w,)=('2f..,v), ... (l) ~ilbill ilciJI , . iJc .aciJtiJb iJbc!tc!c -O
I or . di p·· ... ·P,
This 1s calle,l l';l~Jroholtz vort1c1t~ eq~at1on. If.we·'-";ite..
·.q . . '· or [~(au~-
. at }b ac
)'-a!!A au a2x.]
ab clt 90.+ ac iit clb ·
·. . · .
. W=~l+TlJ+~k. !q=u1.f-UJ+wk,. l .I [ i) aua i) Jb:. i) '"i)2 i) ;:,2 .,·
• thenthecartesianfo,Q'f./.iJ? a a ' l . a a a' +.:;ai(abf-a~ab)-a~fi+a~'aT~l·
(u,+u,+w,+-ai)(~)=-(~-a z z] 0.
2 2
p p ,: +ri;;:;+~;-)u +[a(owoz - ., -aw -a- +·.aw
- - - "' -iiwilz) - -o-
a <JYa a a )( n)
C/X
( a · a .a ,
QZ VJ C/l ,
at 2 ab oc· oc ab ab at ac .ac at ab <I

:, !Ii ( 11 a;+va;+waz+ai p =-;; ~ax+ria;+~azJu


1
But .~A_auau au axl_auau
;~ a . a a a )(.2r ) - -1 ( -,+11-+
a a a ') •, ab at ac - ab ac • ac at ab - ac ab ·
'lIl (·u-+u-+w-+-
ax <Jy • a. at p -.P ~ i'!x i)y ~ -ili: ,, w· Hence terms outside the bracketz cancel so thot
Remark: \.-or p: canst. (1) was originally gir;e. ri by Stoke and Helmholtz and i'! ( au ox au a '( av £l. av ~) a ( t!.w c!:11)
' .. (Mec~ut I/111 2) at ac - + ai /lb ac - ac ob +ai ai - ab
! !1;
,later on exended to the abov<1 form .by !.fa.nson. ·· . , i'!b i'!c "O.

!il . '\
Theorem 7. Ca.ucby'.l9}~tJ,J. ~llls :.Lagrange's .drodY,71am1cal equations are , te=at'nu w , , ;..•.',• 1 ·:· _,. ;:):, ;.. . •
aa,,a,
x ilx ;lnlf~ii a<_ an"::'1i· ~ ,\' . a(· ou ~ · ; ; · ., "(a:~~
N'. 1

ml
'i
l
2 . ,n -~,.·av £l'.) ~w ) ... (8)
...,. ,,,

~
a,eao ado ao ao p ' ab lo - a, a, - a, ab ' a, a,-,;- •'
)1;l
,,~

r
{l: 11
,< ;,,,,,,,.,, ,. •··'·•·:::~;;:(:!:::!;.'., .. :·,:,,,,;•,:••·•''t••""'"'··"'''..•.,11:;J?liil;;;;::';!~::c'.:T~:~;1}Jl!l!:\l~~lt/?,ffll~~'!<"::w'~.T~E'.'i),."''if.<'-!i1')li•".fJ.;,fll.iic:i"'i•"l'.~::,~f,W\<,~.,,;,,v,'.»~•'>::1'tr.l!~';':,"~:,~ffll-•~""""""'ll"' ' ' ' - · " " • • " " ' - - - . . . ; . ~
ii }!
~
~l
~l,,
\I "' ' :'"""•!:~~:::.:: . ,~· ;•~~
ll>'i :j ,':,•:«: ,.,o ,;. " _,::::::::::::=· ,"\ :· :-:::: ::·::~:::· ,·

}
~3,f.'
.b
\
·i{,·•a,,;,.;·:•,,•i-, ,, .. ~.,•:~~·•:~. ·: .~i ~~:::·: (:·:'
: ; I : ; : ~ ·'.; '. ~ ', : '. ,. •, ; , :

t,A' ,,!$,_$, - .n,•.-,si.:c.;;,.;,;,,_~ ,,,'i•·· ;r.?i~;'M'rn◊!.J,~-;:[~:.


.
-.-'.,~---_,.
. . .
.....· _-.
'
__ .

.:,::::!:-: . '; .. ;. '~~- .. · ., .· ,:- ~l ;: t; -:;., · . , ,f::~;..;,I'


,, :~···· ,,_.
,,
:1
:'1
~~ X"'1',!','l~""""'!IN'l:t~ttr.i"(\.~,.;_,;,,,,,.,h••t:,I.,.""'"""'"•-""~'-.''"'• ,A.•••••·i·~-.,J'J.1<1~~~-~0,~wr.•~.,.,,1,-,-,.~, •.. , ..,,. ... , ..... , r ... ~, ., -.. -~:_:·"' ........ ,.~+ti.t~!,~~,1"11~n'~.4~i·~;,~..: ,:.~ _._, \~;. -.. ~-: .,:_~t.:J.:f)!U!.:.: •. . .. ,,,J11)1.~n.,,,,..

~'. . l '
t11 I ki;,JATION
I '. .
OF MOTION' ',

~:., .. "

Initially, i.e., at I = 0,,: =· a.;y,. b, z = c so that


FlVID DYNAMICS

I· I W '"iY.p ' V)0 r.


,.; : :~~-: QT p=',
{ qx .?.l. c!t ax £}'.
0

aa = l, ab"' o, oc "o etc,


' '

ob,= 11 fc =;• ·De1ductl'on 1. To prot•e.the print:iple of permanence o(irrotaticnal motion.


~
Subjecting (6) to this condition, · Proof: If WO= 0, i.e. if-~= 110 =!;a= 0, then (10), (11), (lt/.)
l .
, -au • 1 ) +(-aw • 1 - o) = c
(0 - 0) 0 + ( O•-
i)c o ob.. . · o Wo"O
=}

~
~"11':'~·=o
W=O

I or
whore
aw)o- ilc 1Jo"~o
,c.=·( ab (au
W ...:~url q ., ~I + T\J + t;k .
.
: ·
!
,I
: This proves that ·if the motion be irrotational initiaUy, then it is always
lirr!,tational for. all time. This establishes the principle of irrotation.al .'n\qtion for a)J
1time I, . , .' ....... ,. '. ' : -·
,,};~.[·
'Iii'
.. au clx cu ~::)\;( av .f!)!;
·( ilb ilc oc ab
oli
ilb i!c t ao ob
~)+( iJw ilz ·aw· ilz)
ob. iJc cc ao " ~
ID~ductlon 2: To prove Cauchy's integ;~·l~ are the. integrals of Helmholtz uorticlty
! leqhations., .. .. . . . . . · . . · ,.
or .. [ 2! ( £.!! .E + au j.£ +l!!.9! )..:~(au.£!+ clu £l,,;ilu ~ )] IQilj To proue-Hill.mh(!_ltz eqqations wit~ the help of,C.auchy's i.nUgrals,
ac o:,: c!b 7>' ~b ' CZ ob i)b ax oc 3y ~c I Bdc ~ au·gives
.· ,. • · " [ fu: .Ei ... ) ] + [ ac<·'j')-'ab(
a, I az ...)·1J=.,o I
.I
;: !
· · iJu· l
Proof: (~O) X ox +(11) x ·cly + (1~) x az
ac ·(...'. )-ab(
·rIii' or ~. il(z,x)
il(b,a)"'+il(b,'c)'lta(b,c)"":'°
o(X,Ylr ,_

/
I
: (7)
.. ,
I
II! I t.(•i)u+'".•.:du ,au)=~[dU.£!
p ... ax. ·,_ay +,. az
£.!:f..e>'.; £.!:f.2!') ,;
Po ·ax aa + ay aa/ a, aa. + ... +_::·.
0

= SQ au+ llo au+ 52 au,. .i. ( ,E; )'. a~ix,rdihg to (10).


·I Similarly - ~ +E..£tE. ; ~ -
il (c, a) I; o(c, a) .:1 iJ (c, a) i; -.'lo
... (8)
' I
Po aa Po ilb r 0 ac dt . p ,.
j ~i ~ a(y, z) r - r ,!(~ c/u + c/U + ~ c!u)'=!i_(s_)
iJ(a,b) .,+ a(a,b) 11 ' a(a,b)'"-,o
... ,(9}
p ilx Tl ay oz ot p
·I -Mulliplying (7), (8), (9) by
l( ay ~ £!!)-Ii(!!)
.
Similarly ££_
p ~ ax .
££+
I .
. /J;i:i ilt
a;;::,
ax
ab' ac°
TJ az - at p
·l ( c1w + aw + c1w ) ~ !£ ( ,!;. )
1 c\Y ~ a•. - ~ •
respectively and adding do1Jmnwisc, . .I and P ,~ 11 ax a1
l•· ~'.i~" ~-
~ il (a, b, c) +'Ia (a, b, c) + i; a (a, b, c) - l;o aa. + 'lo ob+ ~o
~ ~ ~
oc ·
i This is equival'ent to single vector equation;

~ .. But • P~=W=Po I , p ax · I\·.29.+~.29.)~si..('i!..)


1(1;,£3-t il:), . az . dt p .

I ,
(c!,:b, c) :·.- .. 1·:. ·· d(W)•
Hence,
, Po · ax
I
ilx
l;-=~o·'!'i:'+110~.+(;o-:;:-
c,: . ,,I,
or;.•' p(W.V)~~-dt p ·
. p ra . QV • ' QC I . . .
.

i
i .

s.
· = ~o [¥ + Tlo ££ + ~ £:.
I or. 1i ·d
dt
(W·)·.
p =.·(W \'
7p. V)q,
or .. , (10)
I>'
t
.!y1, • .
p Po· ilb Po ac·
Po ·ila
I . . .. . I · This is known as Helmholtz vort,icity equation.
Theorem 8, Equnt!ons for lm~ltlsfve.A,ction: To obtain general equations"
s·Jml·1 ar 1y np = &l!•Ei +110 .?X. · ~0·9.l:.
Po ;aa Po db + ;;-;; iJc '" (11)
, 1 : of motion for impulsive action. (Kiinpur 2002, 2003, 2oot;,1
. P~oof: Consider an arbitrary closed surface S moving with a non-viscous fluid .
and Lhilz-t !Jo~+~.£! '" (12) such that it encioses ·a volw:r..e V, J..et q 1.and ,Q2 be fluid.velocities a,P within S jusL ·
P - Po _oa Po ilb Po ac '
before the impulse and just afler the Impulse; Let p be fluid density at P. Suppose
The euations (10), (11) and (12) ara called Cauchy int,gra/s. The vector form of
these equolions is ·
I is the-~xternal impulse per un,it mass ~q ii) the im,pulse pressure· on surface a
element dS. Also let n be unit out',;,'.ard noi;ri,lal vecotr. · .
:i\; J!.=(~L+!J2+SQ1-). r · Change of momentum =Tota\)mpul_sive forces
: ;

P Po oa Po Po iJc ·
"\.1 . <«h,·,.•••••
•.~'.ti,1,
wm~~lH:~!:;,::;;;,;~};~;¼jgj*S~81Wl81~J;~i.~t;~i~{l,~;iH~~i~,;~j~:{'.:~/~,i;::~❖~f-i~t$tll1/,'.t;)>'.,;,:::,',,:":,:::~.t•: .. \:-::•\)i,_~{l\!t'"X~~xi.t:•:.'h

·~· 1'

FLVIO 9YNAMICS EOUATION OF MOTION .· 4'2-

(P :.i i' =f
~j'(._Ip dY, t J- n~
i.e,, ~ Inlegrating~p ($2 - ~ 1)_ ~ ~ neglecting con~:tant of inlegr~ti~n
j (q2-: q1) dV •
I I

' •
.1I I or oi=p(ti-$1),

. [For ti) acts along in~aMnormall tm) 'l'o prove w-= p$ under suitable ~ondltions. Let the extornal impulse' be
By Gau9s thefrein the last , i ·: I absent so that I= O. Also letp be constant and motion starts from rest, Then (l) gives
;i.

f '
IP(<?,2-q1)- ..Ip+ViiildV=O.
i; ! ' I qz - o = o - ¼'l?wl[
•I
l
~Inca tht motion.starts from rest by th~ appliQation oflmpuJtive,prossure henc<1
I
Since the sunre s is, arbitrarjl,and hence tbe in~egrand of the;
:
it.must be irrotaliooal. Then. q = - V<j), \ · .. · II
vanishes. · ,
- v~ =., lp Viii or v (P<b
'
- w) = o:· !.
P tq2 - q1) - Ip+ 'viii= 0
lntegraling it, p¢,- iii= 0, Mglecting cons~a.nt of lntegntion, , -. I:
or . I - -l v-w . - . . . . I ~· !'
... (l) w = pip,
.
qz - C!1"'
. ' p ' '
or
If p·= 1, then ii)"' ,p,
1
'
(Kcmpur.2001)
i' I
'l:'hisis the required equation for impulsive ection. If
I,;. l (X, Y,.Z),
then the cartesian equivalent o{(l) is
'l2 = Q2 (U.; v, w), 'l1 ~ ql (L<o, Uo, Wol,
· ·
R,eruorlc: HI= O; q 2 =.0, the_n
( 1) = -ql =- l "Joi.
.~ .
:
~
\

p '
. . X "loiil ·, . z ;:>
11... -uo= -Pax-.' u-110 = P , w-_~o= - . Further,ifvelocity has one component, then this gives :·I
Deduction (i). Vorticity in a non-vi co us incompressible lluid is never g~nerat~d
aw 1
u=--=-
1 · ;,

. p ax' p \;
9)1 impulses if'the extenal forces arc con~ervative. · !. ! ! ::
· Proof: External impulses are conservative ⇒ I= - 'J!t or . dw=pudx."
Fluid is incompressible ⇒ p_is constant. T11is equation is uery important for furi,her study.
:def. Flow: Consider any two points A o.nd 13 in a fluJd, Thevalua.oftba integral ~~
-=:-V( n+?) ! ' ·
or
'By(l),
'9
.q 2 ~_q 1
x (<fa.-. q 1) = 0 as V x V = curl grad" 0 ,f A
8.
(u clx + v dy + w dz)~ f
A
8
q,dr
\
i:
or CW'lqz=curl<;n or·,W2;,W1,·
From. this the required r~sult follows. ·
· (ii) To prove ~2o') ·,. 0 under'suitable conditions; .
laken·along any path ,in the fluid, is called _flow from A to B along that-_path. If the
motion is irrotational, theo the flow is
• B
JA
B . 13
J -. I I
~
Proof: Let the external impulse- be absent.so- that I=: 0. Also let P'.be·constant. q.dr = V$.dr " - dip ,q,A - 9B
Then (1) gives · . A · A
where•<PA and <Ps denote velocity potentlaJ9 at A nnd B, rospectlvely. \.
q2 - 'li "' - l;7 ( %) : ,.u (2 ) D:ef. C!rculat!_on :- . i
t:

or '9. (q~ _ q 1) =-V. v(fil) .. -v2 (. ~-) .


F:ow along: a closed path o ls defred a1 circulation. !'.
i:{,
or
p
v' 21ii = p [-v',q 2 + 'v.q 1J: p
p I ' I
I- o.+ Ol or vzw =-0,
: circulation'= J q,dr, ; ·
' ' I
For v'.q 1 = 0 "v',qz is the egua'.ion of continuity, lfithe motion is i;rotnlional, thcn,circujation;: ¢A.:. ¢1B = ~A - ¢IA" 0.
im Remark: If the mo'tion is irrotationaJ, theo ,;:
:f\t F~r a dosed path, points A and B coincide, . • , r
iill
!: T~eo,rem ~,IKolvin'a ~ir,~u_lat!o_q. thdorem :.~~1~ circulatj9n along any closed
pi.th moving with the lluid31,f.cons~~n\ /dr all ·ti.~flf;tf thinxi·ernal forces ar,e
i
"··•·.
'I or V [p (<bz '".~1)- oil;,, conserivativp and density p iii"(unction _of pr~ss\lre p only.
! {Gc,rh wa.t 20011 M,.r&,I 2002; Ailra. 2001, _2001)
:· . . t' ,,11
i~
--..
i11!~ ,1
:':'.:\:/: :>;:' ".w,.,•.,., ... :'' :• '~"-'.'·' ';":•·: .:'.'.11!1iitll,::;;,;:· •;:::,: '• •:;1':'.'.;';':!~~)~:t.!\1~::~:~)'~t.'fil:!~!f.':Sli';!\Mii:'!'</;\<·•:1•::t.,:<::~,,:,:..,,,,.,.'.':>' "'';":,:\1!':!i:r.:\i:;:;;.;:;,:,s,:;·;."·: ',~~'1N':11'1:')!lffil1Nl::t"i'!l"~;>,x'll':ll-li'~'l;~!IR~,-{f,,l,'ff!N!l<',\"'>.....-"-"'!l'lfi!i),'<>!'~1'+1~'-;:,
f
Ii.
!!'
l , , :::·,: ::·::-,~ :-;;,;:;:!::;··· ,•,:,'1'',',''•'
,,:,:,,:,:,:• ' .•.,.,·,··:::::~~::?::?~~ •: •' .,.. '

~. . :: ,;;:,:$~~'.'.·:; ':::: ., '' . '~· .. '' , ~; ,! '.~'·:~~·:,,. i· ·;,:~;::::•; :::• ;, ,,, ., .


·,······◄ I I< !l'i(O, •I -.l:!1Si,i,l,,,·,,.,.•,b.'..),'i',',•,•,.,177.f
·. .~::::~:...:..:.G-~ _-_.,_,-~~-

:.:·,,,,\:. ·,,·.:;:,;«;:,! .

. ,.;,,•,•·

' .
.. ,, .. : . ,... ;·, ·r;t.,.-;.~&~1"~)<.,,H1:,1r;;i~1:·.~.::""~~-u~~~;-·;. •·:;:·.:ac.:.H:t•,. ·. ,, . , .....1... ,, ..,•:\•'..: ••;,~•:.n:~1•1"''"'
~r-~O'\'-~;J:,~ui..~~"fU1.'l.'1.V',~Vi\:llh:~'-'"'.,_._,,,.., ,~ ....:~•••·••'•'""~••••• ...•.-.,~~J,~?\g1',~~•'l'Jilt~!~t?."tiil\t•.r.,,,..,.._,, -., ,,
I •

;/ fLUIO DYNAMICS ;_ \.·


EOUATlf>N OF MOTION ,. _ ___,_.•_
r\·.·
!B.
Proof: Let c be a closed path and cir denotes circulation. Then ' eiemerit dS, Let the force ba_couservative so.that F =- v'O. Since force potential n
''.IIi
~ I
cir= f,. 1.dr. j is,supposed to be.independent of time, so tha~
. . 11Q.
Tt = O.
d
Further di= at+ (q, 'v)
ii
1 ·

an ,, ·
1!
;~
'I
'9i (cir) = J. [1:Jl . dr + q , }i (dr) J Hence
do.
dt = Tt + (q.'v) 0. 2 (q.V) fl • .. (l)
·,
1 :1
';)
= f ['!ff ,
C
dr + q.dq ] [ For q , ft (dr) = q.d ( ~; ) J
.
· Let T, W, l denote kinetic energy, potentlol energy and intrinsic energy,
respectively. Since Q is force potential per unit mass·hence

1··. W=J fldm=J. Q·pdV


I = J[(F -¾ ?p). dr -~ d ( ½q2)] t
Jl . dV=-lJ q pdV
·1:~. :C .
by equation. T= -pq 2 2
, . 2 • ... 2

= J. VQ -;;1 Vp .) , dr +· d (. 21 q ) 2
:. Since d ' mass remains lnv;iriant
. . element!lfy
di (P dV) = o.
. throughout the.motion
, hence

= f [(-~
,C
2
~'.d~}'d(½q ) ]as dr:? =d
.
,
·Now
.··dT lJ.d 2
,;'_'"'-=- SEJ.::.pdV+-
lJ : f . q2 D=
d ..,.i:
q \£9.pdf'+O
! ·1t··•1d . dt 2 dt. 2 .' · . ' dt v.

'•1
"' ~[ Q- 2 q2 + -; 1"0, . I , .. (l) . , las q2 = q2 = q,q]
I

'
For, on R.H.S. of (1), the'qllanWties involved hro single vnlued a;nd on passing
. , d· ,
dWJdn
dt• "' . dt. p dV + J·'Jdn
. n,o = dip di(+ D
onco round the circuit, the chnnge ~xpressed in (1) Is iero. Thus di (cir)= 0.
Intrinsic energy E per unit mass ·of the l1uid is defined as the work done by tbe
~
Thia"" clrculnL!on is const~nt'tllongc for all tiµles, unit mass of the fluid against external pressure p UQder the supposed relation
Tlie'orem 10, Permanence of/rrotntiona_l motion 1!(the nio1io11 o(a 110n uiscous beti,.,een pressure and'density from lts aatual state to some standarci state in which
fl,uid Is' once irrolationnl, 'it remains irrotef ional ,;y.ieii a'ft,twards, prvvided the pressure and density are pt and Po respectively. Then .
e:,;ternal/orccs are conserual,iue .and density pis cz fimction of pressure p only, ; ' .. . Vo· . .·
' I '
·, Proo( : Let ; denote a closed path moving with the fluid and cir denotes
(Garhtual 2001, 2002, 2004) l= JE p dV, E= fv p dV Where Vp = l
circulation,
· ~hen ~ir,. Jq/dr = Jn, ~url q dS, by Stoke's theorem, .
I

.iPo pd ( l ) = -
"' ·
p, p
, .

·r• 1 .
~ .p
dp
C / . S \

· theorem cir ls constant for all times along c. Oonsequenily cir along~ is zero for nll
tiOles 1 · · 1
i
'
. ·StiP.POse 'motion ls onc,e ii"rofati.onnl, Then ci'r ii long c is zero. By Kelvin's E= l p·
.1dp, ~encedE=1dp
Po P · \' •P
. l'
:! J[~~
.. - ., ,

cir;,, 0 ~ I along c
i.e.:· .
7 piiV+Eft(pdY)]= J~~ pdV+O
· . The1,1
. .
Js·h.. curl q dS ·= 0, Also S ls ~bitr_ary;
. I
' . '

·. · He.nee 11,curl q,. 0 or curl q = 0, this =;, motion is lrro.t.at!o_nal for all times. I
I
.;f. dE sl£.pdv=f · .E..sk!.pdv=J
rip •dt p2 dt ·
t42a:v
p dt
Hem:'11,motlon ls pennrllle11tly !rrotatlonru, . · · ·.·
•'
Theorem 11, To pbta.i,i equatio,n of,enercy, .. · .:.
I
· (Garh.wat 2004)
I '
.; s·~(-p V.~) aV.
0

··u F£ +p V.q = b
,··:?.root: Consider a.ii ai-1:iftrary closed ·surface's xnovlng with a non:v!scous fluid
s,.t. it ,eocloses a volume V. Let n be. the. un!.t Inward drawn normaJ vector on an is the equation of continuity.
' '. J~?:;;;:·~.
.I "'"' ·-···

~-', , H ,:f,\:!1~~.:t1l:.t1~,;:.{.:~);'{H~M'"f\'•: ,:, ,;,~,.,Mi.\,;(\ ~l~K!U'l'Pffle:nnntnfffli~I.-IU' ! lH~<H.!i!j 1);H»~


,:i-:~;,;~;th'l~·HHaMa)~H·•st<'.,.S}i{\ '1',F r'.i(, J' :,·
,-' -.',\'I),'•'.,,.\~,:,'. I'•'',',•. . ,·.,.,,,~i(,:,'' ,.,, •., ..... ·•-.•:11'.,-'.,t1 · ... ,,.

I
•...::I I ' . 1 ·I
i ~-
.... b•! •'" •~" !° i'"'°' l!OUATION Q_F MOTION

I .. WORKING ~OLES .. ~ •
. .
.....~

01'
'dl
di
·.' · '
.:':",f p (V'.q) dV
I
: I. . lnjorder to Mlvc the equations ofmotlo~, we
·au ' l CII
ndopt U:ie foll'?wln'g tcchniqu~.s:

I . S!!l
'·.
'J
' . : I ' ' i)I)
( l) Equation of motion is at + u ax° .. F\- P~ · •
Finally, dT =;
-d q. ·. -d
d !p (/u; dW = dQ p dV,
. -d I, !I 1' ... (2) . ' ,;i)Q '
t t . t t ' '1
I' ' I
where { F=--
·ox
dl = -
dt f P (V,q) dV
iJ,i.•
I!
I ... (3) (ii) xv ,. F (t) for cyllodrica.l symmetry If p= const.
x
(2) Equation of continuity (i) 2u =F(t);for spherical aymmcti-y lf p = const,

· . do . l· '
By Euler's equation, =d,. - VQ -·- 'vp
. . t p .
(iii) ir + ~ = 0 (gcneal case) .•

:•' . q, ~ p dV= - l(q.V) .QJ pd~·- (q,Vp) dV


Oeneratly
r the fluid is aasuinod. to be at.rest at infinity, i ·
.i.e,,x= ..,v=0;p=n,say,, • (
Integrating over Vand using (2), j . sphere), tlien x= r, u =;, p =O,
1· · ·
dT _ (q.'vQ)pdV+ f ·.
1·1, I
I (4) !fr be the radius of cavity (or holloi
(5} ¥fhenr=a.,·u=Oso·thatF(t)=O ; . '
dt+ (q,'vp)dV;,O I. Boyl~'s law·: p 1 Vi ;, p 2V 2 := const. Its alt~tnate form is p " ,; p.

dT ·Jdip . J· '- ·· . · I
I,
n, Flux= cross sectional areii:, nonnal yelo~ity. density.
or · dt +
dn
dV + . (q, 'vp) dV"l,0, ~y (l) 00 tquation of impulsive ·octfon Is dw =pvci..u pu' dr' ~
n/2 • ' · ••. ~

J J sinP oco~q e~e 7 r( P; 1


), ~( 9; )/zr( P~ i + 2.)'
1
or di":
dT dtdW +, (q,'vp) dV = 0, ~m. 0
. rt ·
r(n)f(l-11)=-.-andr(n)r (
n+-1) = v,i'f(Zn)
2 1
. But V.(pq)~p V,q'+ q,Vp - smmt i 2 2~-
,. \
I
'r 01'
1 v'.(pq)dV-rp'v.qdV= r(q.'vp)dV K.E. of~he liquid= wor~·dona = J:. pdV.
f
r. d.J J ,, dV, (10) If a sphere o( radius a is annihilated, then· when x., a, p ·,;, .6 so that
=x"
o. . ·, .··
J
h .
or - n. (pq) dS + dt = (rp) :\lY_(,3), u
l . as;;. is lri\>ia'rd normal.
U i) Ifa problem contains cxtenlal and li:iternal radii, I.e., R a.ndr, 'the)). subject
. the re$,ult(which fs obtn,!ned frora tbo integr,atfon of the equtitlon ofniot!oii)
I d ,
di. (T + W + I) -
J .
dS =
I •
to tl11i two b9undary cond!Uons''for R. anil r,' rn ·this wny we ,obtain 'an
: equation free from constant of.li:itigration C and pressure p. Again we .,,.,.,
I Now (4) becomes n,(pq) 0.
- integrat0 this equa'.ion to obtain tli:~•required result, ··
I d r if 1 ' I
or d {T + W +!) = J n, (pq) dS · , I I SOLVEDEXA~PLES
I s I 'I ' l"robleik 1.A sphere Is at rest in an infinite ,¾.as~ of homogeneous liquid ~fde1tsity
11 This is ener!l')! equation, This proves _that: rate of chan!Je of totn1 ~Aergy (KE, p, the pfessure ,at infinity being n, shol;') that, ;if the radius R. oft~• ~phere &aries in
+_Potential+ Int:rinslc),of a poriio!'I of fluid is equal to ~he work dbn1 ~y,cxternal any ma,jner, the P_ressure at the surface of the,s'p_liere al any time i.s • ,
pressure on the boundary provi_ded_ .th.e external forces ru;e conserv~liv'eJ': d 2R. 2
, Coroll,:i.cy:Prln~iple of enerjl'Y for ln_compressible fluids; trl' the present
case/.= 0, Henco the' rate of chaog~ of total energy (KE. + P.E.) is equa) lo the work
'
. .
l
,
dR ) 2 , !
n+zp[-2 +(di
dt
' .
l -
done?~ th!l_ pressure on the bo1:1lli:la~. : . · , _. • : ! ., l( R: = a (2 + cos nt), show pi~t, to ~,:euent!cavitat_l~~ _tn the 11,uid, n must not be
·>~::,.:::-· ' less tharl 3p a-2n2._ ',-. · · · ·:-:-,,:. ·'·• · · · :,_";':-:'·' .·
., ....":. .
I I

't"'.!:':,'~l~~;::~:~~::::~1i1~f1tf1.,,r~tr~»uw-rn~""R,8fH~1iiv;·,•.,...r,·.~w•i1~,.,;.,!.,...,-.• lo;...,,.,~r;·•••~"~•'""·'~,♦"l.,.!-~{'f,H.tt.,"<.;,.,'-l...._'<1,.\:
'·::!:~::·:;':

' ·:-::·\'.-' '-:::·;{~:-:;::: ..,.. :r::'.:r:.:. '-"-'.,r,,:~,;~;•;•;i'."'.•'•'•'

J,:~ \>;! ::• /\>; ,, •., ,; ' ,,',!(1'. ')\','1~ .','"• : ,,.,,·.. ~.. · ,~d· •,·. 1 ,, ., .•;·:,~~l~•3·:;::v '· •'.··. .• .-~~~;~~;:,1?,: ;v, ~'.--~'..'.~':;:, ' ,l'',,'.',,

ijm
------~·-~..,, , ,.,i.~~.GIJl, • .;.;.;~n.,.~,i:,,.,,,\,.l.r-;.;,'.bh•~MOOv!,,:7;l:;7

:-:-: :,:) ;,~.\:•,.,;,


·:,~-·-~
-~· .....................~ ·.·- . ;

,,,..,:,,C,,·,.>-,·- ,,•, ,_,;,•,·•<", \J,sht!h~i;\t,-'..,


j ,,::,::,,,. ::,,,:.,!
•~!n:·'.• .

. ; ' '
.. ....... -~--~lit1.l.tffl:.'a:¢•1~d\tnl't'li'~~\......... ~ ~ ~ l . i ! . . ~ ~ ~ l ' b \ ~ Y i , , , , . ~ ; , , . , . , , ... ~' .... ,.,.,,t~, • .. , ... ,,.I>•· 1,' ,d,'e.- '!, .... u;r~~Jf;,;,,r~t'AUk~-~,.:J :..;t:;i~·a.;;li: ••.:.i~.:..; :,;;:r: 11:-.~;,.t.1'.1:.~~lt!W."''""''•.;~1~... ;,, ...-1•j,:.t.,1?:. ~t/(~~,d~t\i!;'\"',\'," ' .
I
! j.

-~
' 'J.::!f •
FLUID DYNAMICS
T
l eouinbN:orMoT10N .. ---.
'"it ·

! .'
I . '
~+;~;a, Q -½ ~
••.
2 2
,· :~\-: SolU:t!oo : Equation ofi motion ls a~d equation of 1· Pmtn = l1 t ½p .i2n1 (-:- 4 - 2 + 0),. by (7) = 0 - 3p a. 11

continuity is_x2 v ~ F (I) so.thnt] ~~


'fol
'IF' 1l.
' %·
J \ ~-
I
~'
,.,,,ff,_r.: . .
:
0 ,
2
p>O =·'·pm,;n>.0 =o n-3pa 2n >0 => n>3p,:i~n
2

Hence F') ' l + -f; (½u 2


) '.I - 'f; (~) as p is constant. ;1'J roblem 2,.An infi riite mass. of fluid acted on by afor.ce ·µr- :v.: per unit mass is
yl"directed to the origin. lfirriti,ally th.a (Zuid is a rest a.nd there is a cavity in tha form
of the sphere r = c in it, show that the cavity will be fi!led up after cm interua! of time
.: Integrntmg
. w.r.t. .x, .:.L:JD. +-1 v2 = - E. + C, "' (1) ; (2;, 5·µ):112 c514, . ·. · ·.
X 2 p
Boundary conditions are : Solution : Let u be the velocity, p the pressure at a distance x from 'tbe origin,
. (2) when .x = "", p "' n, v ~ o, "' (2)
then t.he equations of motion and continuity are respectively.
' au i'!u • 3/2 1 £1?.. .
(3) Whenx=R,P"P,U"'R, ... (3)
a7+uai=-µ:: -pa:: .•.

Also :: 2u = F (I)= R 2 R ·
aridi. i. 2 ·
.x u =F (t) s~ that u ,.
Ft
y, a·
0~ ~ / )
F'

F '. (t) = 2.R R. o· jfe.)2. +R2 ,, 1


!:.J!.2-+ -a ( .1. 2
1 ae
Subjecting (1) to the conditions (2) and (3 ); ·' 1...
• -v ) =-µx-a12 --.·
, l • , ! j ,:Z .C . ox -~ · p ·ox
. -n ., . I. ilbtegrating,
1
' • • F• (I). l
2 2. . .
0:+0=--:-+Cand ~;·:, . +- u = e.!L 2.-+ G. .. , (1) '
! ' ,: 2 {£ p
F"' 11 11 1 • · . • ,. ·· ·,h n are ·
.-~+-mf--
R 2·, ·
2 +o-~~+-
p .p p

1
Boundary
I
conditions
. •,,
: 2,. Vlhen :c ~ ooJ u • 0, p = O
or ; 2. = .!L l (R.)2 :t -$J
. p p 2 "·
l2R <fi)z + Rz RI
.
.;i. · ,Whe,o.x.:' r, (radius of cavity),):'= O,.u "'r
4, Whenr=c,ti,.Oso-thatF(t.)cQ, .
, or ; .ti = n )½ P (3 (Ri2 + ZR RJ .. : (4) ,;. Let T be the required,tii:ne of filling the cavity.
'. Subjecting (i) to th~ corrd!tions (2) and (3), . . . . . ·
Now
2 2 '
da~' i-1ni2., ,fr C2.R·R) + R.2 ~ 2R.2 ~ '2.Rk + R:2 : : . . o+\.,'o·..,o+C ancl -Fr',(t)~-1 cr2i=2JLo+C
. . . 2 . 'Ir
' or .' - F' (t) .!·z.. &!!.
· Now (1) becomes p,~n+½P[d;f +R 2
) '" (5)
:-~inc~
r·• + 2·r - fr'
r2 (i·)"' F (t), r2 dr =·F (t) dt,
. Second po;rt : Let R =a. (2 +. co~ nt) ... (6). LAlt there be no cavitation in lhe Multiplying by 2F (I) dt or 2r2 dr;
fluid everywhere on the surface so that p > 0. Then we have, to prove that ; ' - 2F' (I) F (i) dt Ji'2 (ti 2 d _ .1J:!. 2 d
n>3p.a 2n2• j .: . · l' r ~. r4 .r._ r-"r .r r
. We have~ " - an sin Ill, :.R " - an 2 cos 111: or · . d[. ,..F2.(ti]=4µ ;aiz dr, .
Ob~erve1that gR R + 3R2;;. ?a (2 +cos rit) (-an 2 p9s [ti)+ 3a 2n2 sin 2 nt .! ....:. . r '. . . ~-•
· ! · ·~ a2n2 (- 4 cos nt - 2 c~~2-nl + 3 sin 2 'tit] I~tegrati~g, -.~ (I) = ~µ . ¾r 512 + A. ... (6)
· :,. a 2n2 r..: 4 cos ~i:- 2 ~- 5 si-112 111! . ii
. : . ;.S.ubJecth:ig (6) to (4), ~~ ¥ 61
2_-l'A
·using this lri (4) . ' ·· , · · · I 0

· •,• p;,, ri ~½ p a2n~ c-·4 ~os· nl - 2 +5 s~2.~t), ; .. (~:) Now (6) ... . - rr2' i->
r
2
•,. m± .(·rs12 _ ci;12l .·
5 ·· ·· · I
t • , ,
1 I

'J\s cos nt varies from-1 and 1 and so.R vnries from a.to 3a, by (6). Thus spherb
= .'E. =;[·]if_ :.(c6t2'-" >(2) ]l/2
· ... shl'h.1ks from R ., Sa fo R .= a tµ1d so ther°o !ii a i:x,sslbll/ty of cav!iation. Also- p i~ . _dt . 5r3 ·.
!llinlmum when nt" 0 or 2mrt. lnegntive sign is taken as velocity increasei1 when,. decreases]
""' .. .... ' .. ,
~

I -: i::, ·:: ,\''' ,,,,.._, 1:,1, ''.l'!ri'.',;rni<l\n;:,:m1'li;;:,,-.,,:u.m;,:.,:, ,,:.:1<1,c;,im»'"''"'"'"""·r.:·:•:»:oc,;,:,.,:,,"""",_~,,:u.,c,.<""'""'·''''"'""'


'i':ll°sll~~V.!iWll.!U!fWl'l!°!~Wllllij$Rl,'l~lll!il'if><!t!m!i!~•\!\N\l{i'.! 11ll,,tl,:1!l•1ffll!l~!!lm'1Wll\,-·l"i!-:•'•:sll>l!lii!,!".~Xffllffi~lll!!l::WAl-'1'1li\'':>i1!"1i"ll•·:·:'•:· . ,''.
;,;{_1 ~i/r:J-:.;u:,.:.~o::~1!Htb.P.t,\~~J·'/iJ:i•~• 1; ;: 1:,r:-,1 ::<m,•1, ~•. r·,,·, ,•
~

) i ; I
''""•••-., .. ,, .......
•::·f .••· i i'lJ10,0YNAMICS EQUATION OF MOTION ).t:S,. ....
. . . Tl
. ·10 . .;r312 . .... . .. ,· T· · .i ' r
u;. dr,. ~ (~·)v2:[ 9s12 .-·;.612 ·]112
- c · (c5l2 _ ,-5l2jl12 dr =
.
t ·(l~ · .
8
:· .
1/2
di • I
. ' dt 5 ,.
. r
3

• or T., ( 5' )11_2 ·,{ · r 312 dr


; "' (7)
Time= T., -(
. 8µ
0
.!L')112
. ,
f- r
312
(co/2:.. ,5-;2,112
dr•

."( 5: r2
8µ )
0
(c512 _ r5l2)11Z •
5 3 2 . c61~:
Put -r512 ,= c512 sin 2 a, r ~. dr = c512.2 sln e cos e dO.
. 2 1'1 '.
I .Q.. )mfrv2. i5 C61~ • sin5/4
0 cos.a dB -(.Q.. )112 ·.1 514(-: e )"12
Probl\11,I 3. Steam is mshingfrom a boil;r through a conical pipe, lh'e diameters of
T .-_ ~( 8µ. .- 8µ • 5 C ; I COS 0 the ends of which are D and d; ifVand u be the corre'spondlng v.e/ociti'as oft'ie stream,
,,1·
.
Q ' • , C .COS 8, .
1
:. , ' , and if the motion be supposed lo be that o(. diuergence from th~ ve'rtex of the• cone,
or
2
T" ( 52µ • . cs·14 • )1/ Provecl.
~w~t ·
2
·
· ·i
.!::. =·Q.. ,<v2-1t2)/2.1 /
V ·,dz·
2
Aliter: Equation of cootinuit1/ isx u = ?q ... (i) u is velodty at di'stance ;vhere
x and vis velocity at: a distance r. KE., T ofliquid wh'en radius of cavity is r: where /1 i$ the pressure divided by the density and si,tpposed tp be constant.
' . . .. ' . . . ' i· . . · ;, . , ; ', · ' (il:fe,enit 1993)
' i"" 1 ' .
T;,,j . r 2 (4JtX 2 ck,R)u 2 I So:Iution: Let ,u be the,v!loc\tt at a dis!an~e,i: from the end·A,'.the equation of
motion'is · '.~ '. '. i ··
. ~·· I ~
u~=o·.:lEE. ,,,.~ e"H

= 2npf x2 (
. r
;,z: )2 dx
X •
i ax p,itx
(Si'nce the motion l's st~ady)
~~: .)ov
: a ( -u =--= ·· Fll!'.Z,S ' .

=2npv2. r 4 ( - ;1·)-, or l 2 )' ' Ji c)n , . t •


'dx ..._ asp"""P
= 2npr, 2r 4 [ , x2 ax2 ·pax !

·
lntog-ra\ing, ½u 2 =-ldogp\c
= 27!pv2r3.
2
If n ls force potential due to external ·forces, then 2
or logp ;.logA 1 =-:vi
U
· or '
~."A1c•u /2k
,
- ~=
<.:X
-m
:,:
as F = - ,&o, Boundary conditions are
... (1)

(i) p = p1 when u = v
Integrating Q = 21µ1 ,
X ,. (ii) p = p2 when u = V • .
Work done by external forces
Subjecting (llo (i} arAf (ii) wdobtain.. p1 i,,A 1 e- "212• ond p2 ·.. A e- \1'1;2,1
.' ' I 1
.; I. Qdm I f <4n.x
r
=
r
2
dx. ;.i} This
Pt
=:is-;::
1
v )12A
•• , (2}
·P2
' IC By the equation ofcontlnulty
i 312
I,. = 8npµ :i: dx Flux at Ab Flux at.B I:
\
r d 2 ' D 2 • I
.. lQ ,cpµ (cs12 _ ,612) 7th) U,P1=7th•) .V.p~I
'
By principl~ of ~n,ergy, work
,5 .
K.E.
. ,! '
or
Pi · V zjz ·
1(3, .. ,:/• ..:··" ' . ·, ;i';°"'frdz ::,:.r. :. ' .. :
o,: 5 n P µ (c~IZ ·"" 2,ipv2r3 ".
........

·,::~;m;iff::~~i·~~~~irnn~1~tmmrr::1~:~~~r,~':t.l 7r~-1: · ·'.•,;;~J ·: ~,•~ . . ::;·:::~~:· ·. 1•1 1 !rt':".· ·· \~m~ttf:'~it~11f!l~~~H,~~~-~""""l't':rr.r.-"'="""~';"'--~,,HtlMWW,.·"o~:

,•,·,,'.\' »:'<·'.·•:, .·:,:::·.:i~:, .


. -: ; ': ·. ; ' ~; ' '.·:: '.: :, ,,,;::.:::; ;,,::;;·,!·:.'· •. ·:,./:.•,•' I ' ,.• ',:,~•.:~,:, :•· ,\: .• -:

-------~=~. n ,., .,, .. ,n, lltxl>~-.Qzfo.,.,i:,·A!d,,11, ·(V1!{,\\:X«,S,$!,:~

..~
,·-.-,.~"'~"'":"":~~ .. -...
.' :,;~-: ,;.;.' :-:, :. '
~ .. , •- ,.,:.~ ... . •_,\•'~! •'. ,

H
~.l
~ ~1'1'U1'Utl.t.l'lt'l:':t~~'"'~"...,.""'~~·,:. • .... ,, ~ ... :,: ..:., .... ,... ............... ,0, ..1~:'-~1,~:u::"''"'"\·.•l'h'HMl,~oii,,,:..t•:-~i;;•::,:: •· .·rs1•:·, ,;:••.\'i'.:U?:::'t1U:t!!:,. ~,,,,. 1..,.... , ......... u,..v,;.H:n,H:trtt~li>J,,·• ~,- . ., ..•, ... ~ ., ••,
j I

! .,• .'-+-•·
0
l . .
I .
FLUID DYNAMIC$
; '
1:0\.!ATiON OF MOTION· ·: • ».

V 02 :
• Now (2) becomes - ,~a~
u d
6
;\'2 •v )12A
2
I
I' 11

',I ~ ~
U
·-=-•-
U.

: · Tb prove that stroom lines are the ·interse~tion of two hyperbolic cylinders,
~-
~.
.:. (6)

·or V il 1•.12 ~ \11)121<


v"'d2e .
i ~qu~tions of stream lines ~re . .

!
.i ' ~.,~ .. ~-
u, u w
Problem •J.A mass o/homogqneorls liquid is mouing so that the L'e(oclty al a11y point .;;r ,,
is proportional to the tim'e, a.nd that the pressure ts g_iven b;y ' Using(6), dJ: = itt. = dz
F1 F2 Fa''
. .e "'µ :.O,z +-l 12 (:y2z2 + z2x2 + x2J,) ~ =_!lz._·,. __ dz__
p . 2. , By (5),
(µ - l) :n (µ - l) .zx (µ - l) x:,, <· •
. proue that thi$ motion may :~ave. been generated (,:om rest by finite natural i~rces
tndepende?lt of time; and shou11that if the direction offr!.otiori at euery point coincides• or , <k =!lx = dz or x dx = ;y dy =.z. d.:l.
with the direction of acting forces, each particle of the liquid describes c curve w,hich '' ~ u q' .
is.the.intersection o(two h;yperboUc~linders, · . . · i This ""' x dx "'Y dy; x dx =: dz,
Sol11tion: Velocity'is P,roportional i.e, q =1,,t .. , (1). lf tim/, This represents two
· Int~gration yields the result x 2 - y 2 ~ a 2, x~ -z 2 = b2•
Also ~ " µ :cyz ,-½ ,2 (y2z~ f z2~2 +.i;~;,2) . . ... (2) ·1 . d.isiinct hyperbolic .~ll!lders. Hence the result. · ·
.: ~.rbblem G.l;ir,.ob~ing .B~!e's l~w. fin;motion l~ a uniform tube ;I small-fection,
I

. S.top I, Let the motl~n be gcne~atod from· rest by finite, natural force F
(corisorvnL!ve force), then lhero ,ox.illte velocity pdteQtinl ¢> a.t. q = - V!j). To prove that · (f;~~;'hat ifp qe '.~}-den~ity.cnd u t~e ue oa/ty at a distance .x from ~,fixed point at
i 1
:F.li"!ndepandontofthno, .:- ':"·: . · · ... , i
·. j
: .• ,•• ··., ··~·· az·.:l(u2.>t k).pJ, . . e _;;.·.
By pressure equation, , .+ ··l!, l
2q
2
·7 n-. .2.t
.
a, ,.F' (I) I I· ,
. .
· •. . :
·.
,.
·-~ 2 2 c/r . .
where k,. . , · ·
p
. . .. ,P
E!. .. ~ - n..:.· l·;.,2,2 ·r':F (t)•. '1 · : ·, . ' (Kanp~r'2004, Meel"Ul'~CO:J, Gar/w;aHOO!J, 2003)
p at 2 • · /.. (3) Solution.:, Equii:tion of cootfouity ls . ·. . . . · ·
q=i.i, q"-V¢> ~ ¢,•t((x,y,z). . 211.c . . . ..
••
• ~.+'a; (pu)., 0, ... (l)
Wrlto ~ .. f, etc.; (3) ls expressible ae .
" . of lliOti~n
E~uatlon ' .,is..., '
l!. ='r- n _l ,.2,2 +F(t), • ou cJu 1£2.
. p ',. 2 ' "' (4)
' a,. +v a;"--~;;-""· • "(2;) .

Comparing (2) and (4), (- G = µxyz, ~2 =i:y2z 2,. F (t) =0 i: By Boyle's law, pr. vol. = c_onst.
...... !' But vol. densi_ty i: mass. 1
Now ,-2ti; r/1- = (v',p/ = ~. ('li'f) 2 = t 2 .V,,2 +f/ +(/J ,,

or i<.2=(;+(;+/.2 or. r.r;=r.y2z2 Hei:ice pr. vol. = Const, voi, ~ ~ ·


.P
or
.This•
!
I

,.. ff - y 2z'2 = 0,
''I. 1/~2 -iz2)., 0..
f/- z. 2:?" 0, (,2 ~ r.y 22 2 .= o
l '.al
'' p
mass
pr, - - = const,
.

:::if= xyz .· · " Thls => •/?; ,,,'co~·st. ==,k,.say ~ p = ~p. y


' i • p .
Wehnvcseen thni." 1f-G=µxyz, this,;;,, ;
:r.yz -·.n = fU'Z. or. F.=: '17,Q =·v' (11- l) x;>-z
*y (2), £!:. + V QV = - !d.P.
iJI ax p iJx' "' (3) {

,;,r ;F., (µ - 1) 'Ii' (xyz), . · . , .. (5) . i u2


'fh(a => F ls ioi:lep,,nderit of I.
· To determine-~·:.,
. ut
Step H. Let the d!roction of root!on coin~!de with the direction of actin~ forcr.
~ci'thnt i . · . , · . ' !
By(l) 112" a[ a
· ·· ..::-.i::.=---~(pv) a ]
- c) [ -(pv)
ax at ·
' . , ' .· af ot. a:.:. · ·
.•·-~·-

r
··:,·•:'·,7;:,·..
-
I
I
'I
:· I '
\ flWIO DYNAMICS .,,,
EOVATION OF MOTION'
j '· · · ~ !d ! ,..

.,P
_e ___ a ,·a~
·a (uE2+p-)
a2u .!(,u21~or +· 2lwp·• ~fl) ko u 2
cit 2· - ax oi .•. at otf " ar c)r + ar2
an~)·. j ·.··( k =-u;-
=--rJ:[ u(-= ao a~)',J.· +P - .... .,. l.. [ v2•.2!!. +2u.'(- fu!. - av ) ]+ 'i ~ · by(~},
ax · .
ilx . p i);r r:,X • · dr .ar ·. 0/ U Cr ;,,.:. I

.,1..[·u·~+!i2-2+pui!i.]
. ox ax · ox i!x .
: o [: au c>v ] +k u, a2
,._ -:-u'2 --2u-
iJ [ il iJ 2 er I or 2i)r ot
,. - - (pv,u) + -iJ (kp) -
.
(pu 2 + kp)
ax1 i3x . · ~x i!x2 a2u
z+',
i) ( 2 au · i OU
u -a
)
·"''•2•
o2u
a2 'a2 .. ., '' or
at or r +2u at . ar. I
OY £:.fl:,.-:'-2 fp(u 2 +MJ·
af ilx Prl)blcm 7, ·A mass of liquid surrounds a •soiid sphere ;f radliis a, and £ts out.er I.
surface,.w:hich is a concentric gphfire ofradius b, issubj~ct ta aclven·const'ant pressure
Problera 6, An·elr;ulia tz11id, the wei,ght of.which is neglected'obey(r;iq Bo;yle's is in n, no other forces being action o~ the liquid. The/1, solid sphere lfW:lden.ly shr,inks into
motion iri a u,n;iform straight tube; show that on the hypofliesis of parallel sections a concentric sphere; it is r,equir~.d to determine _the subsequent motion and the
the velocity at an:} time t at a dls/ance r from a fix.ea, point in the tuO.~ is defined b;y impulsive action on the 'sphert, · . · ' ·
· the equation ' · · '
U
Z . 2
+ l. ( 2u ou + u 2£ ) = k £J!. .
2
Solutioxi: Equations of motio,n.and. continuity are
. au· ov. l iln ' . . :
a12 or at or ar2 -+v-=--=·
at: ax, p'o:t · '
. . . , ' ~ ' l
·,Solu ho?) : Boyle's law is p = k as volume!"' p. 1 . X2l/ ::F (!), ' e
... (2)

~ =~-
I
,.Equations oC continuity and motion aro i .
Heoce !..--1::!t- 1 2
F It) i) (' -u i) ( ll.' ) .

.@. + £e!!_, 0
1 xz axz ·o:ir~
a, .i!r " ... (l)
'i Integratiog w,r.t •. x, we get l
i
.2!!.+u.££,.,_l~ _E.'...ill+lu 2 =·-e+c; · ... (3)'•
i!t i!r p i!r • ,x . 2 p , . , ·1
I
Since'the liquid. is contained between-tw'o spheres r = a, r = b, so we suppose th~t r
i,e,, 2£ + ti OU = - ! £e .,. (2) and R are internal a.nd external radii' at any time t and t.he correspon.din·g velociUes
<ii i!r p i!r
are. u nnd U, respectively, Bo.undnr.y condltlons· are
02
To· datemiine " ,
2
By (2), we get when x = r, v = u = p = 0, r, ..... (4) f.
i!I • (Slnce pressure vanishes on the internal boun't!ary)
_aa,22 u = 1. [ u au +!d2 ' _•l.. ( ~ !u. + l!. 2-2)~ When x=R; ·u=R.,.U, p,.n,' . ..:(5).
\
iJt ,cir p arJ-ar i)t·. pat (Since outer su,-race is
subjected to const~n,t,pr.esauru l
i!nu)]
a [. ·( : · au k iJn ) +· •-k.(· - :.=. r =a, u ·=i. r.•r O so that F (t) ,. 0.
I,,
i!r U, - ,. U -i!r .- p- =
- bY, (2). . ,.
• ar . p ar ' . Subjecting (3) to tho cooditions (4) and (5),
a
2
<l.or \ u2 ~ + uh ~.e +hu £2 +k ?.Y,] _:11''(tl+luz-o+~ :jI'·
at 2 1. ar p ar p. i!r ar
.i . r:
F'rt, 1.,f).
~
n,
r :•
::,.
- :.._,;:.i. + - v- = - - t C.

'fiia[ u Tr
auk~]
+ii a;, " ai a;. 2u1 )+ a,:D(k log Pl ]
a[1>(1 . R 2 :p I ~
'j
2· . Also r2u = F (t) = R2U upon subttactlon, '-,

,._a_(l\.1
a, ilr 2
2 +k
..
log P)·
I F''(t)l½-~·l+½,;~1-¼~-¾] .. ; • .. (7)
I!·

a/J({ v.2~\tog p) l I ., :. . •, .· ,.I .'.!,,t,•.,.,;··. '• :...,.,


•,·,.f'' r .. ,; I
. ~ 1.[·)',£!;/!..~'.Ji' Since ?u. = F (t) = ,R/U i,c:,
';.:ldr f" F (!) J(;;'Jl 2dR.'· '.,\..',

iJr ~ ·a, p. ar.'J . Multiplying (7) by 2.1<' (I) dt = 2rZ cir" 2.R 2 dR, wo get !i::
!,
,:

:: :,··•·:.'ullamntm~~'.",'.';"'!:~,:;.,'.!'.~n'dl:fil~·,::i1;:a:i;;r.·;r;::;;:B.;::::,r,;,:;:,:•,~7~.,"'~,,.~,,..·,,.:,·•:,•,,~,,·,,·,,J¾,·,,,.,..,,.,,w,,,,.:s,,«f:il<"Jll:·:,~•i:r\ffl':~1'i-i'"'~m™"'1"'ittliWl!.<!-"'-''""·=--"1'~~""'"''-.-,.,•.....,..,,_,,.,...,N<,i•
,:~l•r,••H•' '•' ,>,t'.'\•::::t::•::••

~ :- '
.: ·:·::~ :.::'~::.,; • ;. : .·,:·1• ·r:r::;:'-~ .i ·-:..•,;,:. . ....:~
'•,;,;::,, I '::•:: ,'.:

'.1;'('1:;1··
'.'•l·
fr.I ...... 1.1,,~~••s:i~...x,,, 1,:p:,,o_.,.,f.1 .. :r;n:r;i:~·~mi"iTI

-, ·::-::?:-::;;,;:
,,,;i.:,(•3•,•. ::, !•.•1'1:•: ,.,

ij
I
,,
,~~~-..~'ff,-...,,....- ••..:.;......,., .........,~~l~c.:U!LOt~,,,1.,,.•:i::M:lil::•~•:m><.:11:c:.·,....:. ..............·......._..,,. .....,.i"'·'~"'·~~·......~-'--·-·'····:••«•"""",;;,,..,,,....,. .• .........: , , ......... ,.•~~.._:;,,.;,o)<1\0,'ft•t~\;,>\',, •,',1,V.:••',HI

\
\ •:.,:..!:l:l FLtJID DYNAMICS
'! :.;.

-.~
•: "-: 2FF' 11.- 1,idl F2 !;k.- dR
Rrf
'.+
,r2R2
-- I!p'' 2r2 dr
· EO~;TIO~Of'MO:J'ION·
,.
:in~gratixig w.r.t1 x,'
'or d [( 1 1J·.Ft]=-
- - -r
R ,
11··2r2 "'r
p • "'· ·
;. :I - t:..'..ill ;
.:,:
l2 via " - ep + c. ... (2)

Integrating,
1 1).F 2.(t):--,TI+A.
•.

(.,-- : 2'r
I
I •
3 be the time. of filling up the ca,'ity. Boundary conditions a.re:
when .i: "'"", V "O,'p,; n, (since constant pressure n is.applied ~t infinity)
"' r · .· . 3. p · ·.
. . '2 3 . I (i!)•'whenx,=r=radiusofcavityi11=:u=r,p!.o; ,
Subjecting this to (6), O=-!L
3 p • O+A • (The pressure variishes on the surface of c.a vityj
Subtracting, we get When r = a, so thati, =u =0,' F (I)= 0•.
1 l)d>.·:·2n.3 3
to
Subjecting (2) the conditi~n (i),
( R-; r~I).= Sp ,.(r - a ) O=--+C
·n or C=- .
n
p p
· ( R--=..c. · 2u}' 2 =-2n , (af- r 3)
) (r . .Subjecting (2)t~ (Ii),

or..
with •
ru3 R _;)
, -
rR
. R- .. -(a 3p
R 3 - ;r3 '"b 3 -
·For total mass ofllquld Is constant
f(
.
.
• 2n s -r-)
,
·
3p · ·.

' t}
: ';,;);'., (I) + 1, ,.,2., 0
·r 2 ·
~~

·:..-~-_(I)+. 1. u 2 = [I s.t. r 2 u = F (t) = ,:i.;, ·


-~". 2 ... ' p
,t~; ..,..

~ volume ofliquid nt any time t volume ofljquidinitially. = or ::E..:.©. +l


r · 2 •. r4
F\. llp · (·
4 3 4 3 4 3 4' 3
~ --n.R --iv --1!.li --na
·;
3 3 -3 I 3 • l Mu1tipJying this by 2F dt"" 2r2 dr, .
. . ' ZFF' . .F2 . . n . 2 ·
To determine the equation of impulsive action, Equation of impulsive action is - - dt - - dr = - - • 2r dr
. F . ' ! , r ,:;.. . . . p
dlii=pud.ta.Qfd.t · · F2 . . .
~ w
.
. )-. R
F'
.,. .x : . . ' . or
·.
d(-):
r
_Q.
P
2r2dr.

"
I
'
,
v.
dlii= . Qf
, r
x
cb: =- pF
. R
1 l
1
1
j- --r, J- - - I
11
)= pr2u lr R,
ntem'ation yields p2 =- 2n ,,a.+A.
-;:. 3p
2
or .. ,· · . ·, <ii =· pr u (-
r - -)
R · ' Sub/ecting this to
. The whole lr.apulso on the surface oftbe srhere ls I \ I_ . 2n .
0=--a 3 +A
. / . 4ni-2lii = 4nr2 pr2u ( ~ ) = 4nr3 .pu ( R~ r ) 3p
(82.~d (!;!) liTe U{e required oq1:1atlons. : ' Ans. : };Tence F2 2n 3 3 ,-4-;.2
- = - ( a - r ) or - ::m· '
.,-(a -
r 3p .. • r 3p
Proble,:n'.8,.An,Jn.flnltefluld In whlcA a spherical hollow shell ofradius:a is initially dr = -[· m_ a3 _ r3]l/2
at·rl!(st under the actibn of no for~es: If a constant pressure n /.s applied at in[i.nily, or
dt Sp ' l-r3
,.
show tha(_the ti,:ne.of/il/lng
I ;
up( )i/2
tlie cavity is n2a n .. 2510.!r(l/3)r 3. _
.e.
INwaliv~ sign is taken as velocity inc.reases when r decreases].

Jo ·' J.[it?..2n .
T D ..
.· • . (!rfeen,t 2002)
Solution: The eq-Ualions of motion and are
,,a ]112
dt = -. a a3 - r3 d.r
£!I. + V di./·= - .U.e.
ol i:Jx · p i:J.x
... (1)
-(1e_
T- 2IT ;.,I,
,ll~
... (3)
x 2v=F(th ·
Hel)c~ F" (il + 1.1
.x2 • iJx
au "_ H..e.
p ii;,:
l·=Ja. (a -r 3,.a 3 d.r.
8 ·
r3
:
=a3 0, 3r2 dr-:. 2a 3 sin 0 O'./S ede
;l :,,, "/•,) l: -'.·':.).'i~;!)'(Qf,::,;1;:~~¼~~7;•;\'):0,''•·~r.:f:•,_~_;~•11 ~t,, ,,,; ,.~;.:tr.."• ...,.. y ..

' ll ·
;1- ,.
:
1 jl
,I
i
't,
l;

rpi
,k, .
I: C!YNAMICS
liI EOU~TION OF MOTION
........
12 I l
j ,./ ~3/2_sio e. 2a 3 sin acos ede .! '
I
:u 11
Jo a312 cos 0 . &-2 ,. KE, =. 211 p r 4u2 ( ·-1 )'" ...2np r3v2
. · X r
·n12 · Work done by outer pressure .
2a 3 sin2 e,de
J
= 0 · 3 (a sln 213 0) 2
. 2 jlt/2
"'...!l. ~- . (sin 8) 213,(~os !:i)o d8
. = Jn\4u2icix) = 4lt n
4r; ' 3 3 '
r r
x 2 dx
.
;
i.
I
.3 0 . . .,
1· =3 n (a - r >
r
. (5) ?lr• , (5) By prin¢pie of:energy, .• '1,
'~2a .:r 6 ,r:_~2..,£,r,;-· r 6 .. · ·
8 . ·2r(.l!+l): 8 ·,r(1~l'\
· 6 2· . · 2}
I•:
i
I;·
, . f. 2r.pr3v2 = ~1t O (a 3 - r3}. I i:
i;
i:
. dr ( 2n )i12 ( a3 - r3 )112 I

iirrtk:ct(en·
br
u .. - d.t "'. '3l) · ,' ~ ii-
·f"'·( .... ' .
'" (4)
dt = - I ( 1Q. )lh--,'t:-·31_2..,,d...,r-::
2IJ , · (a3.,.. ,a)112
~
::
Recall that • • t, '

' :
r{n}r(l-;)=-i_11_, r(iiJ r(~ +~2i )=,r,;-tpn; . From· this t~e required resul~/ollows. c

'• snrm. . .zn•. P~oblcm 9. A pulse travelling along a /i.ne straight uniform tube filled with gas
F l ( l ·) .( 2 ) ( 2) rt 211 . causi?.9 the density at time t and distance x from the origin where the velocity is u to
om = 3 ' r 3 r 3 r 3 = sin (11/3) = ;;lJ become Po ♦· (I.ii -:,;). Prove.that the velocity u(at lime I and distance x from the origin)
0

.. ' (u 0 -u)$(ut)
and . r('!)r(.l?.).,-rnr(¾)
a a r 113
,s giuen by· · ut i > (ut _ x)
Solution, Equation of continuity ls
Hence 5 ) -/ii' 2l/3 . 2it . :,,4/3 rr-1 l Q.Q+Wf.-o
qt i'Jx - ' ' '" (1)
"r(½) ·~.r{½f~'. '·[~(.½jf
(
r 6
we ha.ve to P:ovo
. (u 0 - u) ~ (ul)
. 2 .2413 . l . . u,;u+ ·Hut-;i:)
using this in (4),
J.,.ar.. •T3'•[ r (l)J
- ' i '•I Qiven . P = Po ♦ (vi -;x) ' '" (2)
. .. s I
' : and .u = u0 when i"' 0, .. , (3) ,;
Now '(8) is reduced to ·.. ' a
. · .: ;. . ·( .Jl
·' T= n2 a r'!··
)1/2 , 1 .
,. 2516 1 , I ,Proved,
(2) => Wf =Poi)♦' (ut'.-~l. f =- Po <ll' (vi -x)

. • : . ·. <[r(})J' I·::' Pµtting these values ln (1), we g~t


· Aliter: Let u be velocity when radius of cavity Is r.' Similarly:u is the velocity Po u<j,' (ut+ x) ·+ u [-,p 0,r (ut - ;,;} + Po qi (ut -x) :u " O
when radius is;,;, Equation of' contlhuityls · ! o , ,
;c .
X2U,: r2U I 11 j , '" (1) or OU $' (UI • X) , '
(u-u)<ll'H>-a =O 'or t dx=O

I , '',I {
X I : u- u 4t (ul - .~)
J;
KE.= ½(4it,;2 • Pl u·2 Integrating, - log (u - u) - log qi (ut - x) = - :;
or · , \,(~,.-u)J(U!:.. i) "'A :1,
:·:. .,>:: .r.:•:~t,-2 2 .. ;, .;"
ru:; in view of 13), .this.,. {~ ~'r.icif$ (vi)~ A .
'" (4)
., .•..
,,:2'( } ) dx "'2r(Jr4u2
!
"211 p r
2 (u - u) $(vi!.. x) J (u·- u 0 ) 4> (vi)
X

.. ,'•·;·:••/•:' ''': -;••,..,...;-{·,1\f,f:".o;i.;~:il·~Uti+H~~M\l"'fN~~~"'!~m~tt'Oflf~~.r..r,.r•,r-.,~-~_;,"'7",~~-~!-:-:"'"'!""~l~-t4111~H.:i~,of.11'\~(1ii:;;,


j;
:::

, ;~ ,l~ ~: ,\: ',,', • •'•, ;1·,:·.:•:,1!' ',\:··,;.;,f;};, ": ·: '· ', ,: . ' : ' ' ·~ :: ' •i ~ :, ; . ,.-;; ,,;,,,,.. ' :; •:i ~ \t~i'~'.:,; ···::;:1:.,:-· ·::~ ;:'.'.;'".'

-~•=o-r;•,l",•"·''.l,,. 1,Me,2w~;,;J~(i
<::'<~•;'. .. : 1};;;!(11::,· ., /::.~~!~,;: ~·. ,•,
•·,·,:.,:t ..•:, .·::,:,:•:;
, ...........,,,... ,•.••'
,•. '· ,,.,,~,,-,

···-··~•"""-""·-_..""c,~IW.')tl,ll".....;,. ........ ·..... U""'' •••.•~.',::.-,,,''. ···- ... ::t:.i,1:-·.i.:..,;,.~,;.;;~•••t~....lef.l:io&Uµ:.:'.-~:;.:.. :::::,.;:.:.:.·::,:i,:.:t. .. :,~ri.!:~h..rt.'.Cf. '"" ....,.. ,,.•• ' .,, ......... ,:,...,.,... , •.,., 1 •:.;',t:;)!,!t•r .• ,. • : •'~,,•,

;:!
:;~
. 'B"i ·:_ F_LUID OJNAMICS EC'JATION OF MOTION ~.'Ii

-~
..''\":
•l}
:: or u-u=
. (u - uo) $ (ul)
N
. ow (2)
.
= 9'h =2s2( 2A:1
·I
- 21 ) or h-- l( -l - -l1' \
', ' $ (11/ - X) I, • • B' . - 2g A2 . .B2 I .
•' I ,
:; J (uo - uH (ul) · l This concludes the problem.
or I<\ ::q1+1 ¢, (vt-x)
I • . Pi-ol)lem 11, Show ihat the rate ;er unit'oftime at uiJiich work is done oy the internal'
Problem I 0,A $lream in a hori.zontal pipe after passing_ a contraction in thepipa at pres(ure _between the parts of a c-ompressible fluid obeying Boyle's law Is
wliich tts-seclional,area is A,:is d.eliucred at atmospheric pressure at a place.where ciuovow) dxdydz •
&he. sectional area is B. Show that i{a side tube is connected. with the pipe at the former
pla_ce, water will be sucked up through it into the 'pipe from a reservoir at a depth
(-+-+-
ax ~ ,OZ • I

s2 ( l where p I$ the pressure and (u, u, w) the ueloclfy al any pc.Int, _and tk'e integro.tir:m
- 7i - 2l ) ,'. . ·
below the pipe, ·s being the delivery per sec<>nd.
1(Oarh wa l 2000) , fxtends through the uo/wne ofth• {Zu.i'd. · ,
2'/1 A B ;
Solution : Let u and V be t~e the velocity oftha streain ot two cross•_sections. ·' '• . dW
Solution ; ,1-!,t W denote ':ork done, then rate of work done -is ·dt. Let
The..~.[luation of continuity is given by
Oux ut the nrstcross section"' ciwc 'at the second cross section q ui t pj + wk and dV, = dx dy dz-. .· , .
i.e., A.up-.. BVp ·=-f, (siv7n)
(For flux = cross section area. . .
.
j),1"
A •
B
:U, V = ' ;: Then w,i~a;~ 'to prov.~ t~at 1 ~f p ('v.q) ~V. , .. (l)
. . x deris!ty. normal veio.)
Also.()= l for stream. i
~ s
Hencou"'A, V= 13 ,
··
W= f ~pdV

ilu !. .?£ as the


~ + p ('v, q) =0 (equation oi~o~tinuity) · ... (2)
The equation of mot!on a.:r =-:: p ilx .
motion is steady, F)g,2.4 dW J· dn · Jah" · . :· ,
~

·or -\n
:zu~ .,_~
il ( l . )
_'a; as p=l,
.dt ';- -=dV
-=
dt
.. - =wasp=pk
, dt ' .
(Boyle's law)

· fd"
7t dV = - k, ·J ,. p ('v.q)
/ !
! l •
br: dW
-;[;' = - k
.·..·' dV, ·
Intell'!'atJng,
'
.12, u2 =-p +C ... (l)
I.

! ;
by (2)"
Boundary condition's'nr~ : ·
m u•~ p•n _ " ('v,q) dV= Ip ('v,q) dV
(Sfnee stream is oeliverd at atmospheric pressure p,. n, say at a pl~cc j .
· where cros·s•s~cii~nal n'raa is B). Hence t,he resl,\lt Cl),
u=u,p=p. • Problem 12.A spherical mass offluid of radius o· has o. concentric spherical cavity .
~fro.illus a, which contains gas at pressure j:, whose m<l$S may'be ·neclected.; at every
.
and (li),(Hg!ves 2
l
v2"' -.n + c point of the i::a~rnal'boundary of tne liquid air. impulsive pressure-w pir unit area is.~ .
· l 2 applied, Assuming lhal"ihe=gas obeys Bl'lyle's law, show .that when the liquid first ,
• 2 u =-p +C. homes t~ rest, the radius of the internal spherical surface will be .
.
~ ~p' ( - ui2 /2pp a2 (b - a)J where p is the c!msity of.the 1/q!,!id,
Upon ~ubtractlon, n - p =2l (u 2 - •h l ( s2 s2 ) .
v-) = A 2 -
2
82
i ; .· Solution : Equation ofimpulslve action is aw" p.u d,: and ecrontion of confoiuity
Is:
; 2 . I.
or n - P, = ~U 2 :- ; 2 )> I! ... (2) , I·
I -.
:C21,/
.
= J.i' (I),
!:-... , dx
·. d1ii = p F(t}. 2
' •Leth be th,e hoighi·of.w.nier colum/i,ln \b:e sld~ tuba which \s sucked fi:om 'a
feservolr, then n - p"' dlffer.~nce of pressure= pgh =gh.as p' = 1. I
f! II . . X

..........,.,,.,.:,, ''

i!l?¥Y.0Hf.f!'IT.{i!,tfdm1Hr:mmmnmffil!tl\ll!lllWWVi!1;~;-,1,rrvln':!1N~!lllW'''i)H,?:H\~l'l~Wil'fl!l"l!."vl!M1fflll1!l'.Rlll!!f;!•)Hii\-:ll',:':ic.''\!O'.(•:·.••:·· ,:'.',
;;:,, l· ,;;;1;,::;,i:O~ii..;m;;wtHUt\Jtm·o:;w: ❖HH~c::h:~n.:•t,,v,v.1,. .,,.

!
'I' I . Il
'; .:··-,,q:i_~:··.: I Fl·1j.10:DYNAMICS
:
\ ',',

[LJ]
EOtiATION OF MOTiON ~ ... SJ..
I
l
- w b ,; I I .. . . . . . .. • • ,.. , .• Wo\o, Air
·J . "F·'dx i Solutlo!1: Suppose that the cylinders A nnd B nro
p 'z ' :.
Th lS
. => cfiJJ"' 1,
fille;.d with-watar aod gas,_r~spect.l'(~ly. Le.~k both~ cross
0 ' . a :r i I sectiqn or each cylinder. The water a.n'd gas both are at ' . ' . '. I1
or -· · 1 (1.)b "- (b:a) .. rest before and after the communication fs allowed c·
w=pF' · - ; ab pF(t). 0
..,(1) between the cyliodei;s .. Beoce initial a.id final both KE. . T· l
are zero. Chango in K.E. =0, · ' • I
Let,. be the radius of i!lternal spherical cavitr and p 1 t,he pressure there, Sfoce
gas ·obeys Boyle's law he.nee _i · :
This ~ Tdtal work done= ch;uig'e in J<i.E, · ,Fli:tG
1 ==> Total ~erk done = 0, . 1
. i
I
43 4'3 --~-
-3 1V P1 = -3 lt a p o~ P1 =, 3 • . ,. ·initial potential energy duo to water in A .• f
, · , r I ,
,= 'Mg/I' in us~al Mta-tlon.
.
{Internal cavity of radius a contains ga~ a~ pressure pJ.
• ' I
t
!
Finally, the liquid is at rest.,

Gain in K.E.
. Ii
=} ·½(4ro: dx.
· .
2
!
.
1 1 :
.. b
p) u2 = 2np} · . :r 2 ; dx
.
as

, f C
= u (kzi)
:
gdz = 2 k':pc
and (inal potentJa) ene~gy.d~e tolwatr o£lieight c - x in A and• heightx in B is
1 ' 2
.• I
1
.}
i
I

i
a • a ' , . Jc-x . 1 ~ J'" ; . ·, .,
·,:

i
· ( - ;1 1
"',2np ~ b- a
)a F 2 = 2npr ° !•--;n, 1. · · . (kzp) C.CU. +
0
(kzp)
2 2
2 kgp l(c -~) +x J.
0
g1z ~· ;
·f
In
· ., 2np (·"ab
_o.).. p2~(b0 -a)z
2 02 . .3 Now work doo~ by gravity:·loss poteotla1 energy
. u,InJtial P.E. - Final P.E .
1 ' ' '
- l
I
. = 2n ab ¢iZlp (b'- a). = 2 kgp_ 2
!c~ - {c - :r) - x J = kgp {ex ~ x 2~
2 !

I • . , . ,. ;;
Work done in compressing the gas from radius a to radius r i:s J.-p dV in Work done by gravity= kgp (ex '-x 2). ' ·" ... (2) ~ :,
A]so some work is d~ne against the compression of air in B. Letp be the pressure y
usual notation • of the gas when the height ofwater•level in B-isy. By Boyle's law,· {
l'\V1 ,,p2V2_ ot pk (q-y)=hpg ,kc. !
2
a ( -,. ) •
dra 3p . .-4npa)og
4nr . !
=- [ ~ 7 Tb.is~ p=!:.e8.E., p peing density of~ater. ' ·
l
, a r .... ,0: .. , . c-y: . . . . • !
But gain in KE. = work done. I· I (For.·press~a = lid/ad heigbl,. de11sitY:• I and Initial pressure of the gas ln !'J !s i
!:
2nab¢iZ · · .. ~ equal t.:o pre~sure dve to 1Holumo. Ji·ofwat,er (given)}. · ·
-
( r,
- - = -4npav log· a- :-J :1
'•
I,,
Work done ag!l,io.st the compression of gil.s·ln B ~
!
'I'
p(b-_a).
r-
'. (, . ,.i)
I I

wb . . 2
or log ,,-. = ---'"'--''---
·a 2 2a p p.(b - a) . I
- P.fl,. in usunl ~otntlon. · ~
o ) I ·.: ' ;
Hence. r " a exp · j 2a i pp-2b(b-a) )
00
• • •
.•
m
J
z
-
(
h ( P - kdy, dV•Jidy
. C -y

·
1
I
0 . I=
Problem 13. TwJ· ~qµal ·closed. cylinders, of h~ight c, with their bases ;,} the same
horbontal plane, are'filled; one wlth iµater, and the other with air of suclt a density •
= hgpck lpg( c.- k).
i C
... (3) ~,
~
as to support a column. h of water, h'< c. lfa communication be open. be.tween them Equating the sum of(2) and($) to (1),, 'i •
at their bases, the height x, lo which the water rises, is given by the equcitton
2 h·1··•·(c.;_x')·,. . i ' kgp lex-~~)+ h~ ~ ~k 105(\ . . J O c: :'~; ; ,,,
..
l
C:>: - X + C i og .: ""'.";7-:;:': ,.= Q,
. ·.- '' ··' c·=, r· · (Meerut 199(> •·'
2. 'C-X) = 0
.+ ch log l-
•:,·
l
. . :.
ex - x
C ::

l .. ·~·;~~'.1~~E··· .. ,. •,1•,·1•
1
' ': . : ...... :f,'i,,·ili,!'.'.'.'c:::-})'.i~"i:•~~~~:.,.~r~~:-,-•>:•:,,:,.,-,i-,,,,.,. .,... ,.,r,.~•::·•:;.~:,:~::......... ··~,.... ,,~:,~}!,,i;-l,''"'...;'

~-
·•:,;:<:,:.-.•,. ' :! . <•:-:~: :::~·-·' · , ·.1-:-;?; ;~:r:r::•i~~i~;'.7:t1;.=1>:~
··•: ,.,, ... , ,.. "'•. ·•:~~;;~n:.'. " ,, . ,,, ":·~ •-:·: ··: '. ~:: ;· ::-: ,. . '
• ;.'1Hi\!i{~I ,·1'' ,",~:.•I·" .. : ~t >"' ·, • ,,, "•:!I:~•}~~,:·.\:

-------~-•·>";::""'T"f~.-;.,~o.,;1:..,1.i:'64'~•J.~,i;'t."!.:.11!-!,·, 1~;,!t!(6~,!&~~;s

.-·?::?:~:=;:,·
::,::1,,:

•,.•',','
.,..,,,.,,,,

'•' >•1••••·,, •l•'V~,"\'fl1t1"/~!!".';:;.~!f~~t'l(..,...,..~_,i~~U~1:~~•• .i\


', ', • .·,,··:,.9~!:,.. /, ,.,;, .......... ., ,1,t\1ff:llf,'t.,b•l•l!':1ll\t~1f,~• ·' ,~,• • .,.,
;!l:t!!.!i.!•:,,.::,:,:,;;:1,1::i

!:OVATION OF MOTION , , .'.:..


5"5;, - f't.UIO DYNAMICS
: Applying (l) and
-~
,,
~; Problem lS (o.), Wacer osdll~tes in a ber/e. uniforfn ,tube n vert(c~l plan.e.' Jf O be c! '' : ' . au . ri
:)
!ha lowest point of the tube, !,:B tl).e' equilibrium Zt!!el of water, et, ~ the inc!inntions at = -.e .(h + x ain i;t) - P+
(a + x)
:f of the tube to the horizontal at A,;B and OA .= a., OB·,. b, the period;'of oscillation. is
given by i· · , , i ' au n
(b - x),,. - g-_(h - X sin ~) - -+ r (t)
i~ -
;:
:~
2n
i ·• ·
{a+ b) ) ! ., .
1/Z t
t subtractxm,
"'' ' \ p :

l
g'(siJ1! a+ sin (3) _ i (Kanpur 2001;
, $olutlon; Suppose O ls_ the-lowest point of the tube,A:.B the equilibrium level (a + ·b} t~ = ~ g:t (sin a+ sin 6)
i ofwater, kthe he!glit otA8.abo've 0, Cl, (3 the lnclin,atlons ofthe tube to the horizontal
u =X, U= X
I ·at :A'hn'd B respectlvely and ~. the inclination at 11 distances from; O. Let OA = a,
:, Y 8 1ln 8 p (a+ b) _;:." -ex (slrt a.+
! : ·/IW x=-µ:,: · (l) wl,ere µ ~ I! (sin o. + sin Pl
II •::.~•••••• -r•--7 •••·••~:;:· A

'Q:_· lJuy
. "' ·' _ . (a :t- b)
represents S.H.M. Its time 'period Tis !J'iven by · ·
M ::~E:i:S... b •••••: 1::J.(:· 211 [ · (a+b) ]l/ll
~ ~-··········· 'I'=
Tµ =21t
, g (sin, .
a t sm !3)
gens 0 ::,;o,:.•;..·....::;;..v..ll-L..!.--"
l"lg,Jl.6
"' Problem 14'. Agiven."c;uantity of liqui.d moves, under no forces, in a smooth conical
· tube ,having a small vertical iln.gle, arid :ihe distances of its nearer and farther
OB ,. b, AF" X, Let water in the tube b,e displa_ced at small distance :,; from its . . ,ixtremiiles for the vertex at time tare ran~. r', show that _
Qquilibrlurn position so that AF= x. Mer cllsplacei;nent'
)2 [
,
2
(l)' p A n, y .. h .;. X sin ci,:$"' OP,. (J + X nt P,
1 · ' i •
; ' ; d r ( dr
2rz+ dt r r2 r 3 ] =.O, .
3,..r'--i7'-7'
1 -- _ dt . r' r' ;
(Ii) p" n,y"' h-x sin ~.:s =.,OM "-(b-x) a:M·
_Let II dencto velocity. Equation or continuity Is.
I ~+g_£~~=0.
l t~e:p}~;1ure i:zt the two 'surfaces befog equal, Sho·w als9 that the preceding equation
• resul/sfrom supposing the uis- viva of the mbss of liquid to be constarit; and that the
~flqc(ty .of inner surface is giuen by the iiqu~aon., . ,. , .
, ax a;; i;z '
1
• • · " '•·,. · er --· :- ' · · ·
-_yi.,__,:___. r•3L;r3,.c3,
orJ fu!." 'o ;' 'r3 (r' - r) ' ·: ·
.. as'
'," (l)
' ' , . ' . ) • f •.
0 a~d C ~eing constants-_'. . .: . . . •· - , .-.
.Tb.la=:> u !s indopen4ent ofJ,:
' : S~l!'(tion : Mari:;' t!me I, letp be the' pressure at a distance x from the.vertex
Equation cfmotlon i ls - and uJh~ velocity there, The equaUon ofmot!on la ·
, £.!l+.£.!l.,_9slne-.!£e:
as r ' ,,
~+v.£!:!. .. o~l.~
i!t 6$
3"'o.,
p I at ax p a:c
...
.,,1,1t 'iJs_"' •e.fu!
sm = iJs and the e_quatlon of <:onti;1;1ity is O<._'µ In I:I /B
'l1_ence (x tan q)2 t>,. F (t), '
·-··2 r'
au ., _g i1_ _l. ( e. ) i,~.. • ' X L' T/.{t}

lntegratlng w .r. t. s,
• at! ' . as OS p : ' : . · Fit,
where'f(t) = ~ . Here-h =-=tan a
i i : : ' tan a - _ ;r:, ,
( r· r )

,·Hence
-; a ( -v
,, ii,+ -
~ · a (."-_·
1 2 ) ,. __ ,, )
·i J ~urJ.s,.-gy~E.+f.(t)
r;/ ,' p i!·;'
~1ceraun~_w.r.t:; .:.•
X ox-2 ,cl:i:p
_- _ .' -
~ = - gy - ep
1

1-
. or
'
(/(I) is conslnnt ofint~grntion):
s
' ' (_,: : ! I_, :l _C.(!Llv
,: 2.
2-=-E.·+c
p • ,'" (l'/
;I
o_

\'i!',1lf!f'W'.~;l'(~(l_:m:~mtm~i,w.~:ffi~'lil'!!l,'iffis'll\i!f>.¾iill\::ll.!D:lm1:1~lla'llfflm'!ll'.11'.!'t:\'.!"'r;mi1,,?.ll''.ll'l~l1-lml~W,91!1)~ii1!,l\!\lll;: ,,,,,:-1<1')(:'il' ·r:· •)

. _._,-,~.-T'iir,.,,1o.'-"' _._,.¥•;_,_,.,.· ~•"'>'•·-,,~' .,,. ,, ,.,~,.••·

• ,,.• •.n.-:L'<t"•rn>'·
~~ ..... ,.,... -·· ..
......... , .
•. tt I FLUID DYNAMICS EQUATION OF MO'fN I I . ·'#:If·
Bouodary conditions nre Again, since mass· is. constant and so is volu!Jle ..
fir -Whl!u = r,p.;. p, V ";.·,;, 11, say l' . . .::,,. . ' ·,"
(ii) when .t = r'; v = U = r', p c p . .
(n
This=:, S r' 2 tan o.,r<:,i /J, t~ 2 o.,r) = const:
.. · !Since the pres;ure at the twaeii:ds·is equal). or · r'
3
;_ r = co'?st. "'c 3 ; say. For vollme ai ~ (radlus) 2 h
3
S11bjectfng (1) .to the conditiqns (i) anµ (il),·
. , , I/\ 1 This co.d:l~des the problem. ..
.I - ~ + - u2 = - .!?.p + C
r .2
Probl,em 15. A portion ofhomogeneou.s /l~id iSj confined· beiwe~,twr/ concentric
-l'...ill+i
r' i
ud . . £+c
p
· i spheres of radii A a.rid a, a11d is allracted /owards their centre by a force. vp.rying
inuersely. as. the square _of the distance t~ inn!r sphericfol surface., lo suddenly
Upon subtraction : annihilated, and when the r,adii of the inner 'and 011ter 1u.l"{acu of I.he /1,iJ.id are i- and ;•
i'i the f!uid impinges on a 'solid ball corycef\lrlc 1uith their surfaces; proue that the ·
11)
( -;--;: rm:2<u2-U2)=0.
·1
: ';l I fl,,
impulsive pressu.re at any ppint of lhe ball fo,r di'fferenl ua.tu.u of,J? and r varies as ·
' . l ·1 112 . ' .
But r 2 et = f(t) = r' 2• U . : [('!2-~•-A2+R2J(;-~)] , .
l 1 ) d (r2u.)
( -r' - -r ,--
dt
+-2l u.2 ( (r') •
4
1 - - r 1 ) = 0 v. =;. i !:
, : ; .! S~lu tio~: T~e equation ofconUnuity ls ~2u ".F-Ct) so tha:t'~ = F'
. . ~-
J9 , Equation
(!l.r..)2 r2ci2r 1 l(dr')Z[r'1-r1]··1. 0'.
. \
of motion is ·
or· ( r-r')[z
:rr'. ,, r dt + dt'Z J+ 2•... dt r'4 ' . · · £!!.+au .,;p _l EE..,_ .I:!.~ l ,ee
Dividing by (r - r')/2r', we get at ib: . p ox x2 ' p ax
,& r· 2r (•dr)2 + r2 d2r ]-( !J.r.. )2 .(r + r'J ,,a(r2 + r'2) =, ~:: .
2
[as µ/x is a force towards the centro).
r di at2 dt or ~
"'2 ax ( e.)
+ .Ee ( .! v2 ) "' -
x2 .£..
ax 2·p
.1:!. -
d2 r ( dr')2 [ . r . r3 ? ]
or 2r dt2 + dt 3- ?- r'3 - ,✓2 = 0. t , t'
Inegraingw,r.,.x, 2 H - 'e. + C,
, . - F" (t) +-1 u"' ... ( l)
. , .X 2 X p
This proves the firot recjuirecl result.·
Second Part: Tb.e vis•via = 2K.E. ·tet r and R be internal.'o.nd external radii ae acy time t. Boundary conditions
I are 1 , · ··
r' 1. ·. :• .. ·..
whenx«r,u-=r=f!sayp=O,.
f :,:2 2 (i)
..= Z , · 2 (ru: 2 ,fan 2 O'.) dx,pu~·= np tan 2 a 4f' (Since P,ressure yariishes on ehe 'int?rnal surface).
X
. . 1.. l ' when:t=R,v=R=Usay,p•O .. ,
2 2 (Slnca··pressure vanishes on the surface df a annil)i!ated sphere).
"'. rtp tan o: f (I) (; - ? ) ·
when r = a., Ji =A, the·velocli,., l£zer,o so thatF (I)~ _o,
By the principle or COl'ISP.l"Vatiori of vis-via, Subjecting (1) to the conditiohs (iJ; ;;µ,d (ill,
. ~ P, ~aiiz'a ( 2 (t). 0-~-) = const. = C 1 .;.F'{tl+l~2.,.ll+c . ·
r 2 r
. or ·( 1
-;: -71 )·(·r 2u) 2 ~ •c2 • ~ .:.E..'..ill+luz .l:!.+c
R 2 "'R
or r'rr r) r. 4 ·u~=. Cz
(-=, Upon su~tra'tion , .
1 l1· Ji'' (t)· + -1 (u. 2 .:. vz) "µ
I ( -l - -1 )
or
2
I.I,=---3
r' Cz 1- - -
Rr .2 rR

Repladlng u by V and, 9,2?,K,£1:iyf_iet


. . (r' - r) r or '( 1 . l
; Ji- -r: ; ·, .! ~ ·"' :--:r f =. µ,r,.,,r::R l-'·
)F' ()' t-2
l F2 1,: 1 I J i '. l
: ·. . ·•-.·r . ,R_ . , . .,. .

I .
v2 = Cr' /r 3 (r' - r). [For r 2u. = P' (I)= R2U] · ... ......·
'<! Ml

.,,.t
.'· ,, ~:: ~'.~:~~ . .,·.,·•:,·•

···\·,1~~;;,;d•'. ,,.,. •:•~'.'·~:;:n '\. ! ;: ;,'~ '.·:i '·. '. ': :, \•, :,,:,i '. '•': • '•· ; \~};,I; c.i;.' >" , ':_.;:•::~\·::,: .. ,,, ;•:;;\:; '·.'<•:· .. };, ,: _'.'f,·

---------=. . ~ ~ , 9 ; .......'V'Zr.1"7:i"--:

. '. .. ~ .. ~. ' ,,-...,-,--


..

., ... ,•.'

'.:;
~,·.
'.:!
'i
/,
, ,i~,","!>ll'11slntrrl\T~'ttT-"""'l'l\t'ffiJ.N:"MWJlfrt/,IT<M•-l._•~• ........ •• .... .,-... _ _..~;.,10,_......~6'+Cl'~<:f\tfCltttl'-,Ml/,l").\h,..,l,°ww--,,,, ,..... \, .,,.,-\:.,. ,, .•. ,,._ . .,,,;., ,:..1,;,..~,__,l.•Hf,,.,.v.c-~1<1f.:IJ~~11;:i11h,).a.!NJ.l~),',...'..:•"•i.1::";:11,;:,•,'. ;•
::
,,;,
~
u I . ,. _. FlUID DYNAMICS .
EOUATION:OF MOTION
.,:1
~ ..-~ ~i Multiplying by 2F'dt or equivalently by 2R2dR = 2r2 dr, we obtain, $oubdazy conditions are
),
i ...:} · 2FF' 11•-R- -rl) .de+z:!J
. ,· .fd.r'dR)
- - - . =2µ!rdr-RdR)
,2 Rz •· .
whenx=oo,p::TT,11=0,
;vhGn x =a, L' =..: =·Q,p = 6,, ·... o
i
i l 1 1 ) : -' · iSince the sphere \ifradius:,a.,ls annihilated e.n·d pressure vanishes on the
' or· d [ \ R- r
~ "'2.µ {rdr - RdRl, ] ' annihilated s-phe:ral .. . ., "

· IntegraUng, ( ½-i )F2 (I)=µ ,,:.a_ R


2
) + f 1
j · Immediately afier annihilation., the liquid has no time to move, So we
i '·, ~uppose
':
·: . S:~bJ11:ct1ng thfa to (iii), 0 ~ µ ;(a 2 ~A ) + C1 2
·· ·
When 1 '"·o,x"' r, v "O,p =Po,.Where r > a,

( n1 ·1)
___ Jtl·,. µ '(~2, R2 _ 02 +AZ),
r
'
•:,'. 1'
.
·,
'" (2) Po = n.( 1 - ·.'r.
!_ ) •:

I1·
''The equation of Impulsive action Is
• Q.E..fil
d'iii,= p ,V dx., JI, dx.
and : :
(1) to Ii) and. (iD, o,. - !1
p
+ C·

- F' '_(0) + 0= 0 + C.,


ii!'' :11 1 .1; a .
J-
1

.This=
0
d1ii "'f
'
p (F') ~-
. . . .'
t,}
.
lii .. [ ~-{ p F (t). l Tl~i,s:! ~. · · . .. - F'a (O) = o= 11p • . ... (2)

Putting tho val~e~


'
or F(t) from.(2) ill this equntlon, ~
~II Aw of (iii), (1r~~s ~ Fr' (0) = _Pop ~ C
. " I ,
ra .. ( ..L.l) [µ(r2-R2-a2+A2l 1 ah 1 Po n by(")
.r R P · · ,( l l ) ' or -.-.,--+-
: ',I . 'ii-; j \
P r p p '· ,.

or lii = i:>[ 1i (a2 - r2 -AZ~ R2) U-½) )112 '


or
r. , , .
. Po,. n l - ;
.
l . a) : ,
~ . ' ' .' . . Seco;ndPart: Letwbe the required ia;ipul~ive p_ressure:Tben we have to prove
· or_. •" wv;iouf as[.<~\r2-A 2 +R;{i-f)J1;z! . ; , ' : [ 7 ' 2)1,(i
that, '.iii·"' -Opa
I : ,, I
', I
6
· '·
· .
. ,
. ..
· .. · ·
. Prpb_lei:nA,A sphere ofrad!u~.a i,; surrou11J11~ /;Jy fo{inite liquid of~e!7-sity p, tfie ·: Fu-~ti'we shall de:ermio~ velocity on the inner surface. Let r be the radius of
pre!sUr'e·at infinity being n., Theisphere is tudiJenly flllnihi/ated. Show,that pressure .inr\.et surface. Then , .. .
' I ; ' .

al a di~tance r from the centre_ i~mediate~:, (till~ •ion ( ·i. -· ;- ) . (Kanpur, 1992)
,).' ;vh~n x = r, u = r = i. say,p = 0 when r <: a
1
• iNote the dlfferecce of(li) and (lv).l
Show {urthe'r that if the liqukt is brought to rest by impinging on a concenlfic
sphere of radius:a/2, the impulsive pressure SU$!ained by the surface.'o{thi.s sphere Since pressure vanishes on the Inner surl'ape. In view of the above con(liti0n,
(l)gives.
7" 11'.2 . ·,. ,' . ,.
is [-,:n
6
p a 2 ] /. • . ·
I , ·
. ::.E.:ill.'+r~2 = c. D.
r 2 p
Solutlort; The.equation or'motlon is .r2u = F' (/) so tpat
, . ! , ·
~t'
o/
=F 'J1l , Ei:juation of
X or
-F; 11', ·l f3
-.:......i.:.\ + -2 . -·r1
n
=-
· 1.
as r u ,. F (I)
· ·
motion is
. .. , r. p . 'I'\t
£1!. +u av = :. l £E.
at ax ' p ox
fvi~t/plyi.ng by2 F (t) dt or eqi.JJ·,al~ntly by 2r2 dr, ~e obtain ~~
,' i · -2FJi''dt Ji2dr n ? ~\
or
p,,,, a (1-u·2) ---
.. :..-.l,!4+- : a(")
.. ;
-,;;;:......-= + - 2- = - 2r- dr,
r r ·P · ;~;
-:axp I,
!
··' · ,
x2

Integrntingw,r.t.x,•
i!x2
- F' ',.(tl +-l v2 ,._ E, +C • "' ( l)
or i
I!
·1 ·
" ~ 11 .,.,
d(=--:,)=-2;2·a;,.
•r · p· ,
:,:, 2 p .

1
J 'i

1
:r,•1\1!1(1).',(\!1,SJ{i;:!'.l~sti!::'.&:!:,:~;;;:;:i:,imro:~r,r,;.:;!'.'!\'l'lf!lm!!l!lis~:'.,1 :::'.:':m!i','l;l\\f~l'iN~il';,,~hi1",!'':·i;,:,:;1,::!;:l!i!rlt,;,g,,,M!llR>'l!<lffl\t)''l)\'.>i','.!i;! ,zi;•{i:l'. . ;:

_, .. . ,.,.
_
•.• , .,.~---.~' ...... "' _,_7.,_,"lc,·
t·1
·,1
,' ·"',:·~/, '
. FLUID DYNAMICS EO\lATION OF MOTiON

Int~grating
.:p2 :2n 3.
;,:-:: ~,a'p '. c +C1 (ii) whnnx=r,u =asay,p=po
"l"!:r ·• 2 · 2· n •.- · ' Subjecting (2) to (i), 0 = _ :!:.! + C or , C =I!.
'-r"u . =--.r"+C,
···3 p . 1 ' ' .. p . . p
In view of(ii), thi~ ,. F '{t)
.. :,: ..+_2·u
1 2 t'r/
= p
-e
· -2 n ,,· · .
=> · O,.~-a"+C ; Subjecting this 'to'(fr),
. : 3 ,P . l
. . . . 1 2 n - Pd
- F' (I) . i
~-U
- r3u2 = 3 p • r3 - · ~
p
a3 ·r•".'·P·"'pi
2 n 3 3· n , 2-3 - 1 'l -P'(Q 1 F 2 n:.Pci .•
OJ' r3u2 = 3p
(a .:. r ). or '2 -
U-3p
.e -l r3 1 r
' +-2 ◄"--
r p '
.. ,(3)
2n :1..,;t R be the radius at any time I. Then / ·
- - - (8-1)
- 3 p' . R "b +a cos nt, Als.o let U,. R. We ~ave
'.ap ]l/2
2
or l(u),c 0 ;2l=
. [ 14TT r r=r 2u=F(t)=R 2 R.R~-!!IJ.s!r:,.nt, ,
F (t)" R2 R"(b + a cos ne'.)2' (- na sin nt)
Equation ofimpµlsivc action is d:w = p v d.x. . I .
fi ' tij. · · ',, a/2 F' (I)= 2 (b'+ a cos 111) n 2a 2,sinz 111 - an 2 cos nt (b + a·cos fll) 2,
This~d:w='p·udr.~
· Jo
r dw=p(u),2a/2I '
o
dr PuUing these ln (3), we ge·t ..

n -p Pq = _ lr [2 (b + a ·cos nl) n 2., 2 sio?. 111 - (b + a cos 111) 2 11 2a cos ~I]


·, .
.
=:> W = p [14TI~l/2·a [7
-.-J· . - = - p D a2
]l/2
,
· . · 3p . · ·2 6 . 1 (b + a cos 11t}4 2 2 , z ni
+2 . r◄ - •n a sin
Problem 17."A sphere tilhose radius at time tis b + a cos nt is surr,;,uni/..~ by liquid
extending to· Infinity under no forces. Prove that the pressure at a distance r:from the
cc/tire /1 less· tlir:m th_e pressure-at.cm in/in.lie distance by .' ·
1,·
= (b + !l cos nl) rn,2a [ - 2a si~ 2 nl
, . "•
+ (b +a.cos nt) cos nt

~
.r
(b + a 90s ~t) ( o: ci -3 sJn 2.nt) + b c~s nt + 7 (b +a cos r.~ 3-suj 2 ~t j
. , .. t· , . 2r I 1, 1
+(I,+ a co~ nt)
..
3 •. •a s(nZ nt]
2, 3
Solution: Let lJ ·be the pressw'e at infinity ana"Po at a d!sldnc~ f, ;I'hen, wo ··
= (b + a cos Ill) rna·[·a·(-1' - ,3. sin
2
2
nt) + b co~ nt
have to 'prove that ' . · · . .
li na 2 r ·· ·2 a
P
·
= ~ (b + a cqs nl) 1La (1- 3 sin nt) + b cos nt +.
r
(b +a
. 32r
. 2
sin nt
l
J
1
t -2 • °1r (b ta. ~Os nifsin.2 nt ] ,
... (1) This proves the required result
Equation of continuity is :i: 2u d F (t) so thn~ cvai ;. E.:..fil ProbJem ur.:"A inass of liquid of density p .whose external surfac~ Is a long circular
.0:2 • cylinder of rand/us a, which is subject tq a constant pressu~e n, surrou,ii,{ls a cqa:tlal
Equation of motion !s :l I long circular cylinder of radius b, The Internal 'cylinder Is suddenly d:est~:;ed, Shot;i
~+v au .,_.!,£I? that ifv' is the ueloc/ty at the Internal surface when its radius is r, the1i
ct •ox p.a.: 2 2n (b 2 -r 2)
or F: tQ + le ( l 0 =_le ( E!.) 2) ~ ·= p r log•((, 2 + a2 - b2)lr21
2
(Garltwal!J.000)

Integrating,
;,;2 ox2
- ·F ' 111 . l · 2
~ +- u .= - ,:. + C
oxp
n

Boundlicy conilitiop:J~:::~;tL:·.,<:, p
.
... (2) ~" clu · l
~.,u~"--:
a, a:i:,i• ••-p .a . • and,
. .
*.
Sol1Jlion: Equation of continuity is 'xu,,, F (t) and equation o( m·otion Is
u
'i.

1 I _(i) whon.x=oo,:v=O,p=n 1. i or ~ ~ l.. ( l ·"2 .') ~· _1(


x . ax ,z ax e)
p
I I
1
. ,,.' !:,,.,, "' •:• "''"~i:!r1:·:J;•:;;;:,.,~1 :•::";'~'.1c.'.l;'.':'•::;;m,iu,!l,,;,T,.,~1,1;i<,:~",'.:!'.'~';,f.~~ffl1lm!f:l~!~:m;Jf:t;~:1:~;~r:::t·:;: -~ .1 ·. .': ... •. ' :,-:;,. ·.• -l" •·::"'. ,r,ir,~,~:•1rnrr111J!<".il!?-llfill~~~\lltn>~':""~'i<~"'!11'-"':~•:1,,•~~:::"".""','.~•"~!tlll!(fl1'1Ml!"')l<ll,

:,~·,~•·:~:-:: :,·,. •·:\ '\~'.;·.I~;- ';' ·!,: •. ,~ . ·:::;;,;f;:,1,"t .. :.•r.~-•:::•.-·:•.•. ,.,:~;::::/<· ; ::;;;~:,:::~:•; '··•;·.;:~:~~:~:~· .,,,,-: .~ .,' :--:•·· :··--~~-~~::~~~.-;. '. :"'. ~

...,....,..,..•._"fo;-. • .1.1 .,1 •··•*A+(l1.UVIY-:'J,~ .. dlf?tB 1,,,;,,:,~;rir,S:~

--.·,::;::-~::::.:?t:~; ', '


. ,\, ,•t•~'..'• ..,1~,,.::;,,;,; ..Y,H"
• :;::·::;;•,:,I

•,•(1,t;-:1,:,:,•.::,·
.,,, ,,,\,~..,,,~L•~,i._J,..._V,•,•.· .• ,•

~ ... ..:-:, "!·,>d,~:•.. ,,,.i..·.,1tt.•.•~,tl'.st'i'4,.-f~~·.:-:;;:,!.w~.·.,,:•. •.: ... :::. '. .. · · :'..-", ',/ ;;.'.UWUrt::.:.• .,,,,' '. -~·
,.
, ,_..,...,.._,.._ ... .._,.,.,...I0#;,.,'1,.">1'tlo'fl..,.,""''"-~l..,,Lt,,...,~, ,..;.1-.,01oH,1~~..Willl-r.-.."~·~Ul"l"~•hl,1lttHtt.,._"'-0,,,,~"''••·• ,-,. ,,..,. ,, ,, '

FL'JID'DvNAMICS EQUATION OF MOTION


!51- ' ' • • ' 'I '

·, Alternate method: Equatipn ofcontinuityis..-u =F (t), wher,d:;,. u, LetR am!


Integrating, F' (t) log x + ½,,z., -! -, C r be cxte;nat and'internal,radii, Since total mas.s of the liqull! is constant hence
\· · 1 F2 n ' . : (~ R".h - nr~h) p "'(II a 2h - 11 !i2h) p · ,
... (1)
F' (t) log x + - • - = - "" + C
or 2 x2 l .p ·br · ·
R2- r 2 = 2 - b2 or R 2 = r 2 + a 2 - b2• .. • (l) a
·, R . _.· R .
ux =F ~ ~F :i: dx =F (t) di :u =
Let .R and r be external 11nd lnte~al radii ~t any time t. Since total mass of · KE. of the liquid=½ f (211:X' d.x) p u2 = np f x.d.x. ~
liquig,;is conslant. Hence mass :or the liquid at nnytime t .: roass or-the liquid nt r· r X
time ii= 0.
(n R2h - rift) p = (n a2h' -n b2h) p n
·
= n ll F2 log (
. ' I'
B.)
i.e., · ii .
or
Boundary conditions are
R~:·r2 ,.a 2 -b 2 or· 2 ~r 2 +a 2 -
; o.
... (2) R r Work done by outer pressure=
·
-pdV=.
R

'
f a·
f- a
2~x d:c. n
(I) when:,:;= R.tP,; p =ri. R, = lt n (a 2 - .R 2) = n n (b 2 - r~}, hy ( l}
(For external bound~ryJis subjecled to a,const~nt pressure Worlulona ;i KE,, · . .
when x = r, u "r" ti, p ~I 0. · 2 ·2 · · R · 11 .. '"', R2
'
• (For pressuro vanishes the internal on I; nn(b -r )~1tpfl2 log(-; )=(2)pr?;Jo1{;:r)
. {iii) ·when r = b, ti= b = 0, i.e., F (t) =o;}:, .= O.
Subjecting (1) to (i) and (ii), -·· •. ··2_2fl(b 2 :...r2l. • l .~
2 ·
or.
1,
1·.
. ·•·
u· - . ·P,r
. z .. · Jog 1r+a
,. 2 -Ii 2 )1,--J2
1 F TT
F 1 (t) )og R +-;. , 2 m - -.+ C ~~·
0
. . .. '

"'· R . P • J'i,oo~~m 19, Liquid is contained f.,etween tuio parallel planes; the free surface is a
.
'
F' (t) logr + 2 . /?- = O + C
i F2 ' circular cylind.er of radius a WMH a.ris is perpendicular to.the pl.anes, All the liquid,
•within a concen.tric circularcyl(nderofradius Ii is.sudctenl;t annihilated; Prove that
' i '1 2}1
subtracting, (log R - Jog r) F' + 2 F \ 2 -
11 " --11 , ... (3)
• ifn be.the pressure at the outer surface, the initial.pressure o.t any point ofthe liquid, ·
qistai;t r from the cenre, is. . . . ...
· , . R .•r2 0
P l ,· . . n [ log r - lof! b.] ·;.. .
,Multiplying (3) by 2F dt·~ 2rcdr = 2R dR,'· . • i . log a-' log b .. . (Kanpur 2.000, Garh1val 2004)
2F F' di , (log;R
, ;
1
·•
- log r) t F2 jdR a·l
r
np ·,
-R - ...I. = - - , 2r dr
i [ ! '~olu tion : The eciu!\tion of continuity .is.xv.= F'(t) and .er.i!,le.tion of motion is
I ; . •· . .. • QU. ' i)(f' 1 i)n . . . ... ..
, n. I
. .~. ~.
-+v-=-·-=
pb
d {(log R - log r) F'2l = - l!r P' .dr
or
!, :
:, .-,
:
F ~ (tl + ,g. ( 1 112 ) :' .! ( _
X, OX 2.. · OX . p
,
2.) ..
j -Jogr)F2=-%r2+.C 1 ·
Integrating,
. I ['
(iog~
ir2 ,:a2 .- •.b2,ll2 ] r=--.-+C1
...?. ' ,J n: .
. I '
: : :l?tegrating
. '. . 1 Z {Z,
F'(t)logx'+ 2 11 =- p :C · . ... (l)
· By{2),thls
. i
:=? log
. ;r . .P , ., • ; ·Note that i11i.Ua:lly (i.e., at t =0) the li~uid is at rest.

I rn'~iewof(iii),this-= Qd~b
"
2 D.+c
. p 1
' , Bo1J.Ddar,/conditioris· htli
:·01 whenx=a,u
, ' ' · · ·
0,p=O,l=O.

I. los
(r2 + a2 - 02)112
,. .
2 · 2 . z· l l
F ,. (o - r ) p·
J 11
1
!Since the outer surfacl: is subjected to a constant pressure nJ.

'
[ ((i)whenx=li,11."X=O,p=O,l.;O; .· .
(r2 +0.2-02 )] n {Since press1,1re vanishes on the surface of annihilated .sphqrcl,
or L
r
log ? . . (rtJ) 2 "'2 (b 2 - r 2) p . . Ci/i) whc~ ..- "r, t =.01 p "'Po soy. ·· · :.,;;, · · ,
.
. I
·i zn (b 2 - r 2) .. l . · j '. We have to p!'ov~ t~at Po= n [ log r-~ log 6 ] .
or 11
" -- p r2 ' log [(r2 4 a~ - b2)ir2J ' ! , .. · . . . .log a - log ti ·

... , " 9:

,, ·1\'.'.-<c"'d'><·,

. , ,• ·•"•• ... ......... '


~
, _____ .,-.,._..,~=-...."-"·•.t'~'lk-", ;_,,!..,'e~•~~ I.,..,,,,..'., '•' • '

EQUATION OF MOTION -~i '•'I""-·

·---------··- FLUIOOYNAMICS
Subjecting (l) to-(1) and (IJ):; , · · · ,. 4 3 '. · · ,1 3
P 1':'1 -=P2Y2 ='const., i.e.,
. ..- ., : ·n . 3 11r p 1 =mn, 3 na ·
F' (O} Jog a: ~ P+ C, . a3
.Fi (0)· log b !" 0 -t d. Pi =mn .r3. .., (2)
and
This~
. .
F' (0}. log a i=--+F'
p
n .,
(0) logb (i/l when x =r,_p =p1, u = r., u say, ''
i 'i ' 0/D whenr=a,p..,mn,u•r=O.
or , F ' (0) log ( i~ ~ (-; ). . I I i ... (2) Spbjecting ( llto. (i),
11 ' I
.• /,

! . ti --1£tl:_ ' j '. ' I 0 =--+CI


p .'
Als~, by (2), C' f F, (0) l,og b ~ • p log (alb) : ,: i ... (3)
I
l'{ow (l) becomes r
ln. view of (iii), (1) gives F' (0) log r = ~ /::2. + C - E.'.JD. +1 •Fz } Jl.::.e.
' ' p
, X 2' X4 p
or - I l ~ = ~£2,_.!l~ b 2 2
p log (alb) .p p log (alb) ' y ( )· · ._ Multiplying by 2.J/' dt" z.x dx [as,?;:.= F (I)], WO g;t
= n (lo~ r- lo.g !l} = n ( log r- log b•): 2fi'F • fi'z . ·n - i ' ,
or Po . log (a/by . , log a - log b · .:. - - cit + - dx = !J..:.E. , 2x2 cJx, ... (3)
X :,:2 p
Problem ZO.An in.finite mass of homogeneous incompressLble fluid is i:l.t rest sul)ject Now we can not integnt,e th/a equation w.r.t.·x asp is not constant due to .tho
to a uniform pressure n an'd contains a spherical cavity ofradius a, fil!ed,with a gas fact that ca111ty contains gas at varying pressur~. So we subject this equatio'h·to the
at a pressure mn; prove thqt if the inertia of the gas be neglected, and. ~o)!la's fOW be condition (Ii) and using (ll), '
supposed to hold throughout the ensuring motion, the radius of the sphere will 2FF' . F 2 l( a 3 ).
, oscillate between the ualue a and na, where n' i$ determined by the equalioii ---dt+-dr=- n- mn- 2r 2 dr
r ,:J. p- · . , ra .
l ~- 3m 1og n - n3 "' 0, · · . ; . 1
( F2)
3
- ( r'2 -ima
d ..:-. =2n
(1iff Yr~ q,;eing : - ) dr
' 2 ! ' or
If m be nearly equal to 1'. the time of a'.: oscillation. will be 211: ' r p. : I -r
the density 9/ the fluid. ,.: . . ! ;, l~~egratin!J.,~
, t 't,fp
F 2n ( sr
l 3· 3\ '\ C
1-ma,- ogr:i+ 2
• , . • . . 2· I , , , au .E.'..ill 1
Solution : Equation of con~multy is x u = F (t). s9.. t~at x =!,\,Fi . x 2 , : 2n · 31 · . ·ri ). · .
!'''' ' or r~u 2 a·-;;-( ma !og r-: 3 + C3,
Equation 'or motion is ' 3
2". + v2!! = _.! 22.
at ax · ax
By(iii),lhis . -=> O,;ID[ma 3 1oga-"
3
]+c3•
p , . P • I
F'tt\'' cl· (-l-v 2) =-oxcl ( -"'") Upon subtraction, we get . .
,or :....x.£+-
xZ ox 2 p 1
ii 2 =p
·ru
3
3' ( r ) -(\r--- a
2n ( ma.log;;:
3 3
J] ... (4)
Integrating w~.t. x;
Since radius oscillates between· a· and iia hence we put r = 1111, u = i- = na =0,
{i - L..ill, +.l ui = - E. + C. "' (1) Hence we get , ·
,: 2 p
/ij 0 = -20 !ma
' 3 l
log II + _; (a 3 •• 11 3a 3 )l
·
:1 Boundary conditions arc p ; 3

,.
ui~I~'
;(
(i) when,:= .. , p " n, u = o.

r
Let be ,tbe radius of c~'?If,~:."'f;~
:.t')le t, th 7.~ r_<;,f
.; ,
[Since the in"fi~ite m,oss ,ls at rest subjected to a constant r.r..iss~r~ Ill
P1,~~e pr~s~u\e ~her ,
· I . !
_and
J or
or ,
i
3m log·n
l + 3m lo~"-""·,,:
+ ·l
3 ~ O, · ,
- 11 3

',Jw.. ·,,_,.\ •.. , ·1,


• .

So'.cond pnrt: Whcifm" l (iippro.~1matcly),


=Of
!, ,HoMo the result I.

I
Since th~ ga~ within cayity ob~y.s:Boyl'e's:Jaw · · · ·
Lilrna,.y,y beingsmnll u .,;..,y, ;
'. ' I
•:
'
'' •::·,.,.,,.·:····•-····"•'~'~!:l:!n:~:''i:·. ,.:,'.,::"::··:·... •,;;~'.:·:·•,:.::.;•,rnRm:7,~ITJJ.;'.~1.;-;:;!m!~llf.F.~'.~rn:;w.~!':!r.~!m!llll!'·~-~~:"f4'•~i~,-ll<';,~:,w';,,,,,,~~,,,,.,.1 1
•·"'·
1
,i<:·,:,-;,~f,!::~.t::!:~~•::::,,t1,.,.~1-.trntr.'".'~iw.,i,-,,,..,..• "'"''l:<i-~""~-- 0
-""'""'""''
l 1 ! ; I

ijlfl
ij .. >';:; :. l • :,,. (; ~::,:•; ;', ;: '' •
. ,:,;::::·:;;;:: '..''.,:· .·~-- .. ·..,.;':':':',• . '.. ·.• ~~:!~:!~~:~:· .,....,~.-· •·r·.~·,.'.:!~~~r?
':;~~•;:1; : '' ·i:::,;:1,•· •J;l•;~HV• '·

11
','c"-"'..,-0··-,\,..<:;',71'~V~~
·:;:_:~.:..~~---\'.;~:',,'-!,, ... -.

... 1 ;~. 1;1, ,•-:.:(~ii~~;;.;;,.,,.. ,, :;.,:~•:.:~ . •,:·.. !.,.,, ..~.""·::.:.:~...1..::~ .i,..i.i.1~L'1~~:.~~ •• ~,, .....,.,...........-:.i ...~~·-·:.u~:.: .:.:. . ...1
,;.;,,.:~,-,:,.. ;,;; .. :!,,.:,,
. ,.,1,:-,, •• ,•.•.,•

' '
• .,., •.. • - · -...... ....... ~,. ...- ~ l O n w . ~.... ...:......;.; ... ::..... ..:..:.~.:iU12..1tla1~L!..::-:;, ,'..'Jii!~.~:..;~;J.::il'-::u~:n,:..:. • ........::,,.1 ....,.......... , \.• ,~.... ~";.;;..~,..-"'"""',t.l~~~ft:lr• ............, .:..:..\,::.~~,:....:;..:.. .:..:•. ;.:.;.v;..:,::.:..~~~----·· ... ~·~••H"•·•·;,..,,'\',,.,..,.,r, ...... ,.,r.1,ot1,'l't"! ,,,.,,,.~,, ..,,, ~-~•.•1,.,,

J 1 ! 1
L !
:s! tj FLUID DYNAMICS eov,,:t1bN op MOTION • I

· Now (4) gives • ,, . ( :! ir.R3:. .111:r3) p,. i 1tc3 P


3 3 ,3
(a +yi_;,2.,ll;[ a3'lQS'(a.;x)+½{a3-(a +y)31] or . R3 - r3 "a3· "' (2)

or y2 ,. ;~ [ 3 jog ( ~ + 7; ) + 1 -( 1 +; y]( ! r l+
First we shall prove that
· ;.2 r3 [(r3 + c3)1/3 _ ri;. ( 211 +,g µc3) (aa _ r3) (c3 +r3)l/3 ... (3)
Expnnding upto second degreo. teTll1$, . . , 3p . 9 ·
• 2rt 3 . .. )[ s(Z-_L..)+1-(1+.:i!.l!+...:...
' 3 3 2 . .t; ) ] 2 2 i This equation is obtained by putting. r = x in the give11 result. Boundary .
i=-(1~:1!+,
3p · . , a. a 2a 2 . a. • 2 a2 cqnilitions are
:(il whenx=R,v=R=Usay,p=TT.
.. 2n( 1L.?.l+ .. ,)(-~) .. 2n(· _!Y..:) I:(i\Y' when x =~. _v =.r= u say, p =0. . .
p ' a · .2a 2 211 2
sn k ··
3p
· i ! Since pressure vanls~es on the ~urface inner sphere. of •
,. - pa2 Y, .. ..i, ! i-!iii) when r;, a,y" 0 so th!!t'~ (0 .. 0: .
. i ,. I .. : . : ~ere also, we'h'ave x~v =R 2 U ',. ~u = F (t),
. DJ"".
uoront I at I ng w,r.t. I, Y ~ - -sn2 2yy'
·....
·:zs, · :• / l:$µbject!ng (1) ,tci mand (ill,
, pa J. '
. . ·' 'p••,. l
-~··:::.£:ill·+-u2:-l!R n
3 ..;-+c
.. · .-··, : · ·'·3n . R 2 .3 · p
,or y ,. - µy .. whoreµ~ pa 2 •.
,. , ,~-F'(tl+lu2.,_!!.fa+c
!t Is tho equation for S.H.M, , : : ,,. r 2 · 3
'l{pon:~ubtractioo, .
· ·· 2~
1-.\cnce. time period " r = ~ " 2n
(J!:.£)l/2
sn . , · /-1- 1 1·F , (t)+-·.F
· - 1 2 { 1
I
3) -n
I . . . .. . --- /1 ="'"3..(r3 -R -
r Fl . 2 R4 r4 , p
Problem 21, A solid ~11hcre of radius.a /.s surrounded 1,y a mass cf liquid
~ 4· · · . ' . · I [as r 2u = F (t) = R2UJ
volume is nc 3, ~('d its centre is, atlract,ed /;y a force µ.,: 3• ff the solid spher~ be
3
suddenly annihi(tited, shou; that velocity of inner surface, when ics. radius is x, ·is
··
or:
, :. 1---
r R
111·· 12jl 1) 11 3
F'(t)+-F - - - =-c:.c
. , 2 . R'l r4
n
--.
3 · _P
given by / . , , · . ,
/ . ·•.. 2n 2 c3 . ! : : Multiplying by 2F' dt "2.R 2 dR "2.r- 2 dr,
. . ,x2x2 l(:i:3 + c3)113;. x) " (Sp+ T)
(a3 - x3) (c3 +x3)l/3 .
r,~ :;, ; 1-r1 - -R1 !2F' F'· ·dt·+ r~ {-dRR2 - ·-dr j =.,. ,-,,.a ·2r dr - 2r dr , -n
I ,.2
c::_ ,
3 ·
2 2
. p
wlwe pis U1e densfiy. D (lie exierll(;jl pressure andµ t~te distance. .: . ' ., · ·\
So.lutloil : The force 'F .. ;_ f!X~ as Is
µx2 a force directed towa'rds th'e origi , .i i 1 : ·.. [ (-1· ·1) · ]. ,·(. · ri ). .. 11r3

I.e.; /n the 11egatlve d!rcello~. Eq1.;'at!_on or ~ontipllity is.x2 u F(t) ·so!that. I = · or • d r -R F2 ~- -'s:"' -; 2r.2 dr,
···1·1 3' n 2
. . ' ' a7
ilu a F' 2(I) • . .e.quat

"' I on Of motIon
I
' I~
~ ' l
_l!itegrating, . ( ;-n ).F2 "(· -7.;-; h ,.:i + C2, "' (4)

l ·an
au . au ' · 2 --= · · · · · \, 3 :' · 2n ·
I •
··! -+v-=-µx
/JI , ax · 2 ex
1
ijy (iii), this gives .. · 0= - l:!f. 2a 3 - r-·; a3.~ C2 ... (5)
. , . . . A,
3
F't" . OU · ·2•'- -
!......IU+u-=-µx i:) ( "-
") I·:·:(4)-(5)gives ( _,_.:,.
1 · l )·,(r"u)
• 2 ==. "rI 3 -r.3J..·
2w, ·(a 3 -r3)+:::i.._(a
'/' :x2 QX p, ax . r R . 9 .. 3p ,
· . . . F 'W 1 2 ·i!tl . or· "'(~+
, .. ~u fil1)<a ·-r
2
. !ntegrntingw.r,t . .r, - + -;;.v " - ~·_e+C ... (l) (R.~r) 3 8) R · ·
. ~ • ' . u p · 9 Ip ·
.. . ,., ;j . , .. ,.'
Le.tr nnd R be internal pnd external re.di! rospectivaly at any time t, Since the [(c3 +,:i:_)l/3 _ r) ';.3 ,2 = (~+ID) (a3 _ r3) (c2 +rS)l/3
tot~l mass of the liquid ls constant henco · · I . ·. · · "9 3p .
Replacing r by x, we get the requlr~d r.est:lt to be established. · :.
...... I ! . ...: ·..

I
,, \;;.:~1;.'. '' ' . . , ' ..•,.. ': .;. ' ,, .
~

..,., ......... ~,.·


; ·I,
; ~1:UID DYNAMICS EOUATION,OF MciTION .G.o .t$.~
I :·:.· , I "!, I

Probl\alII 22i A mais of liquid of d~nsliy P, ~nd,·.u~/uriie t l!i'a, is. ~n: ih:e ~orm of a ~lultiply by 2F dt.o~ i!s eqtalent2R;2 dR = 21·2 d,r,
(J_1..) 2F' Ft dt + p2 !9:§..-:- dr l
' .' . ' ' I . '•• . ' I I ·.
Il: 2r2 dr
· splierical shell; a constant pressure n !nxeHed 011, th~ ip:tet,:ial surface bf¢he shell: R
~ R ;~
!" -~
P.
there-is no pressure.on tlie in.tern.cil.sulface an.d n.o'oth'er for~e:s'act on.lt~idiquid;
in.Wally the'liquid fa at re.t and the. in.tern.al hidius of the .,hell is 2p; prbuf that the
uelocity of the bjtern.al surface, w)ien. it. radius 'c, is is
!'
or. d[ f¾_: jj) F 2
] ., - ~ • 2r2 dr.
' (14n' ~)l/2
3p :!12113 _ 1 '
, J,ntegratin~. ( f_.t)p2,i,_2n
r R · 3p r
3·+c l• "' (3)
:In. view If '"l , 2n 3 ·' I'
Solutl9n: Equ<!tions of continuity and motion are o (111 0 =- Sp, Sc ,+ Ci ... (4)
\ . :t?-u '"F(t)
(3)- (4) ,... ( -1 - -1 ) F, 2 (t) = - -2n (r3 - 8c,)

and av+ u av= _l~ r R , . 3p i
· (Sco - r i), . ·(2;
·\
ill · .. <IX p il:,;' . 1 1 ] •2 2 2n
or = -:- usmg
Hence ~+-(-112 ·)· =--i,:,;p
F'11, a· 1
..:,:2. 'cix 2
a (·E- . [ -:- -
r

(c 3 + r3)
l/J (r u)
3P ·.·

Integrating ·-;;·<1l+½v2 ;,,-~+q. ...,1)


Putting , r=c,
, '[ 1: ---m
C
.1 ] c,¼("z)·
2 C
2n (A'
.. ,.,. , =-
3p
oc 3 -
1
1

. Le~ rand R be internal ;ind external radii respectively. Since the total mass of or · '[ 14n zl13
(~),. c"' · -p-. 11-3-
]1/2 Proved,
the liquid is constant therefore ·• ! :• . 2 - l e

4·34,34 3 3 3 3 11! Probl~m 23. A mass ofgrar,,itatlnc fluid is~ I rest under ils own attractioll only, the
3 - -3 n r = -3 1rc
- it R or R - r =.,.c . , ::
(2)
free surface being a sphere pf radius b-and the inner surface a rigid conc~ntric shell
1
Botndaryconditions are · ; of radius a. Shqw lhat i{'lhis shell sudden/} disappears, the initial'preqsure at any
(iJ wh'eo.x"R,u=f:r.,.U,p=n, point of the fluid at distance r frof the centf.e is .
(a+b ) .,
•Since .exteri,.al siirface is subjected to a ·constant pressure n. . ' 31!2' 2
P .(b - r) (r - a) - r - + l. •. ... (l(a.npur 1991)
(i!J .,;,hen·~= r·, v =.r
= u say p = O · ·Solutlon: Let r, be·the radius or inner surface at any time t,The force F of 'i
Since there Is no· pressure on the internal surface, attraction at a distance x•from the contre of the liquid !s ·. ·
=
Hii) when t Q arid r = 2c, u =·o
so that'/il (1) ." o; . i .. ti:i:3 - r3) . [ F F ,:_ y 'l'J m2 ] l
For internal radius of the shell is 2c. " I ,j; 3
rt p
x2 .
•.
.
or
,
- . 2
d
, r
I
We want to prov~ that .. Equations ofcontinu!ty and motion are; ,
. [. 11n z113 ]112 · ilv ilv · . 4 (x 3 ..: r 3 ) l iln
(u),.,3 T p · ~ · .. x 2v "'F (t) and -
[ii ' , I . at • v -ox, ~ - - npy
3 ,:2
- - ""-
p ax
· Subjecting-(1) to the cQn,qi~ions (I) and (ii),· las the force is directed t~wards th~ origlnf .
- F' (t) + f r;2 = _I!+ C

.I
I ' 3
· R 2 p or F
~ • ,p
+ - -v· a(
1 2 ) 1--,ipy
4 (· x--.
. , ) =- 'c- a ( ,, )
x2 ir.c 2 3 : x2 . ,ax p . '
--F'lt~ l ·2 O C
~+2u = + :E.:..ill, ( x2 r 3 )
l ntegrating, . +-ul 2
=--\34 1npy -+- n
-c;,+C· (l)

'
upon subtraction, F' (I) [ ~ -½ \.+ ½(U 2 2
.- u ) = -~ i
i Boundary conditions are
X, 2
i
· 2 X p ' '" i
!
1 1), F2 {1 1!· n I I
i (i); when t =0, ~ = 0, r '= ii, p =Po fare '. .' ': . · . i•
or ( 7- -:Ji F _(I) +°t l 7 - 4 I " - -:- ,· II !For initially lhe radi;s::~ftne-"inn~ihr}ace !s'ii'''ri'h iiso't~ksurface conteins i: :.'¥:;
' , , ·,, }A~.f· :.;-·,:'\•~'.·;:!:\~:~·~-.. ~ :•··
_- (For R 2 U., F·(t) = r2u1::.•"'''' ·•·: ··::::, .
I'. g-ravita~ing mnss and so there wll! be prcssu 1c on ft!, . · !, .........
:l (ii)_ when I=· 0, x = a, v = 0, l'o" 0, , f
' '
I

.·.;mmmr:z:~~;.~;~:~~~~~:;r::;ns:-~~t~?:':!~:~1:.::F~tJ?J:·::~~:.:.t:·
I ,I
·.:·····'.'''''':,•:.,,.,.,,~·,,m~/llH:l>;;.s::;ll!.~m!Wtl'>Wll!l~l"mtll-~••!l•>a.. ,,~~···.... -,.~=·'"·",.,.,,..,,..,.. ..,......,,u/( _ _ _ _ .]i

·I
•. .-i~:\;{:·:;:. : : ' ''),,"· :· ,,;:--:f:''.f."!if:!!:1'.'\'.Z'-;;,~
;j •,,:,•;;,Ii';: ;:;.:::
::-.:~::;•:;;
.,,: ,:,;..
I?\➔ :.••a~1<Jt,,Ha,w,,1Cf_.f~~•P7,

.·. ~~::,: ~·-~:'.~ ·:-'•


"~-'-•·~~--·

,:·:,~::, >·,:.,:,;.:,:,~-~··,,~.'; •. ,.1:;.


.. , ,, .-;t,)!<>}•;.

h
,,
[.~

~ .,,...,.,_,............., ...,..,,,.,,:,,a'.:'--·''··'······..:.:::, ... ,......,,,~:-·;1;'~•:~---"'-~~-....~.,. ,.;.'"·•••·"-·"···-''.. ;'..:. . . :,.: . . . :..:•"·('~"t~~~~--""":"~~,.""'·~-'·"'.'''""·"'"""'"'·•:.~....... ::·············-·-···--..;,~,.~,."·:,·•·r».


~'I I
I FLUID DYNAMIC$
· ·: r;bt.\'~Tio~oF·~onO!'i • · . . . , ·. · ;}~ f.-
, GI
j
•1
..·-~..,
':
• '
I
For pressure vanlsnes on the annihilated surfa~e.
.1'
. ; . ,: 1 ' .• ~

' ' •--:s.olutlon : Let r be the radius


. • .• • • ' •
of i'nner surface
' '
o.t any time ·t. Tha rorce For
. • .. • • • ,... , ••

:, · a~ti-iicelon at a distance .i: frorn the centre oithe liquid is · ·. ·


;' (iiO when t = 0, x = b,Pci= 0, v "0. .. . . .. ; , 4 :,:3 - r3 , . .· .
[ For. F =. !:!!ld2m2 ]
I
!Since there exists no outer pressure). . · -rtpy~.
·3 :t:2 . '
We want to datermine the va?ue of initi~I 11ressure,
EquaUons_af continuity and rnolion•are
, • ~ Ii',' (0) 2 3
. .
Subiectmg (1) to (1)'. . : ~ x · - 3 n PY ( x +,a~ ) - Po +,C.
4
2 p (2) ov ov 4 (x3 - r3) -
=F(t) and -n-=--npy
Subjecting this to (ii) and (iii), ·
at ax
3 . x2 p
3
- F' 10\ 2 3 F' m . a ( -·v
1 2 ) =--npy a (·.!?. )
r ) --
4 · ( x--
~ : - 34 npy
· ( a ·a )
2 +-;;:- +C' ... (3) ~+·-
:,:2 <ix2, 3.:· x2 oxp·
( b~ aa ). F, (t) l i . . 4 ·;.3 ) ,,· (':~2 .
I ·
4 ' . ' ,
.,--npy. -+- +C ·Integrating - +-u =--npy -+- -"' + C • ... (1)
. 3 2 b . ' ,& ; , ; : : · ' · x 2 .3 .2 .x p .
Upon subtraction, ' i · : ; .lJetR be th~ e~ternal ro.dius at any time .t. Since the totnl mass of the.liquid is
I . cons'tant, , ·· . •
1 1 j .
4 · . [ ~ 2 - b2
. •-·-- F'(0)=--11py - + a --~
3 ( 1 . :1 ) ] ' I I , '· .
1b a .3 : 2 a p ' , f ' '. •· ::i itR3 p - i nr3 p 'rrc3 p or R3 - r3 =c~. .... (2)
. . 4 • b 'z '; i: i . 3 ... 3. ·
or, F, (0) = ~ 3 n p ya!> [ a ;
I • •
- T] !
,
; •'Bouodary ·con,d!Uonsa are.
wh~tl:.: ~ji,,,v ;.·,k.;. U say,·p = o:. -.~ :·~.
t' ' ' or F' (0) ,.•:~J. 11. p Y,a. lb (e.i + bl.- Za 2 ) .
. 1.
... (4). T there b°ii{ri'g ~o extf!Tnal pressure .
'" (2)- {3) gives when x =.r, 1.1 = ;.,',,, u say p = 0.
2 2 Subjecting ( l) ·and (I) and (H),
F' (0)· ( -a
1 1 ·1
x
4
3.
( :r I- a
- - " - - n p y - - +as
2
11 · · l f ]
x a
Po
p
·1- -- -- F' 1 4°'(. R2 r 3 ). • •
! 2 . ' I -R+2U2=-3llPY 2+R +C
i or 4
Po=--np 2'y(x•a) [ ll·](.i:-o) p
- - -F'(O)-.-.
·... F' (I) 1 2 4 ( i-2 r 3 )
3.. x xa - +-11. .=--npy -+-- +C
~
1 a.· a2 )·
= - -2 11?'2 y (x - a) [ 2 ::.-1.!::: +~ ·(x k,oL.] '(.2 2r.
3, .
- -
2 x xa_. (2/3) npy. · subtracting, . . .
i '
~ = - _a np 2 y (,: - a) [ !x + a) x - Za :..: :.£.. lb (a+ b) -
' 2
2a 21 ] , by (4). ., !;-R
1 l ] F'(t)+2<•.r-u
l ,,12 2)=-snpy
4· [-R~2
--/2+ r~ ( R-;:
i ·l)] '
l .
3· .
,,._.2itp2.·(:x-a)[x2-b2+·ax-ab.]
,: . . . xa . . _,,;.
! $in~e · ?u = F =-R2U.
.! . 3 T. . . ,: ·;'.:Hence,

·I· ' "' 32 11p2...J (x-


: a)(b -x)· [ l +:-;-
a-t-b]: ·,; ;., {t)
,. !
11- 1.1 +F.2 1. _.11., ~ .1·11,oy[ R2 -r2 +;3 ( .Ll)]
r R f_ 2 R4 r4 3
j' ..
2 R Ir
. Replacing,: by r we g~t the required result . ,;. ~hiltlplying.by 2F d?:::;'.2/2 dr = R 2 di/?, . \ . ·
. . I, . . . . .

Probletn._ 21. Avol~~e t hc3..of grauitati~i 1,·quid, of de,isity p; ls in_it,'all; in the • ' ' .
: ! ~FF'. -;:.-Ji_+
{1 I j · ! RclR2 - cir~ l """"'--3J npy[R 4 dR-r1 dr+r3 2Rd.R-Zr 4 drl
....2
I r- . .
form ofo,'tpherlcal shell ·of infinitely great.'rad1'u<1,'lfthe Uqulcl sh~/1 o:mtracis under
the ln/luenoe of its own attraction, there being no external or intern.al pressure, show
l'hal when the radius of the inner spherical surface ts x, it~ velocity will be giuen by
,°" a[( f-¾ jF2 J~-tipy [(R1 d.R- r 4 dr) +2i-? (R dR - r cir))
2 4npyz · 1
V "' 12z4 + 2z3~ +·~2x2-3zx3 - 3x4) ,, - - npy - r4dr) 1 2 (R 3 •· - :2r'1dr/.
! : . 15x2 .. ·. , · . :. ,3 . '
· J·

l where y is const,ant ofgravitation and z3 =:i:3 +c3,


-~-· .. ...,_ .
,,, I'

. ..:~:~,'i, ~~•,j~~~l.il(';!,~~~i''i'',t,f,•?'. !'. ,,;: '.>'.; ., .'<.•11 .' ,;~·•1~1 it<S~l.,?,i!MY;.'1) -~~,:\.\:~-).,!, :,:.,, ··{ ,'/-<~•"N:,~'. ,:,,f$.,~~t"RMU,>ll1;H'M'l-l:h'~•.:O.(,•.)<!._,J,.!t'QSJnl-M'I
-~! •·.1,~·; .. ~'• ',, ,.

· ···•· ,,--, . . ,, . , , • H • ,-,, ,..,, -~. •4


.,-~',>-'•'•"'·"•'·"•'

>:Ii•.,!"' i, <!•~ •; 1,.\.~-:; i;~r: ,;~,wt:'lt,:fl;•:-Mt.•.rn!•:,~; i,s•·•-' .,,.,,H.•.;d , .• 1,·, -

·-·-· ,....... ~,,,


-,!'-1. . I
..:, ·;~f 1, ii.L/10·0YNAM1cs \~. ,, ' "z..'{fy.:i:J
,...-----------------------....-------+-==---"'="- EQUATION.OF MOTIQ_N·

i • t F2
Integrating, we, get ...
~ere r2u = F(t) = R2U so that U2 =- 1 u2.; F~
5 5
2c~R 2 ]; ; I
(---)Fz
l l . 4 p,,S .:. r - 2r ··•~ ~- r4 . Rt •
r 1?
=· ..· -3 · ,5 . + 5 "'I. 2 :' : 1 1 Subject (l) to (i) and (ii), ·
neglecting constant: of integration. But r2u =F (t).
,;
. .
4 :
u2 =- ...... npy !3 (R 5 - , 5) :.:_ 5c 3R2J.
'' . . R' . ' .
--;-- ; i a
-'F'{d 1F2(1t
R +-2 R4 . =----n:+·c
P:
{
15 . r3(R-t)'•!.
.,.{nAY
16 , ' ,.s
-
li[ 3 (n§ -,R5} + 5R2 (R3 - ,s)iJ i
· fi '- r
• i

'' .
I
~/(2)
lJpon subtraction,
- E.'...ill. + l P2 (ti= - l: + o. ;
r · ll ,◄ P· ··
,•
l;-
... ..i.. npy•B. )2R4 + 2R3,2 +2R2,2, .. ;JRr3 ·_ 3,,:1.
16 ,3 ·· , : ;
I-i - -l),j,· ('1 l)F2 'n, ci3:PI
'I (I)+ - - - - - = - - + - . -
i:
I:
i:
r R . R1 r4, z p ·ra p ·I'
Replacing R by z, r by x and u by V, we get the required result. ' / ~
' ' ' . 1. MuJtiplying by 2F di= 2r2 dr" 2R2 dR, we get .
· Pro bl om 25, A 'mass o{ perfectly incompressible fl,uid of density· p, _is. pounded by 11 ' 3 ' '
concentric surfaces:The outer surface is contained by a'flexible eriuelope which exerts 11
. 2FF' ---;; ! dR- :
1 · l +r ,
2r ·
j -~ ( dr·j =:"1[ - C1 Pi
--11 ] 2r2 dr
contlnu9us/y a uniform pressure n c.nd contrc.cls from.radius R1 t~·ro.ditis R 2. The /;
i
. r " R P r3 • '
hollow is filled wi\h Cl cas obeying Boyle's law, its •radius cohtracts c1:io ,C2 and the
predsure of the gas is· !~itially p 1• Initia;lly th"e whole mass is at i-esi. ;P~oue that, o,r d lJ Ll)R,.' .]=. l!.[
r
F2 ci~Pi -
p .· r,3.
n ]2r2 dr. 'i
negle~JinC the mass of-the gas, the uelocity u dft~e inner sd,ric:ic~ iohr.n the IntegTatlng, I.:-
. configiJration."(R 2, cz) is reached, is giuen. by . ' . : . ; . • I ). ,,
3 ' 3
).n '
I ( ·.. ,' e-i 2
Hci3P1 log r -¥ r3 ]+A
.
I : I '
1 . c1 [· -l ( 1 - -cz --:- - -Pt log-
._c, ]. ·-1 ~--.-
c2 )· !, 1. 1. 1 )F (t)"
- :1 2 = -
!i,, 2 cl · 3 P •P. cz · R2 , cl. rr· view ,01 (iii), this ~- · I, . ·
. S9lution: The equations· of continuity l!nd QJOtion are. ; .- ··o«¾[ cf p logd1 .,-1?-c? ]+A
- t•'
1
:·,,:,:2.u=F{t)·· , av au . l a"'
and -+u-=--=·
a.;. pilx
' ,ot ·t]poo subtraction, · ; :
'':I

'Hence !!:..!!Lf>i. (l u~),,: ~.1. (S. ).' .


u 2 ,.] • · ~ [ - e18 P1
P (R - r) r 3 ·
iog(L ). - .!1 (r 3 - ch]
x2 ax 2 ax ~ .j Ci . .

Integrating, · · For configuration (R 2 , c2~, i.e., when R =.R2, r =·c 2 , the velocity u la ili:,ren lly 1;

\lj ·
_ F' (tl
X
+!
. 2"
~2 ,. _

F2
E. + c
P
1 2· l
2u =z(u
2
)iRi,c:l"'-p, (n
.
1 Ri . [ 3 . ( Cz) TT 3 ,. 3 ']
) 3 C1 'Pi log ·;- -3_(cz .,._ci)
"2 -Cz C2 l .
I!
I
I or ·- F' .. l
(t) +,..,._.,._e.+c- ... (1)
1 2 .•.
3 ;,•.
P1 ·. '('~1 )' .n ( 1--.
. ---log':-,-+-
3
c2 ) ] I (',1---.
. c2 ) 1:f:
'2 xi X.p · or -u ci [
I
Let rand R be internal and ex.ternal radii at any time t. Let P: be the_.pressure r
2_ . c/ p
. .
c2 3p ci3 ... Rz
.
f
'l
at !i distance r as cavity coi,tains' gas. By Boyle's law, ' Probl.e:r:o 26.A 11phere ofradiv.s a is a/one in a1i unbounded liq11id l!Jh/clj iti at rest !r·~
4 . c3l 3 · 4 S
at a great distance from lhe sphere and is subject 1.o no external force. Thd sphere is :,•
311, .P=31t'C1 Pi or P =. 3 •Pl forced'.to uibrate radially keeping ils.sp'herkal shape, lhe radius rat cmy time being
r 1Il
given by r =.a +·b cos nt. Shqw that if,n is tJ;iJ_pre·ssure in lhe liu/d at a great·dislaitce ~
I
Boundary con,<lltions are from (he sphere, the least pre~surii (assumtd positiue c.t the surfacf?, of Ifie sphere
=
(i) ·..,,;hen x R, 1.1 =R= U, sa;\, p;. n.
! \
during the 1notionJ is n - n2 pb (j .+ b) . . : 'i,
:;

.For the outer surface exerts a hniform!Jlressure.n; Solution: Equations.~t;;¢Jl.n!i~~l~,,ilnd tnot~o9,Ar,e;·• .. , .i,
\\;1 <in· whenx =,, "=;: ~~l'.ifi;-i·~''fii;-
Oil) when r = c1, 1.1 = ~ so.~hat F (t) = o.-.
'1)' ' · ' ,..... ·:'"'• . <Ju• i)v l;)
x2' u =F' (I) ;and I -at + u -i>x ,. - -p ££
ax .
i
!:,;
..
.,,

a
ll,, ·.. ~! . . ' ' .
,•,•••,• .... '' •.•• ..::l!!f~j:!:;:'•·,~•'--:••~,•••-.t•-•"•._:u••.,:••••••••·•;JMl!h~: :~-:,•,_.-~~.~~•~:,,~~•~t~';~-.::-:,:
·. . '••'•,<•,\•1,•,\',";.'.~J:>\'1.c\o',~~~~~..,~~~~~~~~...,-,.~_.•-.ll••·•~;i.tM,-...,.,...,.:,,,.o."•--~:~•·!J!"'~~':':~"..,.,'_,••"P..•111"'..-..l'~~Nf'o"-'I-,~•
.~ "'i"T"
!!

:·:::~ ::•x-~:-'-;"::,·-: ,,._,, ·:::!:):':


<·/,'.~~{n;-• .. ·-;·· -~ :, _.
'\\",~{i~;.,lfoi' ,: '1\ ·,·,~• tjl,\'•::~~ .,._,

,,. nmdill.if,h>~,I••. ,,!I,,,•, .~R-;'";-17<'$'::.!


,,·,,:i,;;\•: .. ••:.'•• •'.!i::;. '.,;;:;,i::,';;,•· ,,:;:, ,,,;·::

. n
········--•..-"•-.-··-"-"-~~•• - •••·-•·,._.,_,. ..... ,........ :•;:.rull.!1·,,:,:1!.':•l::•,:, .... , .. ,,::,.,z:,~i,,;::";::1.m,"'·'" ,',: .. : .. ,: ,....:.\ .,,, ..;,,',..,_..,,,,,&1,i,~e~N,k,.i1,.,,,;,;;.i.u.,,;.,;J.1,;,;;.:J:.!ll!.~::;,:•...:;•:.,·:•:·::,:•n!n1:m:i,.:, .:::.·:,:.,:: ......:..,.;,.: •• ,.,,,,;M,1',.••··•,,.•, .. ,....,,.,,,.,

;~ I ...~ t
:FLU!~ OYNAM~
f ,,
~d~AtON OF MOTION . ~~~.ct
·::....
;.--\-:. Henco x2 + l.(l2 vi
. F"Ctl ox ~)-~l.(2.)
- 8x p • . 'I . ' i' :
': 'i'his
·
F; I/\ I a ( l . ) .
~ -\!I.+- -u 2
· :l ax 2
=-=--axa ( nP )
II

x2
<:. •

lntegrating, . 11
F' ! l 2 " . "
integrating,. -~+-v
·x 2
==c.-"'
X P
+ C. ... (1)
- E:...ill + 1 v2 "~ 2. + C ... (l)
2 .Px ' Boundary conditions are
Subjecting it-to the boundap" conditon, (i) when x"' ".", v = o;p·;:.fl.
. . . n Let r be tho radius of cavity at any time t. Then
wh.en x • -. v "0, p"' n, we get O = - p + 0.
,;ii) whenx"r,v=)'=usay,p:0,
- !:.:..ill ~ .! :v2 .;11.:.B. Since pressure 'l'anishes·on-the surface !)f eav.ity,
~ 2 p
(iii) when r = Cl, v '! u = 0 so th<1t F (t) ·= 01,
when~:"' r, let p "p 1• Thon v ~ u., r so that
_ ~.+ l u2 =·n- P1
~ubjec.tin~:·!?Jt.o (i) and (ii), 0 =- ~ + C:
; .. (2)
r 2 p
- E.:fil + .! u2 ,. .!:!: + C
r · 2 • r ·
Given r = ~.. ;t.b cos 111.
f r = tJ. = - bn sin ·nl
Hcnco .:-,-.... •F'(tl 1·F2 '.!:!:: n · .. 2 .. ·,
F' (t)" r 2 u.'=
(a.;+ b cos 111) 2,(- 1;,n si_n nt)
. :hese two equ~tions .,_ .19 .- . ,,: . +J•,7 = r ; p. as ~ u = F.,,(tJ
F' (I),. 2 (a+ b bos nt) (b 2ti~-sfo 2 nt) -bn 2 cos ht (a+ cos nt)2 . :Multiply by .,.. 2F'. dt ':' 2r~ dr,. · .
. .
F' I . . '·;
·or E.:.fil =n.2b l2b sin2 nt - cos nt (a+ b cos nt)).
r . .
, 2FF' r- ( ,i n ). :
----dt+-dr= +- 2r2 dr- .z::..
r . , r2 · r p .
tJF' If\ . '
This~ - ~ + u2 = 2n 2b [- 26 sin 2 nt +(a+ b cos nt) cos nt] + b2n2 sin 2 nt
r , .', . , . . . or
1 a[~~]~~ r dr. +·~ ..2r2 dr
. =n2b [- 3li sil2 Ill~ 2b cos 2 nt + 2a cos nt)
Using this In (2), Integration yields,
F1
.., _ . , µr2 2 ffr,3 + A
+ __ ... (2)
r . •,3 p .
2 (p 1 - nj ;. n 2 pb (3b slo 2 nt ~os 2 nt - / ! ' . ' '
- 2b 2a cos ntJ. .•• (3)
2 3
In order t.hatp 1 is least, we roust have t" o. . ln view of(iii), ,. . +.&Ila
· O="a 3 p +A
",,; 2 (p; -r'I),;. ,/ 2pb '[-
.
2b - 2aJ .
·· ,Upon subtraction,
or , Pi·= n-n 2pb (a+ b),' - -
F1 : µ cr-2 - a2)· + -2n
- I {r3 - 0:3)
. ,. '
r . • .. , 3 p
Prob'fom 27. ;\ centre of forcc:att~acting inuersely, as· the square of the d~tance. is at 2pn (t'13 - r3).
the centre of a ,:Spherical cavity within an l!)/inite melss of incomprelisiblefluid, the :or. r3u2 = ft (a2 - r2) + 3 ... (3}
preuure whic,{1 at an l!)/i.nite ;diatance is n, and. is such that lhe work _done by this I .,
1 It is given that

prfssure on a unit area through, a unit le!'9th fs one half the w¢rk done by .the
attra,ctive force on a unit uoluhie of the fluid. from. fo/lnit:y to the iniiiat bciw1dary of , Work d~qe. b:t 'fi on uqi( area thro4g~ a unit length

·~ (*r 1 ~ ~ r 1:
the cavi't,,: ph;,ue tliai the tirne;offilling up the caifity w(ll be: ' '
2 2 ' I 1
' = ½· wo;k_ done by..;~ o~ a unit Yolume.offluHI from x ="" t~ x =u, ~:
...·.•, 2 ( · . X

• ·1 ra .
, a· bei!J8 the Iii ii/al· radius of the ca uity and ptlie density of the fluid. .Hence n.1.1,,;-J· _£;,:'d.x=!:!Q
.. , .. 2 ~ ;;2 2/:l.
· ' · ~~lutlon : Eq1,1ntlon of ~pntl~uity ls x2u=F (I) and equatio\ of motion is.
.. ·Qu au ", la,,
--+u-,.--'=---= · · :This~ µ':"2a ij•
'.,.i:lt·a~ fl
··\
:,:2{:lcx
I
,•'
.... , ·•

. •,q •• < ., >I~, "•H••~~, '. •


·.t.1·'.•'.);t!'•'.>r>''-",·,V,~1;,•..• ,,,,<· .. ·, -,,·. • •.,, ,,,,.,,,,.:;;._1;::,::,)t~'. "..,. • ~, ; ',·. •,

.,,~··. , .. ,,. .' "


., '·i.'.•" .. ,. ';
',,:,,
65"
I I .i
i' 'FU10:0YNAMICS EQUATION OF MOTION ; ~::··•&:..
E. = _ilr' _,,2 + (a.3 _ 4:,:3) ,.2 _ ,x3 (a3 _ ,6_ ; • ~ , ~! I
I '. Iioundary co11ditio'ns are .
fl 3r4 x 2 • ! iI, ~oi .whenx=R,v;.,.k,,,u,iiay,p=n
. ., . ! .. I (Since lhe outer sur(ace is exposcd°to atrno~pheric pres~ure m,.;-
Problem 29,A .pehre Is at rest in an infinite mas.s of homoglineou~ U~uid of densi!y
p, the pressure at infinity being P, If the radius R of the sphere var/e.\ ~~ such a. way
.. . . ·· . i a3 n . .
(u) whenx" r, u 7r = u say,p "Pi =-3-.
that R = a + b cos nt, wher.e b < a,' show that pressure at lhesurfacr o(tl1e, sphere at . r

any time is P + bn] P(b - 4a cos nt - 5b cos nt). '


We want to determine an equationiofforrn; " - µx. I
Subject.ing (1) to (p and (ii),
Solutton: For the sake of~onvenience',we writeP = n.
Prove as In problem 26 that'· ·
~+lr.J2.,_Il+c .•
R 2 p
2 <Pi - n) = n. 2 pb [3b sirlnt '- '2b c<is 2 nt - 2a eds lit): -F'"'
~ · + -·1.
u 2 =---+C a 3.n
p r3 ·
!This ls the equation (3) of Problem 26]: . r 2
. ' ,31
,. n 2
Po[ %Is (1 - CQS 2nt) .~ 2 (l + CQ~ 2n.1:j:.. Za cos 11/] Upon subtraction, \ ~- #) r _<t)_ +½( U - ,~ 2) =
2
J(;3 -
n2 Ii' •. For sm,~11 'oscillatio~s. rfa•and u2 ar.e sm:al-l quant(ties and heDCe neglected.
"!1.:.,P_
2
lb - 4a. cos nt' - Sb' cos 2 ntl , · · 3 !3 '. . I
n.2', b . ·, ·. : .. F'<tt .,D.(!L::..!_), _IL
or
.•. ,
Pi'." n+ 7 .
lb- 4cz c".~ nt-;: 5b cos 2
.,
nt1,. • i
I I
p (t~ a ,
2 ,. =; p • (t) m
· .• P
Zru'Z + ?u. = r 2 as u 3 is neglected
r 2 '. . ll - r
I
:u
Problem 30, A mas. of uniform liquid is in the forln of" thi,;), .plicr~cal ,shell
r u=r r~pn (.a,;:-r- , R-r'
. 2· 2 .. :
3 3
- r ) R
bou,,ded bj concentric spheres of radii a a,nd b (a< II). The cauity is fi!IJd. ,with gas,
the pressure: of which uaries rv:cordl11g lo Boyle's law, arid is Vnti'lll:i. equal to
atmospheric pressure n and the mass of which may.be neglected. T,fi.e•ot~r S/./,rface Since the displacement is small, leii r,. a + x, R = b + x', Then
of the shell is exposed to almc:spheric pressure,' Proue that if system is ·,~a· 2 " n [d - (a + :i:) 1
3 3
b +x'
symmetrically disturbed, so that particle moues along a line joinir,g it;to the centre, (a + x) x =- 2 •b ,
~~: f P : (a + x) +x - a- x
0

2
the limeofsmall oscillation is 2rt a [ p • where pis tl;e denbit;; of the liquid.
[l - (1 + x/a) 3] (b + x')

,_
er X- . 4
·solution : Equ~lion of continuity Is x2u"= F (I) and equation of motion is , ap ('.i. ! ) (r' - r+ b - a)·
av . . au . 1 £e. I
' Ti + u ,ax = -:- p i!x . n (- 3:,:/a) (b + x') .,.(3)
"cip (l + 4i/a) (:,:' - x + b - a) npp~ox,.
LJ2 )= ~ 1. (
This
.
='> E.:.Jfl
:,:2
-11-(.1
QX 2 ax p (2) = (b +x'f - (a +x) 3
= b~ - a 3

~·¥)
.
.
Integrnting,
- F' It) l 2 . n
+ u ,. - c. + C. ... (1) :::; b 3 3 3
( 1 + ;' )- a ( 1 = bJ - a 3
X 2 p . . • ' ' I '
• . Let_r and R be intema.J and external raqii oft)le.shell at any_tim'e f- Since the
I
, aZx ' I
shell contains g,s hence there will be pre~s_ure· o,n lpe inner s_urface.!Ll11 ~pl when "b2'
=>X :
x = r, Since the tot.al mass ,or the llquid is co_nstaot, · ' i ' · ' · : : Sx . 3~· · ·
Nof(:ll=(,--. )<b+x')=--;-(,b+
3xb ·
)=--, x2 is neglected.
. . . (:i,'ltR3-~:ltr3. ),,.,(.iitb3 _fru:i3',)'p . . · a , Ia a
3 · 3 ')" 3 · ,3
or R3 - r3 ':' ·b3 - a3 ... ... (2) or.
·(·'Sx) .. 3xb
, - -. (b x') = - - . +_
q. . f .. a
.4.3 .A 3n . ,· -,_'• .i . .·< ",• -~ ,','.·'• ·· ........
By Boyle's law,
. . ,
'a'7t r, Pl "'J7t a
·:;~tJf~,ar ::}~ltt:\-.,:~:

. D' of(3j,:J{{i ~ 'Jf/- ~ +
2 • )
~~x+b-a

'
, _ b2 ..---..
j [·; initial pressure orthe gas is equal to atmospheric press\ir~
I
·J.
:~
~ ? .• ,1,'.,'•'.o;!\;iH•,i,1:•: ..~tJT\<li'f>ilmHt~~~~m~~~~~FRMnam,a~•m:-:.l:,•,\•~·••l,'l'l ►!"':*."'''wl',"''"'''••":,::,:::-::\"?~, .. :••· .. •·.., .... ~,,'i,r-...-.1r41.,,t,11tl~YI)\
~ /.',> •• >',,' ,, • • "••'•!·,~::',!!'.!','.':/• • ',H;\u1,\• • ,, •"~••:•,~M•:•:"' --:~nmrnt~~'f,;,l~~t;:~~~~~!.·n~.mn~,~~!!rx~m-:r,,,,o•'.;.~:~~:~!:"".:'.~1~{:Y::"! 'i;'' '
-~
~~
.·,•{:::!;:;-::
.. · •:.;~•:•?:~!,~'. '<. . '••,'i.<·:, : ·,·.:;\'.,:: :-•::},",
, .1.;:\}:\>i':,~,..-•:d.•:--:: '•,•·'., :.,;,_.~··,H,:tii•t 1'•
--.-,.~.~."itl'Jl'n::rn:,,,, .-',',, ,JJ.M1;1L!, • 1 ,\,❖l❖!-:~,:rri;::::
,/i.:.':i, ._;.,;,t;·,,,,:.
:, ·,; .. , ~···
',·',\"'i',!,J,J'•
,,.:,,,,1. '

.
··•--·•·,..,_.... ,., ... ,,~·-•·~•"'•••· ..........!>..·•··••·•~·:~,,;,.,;,:,,, .... ,.;.:;..~..v,,~-..;,utt~~~i~::.:.:~... ~l'.•J.•t:J:.:.:,,,~:.::.•;r..,.-~1;.-... ;......... ...,.,_._,,,,..,..,un\t"r.:','''r,l',1•t•,1•

:~·i1·,
.'fl,:_ FlVIO DYNAMICS i eouArioN OF MOTION !5:t' .!
... ~
· Now (3) bocomes
I
I
'r

l1?legrating,
·
- /i" (I)
+ -2 v "' - + C,
l: 2 .e ... (l)
r3,,2 ,, 2a n {a2 _ ,2) + l .!1 ca3 _ ,3) . . X ·P
p 3 p .. . '.,. Lot r bflhe radius of cavi'ty at any t_lme t. Boundary conditions are
• :,_ ( fil )l/ll[• Sa {a2 -'r2) + (a3 - r3) ]l/2 · (i} whenx .. ..,,v=O,p,.n. .
. 3p ,.3 (ii) when x = ,-; u = u = ~. p .. 0.
{Nagal!va BIIJTI Is tuken bofor,e tl\e rodico) sign bQcause r decreases when 1 . Since pressure vaniehes on th~ surface of cavity,
ioercases), ' ·
· (iii) when r = a, v = u = 0 so that F {t) = 0.
i;et T bo the required time. Then
r .Subjecting (1) to (i) and (i!),
t0 ,a12dr
.:. F, (I) t ! u2 ·= 0 + C.
( dt=-(]P.
),0 ' .J. 2n. {3a (a2 - r2) + (a~ - r3Jll/2·
o.;_.!l+o'
·P ·r 2

pr
2P.. )1/2 Ja ,-312 cir This ~
1J.2- n·
+--::,-p
... (2) !as r'lu = F (t)]
Ta ( _i:m
.r 2 r4
. 0 {(a -r}(2a+iiJl/2.
; by ZF'dt (= Zr 2 dr), we get · r-·
=(§Q:)1/2
2n ·
,~12 d.r I -w'fr,,•, dt·. +F2,4 . r 2 d r = Ilp . 2r2 dr
(at ;>1 12 (2a:+ r)
P·ut.. r" a si~ 2 e·.
. . . /1)2''. :•· . . ' .. ~] 2Il '.
=- .Adr.
7',, ( )l/2 ·I ];aa 12 sin 3 e; 2a sin 0 cos OdO
·•.• · J[;:...:...
I" p . .
o· • a 112.cose.a(2+sin 2 0)' . . -.=-:-r
-'ifl 211 3... +A•.
I.
,.za(:.2Q.)
. _2n .
112
J" 0
12
(~in ~;..2+ 4 :)de
2
· : · 2 + sin 2 e
I Integrating,
i. ,, . .
· ·z~ -,1ew ot'(ili}, this ~"as o·.. f ~ ~
. . .I:'.
3·+A; ·.
8 p. : ..

. ;
•2a(.2Q.)112[!L2.n'+4J
n/2
~c2ad: ]
·
·_p2 '2n
- r =-3 -p (r~'"' ..u3)
lln 4 -2 0. 2 + 3 tan-. 0

3n · f'". -du- ~J,. tan,S-=11


; ('3")1 2[ -.~+4
1 ...0. · 2nr 3 -r)
:3.
.,2a ~ . or i' .· .l'-(!)=--(a
. 3 p ... (3)
. , ..n 4 . 0 2 + 3u 2
· U:sing this in (2}, we get .
.. Za
,... (.2Q.)'l/2[~~
2n
i !l - ~]
4 + 3 . 2 .. \/ 2 F'"'. n 1 F~ n. 'n
_.::.,.-..,.::., =- - - - =- - - - (as - r3)
r I P 2 r 4. P 3pr3 · ·
-~(.fi y✓2[ 2 M('i t2J :ot • ,,•·.3p,-2
F. ' ..(t) = -1L (a3- 4r3):
,
... (4)
, I4 [
For integral P 3 ;
l u '1 · n ( 8
·l .
✓2/'3 :an . ✓(Z/S Jo" 3 · 2 2
r =n(~
I.
· ! .Writlog.(1) with the help of(~) and (4),

, .n 1 !a 3 -'4r3) +-1 1 _nr (. 3. 3) n TI


Problem 28,.A ipherl<;,al hollow ~(r~dius a initially e:dils in an ln.ff.n/1J{luid subject -3p
- . -,2 . ;,: - . -p a. -r =-"-+-
2 • x4 P P
to a constant pressure at infinit.lj; ·show t~at the presijure:at distance r (rbm th2 centre
. '( whe11 ti,~ radius pf the cavity is x; is·'to the pra;sure ·a.t infinity as i,=: l + ( a3 ·~ 4r3 ) _ r (as - r3)
..... . • S.t2 ,1 + (as 4xs)·r3 (a3 _ ~3j·x3:
M M (Gtirhwa./ 2001) IT. , 8r2x 3:,;4
--solution: Equ~tlon of continuity ls .,; 2v .,p (t) and ormbtion is · . . ., 3:,:~ ,.2 +'(aJ,;,. 4r3)xs,_ r.3 (as _,.s)
., .,. ' .. ihl ~u· .. l il '' ·•.. · . ·
-1-1.1-.,--!!E.
at ax p ox · .9 ,·.
·,.> . . ' : ·.. •.·,•.~~,?· . ·'· .
; .Tois gives t)le pressure at a dis~:tfi:e:.f; \\;heil°r ls th~ radius ofclivity. In order
Thls · c(l
~ F' {t) +-.-·u
,..z ux2.
i) =--a ( n ) •
·.c.
c):,p get the press tire at a'·dist!uice r when. the radius of cavity is x,, we replacer .by x :
·anqx!,y•r.'I'hus · ·
,~ ...... ,..... ' ....

, r.. ~V"L"•'•-,.•--,....,.-., ., ···•· _,~-~ • •>. '",. •


.....,..,.. . ·••' .,
····•-~--r-••····
,_,__ .,, ·~-(J,}','0'.,1.1 •• , ·_•,• .-.<•.-.. '~." •·;,;-. <· ~ ·' '. : •.•. • ,"'.,\.: ~. \'i".•,•';,.' '... .,,.

,, II
,,.___,________ ,._,__"'"•-·-·-
', ...,.,- . ·
! :I I i..
iFtJ,J o:YNAMICS EQUATION Oi' MOTION "~ :,.;,\';
a2x . 4:t:b I . f ;
=--·x+b -a+--A~ 1
I

b2 . ? -, i I I we fa.s~rlq.fl.dS, ... (1)

· , =x ( ·1,2.
a2
- - 6+-
a, ·+b-a
4o) · Hence q musl; .ba qonUnuous ·differenUoblo over S. In presc,~t cil.~.e q ls not
contin1Jous· at tha orlgin but origin docs not lie.inside tho rectangle so that Stoke's
'i
theorem is applicable. By. pa.rt (i), cu.t) q,. o. · · ·
Therefore' (-~)(b+:r.')j[(1+-:;)(x-x+b-a) Now (1) gives r" 0 I: Ans •.
1
(b) E1uationofpathcisx2 +y2:i:1.:
=-·sxo -~l" i+(f!:-s+
a b''-a b2
4b
•a -a · ,This circle c contains origin, the point of sing-ularit;pHence Stoke's·tncorem is ·
• 1

not applicable. . . '. ' ·· ·


3.tb
= - a ('b .-a)' using. this in (8)

..
r = jr
d
q · d.r "
f' ·_::_:t_
( a
,c.x-';I.
2 d.x +
...ElL,,
s
x+y;J
'J~
x=-~.lL=-µi:
a (b -:a)• • a p
Tlin.e of small osclllation is = f C ,
(Md.x + Nd.y), sny.
,
, ... (2)
2n _. · [ ~
Tµ - 2na 3bn dM _±::£__'pj!. '
'f'roblcm 30. A ue!ocity /i.eld.,is given by
cy - (:,:2 +:r2,2 - ax
e
. . _'{- I.)!+ jx) ; :. Mdx + Nd,y'is e~act ..
q - 2
:r+y
2'

Determine whether the flow ls irrotational: Calculate the circulaiio>t roµnd. (a),sqv.are
with corners at (l, q), (2, 0), (2,. 7), (l, l); (b) unit circle with centre a/the origin.
'
f (Mdx+Ndy).= f "'z-fy 2 ,<1.x+J od.; -y f
.
S0 lUt!OD : Q = - i:y
n
+jx = U l t VJ
. ' ~-fan•I (; )~ - tan·I (;)
2
(>:~ + y ) ' 'Now (2) becomes

f' ;l =-[ c:r~:)t


(i) To deter~ine the oature of motion·
, I . k 1 1
r= q.dr ..,-[ ~nn· tan~
a § ·a .
Curl Cl=
-v
'a;
.
cJ:t
X
az =.,; [tan· 1 (cot 0)] = - ~ ta.n· 1 i- ~)) l
0
1
tan (
-'--'--' - - 0
x2 +y~ ;z +y2 .
=-[(~-e) = - [ ( ~ - 211 )- ( %· 0
= I (0) +'j (0) + k [ I+f. I~ : l]' f ! /+ y
<>X :,:
'2
,v.,, \ :,: + y, i
r 21t. An!j,
-~·):

,.
=k[~2:>;+ ,~2;;2)~].~o
· 2 2
; .,.
, ;. · 2' : ; , I
1 ''

I
,Jt,
·l
·Motton ls irrotationa!.
(ii) Let r. denote circulation. Then

,r = f q • d.r, -~here c is ~losed path ..


.
1
D (!, I) . 1)
Frobl m 31, Show that
7

(rt+ 'c 2 + c) e Jcdl are con.start ts. ·


2
if$=-½ ~x 2 +
(uhere a, b, c, I, m, n are fun.ct ions of time and a + b + c =
• I . 2
possible with a free'surface of equlpre$sµ.r'e if (I+ a + a) e
2
+·c. 2), V = (lx 2 +mi~ nz 2),
2

8 ' l 2,
,
Applying Stoke'~ theor~m
C, •
0\ A (1,0) . ~ (2,0,
Solution!: $ ~',
2 (':"'
2
ft l>y t cz2)
( j' 'F . dr .'"
'•
fs curl
· Fig'.2.t8
,; (i)' Mottoo is ,rrot~'dari:~l'!f72~~0 :i.~~

, •:'"'···•R1'~ii:~""-::t1:ft'!~!f¥:1~~·~11~~1H'.'1~~\'lfln;•r: :11-< 111-..'. 1• ,, .,......,, 1··•::~:i::~;1'1"'.V••"'•'•••1',,.• ,\,"1n~r:-fflR-fP-'•1••~1;-:


!

',;::;:.::·:!.'.,'
·M:••:•~n~,:::~•:: . , -,:••\-~
\• ,: •:/: '.'~:;::, .. ,- ;,'.::~~w~:.:•
·-------=T?'::""~r,'-'«"~-.·,r~:
____.....,_,.. _____ _ -~---·

w":,,Hn'." • .,.,,, •' • '•"• " ' n->'•


, ·' :: {~: ;:;; -
,><.,< . , .

,,•,•,·•·· .

• p ................... ,_ • ...,_w,.•-11,1.,~~r~.f:.l:ftt':!'Jt(HJ•,h~,,.,,~,,.,. ,,,,,.,.,. .. ,,. ·,. • .. ,.,~ • .,.,,,.,,. ~;,i, ..,;:,,Jo\l,lJr,!,:,,:,;~;1,s;l..:,,,,;;.•~1,o:i.:: ::i,-: :.·. ',.:.:::,u.:u.c..... , •; ••n.' •••• , • ....... ,,J;,\1•,1· :...,v ... ,u ,►1hr~1•,•· 1,

'1 ;

~ _F.i,.\JIO DYNAMICS ~OVATION OF MOTION


1-. ''
-~ ~· · a (£t) : a . · : I.:

-~::.
2
0" V q>,. :& ,,~2 :&ex i),: "-t~ (~_ax)
m ! .: 1
or L:i "'0 I' ;;~y(5),
· ·
J(LlZo. ~+.a.)dttf 2adl=O.
l +,a + 2
a
or a+b+c•O · 1
(il) Bernoulli's pressure eq~atiqn for unstead:,'motion is ..: or
! . log(l+a 2 +a)+2J~di=·logc 1
1 ,
P..+ q 2 J"f;f+ V=F(t)'
a" .. , (2) ' o·r
I
(I +· a2 + a) e2f oilt ·= 0:1
p 2 ' 0
il<1> ·1 .
(1) => ai" ~ t- ax2 ... (3
, Siml!arly (m + 1,2"' b) ezf bdi =c2
2
(n -t- c2 -t- c) e2 Jcdt = c3
q 2 = (ilqi).'(Vq,) = (i7¢,J2" t( ~ )2 = t (ax)2
Prpblem 32, Fluid i$ coming out from a small hole of crosNection _o 1 in a tank, if
Putting the values 'in (2), . \
1 2' 2 ' 1 . 2 l
the minimum cross•section o(the stream coming out of the hole ls o2, then sbow that
' i ' .
2. + - l: a x +: - uix + - l: lx 2 =F (t)
p 2 2 2 ·
~ 2 2
. a ) + l: ax -
p = l: x2.(I+ 2F (t) ... (3)
:: := ½- . . ., ~:-:,::-:-:-:-:-
.for a froe surface of equipressure : . : ',Solution: L~t-PQ be the hole and l?'Q' be its iniage on -§~S~~§;;§§~~~§
. si,f-,,o; 1
tr,_~ ,oP,posit.e wall o_r_the t~nk..Let p 1 b:e the pressur, at PQ · P' .~=:=:=:~:.::::::Le .
t i ,. . ; ,yh!en~e hole ls clqsed, Letp 2 be the pre.ssure and q~ be thE\ , :!:~!-:.:-:-:-
. or, £l?.. £a
ax +v 22. ·gp_
".l' + w az =.0
' _J..,,_._l~;v at the 'minimum. cross-section, The velocltY, of the Q ·:::::::::::-·
a1 "'u i;oming out from· minimum ctoss-sect.ion is at right
a:n,r)es. to the
hole nnd the direction or vdlocity, wUl be
l"lg,2,9
or,. EE.+ ax =P
cit i: u £B. '; '" (1) · ~o~izd11 tal there. · ·
. i : Equation·orm.otio.n is
By(3), ::.1 EE. = i:2' (i + 2aci) + i:°<ix 2,- 2F ' (!)
p a1 •
'
..al (p J '- P2) " 0'2P92
~

P.2. ='t x2 (i'+ 2aci +, a),- 2F' (t)I etz · 2


br ::.1
p ill ·
.. => (p'1 -t P2) =ai pqz '" (1) '
:-
t:
j;
By(s), , ,::_g EE.= ~ 2x (l + a 2) + l: 2cix . .Bernoulli's equation for the steam line c(?llnectlng a. point of P'Q' and a polpt of
.1 : p c>x ' : • n\i.oimum ·cross-section ofthe jet, becomes- · · · l:;~
or '.:.] P£ ='2i: !l +ai +al x P1 P2 ' 1 2 . l · 2 ~:
p, ~.l: ;
,.)
-p =-p + -2 q2 :'} Pi· - Pz = -2.pqz ... (2)
,:\:,,
:1(
-~
u="' ax"~ From (1) and (2), we have
·,,
I\ Oz l ~· !j
·Putting th~se vlill/OS ln (4), , , . ~-
Ih\
ij
!x2 (i+ 2<ili +a);-2F''(t) +l:~a~2 (t.-i-a 2 + a),eO . /· . . a1 "z. Proved.

or . l: .,z ((i + 2aa + +·2a (I+ ~ 2·+:a)l - ~F" (1) = oa) ,


~robiem 33. A horizontal .tra/ght pipe grar:luaUy rer:luces in r:liameter from 24 in. ,·
· to 12 ih. Determine the total longitudinal thrus·t e.rerter:l on the.pipe·iftl,e pressure at
· Jt"is ldontlty. Honoe eacb co~fficlent of.r 2 , y 2,.z 2 vanishes identically, i t4ej(,if;gerend Is 50 lbfltn 2 a_nd tl1e u,elllcity_ .of th~ iu_ater is 8 ft/s~a . .
. (i + 2na ·+ a) +·2~.(I ·+ a 2 +a),; Oetc. . . , .. (5) , : : :splution: LetA 1:nndA2 be the cross:s_i!~tioJ1_onhe larger a.od t)le smaller end.
and· . , ft" {I) ~ 0 ' , .... (6) ,Le~ qi, and% be the velo~ity and p 1 and'p'2 b-e the pressure at the larger and the
'Ip,,
!, · Integrating (6),. we get F (1) a= constan r 1.· . , stn~ll~r eod pf the pipe. Froxti 'lh~ equation of continuity, we have
~..
. ·. l!1 '•\' .... "~
i

. 1

':!f<:\lll~~•~f,!'!l'•:t:,:e;;:1.t:ll?l:I'.~i,~!:!litnITT11W(;R\t-;ir,er,:,~1mtim·;;1~,l!~l!:•t~'~!.li'l~i••W,W.f.il!Rl!-!H'<il-)~l!,Y111'!m\HHi,sli:,,\1,<. ·W!'<l:,,,,:,•,'
'\I,:••,•,·;!::,.'.,•,·\'.•,•,~\•,',,•(.•.;\<'< •••• C•'• I •• , 1.,t;.,::,._~:,· : . ' .

&.

I
:
AMICS EQU~irrON OF MOTION ,. '-:)

· A1Q1 i=A2Q2 I I
$ince the velocity at the .top I.a t.he ;o.ma that at the bottom, Bernoulli'~·
2 ... :· 2, . I equation written between these two levels gives
(12) q·l "·°!I (p). '/2 '"° ,4ql l" qz ... (1)
Ol' 7t

Q1"811:/second = 8 X 1.2 inches/second·


Pr ·
. Pl1 = - 8
·
ofliquid .. t
= 96 :in~hes/second ,i(• below the .atrnosphc_ric piessure, . . .
By Bernoulli's equatl6n, we have Problem S!l. A conical pipe has diameters of JO cm. and JS cm. ~h.e t1.po imds. If at
the velocity ai lhe sma/i,er end is 2m I sec, what i~ the uiloclly at the ()Iner end an.d the
~+.!.,;f=P~+lq:f discharge thraugh'/Jiepipe 7. : '
p 2 P;.!i 2
or
·
_Pl - P2,"'
.l '
2 P (qf- ql} .~ 21 ~ >.< 15 x (96)2. by (1) .. ·r (2)
Solution: Let q 1 and q2 be.the velocjty at the smaller and larg~r end. From
contilluity equif;ion, ws have . . • · 1
• ·

To loogitudinal thrust exerled on the pipe q1A.1 = q:iA2• . f


"'Pii -p2'½ Here Q2 = 2 m/sec, A1,= (~/4)(0•1) 2, :A 2 = (n/4)(0,15}2, l
i I
=·it (12}2 pi - n (6)1, 2 ,•
. A1
= 36rt (4p, -·p,;, .. ,.. (3) Q2 =qi A2

From (2), we hiive 2


= 2 .lQ:.!l.:
P2 "Pl - ,! PX 15 X ' (0,15)2
2 ., 0·80 mls~c. '· • Ans,
Fr~m (3) and (4),.~e have Discharge through the pipe
Total thust = _36rt P1 + ½p x 15 l X 96 x 96) Q = Qi A1

. = 36 ( loO _ ! X 62•4 X 15 X· 96 X 96) · ,.z(~)


" 2 12x 12·x 12
m36 X 26,jQr: • ·Ans. "0·01,'i7 m 2/scc. Ans.

Problem 34, Llqul-d is discharged p.l' lhe rate of 3•86 fl, I sec from a slpho•1 in. ihe 3 Problem 3G, A horizonlal·conical pipe /r.ai diameter /i/5 cm and 40 cm. at th, lwo
.\ reseruoir. 1iha siphon has a diameter of o inIJrin.d the e/euatlon. • ,ind: t.\e fluid end$, (a) Calculata the prenure al the larser end If lhe prauure at the 1mt:1Uer rnd Is
.··pressure at the top of the siphon. ·. . . · · .. ·5 m. of water and ·rate of flow is 0·8 m 2 I sec, (b) Cali;ulat.e the disa/iariJe t/rrough the
Solution :Bernoulli's equotion for the three points on the same strea'rniine Cln pipe If the manometer corinect~d be/ween the tUJo ends reads 10 cm.of mercury.
·be writteo as ·· · . · · Solution:. Let q 1, q2 be ·Uie velocities i'fd p1, p 2 be the pressure at the large,r
I 2· I ·
and snialler ends of the conical pipe, Let Q lie the 'discharge through the pipe, then
· qo Po q1 · P1 qz . Pz ·
-+-+zo=-+-+z1=-+-'- +z2
Zg pg 2g • pg ·2g pg Q=A1q1
Here q0 =O,p 1 =p 2;z=z 0 ..:z2 (let)
ql
·9..
=A =
Q,3.
= 2·38 01/sec. •.. (1)
3-86 l (it/4) (014) 2
q2=n(¼}2· Fr'om,the·continuit)' equation, we have
A1q1 =Az72
•, 3•86 X16 X 7
= . 22 · or Q2
A1
-A· Qi=
JQJ2:. x 2:•~8" 6,l() m/sec. ... (2)
= 19·62 {t.1,sec, Z (0•25) 2 ,

and ql .. 211~ Usinll' Bernoulli's eq~ation, we have


19•62 X 1(),62 Pi qf-:-::·P2,··' 'qt
-=,
.
2
:'\:1:,c,,,:?:1<i'~2'l·
= 6 ft. (opp.)
p + 2g "•p '123: (Hero z i "z2)
--..
, \.,

•!•, •'·" ,,,. · · •• ,.,,·, ,. ·,:,:i: :.:\'::,,.,,.,,..v:,::,,··. -~'.".: ' . •' ::";;;ffi"iS'llliis°'i:::~:'.T,:;Jii:;c'.'!':f';;!.~:rt:J!lll!!!!'J!~''.'lil'i~l':':!',".t}l~E'i\~\'l'il~;)';,,,,:•,Ji•'.W-:?'·'"~'"''·''~-fi<.fi''~'.~;,:i!!j'{S/~~'8Wl~•-~i"'~:-.lm:r-:r;;:;1~:::t'ilW1,:~::::··:·· :· .....~,·-·.~·..,_,...!",~•,~·~•'!o('>"" _ _....,,.,...,ca,.~w,..

.,:,::":;•:::" .. J'••'! ~~!:.-~.;::,,··

. '.~\: :, 1:, :~~: ' :••:•:·


·, ··~~:;:::]!'.;,· ..
•;i:,~;-:~~' ·•::«:{•:<•·
.... .... ,.~ron~~~~:3,jr,!Ip!i:?~'.t:~

-,-.-.. -.-......;"~'t"'-.- .-. ~ - . -


,,

-,: ..l:·:•.•l<• ,' '.;.;.,


.•: ~:.:;:.;H~il~'. ...
!I,j
l ;~

•~.!\.'\.~·.'1':tt~.~~~~...:-,.;._..:_.:....:.,... ~-··· •.:..;.. ' •1''(XC)• '·•11 '\!In' l ! •·' . · ~ ~ ~ l ~ . . , _ , ,...,.....:.. , ,i.;;,;,, .:..;, .:., J••• •.:. ,,;,:~!,\.:~;,;.,;~~~~i!.!l~-::£~~\J.!j'i~:; ,,;1~ i :.:•J..::~~4~; .-..:,,,;,:.:~.. ~~,-......_\tL',Hj,,;.,,;..,.~~•~i'l~U-rv.i'l.lf,,, Jr'I"•; "·"'"•~"H",;:,

,.
r
~ FLUID DYNAMICS
..,,. r !
' ·~ -
.,.
or 5t 2x
J.2·38} 2 l !!.:1. {6-10)
e-s1 ; t x p 2 9,s1
2
. i. :.:·
P.:1.1 : (-'!·38) 2 - (6·10)2
or p • o+ 2 X 9•81 ' !' !
~ ~-4 m;_ 0,34 kg(cm2
i.-

i
2
:Pressure at tho larger end:= 0,34 kg/cm , Ans.
•:Cb) From mnnometer,'we have
Pt P2
O

- - - = 10 {18·6-1)., 126 cm. = 1·26 m.


. '.·:: J
..
p p '
From continuity eguat!on.'we have
A1q1 =A2Q2
A·1 · ..(Q:il.:.,
=> ll2 = -A Q1" Qi= 2,56g1,
' 2 (0,25) 2 ;
:U~!ng Bernoulli's equation, w,e have T ~ . ,l
• Pt l112 · P2 q22
-+--·=-.1.-
p 2g , p . 2g .
40cn!
' j )
~
1
~
' '
, :
O'
A.,

'oi,J8 1
Ie
w
:q ,
,,
,.
•.f'\'

•·
•t'

or ..,..
2 2
iL
I .
( .'?1 _1 )·=. ~
2g qf p lOem
:I

.. ql'
= -2g ((2-56}2 -1) I
I ✓
. q1,"l-2612 x 5-5S
1-26
.
9,81
= 2·11 m/sec.
Flg/2:11

,
'1

I a
Hence discharge thro~gh;the plp,e I~. · · · · ·
. Q"Alql
I•
' ll
Q:;; x (0,4)2 x 2·11 = 2,6.5 m31sec.
'
. An_s .

. Problem 37,/;{plpe o/ 10 cn.i ~!~meter is suddenly enlarged to 20 cm diamet~,.:·;;~d


.the lofB of heat1when 60 litre/ iec of water ls ffowing. 1 • • 'i
·· Solution'·: !,ot q 1 arid ;Q2 ,be' the velocltie'.s at, the smaller.and lllrger end
. pipe, than '· · ·

'(
,,,
or
Q '"Aiqi· '"Aiq2
. f. Q
qi "'·11 1; q2 - A2
Q ~·
'/ i'.:
: 0,05 0·05 ,.
or ql,. (rt/4) (O·l)' q2 ": (0,2) 2 I
ll'
0~ q1 "' ~-36 m/sec., q21 = m/sec. !
.:~,,· Loss of head duo to sudd,n enlargement · . ~
2 !
(q I - <7~) . (6,36' .:. 1•59)2 l !E)
.:1 = ~ = 2X9•81 = . m A~r . . f,
T
!i:l\lf.':~~~~;i:•:'!f~;t1;Wlri:RiliNl!',$1lff~ffll')!m!~.ll'!l'im,,00'.!•:~!'.~1(''1-ll~ffi!llt~l"l''lf'f'im,:::~r~~lffi1',~l$1ffl9.l1>.1'ilf4i}?f.tJmti1 :'l' i'.;';i.i!f.i' ,,: :· :-:;;-:' . ,;: ·..:: ·::: ! ;,:;:;•:;-1,r-·': : '··: ,,·,: : · : ;,,,,:;•~(I}};,;,,::,,;,;:,:,);:<•:::;,:,::,:,·,:;;,·.:·:('-''.!~~~•;m1?);{l,'l:i ,01;;,.<>; ;,m;w;•:•,','.;cllff-Ja:lllm.l)ffllll'l)'.•:•l<J.'/•,'1,c'il
.,.,,·,,.>,..._:~.,,.•I •'.n'>~ .•~••·'

...... ,...
.. '·bf
SOURCES:SiNKS ANO OOUBLETS (MOTION IN TWO DW,ENSIONSJ
,._,

145.
... I
,. ·

I '
Remark. (1) It is clear tho.t .tne exist.en~ t>f'a strea.ni fw:,.etjpi:1 i., n co~ei:1ec ' •

.3
of strea.m )ines and equation of,i::ontinuity for Lncompresaihle O.itid.. (2) Stream·
function exists for all types of two dimensional motlop-rot.atiooal o,r ~ticii:ui.l,
(3) The nece~sary conditions for the ex:ls'tenca of\jl are ·,
(i) tho flow must.be continuous
(ii)'the flow must be !nc.ompressible. 1
3,3, The difference oftho values of \j,I at the-two point!! ropre.Mnts Ole
flux ofa fh-!ld aoro~s any curve Jolnl11g tHe two points. ,(1{11,,,i.pur ROOS)
SOUR~ ES, S't'NKS· & DOUB½ET:S Proof:,Suppose ds Is a lio.e·element at a point
P(x,y) of a curve AB. I.,et the tangent PT me.~e an•
Yl
'Nl'-- t
v-, ,.
+:e ~--~o
(Mo~ion in tv{o Oimensi?*s) angle e with x•tlJtis, Le~ PN be nckmal at P, aod · ~~,u
iu, u) the :voloci'ty co,;nponents or the flu.id iit P.
.
3.1. Motion ln two dimensions:
. · · . · . I, :i;; iI ;I I?irection 9osines of the normal fN llfO
I!' the lines or motion are parallel to a fixed plane (say, xy plane), ·a.nd: the if ; cos {90 + 0), cos-0,,cos O\I,
velocity at corresponding points of all planes has the 11amc i:tjagnitude and 11r~ction,1. le., ·; - sine, cos a, o, . : -:,"-;;,t--i;.;.._ _ _ _ _ _ _ X
then motion is sald to be two dimensional. Evidently, ln this case.'w = 0 ai:id· ' ! For Pf" mak'cs angleiso ·+ e, e, 90 with x, y,;
u = u (x, .)I,~), u = u (x, .Y, t). .. . ·o:k·, a .I
, ', ..
, x, axes respectively · ,T
Inward normal velocity= ~.q, in psual notation• l'Tg, 3,2.
fo the dia8:\"alll. a normal ls drawn through p which '
meets x'y', 'plane In F', The· points p and P', are· YJ
_oorresponding po.int., · ·
I :. au (- sin'B) + t> (c,os 0) (0).0 ~
+
"' - u sfu e + v cos a ·
3.2. Lagrange's stream function : Flux across tho curve A.B from right to lei\
(i.e. current function). , S-l,io. ! ;X = density. normal veli:1', area of the cross section
Suppose t_hc·motion is two-dimensional so that w = 0,
Tho differential equations of streai:o lines ll!'e given by y,____._ _ _ ✓,

Fig. Sil..
= f·
AB
p (tq) ds = J AB
p (- u sin a+ u cos 9) ds
dx Ett. ;L.e,,
-= .
U V
. u dx •• u dy ,;, 0 (:' M dx + Ndy)' r· · :
•• , .
>.• ,( =pJ, [-u$:+v;]ds as\an9=* .
1)
1
ni.,, e,quatioo or continuity for lncor:ipress!ble fluid in two dimensions fo; !
au+ av,. O
a.- i)y,
cp tJ ('~}:iy '+( t;)dx ]7 pf ~ljl "p lw 2 -w1l
• . ~ ,CV (aM aN) whbrew 1 and V;J 2 are the yalu~s· ofw at A and'B respectively.
= ".>'_,:il:i.'
_Tbi:s- =
~~ Un1 Cl) is an exact differential say dlf(l.e.
'"ax . .,,
l:!
FluxacrosaAllisp[\lf2-:1j11l, · ·,,': f :'1.,

,.,·,;,_~~\il;,,f(~I. ' £]!. ~


This. prove~ the required res.ult. .
~d::t:.-ydy=d.V;JE ;ix dx+ ily dy. 3.4. !rrotatlonal motion In two dimensions:·
To show that ln two-ditrienslonol irr()to.tional motion, stream /unct,ior. and
-11-~. velocity potential bath satisfy Laplace's equation., (A.gr<$ 2001)
Proo£ : . Let the .fluid motion be lrrotational so that 3 velocity pot.entW
'--·, !
4> s.t,q = - 'v 4>, this =>
. '" (2)
U"'-~ t i " ' - ~ ... Cl)
~~-~ orcurrenlfunclion; ! . , o;)'_'.:: -.J·: .· .
QX I • '(,,: •
''i:Z'·v•·;.; ~ t . It follow~ that stream function I (!icrc tho component w doos not exist).
,':\~
04-0
~1:;•~•"'";·~:•~•~~·~~~'.t?~Wl~1'.:f'"".;'.l~~~~~•''.~;'·"~·7:."·,::;~nmum:i7,"'~~:C,~ef~~;~,rn~!~!r,'~;t~?r:"J:~:r,--'.":•:::::::;:, ·".;:::·::·:-·::·::···•~~··''':<",">1~~}"'.l7·)f.tl-l~f111W.'ff1',<.i.f\ ..ttffit!FH~h'!)lt.ffelffl~!,l1H.ff~~\ ..~,....w,~~1>~•~t~••~~;•.~.~-:~1-.;;,'.'~~••~¾~~!"'JTi',t,lf,N'flll"'"A 1).',~

·1'.,jHg,~:1,'; . ~;: ;~::;>~ii'. ' ' '. :- ; '. ~ ( ;( ·,: , 1;~. '.' '~ :; ,;-;. ··::,::.:;:'.'.,' ··=:-'.:·::.;;",,' . •:::~:·:~:::::::·'

.....-.--r.:,,-:;v-r::-.,,. 1:1 •• ,,. •m1w..~:.ffi11fo 1!11,',•.1,1. ;{~~•r.:;::;~:-;:


~'.-
·
~ ..

:-r:-:;.•>:-:•:.· _..-· .~:0!·~~~•0'"•:


•.•;."~:.:..i:,·: .. ,··; ...
.. ..
~·:: .. , . :•~_;:.. +·-· ....
.
__ _... ... ~--·-·•

.·.:·:::l!:-:·
•.•,.1,1.,;1.,,,,.,
·.:,:1'.<•)',,:,.
, •.
·.,
'1
11
u
lJ
. 1'!111:C " ~,_,..,.~A'r~.J--(~•1-'.'\:•.l''-"!<h~ .... ~., ...,,• ..._,.:.,,.., •.• ; ...... ,, ,,.' ,.~,'.,.,,.,....,;..:,. _ _ ; , i , . . ~ ~ - ~ ~ .. ...:,.'t.1:~£:::,.'~:•; 'l, ':•, ',:•.•,; ~I. •~••i •. , .,,,,.,,.,,~.,/,.,, .,, .. •• •::~•: ,,d!S:••">rl#!t,(,,, ,,,.

~ FLVlO DYNAMICS :. r: .
·~·
, ,::r
<(,
147
II .. ,,..
, ''ii:tfiia. fnedcm., !.hell .,.
SOURCES.JINKS ANO OOUS'LIITS (I.JQTION IN rwo DIM!sNSIONS)
i
:~:i
....... ,,._h u .. ~ tr & £!,., E!
.. /rt' ox'
I
: . •.. (2) or ,
' . ' . . ";r
-;:-= h
~ =m 1,say,w ere$.,,= ax'"'.r':' a •

m1 is the grl!IC:llent'ortangent to the curve$~ const..


:;;
I

Skip I: From (1) a.nd (2), F!e~e


I • •
-~--P..'1( _£1,.P..'J! . ''. ! 'II.- d ,
. i),: i3/ izy ax·· · : Sl-'9ilarly 'l' = eonst = - - = j; = m2, say
: i I :. 1
. ' . '11.r ., .
. Th.ls ~ ~~ 9~'¥ ::'i:]henrn.
, :· ! m = -- · •h· ( -~-w.. =-=---......
o,. · ljl,. (- ul 11 '
2
,.: ax al
a· ( - a.. ) + -a ( a.. ) =O.i·, ;'il.f.l 1•··2 ( tJ.,.ljlJ
~y•\Vy (:-u)(-u~
or· : ri; 1~ 2J= • l, thi~ prqves the,reqU1red resu,tt.,
--- " - 1
a___,
iJ:r: . cry cl-j
.:::;.;;r.
/x .. :.
.::..:t

'This""' v¼ =.O, Honea th~ result ... · '. Prcibl;;tn ~ ]f $-~A ·(x2' ..;'.),2) represents a possible flow phenomenon, determine the
Step lI.:.Tii;how thnt $ sa.tlsfles Lnplnce's equntlon .. '<Meeru1i19s1; stream fu,rtetiian, ' ·. .. . . . .
. '
Soiutton: We know thnt iSo!\ttlon: Hen 4> •.A (,:2 - y2),
'
' · ·
· •u .. - ai
ax· ti'• - ai
oy' s1J~a: .·. ~- ~ ~ ·=> £::I!. ,. ~
QX :...C)' O)'
By equatlc;n o!'contlnulty, ,.,.~<fo_M.,:j'>t-C, I ."
au
ox + au
c)y = o,
where C Ls ao integrati~n constant, which is the required S'trea.m. funf\.1on. Ans.

·~rob:l~m 3, The uelooi~"potentials ~J = :i: 2 - i and $2·= ✓r cos (8/2) aie solutions of (
·l1e., 1..(·- ox )+ 1.(-£!)=
ax .£.2. 2ty 2ty
() the:Lp#api: equation.. Proue. that the ueloc{ty potential <j, 3 " (x 2 .:. y 2) + .J.r cos 10/2) ..
~
-·-~ ~ !',.' satisti.e1; 9. 2$3 "' O. · · · . ·
OT . rlx 2 + a,)' 2 ':' 0 or v,,.!'." O,
t . . . ! ~d,fotl® :.Here ~l =:i:
-y and h= -/r'.~os.(e /2) . 2 2 ~
Henee~the re!?lt,
Note the-following points:
. ·. ·.. · ·

(l). The stream function 1J! exist~ wliather tbe motion ls !~otati~nal or
1
. '+'~~
~11,.placc's eqµation In cartesla* c~ordlnates and cyUndr!cal polar
coor~9~t7s ls given as ,. .· .
' . i,: i . 2 , c2i 02$ l
. .•
.

.
. .~
(2) The velocity potential $ e:ds~s only when the motion is irrotational.; . , .,;., *1'"~.+-=2-2=0. ~
(3) Whe·n motion Is irrotational, $ exists. i !. I • 2 ox . a; . .
(4) . $ and
1j1 both satisfy Laplaca' eqLation, i.e., 1
. , : : . . ;,2$ 1 a2$ l a<l> 4
. 'v\~o .. , ~ and· , ; 'v 2~,·=--2 ·+-_j_+'-.....l ..
• ar2 ,.2, 002 ; . r er . .
Also •. ~.r • ,,y, ; $y = - 'l'x
or
lj i
·1 · l
v' $~ ... - ~ - cot (812) - ~ co~
2
1
+ i,]n:cos (8/2) .. o
. -~
i.e.·., ~ .. ~1· g! .. - ~
i),: a»'. 0:/'
I"roblom l. To show that the fo.mil:r ~f cu rues ,p (,:, y)·= ~onst. _o,nd
i),:' .

'V (x, y) = co_nsl


,;,,. tbat <!>i and.4>2 so.tisfy L~place's equatloo'. 1
cut orthogon.a(ly at tliair!point.ofintersection, · · Th~s. . '11 2 q, 1 = 0, 'v 2 q,2 "' 0 \.,
I • •• ,,.. • I
. . IQ~. Adding 'v 2 {<l> 1 • <b) = O .
To,Ul,()W t,hat the curues•of constant pot~ntial and con..sta'nt,stream fimctio~s cut B1,1t : $1 + '112 "$J
crt~a.Uj at their po[nl of Intersection: · · . . ,;,2~; := o.
Sohlt:lpn. Curve or const11nt potential is given by
(>.,.con st, this - d~"' O = .f
d
d.:t +:a . dy"' 0
Prpbl~m1-t Sho41 that u "'21:.:ty, u .;..4 (a 2 +.:i: 2 -y 2)'are the uelocltyl:Ompc;n.en.bt
po4si~{e·1foid m'ction, Determine the stre~IJI_ funct/~n. ·
1
i S9.l.1 2 2
tlon: Here u "2A.xy, u •A (a: +x -y ); ·
2
· •..

I j :: .
I : .

•'1> •••··r·.
. ,' :::1::.!;'.~15.'.. ~

14.5' ' I ;i
FLUIO OY:NAM1q\
I' i
i .
AN.o ~oueLli:TiS°<Mon INTWO_~ ,147
li•v b • - - - &uct-. lheo I 'i i ·~
. ~ . bl .· <P.. 'di ', .· ;.ilt .::~
·•.•-:- ~-11 .. ar· ... 12} er .-.;:-= ,.,_ =m1, say,where$x=_,, •9:," :...•
~ = · , · d V
Step I : ~ .(1) ru:id {2), I
! Here m1 is the gradient of tangeot to the cw-ve ♦ = eoa.st.
. . -~=-f;,-~=~. I
.

'Vx
'V " coo.~~ ~- - iv"' d.x =m2, sn, ..
':!1_ . .•. 1~···
' (

m., ==-~>¾t
·ax:z cy2 I ,t • ·I

The!') m1 m2 =
y
.
~\V"')=~... w.....
(-·u).u =-1,

.
.

'
l , ,

a( a.. ) -. a (·££) q,.Y. \Vy u) i- u)


w.Y • +
= ~ - i + iiy ax = -0 •• or 1
m 1m· 2 =.:. l, this proves the'req\lired res \llt.
This=> v2.i, = O>Hence the result,
. I . , •
· Problettf ~ If f (:r:2 - :i) 'represents a possible flow phenon\qnon, determine tJw
Step n: To i.;how t.hat 4> satisfies Laplace's equation. ,. §treom fu,nation. . · :
Sohitioil: We know that Sol\ition: Here q, =A (x2 -y2 ),
u=-21
ox' ,.,.,_£!
.oy Sine~ ,. ~ ~ ~ = 2A:,:
Ey equation or contln\\lty, I· ey a;.,
=;, \jl " 11.Axy + c, . .
.

.
au + au O where C !s an integration constllllt, which is 1the required stream (unction. Ans.
ax cy"'. w C

Problem 3. The velocity potenli~ls ¢1 1 = :t? - y2 EI.Ild </);,. ✓r cos (8/2),C,![O solutlon.s of ·
:.!
ax (-_ax cy (-.21)
.[9.) + 1..
i:e .•
ay. = 0 the Laplc;ice equation. Proue that the velocity pot~ntial h ,. (x2 - } 1)'+ ✓r cos (8/2)
or &2 + &
. 2 = O or .. o,
v~ = satisfies v' 24>3 = 0,
":'.
ax ay Solutio?: Here; $1= :r:2 - y2 and $2= ✓r -cos (8 /2)
Hence the result,
Note tbe following points: · . ·
!
· ' :(
The Laplace's equation in cartesiaq. coordinates and: cylindrical polar
coordinates is given as
I
l,
(1) The stream functlon \jl e_xists whether the motior is irrotational or P1.~.. \
I
(2) Tho velocity potential ~. exists only when the motfon•is h:rotationnt.: . I z
y 4>1
02
4>1 iflf1 = 2 - 2 "'o.
=--2
ax, + -2-
,ey . j
(3) Wheo motion is irrotaHonal, $ exist$: . . 1
(4) .p and \j/ both satisfy La~lace' equation, i:e., ! l a2,,,c__ +.l a,p

I,

v\ =O "'v¼ I: and ?24, · = _..-.


2
+'I ..!...1. ¼
Also
ar 2 ?- r- pr
<Px=\Jly, <l>y=;-\jl~ • !
. !;l "£2. .e:l ' i' o, v 2 1j>z,. - \ 12 cos (~2)'"' _l ~ 12 co~(e't2) + \ 12 cos (612),. O

· ,.e., • = a;.,· ay = - oi · , .· . 4r \ , ·-Ir 2r ,
= that ¢, 1 and cJ, 2 satisfy Lapl11ce's equation,
Problero. 1; ;ro show that the family of curvas q, (x, y) = coMt, and \V (x,y) =: Thu~. 'l 2$ 1 = 0, 'v 2$2 = 0 .
wt ortlwgonally at their point of intersection, . '
Or, Adding V2 ($ 1 + ~2) = 0
But $1+$2=$3
Th~ Lhat the cu.rues of con~lanl poteni'ial and constant .tream functions ·cut
~ a l l y at their point of intersecdon. ' f v24>3 = o.
So.llztion. Curve or constant potential Is give11 by
<- = con.st,' this=> dq, ~~:,+:-®dy = 0 Problem 4. Show lhat u ~i'.,' ~- cc/!,:{~;+ x 2 - r:i:.a:@?fai~r~d,~ty comp,wen.u <( ~
u Q . . possibla fluid motion. Dete_rmine the stream function. · ·· i.:::'•
.Solution: Here u = ~:Y, u =A (a 2 + x2 -i), ·

. '
', :-:;·•':·f''";-;:-,-~:l!~l~"tl.'•'.~~s~~~-~.~;.:~'i+:+.R,\•~i~l~.,~~~~,'jY~1\:,v,;•r;t•..:•..!·.. ~·"'• .."•"'','"' :•,• :··•~·•:;~.?•:'"' ...... ••r\,j~l~~'.i~1'~~·,>N>1,);,:,

';:; ,:);5~ ·.;.


.. .:;.::;y;:;> >'{{ :~•::.. : ' ·,; ,:;,,~ 'i,~\l,";.',:~~~};::~:;~,.: :!i!:;•;;
·•~-:••H•:,'.1. •:;'·'.I,'.';:,'

,,.:::tJ.i,$4\i,,M,,..,__,~~i),;>;lf,},~
----~--··

,:..:,,:,,
--;..1;>.;,,1;,;, • ',.:,;,·•;•,.

'
q ~ ~ e , t ~ 1 \ ~ h . ~ . . , u . , ~ V , , , ~........ . . ! . . A ~ ~ ~ ~ ~ f • " l l i ' t A « > ~ • ) , - . . > 1 , , , . , , ; , ,·
~ • • ·-1 I :. .
,;1~· •··-• ..... •\ ,,,. .:i,..;.:.,....::....;:~'"""'~M,t,i,*~~
.. 1
~~•.'.,::•,, ',,, • .....

. . .. . . .
-~!U!iUl:l.W,.t ,, , .. ,.-, .. w,,·.. ...;, .. ;..,)t.'1,v,..•/l>U'1lN!l,,vr.,,,..,,,,,,,,,\,..MI.<•

.. .•
,
•· Ft.UIO OYNAMICS I I SOUR~E~. SINKS ANb OOUBLl!.1"$ (MOTION IN TWO OIMENSIONS) ' 149

':-.~
L.e., This 'Wl"ill
. l:>e a possiblo fluid motion
. ir I~ satisfies the equation of conti.n1,1ity 1 1· whlb!i represents par. e.lle. ! fl.ow i~ which stream. .y
line~ are piµ-aHel to X•a.xfa. . ·
au riv , ' l' i'I;h~ .. correspondi,ng stream lines and ~•Cons1.
-+-"
QX o;)' 0 ,::(> " "
-V -2.Ay=O
0
: I 1
eq_ujpb_l~Dfial lines are represented as follows (Fig.
3.3); ; .· , . (
wbich is 'trua. Ther~fore, tho given ve!ocit~ components constitute a possibre fluid
m~~ , : : ' j
'l' ~ Const
Pfq~!;op> S, A ,e/,otty /Wd i, gluoc by · 1
· We knowthat • u=-~andu=•f·.
a a· q :!f i- .id t (y + t} j. Fin. d the stream function an~ the . •
sti'e~'il lines (9r this field at t = 2. . · ;0 'X
,~
i
jS<\lµt!on. Here q .....ul. + Dj "'-.i:l + (y +t}J '
S
o .
iJy ,. -
2A.>:
}x "' a +x2_;,2>
y, J M d~iA(~- , :1, : I... (1) !
i !: : ~. u=-.>:,v-=y+t ' .·•· .
·F!g,S,S.

"::·· By lntoirratlng, wo h11vo


.' .. \j/=-'.Ax.Y2+/(x;t);
·1 • '' •
jt~ :mow ·th~ .. . ·
: "' (2) . , - ,i - ~
d)'
= u= - :t and v =·;; + t. ... (1, 2)
Oitfer.entlatlng (2), we have ' l 'I .

·' !By i'ntegrati.ng (1) with. regard toy, we havci


~ --Av2+Ef.·
ax - ., ax· \ .... (3) ·' 'f·=:ey +l(x, t), ",' (3)
From (1)'at1d (3), we have t} is an icl~gration c~riatant..
· -A.,,2 + ,g[ = A (a2 ,;.x2 _ :12) b EL,, A (~2 + ;2) · • or, ~-.;;;+EL -~'..·
... {4) ·
ax ox . ' : ., I c)x . ~,:
By integrating, we havo i From (2) and (4), we have
. ·!)1 ,V
/(x, i) -=A ( a 2x + 3
l x 3 )' 8 (I), + y+d;=y+t=-'Fx=t
Substitut.i.og-the value off {x, t) (2), w~ have in ~ f (x. t) = xt +g '" (5)
.
~ I ( l
w=¼ a2x-xi;+~x3 )+cu), i.
iFrpin (3) and (5), '11'.e have
' i 'V = x;y' +?2 +g (t).
which is the required stream function., : iAt : t = 2, ~ = x (y + 2) + g (2). .
• ' I ,
Probiem 5, Find the stream (u,nclion: \j/ the g:'ven velodit:1,potential ¢,=ex,for I ' '
i'J:ll).c,stream lines are given ·by ljl = consl., therefore
! '
II ..: (.y + 2) " const., . '
c Lt con.stant. AJ,o, draw a lid ofstreamlinee and equipot'eiitial•Jlnes.
2
Lnplac-a equation v ~ = cf. j . · ' · l I J
· .
Solutton 1Tho veloclt,Ypotentlal ~ "'~ rilpresents fluid flow becau\eJt sa~iafies
;.
, j :
w~ifh, represent rcctengulll.f hypcrbolo.B.
·
Ans. '
.f.i
. a"· , F'~dbie~ 7. Prove that for t.~e comp!.;: patent/al the stream lines and tan"'. 1 z
,
•- .
S,.,.C(! -.-~=-c,.,.u·and u=,-~·

i; " ~ 'V
:i,,, . '
· eq~l,pote'ntials are circles. find the velocity at. any •point' and examine
sing.ulartlies at z = ± i. .. . '
.
Therofore c '\''"'1 : f (~). , .. (l) '.S~hi.t!on. T~e_compltlll potential ia !Pyen by ~
Di!J'ercotlaUng witb regard to:;:, we have ·· w = ¢ +i'ljl·= tan-l z, . ... (1) • 1
w"' ,j) - i 1jt:: t11,n·l 'ii!.
~=/.'.(x)
By ~ubl::ractillg (1) aod ,:2), ~e have\!
I . . • . -
... (2) ,
·,. 4
But cl:t!. .. v =- !!.!.
E:!l iJy ==-. il..
ih:- 0, as ~ .. o
iJy
21\jl = .trui-l z - tan- 1 z tan- 1 · z -z~
.
=
· • l +·zz I
.
ta~ 2i 'V = · · •~~ .. ::) x2 +.;l+ l

i
<;::i =O, = f (x) = con st.
(,(x) or: ..,,_t Zy coth 2\jl.
Tho stream function IV la given as 1 + +'y 2 ' '
'l',hnil'1:rA"m lines IV,. constant repr-~se11t the· circles
'JI= const. t cy,
r2- +y2 + 1 = 2y coth 21jf. · · ... (l!) ! .
.,, .., ··+·-·''

;;•.',:,;t:
0

~~<)'f~l~'.:,gf;Hl~~lJi!1'liill'lffiRmr.i·1mttl\lllmlicHt'.·1;11:;J;,•::::;1\l:>\1~m:l!i:-',.!lfi,:1t,f.~l1$\1l',>'ilffrillm!'!tli!MWllll1!llritH!f!:::'.:,ii!ec:-:.c,\'..:·;,• · '
":-·u. .'.'.·'..1~'-·~·1• .. ;-;i.;, ; .. "·' :.:,1:, ·'<--'.-"·,·, •:,:·-:-::

j
· , I i \
~----------...:.-----~---------- FLUID OYNAM,ICS
SOURCE'S,' Sl/'KS ANO o'c>,ueJ.e:-rs (MOTION IN 1WO, OIMENSIONS) 1151 .t
my, by adding (lj arid (2), we have I
2$;,tan~ 1 z+tan" 1 i'=tan~ 1 :z+i.:;=tan:..1 (. · 2X. ·)'
· , 1 - zz , . l _ x2 - /2' lfll!!.
' dz
r i
2 Prooi., w = $ + i-W, w = f(z). I
or. l-x -i=2xcot2$. · ·, .. . '. ... ;(,1) Dilferentiatini w,r.t. ~ •.
The equi•potentials,t,.;, con.st. also represent circle~ which are orthogonaf tb the dW ai ~ . .
stre.imlines 1V = const.• and form a co-axial systei;n with limit points at z =:; £.,'
velocity component (!t, u) is given by . . ; ,
dz ',OX,. ax + I " - U
.: + IU •
'j

dw . . 1 .,
dz=-u +iv=-2- - ; by (1) . . . ! 'c ·~
. +., 5
I. .
I it
· =- LL + I\)
I
11.9 Z ,. X

t
,
• j
t.)li~
!

azJax = 1,
't ,,
·+ ' dw . . . ,· .
z l
the denominator vanishes at z = ± I, lher~fore. it represe~ts the singular:iti~s
points.
.

• · •,: ·
:
at I I= ✓(u + u2) = magnitudJ orveiocity,
Tbis;>= dz, 2
J:
f,

At z =ti, substitute z = i + z1 , where I :l'i I is very small Henoa I': Ircpres.c.ot.a magoitudc!orJelocity: / 1:

, dw l l R ~ ~ Thie ~i.a; wbero vel?city l~ zero, are ~allcd ~/~gnation points, \


- u + w"' = dz 1 = 1 + (- 1 + 2iz 1), c 2fa1.' .....
• .,.os '..._ .... ~_
.""'.' _ - "_
"~,,. • .
pomts. d.,.r;,
dz .. 0:' . F
by integrating, we have
' .1
i'.'
''. ' . . ' [:
~ W t . n cqua!.lons In polar form.
w=--ilot.z1
u,·. I f.;I:'>.,. !' ♦ + '••
' 2 '·
:;:, that the si€gularity at z·= i ·is a vortex of strength k :=-.½ with circWa,tion HC,OQ!
.
• ++ fip rer?'!
;e "' ,.,;a ,
ij.
-rr.h, . ·; Difl"i:::rmtiJd;mj~ w..r.L r ~ 0, ~~.
:,
Similarly, the s,ingularity at z = - i !s a vortex of strength k ·.,,½with cir\)lati~n
, ~+i~,.r<~~ . . ;:
rt.k.
~ ~,•~•-'Dr.(i,,iB).n~ '''
3,5. Complex Potential. ; , l .ae . c)O ,
Suppose $ ;i.nd 1¥ represent' veloelty potential and1s~reatll f'unction or1 'tw,o a Combiniag t;'beiie t.o ~ l:
• dimensional frrofational D;1otion .bf a prefect £luid, Let w =i;· + tw. Then w Is d,efln~d:
as complex pcite'ntial of' t,he fl\tld IXlOt!on. Since ,ti= "Hx;,y.), w.,. ljl (x, y) ~nd soi
w = ¢> + ilV can be ·expressed a:s fuoctio·n of z, Hence w =/(z) = $ + i\1/ ~vl'jere: E '. ~
i
ri[( ~) +i~ }·i•i~ I'.
z = X + iy: !. I l q\lF :. •
p,orta, ~l},\ll~
\:
Also we know that
: . ,.~~~.r.i=~ '. ~
-~= u =-~,-~=v=~ Thl.s~~=!ft,J.~ .. -b',. f !'
a,- r ae· r-00 ar , ~;
.
Le,,:
£1. ~ 21 mil ·
,CJ: = oy I oy o, - OX These two ~ a:re lcuowu a.s polar form of f
which are· C'a.uch;y,Rlemll.nn equations. Thus Ca.uchy,R!emann equations: arc
Cauchy•Rle.tn.ann e ~
Fig.S.4,
/~.
satisfied so that III Is analytiofi.u:1ction oh, ' . :3,7, Two dlmenslonal sources, sinks., ~
(!) Soi::r:r,;.es 1 (Qarh.ulal :zo04; Kcinp1.1r 2002) , 1·)
Conversely, if III la analytic l'unctldn, then ita real and Imaginary, i.e., q, .l\Ild 'V A sourcc(two , ~ n a l si.a:iple source) is a po lot from which liquid \.s emitted
give thil velocity potential and-· stream fun.ction for a possible two dimensional
radial.Jy and symmwically in all directiohs1in .:ry-plane.
,i.
:.
irrotatiooel fluid motion.
· (ii) Sink: A point to wb.ich fluid ls flowing in symmetrically and radially in all
Theorem l. ,To prove that any relation of the form u.i = f (z) where.
w = $ + iljl and z "'X + ii, ·represents. a t\\'.O dil1\ensional irrotatioi:aL !]JOtiqn,i ir '
direct.i?o.s is ca.lled sink. This sJnk i~a 1egat(ve or~~~:,
Di.fl'erene<i Q<Jtw~ri S-Oi!-ri.1!·a~ci.s(nlt,
7~.: , .',
..... ,:,,,;,..,. ·. ,:• ,\·.:
whlch the magnitude of velocity is givel'iiJ?Yif/'.'•'::'.·t:- . · · •- :{11<: .;;,;.,;,'. ·, ·: ·, Sour,ce is a poinfat wbiqh llquid is co'ntinuously oreated nnd sink ls n point at
· which liquid ls continuously a.nnihila~ed, Really speaking, sourco o.nd sink are purely ..
'.) abst.ruc.1. conceptions
I
which do
not o,ccl.ir !n' n!'lture. ' i:
/,

,,
11 ~,r · ·:·. ;;7~~!m::::;;:·~~:t!"i"r.:',~R~fl\~~~rr•~·">'il''!>~•-~~-~IJ,J'"""°~'~··'"'""~'""::T"":'~··.....-.~u~Hil-'HW1l;,"Mt,:•i,'':;
H
\I11
J ,.,;:; · ~; ~· .v:-;;*~::::•', ., :;••.~
·:·.;:!:·, :;,,:.:;, ·•::!:.:!'::·:
\l ,:;r,(;,•,:~1:r; · • : {. 1' ~'. )', ~ i: :. ;~ :, :,:: ~·:: ,: :'
· ·r~•t1:'."•>
, ...,, , , •• i ,,,,, .• j,Jl,,,:iO!.·CIHi','lOiffimn'.l},.fo~?a,.!t,,!U~-~~~~:
·---""~""·=·,-.· ,._ ____ _

.. ,,~ '• •'•' ,,,,,, '·'

Ii
l! ...... ••••-•••n1......,.tw'<'\W'it'nte\111M'C'ff\~n11.o;., _ _ ~ . , ; . ; . . . . : . . ~ ~ ¥ . i l ' l . ) . f l l % J t t " ~ ~ ~ ~.... ,1_.;.,:,t,.~:'..:~,.!•,.·;,1•,',..;.;,~;;.;,!,..;.:,,r,:.,.:~L.l.:.,.,~.¥~~~~.::,:.:;_...l::l:~'.;.::u1l';"',:.~.;~~.. ,;:,~ ... .:.:...'.1.,;-4..,;.,;,1v.,,,....:,,,l;,,'•,1i1•l'\1'U11~J111,;.,,~,.. 1~1i'<'J.rr~
I , , . ' • •
~i
',,.!
1·62
1 '
FI.UID DYNAMICS
l,
SOORC~S, SINKS AND DOUBLe:Tll_{MOTION IN TV-1O DIM!:.NS!CNSJ
,

-~
. .'\~
(UI) Stnm.gt,h: Strength ot' a !!O'\lI"Ce ts defined as total volume offlo\v per unit
time from it. · · · ' ·
. '

= - Z m11 log (z - a.11 ),


Thll8, if2,r,,t i, tho tct.al volume offlow acrpss any small'ciicle sw-rounding the II" l '
e,o~ then m i:s called m-en.gth oft.be source. Sink ls a source.of strangth -m, . I
:Hence. ~=- !. m.,.logr11 , W·=- !. m,..911
3.8. Ccmptn pote(ll:lal duo to a soure'~ : · , :
T-o find the eompl,ex pot.ell tlal for a two dimensional source of strength m p)aced
aL ~e origt.n. , . , . (Garhw<2l 2002, ROO.JJ
whe~e! , z-a_.
n• l
=r,. 8•·· e1
n• l
i
Proof: Co:i}sidor a source of stren!I1,h m at the otjg!n. We are requir~d to 3.~. Ty.,ci. d!menslonal doublet.
detcrmino ;:omplex potential w at a poln~ J: (r, 8) due to this sourc!l, The veloc\ty at
(Garhfr)al ZOOIJ
;Aidoublet ia defined as'.a combination of souice +m and sink - m. a.ta small
·f
P.due r..o Lbe sourc,e Is puroly radl!fl, let .tws velocity be,4,. Fl-:,x llel'OSS n c_ir~lo o_f diSt!J-llP~ f;i; apart s,t., the product m& Is fin!to;':(Slnk,- m means sink of strength f,
~a4.lwi r surrounding the source It O is 2'ti'rq,, By Y,I · i : c · ;q,. [ . -m>~ ! I: l · .. · . ·
1 'Skenith of dc,ublet. If m& ':' µ., finite where m ~ oo, 6s --._ 0, thenµ is cal.loo
dolinitioo of str"ength, • .
strenllt~ _ofjtbe d~bl~t and line !is is called _t)'ia axis of i.he doublet.and its direction
~
2wq~=2nm,henceq,=mtrl. : · ,·. p, u
'
ls taken .from sink to source.
I ' • ll ~
Then II "' qr COS 9 ., !?1 cos 9
r
'/ 3.1 ci. ¢6:rtjplex po'tentlaJ for a doublet : .,
·,,/ t 1
'. ; : i i', · · • (Ga.rhr.oat"2003, Kanpt;,,2000, 2005) 0
u "q, sin O = 7 s!o .El, . /': :Let a doublet.4.B of.strength µ be fon:ned by a sink - m at A (z
"m r.t'.B, (.2:'., a + oo). '.'"•!'' .
=,ar and source
•!~'"
We ~ow that ..-~•,,. 01
• •• ~
u = rn.AB.
· dw .
di"'-u+w Fig, 8.5,'
6a ., AB e1o., .. , (1) fl.
., !?1 r- co's 8 + i sio 0) {For z = re1'1
r as et is:thc locHnirtion of the axis of the doublet x-axis. The complex potential
'-1
(]T' ~ dw = _ !!!, e· 18 = _ .!::._ w due 'to: this doublet at.any point P:z) ls given by
@.
'dz r reie w:=
+ m log (:·-·a)- m log (z- (a+ oa)J
"' ··- -m
. I.nt,e,gratu:ii, w .. - m log z
z
di,
or dw ., - m, -
> z•
1
1
.l. (1)
,
:=-m Iog ( z -·a -oo )
z-a
•.;"' - m log ( 1 -
z-a )

@
• ·(neglecting conattint of•lntegTation)
(l) u tbc Nlqu.l.rod cxpro~sion, • ·· • ~a. ;;;;; m. ( _§g_ ) upto first approximation, Ci
. z-a
1>,,,duotlo_na: (i) µ'the.source + m is at a point z • z1 In place ofz .= 0, then by . . z2 z2

"AL.
shiMag I.he c)l'fgin, / · · · as - log (1 - z) "'z+,.z + 3 + ... aCt••+~a, .·ft!
:·w,. •
WC baTO
f ••
m log (z - z1),
I
Thia b the i::equl:roo (!.X'])ms~lon. for w in this cl\So, ·
(U) T~ fi",uJ 'th••c:omp(a potential U/ at any point z drie
f I


to't~krcea
'

:
or, . W•=
'
' ;

Im.AS e1a
-z-:
-.- , by
.a.,.,
e1a
(l}

.
11

iZ-a
e1:,.
= i=..::--
·1,1
l
.~
m 1,m 2,m3 ,, .. ~ a t a 1,a2,a3, .. :•;·. : · :
of strength ; ·:w :"'-~is the :requ!red_expressioD, -~
Proot: Step 1, Tu detc.nnlno w due to a· source +.mat the point z = 0, (Here D~ducti ODS (i) Ifthe ax.is ofdq_ublet is a.long ~ m(z • n)
prove ns ln § 3.8 I.hilt u, " - m log- z . · .t•ax.is,-then • ' (it
n.
Step 1r a 60'W"CtJ or ~gth· +ml is itt z .. _z i, r.hen ''a= Q, so that w = !: e1,o i:: ....IL.,
~
: o K-
1
w " - m1 log (.t -z 1}, by ~1,ep I. 1 1 :· • z-a ::-:a
n
(19 the ax.isof doubl~t is along and-' ~-ax.is Flg,S.6.
'l'be roquirod complc.:i: J)O'tt:ntlal ls given by
w"' - ml lc,g (.r - a1) - ~ lo,g (z -
,
a~ - • ..

.... ,--. -~..


\
the d_ou,b)et)s·at the orig\D, tl;.en cl.= 0, a ...q so that
W"'
1! 1 .

Ji I
·I
,j I
/Kant/i~,i-1975) "
'~

·.,~.•:<• i.;;:>1!>i!G.'!-!ti5~;~:-;\~~~;.; •'.,' ::•;,>;


•••• _ . . . , ~ . . . . . , . . - <,
·0··.·.-·,,.,..

:··· i154.'
.·~~* ; I
SOURCES, SINKS AN0.00U8Ll?'!'S. (MOTION• 1N_TVvo_ OIM_ENSION!;)_ 155
FL\)10 DYlll~MICS

(iii) rr a. ~st.crn consists of doub)ets of strength µ1, µ2, ... p)aced at


11 sou:z;ce of tbe ss.mo strong-th situated o_n ,. m\
the opposite eide of the line ·ut an equal •••••••••••••••••
.· _l' m- t•··••··••··t••·•
"'' ,. •·r
: "a 1, a,., .... I.hon ui chJe to this system is giveri b,Y.. · ·, . distance. B', M · A' · I
(U)Imageofa.doublet w.r.t, apfone. We
are to fiad the image oftheaoubletM' w.r.t,
UJ=
~ ~ln
:i::~ y•axes;, Treat· the. doubiet AA' o.s a
• n Cl z - an comb!11ation of source + m at A.' and sin~ •-b.: "~p ·x -·c;...--
..,,.here a,. is the inclination oft he axis of tpe doublet of strength µn with
l'·i.
- m itlA with its axis AA' inclined· at 1 an
anglll a with .x•axi.s, The images of- m at
,3..11.lmage. ' I
A and+ mat A' w.r.t.y-axis are respectively
If there exists a curve C ip the .xy•pl:lme in a flu.id s.t. 'there is no flow acrops• it, .•
• m at B and +'m at B's.f
t.hc.n the system of $Ources, sinks and doublets on one ·side of C is said td''be the BL= LA, B'M = MA', Hence the image is a doublet BB' of tho same strength with its
im.ngll-5 of the sources, sinks and douhlet.s on the other side of C. , · .. · ' · axis anti-parallel to AA'. I / ·
1
Slgnlflcarice of Image . 3.13 Image of 11 source.In a circle. (Mee~t 1ss2, 91)
A two dimensional !rrotational motion when confined to rigid boundaries is, W~ are required to find the image o(a source + m at w.r.t the circle whose A
regarded to have p~n caused by th!l preaence of sources and sinks, If we take the centre is O. Let 13 be lhe inverse point o.fA w.r.t lhQ ~ , P
· set of sources a11d sinks (imagining) to be on either side of the rigid boundaries, the circle. Let P ba any current point on tho ctrclc'at ' .
velodty non:nal to these boundaries wlll be zero,.'As sch ,thes.e boundaries- ·~an be . ~b.ich IV is to be detemuned. ,. a . 8; . a1
t.a.ken as stream lines. This is due to tho properly streap:) lines that the v:eloC:!ty of Placeasourco+matBaodsink-mntO,The e A
prependicular t~ strenm lines is iero,. This set of sources aria siryks on eitlier· s(de is value of~r due ~o this system is given by ' • in
· ca.lled the ini.ago, Tulis the motion is no longer constrained by boundaricslso\hat it
t· is possible to predict the.nature of the velocity and pressure at each pohitofthe fluid. IV" - ma 1 - me 2 + mo I
s I
3,12, To fi.nd tM image ofa simple source w.r,t a pl/J.ne (straight line) pnr,f. fh:ow
' : '• I;, •
, or ljle:-·111 (8 1 +:s 2 -e). F'ig;U. ... (1)
~ ·that the image of a doublet w.r.t.' a plane i$ an equal doublet symmeriico_lly ,p,focad. Since B ls the inverse point of A,

~ Proof:(i)Tofl,ndtheimageofasource
w, r. t a straight line (plane), We are to
· .
;,y\P
. (K,mp\'r 2002, 2003, 2004; MeJru1t
j ·. f \
: ·. ,
2op~)
~
~
or
Oll . OA = (radiud)2 = op2
OB OP.
- = - alsoLBOP=LPOA.
OJl 0A
'
.
• determine the imag~ of a source + m at
A (a, O) .w.r.t the straight line OY. Place a .I . .
. :
'. :·
·
1 .ll I f I"
t slource + m at B (-a, 0), the complex
potential at P due to this system is given. £82 ·1 X
i) by w ., - m log (z - a)..; m log (z + a) B 0
·(- •• 0) · (o, O)!
= - m log (z - a~ (z + a)

·. l
F!g,3.7,
~ = - m log (rie10 1 • r 2ei 8•)
it = - m log lri _r 2 11 ce, •.• 0:1) [ where PA"' r 1 , PB r2 )
'd I). 6
' .
I II A
r,_.,. S.10, . (
or '<I>.+ l'lf = - m (log (r 1 r2) + i (€l 1 + 02)l ·
ill Hen«: 60P.fJ and ti.OP.A are similar, T\ierefore

I:.
This =\II= - m (01 + 82), ... (1)
L.·OP.8=.,C,0/\P,Lt,, Eli-; 8 =:r:·-01 or.8 2 +01 -8=n.
!!
I UP lies on ~-a.xis; then PA "'PB so ihat LPAE =. LPBA, ..
··! Now (l) b«ome,i v ,. :- m-n: ot w= cbost, ··
i.e., it - e1 = e2, it= e1 + a2.... (2) ·
'i' I By (l} a..nd (2), \ ; .;
! ::1
i. !
Tb.is dectare6 t.h.a.t cirde is a stream tine so that there ex.!sts no flux a.cross the
boun~.rtrn.eansth.nt:>, _ ,. . , r: .•.· •. ..,:J:::.:.:,,.':.J" _ · .!
. ·l
! ,' ' - . ' • .,_-.,.,.,,•-_c· i' 'i lmc.ca of &()u.ree + m af A;..; a source+ mat 13/in.e'·in.vers~ point of A and sink "' ~- :
it ~.
§
I. ., IV ;:, -, m:1t er \\I ;, · : .:
It i:neans thaty-axis is' stream llne._Herice thedmage of a sourc~·i+;m' al -m at:tMcr . !
l ~-
i
~
'j
Ii A (ci, 0) is,a-sour-~e +mat$_('." a, o,. '!'hat is to say, ima,ie o( i;d1our~e w.r.:t.1 l~~ is
!
I
~ '• ,. ,. . ,; ..,,,.,,,,,11;in!i:'"~:t;,.;;_,1"(·~•~1:"·:• •-'.···:;11m1im:r;;ri·"~·r""''!\"''f:Wi!®::1trr.~1:l;l:"fl''°""'TH-:-· ··,~. r:. ·_
1 '. :· ""!~-' •. ·~..
'••'' , ,. "'''#' ;;'f'W,,•,.,,;,i'"'fttrm·tlj,?u:JUttff1tf=w~~~~~
: •I i i , : • " ' I ••• '
..:·..11,i•tt"\"1l4)}:.0"""'''
'' ' ' ,
.i,........;,.• .. ,i,,' '...' '"'• ..- ... ,,.,,,
'
~-J~f"¾l"fl-QH(tltH.f'\\,1/\
' I ' •

ti I 1

:1

:j · ', '•'"···•,•::•:·::}}'. ·. ,-.,.s:~·:·:i:r-:w::~~f~'.:!'.':Y· ,.;,-·:--·-u;


\\HSH\: i! 1\~'.;{>1 ;,., ) , ,, ',· ,,:.;~~: i,,:;:_; '. :; :, '. '' ,: !:-:: f~i~:)'' ' · ;.;~\•::;(~l:~:, ·,:,. '?ii.•'•'.:' •,•,•. '•• •.' ·.,: . :.,:y,.;-:,., .'.'/:' :, ,·•i'.:~t:·[~:'.·;•:; .

- - ~ - ~ ~ . . , . i_.,,. d .,m,dMdU!'St,.rmr.,:,~1;t>.1 a. ...-}l1.-::·h'.',~-!i16PY~F;1

·::?~~J;;:,.
., ..• ,.

:, <·, •:-~;;:,:·, ·'. ...


.
,;.: :•• ',,
·, ,' ,.;-.;!t.t•;~/1' "• •' •' • . "

it
,,
H
,, ,:1\,,~;,t~\ "." 11 \o,~\t1Mlt.!.IA~ .... ~ ' 1 • • '.;"-'}~,\J;,!,._: (, : . '.,, \ ·• :·.•:.::.=·... ' ,,,1•'•\:,:, . ~-1'•~ ·,, \\, ! .,,

u,. ~JMtJ-l"-t,,...~W1~~}t\l'!'ltt>'/'tt."1'M,w,(-~t1~,~...~,~{N,l",!I.••' .,.1.. ... ,;;Ji,,. ...... ' •••••-:~~\~~•.1,:J,;oJ<WMe-o...~r,..,.Um~~~u-,...,t~h~!l'.1>.-........ ,,., .......,

~l•
156 I \
FLVID bYNAMICS
~ SOtJRCi:S, SINKS AND COUBLeTS (MOTION IN TWO OihlENSION§) 157 , •

......... 3,14,,lmaga of a doublet relative to a'clrc:le (Kanpur:2003) ~ \


Wo p.ro rcqulr<id to flnd thlliinogc 9rt.he doubtetAA'w.r.t'the circle whoso centrn µ'" µ2
ls 0, The ax.ls of the doublet is inclined at ,-n nnglo o: with Ox. Let DA"' f andµ the ' . f
1A!s6,
by similarity orlriangl~s,
strength of t.l:,!1 doublet, Trent !.hls doublet ne o. cpmbinnt.ioo of aink - m ;at A and
source+ f1'I at A' so thnt µ • m. AA' when 17\ -f ""'• ind len,tth ofM'-> 0. The image ' L. OBB' = ~OA'A ~ OAT Un limit)= tt, =
or sink- rn ntA' (ll,1 ls n slo)!. - m at Di,, the lo verse polot or A and ao1.1ree + m ot 0. Tau's the image ofa doublet of strengthµ at A (where 0A =/)relative to a circle
The lmngo or source t m nt A' ts a sourcn t m 11t. B. 1 ; t.he i!werse point of A';and sink Is n douh)et of strengthµ'~ µa 2// 2 at B, the inverse point ofA, the axis of the doublet-,
- m at O. Compoun'tllng theso, we ,i,nd that source+ m ruid sink- m both atO cancel maJ<ies supplementary o.ngle, i.e., rt - a. with tho radius OA. · · ··
each other l\hcl thore ramnlM a doubl~t of atrongth µ"' nL DB' at B, the inverse of
A, . ' . . . . 3, 1~. Clrcle Theorem of Milne-Thomson ritanpur llODO, Garhwal 2003)
:sµppose f (z) iS; the complex potential of a. two dimensional in-otational. :notion ·
of an incompressihl:e liquiq. with no rlgid boundarles ..Then, if\a cnrcular c::(linder
I t F"'a. 'is inserted in the flow field, the complex po~ential ot the reslll~ng motion
is giye\i lly .· , ·· . ·
i '.' l W =f (z) +f (a 2/z) for ! ; !~a .
'}
'r
pr~vldid (z) has 'rici'~ingularlty inside I z I =ll.
i ' . ):·· .• '••' . . l' (Meerul 1992)
• Probr: Let c ofilie cross-section of the circul8l'. cylinder I ~· 1"'a. Th~n on
r
C, z'i., i.:i 2 or i "'a 2!z so that ('i) .. f (a 2lz) nnd so f ('i)'t=c 7(a 2lz) .
Flg.S,ll,
. r
This'= w.,. (z) + i<a 21z}"' <z> + Ci)"' fez>+ (z> . r r r
ForA' ➔ .A~.IJ'-1.B. ' · ~ w "'· real quantity. '
Here we havo 013. OA = ol"' OB', OA', .li:qutiti.ni' Imaginary parts on ·both aldea, II'"' o or w .. const. f'?r .o is also a
a "' radiu.a oi tho ci:rclo, constant.'Thus 'II' ls C(!osta:nt aloofos Uie bouqdary c~:n r:oe~s that C ia a stream line· . ,
in the new flow, , · .• , . . i
,.oii··
H e:qpe, QB' OA'·;,,1
"'o:::f'
'"013'
$0 .c...>
;,,oA·
"' ....., , We know that lf:,: li'e·s outside C, then the poin~ a 2/z is iniitditC, ~s·o it is ~ .
This ..,. t:.OBB' and AQAA' !.Ire siml!nr. tbatf(zl hns singularitiespu'tslde c. Consequently,f (o. 2/z) 8.ll.d f.he:refore i (a 2Jll:)ba$,
BB' OB sin~ari~ies inside C. }t·mea.ns_ that the additional term/ ((/"tz) introduces~ SWff :'; · .
Hence
A'I"'o'A' singu.liµ1ty outside C. In pahicular 7(a 2/z) has no singularity atz ="" ae. /(t) ba1ie;'' ' ·'
1

13 N singularit:Y..at z ,. O. . · . . .~~,
Si~~~ the motion is irrotational and fluid ts incompressible, the ~ { 1
s'a~4Jfy Laplace's equation nnd therefore w will satisfy Laplace'-,~
.twa dirp,ensional irrotational fl6.w of. liquid with C inserted as ·does tbit 111 .
f {z) ~ th~ absence.ore, · . . · · ·.
' ~citti~lt: The Miln.!l•Tbomscb circle tq,~orem p,9vides a Cl.)fflT~
for fitjd);:iff the· i.l:Qage systel"l:!-. of ll given two tlime~siooal syst:ei:p w2
a circul'ar 1boundary. For; it w=f(z) represents tb;: ~ ~ -
0 .' B' O A
F!'g, S,12, the ci~ctil~r boundary I z ·1 ·=a, then w i= f {a 2!z) r e p ~ ibt,·&, ··
)'
I
3, 16. 'trnage of source w.~.t. a 'eirele of ~--(Leit, .... l'arllus
This =- m • IJIJ' = m, AA', gf, ,.I
l
: '013,13). . . ' ·..·. . . '. \ ·.··'
Ooosld-er a so,urc11 ofat:r:engt.b +. ,n at.,-/! !;ha\ tbo. ""l"°- ~::,

. ..B DI ( .AA') OB. OA 'i Lhissourcflls "· · • . ..-· ;"•·' ·. ·~·· ., .. ·'•;,,:. ,,
fi
1.:
• m • .., " m ~ , , i : /(z)=~ra}og~;,./j. tt,,
Tnk,ing limits ns A' -1 A, so that .
· B' ➔ B, we get the
:u.~
a ~ircular ~...
c;orple':t
~
PQteo.tmM:lgifl• .,;i, .,•~1•··•~'
r:.... 1,,
~

' 1· ' . . ~ ,., l:,.


I • ; ' ,
' .....

···-• ..... . '


~
._,, ~,~,-•,'.,•,f'!:--,.;,i!..'I•,•,~•.•••••:,.;,,, ,, . L< ",r'}r,',':•, "J• • ' '.•;.·..:., ··•·'~',:_,.

"1, ,·-~::;;.)!fu-ff}.;i.{HH:Sl}l•i~,t}h'.'t!,',;;N ~i',n'f '?":·:••tp:' ';. <"' ~·H .:-·,1.~

I. ........I ,,,.
SOURCES, SINKS ANO OOUS!.ETS (MOTION IN iWO DIMENSIONS) 159
FLl,ild D~N~MICS
~ . . . 2 This is the complex poteotle.l due to
n-m. ;. log( g_;_z -() . . :•,
\"~.;.:~Wg(.. ~ at z =/witlt !ts a.xis Inclined at an Anglo ci.,
(i) doublet of strength.µ
. ;. . J/, 2 ·µ a 2t/ 2 ·o.t.z ,)a2lf, th~ Inverso pp!n.t o'tz .. f, Its axl_s !s
(!I) douqlet of stron~h
lflr-/)-m log(~) . inclined 11 - &.. •
. z' Fb.r this complex p~tential cir~le is a :stream line I\Od hence the Image system '
•·~Cz-fJ-m log( -=f)( z -'~) for a 9oublet of stre11gth µ at.: ., f ('outside ,the circle) is n ~dOU;blet of sti•ength
µa:
µ' = 21/ 2, and its ~,s:incl!ned at an angle rt '."'io., ·
.;,s,,;,. • . . . , 2
-~~•.io« (;z ... f) -m log~z _ f!.. )-m log (-f) + rn logz 3.18, Blas/us Theorem: . •
. . \;1. f .
n, we g_et

,
··Uw ~ t term - m log(.:. . In steady two dimensional motion given by the complex potential
. 2· w = f(z) = $ + i•lf, if the pressure thrusts on the fixed cylinder of any shape are.
. 1J;J = - m log (z .. f) + m Jog z - m log U-7) represented by a force (X, Y) and a couple of rrioment•!f·about the origin of
~ Tb.ls i.s the complex potential due to coordinates, then neglecting external forces, '
ff) ~+mat
Gi) sink-· m at
z=f,
z = 0, \ · X-iY=!f f(:fe
dz;
(,iii) eocirce + m .at z = a21(. . ·i · . . r ,l; ) z dz]

d 2 .
For th.is COI.llplex poter.,tial, clr~le is a stream lloe and hence the image ~ystem and n" real part of[-·½ P,.; (
for a sowce· + ri:i outsi~e the circle coo.sist.s of a source+ m· at the inverse p9iot and
i . • ~ C

,.
t· Mk-mat the origin, the centr~ ?fthe centre, Since/and d2i/both are irivers~ P,Oints
w.r.t the circle· 1 z I a. 1 ·= . "'

3, 17, Alternative method for .the image of a doublet relative to a c\r~•1J.


• ;:,; '
where pis the dcrnsity and integrals
, . . ,
. Proof: Consider an element as of arc si..trroun<!Jng
tlio point P (x, y) of the fi.xed cyli..nder, c dinusot.es I.be
.
~
are t.a.ken round t.be cont,ol.ll'·c oftp<1 c:yl.li:id~.
I
(Xa,,pu.r 20-01/ ;u,,.,,..,, 19$,1/ Ap-a :ZO()iO, JU»/)
'1

·
,p<ti.""' o
J The comp lox potential f (z) due to a c)ou'blet of streogtli µ at z = fvrifh :itd a;lds boWldary oft.he cylinder of any sb.npe and~ Let lhe
ioclioed nt an aoglc a, Is given by '(l
1 :, ,
l : ta.ogent at P ma.k.e an angle 8 with'X-axia,' 110 th.at t.b.e ~- ; -pJuine
t f(z)Q~. I'' in'« ~-t9. norm al ~t P w.ak.e a.ri.g;le '90' + Owith.'.:t-uia. Tb.e
. z-f , , 1 thrust pas at P acts along i.ri.ward nonxuJ, it.a
f When a circular cylinder I z I = a where a <(,is inserted In the flow of rndtfon,
then the cor:oplex potential is giveo by · · , !
I. components along,: aod y o;i::c:a a.re ~ ·
pd.a c:os COO" + o,. pd:, • sit\ + e> coo·
I - pas. sin 8,p,d.,. o:aa 8. Fig, 3,13,
j w = f(z) + {(~ 2lzJ..JI:t. elide theorem ' i..e.,

j
=~+[(~·)
z - f · . (a 2lz) - f
t1e'"'
z '- f ·.
+~
(a 2/z) - f Hence X= r -pd:s~O.,Y~, ~-= e
J;
".;a . 11; el (n - a)
=.c...=.--
z-f a. 2 -tz =-+
i'eiix
~.-/ f{z-(a 2tnl'
µz e' (n ~ a)
.cos f

, t . . '(. n- a>( z .- -a.2.+-a2J


,,P,
Th.is

=> X - ty' z
J p (- sio

O -, i
!
l'J) ds

= ~ /... . . r r ,.: i "I i JP (/:,:Is O - i inn 0) a'.s, ~


z -:'( · (lz - (c/•1/)J
i I . e . .
-~ ,I! /(11~0) 1!£ i(!l-.a) Ber.nonlbiaei:~·fur'.IIUJO.dy diotioA gives ,1
Ii 'f - z- f +f e + • 2 • a 2 ). l!..'1-l ~ ,..,4.. cr:,a.st. •
T (z-7 . :
i !,
·:
p 2
. ' :' i
'
II lgnoring- the constant termµ e•<n~ a>11, we ge~ I
or . p=(A~½irk f ' .. (l)


r>.
rt
··bl•eia., ~ e/(1t•,!:l{:~,;;,i;,,.::
W"'.z-f+ /2 '( . aZ')'')·C,·:,:,;.;•,•;C·
. z--
r
,.
i :i
; ! :
'i I ff;
~ - 11'= - ~ f (~ :\'i)<co/~ -i:sln 0) ds

1 1
1'. -'• :· "'?' :r·1~; l!~~Y'.··'.f~~;$~~~:t'.•P.l1.n~m,;mt~;.~~{lmr~~•'.!}~;~~,'\\~~···~··'' • ··:~:~·,~·"'·.' :~· '. ::::-:~;i::::-r-·-,~•',\l. . ,-:,,,~~nj'!?/1!~fiN· t~~;
' .. •.;1::-:1; 1t•······~'""~• ,, ' 1 •····· ii. ''•··t• 1~J:mm1t~~'1/.':'1/~~\;,j:½'~~~1.,.:r~~~i~;ff?f,ffl"V!~~~.~{Y!'-~r'.--~~'.~. .~:~~tni"'f~;it: t·:~·,'

. ·':•~:::,.:::::'.' ,!;,~:-:(.::-,-, ,'!.,

:::::;. .·?::,:::);'
f :i ,: ·1~:1, ,: •, .·..<
'. ( ~1 ..:~ ''.f',
·.:.~....::..." •.~-·--"~

,'.: ~.:~: :·-, ;. '


:""t'.• .~·'•;
',> ❖:•:,:.:,

.,)-'n"lt'
',, ... ~ ...-...">'"•""..... ~.••' •.•• '

' i
.,,.,,,""'""'"''""'"'"·~•,•;,•,rc"''"''"'""~~,,,,,n, ,,.,,.~••,.. "' ,.,, •.,. a:,.,JL,,:,,,,.. ,..i:,u•.1,u:.~·••..•a,• ,,._.,,,.,,,:;..;;1~·•1l;;..,,,•• ,..,......,., , ..., ,., •,,.._.,,, ,.,.,,,. , <·-'-'"'''''"'·'""!""'-'~~•-••W:.;;,.ill<l~:;,;:!:lll~,il~l/: ..
I i ,; ·' ; . . ' . : :.. . . \.' .-;::.-i ·.
. ~-
FlUlO O'mAMICS
160 ii \s6u~css,
, , SINKS ANO DOUBLETS (MOTION IN TWO OlM~NSIONS}- ;:;, .L -~· . w
...,_. •~ i crA"Jecb'-;p_~'f
f I ' -I'
(-IJ,:_la;J.r:i 8)cu
' .
'\
I
1
\
'.
: , ·., ·Real part of.[• ff; (~~}<k], by (,3).
\

.
d,: !a. . i I ·,This proves the aecond-required,result. ~
di .. C09 e.. di! .. m o ,.,, tan e .. d:/ld.x.;
But
. :1
I

;: , . 'Solved Prob!e'ms.
.
, , .•
x·.:.d .. ~ {it,e•rocts ltpA J}cu-idy).
'. '

P'rp~lem· i'. A line ,source is in th_e pres~nce of an infinite plarie on wfi#J:i is placeji a'
'·! semi:•eircular cylin.drical boss, the direcaon:o(tMs9,11.rC11 iipal"allelt&,2'!i~ axis rifooss,

But
.
fc~ -
• .
i dy) = f~
• .
= 0, by Ciiuchy's theore~.
~ !
tlie sourcds at il distcmce c from 1he pla/le and-the.axis o/boss, whose·radluf is ih
siiow thr;i the radi11.s· to the point on thii·bc~'s at which tlie velocity is a. maximum'·
i:nr:i:k;'s an i:,ng/e 8 with I.he radius to the sr/il?ce, where
. ' ··~1 'a2+a2 .
~ f~ 2 ... (2) e = cos. [Z(a4 +c4)]11'2
Hc.tic-e X - iY = e- ia ds,
' ' '. . ,
·! l i, , ; . Or , ,.
Let u aod' t.ibe velocity componen:ts. Then we kno.w that
dw . · . ,. D e) i {/ 1th¢. axis ~f Y ~'nd tht circle x 2 + = aZ, are fi.xed boundarie,s an.d there is ai
, e, + iq sm
{ · " , •
-dz "' - u t w ,, - q eos .• " "'•-
. q, cos
. " - ,. s,!J. ·,. , iwp-dtmensior.al :~ource at ?he poi11t (c, 0)1where c > a, show thqOhe r.adiu.s drown·
lrom ithe origin toJl,e poin.t on. thi circle, wherl! the uelocity is a~4na,;{nium, make~'
or
. a;; =-qe•i 8 , ~r ( : }2 dz•'\~~1t?.i 8,.-(dx+ idy) l lf•~h the <U;is of:can angle
i· ' : ;' •l [
2
1.1 + s
2
. •
J.
'
. . or · lddw
, " I
12 ,k "'q 2 a· 218 • (cos & + sin &) d.; • q eA i& ds·.
, :· i o
2 ,,,($)
'!
·COS
. ·
. (2(a4 + c•)111Z

, Uelng thl.11 In (2) we' get the flrat required res\.l.lt, nall'.lely "2a, ,how that the required angle /s·cos~~ 15/✓(34)l, ·

,, . .:
··X - iY •
,
fa
2 ; dz
f( 2
dw -) die.
,;
'I
\ i,
we consider.anticJockwlse momen~s aa poslUve, . . \I~•
'
... The titOIXfijµ(ofthe thrust,i,ae'.~b'oufthe origin is ' {~

\
N,. f• t~ (~ pds sin 6) 5, ~ (j,ds cos 8) xl
• . I I, I
x' +m
A~•I
(-o, 0)
+m
A"
c:.!!.,o>
e. , 7,0.
al
.
)'
+m
.
A (c,c,)
X
~
l
l;
"'J p~ si:h ~+,:co~ 0)
J
~s = f. (A-½ q~) p (;y, sine+,,: cos 0) ds y'
C
/. '
C '
.. I
-·: I I '

~ f q'l ~~in~~ xcos t ds


'
, · ; Flg. 3.14.
f
=Ap fy sin}+ ,::co; e~ ds -
t ,
! I iS~l utlon: The objectsystel:1 consists ofsoureti t m atA (c, 0) with semi:circain:: ~-
J •. I . ~I !

I , C
_bqu.11:djttY and. parts ofy•ax:!s lying outside. Image· system consists o( {a)(i) SQlJ:IU> ~
"'Ap J•
(y dy + x dx) - tJ , e
q 2 0' sine+; cos 8) d.s ,f ~ hi1at'A', the inverse p<:>iilt o,f A..so that OA'; OA' a. 2 or.OA' !=.a 2ta
1- i (Ii) ai.nk - m at O, the eentrli (origin), Ith1·due to circle,
' : (b) Above systel:t) now g1ves l~_dwn Imago; as '
=

B\ltAP fC
l.;y dy + x dy) .. ~P J;r d;r ~ Ap f xd.x. ..
¢ ,C
O + 0, by Cauchy's th'eorem, : (lY; sourc~ +.in ~tA,'.• ailc) ;."t:-< .,:.:, ·.
'.(This is fhe imag~ btA'.relative toy-iµds) .
(z ... -
- ~ (qZ ~ /liids J .
(ii/ BOlJl'.C!,l +m.atA"' (z;.:; c) ..
Hence N
.
=tteili'iiirt ~;[
, . Jc ( T¥s. is the image of A relative to y•axis. ·
I :niiYsink- m at o.. .
............. , ;, \

\
I
- , -'.<'• .....,;•~·-·. ::~~··~·•·.,
~.," ,)::.!-'.'.t:.,;.,,.:,,.,._,_;;-:i);,;._\,t .. '.
· .. ·., ..... ','.)(;,~.·t; ,'. ...... _•. ,,_,_'

so~RCES, SINKS ANO ooueLETs (MOTION IN T'WO 01M1:NslqN~ · i ~(


164 FU.110 DYNAMIC$
:·1· w= -, m
. -1r(r2+a2} ·] ... (:
(z+a)(z-..a). ,..'.. ..:2-~ or,: tLUl '- (r2 :., 02) tan E)
or = - m. l og . z . .. - m.,,,,. ::-::;--
\·;i ·. >
IJ./
•I,! By(l),atr=a,'!f=-m7t/2=const.
or log(-;-} w =- m · ' and ate =. 7tl2, w= - mn/2 "' const ·
. Also when 8 = 0, w= 0 = const.
. . . (·~J~')
Second Part.: We have w ,. - m log -z-· • · 0A \s stream line when 8"' 0
OB il stream line when 8 = n/2 i
or ~ + i'!' = - m k>g (r ~ - a~ + m log re:1~. : ; and arc AB is stream line when r,; a .
Eq uat.i.ng iro agi.na.ry p:art.a, · · .~ 1
Thus the image· sys:tem for the fluid moti6n bo{mded by qua\idrantal ar
'. ;

=- m i-2 sin 20 ) t an -1 (,-sine).·


--
Q:4BO due to sink - mat 0, source+ mat A w~uld be,a source -t· m a.tA'. ·
111 ( ,-2 .cos 20 - a2 • ·t.'m
· r cos 0 . I ~'.t~blem,Y.Withiii a circ_ular boundary of ra.diiis a there is· two dim.en~i~nal liqui,
1.-
,, ~ m [ tan·l ( . r2 sin 28 · ) .;. taii•l (·§!!0)_·)1 : rtiotlon kue to a source p,:oducing liquid at.the rate mi at a distance {{tom the cen.tr1
r.2 cos 28 - a 2 . . cos 8 J . : dnd. an equal sink cit the centre. Find the ueloeity pote,rtial and show that the resu.ltan,
'df. i~e pr~ss'iire·on .thr. boundary Is p m2 f~l[Za. 2n ta. 2 -/2)), Deduce ds a Umll, tlu
_ t . "'I [ ,-2;(s/n 29 cos El -sin 8 coa 29) -~ o 2 sine] u~ld_cit;y potential d'ue' to-a .doublet gt the c,entrc, ' (Apa :JOOO; Konpur 90)
.. man 2· 2- . 2 .
· (,:- cos 28 - a I cos e + r sin 28 sin EL : ·, , Solution: Liquid Is g!!.ncrated due to a source nt the rate m nt the pointA
For tan- 1 a - tan~ 1 &:= ta.n" 1 !; :b . ·. ·] . OAF f, Let k be the strength of the eo.urce'1 tbon:
07 def. 2rck = m or k = m/211. the object systom ·
A1 P

consists of (i) a source + k aL A (ii) sink - h at'


and log (x + i;y) = -1 iog (x 2 + .:,2) + i tan".1 ,t. O:Thi:i image system consists oi(I)' source + h
A 2 • • X
2 ~t:A', the inverse of A eo that OA', OA .. u 2 or·
or \II"'_ m tan•l [ ,-2 ~In (20 - fl)+ o slo O ] !oA''.=f'=a 2lfa.nd ll.su:.k-k ll.tO, .
,-2 cos cze .;. a) - a 2 cos e ·, (ii)' sink k at Infinity, tho inverse point, 0
2
or
2
W == _ m tan•l [ (r + ) sin 9 ~ a ... (1) . and .a source + k at 'O. A'
• k
. . · (,-2 - a 2 ) COB 8 1 .. ; Source + k and sink - k both nt O cancel
\v = - m (ll - a) gives the stream lines whJch make anglo a at A. By m Md (2), ;each other. FiDaJly, the object Md its image ·
jsy'st~m consists of source -t k atA, source + Ii at :
-m (11- a)=..:m tan•l[ 1,-2 + a2) sin 8 ]. )Af; sink - k .at 0, Slnk at lo.llnity ls neglected, Flg,3.10,
. . . cr2 ..; a 2) cos 8 :slrlce-it has rio effed on fluJd motion.
1
or
• (?. + 0 2) sin 9 , \ \ ·. ;The complex potential due to objectsystoai with bouodary is equivaJe.r:i1•.
-tl\Jl a._= (r2-a2) cos e, · ;to ,complex. potimtial due· to object. sysem. and its image· system with no rlgict-.,
'.bouodary, Hence w is giv.eri by - ' -~.
or .:. sin 0.. cos 8, cr-2 :-" a 2 )'== cr2 + a 2) sin 8, cos o. · [;: i ·
w"'--:klog(z-1)-klog(z-/')+klogz. . .. Cl?fl
r
2 sin (o. + O) = a 2 sin (o.!- 9), :' 'Equating rea! parts from both sides, · : '1~
or
Remnrk .: To justify tbq h:nage sys\<Jro of the above 'p'r'obkm :
·. · .
i ~=~k l~g I
z - I - k log I z - f'' I + k log r I" I '
Let OA be o boundi.ng rndlu$. Co'nslcler a source+ mat A, s:tnk • m !jt O.;Take · · =-k IogAP-k logA'P +k log OP
an image sourc·e + m 'l:it'A' s, t. I ·· · · ·. · ,;, = -k log
AP A'P
op,
=
OA OA' "'a, Then complex potential W ls given by ,. I I Jw'
k . k
w =.,.
m log (z-:- a) + m. log(: .:. o) - m log (z - a) · Second-Part: By(l),-:;:- =-
. I ', . z- - -z- 1 , +-
z
"-" f
this gives · :
ll. ( ' )2 ~ + ~ + J.. + 2[:;5;;,;:::;-.3_., :.,,;,, ·- __ 1 - - __l
~.
'41 "' - m tan- 1 (
'
~~:;~:;.~;:,J(By
> • • / ' ' ;- • ~, •: -'• V,: '
equatt~~JH,~f
• •
ih. nbov~ solut:onl

. ~2
' .
C

(z ;. fr ('Z :;_ f"? .,z, · (ii,-/) (z


.
- /") Z (z - f) ti,; 1:/ (Z -
,:. ~- -~.
:.'.......-•·

.··- -

:::•.';t::. ·:·'""•:-::::::r·•,,.,.,,.:,;
>,: ,·.

:, \~~-; :~< .. ''


:,:;:,::;:;•,. -------=-=· .-~~:t7',",t

~-~-T--,,
,,.·' ,' :1 ..','~:...... ~··' ···•'·' .-•~.,,•-•.•

-~ .. I ·. ..
so'uRC~S. SINKS AND OOU!3LeTS (MOTION IN rw6 DIMENSIONS) 163,
; .. N, ~
'- a al;O ~tiVll toy-axis). . 2a2c~
Lli!ao tq object ~ m wHh rigid boundarr!s cos28o:.~ ... (el
a. + C ,
':illll1 its ,lmagl,,,irys.t.em withoii rigJd boundary. 'Now
-1[
"" '
Ett•-~lo,r,(z-c),-m log( z- c )+2m'log(z-0)
. l
0
2 •
or
l. 2a2r.2 . ] .
e .. -2 cos .I -a 4-+c
.- 4
This givea the posltfoo of the·point where.pressure is miniuium as:
l !
!+,½q2•0 . .
i

; !~:! ,1
- rr(JiS~ (z + c) - z+- I
' I •* 4 '.
~! ;_ suggests thatp Is minimum lf'q Is ma.xlm~m.
w "' - m log (z 2 - c 2 )- m log( .v 2 - ; 2 .)+.21n logz 2 . 2azc2 ,
BY,(6). 2cos 8.-1.=-,-.-,
· a•+c•
)· dw
-=-2m [ - -z + -- . l] ·
.. '·1! 2
.t
dz
'
z2 - c2 2
z -
;:· ·or 2 cos. e= !a24+ c2,2
4
.or : .·cos2 e - (a2 + c2)2
2(a4 +c4)
· •· a +c

dw:.._ 2m(z 4 -a 4} a2· + c2


· I or or CQS ~ = -,---,--.,..,.,.,,, ... (7)
dz - , · 4 .. •
~·... [2(a◄ + c◄i) 1/2
. '.r,~. .·.
. z (z 2 - c2) ( z2 .., :2 ) ::::'.~ ., . . l
·,,;, ' . (l+4)a 5
8
If l c=Za, coa,8= f ◄ )In C -1(34)
Ifq Is v,eloclty atz "aei , thon · , i; •l2{17a ) ·
. .
. • a / !l:!e / a 2m I a·4 el4B,_ a4 l Sim.i.ID.t' P-roblcm: lrl'a two dun.en..storial motion ofan lnfinite !iqui.i:I there i.s a rlgid
q dz .
!ae'e ! • I a2e12e_c21. j a2e;2e_JL
. .
'
c2
4
I 'I. bou,ddry COM/.stirig,qf /hat pa~ onhe circle,?+ 1 2 "' a. 2 which lies in the firs, and
{ou1tl1 quadrants a.ndiheparttJ 0{1-a:i:is· which lie outside lhe circle, A simple source .
2mac2 !'ei48:.. i ! .. of str~ngth m ls pl~ced. al the poin.t If, 0) where:f> a, Pro1;e that lhe ~peed oft he flu.id .
or
q" Jaa,;2e_c2 l, ! a2er2e_;a2 .,. (1); , e,
at the point (a c,os a sin~) of · the. · semicircular /Joundar;y . is
But Ie 140
- l ) 2 ,. (COB .'rn - 1)2',+ sfo.2 'iia
., '
4a;;,,"/ 2 sin 28/[a4 +1 4 .;. 2a 2f 2 20). · co~..

I Find ai' what speed of(he bounda17 theprimure ls least.


.. 2- 2 cos 1Q"' 4 (sin 2e)2
Hlo t: ·Put c "(in,tho above problem and refer equations (5) and'(7).
, . e110 - l I "'. 2 sin 20 l i "'~.
I· t 2 e' 28 - a 2 12 = (c,:: cos 28 - a 2) 2 + (c 2 s!n 29)1' liii1 Problem 2. A region Is bounded b;y a fi,xed quadranta·~ ara and ita ra.dU, with a source
and an equal sink at th.e ends of on.e of the bounding radii. Whow that the motion is
"~~ +a4. - 2a 2c2 cos 20 • j
'''.' \y
'<:!) 'I' given b;y
Ia2 e' 29 - c2 !2 w1(a 2 cos 26 - c\2 +(a 2 sin aa)2 '
~2 a2)
w=-m'log ( ---,-, .
=a 4 +~4 - 2a 2~2 cos 28 • z: ' 'i'
Writing'(l) with ~he h~1p or'{2), (3), ~,i ), · and proue th.at the stream line leaving either the source or the sink at an an'g/e o. with
I'
4mne 2 sln 28 . the radius is r 2 ~ia (o. + 8) "'o.2 sii:i (o; - 8), _: ·· · Y
q "! (a ◄ + c 1 - 2a 2c2 cos 28) S!j!lut-ioo: The object system and.its image sys tom a
consis\s of (i) source + m: at A (z = a), (ii) sink ,,,•··
q b maxiroum if .!l. [ sin 29 · J= O·
1
- m at 1Z ;,; 0, (ii!) source+ m atA.' (Z,:: - a): .. /
de a 4 + a4 - 2a 2c 2 cos 28 Ii
Th~ complex poiential due to objec~ ~ystemy,ith /
'l'hi.i, giv~.
is
tigid boundary equ.ivale'bt to tha comphiX po(entiaJ --A,;-,,---,oc+/---A~- ·
1
N
2 c,o,s 2 O (a ◄ + J◄ - 2a 2c 2 co1/2e)'- sin 28 (4a 2c2 sin ·29) ~ O due to opject system a.nd it.s image .system with no rigid .,. m - in +m
3
2 (u~ + c ◄ ) coo 28 - 4a 2c 2 = 0 boundary, heoce complex potential !s given by Flir, •16 ·

' ' w., - m log(: +·o) +,m log.('!:-;- 0) - m log(• - a)

.~w;'.~StiliWJltiJ\:\,~8ti11.:11.qmr1i:m1m~:iti.r,·et,::i!~l1m:rnmmum,mrftti!:!llt~~1:mnffiffi'ir~immHmr-::•~'cc%i11:i:,,~:':;,,,.· •..

.,,.,. • #, . ~ • " · ~ ' .... ~ ,-•,·


,, .... ·.,·"

, iJ I l .. . . .
'iW • FLUIO ~YNA 1¢s ' SOIJfi~ES, SINKS ANO OOU8Ll:TS (MOTION IN iwO DIMENSIONS) . 167
' . . .· ' i • ' - • ' i i '.
The poles inside the bou,ndary c Qfthe <:\l'cle are z ~ O and~-"'(, Hence ;th~ ~up. . - ' 2:'
otthe residues .o,ftthe function i • · : , : ma
=,."' 2nfr
,I, e
cos .
• Ans.
l(dw) ;,!1 is the required velocity potential.
.-k"', -dz at z-=. Q
. '
, ,, ,
' ,..
Remark. By (1),
tind z ={is obtained· ~Y adding the eoeflicien~s or¾ a~d z: f or w = - k log ( 1 -f )- k log (.,z - '
a( )
2

Sumo f res1'd'ues=r-r'-7+7+7+p=(/-(')f'
2 2 2 2 2 2/.
· Dy Cauchy's resid\ies theorem, · ·11 =- k 1011,( 1-1fl-k lo.;( :..i-5. )( 1-~),
J]z ( ~; )2 dz=. · = - k log ( 1 - f )- k l~g ( i -;;i} ·' negfoct!~g constant,
C

dw )2
2ni [Sum ofrasldue;J·

4nfk 2i ·
. •
= k [ ~· loi:(1 -n- l~g ( 1 -;~ )J
f + .~,) + ( ~ + "' )1,
or J_ _

dz = :- ifrf')f'' ck [(
By Blausius theo~ern,
rj_w ) 2 dz ,..ff?. 4nfk 2i . ' :. :!
or . [l ft.]
w=k z'.;.·a2· ,
' .
X- iY= .ff?.
2 df ' 2 '({-(')(' .' 2 : ~
~ :,·2np f k2 = Znp f 3 m2 : :j Ir we make ( ➔ 0 so th~c • 00 , Uien wo got a doublet at tho cantra and its
I l'I

-azf ( --r
az
f
) 0 2 (a2 _ fz) • iln:2 ,.i i strengthµ" k(. Thon w;. l!z + 1£.
a2
or X - iY = p / 3 m 2l2a 2 n (a 2. - / 2), r.
': I
. • ryal
Equating i parts,$=µ
,· - +2 (lr) cos e. Alls,
Equating real and imaginary parts, wo get , r a
. X = p f 3 m.212n aQ. (a 2 - ( 2), Y = Q, · ·- Thus we get two n.nsw~rs for the two limits namely f ➔ 0 °and-(➔ ...
Resultant pressure op thn' boundary. Problem 4, A s'ource of fluid situated in 11pacc of two dimensions i.;.of,uch ·str~ngl/:l
· +
= (.1:2 f'2)l/2., pm2(3/zn 02 (a2 _ (2), that 2~pµ represents the mass offT,u!d of4enstr,y·{:i'emitted per unit oltlme, Show that
the force necessary to holi:i a. circular di* at rest in. the plane o/source ts
Third Part: To deduce velocity pote~tiol i:Jue to a doublet at O.as a
. t 2rcpµ2a2/r(r2-a2), '
If w~ take limit as f--. ""• then A-+ oo and hence neglected.'-AlsoA' comes ~e~
the point O. We have alreatiy a sink- k at O and we have brought a source ~ear it. where a is the radius of ihe disc an.d,: the distance of ihe source from its cent,:~,.'/~,
This c-ombination forms~ doublet ofstrength µwhereµ= k. (a 21f) as f..:, ,.,;. · what'direclioit is the disc urged by the pr:essuref. · (Kanpur 2006)
Now w becoroes 1
. $elution: Let X and Ybe the coiupbnents'of the required force. Then we have
I ',. to pr9ve that · : i
w"' - k log (Ji; - /') + k log z _as source+ k at A is n·eglect~d,, ..
:i . . 2
·, 1 : ,: j
../ (X2 +y2) = . 2rpµ
a~· ,
2 r''
o.r w=-klogJ(z- 0( )=-klog(l:-/:i) ! ;, r (r - a·) 2
:i;
This;> r > a. By Dlausius theor,em, t
.
=<k
[ 0
+:
2· · 1 ( 0 2 )2 , ] --2
fz 2 fz' + ... Fo'r-log(l-x)=:r+'°z+,.,:
'. ,: I i
. , I. d J .. f.
2 . X - iY = !p. ( -d~ ) e4•, i·
r k(z neglecti~g ~igher1,gi-ee terms A i:
2 ".{, ~ 10···, ·:-..-: · where c repres~nts ~he
the 601.iudaJ\~r fl ;:
Q •
. ·
rq r, ----.-
m.a ma e
=--- I: disc. Sinco 2rcpµ represents the mnss o(the ::'
~-'•
2:J< (r •'-0. 2wf nuid ,_emitted e.t A h,ence strengi.h of the Fig, 3,l7, ti
t'
' ~t?~,,~.,,••~•• •
~,•,~:•, '" :• •,•:~•;<••~•~ ••:•~ ,, :v":,~~~¾ ~;~~~:"'.'•::r~:~~':":"'~.~,~~~~~""?:'!':':1'~~}'.~;'l:~~-~. "' ,,, '"/' • ►v,:,,• ..~...t~.,~~~.')'/':,'t,~~!rJffff~'}.~~~l;~.''l\V•~-lo/"'f•~•'1""':"''"~g•/~~~•~,..v•• "~,,......,.....~1)~~~11;;,~(,:.;..:,/:

,,,· '. ,'',~ '' ,:,:, \.'•: ';n:~•.<•• 'N;'' '~. '' :: ... 1::~:::'

---------~...,-=~~
. '. . '. ''' ·-·· ;
__ __:......---_..:..-----

' , .r<'_,1.·
· .. ,.:,1 .. ,.-,.,, ..

J1,
~,i. . ,,._,._ ......-..-... ,..... - ...·----'·""-- . ......
~ .,, ·.
.• ,.:, .: ' :, ·.: .:.,,,,c,~·•~'-'r~-~IJ''"'; •:=~:~!!~,;~-"',~:tm,.W~1'.f),'::.:"'•''(''''"'"/.i"';'.~'"'"\~H(\Hi:••~"''.' '"•'~~•
~ 1 · 1ea Fw10 oyN111,,<.cs $QU1RC~i, SlNKS ANO OOU6l.lITS (MOTION IN TWO O!MENSIONS) 1
1 e9
.I I ·

I
t . , ,. 11:; , .
sources lsµ. The !mngo d8<lul"C1l +µat A (OA = r) is e. source+µ at the inverse point stre'ar:rl lines rl!Veals thaL the pressure is greater on the opposite side of the disc th all·
l A' s,t. DA.QA'= a2 1LI:1d sin,k- µ at O; : I ; that'df.t.he sou:,:e, · ! • 1
'•''""
Then OA' • ai,,. ",', Bllf, ! '. lR~~n.rk ! The_ above problem can be expressed as :
ll The ~or:oplux ?<JUlntfal due t.o obj~ qstem wi~h n!!id b9undlU}' is eqJive.lent \ ·l S1ofu that the force per uni: length e:terted on a clr,culcir cyUn.der, radiu.s a., du.e
\ to the complex pot.on ti Ill due t.o tho obj~ e)'l!U!m and lt~ image. systr witn rigid
bolll'.lda.ry. Hence • ; , I
ro to;a. 'ai,urpe of strength m,. ~I a di$tance II from the axis ts
i' l. ! 1. a
2itpm. 2 2/c (c 2 - a2). · · '. (Kanpur 1993, 91tMeerut 91)
1 ' . , ·
w • .~ µ log<•'.~ r) • 11 log (z - 1{ + µ log (z - 0) ,
1
·
!!!e•-µ[-L+-L-1]· : P.roblen;i. 5. What arrtmg~meiit of source~ a.nd sinks will give rise to the function ·
d: · :-r z-r' z .' . . ' '' \ ' i '. ..
1 du, 2 1 1 1 . · :. . Z Z ' .Z =
w lo-g (•,-: : . ) ? D~.w a rough sketch of the'st,:eam ~ines in this case and prove that
, ; µ2 ( 7fi) " (z - r)2 ;-:-;:;+ 1ft. +. (z ~ r,) (z - r') - z (z - r') "; ~ -
+
two of them sub•diuide into the circler"' a and a.xis ofy, .
(Kanpur 200:J, 2001, 2005/ qa.rJ,wal 2002, 2004)
The !;unction 2 ( cl: ) has po}es ·z = o. and z = r' wl.lhill c. Residue e.t z., O ls
1 d 2 · · . ; , . . '
· ·: , · ( ·2 I
tc µ . ' J • ·solution: Given w .. log 2: - a.i: } · · · .. · ,
the a~ of'coefficlents
. . orlz whJch is,, e(}unl to · .'.!'?ts\=>
'.; w "'log\,-.-.,,-·
·- /'z ·-a ,,. log=--.~.
2 2)
(z-a)(~+a) • ·

[------1
!I f)2 ,. · ·. l
~l!--

z-r' z~r r-0.=2(-+-


r'•r. ·. or ! i'; w=log(z-a)+log(z+a)-log(z-0). , · ,.:·,.'. ,.,(1)
Residue at.I!"' r' ..:sum oreoefficlenta of' l/(i .:: r')
2
-- [ ---- 21· 2.'2 .,,
, , Th!sishows thiit;tlie given arrangement consists oft:wo sinks e~ch of strength·
- ~ a'.t ~ :=:a,, and: =.,. a, and a source of strength + 1 at the origin. Ans,
z - r z ;, r ..=----
r' - r :r'· .· \ . ~econd Part: Te> determine stream lines. ·
.Sum ofre~!dues at: ., O z = r' I and 1 By <i),
9
.
-----+-+-~--_,,,
2 · 2 2 2. 2r:
. . . .,__ 2a 2 · i ] ;\ l $ +i'!I = log (x - a. + t_y) + log (.i: + a + i,Y) - log (x + ij), .
(a.2 _ r2) r'·
r' - r r' r' r (t:' - t) r : ~~u~t!ng imagi~ p~, · . . ' ·· . : . ··
By Cauchy's residues theorem, !. ' ' : • \jl = ta.n-l ..L + ta.n-l ..L - tan•l ,'):,
· · · .i:-a ' ···x+a · x
J)i (9f yd::: 2rd, ~utn;ofresld~eswltbl.!l C
y

. ' ' :za2 .


" 2r,;l, -----.
(t'.12 '."·r2) r
We have seen that.
:r .~
. X-iY"'lf!J.(Z) ,r:b:

i,ii
C
2nt 2a2 i '
¼2 nµ2p
~ X
·,,
"' ·~ · (~ _?) r,
1
~ r(r2:- a.2/
2llnl1~ . ,
· This ~ X"7" 2 , y .. o
r(1· - a')
. 2 2 ~
"" ..J (x"l + Y~)., ~ 2£ .2.
. · r (r - a ) , .
,
Thia. also dP.cl1trP.a that tho force Is p'.u.rely along Wt, the disc will bo urged to !·
move alonr, OA, Alst) tho cylintlP.r I~ ,d.tra~tud towards the source, and sketch 'of the Pi;/, ~.HJ,

il!m't.1i1<-il!<l\l~:,.,1f.,;'.iilt.t<r,r,:t6~~~:c1r.;,;1:tmm,i1m,,,,1,,m,>;1~>:r1: :1.~~~!,l'l.;;,,;im1•:,:,,•,1,,,.,w,;1?.l:1,i;::,J,li:i;g~u11>111Wfl;1!FH'i'J"',1:,, ,,•.11':f'" ,,•i·, ·,,, • •: ·

\
,.,., ..... -.•,)<.•. · .• ·,1 ..... ,.• ..

. •: -1, : :.: ; ,:-; ~\ ;.: ': :Ot\...:,>;{-:t,:•;~»:IM!~:,•.1 '.•:,•n~h''"'~: ':l.1

"
6
I'
i . . .. · .._, ............,. ,,
170 .:.'· ;-,~ir; FLUID o{Nd!.1J , . I . :
SOURCES, SINKS ANO ooueurrs (MOTION.IN TWO DIMENSIONS)
·11
171
-i[ ly/(.:i:-a)+y/(~+a) J'·-t an ~ll i' I'
,I:: I
' · I.i'
"ta T~ei~ages:,1em consist.s ofs9urce+ µ'at~; and sink-'µ etO .'•rnke O OS ori,gin
' n "1(
. ' - 'l X "i a
) ·1( , ,y X + a) X ; ;1
OS as real axis; then.the equation complex potential Is given by· of
=tan -l 2xy ' ·.-tan- 1 J:: · ' •,.
·i
.22 2
x--a-y,
.
.:i
. 'w = - µ log (Z - r):-:µ log (z - r') +µlog 0) (z -
= tan~l I· 2xyl(x 2 2
,- a -/) - (ylx) '] ··dz
dw = _ --;J!_ _ ...JL + 1£
z - r z-; ,.,- ·z
· • l +fJl/x),2xy/(x 2 - a 2 -y 2) _hL ~
2
j,,
- :l ·x (x2, + y2 + a 2~ , .
- tao n
.l
.-.i
.
I I · I dw
q "' d.z I= I..=.Jl..
z-
: -1 · z - r' r
I
r - z - r +·z . "''µ _· (b - r) (z - r') z I,,
l · X (X 2 + y~ - (I 2) '
·z2 - 02 I . .. . .
. Stream lines are given by \II "'con st,.
_ Y (~2 .j. y2 + a2) : .•
I
=f (z - r), [z - (a 2/r)) z ' ' ; .
' :· • ., ·
tan 1 . 2 2 = const,, In order to de\~r:n:i.itie velocity aL any point on the boundary of tho disc, we shall
x (x + y - a 2) put z = ~ .eia, . . . .,
Y (x2 + ,;2 + a2)
or
2 . 2
X (:>: + y
.2
- a ).
= const,, •• i
: l'.

•t'. Then q = µ. j . 0 2e2,a _02 .


..
I
·
If corist. " 0, then (2) ~ y (x 2 + y 2 +a 2)" O , (ae 16 - r) [(oe - (a 2/r)) a16 e/ 8) ,

~' Y:= 0. for x 2 +y 2 + a 2 1' o. !I,.! _ or -:. l


q-µr
, '. 2(cos228 -21) :+ (sin' 20)~ 2 2
~ ]112
'
' ' ' .
If const. '; "",.then (2) ~. ~:':(:,:~ +y 2 - a2)" O.,,;,: "o:r2 = 2 ,. a . ,'
I
i l(a cos a -r) + a sl11 e1 ftr cos e - o) + r2 sln2 .Ell
_:,. ~ X·=O r=a. .· ',. . i , 1·1 2r st:ii a '
or
. . But x'= 0 repr1.lseotsy-axis and~,. a represents circle with radius a and 'ce~tre: q f µ (o 2 + r 2 - 2,ar cos a)l.12 (o 2 + r 2 - ~ra cos 0) 1f.!
at the or!gfo. ~hus we see that'. partlc'Jlar stream lines nre y-axls and th~ cJ\-~la, 2J:!..r sln ~
a.1 or ... (lJ
r =:i I . • q = o. 2 + r2 ;. 2ar cos a'
. of the strea~ lines is as given in figure 3,18,
A rough sketch '
For q to be miLXimu.m, ~ = O, this~
1
Sirnilar Pl'oblem. What arrangement of sources an.d sir1ks wttl give rise 'wthe. , ' I

f1mctio1r w = log (; _¾) rDraw a rough sketch Qf strea/'/1, lilies. in. th£; case an~ proie• Zµr cos j e (a2 + r2 - 2ar cos 0) - 2ar sin2 8 ) = 0
(a 2 + r 2 - 2ar co~ 0)i .
that two of them. s1,1~diuide ,lrllo the circler= f'an.d axis·ofy,. · (Meerul .Jl803)
Hint, On replacing ci by l in the abov_e problem, we get tl\ls--prqblein. ': ' or r2 - 2ar cos 9) cos e -bar sh:1 2 9 ':' 0
+

k· ' .· by · so~r~r: : . ~s e "'2art(a 2 +


or ... (2)
Proble~-·~-·/n. lh.e., ca~e· of·tuio dim~n.sionalfluid motion. p~oduced .. a of
strength. ).\' placed at a. ·paint S · Q Th~ value of!),.given'by (2), gives maximum velocity.
_outside' a rigid ·circular disc of · . _ 1 .' ! · (2) => sin 6 = (r 2 ~ a 2 ver2 + a 2). :
radius a- wh'ose .aen.tre is 0, show· .
that ueloaity ofsllp ofthe'{luid.lr:i · _.,,,--- ,
contact with'lhc disc I• 'greatest al .. /
P
'

- 1·
1
B (1),
't qm""
.
;,.,2µ.rj k (r -a )/(,-ta) !\
' 2

a +r,2-•2ar(2ar/(o2+,.2)J,
:2 2 2 ·!

the points where the li1i'es join.'in.g ?. 2 !


S to the eri,4$ .· of the. diameter at I _ 2µr (r - a ) _ ~
1'
right angles to OS cut the circle,
am.J proue that its magnitude at
0 \V c• J ::::-,s r ; '
- (r2 _ a2)2 - r2 _0 2
Ihese po in.ts is, . or . qma:c··" 2µrt(r2 - a 2).
2µr/(r 2 - a 2), where OS= r,
,., The velocity will be nlo.ng the ~ o n bf tangent lo the boundary and will be
equal to tbe velocity of slip as the boundary of ~he disc is a stream llne,
Sohition, Let S' be the
inverse point oC S w,r.t~ the circle Rema:rlt. This result is o.ls-<?.exp~iblc as
-~'· .... ,.,,.
so that
OS'= a 2lr = r'
OS.OS',=a 2
·
or
Fig, 3.19,
qrn,ir - os2 - a2'
•,· '

--..
,.-~
•, ••~',J!?;~!'.); ,,1, ,,,.,,\.~,.... , • '"·' :1 •;,:\1,•~~·-• ,~,•'f'l/~~flW;'~':~~~"'!{...,~";~"l•'tl•'X~~~nffi'~~\...,~~-::mm";....,, ..... -:-;;; ·:~::··;.::i' . ,,.• ,i ~ ,,...... ,:~P.\•'.J'IH~HOf;~:,::~~;"&:•V1}l'!~tv.\:/11H~}~rJll''·"':""!*~fJffl1f,~ :~i,, ··-:·. ,. '' ,, .••.., ....... ,.,' .,~, ··••H•;~•:~1::1'.'""' ''"' .~ ,.,,.,,,,.,n~i.,;•11!-{l'Ur-i),,"l'_,ikj.'•.

·, ·;•::·,:::,~:•i · .' -:;:-'.•.::-:-~' r~ 1'.:,f}::·:: •:··


' ;'.¾~:••'.~ ,',. ' .1,_,:,;<1:,:,: ',·, ·•:-:1;:;:•::••· ·: :. ' -~ .;, :( .' .' ';~:,: ;:,,:,

_ _ _ _ _ _..,.....,.,......-,-~,..~.-.c,~~::,·,~::-m;1!
' ,.P;~,,; .
·:..•·
... ,,. ,,'

. . ,... ,. ··· "l·••· ·······'· · • ,;. ··'"f···'""':·;,:·,,••..,,..,.....,~=;=,i·"''":, ,·.,., 1;,,.,,:,Nc-Wlrn,;,,.,......,•.:,..'-'··'·'""'"""'•'·"· ,,,,~,,;irn,,,.,.., . ,,,.,,,, ...
1'1..IJIO OYNAMIJs ..
172 sou~~$, $INKS ANO _OC:,IJ6L~~MOTlON IN TWO DIMEN§I_ONS) . • 173
, I
Problem ' I , ~ t.JMtfi:-l:~ O•~ c>n.d e .. -it/4, there;is a two,. ! :· i . _ , r• (a 4 ~ b4 ) sin 48. \ ·.],
...
:~.,:
dlmen.Jicn.al [ud4 m,~-.o,ub,enc ~ o f ~ mat the point (r =,a, :e,. O) t:md
an equo.l rin,\ ai tla(,•b,& .. ~~Ula1. tt~s:t:rf!(.Jm/lmclion is i' lcleoco. . iv=-m te.n 11L ra-r4 (a4+b4) cos 4e+a• b4-
. t::im•l [ r4 (a,c; - b~ sin 40 . ']. ; this completes the first part.
-m r'-r◄ (a 4 ~b°')coo48+oh 4 . '
n:
and /Ml. tb.t Pdc,d.q at (r, 0) i& dw ;. _ m (4.z3) t 4mz3 •,
dz z4 _ a4 . 4_ b4 ·
_ 4m (~ 4 -o 4)r 3 .. . 2

. ·:.. ,(r. - 2a r◄ ctJs 48 +·a 8),1.r2 (r 8 .;, 2b 4 r4 c~s 49 +b8)112 '


8 4
'
.. ,
.
(Meerul
. ti(
"-4= a4-b4
(t~ - c 4 ) (z~ ,..
.. b. 4 ) J
Solatlon..ConBlder tho trii.naforni'a.tio?) ~ ·., z 2 which tnaps point.!! frora z-plane ' l dw I . . 4mr3 {a·◄;~ 1> 4)
t4 ~-plan<),·~t" "'r e18 a.od ~ = ge'a, I.hen q =· Z '" I (r4 e'40 _ a4) (r◄ 6'48·_ b4)
1
. )' I •·P • 11 • ""'·
.S''
a - tt/4 ., .
6 ·- 7<12 l;•pl~ne
or; q=
. · . ,4m r 3 (a 4 - b 4) ·
I ---

'• ((r t a - 2a 4r 4 cos 49) (r8 + b 8 -


8 6
2b r cos 18))112
4 4

.This complet~s- the pro9Jcm. ··


Pr'o,ble.m 8, Between 1he fued boundaries a= and e = - 7!.l&:'~here is a twoi't/6
+111 i +m .·-m -m 1
-+-m di,;w1sional liquid/motion due to a source at the point r = c, e = ct;and ·a sink at the
0 x: (:_al,O}(• bZ,O)_ (_bl,,Q) (ol,O) \ · o.rl~il'l, absorbing water at the same rate as the source produces it:-'Find _the stream
f,u1:ciion anci show that one of the stream lines is a part of the curue
l ; i ;: ; r 3 sin 3a" c3 sin 39, (Ka,npur !WOO, Garhµ;al 2003, 2002)
1
; ~oJution. Consider the map ~ =z 3 frorri z•plane t.o se ~-;:,lane. Le,
9 - nl4
; =: r eie·, {,. R eiP, Then R e/~ = r 3 e'38 , this ~
~ - 1r.l2
~
'.!'lg; 3.20, R = r3 · · ;., {l), p= 39 ... (2)
By: this: map the boundari;;s e = ± 1t/6 al'1? mapped on. the bound~ies ~ = :t rr.12; i.e.,
C: =z 2 ~R e'6 "',J, ~l~S ~R = r2, 8 •dl& n-exla !.s the new boundary In ~-plane. The point~ (c, a) tnd \0, 0) !n z.•plane arc
=
Also e :t. ;v4 so that 6 = ± 'Tfl'.2, i.e., T)•axls, mar,ped respectively on. the pQLots ·(c 3, 3o:) ntid (0, 0) by virtue o!' ('J.) and (2). The
obje.r;~ system co!)slsts of(i) source+ m a~·(c3, 3o.) aod {II)- mat (0, 0), The image
1
By thls transror:viatlon polnts.(a, Q) aod (b, 0) ln'z-plarto are iµaru,ed on (a 2 • OJ·
and (~ 2, 0). ill ~~plane. The;lmages of'+ m. at(a 2 ,. OJ a.nd - n; 11t.(b 2, Orio <;-plane w.t.t: s:,,st.em :con.suits oftl) source+ mat (c 3, n - 3a) and (ii) sln,k- mat (0, 0) w,r.t. T\•!IXIR.
11-axls·are -1< m. at (- a.2·, 0) and - mat (- t-1, 0), respoc:t!vaJy,' . · The complex potential ·i.,9 given by · · •
· The complex· potential due to object system with rigid houlldary !s equivalent \.u ~ - m log(~ - c 3 e 130) + m log{~ - 0) - m log{~ - c 3 e1 (n- aaJ) + m log (i:; - 0)
to the complex potcnt!,t1l due to objec~ system and lts Image system without rigid
boundary. Th!o ~ /
y l . Z•plane e~ '1116 ff " p - '1112 ~-plane

w" _. m log.(~.- a 2) - m log(~ +a 2l + m Jof (~ - b2) + m 1; (~ + b'.2)


=.: m log {~2 -b~) + m lps;(t:2 .- b•) . 1 .,. :n
Ol" w == - log (z'1 - a 4) + m tog.(z 4· - 6 4) , .. (l)
-m
0

4 48
= - m log (r e' -a") + m log (r 4 ei1 8 - b4 ) 0 ~ X --~~t;)J'-' ~

Equating imaginary pnrts, ·


ljl ~
= m [ tno•l (
4
4
r sin 48 \ )·- ta.n·1 ( ~ sin 49
4
r cos Ml - ci-1 . r 4 CO$ 49 - b4
'"'··· ' 8 • - n/6
Since
'"
y
tan· 1 x - tan •i ·= ta.n· 1 (x - >')/(l
.
+~} f:\•-n/2

t~:< :·. ·· ;.t:i·.: i~:V,'~:.

l!iffi,1~Xl'l.!f!~J~\l~:mf!~ffl\!llfj,"(!Jl;mtrt:\5'.':U'.f]ffr,\tl,',',;~ffl~tlf"1i•f:lli;;:t;,1:i,i,~li:1►,l'il!.ltt;,i'.M!!Utiffl~W&\ln(!\1l\il'.''l:•li\!'i',)t1''.: i•• ·,·,••·

. ' ,;-:··_i: ,,. ,•,•:•'. {(.' •,..•.


•.~. :1,-\'tp•, 1-:•·,,
.·.,.•·, .. ,-. ,'

, 'I
t
FLUID GlYN/\MICS
"l

·. : . .
SOURCES, SINKS AND 0,0UBLITTS (MOTION IN TWO O!MENSION_$_)
'
i~ th'e strength of the source nt F·whero OF= c, Then liy
176

"'. .. 1'ic" l; - mlog(~..: c 3 ei 3~) ·-·m lpg (~ + c 3 e· 13,;xJ Solution. Suppo_se µ


~ .'~=z3, .·. . def. of strength y ·
21tµp "'mp
- - ~ l_og ?-
m log (z3 -c3 lla) (z3 + c~ e~i3a) ml~=µ,
--~ •'6m logz - m log (z 6 ..: c6 - 2/ ~3 c3 sin so:i ' . The boundary isy•axls. The image of a iource Q
= 6m log rel(! - m log (r / 68 ".' c6 - 2i c~ sin 3ct, r 3 / 38 )
6 m
+ 211
• I "'•
at P (c, 0) is a s~urce + m/21'. at P '(-'_ c, 0),
= 6m log r eia - m log [(r 6 cos 60 Jc 6 + 2.1 3 r~;sirt"3a, sin 39 1

Now the complex. potenti!l-1 \s · m
~ \• 1 -r,;
m .
. • . . · ·
&quat.ing 1magrnary parts on both sides,
•. • + i 0 sin .60 :.. 2r3 c3 sin
•. .1
cr aa ,; cbs! aa>i w = - !2l 1og (z - c) - El log (zl + c)
l'' Q p ........... ~
211 . .211 .
,, ., Sm t,ai,-1 ( r sin 0 ) ·· 11.I I
Q'

~
. r cos .8 . = - ..!!1.. log (zl - c 2)
2n •
- m tao-1 ( . ,.e si~ 68 - 2r3 a3 sin·s·o:. cos ae' ) . ,I
Fig, 3,22,
· r6cos 68.- c6 .+ 2c3 r 3 slo 3cz , sin sa . ct.w m 2z
Strcao:i lines are given b·y · ~ .. const., i.e., ; d.:z = - 2n . z2 - c2
.,
. . 6 ·
• errie _ m tan~! ( . r s!n 68 -2r c s!o.3ct, ~os 38 ) ,. const,'
> J · . I l 1 ·,.
t . ,:':, cos 69 - c6 + Zc 3 r 3 sin 3.!;t , sin 30 • q= I: I" I~ • cz I z2 [ ~

Taking const

.•,o, we'
get pa"rticular stream lines llS . . . '.
I I For an.y point on,-,axls,z = iy, so that
· 6 •. 3 3
6m0 _ m tan-1 ( Sin 69 - 2r C sin 3<:t, COS 39 ·)"' Q
f'
qU ni j-
· z- 1 ·Iii m 1· iv
~ I '
~ m:..
r 61cos 60 - c6 + 2c3 ,.3 :Sin Sci. sin 38
- - 11111
n z2:.c2 n -y~-cz n(y2+c2)
. 6 • · 3 3 , . t
or 60 ,. fan -1 ( 6 r. s1n 60 -6 2r c3 sin 3a , cos 30 . ) . :•i This is the expression for velocity at any:pblnt ony,ll.X!s. By Be~nou!B's equation
r cos 60 - c + 2c r3 sin 3a . sin sa ". steady motion, ·
or sin 60 . (r6 cos 60 - c6 + 2c 3 , 3 sin 3ct. sin 36) ·. .12.+l,;,2:A,
· = cos 60 (r 6 sin 60 - 2c 3 ~3 sin 3o·."· . ,cos-38) :.
p 2
Subf:ectlng this to the condition ,
or :. c siii ea.+ 2c 3, 3 sin 3a. co~ (69 -:!e) ~-6 · •
6 · \ ': I
p=JJoWh~ny=«>,q=O,wogetA"Po/P, , I,
or 2r 3 sin 3a,. cos}a-_c 3 sins~:= 0 :)\ (· '
(Sinc_e velod(y is negligible at gre_at <Jistance),
or _ 2cos 39 !r3 sin 3'et - c3 sin 39] 0 = ·i '.) \ i .
This _·=cos 30 = O, _ : 1•• ! 1.. ,(3) Hen~e I!.+ 1 2 Po (
. .
. r 3 sin3ci =c• 3 sin-38. .
.
· ,, :
' .
i1
1: I' . p zq "~·
, I
., By (3 ), a=± _rr/6 wh! ~h giv,es· n~: ri~w .stream Jines a$ these are the giv~n;.~tte~m
lines, The 9ther stream.line Is a part o{the cu_rvo . , ,; \ ?res.sure on QQ' ..f -I
pd:y'.
r 3 'sln So." c3 sln 8, I ,·:; \
t ' •

in
. '!

Prob him 9 1 hi t'tie ca,Se of motion. of liquid a part of a plane ·bounded by sirc.lght
"line due to a source"in theplar;e prove that ifmp is the mr:;ss o(the liquid (ofd,ensity
a ·' But" P - Po = - l
.
zP<f'.,,,; ./ to - J' dy - z1 P I
q2 dy

p) generated a.i the ~oum per unit o(l lme, 1)1e press1/re on the length 21 ofthe bou,11dary1
Required difference of pressure
to
im.mediately opposite the source is les$ thri.n that O/.l o.n equal length at ti creat f
distance by . . .: .. . ·. .
l2. >n. f? (_l tan-! ff_·.,.:J.,.()'.:
2
· ' ·
i .· r1 l
--
'ft?. c. c' i2 + .. c2 "J /po - p) dy = zP
. -1

,
where c is .the distancJ
l.
of the source from the boundary,
,
(Kanpiil"
,
2boo)
. , . •· , . . ,-~r,~.-r;~~•l"'~~~:i-~t~:""~•~~~'"t"?l~H~i.Juu~u,1,,:
.
....,, .. ,,... ,,, ...,,:::'.~U~m!:'.'~~"'i~•t'.1!il~::::~··\~\\ID:\~-~r,;"!Wl',~~ml1[!:,;rr:'::o:i'!1:7l:j':f,'?.'.:,i:!:'::7'.'"'."::··;:•'.T'l;'.i'.':''.'~!"l;,,;r.a:,,':"'1,~,,llflllf~riw;\~~~ ·

1/'.:~'.i~:-·· ,. :•:: ...'.:: '··:·.:~:;::;i'


,;:,;:,•,;,
', :;;:-•~(•'.<·:,: =.,·,~: :·•:~-::·
' '.O(f.~:!~\'.'·
"'"""""""·".\l'""''·'',;;,, .. ,..........t t , ~ - ; : ; ; l t

':: ~:·:::::·::;
\}.
': ;, '~ ', t :·
••:.1!,'.,:.!"',:,: ,':,

. I
•y--·•. ,., •-• .. __ .........._, _ _ .__,_, .. _~~ ........~ .. ~i.L,.,.~H!IHl!.....,,1,..,,..,~"'•••'' ~·• :, ... ;.. '.', .......~ , ,,', ,~ .'. ,.~,. ~,,.,..,,,,..,,.,~.,.;,,;,,1,1~ol:1~~!4-~~U.•4•< ~~:.~,;. • •.: •. •• •. '; '.}i'!Utalhlli '•• ~••«• .,.,,,.,.,,1,"••••• .. i.. .. •1H;,;~,•1fl"lf1:H~t-Jf'r'\fl'•:, .. . ,.,

, Si::,\JACES, SINKS ANO DOU8l§I$_ (MOTION IN TW_O OiMENl3iONS) 177·


176 PWIO DYNAMICS
r!ere
... ~. ";
m2
i
ll1'L
2 Bet2 µ Bet2 = 4µ. ·
_Ldy'. µ' ,._i:;::....., µ-- =
•7p (f + t?f ' '',.
r2 cP'- zo:2
, ! i l . 11 e'Jl/2 4U 1--P
fo. dy .. c ~ O dO J •
(Put >' = c t.ac ' · : ·'.Phus w = ...<:.::..- +- -

· ;.~
.. 2. e1
c
2 2
~ ◄ ·~sec~8d8•~8
f \ : . 2
·
81. . i l
'sln 2 erl'9,wh~retan81'7c'
i; !
:,, i
/1: - i Sci. .z: -.3¢.
... ~ - - + ~ ·
x
i.,
+l (.y -
' '
3a) · {,: ... 3d)
4 '
+ l;i,1
rr o c. se-c . e:, : · rr. .
c .o, · ,I . ·
_ µi Ix.:. i ()';.. 3o.}l. •· 4µ. l(x ..;•so.)·.,. i:tl
..
~
2 f 0
0
I
( l - cos 28}..aO
(
'

•. ),
" -:,-
m2p [
211""¢
8 - ;;1 sin
"
, 28
'
I• ·
' - x2 + ()' - 30.)2 + . (x - 30.)a +y2
Equadbg r~al pa.rls Oil both sides, '
$ . [·. y - 3x + . 4 (X - 30.) ]
2
":' m 2p {8 1 - sin e1 c,os ei1J =·m
7
2
Q.[ tan- 1 -( - 2le2 ] = µ x 2 + (j, .. 3a) 2 (,:,.,. 3c:.)2 +y 2 • ·
~11 C . 211 c C l +c
This con~)udes the problem.
m2n[·1- tan .1 -l· - -l- ]
=~ Proved.
'• .~·' '

. ?roblct:n 11. ln the part of.an infinite plane bounded b:; a circular,"quadran.tAB and
, z11 2 C C /2 + c2 '
· the' production ofj,fte radii OA, OE; ·there is a two dimension.al·•motlon. due to the
· Probl~m 10. Within a ~igid boundary iri. the form of :the circle ,production of liquid at A, and its abrorptic:1ri at B, at the uniforrr; ra'te m. Find the
"(x + o-,2 + (y ~ 4o.) 2 ,, Bo. 2, there is liquid motiori due to doublet o( stre,iglh µ at fhe : vel.oi:iiy potential of-the motion.; an.d show that ihe fluids which issue from A in. th~
point (0, 3o.i with its axis along them:ls ofy. Show that uelocity potential is . direction. making an. angleµ with OA follows the path whose polar equation i$ 1
, µ [ 4 (x - 3Ct) + : y - 3o. ) I
'::
! ;
r = a si.n 112 28 (cotµ+ ,!cot2.l""",; + cosec2 28l) 112 t ..
(:,; - 3o.) 2 + y 2 x2,+ (y- 3a)2 \1~/pofiliv~ sign being taken fonill the squar.e roots.
f.
i ' (Kanpur 1991) j i ,:;Solution.. The object system 't
~Solution. The r!g\d boundl!J')':is a circlo g\ven by :cbosi~ts of source+ ml2n a~A and ( ·,n · n1 )
f
:-iml2it at B. Tne ·Image of· "".2n•.-2,i I B
(x + o.)2 + (J' -·40.)2 ... ac:2 •. '+·(m/211) at A w.r.t.,.ctrcular · ,,,•···· f
Tho c~~tre Is (- a:, 41l) and ;-ad;ua '., ✓(Sc:2 ). :bbJo~ary is a sourcCl +m/21! at A, . /
1 ~
Oliject doublet ls at P (Q, Setj with its axis along y•aicis, CM and PN 11.re
perpend/cuJa.:rs on X•.BXls and CM :rcspectlv~ly, Produce CP to meet x•nxis at Q.
the. in.verse point ot A aod .sink (
-'m/2/t at 0. The Image or sink
,-·m:12natB.w.r.t.clrcla!saslnk
• '
Ai--;--.,;,-,-='----+
A ( ~nn, ~)
n ~--rl n,. •, I N •
Evldeotl;y, ON" NP •·a so that £.NPC "45' lfnd therefore ,t.CQM"' 45' so that .. ,
- mt:Zit nt B, the inverse point or \· /
(4'1i .«
.,
CQ • ../ {(4o:) 2 + 2J • 4a'12., . . B ·and so1.1rce + m!Zn at 0, Th.e •••., · ,,,,'
Hance CM "'Mfi• -la, IY SOUNe,+m/2n and sink :--m/2n '••··it'(' m Ill)
Obsor,,'e thnt 1 bpth at O cancel each other. . · Tii' 1;
linage w.r.t, bounding plane. . Fig. !U4.
CP. CQ,. 0.12. 4q✓2
• 'i)he image of source. f . . .. . .. . . ' fi
," aa 2 .i (rndius>2, :+·mi2nbt ✓l is a source+ ml2n atN. w.r.t. Line BB'and' image of. iiink - ml2n at .8
Honco Q IB tho Inverso point 'W;•r;t· ithe line.~' is, a sink, ml21t at B'. Also the images at A and B have theLr -
or P w.r.t'the circle. Tho lmogo of : ,ilJlages +m!~n and - ml.Zit at A' and B' ~vdly. . . • ·
the doublet ·µ nt P (0, 3a) w.r.t ,:
c!rcle is ·a doubletµ' at the inverse
i Ir'21
i'T~ c object and ·its image sjirtern ca~slsts of 2 sources of s'trength ii
point Q (Set,O) mth its a.xis along
· x-axis, For object aod !~age
1-r
:+
1

! Ir\ie
1
at A, 2 sinks· pf :Straogth m/2li: ai B, .two sources + m/2n at A'. two sinks
12rcatB': ·. . · . • ·
M

,.,
compleJt potential.due to'ob;cct. ~ with rigid boundary is equlval~ot
~
doublets tl).ako supplomontary M O ',,; x
jtd th.e t~mp!ex pet.ontial doe t o ~ - ~ o.nd its Uriage .systems with no rigid fill
. angles with the Llne CQ. Fig, S,23, b~ll;)d!UJ', Thu!! .
1
,,, .. ,.,",-..... 4-

\
.'', '.: ;"'.'·""';;',!!)''.' ,,.,:;)'~.;(1~~;::~}'atffi~'

• ,. :-:•• .': '•·• , ~\ '>'i<>',"'• • ' l

··.·~•-"···~· .,,
; i\• !~';;.';.~.-~; }~•:H}~,(1)'.,',{i1Uia,~~O:'}!~r1 '· !·11 •,r,,,,)~ , .. ,.., • :~:~~ '.>n ·, ,,,

FLUID DY,NAMICS. SOURCES, SINKS ANO DOUBLETS (MOTION IN TWO OIMl:NSIONS) 179
~ •.• I . ,. .
2m 1 ) 2m I . ( · · 2m ·:( , 2m l . I. i' . ~Y;roble~-12; Two sourc~s, ec.c~·of strength m, ar~ placed at the poi_nts (- a, 0) (,t~d
'" = - 2it og (z - a - 2n og z +. a) + ~ 1og z - 1a,1 + 2n ~g 1z + ;a ,
' / (a; 0) a~d a sink of strength 2m is placed at the or/gin. Show that the stream lines
. are curues .· ' . . .,
,., w = - !I!:. log (z - a) - !I!:. log (z + a) + !H- log (z -•ia) + !!l Jog (z + ia) ... (1)
(x 2 + y 2)2 .. a 2 (x 2 -y 2 ·+ ;1.xyJ, whe~c >. is a p.irameter,.
n 1t . " ;i;
Equating real pa_rt, Ol) both sides, Show also that·the !Juid speed at any point is 2ma 2lr 1r 2r~ w·irere r 1, r2, r3 are
respectively the distances ofthf poipt from the'tolfrce and che sink .. (Oari,wa·L .2000)
~=- !I!:. !log Iz - a I +; Iog I z +a I~-. Iog I z •- ia I - log I i + ia Il
1t
So\utioo. The complex potenthl at anr point P (z) is given by
, p _. I
n Oog PA + log PA' - log, PB - log PB'J
= - !I!:.

or m PA, PA' 1
¢=-;log~. rl.

This is the required expression for velo'cJty potential. Again by (1),


l
\ ·1, I
o + iw = - ~ l~g (z 11 - ti 2) + .1!:!.1og (z 2 + a 2) ' I
1t . •71 I
or (;) + iw = - !!!. [log (r2 e12e - a2) .- log (r2 e12e + a2)J. B · A(o,0)
·1t
'
(-~,Olm 0-2111 Ill
!.
Equatl:ig imaginary parts, Fig, 3,2G, i:
i::
·l( rz rcos2sin28) 2
t -1( r sin26 )'] m. log (z +ia) + 2m log (z -
0

m[t w "' - m log (z - a) - 0) f


= - n · an 20 .; a 2 - an r 2 cos 28 + a 2 ... m ~
\II
or w = •· m log (z 2 - a 2 ) + m log z 2
m t -i (
2 2
2a r sin 29 · J or <I>+ iljl ='"Flog cP - a 2 -I}+ 2,'.zy) + m log (x-'2 -i + u;;;,.),. ~

"' - ii 4 2
an r 1 cos 20 - a'i + r "sin 28
2
..._(2)
Equating !magio,ary parts, . . 1 ~
·For tan- 1 x -tan·l y = tan· 1 [(x -y)/(l + xy)].
\
Ill= -
_
m ta.o· 1 2xy
x2 _ 0 2 _ :r +_m ;t.a.o" 1 --1::L.
x2 _ ,2
!
I
For a particular strcamyn·e which leaves A at an angleµ, ·
m !
\j/ = --; µ. .:.; (3) " - m t.a:.n
. -I 2.dl lry
-=--=--=-~~-,,----- /:
I ex2 - Y') (.x-2 - al - y) • u-2y2 I

l
fly (2)'nnd (3), i :~ Stream liiies are glvco by 1/1 ., co.o.;;1., i.e., ·
m m t I -1 '.2a 2 r 2 sin 28 ;_! ?..:i:?;:Y
'• : ' •l ( I2), say
- TT µ "' - TT an r4 - a4 · ' -
· -I
m-t.an , 1 . .JI.. 2 ,..:...2 .. - m Lan
.
. i.r--;r1(.;r.·-cr--;r1+ .... ;r:·
. .JI..
f
Thi t 2a 2 rz sin
r• - a·'
28
. :•.1 or •,i.,i.:r:,,; c:c-2 -:?r _a2 ~ _ y2} + 4J:2y2 :~
l'
a

=(r2J
,:;; an µ =
or -_-).Q:i_r}'" (r'1- + :?r- ~-~- :r) }

This is quadratic in r 2 ,
2
2a 2 r 2 sin 29 cotµ - a 1 " O.
-
or =
~ + :t¥ a 3 (:? - -? + l.x:r) where?-- is variable paramotE/r: a ..;{-~ '
I, Thia eo:m.plctes tho 5:rst ~ of the problem, ·1
2 2a 2 sin 20 cot~± .v/4c,. 4 sin 2 213 cot 2 µ + 4c,. 4) I = I·-:- ?-a
~ ~ I ,,
:.
Hence r =
Tl'.king positive radical sign,
. 2 ·
I
!
,_J.
Flows,--,_"
, 4,,,
dz

- ..
2 + ·z · ..,.

2111'1 2 2 2
2ma2
lz (zi-a 2) I
. - .:..1!!£_· •, .•
.,,
1,
~
r = la 2 sin 2S cot
~.;
cot2 µ +. l))Y~ .
I . -. lz'!/:::l#-;sA.1~.:•I: +.aJ~ifl~it:s\":'~tt·. ., ..
. "·:
or r =a {sin 28)JIZ [cot 11 +:V/ifr,t' co~ec-2. ~effij2t; _ This t"Oncludc.~ the problem.
This is the required ~ath.

,',, :!,i,:;:,., ,, .., ... _,


. ,:/~i!.!f.F~H\..!;! 1,' ,,,;,}'>',,.\:0,~--·,:1 :i:;;W~i)~H;•;•~~ , '~ •·:·· -~ ,; -~·- ,,,:. (j)t:,;-:,.;:' •' •, ',•,!::;j{f;?f ;; • ',:,-:
-~~;.\;}?{;--::, ,-:i,:.;%';;~~;:/'.1.:-:·.... ,:•·-~··,~::-;·,· , .. ..
,., ::•::::•:,/ ', .:>:\~:?~: --:~';'::..-:~ ::<

...... ,-_,"-··.
I
. ~-
',.;·;~:,;: ·.;./. ' .. .,, ...,,.,,~'..:•\: ..._..,,
.. ,·
'' •.• , -~ f, '
.' ',-.-::.-:,<·'
l\'-;;••i'.
·,.:1:--.',,'

'.!
·.. 4::.i.'..!..•.. ;.:.,,~-~-~...'"4\."""'1fl>H~/.ll••·u.. ~ ! . ! : t f .~.,,~~•., • ::i,, ·; :·,· ;.;1::1.:tuJ".;1;~, , .•,,, .... ,·.,.'..(;, ,,;,\, .~•«;..pv•, .,, .. ,\~i••t1Hh)\,\•• ... •• '• '
...... ~.-~ ... - ..---~- ,__ ...., *"' • -- ... - ..... ~~~~•~ltitUMtlt1~1'cftllil?(!<.'ro."1·..':'.·,-.,,:.,..1"•"•·••'M .~.:."
,,
, 180
, I . .
F\.UlO DYNAI.IICS
.
: .: · : '.
SOURCES, SINKS ANO OOUB_tEfSJMC•TION IN TWO DIMENSIONS)
\
,01

Problom 13, The spAce on one side o'( an 'itt(Wte p1;.,,~ waJl:,,. 0 I, fif/.ed c,;i.th i.\ ; n l ~ n 1 .,2 1 2 ,.,,. n-e,
lnvl$Cid, incompres,ibla fluid, moving at iplini-0' with 'i>tl~ U i.ri tM dir~ccidn of · ·'I'l\us "-+-q• =-+- v- or -(q - tr)= -
I ! , ,p 2 p 2 2 P ,
x•axis, Tr.e moUon of the fluid is wiw/Jy two ~ I in':r.:,-plan.e. A doubfol·of
stren9th µ is at a di,,ta11ce a from the wall ajtd tM JX)ClWJ ill t/i¢ n.eyariue direclibn of or 11:..e. = 2µ2 (a2 - x2)2 + 2µ U (02 - x2)' using (2).
x,axis, Show that ifµ< 4a 2 U, lhe pr-d.rure oftJvr: fhri,J. on t1i'e wo.11 is maximum at · p (a2 + x2)4 .{a2 t x2)2
points distant a../3 from O, the foot of~ ~ l o r from tM dopblet on ;ihe u.:all _ ,! fl:E. = - 8)f2 :C (a2 - ,:2) - 16µ2 }02 - ,:,2)2 • X
and ts a minlmum·a·/so, •· I. 1 · J
p d.x · (a2 + x~)4 (a'"+ x2)5
Ifµ"' 4a 2.u,:~~d points wh.ere t h ~ ~ ofu,;_,fbnd, iz r.:erryand show that stream · · 2 2

.lines include t.ite circle. · .


,:!~+·(y-a) 2 =4a 2,
·
•·
tt·
•·
Y ·
+· 2µ LJ C -
L(a.2.+.x 2P,
21; - 4x (a - X ) ']
(o 2 + x 2)3

··.: . Solution. Since the points


doublet are In the n()gllttve d:im:tion
klhe ·• · ___.,.,u 2 2 2
=8µ x(a -x )a2+xz+ 2 (ci2-x2)- 4µU:c a2+x2+ 2 (a2-xZJ
(a2 + .,,2)5 (o2 + ,?)3
of x•aids. so tbnt·tht; doublet make:s \'lrl
arigle-nwjthx-axis. l.tnag,,ofthi!.~ :0 X
i 1.42. = 4µx (3a 2 -x2) ~Zµ (0 2... .,,2) + U(a2 + x2)2~j
: p dx (a2 + .,,2)5 ~ . . · I
doublet ls an ~ua.l doublet llimDarl,y i . ·. ··, d• .
oriented at z., - ia. · .. ,Fpr extremu1;1 values._ofp, ~ = 0, this => x (:)a 2 - x2) = 0 soJhnt:,: •. 0, :t a ✓3,
Tho S)'S~em cociaist.s o_bject ..
doublet, imag1,:"doublet and rtremn
with vel6ci'ty U parallel lo z•e::rls. i\,hus, ifµ~ '1g 2'u, then d < 0 so thatp .is maximum whe~i.=
2
f
Again,'ir , a✓3,
· Hen~ ·
F'lg. 3,26. i !1 : •• • dx . . .,,
~ !<'4atU. tben d 2~7dx2 > o where z:: 0 so that p is minim.um. Consid~r the case In
w =~+~-:Uz' which'.µ= 4a 2U. .
Z - Ill Z + la '.
· •: • · dw '
. ' ·, Let the fluid velocity= 0, so that dz = 0, then (1) =>
" - -4
z -1a ·- -L,.
z + 10 - Uz
4a 2 U (a 2 -
·U+ 2~. .,, z 2) •
"' - - ~ - Uz . 2.. z.s 2 =0
z2 + a2 ., ,. .. (a, +: J
·,
or ·, (z2 +a 2)2 +' 80 2 (a 2 - z 2) = 0.
·-;.: ddw " U + 2 2µ 2 {lz2 + a2 - 2z21 · On the wall this becomes, .
'_.fl.!£. ..
Z (Z+a,)·.
.
V +•2).l (a - z\
2 :. : I (x 2 +a2} 1 + Sa 2 (a 2 - ~) "'0 ·· .
or ... (l) .... i t' ' ' '.
dz • · (z2 + 0 2)2 . .i;: ! · :i:4 -, 6a2x2 + 9a 4 ~ 0 or (x2 3q~)2 '"0 or x = :t o../3 1
-

I• !.. Iu + 21¼ ·ca2+ i z2> I 1·r :A)ns. (± a✓3, 0) are the points wherb velocity vanls~es,
I_·f!E!
'
dz 1?1
!
(a 2

,2)2
'
' · !iro determine stream lines, . · ·. ·
J' ! '. ' ~ '
I
For MY, point on tho wall, z "x·so that, li'.Wehnvew:--,,~;..uz .
, U + ~I-! (a2 - "'2l ' , ·• j'\ ,, zt,~.·. ;- ..
. Q" . (x2+ a.2)~ .': !..': 1' t +•Ill=
. - t
2.4a 2 U (x M cx 2 + o 2.- y2 - 2l xy)·
=:::...::"--"H-~-""--;:'-;:=-'-~~"'-'""-'- Uz
. 2. ..2 _'.4µ2 {a2 - :i:2)2+ ~µU (02:.. .,,2l
;: i (x2 + 0 2_-;;2)2 + 4x2.,,2 ·
"Th'
IS => q - u- -
(x2 + a2) ◄
n
(x2 + o2)'
, '
... (2)
or . - IV "' 8a2 U I- 2.x2y ~•:,, (x2• +a2 - y2H+ Uy .
To determine pressure at a,ny point on the wall. By Bernoulli's equation for , +a2 _ y2;)Z + 4x2_y2 ·.. , .
{X2 .

steady motion, fl.+


1
- q2 = C. Subjecting this to the condition p = n, q = U where
1
' .S\reamlines are given by '{I = cils.t. Take cons't. = 0 Then str.eam lioes a.re g;M:l1!1 1
p 2 . . i! : . 8o2y2 (- 2,;2
b111J1 = O, i.e.
u2 u2.-;i,Zr .
2 2 + y =0
: . ·i·'i1
n l U~ = C.
z ....., so that-+ ; , ;
" 22
(x +a" -y ) + 4x y · >'
2
p .
•· ......,.,,,. __ _
· o~ · · 8a 2 !a 2 - (x2 + i°>J + (,:'2 - .,,2)2 ~ cl~ f 2a2 {x2 - ,r2) + -ixZ.,,2 = 9.ti,fc,.' · J

~~uvit~tJ:-Ht,r,,~mttH

~~~~1i~l!~!~t,~'-~~t;;~;~~~~Jm;:;t1:!~"t;.>:>t't~t:,!,-:i ':{',?~'.: i.~-. •:


,; ;,.;;;;.:i•;_~~~;:,~;;::;5:~Pi~.~;.;,;;r.·i :1:i::-;,:i~.~:y,;· :,:,M,Mg! •.-:,;• ,;,:

........ "·~·
i ! !.'
• ' • I
I
t
. • '
. I
FLIJIO OYNA))116$ SOURCES; SINKS ANO OOIJ6LETS (MOTION IN 'TWO. OIMENSIONS) !!;!.'
9o◄ - aa2 (x2 +ll + (.i+ y2)a + 2a2 (x2 -:,-2) ..i: 0 ; I' = nr {cos :i;t + 0) - sln (nl + 9)1 , ... (1)

""
~ + ?)2 ;. 6a 2; 2 - lOa2,./• + 9a 4 = 0
(.r2 .. y2 _ 3a2)2 _ 4 a2y2 '., 0 ·
' i
This,,;.~ [r cos (nt + 8)]' = :t {rs!n (nl + G)J
l.o'tegrating, . r cos (nl + 9) - r sin (nt + 8) ,. A: , .. (2)
cv (r2 .. y2 - 3'12 - 2ay) (~2 + Yz - 3a2 +2ay) = 0. S1rbiecting this to initial ·condition, when I" 0 ':
This Includes th~ circle x 2 + y 2 - 2oy - 3a 2 = o. r cos e =X .. xo, r sine "'Y """o• ... (3)
• ' • ' j
...,,_ ,,2 + '(y - o)2,. 4a2, .,!1 we ,A = Xp..: Yd• ' I
~bk:;m 14. Fiitd tlie lines ofilow in two di~en.sional flu.id diotion giuan \. cos (nl + 9) :_~sin (nt -1- 0) ~ xo - Y,o .'
. ' n. . • 1 d d ·.
¢ + i'II,. - 2 (x + iyf2 eZint, •
Hence, by (l), di fcos (nt -t- 6)] "'di. [r ~in (n.t + 8)) "n (x 0 -lo),
:. ' ~

Prwe or verify that the palhs of.the particles of the fluid (in polar co•ordino/es) ln'teg:rating, r cos {n.l + 0) "r sin (nt + 9) = nt (x 0 - Yo) +.B(, (4) ·
be obtained by eUminaiing I from the equations \ .' I This , ~r:cos(11t.+8)=n.1;(x 0 -y0)+B.
r cos (nt + 0) - x 0 =.r sin (nl + 0) - Yo= nt. (x 0 - y 9). 1' I
Subjediog this to (3),. x0 .. 0 + B. ·. •
Solution, Writex + iy = rei 8, it is:given that
,.· cos (nl + 8) = ,u; (xo - .>'o) -t- -"o•
<b + - = - n (x +· iy} 2 e 21111
i.w · =- n. r ze'·20 . e1·2nt, :1.·
or r cos (nt + 6) "xo = nli (xo - yo),
2 2 .j
Ag.aic, (4) gives r sin (nt + 8) -yo= - iii (xo - Yo)
.,w .¢ + iw = - ½nrz e12 (ni + OJ • '}:
Combining the l~ two equations,
2 . 2 i
This~ ib = -
11
; cos 2 (nt t 0), w" "'. n; sii:1 2 (nt + 6).
r c~s (nl + 0) - .to= r llin (111 + 9) -Yo = n.t_ (xo -;~), ,.
! ! This concludes the problem. · ~ hI
Lines of lol<' are given by \j/ = const:; I.e.,
"'j'Jo > F'roblem l&. Uu the method of images to prove thal If there biz, a source m al the ~
-n; 2 '
·•in 2 (nl + ()) = const, ,::;j), po1.n.l "il in a/Iuui bou.rv:!ed_ l:r.i the
?"'
U~r
8 "!° and_~ = iv3, the solrttion. is . ~
Le.. r 2 •sin 2 (ltl + 0),. const
<> + i',I = - m log(,: -•zl);(za - i:6)l, • l
woere :to= "o + i;yo, :t' o ~ xo ,- iyo.
!'
. . ~ : ( Kan,:iu_r .20021 2004)
By.def., r =-or .. nr cos 2 (nt -t- 8). % "pl.a"'

l
y. ~ ~ • plane
· 1£2 l .[ nr 2 .. · ] 8. x/J
rll = - ,. ae = - ; .T sin 2 (nt + e) • 2
zi
•m
or 8 = - 11 r sin 2 (nt + 9) 1:
I
-1; Ir cos (nt + 9)) = r cos (nl + 0j - (11 -t- 0) r sin (nl + 0) !l - •
I

" nr cos 2 (ttl + 0) cos (11/ + 9-)!n - 1t sin 2 (111 + 0)) r sin (nl 1+. 0) ·
,=nr [cos 2 l(tti + 0) - (nl + 0)) - sin (nt + 0)1 . •
nf
0 8•0 • -cir-•r'"•"" ";i":";; ~
= nr [cos (nt +·.01 - sin (nt + 0)1. i Z:Jo (•ml ,,
Similarly wa can·show tha't '· F!g.'s.z1. i
f [r sin (111 + 8)) = ~r [cos (111 + 0) ,._~in (nl-t; 0)). ~
Soloi.loo: Cooaider:).}'e Ill~~- 0 ~ 3 rr~~·- ~;!)lai:i.t.. to· ,<;•plane, whel-~ I:;
z"' re18 , <"'
ru, 1ll so th.at RJ!l·~ r3.P 8, This ,b R :...}3 f'tl= M."hitii:e tho source+ m at •i;
H~C'<I we see that
d
dl. Lr t;of (111
·
+ 6)
• d
= dt[r sin (nl + O)Jl , , , ,·
zo in z- pln.oe i.s mappo<! on tbe source + m at z& ln ~-plane. Also th~ boundaries Ji --..
. \ ' . :t : e"' o, e =wain z-plooc become 13,; O, 13" n1/,e.. .:;-axis. · !).
ii
iJ ·:·:•·:•,1:••t•'·':·•·Y."r.';:>,'.",!llilll1!1t'l,~!~l!!>'ffilfl~1~1!!~~1!1/l;~.,,,,.,.:,,•~•~•-•...,:-;;-::.:~~~--•~~-~"":r.})1
:j !!'."'"''.'""''':'l'~~mn:~~~~~i::::::•--·''"·'~::"'":;•,;1~~!'?.~~~~~~l!:m'ir.::ct·':::::;i'.::~-i.lr.:\r$:': ,;:.·
:1 '

':,l ,·~·: •:•:: :,•,;:· •' , •;-·· .. ,.~,,n;~:";i;, .


..
:: ;,•,·,:··· ··>:·::.~:·•;'.,:
:, ~~•'.·~c,;:' , . .·,,:,:•.:;:,;.,' '·:::~~ '.'.t>,.

.:/:~:~~:?: ."
'
'
.
•:..
. '

---~~~;...u-~--·
. :'
. . - - . .

~~"'--

•, ,,,, ........ ,,·,.i


' :~ ,;-.:,:, ~-;,.,'

. '·,•,•.~,~-:
... ~·-~ ...
, ,
._,,,,_,,,,,' .

• i:r,_.~.....,,"_'l'1li't'Nl°'fiX'f.,~~tvi."f'.N11.-.••,...~ ....t ... - - .... _,,, • .-:... ... ~ - - . . . . . _ . . ,.....- . . , ,~,..,~"'"'·""• ......... ,, .:_,. ,;, ... ~t".''.-,,,~· " •.,.,_, ·..,., ,. .,.,.,.,_. ,_, . ,,. ,-,,:,•:,,._;,r=,.... _~,,.,, ..,.... ,,.,,..,.,,,...""""'""~"''''•'•'·•·""""'

I:!A 1 '; '· I :· l


I ·1 : ,

..-~ ~ 184 FWID'OYNAMICS so&Jdsk. s1w<'s ANO ooueLerrs (Mor10N IN iwo Dlr-lSNS!ONS) • 18,S'
C ~ · ,. 1 • -: , • •• • · ··
.--, .~
;
;):i
·i •3
The image of source -+1Hat2 30
, . ,
w.r.t, s•nxis· is n source:· +m
•'
at ·I I I ii!'::
I:
1 · i ':
1 ·_...!£,=m
,
d
dz
[ -·----+--,---,
1
%- C
l
+
l l ] ,wherec'=a
+ Z
.
C
\ 2le.
Z- C 2 C :
~0 ,wherezo ""o-tYo• .: i .i . ! :: : _
1,. I -
:\., }. . . . , 1 20 2 (a2/c)
.• (z2-a2}(a+a2/c)·
The complex: potential due to obieet ?YStem W'lth ng:!d boundaries is e'quivnlent m [ z"""i + 2 4 z.. ] = 2m 2.· 2 2 •2
i.I to the obJeet and Its imago system V\'llhout rlg-ld boUlldarles. Hr,i:icc w is ~vt:i by I
1 :

·'.'l
I i ,: · z ; c z - (a le J (z .;- c ) (z - c )
.· ·1 ;h · I i I' 2 (z - a)(z + o)
B . o•~
u,m-fn!og-(~~z)-m.lo"('-:o I I· ' . . '. =-. 2m· a +c
,. -
0 -. '' . ' ~4
or qi+ iiq"' - m log (z 3 - z5i'{z 3- zh . . . • Proved. i I i ! · · . ' .:. 2
(z - c) (z +c) (a - a lc) (z + ac )
~of~~ I, i~ j_•ng m.odl~s of both sides and noting that i\uid velocity"' j- !; I, we get
·,'

,(
:;; Pr~blem 16.A~ource Sandsi~h s ~ "? OJ!S:ituatedwithi~the space
1;.
,, • ..boun~edky·a~1r~le1.11hosecenfre1-Sd(lfSandTOJ"l!atequaldist~ncefromp~n
:· opposite s_ides_ ~f 1/ and ?n the. same dfGm.eter AOB, ehow Iha/ velccity of ;he liquid · 1
1
· OA2+os2 . . PA;PB .
.v;e ,o. = 2m • :os . . PS PT .. PS' pr,• .
'!
' ··
Proved,
}! at any poml P ts • 1 : · • .· t • 1 ' . . •. . '., • . · . · .._. •·
1
ii
(
. 2m · OS + OA
2

OS
2 /

PA. PB
' PS , PS' • PT, PT'
·
' '

' i
'

. --P~.obl~m
... ..,... ·
-- I •

7, Proue thal u."' - icy, u • - ()l:;, w = C 'represents a possible' moli.:m of


· uid. F.ind the streair, fun.ct ion ond. sketch streom lines, What kthe basic
• •

·l
~ •"" of Sand
where S' and T' are inverse point.$ . T w:r.t. the circle. ;,,iffer~nce ,betuie11'n·· this motion and one represented. .9:Ji' the 'potel'ltial
-~
· ' · 4 (Meen,/ UJ!JJ, Kanpur DO)
$ ..;A )dg.r,·r (,:~-.:h• 2) 112, .. , · =
•· i ·
Solu't.!on,Tn.ke O ns origin and OA u .t'•axls, .SoluUoo, 1,'0onslde'r:the motion defined by
Let "OS.,OT"c,OA=OB=a.:Then . 'I· iu=·-OlY,~'"~•~=0, .
·i ,,. \
Ev.idently.!t Is two dimensional motion.
l' I! ·, .au, + ,+
av 1 ow, =0 + Q + 0 "'0, ·
i
. I . v:c <1y vz
;'f~is declares.that the liquid motion is possible,
.' .· a
dp~dx+icly.
a
-m (-;m ! j, +m ) _· + rn x ..:£t.-u'.,-~
ax. - '-.oy''- ~"
ay u = 2.'ax·r
T' B f 0 S· A. ;S'
. , . dw = v clx -u dy_= wx dx + (!),Y dy =ci·[ ~ <'12 ·•
1
i\}
1

fig,Ua,
i 1:niegrating, Ill=~ (:c 2 +y2) +a An.s.
l "I: ' ' '
,Tlils gives th~ required stream fu.nction,.
' : I :Stream lines irtl given by \ji = const. =b, say, so that ' ·
OS • 0$',. a 2 , OT • GT' ""a 2• ' 2 . • 2 '(b - a) 2 , ·
Hence OS' "' or = a2tc. . . X
. + j'~ ": ·
·. ,., , " C 'lli X
I +y"'. :, C. . . ' ::•{
Tho object systlirc, coliislsts of 'It meo.ns thli.t stream lines are concentric circles with theircent:res at U...
' ·, .. ' ; • '...1. ' . ' '
(I) source·+ m at S (c, 0),
(ii) sink:- 1..n atT (- c, 0),.
;
I I, l ,IL Nex-t, we. consi!ler the·m!)tiop defined by
. . ' A
The lmage system consists of . $""~1ogrr,Jlog(.i;: 2 +y 2).
(l)' so'ur,ce + m At 8' (a 2/c, 0) and ;ink-mat 0, . i· it· Ax . ~ ,;:. Ap,2-.,2)
·Tli!s:11 : : - - = > . . ..
(II)' sink - m nt T (- a 2tc; 0) and 1i'ourc~ -t m at 0.
1
I•,.: t:e _..2+.)'2: o,:2 (.t'2,,ty2)2 ..
Source and sink both at O cancel :ea9h other, \•
,,,) 1'72 qi,;~+·~= A {y2 ;.;,:c2) +A (:?-y) "C I
i
·H0nco· , . . . ai:2 . izy2 · ·(:? + :l>~
w J_ m log (z - c) + m log (z + c).:. m. log (t - a%)_+ m. log (z + a2tc)). ! 'H' ~ l'
·... 7n-..e . I m.0,,1.on
1qu1c. •• • ~st'bl.e.
1s 1.·
\

·,·,:·· .

... ,,.-..._,, .... ·.·


,.{).:,;.<~::--;.~·•~.J-";',';':,'.<!
~:-::.: ..

.,

...... " .. """ \


! :\; \
-iiif'i;*f l'LJ1d'b1N!M1cs '
SOURCES, SINKS /\ND oousurrs (Mlbr1ON 1N -rwo 01M1=NSIONSJ 18"/'
Il).. Di~~r-ence. Th~
b~sic difI'e~~~ce in the~e two rcotloos is il~tl ielocity
c) 2 + y2 J - 2 (x + a) 2 _ [(x :- a)2 + y2J - 2 (x - -~)2 ·
· potential dqe;s_0ot exist in ~he firs_t case whoi-~aa,in the second ~.a•se.it eti~tsl ~$ ·_ {(x +
c1:,:2 - l(x.; a)2 + y2J2 · l(x _ a)2.,. y2J2 •'.
~,e-m lS..A two dimensional (lo~ field ls giuen. by w xy. Show th~~ iliel/Z~w1is
irri:>tation.al. Find ueloclly potential, stream lines. ·, · (Ga !iwal 201'.k))
= 2
.a (,, /-(x+a) 2 _ y2- (x-a) 2
or .. , (2)
Solution, ljl = xy. 1 ;: ilx 2 f(x' + a) 2 + y2J 2 [(x - a) 2 +y 2]2
• ~ . 3,,, By .o$ _ Y _Y • i
1,1 =- ;i., = - X, U = .::.;x; =y
Vf , ,~~X i!y (x+a) 2 +y2 (x .. a) 2 +y2
q = ui + vj = - xi+ yj. I> iJ 2,p = (x + a) 2 +~ 2 - 2~ _ (x - a) 2 .;,y2- 2y2 _.
'" (3)
curlq = j l j k a_,,2 [(x + a)2 + y2J2 [(x _ a)2 + y2.J2
·. a a a I
Ad,ling (2) nnd (3), ~- + ~ = O or· v 2,p = O.
f
a; .t _a;
l-x
= i (0) - J (0) + k (0) = 0
y 0
id
' i ! motion;;
,
· ,
a:r:c ay
,Thus ~he equation of continuity is satisfied and so (1) gives
.· •
apossible liquid
Motion is irrotat.ional.
i Se~ond Part. To determine.stream lines. ,
(ii) = 23! dx +~ cl" = - u ·dx - u dy I · a ·· a . a a ,
= u =- ~- - £.2 "' u - ~ . '
'ox qy· • ' I - £.2
OX ay' •. ay - ax'
=:r: dx-y dJJ.=M d.x +N'dy, say.
Hen ca ~ ., Qyt. ~ = ~ £11.
PM .. o= m· ox ay• ay. 11x
' <)y _ex N ! i¾r x+a· x-a
M_dx +Nd:; is exact. Solutio11- Iii ow ay"' (x+a)2+y2- (:i:-a)2+y2

f f f~d,P = x dx + Y dy = x2 ?z + c
: Integrating w.r.t. y,
IJI."' tan·l' __:t_-tan· 1 __:t_ +F(x) ,.,
x+a x-a
... ('1)

or
;,12-y2 where F(x) is constant of integration, To determine F(x).
¢i = 2 +c 'Ans.
ilw di!>·
This is the expression for velocity dX " - 3y ,:; - (X + a)2 + y2 + -(X---a~)-::-2-+-:,,""2 .. ' (5)

Stream lines are given by


By(~), .2.'.I!." - . + _ __.,..___, + F'' (x) . " (6)
VJ= cons~. .l;lut · . ljl:xy : ox + a)2 + y2 (x - a)2 .,. y2
(x
xy =c Equ,ating (5) to (6), F' '(x)"' 0. lntegrat!rig this f
gives stream lines. :Ahs. F'(x)!= absolute ·const. and henca neglected.
'Problem 19. Show that uelodty potential j lI Sin~e j~ has no effect on·the nuid motlon.
2 NoW: (4) becomes
(x + a) 2 + y ]
"' -1 log [ =-'--"''--'-"'- , . !. 1i
2 (x-a) 2 +:,,2 . ! ·•:; ljl "tan- 1 ~L-lan· 1 - L ... (7)
i l x .. a ;,: -a
-t • ; :
• I,
giues a possible rnolion.. Deter.mine lhe for:,m of strgam lines and the curuef q( dqual, -1 - 2ay
"tan 2 a 2
speed. ' i x -a. ,+Y
· ·
SolutioD, Given, o ~ 2l log [(x + <>).2 21.
+ y I -. log l(x -
2
22
a) .~ y J. .\. (1) S!;M;a.m. l.ioes ·are given by, \jl " cimsl. ,. i.e.,,
2a~(1:,,·· :.. ;, :,,·~·
: •.:•. ·
5,-:-:•.'<· . :. \;
~- ' \ X +a ·(i''"'a):··,/~l·. ,.i,f,,•:' wi- [
l .
~
-
],. const. or
:.
· = const.· •....
x 2 - a.2 +y 2
- a2 + y 2 ·
· ax :: (x + a)2 + yZ - (;t _ a)2 +y2 . lfwe take coost = 0, then we gety = 0, i.e .. X•tjis.
' lfwe take cont:s. = ""• thon wee-et drr.lP.·x 2 -n.2 ·+ v2 :: n

:~!7', :, . '• "'•" ""'.'.':~•t::''. ••~;•,"r'S

:,-:::'.•;; ' ·:~. . !::,~:· ~:·:


-:,:,,•1,;·:\•· -. ...,:,!,.;''.,: :,<;'.,';;~'
·;,,i•,•,;. ..
- ;:,,;,;;··
,'',\!,.':','
,•.•.-'•·
',/

11i
~ • • ••· .. •••·•••••-••·••••·•·••••-•••---•,.••••••--•"""""""•;,,<;,~,;l!;I'.•'.!(,''.¥·, ''.' ,:•:.cl!~J:,,,:.,•1:l,~,,,:,.,.,.,1•"•:·'""'~;,;,,,1,,,.,,,.,,:.,1,,J.s~,,,,,..,__,;,-•1r-~;:,,...U:,"">'-"l'C,'.•!,'l,•>•,;;c,;_,'.:.".::.!:'~~..- ·..••~~=-~!N,lffll(~~Ua>;':""""•"•:~-"'"
,,~
~ 188 FLUID DYNAMICS
s6ui:i:::e-s. SINKS ANOO.OUBLE7S_("!OTION IN iV'/0 DIMENSIONS) I 1 ~9
:1
~ l! , - . · 82
x2 +:l., a·2,
I

II . w = - I m log e( 1 + 2 2 )·
l,e., 11
' I\ 1 II ' n j't'""
Thus str~am lines ioclude ;r; nx{s and circle. A.ns,
Third Pnrt. To determine, curves of equal
l , speed.
. Ol'
: . .
\w=-mloge. l1-·~
( 132 )(
i+
92 )(
y 1+ 3~i.
92 )
By (1) an,d (7.), wo obt.llio . 2
w" ~ -t- iljl,. ½log f(x +a)2 -t- / ) -1 log [(x - a,Z -t- .lj = - m k•g sinh e = - i;i tog s!nh ( 1;:.:)
or· : w = - m log slnh (r.zla), ... (l)
+ i tao" 1 ....):_ - i tan· 1
x+a x-? ''l?hi; prov-es the first required result.
= log ((x + a},-t-'iy) - log [(x - a)+ iyJ j }f 2,t~.Ihat,w = - m (z - a) due to sourc.e;t.m at z = a and w., ml(z - a) d\Je to
= log (d+ a)-·,J~g (z.- a} . . . . ·. . .d
d~ufl~ft mat z = "_'~it~ its axis along.;.ii.x.is. i.e. w =:J; (m log (z - all for a doublet
dw l . -2<%
ru ., z + a - z .... a:= ~ + O) f:~ 11tzi=a'with it1Huis along.i:-~s. . . ,
.
I7h I'"'l"!~.,,.al,lz+a!
: iTJie.r-efore the complex potential for,t}'ie doublets of strength m .aUhese J:)Oints
dw ·2a ·· is.negatfve denvaij:ye of(l),.so, that . · · ··· ·'
'
Write ) z - a I " r, Iz + a I "' r'. Then speed "'r?• ·
2d

• 1
i \•: ! • !
w
mn,t· ( nz )
~ !i!~·loifslnh (m/a)l.
· ( 1tZ )
'fhe curves otoqunl speed are ll'!vec by . i.e.,: . w = - " 90th .- ,. µ coth - ,.(
2a . , ; C. ·., , ' •· i .•· ·, a.:.:; a. . .a
;:;; -= const., i.e., rr " const. whlch are ass1ni ovals. 1T]:iis proves the .se'i:olid requlr~d result.. . .
·i ; · · •·· Mlscellaneous Problems
t
Problem 20, Parallel line S'OU~Ce3 (p~rpendlcular to.the ,:yplpne) ofequ~l strength
rn ari!placed at the points z., rtta, where n = .. .,- 2, -1, 01 1, 2, 3, ... ,prove /hat the Pro;hleip. 21, An 'area~ ls' bou.nded by that part·of the :r:,a:r:is for w~:'ch x > a nnd
complex potential is . · · : lhci/ brcinch of x 2 -y 2'= a 2 which is in the positive quadrant. There is a two
~ w = - m tog sinh (ttz/a). · J ditA'.e,isional unit sink af (a, 0) which sends out liquid uniformly in all directioM. (
H~nce show lhat the complex. potel'llial for lwo dimensiqllCII doublets (line doublets), Shou.: :by .mecuis C?f the' t~a,:,,sformatio11 W clog (zi - a 2) that. in steadj motwn the
with Aheir a,.,;es parallel to the x•a:tis, ·of-strengthµ at 'the same points; ls giuen by strebrri; lines of the liquid within the area A are portions of rectangular /iyp'froolru. l
· • w = µ colh (r..i/tl.), i • · · (Y,.(P"a 2000) Dfa;w ,t~e . stream Un.es corresponding to IV = 0, 111~, and 1112. It p1 and P2 a.re tJu.
Solutlol'.1, Sources of oqual strength m. i;ire placed, nt z .. ± ~ia where di;stp.:ipe~ of a. point P within the {1.uid from the points (:ta, 0), show th.at th.e. LleLodry ~
n., 0, l, 2, 3, ... Tho.co~~lex p:tent~~l due to t~s s::stem ~t any j:,ointz s given by of: the tJr.iid al Pis measured by 2.P0lp 1p2, 0 being the origin, (G<u-hu><d ~ }
1 il
sa16t1on. Seep I. w = log (z 2 - a 2) is ex'preselble as
w,. - n1 log (i; - 0) - l: • m log(::,; - nia) - :!: m Jog (z + n(a) . '. i !: i ¢, + iljl .. log ('112 -y2 - a2 -t- 2b:y) '
., : I)• l 11"'1:l ' .
,
I
1
, . ;1,,,. ~

21
~ ~ 1
., - m log ;i- - · I: m log:(z -t- n 2a 2 ) j ':'~i? => 111 = tan· 1 ( 2 2
; I • 1 • · x - :; ·- a f .
n• 1 i
; ~trqam lioeii are given byljl., con.st, .:.~, say, th:en
, ·H
"' - I: m log
1
(
t• + L2 . 2,!~·-'.n. 2a2 . z . ·. .., ,' I . ,,ac -1 ( 2xy )·="<
ll•l

,. - 1:
-m ·.
log ( l. +
/la

r~
} !!:. - Z m log ( n2a2 E. )· .
~
or
·· . .Jl,-y2_ 0 2 .
tan k = 2xyl(x2 - y 2 L a 2) •
11«1 •nc 2 a " ~ l 11
=
!fh 0, the~ (2) ~ 2xy = 0 ~x = O,y = 0, .
Noglectinrr co11st.ant, w,. - i. m log :IE. ( 1 +I -{0
· n~ l · · a
- 2
= x
lf k '112, th;m (2) ~ 2 -i- ~2 ,,;_0 ~.,,?.;. :/· = a2.
0

ti""n"
Tlw,i stm,ih !in~ ~rep.arts ofth~ ~~~~;,:t'.::. i 1 ..,c:2.::::.,,
1 (1 !;,.·
Putting •r~• ~;;,~Ve /nan ar~a A bounded byi- "'0,:,,. O,.:.:~··'.i' , . , b.Uia

1!11'~t,l'!~}~"'&T,;:.!T.'-•~i!.!ll!!l.n!!l1Wmr.!1ffl!Nl\!l,11!!1!!;1!;(f:C:'ll~'':':''.\f:!·~,ll'!!!ll'ffl!U1f<li*•'''r,o'1:>.,·,1t;,H/.llOIH<llf1R\im?g!ll}Wl!ii\!!!!!!'''.'•'ill''':'>l''''- ·

,.•,~i~•.v .... ,,.~. ·


. .. .- ,.,.., '.,,,,
·, ~
..
,,.,.::,: :,:,\.

,,

...... ~ ... .... ...


~ , ~,

' -. " .. • . !
SOURC!!S, SINKS ANO·OQU6LE'TS (MOTION IN TWO OIM5NS10NS) .I -191
log,(rl - aZ) i~ expr~ssible as·
.., a

"' .. l.og;(z - a)+ log(~+ a}. . · · , i


!
Btt u=-2-~u=~
iJy' iJx
;I
. · ~ tb.ai th<i liqu.id ~otion :is generated by two s'inks I
NI w. the last gives ·. v dx - u dy = d,'
pd(-a, 0).'Consequently, the irnag~ of sink - 1 at (a, 0) is
remth-8 to~axis i.e., relative to the area A. or r f!l=.!!.
I d.x i;
Sui, ID. To.show th'at velocity, q = 2. OP(p 1p2 slope fl'li or the tangent to th9 cuiye (1) is in1"' Y,fu,
w ,;. l~g (z 2 .:.. a 2),
·Waha",IV

flence
aw '2.;
.\, B~t slope of direction of velocity q is t• 1

.7!;. .. z2.,. a2'. . Consequently, direction of velocity is tangent tow"' dlinst;,


Tb.is =:> •=j!i!£.j., 2!z!· Problem 23, A velocity field is giuc11; by q"' - xi +(:;+I) j. F:ind the #reamfunctlon


q dz I z. - a I . I z+ a I ' and steam line$ for this field at t = 2. /
U.t P be a pojnt within the fluid. Then lz'I = lz - 0 I = OP, Solution. q = ui + uj_= -xi +j (:c +t) • '

, ) p1 = lz:.. a I = distance between P and (a, 0),


P2"' lz + a Idistance between P and (-a, 0)
Thus q zZOP'
-., ·,
I:'
!
i
I
I,
But
'
. u.=-:c
u=-~
a .

v=y+t
1.1=~
~
~

,
,, . Pl J:?2, ay =x • =y--·t
..
~
$tep rv,:, To determine stream lines corraspondin{to !
'1t 1t
::!:! IJ dljl = = dx + .;;..,:..
'ax
<)111
dy
·il111" C

ljl = o, 4i 2' . I
Q,)'
i' rJw = (y t t)dx +r dy
f
,
' '2xy iI , = Mdx + Nd}, say .
By (1), tan ljl = - 2- -
:c -y 2 -a 2 iJM _ 1 _ oN

, P '
utt1ng \jl ~ 01t1t
, b'
4, 2, we o tarn
ay- -ax
Mdx + Ndy is exact.

, . ~
-2- 2 2 "' tan
:c--:1:--a
0 0
= ' 2
x-y_-a~
2:iy
2 ~ " tan 7
.. ii . 1

= ,
Solution of(l) is given by
1
\It =J (y + t\ dx + f 0 dy ,. x (y + 0 + a I
'=""
•; i.e.
2.xy
x2-yZ-aZ
xy = O; x 2 -y2- a 2 = 2.xy; x 2 -y2- a 2'= 0
or lj/ "'x (y +I)+ c ... (2) AJJJ,
This is lhe required e~pression fontream fuoction, Stream llnes at I= .2 are
i.e. x" Oy = O·: .i::'2 - y 2 - 2xy - a
2
= O; x 2 - ::l = a2• given ~y (W)1 = 2,., const. f ... ·
g Thus stream lines Jie·aJonl:l' ~ .. i or :Ji (y + 2) + c = const,
or (y + 2) = a, A.us,
tp
,
X
X and y-,.,;es ' rei.\itiyo \jl "'p
f thecurvex 2 -y 2 -2xv-a 2 =0 releJhieJo~=n/4
Problem 24. Proue that In tha two di'meniional /iqui.d. mollon due iJ any number of
,., (ii) ., 1 ' . I sources at different points on a circle, the circle /.$ a stream line pro1n'ded that there
relatjve!~ ~ = !l/2,
'
(iii) ractangiilar hyperbola .i: 2 -y 2 = a 2 is no b~undat and that the algebraic s,um of strengths of the sources is zero,
' \ . 1 l i- I !i
Problem 22, Show that the velocity vector q is euerywhere tangent lei i?ie lines /11 Show (hc,t th~ same is true if the region of/low is bounded by a circle in wMch cuts

' ~
xy,plane along which ljl (x, y) = const. ·
Solution. Given ljl (x,y)
· (l) ~ dw ;. o ~ ~ dx + ~
·

a'
· I ·! '
6 _
1
·a=.~9,n~y;,
· ... (1) t ,.
,· . .;,,d-,,>·~•·1'·',.•c•;•·::
dy .;, 0 . . .
.,.
."•
.
",:J<,
orthog~nally the circle in qu~slion. ,
·
ml, ni~1 m
3
, ... ·
. ..• . . . . . (Xa,i.pur 1991),
Solution. SupposeAi,A. 2,A 3 .• ,_•. !)J'e the po§Jt,;i.ons,.o.qp~;sources of strengths
,. , :·:~:"••·..:·:,i.,, ~ ""· ', ·,·:· '-:(::~·=·· · ,..,, •. -...~_.. •r;1·· , · ......
..
•-
.,, ...,,,,,.,,,
: ·m:1~('.1~~-·,
dX· i)y

•• .·..,·;'.·····: ... 1:'/ • ••••• :~:· ::'>·' ·· :~~.mttr~~~1~:~~{t~~n.,,~,.,ttft!)ffRfl~Y!~'I~·~;·1:r~m;-;:.~:.·:w.;I'!: ·r.. :·, ·


Tako any point P on the circle and the diameter through It a1:i' the in.Jtial J!ne.

·•·•··~•,.,,,,:,,!f••>.,~,1]A~?f~!ff::!~~"'1<1~~~~4K~•l<!""••••··~•:•u-'1•~1••M"""'••••:~·~"~":".t:;~t;:":·~"" ......... ~--i~•~v"'l'.-,H~i'C~Hh)l\'li~~\i(

.,,;,.•.;.'. •,·.;·,;,: . •;,, ·,·.·. .••::::•;:: ' . ~ ...:,:: ': >:~:l•::.~:.,;.,, -1·:

\
,;w•••, <

,:,:,. !;\(•~•.•

. ' . 1
,,-.,,t.4"1:fftflllNfr)ff'l.1'11111Mtlt'.\'"7""""~1H:\H~-----..... - ......... ,.;.!J.uJ.41}1.':i1t!llt.nl~'Z:....:-'.. ~~·~~'~.4-,.,:..;.,~.;MA1.l ..... .. ,~ ... ,,'~},; •.... w.,...: ... ~.~-;.,,,1~,-'t)'.l ... .ttiY;'... ~1-.IM-'l't-flld.U:<"'U'~Ci·•llii,;.$,l,~.i•.:'.~:.:.:.:1.:.:.::.:..:.:s.:..:.:.:..J1.:i&."'~~~-· ... ....;,_•i.,.cJ4'.n..;,,l>.r,11Jft){•t:t~HtO!~tnfto .. ,•h•l ,it.,11, ....:.

'': i ·•, 1· t ;< t


i ;I ;
FI.\JID DYNAMICS I I j ·
192 SOl/flQ_~_,__sl_tl_KJ!<Nl::i DOUBL~~it,1__9-T_LONJ):l iWQ t>llASl'/SIQNS) 193
.... ,, ...

.
IA I i .• I
!At -'A,PA=e·, LAiPAi..,a 1, ~A2 I· : ! ·
: Smee . OA . OA' = 013 . OB'" a . · · 2
..
k'e~ce poi.nt.sA;-A', e: 13' ~re coocyclie. Let lhe eircle thtough these points meet
...,--·'\: . l
L A3 PAz" ci. 2, ... etd. 1 · . i I
.
Then strooru lln e ls given 6):'
'
· l'f"'<i"'"'= , 1y IA ..the!cilvi'ty in-C and C;, Then OA OA' =002. ~nee oc is tangen(at C to the circl~
\j/ =- m 18 - m2 (9.,. o:1) -m3 (64- Cl1,+ ci.z) •;• thdnig#, B, B', A, A',, It declares-the fact{hat the tw9 circles intersect.orthogonal!)~
A:lsb the-cir<:le tl::.rough A, A';B, B' passes through.A' and 8; I.e., the same source and
.. ..: (ml'+ m 2 + m 3 + .. ,) e,..; [lii 2 o: 1 + m3 (0:1 ~ Cf.:2) + ... ) sinA, ~ce lt must be a sye?,m line. ·
or \jl" -:·&I:+· -
const. . . I '. .. ' . ~ . F/g. 3.Zl), P,~bl e;,,
1
U.,Prou~ that for liquid circulcz/ing irr;tatio11a.lly in part 'af·.the p/an,I.
For ..whate,;,er be the posltioti' qf P, a 1 , c::L;i, , .. elc, do not · · 1· ·
betw~ir'-~'.two non-intersecting circ/e3 the curves offonsfont uelocity are ca·ssini's ouals,
ohaoge;·
.
Since the o.ng\e subtended

·at• the circu.mfer.ence by llfl. ar:c is always the sa' I e. , : Sbl,utlon. Suppose CC' is the line of ce~t;.res,. Take two points A und B s.t. th~y
l~ the•algebr~c sumo~ the s~ngths is uiro, i.e .• i,f Im= 0, then ir = codj;t.. ate:ioverstl points w.r.t. both the circles, Consider a point Pon one of,the circles.
meamng thereby mete ls a Meam lli'le: Hence \he ~t... . , l 1· ' : Write PA = r, PC~ r 1,
Seoond Part. ·Let O' be the :C1:lntre or a · 1! ( •

circle whlch•cut.s the given ~1e ortbogunally. i , ~i~ce CA, CB=:. CP 2, hence · ·
JolnO'loAj, UO'A 1 cuts the original ntA'ithen t:.CP.A, .abd t:.CPB are similar so
A' Is the Inverse point ori\ 1 w,tt. circle wh~e
,· ' I ,,
!Jia~ :; j . ·..•
; QP. .,. 1PA . CP,.: r ·• r
centre 1.s O. ., , · .W. 1';' PB' 1.e., cJi•~ ;- or ;- = con st..
RelaUve to the circle whose centre is er., IA I : ;. . ',<, . _l l
,:'the image C/1 source + mi at A1 ls sour,~e a jU • : 1 I
I, j I . •
.... ~•:
- . •

. 1- m 1 atA'.,,Md slrik -m 1 at O'. lithe barriers" i . ri mil MS the equations or the two
are aOlitted, wo are 1~n wllh system 2 rm on cltcle~!,can be written as Fig, S,32, 'I

the orig{oa.l circle 11nd - Dn at O' and as · 'Fig. s.so, L "'k 1, J:.. = kz, say.
!m .. o. we again get tho. urne result,\ •I r1 r2 .
i,e,, $,. ,CO!\/lt, ! Alb6 these two ~lrclea a.re strea~ line~, henc.e IV must be of tlte form

I
~
Problem 26. Find the velocily·polential when. tlie~e Is a ·source and ,u1 equal si'nk
- ,\V='f(rlr1), bu·t f(r/r 1) ls ·plane harmonic: Co.osaquantly
'¥ = f(r,'r 1) = A log (r/r1) as log r )s the only functi'on o.f r which l~. plane hiu-rnonic.
'
iMide a clrculc.r cauily and shdw that one oflhe stream.lines is an arc:ofthe circte Hid~:' . ' .
. which passes through the sourc~ and sink 1.Znd culs qrlhogonall;y the boundary · • ' •:. · · · ·1 a.- a'
'cavity._ . '·. , .
Solution. Consider a source + in at
. ' ·rI(,mp1tr
$=-A (0 - 0 1) as -
r
=
aa = - ~ c)r
A a.rid ulrift:- mat B respectively ilside an w = $ + i',1 = -A (9'- 61) + iA log (r/r 1}
cirNll\r ~s.vity whos11 centre is O and radius
ls a. Let OA.,:. b·, OB= c, LBOA,. a. Let ' ! = i A (log (rlr 1) + i (8 - El1)l
'. I.
A' Md B', bnaspo,.eUvoly Inverse points .of . ..-:;;: : t: 0I ; A' • l • ' i8
A and D respoctlvely. Then : l
!. , . r1
.. i A' log['.!:.. e' te • 0 ,l],. i A log [
\' ri·e'{i
!:!...,.. ]
OA '· OA'"' a 2 " OB.• OB' / I '

The Image of source t m at!A is a sink


,' jCho?sing A to be(:.. a, 0) and B to be (a, 0), then
+mat A' and a sink- rn at 0. The hnage of
................... ,._ .... ,,"
Fl'g. 3,31 w " 1.A log (Z.-.tCl)
- , =>
. 'This
•sink- m atB is a .sink- m atB' and i . so·urce z-a
+ m at 0, The source+ m and sink-! m both
at O Ca!)Cereach other. Thus w Is given by
. . . . 2
'. I I · I
dw
9 ::: -d_z --! iA l, -
1: j 2AD
z+a- z_-a -- !i;t'al. t.r~aJ
0
,w=-mlog(z-b)-mlog(z- 1;, )+mlog(z"".c•e1a)+m:!og( z- e1") or 2Aa
q = -..
: , rr 1
Equating real a.ad Imaginary parts from both sidas, we can easily get velocity C~rsicl, of constan~ velocicy an, ~ \ v
Il,lot.uit'!al and strenm fuMti'on, r'esp~ctivoly. '
,,.,.,.".,"•-·

,,.,,, .. ....,..,
_

...............
. 1 t ... """ ""
; ! ( .•

!!:4_::~~•*1/:'' 1 ~LU(C~bv~,t1cs . SOURCES, SINKS ANO. OOVSL&:T~MO'T~ON lt,41V,1,i) OIMENSION!:1, \gl{.


I
· q., oonst., l.e., .&i.S.
rr 1
.. co~·st.
•.
i.;1,, i .1 \-eJocity;potential = q, mAr2 cos ( i + 28) .. -Ai- 2 sin 20,
n- 1 ,;, const. which are clearly CMsini's ovals; .
or 11)" _ ::fil·,.z
2
sin 20
P:roblem i1 .If a homogenaous liquid is 'apted on bja. repulsive force from the 'origin, ·-
l.he magnitude of which at distance r from the origin is µr pre unit masJ, s'iidw that I· '\I"' ,2-?rQolem 28, If the fluid (ills ·the rettion.of 8pace on the positive side ofx•a.xi8,
ii Cs possible {o"r the liquid to move steadily, without being constraihed b;y any -' /{: , ' a rigid l:;oun.dary, qtld if th era be a source* rT?, yi
boundarie!I, in the •p.ace betwe'ln one bro11Ch 0 j the hyperbola: :,:2 _ ;y2 ,.\ 0 2,·dnd the . \~--,::..{ ~t the point (Oja) and ~n equal sl,nk at,(O, b), and . • rn ·-13 (0, b) '.
asymptotes and find the velocil:tpoten#,Jf. ' · · . ; . : . •~ri if the prusi.t-e O(l the negatwe_ side of /~e I
m A{O,a)
S~olu~lon. The liquid moves s.teadlly'between the_spai:e given by o~e .b,r~nch of •·'i I bo~nda:,>1 be the same os the pressure of the 1uid
x2 _ y· = a2 • .,(l) and its asymptotes givell by , .• • , .. • ·, . I at infinity, show that theresullantpressure on, the • ,
2 2 ·o · · · , ,1 bo1mda.l':)' is 1tpm 2 (a,- b)2iab (a+ b), wher, p· is·
' X -y " , ·, , j ';' (2) ' tM deM(ty of the /7.1.JicJ,
(1)and (2) are clearly stream Jines; For x2 -y2 is a harmoni.c function a~ it ~,ti~fies . So!ut:ion. The obj~t system consists of ~ / X

Laplace's equation. Thus , · , · ·. . . .' . : ;: I , · source ti' m _at A (0, ~), l:e:, at z = i.a. ~~: sink 0
_ z_ 2,'- 2. i _. z' _ 2 · ·. I !I 1· · -m at1z=.1b. The image system COIJ.Sists•of
'V_-A(x Y )_-Ar (cos 8 6)-Ar cos2S · , :I I, srn source 1+m at A'(z=-ia) and sink-mat,
or =Ar 2 sin ( ri
w_. + 29 ). A beillg a ~on~~a.:it. \ :\.: 1 : ~' (z = +ib) w.r.t. the .positive line;O~ which is
1

" ( 2 • · , · 1, i i n~d b~u.ndary. The complex pp~llt:ial due t.o '. m!A'(O,-•J
~. l £.\I!. . ( rt •. '11 I obJect syst.em w!t.h rigid bou.adar.; !,s eqt:.!vaJent
Using ar =;: as, w.e get 4' "'Ar 2
cos 2 + 28 J .: 1 i to the object system and •It.I! 1.n:ui:ge system with • m· B' (0, - i)
• ' no rigid boundary ·
w=.P+lw=Ai-2[ cos(i+2e)+lsinrntze))
·.. . ··:, ·
I .. :
w~-mlog(z-~)+mlog(z-ib)
-mlog(z+ia)+mlog(z+ib)
Fli;,3:s:i_.,,·

or w. = Ar2 e1 l(lt/2) 281 = Ar2 ·e' 28 = A (re 18) 2


· •
or ·•· ' w = - 1.n log (z 2 + a 2 ) + m. log (z 2 ·+ b•2)
For el(l!/2) .. 1, 1
dw :o:: - 2niz ·[ -l- _ _L..J 2mz (a 2 - b2 )
1 ] "-=""""--:-!"----,,..'......,-=--
Hence w =A.% 2 , · 1-lence q =
. Idd~ .I= M. Jz I = 2Ar -
·dz z2 + aZ z2 + 0 2 . (z2 + a'Z) (z2 + 02)_

I~ c!is~ ~f steady motion, the equal.ion of moti~n is


- dz
q - / S!!!_ I.,
2m. (a - b ) jz I
Jz2+a21 lzz+b21'
2 2

E. •½l + O= const. .... (1) Fo~ any point on x-axis, we hµvc z = X so that
I ,/ q - 2nh (a- 2 - b - 2)
Given - ~n =µr,
or •
[F = - v'.QJ i + a2)(x2 + b2)·
- (x2
This is expression Cor velocity at any poinfoJ x•axls, Letp 0 be the pressule at
This=> Q =. - ~ r 2 , negjecting constant. :,: ..... Br Bernoulli's equation fO\Steedy motion.
P:Utting the values-In(~). ;+ ½q2 = C.
2. + 2Ai
p . . 2,,
r2 -
H.,.z const;
.
= l n view of p = Po, q = 0 whell x = .. , we get C" Pi/P,
Subj11cting !.hist~, the condlt!onp ·.1, const. on f~ee surface, we Po:-' P l o
-p-=2q".
get 2 r = O . A., ✓µ12,
2A2 r2 - l! 2 or Required pro.saure Pon ~9un~ary is givoo by
\ E.::fu ' . - ,.. . .l
Hen<:o. q_= 2Ar = 2 . r Pc. r <Po:.pfr.1.-t=:f·-~·:½rxldx
.,,__ --..
~''!"'\~:+~~~,~~~~~':~-~ . +-~~~;~.;!'.~1;)·rn~ft~~t~!nmimmm~~;rr.~::-;~:r:~~~~r;?:i:~x-~::.~:·;;:.:.-~•::~]-:t:~:::::-·•"'"'"~~'.' ~:e'?!t~:t1mHmmH1ttoffl1~lff,r,?~~~:-:,:.~-~~•::·~~,m:--'!~~l'-~,~1t'nMrn~""-~H1
1

.,
l
.~

:{ .::::/·:>: ,: ::::,r;•~t•:~~~~f~ .~•:•:t~:;•:~


'.·'!;!'I"··:, ~: ;:_ '.~;;:I . ,; ,;'.::;~,::· .: ; . ::· -~.:: ,,;.- !·

. '·-·-···----·· •. 'f"r'1-,-;-r:""t":"t,"'!';'ll'TT7•',,~7';"\1\;?,'

'::;:::::·i~:}·
.
,:.,.1:,:>-,Jr, ,•,·J••·
' •, ,\';\1::•.•, •<
. ! •: .,,:, '·• •::·>:-:
:i
.r,,;.; :;~;:~:;;;.~/:•;

~ I
tj
·~ ,/ ' l: ' '
, • ·~•-•••·•• u,•.,•- ...,., • • - • - - - • ~ . _ , , , _,,:.....:.-~~-.';..:C.!ul:.fl!.1.U!Jt.",',. • ..,.;;.,.i;,;~:; ..•,1.0:iil;,l:.,:r,,\:, l~u!/1•~:k1,•i'INl'l~,>;'t",,:,:,,,•11,•,•~•,;uttt\l"l,'l1,\l.i,,I,'- ,,.;,.,:..,:1.;,,.:...;,:.::;.c~ .. •..t.~:.!t,!~'IIU,~~t,t~!;1~1;D,11i1UU.t:1:.' ';~1;,'-4.,..::..._,:...

' .
IUt f tH~r.:'.t~:111:T:UIJt.-.,•~•1~,-.~f.i't\' '•••• •o~~trtl :1.-.1' 11,,,,,,,.,.,~,,•,,.

j
196 FLUID 0\'NAMICS squ~ci;:s, SINKS ANO OOUSLETS (MOTION IN 'l'NO OIMeJ\JSIONS) . · 197
-~ 11 .
'\:
=. -1 p
2
f.. _ .. (x2
4m2 x2
...:.;;.;;...;......{a2 -
"----:;...J-..dx
+ ,::2)2 (x2 +:b2)l
·/H'·
· .
t/ . '
u.(,
:~qr~K.E,"'2Jo
··1r
M'

9m.? "2
2
00,


o <J.npqyq dy,

. 2 2 2 2 r- I x2 J, i,
i, ;
! , I
. . YI .· .
· • 4pm (a - b ). J ('<~ + !,.2)2 {x!- + 0 2).2 i,"

-
fL.!..£.. _._l_ .. _1_
2
0
2
! •

, . b~ 2
1
i

11
: "4m2 p
· J
0 [ 0 2~ 02(x*+b2 x2+ 0 2l
- a. ·- ,
(x2+a2)2 (x2+p2)2]
cix

!
,·2P[a+b(n
..• ,m -a2 _-
b2 -
.. . - -
Zb- -
·2a-
n -
2
n]
4<l -4b .
2 .'
it} ' I •

1•
,, ' lf
I

_npm 2 (a.-6) 2
1
, .. , J
i
!
A.t:,s. l
i. 'I

~
- ab (a + b) • ·1
· For .·f- --1kL,,[lt.an•l!r=...!!.
x2
o + a2 a . a .b 2a

(x
r1:.c
2 "72
+a )
=3
a
1
jr- cos
,
2 ·
8 d8, .i; = a tan e
·
O
rt!ll . ' .,.
Plane
.
,
.....-
f
)Po
l
3
(l +cos 2tl)d6 =
., n l.· n
2 , -3 = -3 .
2a4a
I :
ti
F)g. 3,34.
I •
ji
•.,:

Problem ~ n infinita mass ofliqtt~d,is f'!10tling irrofati;),u,,lly and stea¥ly under


the influence of a source of nrengtli µ and.an e.q;ual 9ink at a dt:ston,ce 2a from it.
;.= rtp J'"[
, o (a
2
2µa2
+:,, )
31'2
"f
J Y dy "' Snp 1,._ a 33J- o
·.
.-.:t..Sf.:t.
2 2 e12
.(o. + Y )
j:!

Prove tl!at the Mnclic energy of the Uquid whi'ch passes ir, urJll lime a,yoss;llie p/ane : '
. j,;,., :le~ 3Jrl2·
. ~int,cos 6
.· '• ,I\
,,.

r
3 td/,puty,;atant
which bl~(.cts at rl{/ht ans/es in tie llhe.1~/'!-if!ll.the s~µrce·~'nd sink is ::4. np µ , p
j a: 0 . . ~
being. the density of the liquid. '. i . • , , . ·; 3
a
SolutioD, Consider sourc~ µi nt..4. (a, 0, 0) and'slnk - µ al B (- 0, 0) s,t., + a, I

1 ,.
;

a4
i '811
': :~[--cos
7
1 '7
t =-npµ8•
8
?a4 ~
, All= 2a, Consid~r a point P (0,y, 0) oil Y-axis, Consider a drcillar strip b~unded by j',J !• '
tho radius y and .Y + cry/Mass or the ll~uiil passing throtigh this strip ls ~
p (2cy, 6y) q... 2n-b qy .6y per unit time" om, say. 1 Pcoble~ 30. A single source is placed in ali in(l.l'l.lte perfectly elastic {iuid, whi.di ;.,. .,.··'
o.iso ,a,perfecl conductor of heat; show that if the motion be steady, the veloc.iiy t<r
of spherical symmetry-.
2
Recall thatr~v" ,Jonst, so that vb corj.st.lr is thEi equation of continuity in case
·
distance r from the source satisfies the equ.ution
-Ij
( v~l)fil!-~g
V dr r.
Hence velocity lit P due to soured at A=~ along AP l ll
• , AP . r =7m
2 ,
hence that ,v ev 14"·
. velocit.y at P due to sink atB = "iiJ a.Jong PB, I •,..

So.lution. Since the motion Is steady aod is due to a s::iogjc ~


flow:is:pJrely radial. Equation of motion is
= -tL-2 along PB . ' :., . d l '
AP .
. .. f!.9.d
t =F--Vp
. p .
AP .. pa, Let L PAO .. a. we h~vo · . .
Thcn·resulton\ velocity atP nlon'gAB d cl cl cl · · , ·. ·' .
·21-\ '"" 8 2µ cos 8 . 2pa di"' ~ + u ~• '-' "'"5;'. F "O et1 ~ ~~.fill
"' AP2 "' a2 +y2 : (a2 +y2)312 "q.

Wl.1:~.illl!T'.t•H~!W~l\•:;ft:Valll~MW.ff<l;!\ff1t\!:!.!~t:lfff~/'l:'..f!~\l!i'il~ISR!Wffil1',tl".i'~:-,;,!;~;;';?.~«llll~l'.•~IIIJ!li'IR!'l-'!1iM!:<!',};··:;,!,t•;~i,;,,.,1 •.c:•:,

,. -• .. ~~~~.'" . ' . . ... ",


;;H~;:;,~~it,;·!Jl"i!~;~f~~{bJ;( ,.,., ,,\''I,:. '.; •( ,,h!r~v,:-, 1'.\.~~Uii•~•·· ,,, , :,;-,1,:i,;.., n,· a;.:·,n ,u,.

•.. ,,..,,,,,,,,,,....,,M,t_,,,
'198
FLUID O'\'Nt,MICS SOURCES, SINKS ANO DOUBLETS (M0110N I N ~ - : ~ - . '
< I

il ) u = - ;l Vp,
iJ u a;:
Hence (. "iii+ (1)"" x + iy = c cos(¢+ i'!I) = x" e eo~ t ~h 'fi.~ • ~ d.ir~,
Eilminating ~' we get. ·
Motion _is steady, t~ = O,p ~ kp (Boyle's,law)
1
II :/ . I
i :' I
2 X 2 + 2 Z a: 1, •
ufil!...,_!dl?. I .I j cosh ¢1 c slnh¾, 1
a . 1
: • •

. fir p ilr' I ( .. (1) Stre_nro lines are i;iven by 'II ~.cocst. By.virtue ofth!a, (2) doci4,l'(!S t.hat ~
Motion has spherical symmetry a1i/.d hence· equ_ation of continuity '.I lines are confocal ellip8es •. Comparing (2).with'tpe equation. • :. ·
[ 2 '.:-\ ,
iln· l a '· .· , ' ! l f_1

Is =at +_,.2- .i)r


2 - (pu r ) = 0' · -f--c. +-j:!-,. 1, · we get c cosh
a+A b'+'- I
\jl ... ✓(a 2 + A), c sinh 111 =·-✓ (b 2 + A) ... (3)
.• ·

But 9ft = 0 as tho motion is 9teady, => c, (cosh 1V +. sin.h·lj,i) = ✓ (a 2 +A)+ '1(0 2 + ;.)

. rJ iJ .. a or ce"' ,- ✓ (c::2 + A) + ✓(b 2 + A) . ·


Hen,:e f, 2
,,r .(p ur ) =· 0 or ur2 r +··pr a.ur + pu . 2r = 0
2 _!!Jl.a
!
or . 'V" log [(a 2 +;I;)+ ✓(1>2' + A)J..; lqg c • , , •., .(4)
· Irw = ~ + iwis the.comple;< potenti.a.l oiso~e.fluid motion, then so !sAw. Hence
nr i)n
u ':::"- + p - +·2
er
CU
"U •
=. 0.
ar =r ; ~ .. (2) (4) g1vea · ' ' .

Elimina,tingt fr'o~ (ll and (2), wo get


(✓(a 2 +A)+ ✓(b 2 + i)J - 13.
W a,A log
Velocity, (1) ~ dzldw =·- c ·;in w = -d[l:.: (z 2tc2)J
. u (- Q!l au ) +.ri ou + ~ = o 1 l dz , .
q- =q= [-,#,u j= ✓ l'c -z 2 1=;✓ [l.c.-zl, lc+zll
11 ,.. .'(5)
k or
ilr r . 1,1•,,

1 ...,. : ., ... ,.,.,,<,.


o·r Zk = ( u _ f!. ) au· •· By(3), c11 (coah 11 ljl s!nh¾,) (a. 2 + >.)' - (!>2 + >.) "'a.2 - 1, 2
- Q
r u or "' (3)
or · ; c 2 =a~,:_ a 2 (l - c2), For b 2 ~ a,ll_(t - e2 )
. 2k r"' ( u - k ) 'au
u
This=:, i:!.r I as u = u (r) or C::i a·e.
I!:.u ) i:!.u " ~ N9w {5) becomes q-l ~. ;I l J.i, - ae I • 1,; + a,,i l l ... (6)
r.:i:!.r
01'
( ti - . ; · (± ae,0) are co-ordina.tea of fo-c!, denoted by
. 2
J , 1 s,a.od S', P Is a J)Oi~t z. Then r1 • SP a j z..; a.e !,
2 - " Of> u·= 2k og,. + log A

Iote1,rratmg, U L

!l2 , rz"'S'P= _lz+aej.


~,r
2 = k log (r
2A
1 . u), where k log A1 = log A, i I
Now (6) is expressible as

q· = ✓(r,ri) or q = ✓ (r;r2,)
2
~r r2u A1 = eu m. Take A1 = 1, we 'get 1

l
-r=-ri;e" ' 14 k : ... (4) From this tlwreqired result follow~,
Replacing u by V In (3) and (4), we ct the two required results. Problem 32. >. dano.ling a uariabt'e parameter, and fa given function, fin(/. Iha
conditia.n th'11 f (x, ;;, >..) " q _sfiou.lfl. be a pos$i/Jle system of atraam lines for steady
1 . Problem 31. In two dlmen.sior:al irrotational fluid motion, show that if th~ striiam irrolational motion in two 'dimens!ons, ' ,
~ lines a1·e c·onfocal ellipses, · ' · Solutlon. Suppose f (x, y,. ,.) =Orepresetit.s stream lines for:dl!Terent values of
I· . x2 ·: "2 A, Solvlng this'eciuntloo, We get · ·
--+..:...,____=1
a+>. b2 +>.. ' : >-•·F(:r:;;1), ' • ·' .... (1),
~ Alog 1✓ci 2 + t.) +, "<b 2 + ~)] + .B . . ..· .. '. i' we· also know that; 'V !':' cpnst. rl)Jlreaehts str~amJines: ~o wo"i:an sµppose tha~ .
i!•~ -~ u11i:!.. t/1e
'II ,...
\ ' (•,:
velocity at 1my pi;lnt isi/nl>srsiiZji) proporl/Mali'to, the squor~
. ' ·~' . ·: "} _,, :'
the
(l) an9i ,j, ma both roproseri~Jhe.'s~~-B.tr~,m }J12'~s~;ltm_ea.J]~ii,hat, ,,. .
11
:i
rectanel•·.im.der '"- .focal radii of lhe po,nl,
Spl~tlon·. The con for.pal transfonx111tlon a: .. c et:1s w
i
\ ·.
1

. d Zit.!
IV ,. 'V {~). Now~•
. . . ~ ·..
2t °a=; --..
:1 · is known·t_odleld the gNen; type· of'cootocal ellipses, : ·I I j : ~
j
; ~: ··!~~•r
1
' ·, • , .. ·~~· : .~1:im:~:~-:\•,:~~-
1 0
~~~ ..~-\• 1 1~r,~~·:"""~;:r~·,!U«Ut~~~!:~~~~'.~mr~1~mm:.~:~:~~:~:rrt;:::~~+~·t·~·-:.: ,~··.- • ·.•'"'.'••:~•••!;>:,;•.1;\~!;;it=,~,')Jif.]ru{mfmftffl~1'ff:~•y;~~~~f.•'-i.;•W,~•~,1.~•-•;~,,~•i.-!*fyt,f'fr.'4~""•w~•i'•a,1"tf~)fl'O,l\.tltvi"l,;!I~
.l'
.
:~
.,

·:·.;;·~;H;":'·',.,' ',' ~;, ;;:·, i; :,~;: ,,;, . >;, •• ,...,(' ,.;~•~·~i::F•' ·· .. ••·, ,;"''1:-~•;;;;,;;'.·' · ,:.~I?;:·:::: ,.. , ;-;;;:< ::•:::::•::,::••. 'i.':'.;!f'•
1
···t:•~~.;rg~:··· · ···""'·~·~

···'' eo .. ,, ,>>,•.,;c;i,nu. · .~ 9 ,.,, ,13.J~ >.h, ,v,v.~i'~:CJ:·nr,:

.;-:=:;:::·;:?:'
-
'--,,,.J
H
p

f
· _ -------~,.~,.,,.,,"c:-..'l/l<.~'lt.Vi'l:.:o,..'liQU.'L....,"''l·.. ·"i,..,1'c.'1.'-."'\.~.;...'lo.'V'Lt"•'1.r.,v,_ .. .,,_,...,,-..;u......l.._ ____ ,_, ___ , '. • .... , ~·. 1,\4,-,• ~-· ,, ••

IJJ!EJK~-~t;~;~i: ]t ii~;tjflfflJ~i~&jt;i, ·, i: ,'.; ~;_;;;j i~.~.'.: .;:_ ;~;:;; ~j; ,;; iJ!:<8!~~;g:~!~!~ i.-.'•:1 ,:; ::: 1 .', ; ·'. '.:; ;,,)/ ;.~ •~J;.:-iivM~~'.❖}~l}~li!D!llf!~tl,-;nHil'{;,'.t?:'::"!·,1 ~n!-!,•,:,• .·n • '~ • ,, • -- , r,

.
. ~,·
•1 ~H~ ..:f 1·1 l,) !

2.
,./ ~ectilinear !\!lotion:. (8..1:-1. lVl.')
,,1t
-------·-------..;, ____....._.~ -· ______ ,

§ l. Introduction. Wh~n u poin}'(or p,llrlfol~\ moves. nlo1ri;,u


''
$traight line, its m.otfon is szi:.I to l)c ·~ roctlllneur 111otlou. Hencr.
'[ /- in this chapter we shall dlsC\l~s.thc.motio,n•of. n point (01•purth:le)
along a straight lin~ which rn11.y be ~ith~1r horiwutal or vHtlcal:
§ Z, Velocity ~·.1ttl ucucler11tio11. '
· Suppose n pnrticle mo'\/:s aloua ii ";ct
~· a straigl'it' line OX ),vhcre, O•is u fixed 0-··" "'T'" ,-.. ·;,••u--:·· .. '
.·*::OYNAC4.I C~iJ:i;, on the line, Let P be ithe posi•
l~~;. . . . :. ·. •(.- .wu:sz~, •·
:ion cf the \lilrlicl~ nt time·,.· whnc OJ>cz:x, If r d1:no\u5 the
;->osition 1•cctof or P and i denotes the unit' .vector uloni;I 0).',
t · I -i,.

!;.••''
l:hcn r=OP=,'< I, .
~ .....:.: Ldt v be 1he· velocity. vc~1or of the p~rtiol~ al /'. Thc11
dr ·cl ,. d:,; di d;,;
~ ~""'Ji=<ii (;,: i)=dt l+:r dt""'di I,
·ocoause i is a cons tan ii vector: Obvh.l\lSly,tll.c vector 1· 1s ~ollinc:ir
vector I. Thus. for':l par.tic.le m~\·ing. alonu., u.
I. :.ine the d.irectioll 10.f'velccit)' is aly..uy& nloni tlJe lluo,l\self.
P the particle..be mcviqa in 1hc ~lrccllon of x
the direction OX) and if ihe mugJ1itude .M its I.e., i l$
sp.eed be Y, we have
.•. d~ I Tl f d;,;

6
V=V J=dt , . \U(C ere tti""I',

On the other hn11d if 11!. P thq;llrti~l~ b~


. \
dirc,·.1iv11 , ·
.
M x dccrcnsing (/,a., in the. diret:~011 X9) rmd n,agnili1dc
·er i1s v.ctocity be 1•, we have
dx dx
. i·=-1' I""iii I, (l'i~ - ~II

Reinem!Ju, ·1,i' tllc


ca~·t of a r.ec1l/111ecJr 11101/011 1he 1•~/ocity <?!
dxldt a/o11g //1e lint lts~lf 1111!/ iJ wk.m with·.
<'II lime I /J•
posl/11•~ or iwgu11,,,11/1:n m:,ord/lig_ as tlHJ:flitr'l/lilt i,1• 1111lvi11,r: J'n f/1~
,, lrcct /011 'a/ s. l11C!el/J'i11g or JI <1,·,,·e~·Jli1i:.::.
', ... --..
i\
'.i
:1
\l ,.,, ·.;··?:'•r ··.· ·1"•, ····-·::t::r. . w "')•
.,, ·,
i~
!, :,5;foi:'f'.":
:lh';";i,;,~ ,,_ ;,
•-~==n=A~>.=.=•,,~t"'l':T:•"'.-;'~;;.~~:::,.;;
: :v -~-.. · .....'.-----------·
______.:.~-.~---- ---
'
,,~>i\''.r'', ' " ' ,,:~•.:, • I •

..:',:.:,•:·

; 54
. Dynamic:,' Rectilinear kvtfo!l· ~IS

I ..-~ Now' let. a be the aceelefntio1i vector of the p:utidc


d '(d,t ) d'x.
HI P. The 11.
·Differentialing'agaln, we have.,.
1/le acceleration=dv/dt ....-·a sin 1.-b·cos l=-x.
. .
~?, : :-,-: .•"' 1f\>
, n""''jf.['::''i'{T dt I ""dt11. . . . . Ex. 2. A point movqs t,r a stra/g(J/ litre so 1/ia I Its 11/sumc.c :,<•
i
Tll-lls thc,;vJct·o·r a ,is·collinec.'l' with l /,e,, thC1dlrection ofuccelera• from.a J1xed .pol/If a!
011.X' time 1 (i pipporilonal to I", .Jfv b.e tha
~ vcloclty and 1·11i1n1c.:eleratl0it al air°J? //me 1, sho1v //rat
tiqn !tal~a.y's'nlong th~. line-itself, If at .P. the ,acceleration be
l • natlng .. fn~thc,9_lrection ~r x increaJing .npd il its mi1gnltude be/, !). · tMemt 1981,•.~ P, 85 S)
l .· . da~ . . ·dix , .b . , Sol .. Herc, dis.lance ,;r:r.:1•.
,. we ha:vc.n=f l== dt' I, Therefore rfii,;,.,,f. n the other ho.nd if. Qt . ,', 1,1 . 1< . ' .. {J)
\
I
P.:tlie acc~limuion ~e ncting In the·"dlrection or;,:. aec~easlng ::ir,d where k is n constant proportioMlity, or I
I
··Int~. mng'nfiude be'f; we'have . : ·Di/Tercntfatirig '(I), w,r,t: ·•,1•, ·we have
· d2x · d1x. · · · the :velocity v;,,,ds/dt~knit1- 1;' .:. (2)
.·R.,,,-f.l=7t1 /·; fherefore-T,1 ... - / . I I
i· Again diffmnUatlng r;),. ·1 . '

Remeqiber, Irr itie c-a.se of a rec.till11ear mot foll r,/ie -alic11/era• -~; the acceler.atlon/=dv/a1-kn (11,-J)'!•·~. -.. :(3)'
;f /loll of a.partlcldl/ //Ille I Is d1xldt' along ti1e_·· 1111e. /(~•al/ and Is \11 =i (kn1n-l)t•= k: 11iz1 2,;- 2
;~'
: taken wllli poslllv.e · or neg_atfre sl~n ·aocord/ng as 'II dct.dn' 1I,c . · .,}, _ n.(kfl(u-1) 1•·'}0.E_
··1 ri/recl/on of ;,:,(ncreas/ng or;,:. decrca.rl11g, . - tn-1} ·
·si_ncc the ncceleratlon Is pr:)~Uc~d· b>' the• force, therefore
,, l

sub~tituting frOJJ): (l) o.nd


••
.l
wbtle considering the tign of-d 2.i:/d1 1 we m.ust no'tlcc
of the .act.Ing force-and not the direction io which the 3:
Ex .. .A ppr(l~le moves a101tg a st,i:a'ight line suc/t'(~'at its ''."*"':"'.;

rnovJn·g. For exacnplc lf the dir:ccioo of.the acting forpe ls that ,


or"x Jncrcasing1 · then d 2x/lit', must b~ tnken ·with poiltive .sign
_dlsplace.menr x,from a point ~n 1l!e-lt'M at·ttli.re 1, .fs $1'Yl!II by
x=1~:...:9t 2 +·24t+6.. · · '
·'.
wlle1h'er th.e particle ls-moving in. the -direction or x.in(;rensiog or Detcrm//rn (() ·'/he /J1~ta!lf wh~ir •/he ac~:eferal101fr h1[co117es:~~ro,
:~.. {.
'
in the dirc.1tJ011 of'>; decreasing. . (ii) the pos/lion of the p,ariic!e at that. inst.ant and '(lit) the 1•eloclty
Other Ex~r(i"sslons ror accclcr~tlon: of //r~:partlcli, th~n. · . [M<i'erut 1.971)
dx ·J ~
Let v::;,-ii. ·, i: Sol. Here,. x=il-9.' 1 +241+6:
I We. can then .t . :. ,the velocity v=d.~/d1 .... J1~-.l81
d1x · d (//,~ ) di' dv' dx dv and the acceleratio:if=·d1x/dt 2·=61-l8, ....
,ir(l""d( d/. ""dt""d°x' di tnV'';f;:' . Now th.e accelcration',..0,·whe11 GI..-IS=b or. t~:1. ·
Thus U, Z{ nnd
I • ,. . . '
v ;ta.re three expressions 'for representing the accel¢raqon is-zero whe~ ·1.,;,3 sc-ep~d~ ..
.tb9 ·accelerntiot'l.·and i\~>' one of them 1n11y be ·u\ed to suit the·
convon!!)nce·ln work.II)& out the _problems. .
(ii) When t=J; .. pi.:>sitlor:1 of tho p'nrticle •'is 'gJvcn by
, x.,,._:t 1-9:,3 1 +·~.. 3 +G=24·unlts;- ., . .
(iii)· When 1=3 .the,•velocity 1'=3 ,3,_ I 8 ·3·+ 24 == ~3 units,
1
.
Nole, .. Often v.:c denote ,dx/d(, b>' ,l; and. d1x/dt 1 b>' X, Thus when r=.3, l?C'Vcloclty,ef the particle !s:3' units It\ th11
Illustratlvc Example~.·: • . direction of x decreo.slng.
. Ex,), .Jf;al 1/me. 1 il1e disp/aaame/11 x· · of a f!articlu 11101•i11g Ex. 4: A par1:d1e·h1oves.alo11g·a .r1raig/J1,l/11e·
.. away Jr.om tlic·orl!Jln•-1.s g/v~n:by x=a. s/11 t+b co.r 1, find the vefo• frolil a fixed poln{on tlia l/11e' Is '.gii;ei:' b-;i' x=a ;J
,,,., city and acca/eral/011 o/ the p<frlicle: · tMeer~t -1977) 1/tat it,: acceleratl,on varies as the distance 'from, ::•:

,,Sol·. Given th-at .t=a ~in r+b·oos 1• cNrectM to11•ards the ori}l11. ,·
. D:tlforcntialing w.r.t. ·'I', w~ have Sol. Wo· have x-a cos (f't+(), ,. ... (I) ,;,
.t11,; v~locity y ... dx/dt=a cos 1-b sin' r. I
~~
}lI';•
... •.·..:.·. - ...... , . ,--;- t

. .,.~, '.'. ,,-·

><• H,,.,,-,,>-<•
runm.iuw;i;WH.iH/iHH}HUWMSUUllBW:!~j;~JJ: \;;~;(~;; L: .:\;~;:d~~.;(1,;)}i(:.~g~J.\}fo ,.~\· ;.':,~;. 1~;~,1.: ;1;;,: :t.\H~~i$!1i.J:w~H·~·..,;,,' i ,: ,:, ",, ,~- 11,' i.·t''

·I ,;•----·

\. .
i 55 · 1Jy1111111ic~
-iI ' '

· ;I?lff.m:,ri\iating. w.1:.\, 1,.wo.get


' ' H.wi/f11ear Motlr.J/1 57

dx{d1.;;;.:..a,, sin '(,ut+<), . Diffcre11tia\il\g: again..lli.,r.l. t, we get


•\ i
i" and d•x/4( 2 --·a µi,90s'(,ui+•)=-~µh:. from (I) dis d's d"s ' · ti' 1s . 1/",r .
Hepce"t/lr occeleratio•n varies as tbe <Jistancc .~ r!om the d(~""ifi'a+ dt.' I or ii~ t=O or d'if'=O
•origin, The.n.eg'ative sign indicates that it is in thenegative sense because t:;J,O.
·x
Of x~axis· f.ei, IO;Nar.d's lhc origin. ' d~s . d (dis) . dis ,
Now d/3=0» ili Tti =0=> dii ... constu11_1,
.... · ·. Ex>S. •1'P.·i1r~iclc mcms alb11g, a strd(g/11 line such that- its dis-
tance, xfr.0111 a fixed'point.011 tr and Iha velocity v there are 'ra/atcd . Hence the ac9elcratlon is constnnt. > ·1
1. · by v2 =µ.•(az.:..,"'z), Prove 11;a,_the·ac~elerati{11 _varies as• th~ dista11ce Ex. 8. A p6i11t moves in a stralgh,t !Ina so 1lta1· /Is i/ista11c11
of'the pa'r.tic/eJr9m the.origin. and is di'rccted ton•atds tha origin. from a.fixed jloi1it /11 that line Is ihe S(JUara rcpt o/ t/Ja. quadratic
•· ", • . • i · (Agra 1975] func·11·011 oftlie 1/me; prol'e that its accelefa;f/011 1•ar/cs inversely
Sol. W:c have (11'.:....~~). t' ... (I) as the cube pf t/ie distnnc~ from tt1e fixed p~·11ii, · •
Differentiating ) w.r.t. .r,.we get • · Sol. At.any time t, let x be the distance of the particle rrom
dv. d1x .d . ,. a fixed point on the line."Theri ·o.cc.o·rding t"o the ques(ion, we have
2v·(/x=;•µ (-2x). :, dt~=v dx=-/'-x, x-=v'(at 1 +2bt+c), i\'hcre a, b, c:,:irc consta1its,
... Hence t'he '.acceleration varies 11s tlip disto;.1ce ;,: . rrom· thc .:~ . x1,;,atz+2bt+c,,. . , .. (I)
. _orlti~. _Th~ ne_g~!ive. sign i~dlc~l:s: t~;lt it Is in the dh'ccliUl) or X Piffercntiating w,r,t. I, we get
d . .
dedreastng.i ,a.,, ·towards. ·the or1g10. . ,2x 7i=2at-!-2b
·ll;x. ~ .. The ·vcfoc/ty a p(irticle mqv/11g along a straight of
1/ni· whe.11 at a'tiistance x.. from:thc. oriJ/11, ,(d,entre · of force.) var!es or
dx at+b
as {(ri;;;:_x 1 )/:<'}, 'Find /he law· of·acceleration: [Agra 1979)
'iii="·:;·-. .... . . ,.. (2)
· · Spl. Let v be the velocity 01' the particle when it is at a dis• Ditrercnliating a~in w.r.t. 't', w\ have· , 1

, tiLn ex from the origip, Then !l.ccording to.pie question, '-";e hi\Ye d'x ax-(at +·b) l<lx{dl) tl):-{a •i'b {(at·i· b)/.q li'. (, J
Tti=---x,:---- =- , , ;<'' . -.-, 10m -J
, v=1-1v((ai.;..x,)/.~ 2 }, ·wh~~e 1-1 is a constant.
. _;:, ,.i=1-1i ("'-xi)/xi=µ.~ (a'/,\'~,..,.. 11 , I _a.~i-(~±£1.:_q (at'+2h't c)-(a't'·i.. ~bt+b 1 )
.. • • • l , - xa· - x·
D1~erent1atrng w,r.t.'x~.we get I' · ac-b 2 I
. . · dv . 2 {. dv d2.x. µ: u·1
. . . 2v·d,/';"./.I . --;:;-... ·: . v;1,;:=,4r,=--;:i ..
,. . .
2a~) . . •
= ... •;;:i ., =.(some constant),_;:LJ' .

!Henc.c the acceleration varies inversely as. lhc cube or the
He~cc the acpcJeralion v'aries inverselyhs I.he cu~ or !he dis- <lis-tancc .~ from tl1e nxed·point.. . .
..~ant ftom\lie;origiri. an.d i~·d!rected to11•ar~s the centr~ of force. Ex. 9.. If a po/11( 111cves in a str1Jigltt !11w Ill such a 111a~11er
·. Ex. 7; The law of mot/or! /11 9 .straight flM · being given by ihar Us retardat/011 (s p1:upor1ional 10 /1~· spl!ed, pl'OVI! that the
· :.= t v/, prove ·rlfrlt-.the .acceJ,,,a(ICm ls.co11staift. · '[Me.erut J979] spa_ce dascribed ,,i any /Ima is pl'oport/Q11al to ,tltc .vp11ed rleilroy~d
ill that ti111a.
ol ., ,·W
· e l\ave •t
1 ~
ids t. [, ••·
2 di
tis]
SoL Here it is givrn lhllt the· rctar<li,ti<.11~• <:1. ~P~lld, ·
·~,,!i iffei'enti,atil\!I W,l',t.,.'I' we·.sei,. , t/1· . , . ,. . . . r . . . .
ds :I dis 1 l ds . . Ids· ! d~s -.7Ji=kv, w1crc,; 1& :i consta.nt o propMll•.nrnlity
;111 1
di=2'Jil +2Ji. or '.iJi".°'~dt''
Ur '-1-:.d.-.:1!';,,1;r for d;;="--•
.
or~ .. I n\cgrating{v.,;•-(i)A·) + A, ·
..•
wl\cr,i 1/ is constant or l11\IMl'lll.io111
~
·.L. ''j
t
th
111.11
•'!•
:liliiH ':,~~:::· ·:r ,; '··:"~-:~v,..,..-r:;1"'~»~~;~. ~·:"~'.!
,; ~ '
'i1 ,",.,,•;, ::l• ::-:; ~: ·:' :-...
,,· ,:,.'.;,,,,!·.",Hrtl ;•~•:i ,;,~, · ~ ·. , ':,:,,;; ~'\·i< •;·: l'.l :~I••' 1 • i •' .:,.',';, ,,; , · • •• ;! ~ :M·li\g, '. ~ ~ ,~J;•' '. ;• ' (, '1';,~1· ,:e:h'. ~, '.•:"• '"'.~':'.-:; ;; ,:•;'

'! ----------~==cc,.·Tt'""'";"T,
. ~----·
.• .. ' ..

:·,,,,.,,.:••..
:,:; ' . ,: :~i; .': ::,:,.'.I;,':,
p
ti
r.i

f1
,:;
58
Dynamfr,,· '·
M
:.1 ....-~ Suppose Ilic particle sl~l'ts from the origin
Then 11 =11, .\'--=0. . with velocity· u.
. )~e,11/l1iear Motion 59
E,x, .11. T/rl! ,·elocily of o p1;mi'i:!e :,n:ovl11g a/011g ·a stra/ghl
,(),.,, - ~../. ,1 or 11 I iiM is g[1•en by tile rela!101i V~=«xi+U.:.:+.c. Proi>e (hat ·:the_·
k· A=p/ P acceforation 1•aries as the dista1ice from ·a fixed point in lhe.ll1ie.
·v u 1 j
SQL }iere ,given th'at v1.=axi+ 2l?x+ c.
X=--+-=-
k k k
(u-v) ·
Differentiating w.r.1. 'x', we 'have
or (1,--,v)=k:-c.' .. ,(l). dv' - , . ·:
IroycdNow the space described in time is,x ancJ fhe ~peed dest• I 2v -dX=2a.~.+2b,
In time Hcnc~ fro_m (l), we conclude 1hat the
/=11-1•,
space d'escril,ed'in•any time is proportional to the-speed destroyed
in that' tin'le. · ·
or
1
. ax
dv.
I =v-r=ax+
·
· b..... a·( ,:,: +b)
· . · · a- •
...
Let P be the- po sition of tl1e particle ~t'tlmd r. .
Ex. ·10,. f'rove'11ia1 ff a po/111 move,r w/1/i a ye/oclry i•arytng If.,= -(b/a) is the fixed point O'; t'.:ten the .dlstiince · thll
as any /mi.w (not .less Jiran rm(ty) o'/ irs d{srance front _a jixeii poi11t . patticle,at tlrne· t fro~ ·o• · .· · · .. :· .
iv/iiclt:it is approachlng, lt•Wi//11el'dr reach r'hat po{11/, · , "
Sol, If xis the d/s1,ance 'or the particle frQ>in the fixed point =·.0 ,P=x-.
• 1
(,_·,.b) =.-i:·+-•
a ,· a
b
0 at any time 1, t!Jrn -its speed 1· at that time is given by 1•=k;,:n, :. f=;a.O'P, or.Jee O,'·P •.
whel'c /( is u constant and II Is not less than 1.... · Hence the acceleration varien.s the •dist.imce from a· 1\xcd.
' Since the r,nrtiel~ Is moving 'towards the fixed. point i.~ .. in point x~ -.(b/a) in ·t\1e line. S" . ·
lh~ .. direct/o'n of.,: decreasing, therefore ·
Ex .. 12. If I be rerarded,;,tp a fu1i~tlon of. velocity• v, pro1•e
rlx/df=-1·
or d:,:/d1- -kx•. . . that the rawofdecraase ofa'ccf_leralion is,g{ven.by/ 3 (d'r/doi),f
Case r. ff n= J, then from (i), we h:,ve being r~a. acceleration. · · ., (Meerut 80, 8~],
<ix/dt=-kx · Sol. /L~t/be pie 11cceleracion at.time ·r.'Then/=dvfdr, ·
, , I d,1; Now the l'ate of dec,reakof acceleration= -'d/fdr .
Or ft
I=---•
. k~ .ii (dv) d (dt.)..:1 .. C

. lnJegrating, 1=-·(l/k) log s+A, where A. is a coiistant.


=-,ii di =~cf( z;, ··' ..
, r.egarcl,ing ( 8.S a functio'n Of V
,,,.,.0, .the 'time t 1ore1ch the fixed pqiht' 0 is given by
. , .t.,..-(1/k) log 0:t-A=co =·-{d_dv (d1)-
di• ·
1
}· ... -(-d! ). ' itt1, •, ~(
d1•
' d(- 21• dfii '·'di.
:
i.~., tirn ,pa'i-ricl~ will never reach·''t!ie fix~d pc int 0.
Case IL Jf11>J~ t11cn l'r'om ('IJ, we.ha·,·c· 'd 1_(!'..!')
.:_(el:.')' ' 'dti.:.' '.d11
1 3
.·. I . dit =. 0 d't.
-;'. dt dt . ' dv~ f dv~
1-
111 ':' -· k ·.r" i/.~.

or
lntcsmtihg,
. I ~.-,,,,,
--k -n+
---·+B,
L
I . .
· E i~ n
wJi,:re·
. "'.{:
. · s ~ o n und.cr constant,·a~crler~tloa, ,4, ,particfe.t110\'eS .
i11 a strafght l/11e iv!th a ~onstar.t acceleration· fr (he 'illlUal velaclry ',
beiiig u, rq dfscu{s ·rhe motto11: , • ,(Me~rut 1sj·
·)(
M
.
Suppose a particle :moves··•· :,:. •t I '
t=~--("'"t,-1--:,.,JJ,...:-,,,;=r +n. . I '. l in a-strnighl line OX .si.arting . · ·
.~.... o, lh.e time I lo rea'ch.the lix3d poi1.1t O is given by from O with velocity u. Take I O _' y . P
~

f'=CO•I-B= O".; ' jlOas origin. 1.e.t P be !_he;posi;_ion of the particle ,M :iny time 1,
l,e,, tlrn Plirtiolc will never r¢ach ti1c llxt:d point 0,
where OP"'x\ The accel7rati.on. -Or pis CO!~Stant_und is/. There•
·1·

He1\1.\c if n;;, I, 1he particle will never r,,ach the fixed nolnt fore !he equation orm.ot1on or 1' 1s ·
'it is npproaching. . I d 2X . , .
\ 7fii=r. , .. ( l J

---· _._ ·a,11. ~


~ I
. . '
.I
,,,,•',

iiiiM!ii$i4ffi&4iii)i4··•··
C ', , ,.

-·- . '., &olV& .1Ei


.•• _ ~~,., ""' .'".!'.oi._,,.,1,"·~ ..1_·,::r ~,r;-·tY •'''" f-"I' ~ '1' • •, .
, ......~· \,.,,
,' . ". ~.•" i"rt" ..'. ,., • .· " .

.,.,,,,,_.,,.,.,."'""....... ,....., ..... ,-,---~~-~...,_- --·-7···

~m;:):_,i.•; i, :,ii/ 1/:~i;~~iM,:,;;; ;;:,:;;;,:,::;,,;: ;,;;g~,;;,;;:~:i:,;:;::i.:!:,'i:if,;.;;.; ,, ::: : :. '. ;,.::,:, :·:::I;i:1~,;:i.lls~'i-ti!li<>>W:<W"•' ,,,,,,., .• '''"'"' .,.,,,: ·,,. ·.~ ,,

.,.. ,,.,_ .. .... •,""""l'""I 11 ,.,.,, ...... _ •• ,. • ,. __ ........ , ., .~·······--; ··-~· .. ; . • • , ' zu;•~~iz' ,;,.:=::.:...', J·..,~

60 I . , . <t."•
. 1 . ,D,v11a1111cs, ·:,;."'
·.:\,.
l. . .' ·, . "
~cii.lln~a; M.~tlo/2 I . 61·
·· }f 11' fs \he.vc:locHy·\jf tlie 'p:i.1·tiil~ !i.t,(any time 11 tliett v"' d:,:/dt,
o infog(at,ing·(I)' !.y.r.t: 1, w¢ s~.t· :. · · .· ..· · .. /·TH~se ~qua.tlof\s_give . ,. . . ,,:,
\''i"'d:(Jr/1..:.11+;/, \'.yhere A ls'c,~i:staht, of integration-. ... x,, +x•..:..2x,=·U-U.i 'H,.:.:21,)'-~ t ! 1
(11 1 -;l-tn 1 -,., 2ti ), ... (I)
B(1t i,nttia/ly ,(t O,·\'o;:itl and /;;.(/; ~hereforciA.;,.u,. 7hus , .. . Bur . ~" x,, .-:, :are•: in .(l.P,; So .·tha:t :xs:=':y{x'(X1i), ·/\lso ·
' . . ~- '
WC
.
· ·:;':;Iii, t~, ·1$ ·are· In ,AiP. whose. common· ·:ct.lffercn,c_o .. is.'d.'· • Therefore
Ji:ive• 1 ·v=-=u+ ft,:
' ,·,1 . -ril' . ... (2) . l1.+ln=2 11 and l3-l1=2d, Putting tliese values In(!),. WC-,gct
The· cqu_:ttio1i'{~) 'gives the 'i'eloclty. V or the rarticlc a._t ·1rny
tin,.c 1. ·: · · ·.', ·
1
x,+x3•-2v(x1 ,'\:i)::tt,0+¼ / _11 +.11\-t-:?
' f ('7+ (,1)11'r
. ·Now irii,cgrating (2) \v,r,t, 11', 11c get· ( vx1,-V X.1) 2 =¼ / (211 1+21:?~(ti 2·Hn1 + 211/:1)]
· x::ut.+ ➔/1 2 +b, ~rhere .{J ?s !l constant c= ¼/ (1~ -11) 1 = ¼/ (2d)' ..Jd 1 ,.
· l.ltit at Q,, ·1;.,0 and x=Q; therefore 'i=O. Tilus we have Hen'ce f ,,,.,r ..jx 1 - -./xi)'/d~. . ~
·' x=·ul+½/17, · · ... (3) . E_~: 14, .Two cars star/ oJ[lo race wf th vclorities u and it' crnd
Th~ .cquati.on (J) g'i-;cs the. positiol} of the particl~ at, an:!' 1ra1'.el /n ti slrmglrt' /fnc with un,(for111. acteforalion.r .f and f' resptC•
t iiric 1. · ., • /Ivel:,,. · (( the rar.~·en.1/s It,, r, dea(l neat I prove thcrt 1/ta Ieng ti/ of the
The equation ol' motio.n (.J); cnn also. be written 11s. ·course is .
• d1• -' ·1 . .. (2 (11-11') (uf"-u'.fi}lU-f')i,
, , ..- =( ·or 21'
. ' =2f,
. . Spl_. · Let ..i be _lhe .Jerigtl_l· of the C0\11:se, BY, ~end heat WC
'
1~ ,·'

fotegratin's it w.~.'t. .x, we got ·m~an- that .each car moves the dlstanc<!' in· th.e so.me time, say' I. s
'Theiu;onsid'eri!)g the inot ion of rh e firs·.t cnr we lrn.YQ :S=II/ +½ft•: ' .
,' ,,Z:J::2Jx+c, But iii b, ·X=O · and·· l'=u;· thercrore C=u;. · nn(/. considering the m,~tion of the second car, ive h1we .r=1l'1+
1

Hc'nce we have ' · ½/' tj, These eqt1nti9ns•can be written ns


;•~=u~+·2fx. · .. ;(4) . -!·/t~+ut-s=0, . .I ".(I)
.. '.rJn1s ,ill equutiqn's (2),'(3) a~,d-(4) we have o,l:Hained the_ th,ec and . . · . L/' 1i +u' t,-.,r=-0, . ' ... (2)·
··well known formulae ot rectiliner mcitiori :with consta·nt acceJe: •ny.. the:.. method of .:c'ross multlplica( ion, we get• from {I)
ration, ' . ' and (2) · ·
Illustrative F;-(ampl~s
' :Ex. 13: A pa,:/ic/e moves' (11 CJ s/rti.lght·. i11ie···•vi1(i COI/S!IUII accl!•'
.• 1:_u· ,i· -.r 1·· l---'-s.:. Tl.·.....\ fT7u
I
I
/era/Ion a11d Its distances from ihe origin 0. 0/11/itj: /ii'ie' (no/ /li:C~S- · u' -·.r · I' -s ½f' \ \-[' u'
1• I ! .
'saril)' tile .position al .t,lme·t=,;oO) at. tlmei, 11, ,~. /3 ,are ,>:1,.X2, Xu
resp_ec1/l'e/y,. _S/mw th~t I/ t.,, _1;, t,/orm, alI A. p., ,,·hose .cb111mo11
M ' fiY=",iJ'.'/"' ,., u=-rri-· ½T/11'-/'10·
dllfer1111c1.1 l,s' d .and x 1 , ,\·~, ,'\!o ar.e Ir. Q, P,,. than tb<J accel•rarJo11 Is Elim!n111i11g 1, wc'have ··
' (vx, ..... -.;.~.)'/d', I · /ii';...u) ,r ( ·&s (/- i
½ (/U -f'11)
1
= !( II - . 11)

\'
. Sol, '·L~t ·O be the origin c
and -9 the
point start' i.e., or ,.,-:----•-DI ): ~ ~) Since s;t-0, 1
1Kcrhfore · s-{2 (11 -u) (/11'-f'11)}/(/-f')'
ihe po~ili611 at /=O. · .., " (11-11') (1(('-u'/))/(/-/')l,
· Lcl Ob=c. Supposfu is the inilial velocity und / th: cons- Ex. 15, T1~0 pnlllcles P aMi Q,mov1• . /11. o ,stralglit line AB.
tant uccelrnuiu11. Let A, B;:c be.th~ ·pusitions oi' the pariicle at
h . ttirn2's i1, i2, la rcspcct!vely and let_ Q..-:i=x1;.Q.B=.\'~ and O(,'=;,ii,
The pnrticle.J{-slar(s from. A. ill tlia: cJ)rcc/lon":"A.p ll'ilh ,•aloclt:,, 11 a11rl
ccmstant (Jcce/ero.tion .f, and al 1/uJ same tlmr Q: .1'/arts from B In. the

I
.The.n· ' . -t'·.:,.;·,... . . . .. ,· ....,~.· dir~<:1/011 .BA with velodt.v iii 011d r.on,rtiwt ,aacc'/ua//011 .f, : (/ tilf,V
x, --t·= ~,, + J/1?, .V , - l'=UI, + ~,,,., .~.--c =,ul,-.· u,,,,, pass 01re ano.tJwr al tlie-mlddla po/111 of ,,/E'11nd ari·li•<' at the otl1N
Mrl.i n.f AB with eq11nl velor/1/q,1•, prov~ tltdt

~'

\ ::,~·:!::::.-;:,•:
,_ ,.•;>.•:-~.;<._~"<ff>'.'-~•~:,., ·~~~.,,-,~;-..

,;.,, .,,., ...,.. ·,:-: .. ,.. '

n,',": '.••'r• . ·. ,.-;~:. • .


•: {I:\\ :,1 ~ ': •" ·,
. :.;,;-;1_;:

"~,..,..-
~::-....,-,--,~~-.
' • <..,•,~ •:.•;,,I:

,1
}
~
~
i;
62
'. . Dy1taml/Js'· · ·
I \
! R.ect/1/near Motion
'
. 63'

ll i ·-~: , I' (11+u,) U~/1)=8 (,[,.1,-/iu)'. .


, Sol, L~t-A:B=2s, Let ii be the vel6city of either pt\rticle after
Also 11+li=vJ/:t-v/f"=v (iif+Jf./1): . .,..(4)
movi,n1i the dist11nce AB=.2s . . Then Substituting the,vaiue of v'from {3) in (4), we get.

J{. · /r -"'
1 ). J-''[. ( r ~~ i . ]·.
1
Y =ir+1f (2s),;,.u1 2+2/1 (2s).
1 2~ . }.. ·
111...:u1: l=t1+ti= (J('f+J:/f.') . • \:/ '.• f' "7.. 2 f· f f:. ·
.r=m-n· Ex. 17. Apoint inovl~g (II. a straight line with' uniform· a qele-
Now let t be ihe time t.aken by each particle 10 reach the ration describes d{sta ~es a, b feet' l11 successive .. l,~terv:1-ls' of, ti, Ii
n;tiddle point or .A.JJ. Then· eAcb .particle moy.es dim.nee sin time seconds. Prove 1ha1,·q1e·acceler.q/lon Is -2 (11b..,...·t1a)/{l1f.a (11+r )], ·
1. Th.er.efore · : , ··
, Sttrllf+ aff 1=:1t1/-J"tf1t 1,
· j · · : tIDinp~r 19~1i .~e·erut.'69; _84S]
Since 1;,1:0, .therefore from (I), we ha've u•Hft=ui+f/il S~l. Let u be the lnit\aJ velocit_r and fbe the. ~.nl_f~rm _a cel/.1•
or ,~2 (u-.r/,)/(i1-/), · . ration_oithe particl~. Then frnm s=w+.½.Jt-1, 1,:e-have
Now considering thc\inptlo'n of-the part-h,le P t'o covet _the · . a=.1111 +½/11 2· .. ;( I)
flr~t halfa_f the_ jomney Alj nnd nsing the formula- s=w+t ft', and a+·b=u'(i1+t~)+il(!1+ti) 2'. .:.(2)
we get •. . \, Subtracting (1) from (2), we..ha~e ·
·. 11•-u, 1 . 2 (!'/-:.•u1) 4 (11-,·u1i . 'f;.•'(tii.:H.IW,•2f1lt): . ... (3).
11
4TT'i-n"" '<1i--7T"·+v. i1i...:..:rr". Multiplying (3') :b)l,/1 a,111:j (f),by.1 1 and subtracljng, we »ave•
or (11+111) (/1-/),...811 (/1 -1)+8/(u-u.,) ('.' IJ-u,,;cO) ' 'ht1-at-,d½/(r2 1::i.2'1 11,) t1:.,...½/.t1:1,
or . (11+111) (h-1)-8 (u/1·•/u,) .
= ½f c,;ui
+1?12)·=·½/ 111~ (td
··
or (u+u,) (/-/,)a8 (fit1"•fiu1,. .
Ex. 16. A train trm•els a dt.r1a11ce s In t seconds. ft starts
· I- 2 (b11 -:(}t,). .'
• l-11~ (l1 ,-t,J
from rest 011d end.~ at N'.I'/, _ /11 the fir~/ prm ofJourney ft ~loves ivf(~
co11S1'i111 acce/rrat/011 f n11d In 1/11!' s~cond part ·with constant re,1arda- \1, Ex .. 18. For, 1'}' of the dlstan~e between Mo sta1ip11s. a'. train
1io,rJ'. S/Jo11 tltnt ifs is tM dlsrqnce betiveen 'the two statiqi1s, then
1 j is Ul//formly ac.celera ed ntrd for 1/n of the distance It ,1.~ · ·uniformly
. l=y[2s (·1//+J/f')), ' . . '\ retarded; it starts om rnt nt·orfe stat/011 and comes .lo rl!st at
//If other,1 Pro1•e that the ratio d/ Its gr,en'tesi 1•1?locf1j, /0 lt.r av(!r,aie.
• Sol. Let v be the. velochy at, the end of the !lrs·: .part of~he
motion, or sny in the heglnnin~ 'of the second part or the motion t (
velocity is 1·rm +ii . : I,
I l) . . . (Meerut 1!>77]
·
and {1 a.nd.ft be the times for the two motions respectively. Then
1=11+·12, ... ·• . . Sol. Let 01 and.Oz be two stations at-a di5r_a'npe s'oparl and
Let;,: be the distance desorib~d in the first pa.rt. Then the A and B two points' between· 0 1 and O1 ·such that · ·
dist:i~ce d,escribecf i~•ftnc .second part is 1 s x.:
~oRsiderl_ng the Q
·
01A=.s/1i1. ' and
AB=.s--.-s/m-s/11,
·..no,'=sf,i:.
first pnrt of the mo11i•n With co_nstAnt acceldrat•1on f, ·we hnvc
· · rr=0+ft1=fl1,·}· . (I ·1.,1.., ·,
..• 1····1\ . ,ii)
"-• ,;,,•·•.;;r. ·
'
V
'
;,:·,,.· 0'
2
Md· =0+2/.'l'.=2/x.
1' . ... (.J) 01 t1 A ta B'ft;l :02
Agnin considering tJic seco1)d prirt of the motion With cons-
innt re111rdn1ionf', .we hnvc · , The train, st ans at rest from 0, ·. nnd tno,vei · with .uniform
0=-1•-J'i:,, I.e., 1•=/'1,, , } ·• nccelcration / from 0 1 to A. Le,r Vb~ its've!ocity fit the point A.
nnd1 • o,,.,1• 2-2/' (s--x) f•,e., ,· 2=2/' (.r-:i:), • ·... (ZJ 11 moves with' cqnsianr v.elocity V from A lo E and .then moves
Frnm (!) nnd (2), wr have · · with nniforn1 re.tnrdation.f' from .B to (Y 1 • The ve/ocity a1 ·1he
staiion o~. is zero. .
S) ; ,"(•-,,
. p'I • ,.2
2j" ·1· 2/, or .r·, f
1-l ( J
f ·rr •
j )
Lei 1., 1.,. l,1 be the times lnken 10 tr:wclthc dis.lances OiA,
AB an_d BO 1 re,per1ivdy.

----------------.,...
iHWWW)!' ;;-;;;r nrn:w:i~im~uuunmtttWi1•; _; , iii i;;~: ;: ;_. ;:.;<: ;;.:::: :~~;.:~r-~~:~!i:f·:;i;:-:i:: ·, .•: :-: ,'. •. ·. :.... :,; ·::.:;•; ;::~;~-:~,~;,. ,J4'(i)f-U•>:(• ,;.t,:1 ;! !1. ,,.Ii,-••. ,p,;:i ,:
4
:.-t ,. . i'

·I
. 64 Dynnmics
Rect.ilinear Mot.lrm .cis
Now thb greatest -y~l~city ·or· the train Li:t the .vdocity of tlie train ut A be V= 22 m/sec, 1:h.e,n the
·. . d·urlng 1ts ·lourncy from 9:, -~Q 0,= V train moves .with. constant velocity. V from A-i·o _.IJ fi,·ntr:ne !•.
and the nvcrngc velocity o_r thc-traln,;s/(!1.+13+1,,). scco nds, In ·the lo.st thQ. tral n. moves- from ·.s tcMhe·,second:,stati;rn· ·
. the required 'r;tio..:.~rentest velocity~
.
• i1_ ·
V
aver.age .velocity. s/(1,+1,+13)
.. · · __ v (1,+1,+1,)
0, un_der on_stan..<rei~rdaiiori '.4 m/s1:~••
the .. lc~&U1me
f.oi
ti1,~~,-,:;,;}e,9.o.n¢r ·-r:1.iu~
7 _tp trav_e_l' be.tween I the .·two · stations: 0 1 o.n~ Qi Is,
I· - s ... . , .. (!) (It+ /if_·i~y_s:ec9n_cts .. · . . . • · ·
. . Also Oi9'1·,;,, 12::km·.-=·I 2000 rnetcts, ·
For motion fr-om' 01 to )f, usi'ng thdorrnuln l'=Ufft, we· havc,; 0

NoY/<u'sing.the f9rm~·ia V=u+ft for the- p!\rlS' 01tf. and··.BO,.


·..
... iI . _·v=o+
.
f11, . f=~·
/1
. of the journey/we have /
.I. _Now using t,hc fortnuln s,;.11t+¼ft 2 for the same motion, J/;,,.22=;-0+l.·1 1 so th.at:1 1 =22, '
we •hi1vc :tnd 0=22-~. /3 so··that t,=~-
•. 2, •
s . V 2 3
-
.m =O+-! -~.11
•t, . ~ow O1f=(~v~rage velocity,from 0 1 to ('l)X'/1- . '
I " 1 / , 0+22 . · .
, Zs
11=-·
. =---::;:-;-.-x22=2~2. metets,
01
. . -Vnt ... (2)·
22+0' 33 363 , .. I
For motion from;( to .B,.AB= v.·1,. and B02-=-2- x- =- - meters.
•192, 2 1

~·,t-. . o . .
!.,\
/,= A.8..,_bS(l11-s/n, 363
• ·V · · .V
F\>r motion from 11. to 0 2 , using the for_riluln 1•:.=11•l·fl, .we h:i.ve
... (3)

-~1;;.
AB= 010~ -01A-£lO, = I 2000- 242;- - 2
Ii
I .. O=V-f't~,. · .'; f'=V/i,1, • :er = 23
.. 2I 53.- mclcrs,
I

· Ufing_ the formula-s=·u1+½ ft'. for tl1~ ~omc motion, we !\:we AB. 23l53 23153 .
1,"'.'v=2.xi'i =,-:--4:4-•seconds ..
·.!....= v,,.-1,'!:.. ,~~= Vt32 . .,
lli'•·

,:,
11· .
2s
2 'I In
lit t}:le require.d:time=(t/+1~+1a) seconds
n· :2i1's3·)·.seconds=--·r
. 2484'7
,;,'':
= (22+-.·+:-·-'-·-~- 44 seconds·

~
ti ..or . ln=·vii' , .. ,(4) .. 2 44 . . ..
Suhstftt1tfng from (2), (J) o.nd (4) in (.!),'the required r1io . =·9 m.in~tcs:25' s-~conas opproxlm.n.tely:
.(It>
: i?:\
: ·•::· . E'x. 20. Two points IIIOVC /11 I.lie S(I/IIC strci/ght ·/Ina starting at
·· • · .V { -2 s. -1--
;-1 ( , s · s ) +-·2S } -+-+
I I l
_ V,11 V s--:-·--- VII ,;~1::.
i

·i:
Ill II l>1 11 th4:amc 1110111ent from. the s1111e point: ·in the .ia111e dlrec/./011. .. The
' ----- s ·. l first move~ iv(th coi1sta11/ velocity 11 and the .rei:ond 1:,,/.th-cq{lslant
Ex. t 9. Th,e gr~atcst possible acceleration of a trni11 is I 111/,iec' accelm1tlon f (its Initial veloci1,,, being zero), S/1ow 'that 'the
_a1.1d the' greate,rt pos}ible ret.ardat/01_1 is ·; 111/sec\ Find the· leas/ .~~·
,;-. grcate_sl d.ista11ce b,itwrni the J!,Ji/11s before the .r~·co11d catches firtt
time taken /O,r.1111 belii'ee/1 /11'0 stations 12 -;;.Ill, appart . 1/ l/1e 111axi- .,f is 11 1/2f at the end of the ti111~ 11/f froni the first.
I~
~,~1
nwn) spe~d is 22 i11/sec.
. I . .
' lMcerut 1972. 76, 8RS]
.
)5ol. !.,ct o. .trui11 stnrt from the stnti9n· O1·and move with ttni- l in time I, then
0

Sol. lf s, anc/ s, are th e cfisti1nccs moved hithe two particles

form ":\c'cckration i m/.;ec' up,td A for time ·1, ·~econds.

l
/ I{
i h, s,=lil".,~.nd_s,~f ½ft• . .... ·,.'.,,..,. .•.,:, .
the ·distarid.Hi'eii;-ccn the t11' o'p'ii/11clcitnt lime I is ,1iYf.rt
1j/lid~>i%
0

: t~
·,i~:;,,. /,;.·};;"'!••~ •w-· 2 f(2,.i' ,..)
:i l
:~•. !, '-,; i, ..... , ~ ,i~tO-, hy ... . tf,
s=s,-s,=111--- . I=·-2. - 1-1 .
1
·.j°:
'J 'f11;''·
'f \h

11:·; ;,,:;:: •:•::?::::,, >:r~:):·:·,


:,:, ·',:;•:>:,:::-. ,,~:,.
. !..~ ' ' '. :: '
11 -------~-~--:-rr, . . --;<;YJ~;"';"':'::~

·/f\\G·:·--·
,.... ___ --..;r,-'",--.~.-.·

T/tl:i~:?-.:· -~){:;i/j.:.- -::.::;-.\H)\<.U{?Fff: :·. -~:·/\U/:·


_...~

'
,. ,.•.,:,:,·,:tt\'· : ' ,' ·, ..
ij
~}

..
~-1

:·l

~ Dynamics 1: Rectilinear M.o.tion. . •


. t
'67, .
l!i
"'::,....
I [·'II• u •z]·.· .:,),
: :~.-: or s=:r ·f•-. (1-. l) , .. ( l) t;·.
Ex. ;q. A Jif(ascpui~ 1,iJth co~siqnl _r.1cuiera1frm I, then wit?i
eons/ant veloeit)' and f,ina/11 sH1ps 11nq(r constant retardption : 'II
·l\1.·OIN.,ds
• <P r Jf.(t-u/f
1<>ru.Vst . ?,;,,O
.. al \f
j.e,, l
t=ul",
,:;
ri 1i,e roral ·distance a;k nded ,.is s. 'a.nd the total. time OCCl/pie is t ;,
-l~
show that the time du~fng whlch°~he 'lift is ascending with' c~n t_ant
-~ ..,.._so
•·1 .t'h,1, ·$rea,es
· • t vaI ue of f ·- u u s= 2 ·.n=-: 27' . velocity is y'(l'-(4s/l))., ' ·• . ·· · ·:
, :Ex,• 21', The speeci of a /rain Increases at constant rate r,.,from Sol. l.etv be tte maximum velocity• produced durin 'th!:
zero to v, t°liei, remains cons,a;, for.an interv{II and finally decreases ascent. Since tnis velocity i,s ·produced und.er a _consiaA.t ac~. ern• .i
to·:urfJ nt <1 ~t>tJ#Ql!f rate /3, I.I I. be the tcttJI dldiince described,
prove ,t(iat r;,e tdtnl ~/me <fccuple(ls (l/v)+,M2/ (I/a.+ l-//3). 11tso
find the· least 1•alue of tlme\wben r;.=f3, (A)la~abnd 1~75]
Sol, : Let,fp 12 , i. be \he times taken to cover.the distances
tionfduring the first, part of the·asc'ent and· d~_~troye~ un.~e the
same retardat1q·11 f ~uring the 111st part of the ascent, there 91e,
the distances as well las the times for these two ascents a.re e · uaj,
Let X be the distance and 11 th~ ii-me for, each of tl)ese tw;q arts'.
I
.«<,' y, z o(t-he' first, second ¾nd.last phn.se or the journey, Whole We have then ' · · · ·
distance l=.t+y+z. . •. ' .. 1••"" 2fx, } · ·
. Equations for the first and )a.st part or the Journey are and . · i v=fii . . , .
2
v'""2r:1.x; }
1 , 11 ... 2~z; }· for the f\rs~and last part oUhe motion,·· I
•and v=«t 1 .. ,( ), nnd v=f3ti .. (2) Also considering the midd.le part: of.'t'h'e motion, we haV ·1·
From '(1)~ on eliminating Cl(;• we hav,e x,;;J _vt1: and from (2), . v (1-211),;.,s-2x, •·· , . · I " !
on clfminatlng P, we hAve z=jl'I::, . ··, . . From ( !) and (2), on elitninating v and x, we ha.ve
Also considering the.motion ror the middle part or the joufr y> .. :: : •. f'f1 1 . , '• . \,
2
· ney; we fawe ,vmv/1, 1 •
f1, (.t-2t/)=s-
1 =s-- 7=.S:-:-/1, •

1 Thus x+y+z=Y (i·t1+t2+ it,) :, I ft1 1 -frt1-rs=0;


f.e., I= v ((11 +td• 13)-f U1 +10)] Solviqg. this as aiqu.ndratic in 111 we get
I _ft±v'(/2t2-4f,r)
or i,-(11+td-1u)-I (11+10), I1-. . '2f . -
the total time ooeupi~d
I ~
i,e.,; Ii+ ti+lo=UM+I (r, + ti)
V~ .(y • .
or 2
. /1=1± J( • 1
~ -.4s)· · 1=
or ,,..._21 j'(· _1 •l4.r) •.•
1
-

=t+ ;-+p)-• [from (I) a~d (2)) This gives i'hc time of as::cnt. with constant velocit-y,
l • ( l J) Ex. 23. Prove that tbe shoriest ti1n.e .from rt.st ra rest fil whlcli
=v+·! v ;;: +.B • . .. (3) a steady. load of P toqs can N/1 a weight of W 1qns (hrougli 1.1 ,wi:1.
Let t·denote the :total tl'tne· occupied whe-n 11.-.:,/3, cal distance bfeet f.r -i{(2h/g:. P/(P- W)) ucon'r!s:
Then p~tth11:1 "-"!'Phi the 11,bove result (3), we have Sol, The time ·will be shortest if the load nets contin.ooi:sly
J v " dt . I l durlng the first part of the as,::en!, Leif be the n.c_cele'ration dur{ng .
r=--.+-, Ther~fore· -d1• =·-- ,,z +-· the Rrst part of the ascent. TKen by N'c.wton's second law,,of
V rJ.. ·
. O:

Fo~ !~as t value· oft, we ha Vt: 1// /d1•= 0,


/
-i,i+;=Q
l
motion,/ is given by
p ..:, W=(W/g)
.
J.
'1, ..
1 ... ,(I)
During the sccond'put o:· the ascent, P .ceases tolnct and W
I
I.e., j-:-=;;:v .
1.e., 1·=../(/a.). then move~ only under graviiy, Theri:forc t/1e re'tard~,don· i~ g,_
Let.~ andy be 1h1: dis.1ance.s and 11 , lrthe-corr'espo1\dfog
Also then the t!m~=2 (h)= .:;~{,, )
""'2..;(lf.,,.), . This time is times for the two pllrts in ,the :i~cent, I ' . '
of
·....• . . '
,r,; be !he velocity Ill th~ ~nd th'e first f)l!,rt o.fthe UCC!}f u,·.
lea.st bec:ause d t/dv'=21/v' which. is positive for v=../(1,:.).
2
a1 the beginning of thi: second 'part· of_ the ascefit; wC'havc then

'-==::=~;!lilt[ t d -

•~ :•:,;,;;~~mm li~, ··~~·=,:\}·: ,,,;::


,,,
.. :.:,

,., ...... ,. 1. ,;'·.,•,,1q,

6&, Dy11a1i1ics Rectill11ec1r MoOoi.


2
.69
v . ;=2f.x\ , '} Q
V=fli, ·,,,· A bullet fired Into a target loses ·half. ilJ'
. Ex, 2S.
'[Eqll'ations for the first pa~t· o( the :i.scent] after penetrating 3 cm. Ho111 niuc/r /urt/icr.ivl/1 ft penetrato·? ·
and· y2=2gy} ' ' [Mc~rut 1972)76, 79S; 83,,SGP, 88).
v'==a1, I ... (3) Sol. 11' u cm./sec, is the initl~I yeloclty · or thf" buflet t~en
J . ' . its velocity ~fter penetrating 3 cm. Will be ;i1.i'cm./se.c. ·
«L .lE_quat,ions for the second· part ·or the ascent!
Le.Jcin,/sec 2• be the retardatioi\ o,f the bullc.t.
~+
' . :.Als'o Y.~(I (giyen). · , Th~n· from v2 =u2 +2/s, we !tiivc
'
. , f'.r;'om;,(2).11,1\d (3r, w.e get
. ·v~, . ya. (u/2) 2 -!1 2 - 2. /.3 !l,iving/=1,NS, .,
.. 2c~·2":'x+y Jf the -bullet penetrates rur1l1er by a cm, then 1rroni v•"".u~+2/s,
:.· .. g_ . :,ve have .
1 l) =_·II, ·.
·v· (
·2 7·:+g . ... (4)
, 0-= lu/2) 2 -2. tu2/8),a •
a= I cm:
·Also
· "
· · · \i ·v ··, •
· : '., .f ' t-;;-"l" 1 •/i,,
+· Ex,. 26. · A loaa Wis. 't'o be i'aisltd by a rope.,.Jj:.om re.tho re.st,
• Q ... (S)
-:Now'. the,.tOtJ\Lti~1e o.f asc:en~ ··.: t~rougli a {lalgilt h; ·1he greatest tensl'Qn wl.'tct1 1/ie.. roj)e · can &o/ely
· . ·( I I)
=t1+l2= - .,.- _v
b.ear Is itW, ·Sh.div that the least t1'rne l11 1vliich (ha asqem' can be
made i:S [211h/(1i- I) gJl,' 2. . ' (Mee rut 1986] .
'' 1 g .•:
Sol. Obviously, thll lime for ascent is l.:a.&t when, l,he llccclera-
=(.fI +gl)J[· 211'/( ·1 l)]
.. 7 +; [from (!4)] tion of tlie load Is greatest. lf 111 is the mass :of• the .Joa<;f ,. then

=-J[ih u. +¼)]~.)[;/!(;+I )] W=mg or m= W/g, Let /be the greatest acceleration of'the
!oad in· the u·pward direc1Jon. Siltce the rope can bear the treatesJ

,=
W.w· +l.·)1 .' · ·
. J[2hg (F=' [from(l)1
tensi.c,n n,w; therefore when/ is. the greatest acceleJ~tion·.. of,.the.
load, then· the tension T~o th'e.rope,-is'nW.. .· ,
.'. by .Ncwton's's-econd law or
motion P=ntf,· we. li:a..ve, .
·l . =)[¥. ;~w} Y · T.:...W=nW-,W=i11/ or /~(n-1). (W/m)';"(n-1) g,. •.,•(l)
Ex. :u, Prove ti1af.',tl;e mean· klnetic · ehergy of a par11'c/e of Let the load W'.n1ove upwards up_to the he/ght)11 ,unde.r the
mas.,s m.. movlng under.a constant j'orce, in a~y lnter1°a} 'O]tlme acceleration/, After 1hat th~• tenslqn In the rope .ceu:s .to act
and thcrefoi:e o.bov,e .the. height ill' th'e load 'will move,undet
. tm ,["tt+,u111,+~ .. ll'ht're !') cmd ui are the fititial
itil!'J', ·: ,• . .
ard gravitylwhich acts vertically do\~nwards. lf ..the· load.,'.com~~ 1,0
rest aflc,r movlng through a subsequent beJght '11 · above . the
~ill.· Let the int~rval or tirrte during ~•hich the piuticle moves height hi,. then according to the question
i)i}t·rr the par\fcle moves .ur..der a c-~nstanl acceleration/ a11d· v hi+h'/,=lt,' ... (2)1
·be't vel.oclty il.l':any 1ime t, ·we ha.ve·v~u1 ft. .
l f V' is tlie-maximum velocity of, the load ~qufred at the end
· '.ow the·tnean. kinetic ef\erg'y. of the par icle d.uring the time T
'fr .... . . . mf.1; . • . ..EL 1
.I
oft ho lit'st part' a.nd r,,, .': a r~ _the timJJ. taken for. ,d~Jcri.\?lryg the
i
l.
.1.
r,;~·: 1mv' dt'"'.n 0·(~17/1JU_d'.~2/T'.,3
0
· V=O+f11
:, ti= V/f
and 0=
.:;nd. · tz~T/:1,I• :-
v-it,. · ·
h1 and, hi' respe'ct1vely;,then ft'orl\ v=:=u+/1,_.we hu~e . .

i ~.~[ (uit/7:tffi ],;,ei c~\-u;)'(u?1~u1 3 1


)
Also from v~i11 1t.2/J'; we '.huve ., :,-1;~. ·.,
I . -~ . ; . :?:,,:i\').~'~7\' . '' :·r·:·;.i.:;lii.;;'1i{'g:fJr, nnd so Uz~J.11 =IT] v2 ;;,.0·+211i1 and 0=,111 -,2gh,. ..-..
I
,. =.m(u1 . +u,u~+ud, : _ .. v:. V 3·'•
.. '
h,=21.. o. nd "/1,=2.g'

I'
i;,···

:ji;{'t'i:::
•:1-;.:•:,:,,,:- :, ·:$:;;~:•.h<

---------'t""""T"~:",.,..v;?;~:r.:~
l ·::-.':

.. ,,,,,:,;,,, .. ·t,.-.,.;, ··>>:•:',\: ;;:/:;},: ;: :,.'::::~:,:::~,., .. ,;;,:;~:·~i;,,., ... ,.......,1..1,1~, ...,,,

70. '. t;, vn 11mlt; s . . n \


Now •C:ro.m'li,+.l11 -h, w,,; have·
1/li!ctiflnear Motion ·.7'\.
'\"'•
v•. vi vi ( 1 . 1)
2/ +4g, ""' 11 or 2 7 +g i=:/1.
Ij
I
P"7l,'whi:11, m=l' and./=L
:. from(!), we have k=I;
• ,

: • . V.7"../{2lt/(l/f,- l /g)}J ... (3) Hence .we .have; P~mf, whi.oli. is .the required 'equa.tio'f1 of
:·,: ·the (ell.$t.tlme. of nscent moti.on of the particle, · · .. .
I •. V. 'V
=l1't,t~=f·
. g
,.__,.,,,.y•--,.,--
( l
J g
l) § 5, Simple· Harmoalc r,,,f.o:ik>'II', '(S,HJ\1,): Deflnftt~ The,
kind of moUon, in which: f)(!rtic!e move:diz· a Nra'lgli1 lute · in s.u~h
J{ (11,/+lfg) 2'1 }. ( l .J) ll way that hs accelera}io11 fs ~!ways directed t;owards' ajix(d J?1fn~
=. ' 7+g · (substituting for Vfrom(3)] on the line (called the 'qentre of for,ce) ,an41 varies as the. dls,/11nce. of
=J[ 2/i { +½}) 7 ' · the particle from the.fixed' point; rs 'called simple ll'lf1mr:>l(fl! 'ltt<>ffo1i,
I , l}]
,J'
"" J[ ~hl,(n-1) g+g
[Mcerat' 3!,16; 78, 79, S'l, 82·,:&S, 8/S; l(aip111·11; 11,
. · Luckn~ff". &of Agu '1]'
•,•

[rnbstitutjng for f from. (l)J Let O be the certtt'c' bf force 'taken a'S origin. Suppose th'e'
• ==·~···--t••-
'':mh
·•i ,(n-'l,)g ·
]JP. particle starts from rest from,thc po'i,rit A: where· 0,1.=;,ct, U'oe:gin
;i

§.3, '',l'lewton~ Lliws,:or Motion, (AllabQbnd 1979; MeHui 81') to. move towards the oe1ttre-of. at.trndtio& 0, ·Let P. ~~ .thi p:ooitio
of'the' par.tic le after time 11, ·wheFe'. :0.?=x; · ·sif. the; definition o
T~e Nii':'ton's ·Jaws of m61lo1t.are as follows; · .
S,H:M. the magnitude of ao~ele~qh atJ:>)s prop'ortlonal to x.
. :.l.ll.lf ~. ev~r.y body .co!iun,ues in It's ·s1at,e' of rl!'it, ~n of 11111/orm.
. ·mot-t~n- in Q ;tr.alght line, unlius II ls compell!Jd b';'·,some, (!X/.drnaJ. ' I : '.. ·· .:f.Q
• /<>~Ce: or /orc~s. fO, change Its stale, , " ' ,i,.;;, ·. v<;t. 0
A' ,) ..;,. ·,A
. · 't,~w .~.. ,Th'e,• r.ate of.chang~ of momentum• of a bodY, ls pro- ;if'
portional lo. the lmpressa'cl force, atfd lakes place In rite dirac1fon. Let it be µx, where'µ is a. ~~nstant pa11~d,· th·e lotentilty ot,ror.c_e
ln'whlch the/orce acts, · Also on account of a cc£i'tre o,f actraetion at 0,: the a:eoofetat10
.. taw 3, To avery,.act/On· there is an equn/ and opposite reac. of pis towards' o, i.e., inl the"d'irecrion. 0f X decrensmg. ~r:er~r

,
tibll,, . . ,,
§ 4. Equatton:, of notion of (J· particle: moving fn a straight
... .. . the equati,on-'Of motion ~f:P. is .
,. 'd~x
. .
.
. . ·
. ..
hne as deduced f.r,om ihe f.le1vto11''J'. Sl/'tOnd law of mot/011, ' . ld12=:=-µx, . . . ..
Lct:'v b'e the 'veJocity'at iii:ie Io( a par~icle of mass in moving whetc,tbe negative sign ·has 1:ie.en,tak~n,because tile' l"orc0 .a·otl.n
in a, st-ra-ight lln~nder ,th\l'.acton of t~e ir:i:lpresse_d f~rce P. Since oa Pis townrds O I.e., .jti c~e d,ireetioll' of .x dec'rt:asirig. ! ~1, • 1-1'
I
f:trom Ne:wton's • sevond, lnw pf. motion ~he rate of change of equation ( l)' gi,ves, the acpilera:tlon of the plirticl'e ftt 'any, p~S'iti~ ·
momentum Js·'.prop,ortio/J o,I to 1he.· impressed• force, therefor~· .tv1ultiplying both si~es. of (I) by ~dx/dt, 'we get · .
d \ '· J · \ I dx d,x. • dx. ·
P cc, Ji (mv), .('.' · b,v de-r.. momenium= nlass x velocityJJ 2 di
d1 1 =-Zµ,xdif,'
_or P=k .d~ ~mv),·"l'!here f is som~ con~tant lntegrntin,g l~ith resP.ecti10 1, •we ge,t

or
dy
i'=~m dt provld~d m (s,constant
I·' Yb(~) . -,..xf+c. ·'r
. where C is a con.stant, ofintegrat'ion· and v· is the ·velocity at P.
or P=kmf,. ..',(!) . Initially· at the point~. X=d1' a11d v,=0; therefore· C=µcr
l..e\ us supposo,that,o.. u,nit force is that which produqes a
( •: f = a.oce!eration =dv/dt). ., Thu:;; we have
.
,• · .
(I:'·rr,.:': IV=-µxl+p.al,
..
unit u.ccoler'atlon In n parilcle of uii'lt muss. Theri · y2='
or .. v~':"'I'-' (a~.:.:.;,,; 1). .•,.. (2)

IIIIIM6il''llfl I ·r·: • 1-·,

:, ,, >··::')•, 1, ··•:.·1 i! '. ~, ,: ,,·1~n;:,.i.mm6~t~;t~:1rye ;'-,. : · ,,.:,.·. ,}:·· ~f,, ·NlJ""}:-:,):,:H>, 1.•:,,:.!••·,:···•::-:,,:,:1•
'!Ull[U!UU!i ·:; ;f;~,tUIWUtJnnmHl~ih; ;,~ ~i;,{~: ,;H!i~~~~:, :.-~~;w;}ii~~H,~~H~m~iil-i•... 11::;,:•;,.:, '~:{;,;,,;;, '·.:i·1,W(~~~i'~HW.*thim;,:<)il ~•mi 01, I':·
f 0(/,/i()!'' ,,c,\,, ,,;, ., ,;


· - - ~ - ~ ~..-,------:-=-r-;:::-·:-~-:-::;:.' .• ·::--~~·· ":::.:..:.:.....,..:...- ....,_ _ _ __

72 Rec1i/i11ear Motioh 73.


The e'quation (2)',g[ves.,the v:elocity at any point P. Fi•oin (2) or attraction at O u force begin~ to. act upon the particle against
w.e observ.e that vi is: maximum· who1\. -x~·..;o or ·x.-... Q.· Thus. io a its dlrection of n:iotion. So its 1elocity g01:s on decrca$ing a11d it
s:H.M:,: t~e. ~elocity l~ ma'xl~ut,n ~t the. centre of force O. · Let co:nes to Instantaneous rest at A' where OA'-OA. The rest,at ;f
thl_s:maxlm1.1r:v,e!~c;it~ be- Vi, 'Then af O; •..:,s,()•; v=v,. So from is onljl instantaneous. The paniole nt 1mee begins tolmovc towardi
(7,J We g_ep1 .:c=µa · or V1"":avµ, the centre of attraction .O·and retracing its pnth it 11gnln comes to
\· · ('-\sO: from: (2) :we observe that · w.hen x'=a' i' e., x::.:. ±a, instantaneous rest at A. Th.us the motion of the particle is oscll•
Thus In ·a.·~.H-M,the 'velocity Is- zero at pJints cquidi~tanr- from
the·icentre 9fJor~: · , I• latory, al)d. i't continues to oscil.late between .A. nnd A'. To start
is
frc-m A and to come back to A called one coh1pfete oscillation,
· . blow. .frgm' ·, (2). on la~ing. Sqf!lrC root, we get
v!ew Impo,rtant Definitions: • ·.•
a:J:Jf.:-:v.µv,(~~-.x 1),. where· th_e-,-lve ~ign ,has been taken. be• L. Amplitude, lu a S.H.M, the itista11ce fronr the Cl!!llre of
·ca~~e at:.,!?- th:~))articlc !S moving. in t~c: .d1rect1on·of x decreasing,
S,~j)atatlng,}he ·var_iab!es, ·.we get J force of the posllion of maximum displacm:a//1 Is calf~d the pmpll•
. . · 1 .. · .dx. ·. . ,:"de of tl1q mo.liori, Thus the umplJtude is the dist,al\CC or a posl•
. - y'p,-,✓,(a•-x•) =, /r. •
t .'.lo~ 06 instaritaneous. rest 'f~oni the ceni1·e of, l'orc~. ln the · for•.
mulae{2) and (4.) of tJ1i~ a1·ti~lc the arrrnl!tude is a·, ·, . ·
lnteg~atipg ·bo.\h'sldes,•~·e get i • 2 .. Time perlotl, [Kaopur. 1977]. •4.1 a S,H,M)-11,e //me taken
·l .. X . ., .
.· ··· -~-~os~ 1 ·a~'rfD._wh:er~ .e;is:iii c,instf)nt. 0

10 make one comp/e/a -osclllatio11 is cal/e;d" 1(Jnc period Qr ii1rlodi~


rim~. Thus if Tis the time period of'the•S,H.M,, thcri ,
\.' .· ·1~ut.inltial_!y at A, x=:a :;.n1 ,;;;o;.theri.:for~. D'=O:
T~1!f-•We ~ave . .- . . 1 U ) =4. (lim~ from ,.( to 0)=4,
2~ 1~,;._ ~µ•• 41ich is lucl~pendcnt or
· :Ycos-:'
·VP,
~=,
a
'or .x=a··cc.stv1!t).
I
<·')
.,, " tbc amplltude o, i[Mccrut 19S9J
3, Fre(Iueocy. The 11umbar of complela o:Scillatlo11s ill zo11e.
1
• 1 [Lucknow 1978)
l ., "Che· equatlon,(4'i, gives. o. _rel~t h.m\:i~twe~n ;,; and /,. _where· 1 :is
· ,., 'the: .. ime ITIB.asured fr9!Tl ;j. l( ti:be, tJ-\e time 1from A to O, .the.n at
second Is ca/lad the fr<:queiicy of the 11101ion: Si rice Che ltlmc take!'\
to mak~ one- com"lete· o~cillation is 2 " .st~onds, therefore If 11 )s ,
:·'c,1.,. e h'~ve t=t, and>:=◊, · ·.So from·(~), 1
w~ get 11=. Jp,,co.s•t 0
.-.
the fre~uc11cv, then-11.~~ ~! .or 11;,-:(i•,
\ 1 I' ..
.
·. . .

·1 11· 71 , .. .. • · . • .-1: . : .. ' • ~µ '~-


'.'i .'"'· 0 k ~=ly!,1!,' which IS_ rndepe~dent of the llltl1al d1$placem~n\ Thus the frequency is the reciprocal of the periodic lime.
-.a,of -h.e.particlc.::P/ws In a S,ll,M, the time of descent 10·11ie centrf Important Remark I. Inn sit.M, if the centre of force is,
·'o/,Jn c:e is.,f,;4~pemiel_l{ of the f1ii'tfa!'dfsplace111en1 o/the par1ic/~. n-:i1-~1 origin bu't is_atthe point x,,,;.b, then Lite equa.ti9n of mot1ion'.
· ., : . . · _:.. · · ./ ' .· lM_eeru~ l984, 85] i~ d"!xjd,~-= -µ (.~ - b), Simil~rly d~x/dt~= - 1, (x+ b) is the equa•
1 also be found from
· · ·: .N te ..The time of·descenr 11 fro.m A to•O ci.\n 1i-~n of a S.H,M. 'in which th'e' cent1!c of force is. at Lhe point

, (;): !th thd hel; 0~ the_ defin:it_e i~t~i,r,Bt<)~J: . . . II· .x=r:po.ri~nt'Reniari,'2. ln the above article' whe~ after hman•
13.)lCOUS rest IHA' yie. par:iclc peg!ns-to move tOWHrds A, we have
Fort.xJng· th.7 Hinits of i_nt'egrat!Qn, \,;•e 'O_b1-e1';ie at x=a . . h . .
. and·· =0 W:h1lc.at O·,.x~0 a11d·t=li, · from (2) dr·=+v'µ,v'(a~-,x 1),' •
, '. _atur·~ 6f :r,-~otl.oo: 1"he-,pa/_t.icle sr~i;ts fr9ln rest at_;1 ,~jwre its
accelerat\9n.1s max1mum-.ind ·1~ µa to_\\>ards O. It bcgms to ruove wher~ the ~ivc sig11 has he~n ,la~CI\ because·thc parIiclc i~ Inov•
· towa:rds .ih·e cen(re· of.attraction .a.and.as it a pp roil.Rh es the centr~ _ng ln the dir~clion of.,· in~rcasil\g, ·.,
ortorce ·o, its .velocity goes on lncreas_ing. When the particic . t IlC vann
Scpar~lrng d,,.
' bl cs,. \\'C IIll Ve· v(<if-,ilj"·· v'}'"''
·'
· reaches .9 its acceleration is ztFI a:.,d·_j :~ \le'lo~ity is niaximU!ll and
l~·ayµ.·in. th~·AfF;~~l?o(OA', .uueJo.;.this;:y~J.99ity gained at O the lnteg.rn!ing, we·ii~t ..;.cos~ 1 (x/a)-V)"i'+E, ·'.·N(lw 1hu 1iillQ
::om .'f lo·,A',.i.s 11iv·µ, Jhercror.u nt ,1•, WC lrnvc /s.,r/-./~· and .... ·: .
,. , :p.arti_cle 1uovcs:tow,a.r9s t_he I.en ot:o·,.:But on account-of the oentrs:

'~! t:1
•."'"·

ll 1:.

~
,,,J ~ .~;,;,,r.•.,7~i:(f;:~:: ·',. "-~,-.--:"'\'
Ji :~·•:)~'>.::::..,' ···~::\":f·:,·
:11 u, ._; .~H-.1t/,:~\ •.o•·:,,•·,v •:•' •·• ·. ·;~, · , ?:\'.:i'H~i':,: ·;: ., ·.·:::~.•.,-:i~~~;:y: .. ·' •:·i, :>'.: •:\::,,• .. ' ,;::~~::,~· '.- ' . ,, '
·>·'.·,:

!l ------==•~.,-;:,r,•~;.~•~:iV,~
,','(••:.;,•:, ,,:;;t;i;t1i:,,

~
lj

~
;'

~
j .. '7if R:~ctlllnear Motloh ' 75-:
.....-~:
D;,mam'lcs
! ,I!-. -c. These give -oos- (-aia)-v'I-', (rr(,y'p,)+B
1 lf OQ=.x, then
or -cos- 1 ('-1 }=,r,-:B or ..:.,,_rr+B o.r IJ,... -2-rr. ·Tb11s -we- have X=O COS wt, ;,(!). ·.-l
-·cos- 1 (xfa')-,Yµ, t-2rt,Or COS" 1 (x/a)•2rr'-,vµ.i As. the particle P moves round. the·. 9ircu:mference, .· the foot
or~= d· cos (2'11'-,,y'µ,t} or x=a cos yµ.1, Thus in S.H,M,. the Q of the p~rpen.dlcu.lu on the dta:m·~ter AA' oscillates on AA.'
~ci.u.11-1ion; x-a oos ,y'µ.t is valid through 1HH the entire motion from
A to A' apd back from ,f co A. · ' · · .
from A to ;4,,' no.d· from A' to i baek •. Thus the· ino~lori
point Q is peri.odic. 1 ·' '
the· oi
• ~
4. l'linse ll"nd Epoch.· From· eq.uation (I)-, we have From(!), we have
d 1x · : . dx ., ·,
d/Z+;.ix=O, ·d/.=-awsinwt , ,'.. (2 )' '.•:
which is a !iMur diffc'rerti·at equation with. constai1t coefficients d'x , . · r
,lnd its generar solution is given by
and 'iJ"i'i" = -aw ~os wt"".-w X, ' ' "' (3)
· ·x=a cos (yµt+.(), ... (5) The equations (2) anq (3) give l'ue· ve_focit:,' and acc1:!eration ::
• The constant• is called the Rln.rtlng phnse or the epoch of th-e of Q at any time r. . . . ·- · .1;
moqon and the qu·anti'Cy v' µI+ i is ca tied th~. ,argu1heut 'or the The equation (3) thows that,Q executes· a
molron, motion with centre ut the origin 0. F.rom equatio11.(l), w¥ see
;{•·
The.pbnsc at an)' rime r of a ,S.H.M . (s the time 1ha1
a/upud J'lnce the'p"artlc'/a 'passed 1/iroug/: Its e,rmmie pos/rlon In the
I that• the amplitude of th.is S.H.M,.: i,s a because- t,he- maximum
value or x is a, .' ,,:' · ,.
POJ'l/l've d(rectlon, · · · ·
: Frein (S), x Is maximum when cos (v ;<I+•)" ls. maximum I.e.,
. when cos (yp.t+•)-1.
The perifdic ~ime-of.Q=The}")ime·. req~ir.<:d by 'P _to. 1u'rn
·. ihro'ugh an angl·e 2,r with. a• uniform 1
'/,
· Thcrcfo're if 11 ls,,1he time of reacJ1ing the extreme position angulaT velocity w' ' ' -· [,,
in the positive direotlon,· th,e n. 21t·
=-·· l
cos (y'µ.1 1+e)= I
e
w ' . ' '
'llius if a parlicle describes a ·l:J~cle w/1/1 co11s11mt . angula~
I
.l
.!
or, v'fal1-+e=0 or l1=-·y1;
~elo,ci,ty, thf faot /f the p.erpentllc11/a1• /r-om' ir on ~ny._ </famrper -
e,tcc:utN a ~.H.M. . •
the phu\i at time i~ 1-ti=t+:7::....,
' ,,Y µ._ § 7. lmport~ot results aboue S.H.M. r
. !
~- Pcrlodlc Motlon. . A point :said tolliave a perl.oa1c motion
(1 We summarize the important relatl_ons· of a S·•.H.M. as·
when it mo~es /11 Jw:h a manJ1(!r 1/:ar after a cerrain /1':ud lnrerµa/ follows : (Remember them), '
t

of time cal!11d ;nriodic ri'11ie./f acqu(res the same pof/.lion and movqs (i) Referred to the ce1H.re. as origin the equation of S.H.M. ·
wirli the J'ame ve/oC'lt)' i'n tl1e sa1i1e, direc:tion, Thus S.H,M, · is a' is Ji= -1,x, . . ·. , .. ' · .. :. I
periodic 1notfo11, I
§ 6,'LGeoo1etrfcal n!prescntatlon;or S.H,M, .
l '
( ucknol'1, 1975] -
or-the-eq·uation.x= -µ.x represents a S.H.M, _with cenir~ a~ the -
origin. I · · ·
ib,
Let a part·icl~ mov~ 1vilh u uni• The velocity Vat a'dist:1,nc~-x from.the centre ~nd
form ~ngulLlr velocity· w refund distunce x fron1. the ccntre,.~t time· I are ~\l~pect{vely, giyen by· _
circumferencll of a circle or rapius v•=,, (a'-::x.') and x":''a ~o·s yp.1; ~• .
SuP,poso All' is. u, fixed diameter of where a is the arnplitudc'und·the time I has been m_easured Crom
the extreme position<l·A tlie .positive ·direction,
the circle. 1r the partiole stnr1s rrvm
· A 11nd,P is its positio1t,·-0.t: time t, rh1rn
'r-- 0
11 ~.;. d IA (iii) Mnximum·accel~r~ri_on=i,ta,
· ·
. '. [~t exireme polnts)·
L ,IO P""-' wt. .. .. ··:, . . ,, · Ma)liinum veloci_ty= v' µa, · _(at the centre·
Draw PQ perpendi6t11nr to the ·' ,
dl1tmeter A.{', Periodic time ~""':;}-·

,0

~.:::-:,:;.~·"'••'·~·, "'"-"''-'~.'.to,·•.-,.•1.-.~~~
·,:1,'..'n~• 1.', •~••. :·,.•: :-., '
,,:,; .. ,,-:.'

7~.
Motton · 77
.. t . ,. Dyna1h/cs ~ec1/i/11iar

(vi) .
Freq11e1icy . I ·,/µ
11=~= - ·, .
T Diff.er~1,tiati1\g' {i) ,v.r . t. I, we•g~t
·2
r,
Il!u lraHve· Eb.mplcis, dx d3x , · ( dx)·
2 di, 7t,27l~1 --2x di .
• . · :<, 21, . ,J'/1e .m.ax/;nun1.'·ve/octtyo/ a body ml)ving with S,H,M, dzx '
Is 21(,/scc'. andkr' p~ri~d ,;,,·~iec, .Wlia, Is its atnp/l(,udii ? .· "ai•= -k 2x, which is the equation of a S, H.' M,
.ol. Let the.·amp!itudc he a ft.· Tllc.n the maximum velocity j centre at .the origi_11 ·a11d µ.=k 2 , He.nee the giv,en motion. Is simple
":"a µ. ft,/sec,";'2•h./sec, (givc,n).. , harrrionic. i •
· · ·• :. · a-,,lµ":°''2'.;_ ·· · . ..,(!) The time period T=2rr/v µ= 211/,J k1 =21r/k.
!so. the time period T=Z1r/,/µ ~econdsb-} seconds (given) N•:>W to find the _;i.mplitude we. o.~e tp find »th! disir,.nce ,from
. . ·, 2,,. ·1 I th_e centre of~ poi.nt where .ihe velocity is .zero,. So p,utting
. .. . .. v'p. di/dt;,,.p in (q, ,we get,O=ti (a2 ..... x2) ?(x=.±a, Siry.ce liere \he
centre is.at ongin,·therefoile the amphtude=a.
....
Multiplying_(!) and (2) to elirninalb µ,•we have I "; (tj .i ·
- R'<: 30, Shmv that If ilie. displaceme.nt of a 'particle /11 a - -
:.!r,a=l •-a=},;• . stralgnt line Is expressed by the eq,11ation x=a cF·nt+li sin 111, It
desui!u~ a simple ham101:1a motion whose" amp/i111da ls ,V(a 2 +,b1)
the requ'ired arnplitu~e=t, ri.~•Oo4· l't. near/,v, and period ts 2r,/n. . , (Moarnt 1977:J
Sol. Oiven x-a cos 111+b sin 111., .. ,(\)
1 E":, 28 .. . ,it what. distance fro111 the centre the velocity (11 · dx/lil=-aH sin nt+bn C(l~ 111,· .... (2)
S./f.M. iv/II b,e Milf o/ the 1nax(mum 'I •· • 0 and 1 2
d!x/dt =-an cos ,ii.,. b11 sin nt.,. -11 . (a cos 111+a .sin 111)
1 1

SoL Take 'th<i cont,re of t;ie ·motion as or/gin. Let a be the _.,;,-11 1x, from (I).
a
ampli1ude. ln S.H,M., the velocity v of the particle at· a dis- Now d 2x /dt 1 =-n'lx is the equation of a S. H. M. with cc11tre
I t.:111ce x from the cent'r~ is tivcn· by. . at (he· origin, and µ.=tf, Hd1ce tile given motio1\ is simple
·'•. . •, .. • V~=µ. (a•-•,>: 1). ... (Jj · harmonic. ·
from (1), vis _in,ax, -:,vhc11 x1=0, ·TJ1ere(ore.'max_velo<Jity=y'µa.
1 The.tim'e period T=2.,,/v'µ.=·'2:rr/v'111=2.,,/n, Also the e.mpli,
· · Let ·x1. be th.e di~t-an,c~ f'iOl'!J the ccmre·or th~. point wher,z. tile tude is tlic distance. from the centre of a-point wher.e ,the vclocilY
velocity is halfof1he,'ri1axjmu_m. l.~J, where the Ye!obity is tav'µ. is.::c~o. Sipce lier1 the ce.ntr1:; is a; origin, therefore ·the amp!Hude
T/len put'ting•X=X1 and v==,a,/µ'ill,(l), WC get is the val'l}e or' x when af.;/dt=O. Putting dxfd/-.0 in (2), we S(lt
· •,! a~/(,;,.;u..'•(a 1-:;-ixL 1), <•r i• a1 ,;.,.a 1 -x, 1 0=-an sin 111+b11 c<is 111 or· t~·n nt-b/a. ·
. . 3 I .. . ,
:. • s.in nt,=b/y(ae+b') and· cos.. nt=af,V(a 1 +b1),
01· ' ',l,"1
1
"=• :·.·or •.l;";=±av3/2. Substituting these in (I), we have ·
··.Thus ihcr~ ore two pointt, ea~h-at a dis.ta nee ay'J/2. from the . , . a .' ; b a1+1iz
I; the amplttudc=a V(a~+bi) '.1· /J. :.J(ci'iH'if "",V(al+•bl)
· centre; where, -the.'veJoci1y'ls ha!f of the maximul')'l. .·
. El(, '2!), ;:q)Clriic/11 moves /11 C Slralg/1i li'ne and ·il.s Yelociiy.. 0/
I . . ":'V((ll+•bl), '
a dls1a11ce ;-:'from /(w orlg:i1 .'s ky'(a~.,:.Cx 1), w/Jefr a and. k. J¥1re con-s•
. r::x. 31. The speed. v of'a particle .moY/l!g, · along' tMt axis o/ x
.10111.r.' Prov~ /h(lt· the motion :~J' simple luumonlc and.fiiid· tne ampli,
Is glve11 · by the re/(lt/011 v1 ... 11 2 (8bx-.~ 1 -l:ib 1 ), Show' that tlla
111,Je tlllij-rlie periodic //me of the .motion. · \ 1'10//011 i's si'lnple harmonlc-1v/lli i'l.i re/I/re 11t x•4b, · and amplitude
=:i.b. . . .
· Soi."wc know that in'a rectili:icar motion !Ile expression for Sol. G.iven v2 =(dx/dt)'d11• {8bx-x1 -1·2b~).·
ve'iochy. at'n distance J.: from the'origin is d.1'fd1.: so·accordin, to Differen.tiati,ng (!) w.r,.t. r, we get ·.
the q~ic$\io,11, we have_ I ~ dx d2x. r. b ,.. ) d.~
' ' •. .;•:·. ·,;: '' d ; 71 = II 1 (8 •-.;.,.~
ca~:..:.•~•>;- I;::/·.• .. :.· , .
I. ". { 1) :\

:: \~_1\. ~:';.:' •!•·'.'~' ,., . 'l;,'::·;;/'• ': .';;::~ ;. ~' . .


,' '•;;J{t~i
",:•,:.:,,
::·'.:~'.·1:'.:: • . ::',.':•~:.-::::· ··:.::.::. ,.

•• . . - - . - , . ~ ~ ~ , - ~ 1 " 1 ' , ; 1 ' 7 " - ; : ;

•:,._::~;~:;:·t::,>·.
. . ..
.. '' . .. .
.' - - ~ • . • -t...,, ....... _ .... ..__ ••

··1',:,•,•:,

•i
;1
'.I
i~

.,,J
;,;
I••
.•,n
::i .. 78
Dynamics Rectiltn~ar MoliOII
i:j ·,
1
~}=n (4b_:x)=---ni•(x-4b), w)iichis·the ei;tuation Ex. 33, In a S, H.1 M, of perioi 2rr/w If the laltlal
men/ be x 0 and the in/ti~/, ve/Gc~t{ u0 , p:ove Iha~
of n S.H,M, with centre 'a( !he poin.t .i:-14b=O f.e., at the point
xc4b. [Note 1hat centre is 1hc. point where the acceleratic,n
d;x/d,t 2 is ze.ro.J · ·
(I) amplilllde=J~ X02 +~.)
Now v=O whe're 8bx-x 1 -l.2b 2 =0 I.e., x 2 -8bx+ !21.i'=.0 (II) position a/ time I= J( Xi
2
+~)-ros{ ~i-ta,11,-I ( : ;
0
)}•

t,e., (X••6b) (x-2b)=O l.e, .-r=6b.· or 2b: Thus the positions c,f
instantaneous rest.are given by x;,,,·2b and .v:•6.b,. T.hcrdistunce of
and (iii; lime/() the position of're;;·:L ran·! '( •Ux•·, )· '
. · W W O
nny 'of these two positions from rh.e centre X=4b is the ~mplitude, Sol. We 'know that 1n a S, H. ,M. the time per,iod"';'21r/y(µ) ..
Hence the amplilude.is the distance of the P';)int X=6b from Since here 1h·e time period is 21T/w, tli'erefore 21'1/y(µ.)7:21'1/w
the ooint X=4b,' Thus the amplltude=6b-4b=2b, '
1.e,, µ=v,, ~ . . . .
Ex. 32. tli~ spaed y· of the point P which moves In a lfne is No1, taking the centre .of the motion origin, the equifti9n as
gii•en by.the relation v1 =a+ 2bx-cx2, where :ds rhe· disrance of the or the i;iven S. H. M. is .
. d 1x
poln(·P from a fixed point 011 the path, and a, b, care constants.
d1~= -w'x. ...(1)
Sho>V't/iat t//e 1/101/on Is· simple ,harmonic If c· fs positive ; deter•
mi'ne tlie. period and 1/le amplitude of the motion,.,. [Knnpur 1979] (I) by 2 (dxf!/1) and integrating w.r.t. 1 1',.we get
dx)I
· ' Sol; Here given 1hn1; 1• 1=a+2bx-cx 2 • ·:.
.l)Jfrer~nfl1'1tlng both sides of(!) w.r.t. x,'•\\;e have
• .. (I)
( di '""'-w ~-.~.~.+.A;· wh,ere A is' a constunt-.·.
2

I . dv ·
2v dx=2b-2 c.,:. initially at~ the nloci;y ~=!fo ..

b )• Therefore Uo 2 =-:-w 2xt-A. or A;,.110i+,u 2x 02, .


d:x
-
<Iv
1 =v -=·-c
·(
.-:-- L. Thus w(_have · · . . ·
rlf dx · c , .. ('2)
· · dx 2· · · , • ·, · ul ,
Slnce c is ·positive, therefore the equation (2) represents a
S. H. M.·with the centre·offorce;at the point x"7'b/c, · cn) .=:""w•x2+uot:-w•Xa2=~• ( Xo,'+ w2"."'X2 .) .
Hence the relation (I) represl!nts a S. H, M. of period · (i) Now the amplitude is ~he· diii'1ance from the centre of a.
,2,., 21T . . ..• ' 0 point wl:ere the velocity is zero. Since here the· cen'tre ts origin;
. T=':7 .., . 1 , becaus'e In the. equJion (2), µ=c,
yµ,-vc, therefore the amplitude the· value 'of~· when velocity. is. zero: is
'T'o determine the ii:'.rriplftude,:putting v ... in (1), we have
:a+2 bx:....,rx'-=O Putt.ing ·~=0 in (2):i' We get ,'l'=±j(x +~:): 0
2

or ,.r.~i-2 b.~-a-=O.
'. b±·./(b1 +ac)
.~.::;:er .......
- - - - .....
Hm the required amplitude is J( :ro +,·
1 110
w
:)-'
' (' Assuming that.the particle· is. moving in.the .direction
(ii) or
,· •• the distanc~s of rhe two positions of insiantaneo1J, rest x inii;ea~ing, we have'from (2) · . , , , · ·
A an'!l A' from the fixed point O are given by
OA=~·V.(b:±.~.C) and OA'=b-,-V.,W-tac)
, c . C
• dx
d,=w · J4
{(' , u "\ -.xL } ,
.1·u'.,.+~, t'

TJic distance of nny of 1'11ese two positions from the centre or r/1=!.. d,'( .
1

,\'= (h/c)is the umplitudc of the motion. w v((.\'o·+uo·/w 1)-.,:"}

. 'h'.;. v(b 2 +ar) b v(b2+arl


t I1c nmplltude:::i-...;....;_..:__, - - = - . - ...-...
i .
I 11cgra11ng, I
r:::--•-cos- , { ·'(.... . .,_ •; } "T'.:'B ,·
, r·.
"' ' V Xu':,- II o/w
'. · C C· C
where .B is o constant'. .,. · '·

' . I.' J'•


•1.'.'H•,•.•,,• ., 'v ,~ ·,,.,,.~.;,," .

· 1 ,· ,\•,, ·.1· ,..'v:,14. ,, .. ,,,,;.;;,',~~i':•~~.•~·•l'i,•'·'·"c ,l··:;,c·.u!C•,•1«• ·

. _,_,, ___ ". ' ....,..., ",, . ,., '" .. " ........ '
.. , .............. ,,, .. ., -~~I~•:••••:• ","7~;

,80 Dynamics_ /1.rctl//,t'ear .Mo/./011 31


Bufinitially, wheh •1'90, x;=x 0 ,
\ .. .

. Fror,n'(l), /•"'S4;, 1/T1 • { Putting
l .
,, In· (2),
.
w·/ ha ,,c:
11= !_
, . . co.s -1, { '-V( i
. Xo ') =- tnn.~1 ( -Uo ) ,
} l j ,
v2.-;.~ (al-x'). or v•Jr~ ... 4,,i·(a!.-x'.), ., ... P)
. "'· 1 . 0
-i {:' Xo ·,• u: ,v
:.. wx (. u ) .,.}.
1 1
:T(':':lT.·-n-,-· +_,.,ta n- 1 - - Let v1 be rhc_ velocity at a di~tunce ja from 111~\en\rl)', Th~1\:
0
I= :--:- cps

or. . co•~I
>
V Xu
. .w
{I..,-( i-1.X i7. 0, } ==-· {.wl-tan-
Uo ~J ) , •
v .~o
' ·1 (-Un
WXo
.
·rU0 :1<,r) w
)t wXo
putling r=½a 11nd Y=l'1 In (:I), get we
'
V1~71..,4,,2 (111--.lal):r,Jr.lir, .'. •
. " I '
: ,.. .,(~)

r { Let a1 he the new uinplitudc'whr!n the vclociiy: nt the point,
or 'v'(l(i'l:+"u;ii_;~5=cos l- w1-.an·
,>: ' l Uo
:.ixo
}]
,,:=-!a is d()ublcd i,e., whe.n the veloci1y ctt the p,Oifit ·,'(..,Ia Is a,!lY
. =cos·( wt-- tan- .!:!..)
1 how made 2~1- Sinte !li'c ·p¢riod rcfnnins i,ti'nchafiged t.her.~fore, I

· •· • WXo putt.ing 1•=21•1, 1i:~u 1 nnd;.-.:,,.Jri'ill (3),,wc g(H ! '

or x.=J.(·x/+~:) .cos (".'r-tan-·1 ..1!:L), 4,•,•T~=>411• (11 11,-J-a•)


.. w . w~ ... (3)
or 4 >: 311•a~='4rr 1 (o/--Ja•) ,[ ',• from-(4-)', 1 11 1' 2 =311 2a•J 1
·whibh gives ~he position of the pnrticle at lim-:: (.
• I Rt1bs1ituting the:.value of X from (3) iit. (2), WC get or a1 '=4~•+taz,;,, l 3·a 2/4. Hcnc~ th~ ampliti!de 01 = (aV13)/2,. new
Ex. 35. Show that'tl1e particle ,,.,:w11i11g S.H.M. requires one
(t)' =w
1
( Xo;+::r sjn 2
'{ ~l-tal\-
1
( : ; )}·
0
sixth.of its period lo mo.i•e fro/II the poi·//1011 o/ ~a.~imu111 displace.
mrnt ,1,1 o.11r in lJ!/iich the dl:pl11cament ls halftlic arnpl/llrde, .. ··. ·
· · rh:· · 1
Putting -j- =0,wc get ·(Kanpur I 9.73·)
Soi: Let'tl1e-equatio1):of S,H,M, ·wiJh centre ~; ririgin 'lie··
O=w:··' (.Xo'+:;;;z
11 2) sin'i { . M-"\Ul-
1 ( ;.,;
II )}·
0
rf'xft11~.,.· -,;.x. ..
· The time period T""'.lrr/-.,/ µ,
or sin {·wt~tan-i_(~.)}:=O'. Ler a be t.l1e amplitude of the 1not krn. Th~.n
or
· wt-,t:1n·
· 1 · ( J.
u":'.. )'(o=0
· .. o~ t=-
· l
ian· 1 · ..~~
( ti )

(dx(di)2=µ (11'-x 2j,
. WXo w w., 0 Suppose the particle is moving from the .iiosition or m.oxlmu·m
Hcnc~ the,!inie or .the ~cisition of rest=.l tnn· 1 di,plac•!men/ X=n in- the directioll of:, d~creusing .. Then
. ~ : .. ' w dx · • l
·-= -v'µy'(n'-•x·) or dt= -•
d , -,- ,
· ·Ex. 34, . Slioiv that iwa simple /1a,:111onic morion (If m11p11nmc r v'11.,/,(a-x·) ... (.I)
a a11d.P,_griod 1 T', tl1a vel~city /a1 a.d/sta•ici• x from !lie centre is Lee 1, be. the-time rroni-thc rriR:dnrnm displacement to· x_,,a.
given by tlie ·,•elat/,J'n v2T'=4;..• (a 3 -x 2). _1hc point .\'·,,,ta. T/ien i111egrnling (IJ, we get- . ·
.Finri the uaw a1i1p/ilude if t/ie velocity were doubled wlim Ifie I 1•1• ,/,,;
partic{{is (// ti°distan~a Ia from the CeJ;/l'e; the period remaining . , " dt.-.. - Vt•, • vra1-.~2)
----· ,
1111alti'rtid. · · . · . I [ '. 1 x
t, ""':, co~·.
l [ . ]"t'
· 1
co,- 1 --cos· 1 rJ
'
..., Sol.· Let the cq11ntfon.of s; H: M. wirh centre a; origin. be -v ,, . -
ll I/
• "'·.,
V µ, 2
diJ.•/dt.2 =-;,:,;. ~
I .r rr
, ,,;,
l-
] I rr
\,i:l'l"_ 1 1 3 -o =-1-.
1 ('lrr I I
.i
6 V /J
,
,=- .
(l1111e•per1od T). )=;.,.
The time period (l) . y / u . •

~
\ ,, ·'
T=211/\i/t,

ilj
Let a bl! thc,;mplitude. · Then the vcloci1y 11 ,at n. disrnncc -~
fromtl\ece~trcis.giVenby, . ,, I
:__As. 36. A µartic-/11 i,\· ~e.,rfllr111ln11 a slmp/ehnr111011/c mo1!011·0/
p~rinr/ 'i qhm;, 11 ff111~c ()_ an((.(1 11<1ss~,.1- -1hro11gh t{';poi11'iP 11•//erc
OP=b ·.J'i//i Pa/ocity ,. IJJ tlie dlrl'cl/Qn (){'; i1rove that thj• tiiJ10, whlc/1·

..
. . . (d'-·.~')· ... (2)
elnp.!N bf/ore //, relllrn.s. io p· /,r.
t
"';·, ~'.· !:,:1:" ;,.::•:;·;? :·:::::··· . ; ·-·:,.-•. . j'.•~~n:~!;:::':~•-··~,..:·,,,-:
-~.:;,; :i:;,.:'" ;,·.:::•:•:••i\·•"·/;o': ,'.·. . •:-::~::A{t:~~ ..... ..... '·:,,.,, ',,.,.,,:: . ::·,
·_

-----~...~.-.....-.-.--.,--,..;""tm:~.~;,-;r.'?;7
.~j-::":._> !
, ,', , t ;, ,1 ~,

·t.•·,
'·\ '•1,)•

...-
'8.J''
•·' . . •:,., ':, . 1,,.::. Dyuamics · Rcctllinarrr Motio.n - ",,"
8~
··, ~
' '
; t:'ti-Jfr,1
', ~·,

·(fl·)· '211'J(x;=-x,1'·)·-: •· · .· . · :·
. vi . v· i ·, , . . . .
,!' ' .. ' ;:· "'.,. •' ' -~11'0 •' . . 1·, 2 - )'\,:·,, ' , .. ,
(Lu~K.Qow l!(7,9; fyfm'i1t 72,.183, 87; 90; Ka~p~J_74J .. •· · .. · ):.1:[Mieru(lr,77::; Kanpur ·s~J
Sol, L;~~.the· iiqu,ntjori:_or·ihe S.H,.M, with c.entre O as orig'Jn Sor.· Let the equatio,l:of'1~~,;§;;f:r;,M:,,:W)•;h,~~~tre· O·.as ,,.ori_fl:tft
be d'i(dt 1 ""' - µx/ · ' I · l be d';,:/dt 1"" -,µx,: ,i'J'hqn the,t_firi'e-pe•rf,p'd',~;..:~'lil-/fa, . . .' '
' , ,.,. -:Ir I J , • , . .,,•. • .,,, .•1,·, , , l•~•"' . . 'I, .' .

. A,
Ifa be the·,unplitud:e;oLtt(e:iijti'tJ.01J°,,1We:ha\ie·,. ·· · : : ·.
0 6 . · P A
. ·; .-f,;;l{~~-~-~i;i/: ::,:,· ·. ..· . ' ... ·
1"11e)ime· .P.~ri~d' T~?.-rr/yµ, ,., . whe.re v is the velocity fl·t;a:·dist"'n&e·~ie"fr~m -th.e· centr~. · .:;,
J'..e(th:e•ufuplitudc be·,a. T.hen (d-.xfat) 2 (a 2 ....;x~). ·:·{!) =/,. . But when X=ix;, ~~vt:.:arid'·wii;h'._.~~.x~i
v,;:=v'i,
,Whe'ri·the p!\rll9fa,p:isses-thr0Ugh pits velocrLy is' given to be Tl1ereforc froni ( l), 1H::,,·h V\l:. · . , ._. . • .
v ,in the direction OP. ·.Also op,.,,. b, ·So putting x=b and· dx/dt=v
in(l),1\•e·g'ei' . . ' . l't 2 ""I-' (a 1 : X1 2 )'.-~rid ·Vl~,;·u, 2 -!,)/);
... i,1= ,u (a2 -b2), These· give V2 2- V1-~=i{(a 2:..;r~li)-"(a 2..,.x;i))= µ-i~t 2-x2~)
'.l;et A be an extre.inlty of the motion. From P · the i.e., . .l-',=.~r/7~1~)/(~\'.7X~~)'.... ~- . : ...
to,
com-es. !nsranta(l~ous res·t ~t A, ·and. Ihen r·eturns back. tp .P. !n .
:Hen,ce th~
.
.time pe_rio',d
. ·.,_;..: . . . , . ', , ··v·
~-.,='.'iiif.V.w:'=.?.,>r.•::
-;,:,2 )
. ~ - .:.•,
(;,:! I
,-!
ts
.S.H.1'>{1.tlie ti!l),Q:fr.;:irn'·f'·I0° A .equa! to 1he tim,e from A to',]',' ' ' • ' ' • ''., , •,, , I ' ' . Vi -Vi ".

, · :, . the,rcqu!r,ed tlme=2\, ,1lme from A 10'.P, · . Ex, 38,' · 4 part1'cle,ls\;i'o,~f~:~":~.1iii. S,H,,~1;- ~,hlfe·. iit~king an'i I·'
Now for the motion from\A lo I', we hnve an excursion /r&m one pos,ltfori. df,re,r(:~p· lhe·. ·or~er; ·II~- 11/siancf'i' ,:•, ·
dx. · ·i .:1 ' 2 \ • 'I' · dx I from the middle poi,it of il,r·path,ai tiiree; coftsecuilv~ ··seco11ds1:q;,c,
'(ii'.., - y' ,uv'('a -x ) or ''dt"i' - y' I-' v(a•-·x•)' observed·to be· X1, x,, Xn; prove th'a't.lhe 'time, o/ Q comple,tq_ oscilla,-
rion is' · · · ·· ·
1,et r, be the time from A ,to P, . Then at A, t=O, -'\'=a .. and
ut /?,. t=-/1 and· >:.""'b, Therefore integrating '(3), we get 21r/ .·cos l-i(x,+;~)"·
, zx",/-C:' • .. ,
J·b -
J.
r/1=7µ.·:. V(al-xlJ;
dx :
;or
j [
fi'""':;J"µ . .. ~os-l a ]b•
·x T~ke the, (lliddie p,o!nt oc:;!iie',pa,tii as ;O~ig!n; Let ihe_
, Sol.
eq ualion of the s'.~.M.'.b,e'· d.ix/i/1~~;-:,-µ\\\ :lf h·¢,ri .the,: dine' i?erfoq ..
,.. --I ··[· · 1 b ·
,Y.I"· . · cos-" --cos-_
a
1,.L] ,.,_
, · .-,/µ
b
l cos- 1· -·
.a .. .· : . . . ; : :: T":=:~:~J.fr/i:-'...::.:),.::·.-.: :: . ;:::<:.·:t·':. ~,:i. r. \·,:

·" ··· , ·· •·· ".: • . ·2 · •b · .Let a be_.tne 'ampli_tutle/o,()!h~.'.:~6'ti9.n,.;,· 01r/.th,;e:, t/rrie r,.;:t>e..:.·
Ftence",the ,-fi;(tuircd tlmc=-2t1,= ;✓,·, cos- 1 - measured' frcim the'•positiqh 6l'iii$1d'nia neiiu~: rest-~·¼,a, ·..wlf)ll'i:;,;e,· . ; ·~
,:, · . · ·. 1.;'.;2:.·· .. · _.,u .· a . · ·.. : ·. ·.. .·.·x,ld;a:ccfr:::.;iµ'i,::.,/·· . ,._,. ·,·:·,,.•·· .·.:::- .,
=...::. . . .{y(a. b/:•P·). :
...; ,~ ,.1an·
1 }=-2:
.v P.
tan:.1 (.
.
V ·)
b..; P. where Xis the distance,oi'the:par!ii:le: rrc\°~1 'thl}'.cenrie at-ti:meJ; ·
Let }I' X2, x~ be th~ 'distnnb~s'oc the pil~li~le' from'·th.~.:t6,;t,e· :
1

from (2)·, y(a 2 -b 2)=.. ~- ]


of /jlli, U, ·Fl )ih a'nd (1i_+'2)'~ scconds •.'·Then. fro'm (,I),.
, [·•: •
, . y/J,

·~2~7'r_'an~; .{b(~l'l'/T)F ·c·: . 21r(✓µ so that v.,u=2rr/TJ


.r,,,;,a,cos-y'iu1;, , ' . ·• v · ·,·.. _(2) ·
Xf=:a cos y,:1(ti'+ !), I ' ._:1;,(3)· .
=F l~~,;i·-(~}· •~,.
x,=-u·cos •,/;, (f,+2).
y,u.ii'+cos, V~(/j J-2)1
[<;OS , ,
,;.(4)

.. .E~. 37: : A0,~int mo~t'ng: In n ·straight· line .1,vlth S,H.M, has. =2a co~ ,/µ(Ii-fl) cos •./1~=2.~;,;os"yµ, (from {3)]1\
've/o'c(liei Vi an'd'v2·when 11.!dfstancesfiorn the centre are x; and x~. cos +,>:3)/2xi ,1 µ or . . =cos~
1 {(~1 + x~)/2x~t•· ,:
Shor/tfi'ai. fhe per,lod of motion Is ... / Hence the
' ,. "'1r ' .
period T= :,,_ ;;,,_ '.,.:"'• ' .,,. ,,. ""''./', ·.• '
v
µ·•-_:'.cc,s~, {(x1 +"Xs)/2xi} ·
(I
Q )

---------------------
f,' .u-·,:,·1:.'t, •• .•,•,•;.,,. ., ......... ,. , .,,.• . . ~! ·, . ,.,

L ·'·-·-------
,________
!
811 , D;manu·c.,. ,~ec!l/i1111ar lr{ot/011 llS
·1
,:I Ex. 39 (11). At'the· end.! o.f tlircit s11~ceaive .rcconds the distances o;=µ.X1, I , .. (!)
of a polnt.movi11S' wirli ·s.H,M. .from ,tl1e. mean poHtfrm mea,·ured fl =µX~, ., , (2)
In VIie same dt,ectlon are I, ,5 and 5. S/io,v that the. period of a u1 =tL (a 0 -xi~). ...(3)
(:ompleie oscif/ati:on is 2"/0 where cos fJ= ':i/5, and v==-µ. (a~-x 21), ... (4)

· [IVfoerut 1969, 72, 9111"':j Adding (1) an~ (2), we get "'t,8=µ (x1+x,),, , .. (5)
Proce® as 'in F.:x. .38.
Sol. Also subtractln$ (.3; from (4), we get
E:.(, 39 (b). Ai the:ind:o/tlire~ .wcus,;i'1•e seconds, tire dl.rrances v1-u'=µ (xi'1-xi~)=µ (x,-x,) (x 1+x.)=(ix+,8) (x1-s,),
o( (I poir/1 _n;,.~:vi:ng "»;ith .rimple hari11011ic 1'•101/011 .fro111 iu meq1 posi• . • ' •..i..,. · •. [from (S)J
tu;n measured w the same direc//tm ar<f I, J mid 4. S/1011' tlwt the (x,-.~,)=(v 2 -u 2)/(ix+/l), This gives the d'istance bet1~et:n
period~( complete oscl/latioi'1 is' lhf two positions. · . ·. / ,
211' , , Now 10 get the amplilud,i a ii is obvious :bat we have to eU-
co.rll'5i6)' , [Me.erut 1987!", 88P] mi1:ate x,, x~ andµ. from, the equations (l), (2-), (3) and· (4), Subs-
Ex .. 41): . A body mo~/11g in (I ·.f/!'ai,~'11 line OAB 11'/t/i S.H.M. titu:ing for xi and Xz from (l) aud ~2) in (3j and (4), W\: !iavc
has iero vi!loiity 11'(1e11 at the poi111's A p11d B 11·ho.,e dlstahces fro111 u'= µ ( 0 1 .:. ~~-\ /,,:,,, a'r';-u•
' 1
0 are.a 1'111d·b respcctive/1 1: a11d has,1,.elcci'tµ v ll'hr11 /1<1'( ,,,·a•, bet• 1,-(l. =0. •
•.. ( 6)
\V/!('/1 them, Show ti,at d,e co,111p/ete,perio/is rr (b~a)J;,,
"' I
Sc,l: fo ihe figure, A nn;i'.B --·-..:·-·· ...J.--------
,I
,rnd (a•-~)
µ.
I.e., ALl_;,i:,-13'=0,
w. 1- ~ , ,
.,,\I)
are ihe'Mposi!i6ns of ir\Stanta- O ' A I · C
13
By tl.le method of cross, mult1plicatfon, -we have fronY (6)
neoµ~ ;:est 'inn S,H.M. L,et Cb~ ti.I\? micdl, point Then C of ,./,8, ( 7),
.is t\e ccntr: of the ?1,0til.ln. Als_o it is '~lven that 0-j=a, 08.,,b. iii 1i · µ. ~ 1
1
, ,. · !The n~itude of lhe ,motio11=•k,1.B,...-l (OS- OA)=Hb-a). u'l'µl-.:-y",';:ii' = -a 2«"+a•p•=-au·l-a 11••·
Now in a S.H.M: the velocity at the centre={·✓!") >;,unpli- Equating" 1he two values of pf found from the above
I ,
equa;ions, ive g,ct ,
. Since in this cM·c the velocity at the. centre i$ given. ro
·I
·1 • ' ' j.
"'"'-11',8'
a (l'i-1,1'l)= al (1•'-1t'1)
2
[a' ("~ -:-P') ]' 1 O,T
, "''"'-11'/i' ·c«•;....p•?
a' (v 1-1('J°"".'(v2-u'),r•
fore 11=½ (b'-a).-.,1µ. or v,u.=2v((b-a). , ("iv,,-,e·•~.(1·1 -11·:) · ~(v'-u't («~1••-p~11l)} 1''
Hence time . .
peri.od r,;.21T/;,,/p.=2,,.
' . ·,
((il-llJ/2v]
l ,
"'r.(b-a)/1•,
• a•= --··c;,r::.,sr5r-r or Qe!' \ ,,.,_p2) \ .
Ex> 4'1, A pblill .axec1.mJ' S.H.M. ,mcl1 that i11 _/ll'o of its posi,
t/011} velocities are. II, 1i tnid the fivo cor~espo11/:ling 0ccei'er111io11s are
tiP~, 42'. A µarticle res1.r ill <':qui//~r(U/11 1111der t!tc iatractio11
-~ol tivo centres of force whlcl.1 attract directly as tlia distanca, t/lelr
·; shoi.i ·,, th(// the· disianoe . beiwa., )lie rwo po~(tlons is
.,;v-' iii:a,isities being I" awlµ.' : 1lw partfr:11! 1ts displaced slig/rtly· to•
rr)/(~+fJ) and ti1e.a111plitude of the n11;tid1/'is 11'11/'ds 011e of 1l11u11, sliow that tlw li1m of. .a small o,rc/llalion is·.
1
., ,'{(vi.:.. 1,z)(ctlv2,_fJ2u~)}l/2 . ,
:!TT/•/(1>+1,'). , · (A~ra 198!J, 86; Robllkband 88J
.. -·: . ·-;•-µ•-.- : [Mmut 11990S; Allah~b~d 77) Sli!. Suppos~ If and A' ar~ the two centres of rorcc, their
.· ·tSol. Let the equation of the .S,H.M, w:irh c.entre as origin intrnsi1ics being I" n,nd 1•' i·cs• · -L....
bed x/4t 2 =.:..·1,x,• · .· · p~ctivdy. Let ~- pn.rli~lc, il,f ·A' d p A
·,1r ..a he thc·ampli!Udc ~r the mo1iti'n 1
I .
J1tiv~
(dx1dl.)',=:j,. (" 2 -xi), .
was.s 1H be in ~q\lllihrium :/1
.8 und c~ rhc ntti·,\cti on of thc.se t WO c~ rurh. lf ,-1 B=a and ,!' 11 =-- o',
edx/dt is tbc velocity at u_.distuncc .Y. fro'm th~ cen,lr,e.
rhe forces of ~11i:~c,ti611.~\ 11 uu~ IQ)I\,.~..~1;11tr£.~,.tJ-anu A' oN
·~... ~c~ x; •and X~.be'the disrnnces,frOm the· centre r,f the i•-,,
posi l,ons-whcfe ~ nnl:i• are, t_he v~ti:iC:tles' :ind), r,n.j fJ nrc the
IWO
o.:id 1111,.,''a' resp~1;'1/vely iii opposite. dfrctthins, As t h.ese two forces
balance, we h.:iv~ 111·1w;.111/"'a'.. , .. (I)
~- '•
· accc, eratlqns rcsµectivcly. 1:hen-

' '!.,;·•;-,: ,_,;_::-' ::•::;:~.:~·::•· ·.; .· ,:.::•::;::~::,. , ,,~-··:,•• :•:::,•--~-r~;:~:t. .~.:.: :~·~~,,
! ~·- '. ;; ,., , :•:<::: '.i)~ .' ; ·'.~: : .' ~:• '. '' . ,:;;::.:::::-'

..--~.---:-,-~~--
.'

·.•'.•

;j
~
"'(.l '•.•
ij 86 D;,:11amicJ· Rectl/l!iear Moll on '..·'sf

I ~
..-~ : Now suppose the p!l,rtlck is slightlydisplape'd 1owards A and Obviously r is least-when d~x/dt 1 is least . n'unbe ie11:st:v_a'luf'
'~ then let go, LetP be the poslc/9h of'the,particlc -nfter time t,
where BP•X, .,
of dix/dri is -.µa. He:ilce· leastJ'=rng:-{1tJ.4a, · , . :·: . ·
The strl!lg wiU remain tlg~t .if:this least · tension· .,~s.. P?Si_tlve
j Th·o at1raetlo!l' ~1 P due to. tµe ecnt/c A ls,mµ,rlP or m,.(a-:-x) f,e,.,·lfr,nµa<:lilg .. · . ,,, · ·.: ' ,, ·'._ · /~ .
In the direction PA f.';:',' fn:•thc direction of x Iner.easing. Also tlie f,e., If m411'1n 1a<.ing · · . : ·, .. · , ,[';' ··. /i":'"41r1 n.1.)' '·' . .' .
atfra'ctlon'a.t p duc'fo ·the. centre' A'\ ismµ'. A;P or mµ' (a'+x) ih ·t,e., ·1r.n!<if(4'1raa).,' Henee::~_li.e·}esu)F·· .: , ' .:_:' ,. · ·,· :·.' ,

I the directio~ }'A,l ,(,e'.', fa the direct.ion of.x decrensl!)g, Hence by


Ne11/ton's occond luw of molion, ihe equation of motion of the
tilirtlclb .a·1 P ls · · ,
m '(dax/ell')=mµ (a-,-x)-niµ' (a' +x),
·
... (2)
Ex, 44,' A ho'r/zont<il.-s6e[f·fs1move'ililp. ·i:zrJf!1</o'!"IJ, ll;tl!J,;S; 1:(.,:M, ·.
of period½ sec, w!,a1l.s.rhe·a11:i?ty~1i/fliff!",.(~'s~h.(e::Jr(,~':-f!e_r )hara·.
)i,,e/g!,1_placed 01(.th~ ,sfi~lf :rnat ~of ·?.~}~:~~ed:
Sol. L~t 1m be the _mass ?f-th'e):iody
-~L~c~~ow_J9:79] f \·
j'.. · J..
whei'e'the force ,in the dlreciion cif x increasing has been .tak'en placed 'On .t.he, shelf. ·suppose·, along wi.th . · ·1 ·', · R
1\ilth -rive. sign and the force in the dir.ec'tion of x fo,reasing has the shelf, the body moves .j n ~n:,identlcal. · _P ;.--_;_,...:.:
·
been· taken wlth ..-Jve sign. ,
Simp)i(y-iog lhe..e.:iuaiion (2J,'1:,,e get .
· .
. .
· S.H.M.bet~eenA.andA,,,Let._Obethe 1

centre of the md1lon so, that ,OA,.,':';-a is the , _0 · · ·


• .~ · ·.
~8
l.
m (d 2x/dl 1)=m (µa.- 1,x-µ'a'-µ'x) • amplitude'. . . .'
or d1x/tl{ 2 = -.(µ+µ.') .r. . . ['.' hy (.1), nii,a=(11µ'a'i The time pcrlod 2'"/yl-':~h (given)
This .Js .the 'Cquat!~n· of a S. !-L M. with ,:;;;ntre ~t the·origin. . :. · µ= l 6'112 ;, :. ·-:· : : ,_ , · ,
Hepce the:mot!on of the particle is ,simple hatmo'nic with centre
at .8 '6lld its time period is 1'"/v(µ+-,.,.'). · . . Le~P-beihepos/t/0'1i"of}h't\ ~O.fY" • .. ~'.· ;'_. ·. ·""
Ex. 43. A body ls a11arhed to one fnd.o/ an ,inelmic: -string, at time 1,.where 0,P=x, Tli'~·,i!'nPi~~s.~dJorce acting ,on the 'bqdy
and 1he other 1111d mol'es in a Jer'lical lina with S,H.M. of 01.Jip/fu«Je Is R-mg along OP .. Here':R is :tlk ·r~action of tb(;sSe!f-•. By
a, mpklng n ·oscl/lations per .second.' Show rhar the strl11g will not Newton's liiw. the cq~,a-tjon·.o'f"ril6\ii:ni oft~e.body)s,· . , l'
remain tight d11ti11g '1he mo1io11 1111/ess n 1<.g/(4rr 2a)· . m(di1/dt.1) .;,,:ft._:.:._tjii,,'. . . . , :
. .. . [Meerut 11>70, 80, 86 P, 88; Agra '7S} . f,•,11iffn:i •~1~x/di 2
)_: _: • - , , . . r tl
· Sol. Suppose ~le string remains tight ·4 Obv~o.usl~ 'R is.l~a~t-.when d 1):ldt 1 i.s.. least !)._nd th~ !cast ia:lu~
during the. mo1io11•,so t'hat the 'bo'dy also .I ; of <:fx/dt· 1~ .-µa, .l;(en~e· lcas.t,R ... mg--rnµa,
, Tb:e body wil.l n·o.t,be 'Jeiketf;:off.:lf· thf's. iea1t 'value ,· r' k
.
moves in nn.Jdent,ioa! S,H.M. L'et 1n be, thl!
mass of the bo~y. . · ·
LeH_he:·.bo'dr. move-,n S.H,M, b!.'1w:e-ett
. l ........rl,
Pt remain.s nof.negat\i'j,./~:•.\(1.,~!f.id1~c... . .
.l.e:, · •1f ml 61: aE;;•'.17!~·.,:.;:.'. f·\'. '.'/P,1=;' l.61r2) .
,.
A'.and A/ and -~uppose·O is th'e ceotre, or 1x . T i.e., ·if a. ,;; g'/(l iiir 2);'."'. if:te.nce;ithe ·', ireat~st
"admissible . val~e of_
th'e am'plitu.de)i.""'i/U~1r~j·.:;,·,:1 '::,',,:: ·_::.:._,:' · . .'.,,· , . ·, · . ,
the 'mbtion, -whero OA-=a,
Sln~e·: tlie -bo.dy 1#akes fr osciliations per
·
ft I i ·m9 0

, Ex •. 4S:,::A.,pa r.t]cf~:~/,'tiiat;·:ii>J/'atta~fied.10 ll light wlrethich I


. '
second, .thcNfore
. •• . f, • •
•lts. time
•• • 211 .
==-·
period-./
J is stretched ,tighiif ·oftw.~,~t}'.'f:1:'o.7/J,;~¥,-p~'lf1':
}dtli" a· te11;1ion, T }fa,.·
I
, . . , y µ II b be_the r!istan~e· of•J~/ p(!.1'11tltt/rpm··t,he two•.•tJ(lds, proFe•. rh t th?
1 This .gives µ•4,,. n-,• 1
pcr/od of .m1allJia,1s~er1~ •O'.l'c(l{at-io1d1f' 111tis,s. iii 1's. ·. · ,
At ·time 1; let the·body be in' a position · , .. · J_ ,, ··2fr , . :· ·; ·. · . . . ·
J'; where OP,~x. The ii'i;ipresscd force acting
.on the bpd,v l,s T.-111i(1Hong OP. Here. Tis
·1nJ~. ~r~1,c ·
,4
✓' l //U.1/J -I , ' ' ~!
t;e.. tension Qf'lhe' strlrig. tBY N~wton's Jaw, the equation or Sol. Let lt light ~i.re 'be strc'tcJmJ. tightly 'between !l1ejl\xcd· li
motion ofth'e bod;' 1$ \ , m (d'x{d1 2 )~r..;mg. '·
points A and B with:a tension ·1::· ·:Let' a. particle or mass ,n .be
. , .', T=mg,•,•m (d'x/dt2), aHached at 1he point·O o(thc wire, where·AO=t1·arid O'B=r-
\ .-
-------~------------'"'--""'" . , ·-- • , I , ,

ti
ijl
·.. :·., ,: ,·. ::'-' ::: .·,..•,, .,,.:,:·,,:fr:•;•l>1!'l119.1:,;1m,H-'"'H,:,,,,.,,.,,,;;c•: :·:' •.c,,.,,;:,,-.1~~:u.~,ii,,: •:,w n, :: ,.,.,,,,,. "•''''"'""'r,_,,momo1m<>""'"'1"'"'"'"'u111\l\
IIJ~it~/}:;;,.:t : , U! ;::/~_ft}fltiRi;Ui~i.1161,h:_;,;; i,:: _:,v;;:;:::/i:;! :~:.:;;K~;J;{,t~;'.;i:j~:~&!,~~❖:~ :i !. ;. ;,·; ,: ~, ,;,-:1 ,,,: :, ,:,.:,::;.,-~·(/.mt~~~US!l~i.;9j\(•>:,:,;:; ;, -.:
1

,,11
!;,
r
:1
_,;:· '\,
.•
·.l)y11a111ic.',
,, ,.;t:·,' ,8.ect ///near M 0//011 g9
:, '

I,
!,
,' -~-_at::e~
:.L,,.,sli~'liJ)Y
••. '"''"; 'b•·.ai,.... ~
pe~p_en~icµll).r· .. ,r ' ~ ,. ' I' ' • ' . .
0.0'=;/,' Let u, v, IV be tlie velocities of Iii-: pttr.tic!c · at i', Q, R
respectively ~here · ·
·! . :. •t::4.8! (f,e,, In· the tra1'1sv~rse. ·. · · · : ·. · .
irection):a~cl.. then ,~i· gti, . .. ! ' I .' . i.
O'l'_=·a, o:Q=b, O'/1.=-,,·,
. l~et P:be the' positio11 _of the· · A ~ l l a. ..,.•_u 1> w
rticle nt acy\tir.:ic 1, where . _ , · 0
1 <1 :; _
0 O' Y, 5/ R I
P:;::X •. SincS-the displace1\'Hlllt is ·~mat therefore the te11sio11
!~·the strin~.in a1_iy. displace~ ,pOs!ti~n ·ctn ._b': ta~_e11 _a~ Twhich 1 For a S.H.M. of amplitud 7 A',-~,~ velocity V rtt I\ di:;lo.ncc X
1J the _tei:is1~n 1n the string. in the i or1g111al pos1t1on. The from the centre of fo1·cc i.~ given by/ '
uation or motion of tho pnr.ticle is· vz=f,L(Al=xi), , ., .. ,(I)
d 2~.' ·· . •. - i· At?, -~=OP~l+a, V~1/
m. ;:n, :':" -(T cos LO!A'+. .T'co_s· L~(B) at Q, s=OQ~l+b; V=i•
· ~-T
. (OP·
-APf BP
OP),=:',-T·
. · (·.;;7fa"'+x
.. ~- - ;.'~:":V(6iC:f~'
· x Y) 2
and
from(l),wehave
at fl.,· x=OR=I+ c, V=w, ·
,·· ·
· '{X ( ·•x2')-I/Z ,; ( ', xl)-l;~t .
·=··-T. a, .l+iil' J·-'t l.~-,bl : r ' ' '
u2=µ. (112-(/+0)1}
2 ' '
or ~-=A 1 - / 1 -i,~-2ol
=.~T [H 1-½:~.i:. . :)+H 1.-r[:+ ... )l.
f,L '
ll_'

Ii = -r·(=:~;b.::);:'nc'glccting
.a . t .
hlg_h 6 r powcrs-~f x./a
1" ..
~rid x/b. f.

OJ'

Si111ilnrly,
(~+dJ)+ 2L a+ (/ 1' - ,P) ... u,
"' (2)

i · · wl1ich ·p.re vcry.smnll


·f,f. ( ~: +& 1 )+21.&+U'-A 1)-o.
· . (11+'b)
= -T ·'at' .~.· II 0 .. (J)
-~ and
. ~ )+21.c+(/ 1 ~.f')=ll.·
I::....+"~
d~x r (a:+b) · . I T (a+'b1 \ /L ' '" (4)
---· - - 1 - -.- X=
. -µ.x, where µ.=--
ma 6- ·
dt' -
This is the standurd i:q1tt1tion of
· ·ma 0 ·
a S. H. M. with centre
E1iroi110.ti11g 2/ o.nd(/2-.-1 2; fron, (2), (3) and (4), WC lln,·r
,the origi~. The tim~ '1mlo,d ·
:11
.u•J
·-7-a• _J ..
a
i I
/l
· ·. r'"'·
. ......
2:ir.. ,._,:2,1
,. · ·/J·\{T(a+b)} ·
, ---r- =2.,,._ J{ -y-·-,:·
nwb } ·
I

-," . • VI·' · . mav . . . {u+u) , ~:'...+i,; b I !


11! ·Ex, 46, lf /11 a S. }I, 'M, u, ,,, w oc· tke ve/ociti'es at dls1a11ces ,, , j=U
. ci; b, cfro111 pfixqd-po/11/ 011.th~ 11raight /Ina ,vnich is.1101 the CC/I/re
,,
o//orca,·s1io11· thdt the period T q g/ve11 by the equa/1011
'H':J

,.,-+,:' ,(' I
i
·./'
' •· .. . \ u . µ2 . Jf'2 ' i
411 2 ' '. . ll'J (I' (I
..... (a-b) (/J-c•) (c-a)=
P· ,
a· b c r.; - ll I
11 I'
,i' . . I I I I I I
:.~.~-- µ.

f
I)'
I \

I \ [Kanpur 1980, 85, 88; l'vk1,rut 84] 01' -


\''.!

I•
I.• +' Ii' /1 ,,.,,o
i (~ so·1 ..,.,:iet ~--,-.!!,nd O' -be' the CC!llre of force and the
w' ·,, _i,,::<?~. .:,:.
··1· /i);etl ,'p6h\°t'' '.~d~'p~'t.t!vcly_· ', on-F'ihc''''(itie of m,otion an(\ I~ t c~ .,-'•
µ. '
Ui C

:,Hj
:'l'I•
~ l
•·1!,

•,ilj1►-Jt:
•jiH

'!1,/j lii''.I,' .;,:•!--:•:.:,· ',:..;,,::;;: ',,;•;,,· '-; ;'., i~ .:'.; '. ··-:·:·::·f:' ·'

------~.:);;i(/ -
,.-;;~;~:i~:(ii·,-
':;.

l ,,,,. s.,;-,•;J,:,;,>.<.

',J
,;i
~i
~I
;j
~
i
',
9U ' R~clill11ear Moliol/ 91
Dynamics
·i al
!\ ' I
a J ul (( ___a_...._.,.......,..~----4..
or• - i b~
ci
b
C
'
I
I=µ I .I

H~2
vi b
C

"l
0 ·A e. P Q
length. Lei /3 arid C b_e the two positions of t)le free end A of-.the
C
1

or
~I I
l

b
a
al

b2
I=// . I ul

a
Vi

b
1,:i

C
String during its .any extension ani:! l~t OE=b al')d 0C'4=_c.
Then by Hooke's law,_ · .
. b-al
the te11sion at B=Ta=A -a-·•
~ ..

I. .J C cl • • · · c-a·
·and . the tension at C=Tc=Aa, ... (2) ·
lfz • v2 , where ,\ is th~ modulus of eJas'\icity· of the stri1i_g., . ._ .
w1 I
or
Now•' we find.. the work aooe
'
again$!•
\
the . tepsion
'
'fri .streicnlng
..
fi (a-b) (b-c) (c-a)•J a b •f;, the string frof\\• B to C. · • · ·: . · ·
Let P be any position of the free end of the string during·its
extension from B to C and let op;._x, · ·
· · · x-a . •
I But-t. he time
' ' d T ... 211
per10 -./µ' ~o that 411 1 the tension at P=Tp=~. -n-::-'. :
µ=.ri Now suppose the free e·n4,.of ihe string.is s·lightly· stsetched
Hence rroin (.5), we have 'from Pt~ Q, wh'ere PQ~8x, .:rhen the -\~Ork .done again'st the:..
4 ,,
' . ul yi wi tensioi:i in $!retching the striqg rrom.P to (2
r:· (a-b) (b-c) (c-a) a b C ,
· · ·(x..:..a)
=T•r 8x=-A-:----,o;i:.
• II ,

.'. the ,fork done against the'tension in stretching the string


,l
from B to C . · ·
~ 8. Hooke's Law : ~J' ~ • a
(.~-a) d:~~}• 11'[ ·(.,:-a) 1 ]'
~
·. · · ·

"1' Statem.ent, •The ten#on of an dastlc st:li;g, is proportlo11a/ to


11X/<:nslo11·0/ tM J!r/1111.bayo,nd Its 11<1/l/ral lt:11g1~. ·
.::~a [(c:...a;~-(b-_a) 1
:=t [{(c-aJ-,.(b.:..~)) ((c-iiy;,:(b-a)}l
..

I "A . · '] • }. '


If ,i js the sl retche:d length· of :i. string of 11ntural Icngth /, then =(~-bJ: ;d<i (q-a)+-;Jb-;-a).
Hooke's law tbe ~/nslon T In .the string is given by T=f:,;-: 1, =(c'--b)d [1'c·l•Tu], [from.(1~ and (2)]
where,\ is cnll~d.the·.m.odulus or c·1asticlty'orth~ string, Rcownber =BCx_(mean of.the tension•a·t Band C). •,
that .the .d!rectlon '.of' ,tk"e tension is always opr,oshe .to the Hence/th~ work ·doiie against the ten$ion in· strclching the·
·extension. ·· 1 · string is--equal to· the product·of'ihe exteinion _ar.p:! the ··mean of
.. i
the initial and ti11ai t~11sio.ns: . · i
Theorem,, Pro,:/! (hat the 1~ork 'don!! agalnJ't the ·1e11sio11 in·
str<?tchl11g a /lg1ii rliwfc nrl!tg, iJ" 1?quC1/ to tlte prod11c1 pf its ex1en- Now we shall·dis~uss n few· sirnp!e;a'nd .. int~cstiirg cases pf.
sfo11 and the 111ea11 of II.,· /fn(1/ nmfjnitt'at, tensions. .[Kanpur 1977] simle harmonic. nwli~n.: '
§ 9. Particle attached to one c,nd or_a, b.orliont11Lelastlc str,fng,
· ,l'foor. Let O A= a, be _the natural, length or a striog whose olle
end is fixed :it O. Let the string be stretched beyond' its na1urai ti particle of mass. Ill Is Q/(cu:l1ed io 0/le l'lld. of. a horlzo111al
elastic strl11r 11'/10.rl! orher e11d'!s /1:redto a pc/111 ·011 a smooth /iori•
i
·/

--~-...~.,-...,••,~------·· '' .:.:{;b • ' AiMAAlbih ~


5 PM
L..,.
!

·.·:.;· ·.;:,< ·:,:,}:".'1:iil~i'.!i~l,i::.:,J:·:;.,:,;:,:,::~•.

•• ....... ~-~K~-......,..... ..,___,..._.......,. ,. , , .. __,~--•;--~->.~~-~~~-~~-~• •~---• •---~·---.


mrn1u:f;1 ',.~_.;;:,;.::;ff,:HJi~~UKS}~liiU.Ur~ ► t~;-i-1·; :,1:;w~·;'. :. ';;ii-;;:; :;?~:~~;~at.t1:\\{-~,.;l:;;,..,;' ,;,.: :,d'Hl,!!,\)•f~W)~p;; ;,c r. 1•'.Jo•: .,,•,~·,1-·

~2 Dj11arnla Rect'ilfoear 'Mo/1011 1· 93


•• • t

ZOii/ai tc,ble. Tiu:: partfrlr\ ls p,tflea' lo (I/I_\' disiallc// ln Ilia d/reclio11' g1tined :,t A the. p.at'ticle con\inucs to move to the left' o'r A. So
of tlip· s/ri11g and then le! go; 10 di;icuss ,the motion. long· a$ 'the sfring is ·1o'osc·lhete is nr, forcii o·n. the piu·ticle to--chnngc
· .·, , (Lucknow 1977; Allababad 76)! its velocity because the only force·here is·th,,t of.tension n.nd ,hu
Let as\ ring OA of natural length a lie on a smooth horizontal tension i~ z~rn ~o long as the string is loose. TI\US the n:i.rticle
tnble. The qnd O of ll\C string is a:tached to a lix,:O poi111 or thi moves from· A to A' with uniform velocity ...,!(A/mn)-b i'ziincd l:iy it
table and n parlicle or mas·s.m Js nHaclied to the oth~r end ,1, at. ,1. Here !-1' isla point on t);1c other ·~ldc ot' 0 sw:'h that 0,1('=
The .mass m is)pullcd -upto JJ, wh~rc AB=b; nncl then 1et go. DA •. Wh~n the p~rliclc passes A' the sti'll\S ngr-in- bccomes,tlght·
1 and begins to ex.tend, The ten.sion ngai,~ eon1es into picture 11nd
l?et•o/(cf;,..) ~·- l~I, b/(~) th~ particle begins 10 move ins. H. M, · nur•now the··rorcc of
i><:l• b . 2;_.,, ~• 0 tension acls ngainst tho direction or motlo11 of the pn1•ticlc. So the
.s·
. \ . ·,A'··
. ·o ·.
·~. ... 1,· 8 a, vclocily of th~ pal'tlcle 5t/trts-d-ccrensi.ng nn~;(he ·particle com~& to
in,tn11~aneous rest nt B', where A'B'=AB. ·Fhe time from A' to B'
. Lc_t P be.the positio11 of the particle after time t, whe1·c AP=.~J is the same ns I hat fn,111 .o' to A. ,At B' the .µarlicli: i,t once begins
The ta~tc being sm\1oth 1 ,the only horizontal force acting on thcj to 1nove tow,1rds ,4' because o'Cthc tensi\in· which all racts, it to•
p:t'rticfe at Pis the.tens.bn 1' in \l:e string OP. Sin.ce the direction! wards A'. Retrilcins its pat'!, tht 17:Hticle o.uin ;,:omes 10,instnnto.•
of tcnsioi1 is nlways oppt,sit'c ·10 · ·,he cx.tcnsi(ll.l, 1111:n:fore, the; nenus rest at E· a11d tlrns it co11tim1cs to nsc,ill(lt';)' bClW\!1:l\ lJ nnd B',
force Tacls iinlic direction J>A'i,'i!,, in the dirce1io11 or x dce-;1 During one co.mplr.te MC_il!'nthrn the particle cow1·s 1hc .dis•
rea_slng. Al~tl by Hooke's .Jaw T=;.\.(x/a), Hc1icc the equation tr.nee betwee11 A Rnd . B and RISO \l\t\t betweon A' 1\110 E' \Wice
of'r1iotio1t t,f·the par1icli: O:t P. is . . white moving II\ S, H, M.- Alm it cOYCI'• ti\~ dlitn11cc between
· d:;,: , ,' x 16t..<, ·,\ ·
. !
Ill • a'""-··A· .. Qr· - ;·=·"·.~ ,\', ,•!, O.J'ld A' t-rice \Vijl\ uniro~rn .v.~locity. y'(;.,/cr111Jb, H~1\c~ the totn.l ..
. . , · ·c I . ~
.,...t
t.t,
1 .. 11111 · ,,. (I) • t1ITJl: for one complete osi.:illat1(1n
i . The cqualio11'(.J) .·show:(tha.t :he ·11101io11 or• the ,1,:uticlc is :: the complete time period of :i S. H.M. wlum: equt1tio1\ is (I)
· '· simple lht'rn,011ic •with centre n!' th.: Ol'igin A.. The equation of +the tin1c t11ken to cover the disu1ncc 4a witli unifimn
motion (I) hol<ls good so 1011g as U1e, strihg is Slrelch'<c\· Since velocity v'(A/(1111) /l
.th~ string beco1i1es sinckjust. o.s the pai·ticlc reaches· A, thererorc
· the. i:quo.tion ( I) holds good ,for the motion ol' the particle from
4'1 .
"'v(>./arn} ,~ = 2"
' ·)(am)
T
4aj((IIII)
Ti T .I,
·.B to ;t: . . .
' V, Multi1)lyi1rs (I) by:! (d.\'/dt) ,\ilt.l inlc~rating, l\'C,gcl ., 2 ( "+¥)J(~t), I
,.\ 'dx)·: ;WI_,.,.....
1-. '. ,\ , wlieri:' c :$ ,I COI\Siallt. !llustratiye Exampl~~:
· \di . am r,x, 4.7. One Mrl of an dosth· string (111nd11/11s of rlasttcit)' ,\)
Al the pcd111 !J, .\'=I.:· ;llld d.,·,'ai:,;t); C=-=(,\/illll) If.
wltos,r 11a111ral_ la11gtlt ts a, is f,x~d to a.110/Mt 011 a ,r111ootli lio(ltontal
. .· . ("xdt-- )': .. am-,\ .. ,(1,i-,,;,•),
1hus11u-h:1.ve , .. ~ ,
.,,(.)
tahle lllld th~ 01/ier /,r tiC'd to a pa_rlll:I.• n,I 111aiY 111, 11•/1frh Is l,l'illl(
011 tile 1n11l,1 , Tl1e particle t,r p11/lcd lo a r/lJt(ll/a fro111 the point
il This equ11tio11 giv'es ·1e!oc)ty in uny 1rnsili,>11 l'n,Hn ti lcl ,,f. of a1111C'/11ne11/ of tli~ string equol (o 1,1'/cl! Its 11a1111·11l l1t11ith all(/
Pulting x=O in (:;a), we h:we thq velocity at A= y'(;\/<1111) f>, ill tli~ t/Jr11 IP.I go, S'fiow that the lime of 11 ('01ttpl1ttr os'cil/nthm is
d irect!on AO. . , '
'1 The' th11e l'ro'm .B to A ls :t-Of lite COlllpl~l~ li111~. pel'io<l ol' ,\

~
2 ('rr •''2)
T J(U/11)
T . [Luc.know .19ll I)
S.H,M, wl16sc i.lquu'tion i; (I): Sol. Pr9.ceedexact\y in the,.s.nl')1e w_a.x_as it1 § 9, Herc, the

t .\,
Chan>.ctcr .or tbc)llotioit. Tht)l,:Oli!,1-!,l from .B tl.l A i,, simple
1farni'ollic·. :Y>'l{~1illlt! l)'t\\'licle r1i~cl';es).-°(11c strins becomes Sll!Ck
and ihc sinipie 'iidr'ti16nicmorioi(cea~Js. llut due to the veloci,y
' - .
pa~ticle is. P~))~Llolf -1!(listal\C.C frp,'!l}:n,e P~\rr19r
~ttacl,rn,~.n! ,,( the·
Slrmg cqLJa\ to tw-1ce its naturnl·kn"gth: · ·Th~r~lore 1n1trnlly ihc
incrc:i~c 1,· 111 th~ Ieng.th ,.,r the ~tr;n,J Is equ11I to :.n-n u .. ·11.
~-..
. I\ ... \ ... ~"' ..... - --~ .........--

·.::;.::•;. . ,:-:i:~:~•;: ....


'·~' ~;";
,1:•:,.;.,'.1;, :,•'.::.:,;,,,1 .,, ~~·· : : -~·'.: .': ·'.,:, :· '

..~,,,1,;,,,,.,,,J,h,,,/7,,•~~..t.'lll

r,,------•-"."•-.~-,••,
.
•_ _c _ _,__ __ _
-~, ...~,...,~ ... ~
. . .

>.,••,'
,,:.i•,:\:,:, _;~~,;;~:;,,,,, ... , .', .. _, , .... _._ . ::::}.";..
~
•i
s
:~
I i
f . .I . it;
11

I
,

94 D;,namlcs ·l Rec1i/i:e(U mot1:on ·. . . \ , ~, .. , , . . . : , 9.5:

II
Il~
Now proceed ns in§ 9, rnking b=a. .
Ex. 48, ,,f. /fgh( elast(c: string 1Mose modilh1J of ela_s1lci1,1 1 A r, the string remain taut ,anc!j'SO'•!he ei:i.t1re motion ofthe · particle- 1_s •·
gove med by Irie above: cquafion:Thus thaparticlc makes oscillalions
Is strt'/ched. tc i/011blc l(s IMg th a,1d i~ t{ed to /iVo fixed p6l111s dis• in S.H,M, about o· and th,l'tirne period o-fone complete osciUatiQn
I 1an1 2a.apar1. A purtic/q i·
mau m tier! to Its m/d,dle point Is dis• ·· of
= the time peripi;l S, RM,._whose equati\:'n Js , (l J ·~:
placed In tile' ll11e of 11, .ming through, a diflance equal' If/ half
/u dist((lnc,a from the fixed f lnts and ~'rli!ased. Find. 1/Je 1ime. of a
com'plt1c ~sr/l/~1lon and tl(a '·111(1xlnwm velocll)I Ol'Q11lred /11 ,11e. s1ih• IJ(
am ) "'!'~
_..,,z,,. · 4>. ; ·am/A ,' . . J( )' ' 1H
I

.reqllE'llt n10flOn. , , ·, \ ... . ,;, . . . , .The nmrli.tude (i,i!,,, ,the tria?im'um· displacement from the
So I, Le1.,a.~ .eJptic strift& of ·natural l,engt-h _a be .11retc:11,ed., centre) of this S.n,M,.is a/2, · j
between two;:~xed points A and·,$ d'istant _2a :i.parti,.P1 .l?eihg:'the_,• l(le 1naximum. ve'locity,={V ~) X /iniplitude
rni'ddle ·point of.if.B.. ,We have, 0 A=O.B::s,G,. · · •.• ··: ···:, . ·. · · ~V.(4A/an.i) ,:-((!/2);:,V,'.(a>,/i,), :. · , , .. ;
. .:
. '

'
£__.
.. T2.
. ,::·,::,.·,
' Ex. 49, . )I particle of. fl'J(ISS t!1 ftXl!CU,te.r .s(mpl~.''.'la(·nt91il~mo(/011
in '/he. line. Join(ng. the poinis ..l .and ll, ,on a smoo1h table and 'ts ·
r
4 o r, P ·c· B· co11nected 1vith these points by ela.s!lb s.lrlng.s M11ue' .·•ten·stons in
:, aq1iillbrium are ·,eac/; T; 'iJiow 1ha1 the time· of.tin c,sc/llalfon ls
Nat11ral length.o{the portions OA and Oli\each is-.a/2 '($in~e 2;; {m/1'/T ({+ 1')) 112, 11'!1ere f. /.', ar~:: the exleilst'ons:o/.(M s'trffrg.r be- .
the·string is stretched 10 Q(>,lible its length),· .k~ardtle of mass m yond /heir nalltral /egths. · · · -::·, . .· · · .
atta6h'e'd to the middle point O is dis·placed towards . .B upto a poin•t· SoJ; !,.'.particle of rims At r o·
C, ,where, (!9 .... 412 and_ tlien let go. Let· P be''i'he position of the, 111 rem at O· being pulled PY · · 'T 8 •.
r,nrtlclc :i.fter .any time t, where OP=x; (Notc·rhat we have t_aken two horizontal strings AO and. ....;;_,,;:
0 as origin·. The direction OP is tlrnr of .v increnslng and the EO whose ot~er ends. nre 'A O : ~· ·p · . ~ · 8
dir9ctfo!:i' PO Is that.of;,: #ecre11Sin_g), · At P 1he1e ·arc 1wti hori- -~I

zorltnl forces acting on.the particle: ·connected to two .fixed points A and 13, Let a, {I.' be. th·e· natural ;

· (i)' Th.e !f:nslon T1 in 1h1n1ring;AP acdng In rl]e direction lengths: of the Strings AO and BO' ·whose ,extension~, ·b,eyonp·;the'ir'
P,{l,e,, 11, t~c dltection of.\· decretislhg. · natui.al le~gths are / an~ /' respeotlvely, Let ,\ an,d A': the res- he -I
. (-ii)· The 1e1nion r, in 1hn1ri11g .BP actiog in 1he dim:1.ion i· pective modulii of el.a~ticity of the two strings AO nnd .BO At O
PD I.e., i11 the direction of.~ incren~ii,g,
(Ndtu th11i the siring AP Is exrended In the direc1lon AP ancl
. -;,· the particle is in equilibrium under the tensions of the ;two strings,
Therefore · · · · · ·
1

. ;ji
]
~o the i'cnsiori- T1 In ii nc19: in the opposl1e dlrec1Jon PA],' ,\I XI' ,, • ··f

By.HOo ke, ' _.·.· a+:r:..:.fo . ,·a-x.:..·ja a·=.7 = (given). r


IRW, 1)m>,,-,...nrr'- nnd Ti""'"--;;j - ·
2 t .,,fl\

Hence by Newton}! second l:l\\' ofniolion (l'=mfj·, lhc equa-


T A
From (I), we have-,=- and :,;=,
. , . _a
T A'
-r a
I
... ,.;
c~·
l io11 or motion or th~ particle at P ls
No~ suppose the pa~t'icle is slightly pulle,d to...,:a·rc!s B nnd
iA-.:
m-;;-,
((/'
..
=T,-r,~ ..1,, (1-•x-a/2

1
• a J-X-aL2
S12 . ~·-.l. • (1 •
4'A.i:
a
1
--,-2 .. , _-· .
then let g9.. It !,eings to move towards 0. · Let P q.e ,f:ie P.·osiiion
of the particle after any time I I where OP!:="'• [Not'e .t~'a't w~ ,nave
0

, d ,'< 4A ·
.. 'il[I ""' .,, am ,r. " .(I ) taken Oas .origin .. The dJrectidn OP is. thilt'of x :ncre;nsing. nnd '. ~~
the direction ·po is that of x decrea·sing.J . ,
Thus• Ihe ri1'otio1t is S.H.M. with centre at 1he origin- o, Since
we have displacecj the ri11rticlc iowards O orily uptc\, !he point C so At.1-'.thcre are two horizo~ta! forces acting on lhe ·particle:
that the portion BC of the string is just in i1s natural length, there, , (i) The tension, Ti irr the strin'g AP ac;ting In th:' direction
rorc during both PA,.i.e., in lhe direction of.'I: decr~asl,1,t;, '
. the' entire motion of the particle
' .
the porlions
', .·
of' '• '•

I
' ' : \ ': '.~ ,;. :'. ,·,:,','\' '.'<. : ~-· t'.• :··,: ~,;,;) ,m<ffUQi\l'<} ii{O(\(fj; i,;\.' ;.,:..:1; --~' ,:,; ;(, ,; ,;:p: tU}t·HJH\◊T:•1H1';; ().!)'11 I~,' ,\\.'/(H 1,"j, •. , ,·1 ~}.1,()Uf.fflD~~,nnr:01: {: t:,:; tl;;~v.r
1
~f~~n.Ms.~m~·m~CTt:ft1-mnm'.flmmu·t.rmnt~m11ri.~Y:-t~;?mrr..rrr~-:r~:$!~m1.mt~nrr::u1::n~:~~1~;1~{<mmm·m~:~mHaunmudM~iH-:r;r:1\1: ,~:.: 5t;;:,~~-·. -,-:):
I
, ·~·., .•.::,::~-.,.:..-V"Y·••,:. ,·. "•
·.· ·, ~ ... ', ....... ,
.-,., _. ,, . ·•.·,·." ••!:,,tf}"''· ,,.
I,

96 Dy11a111ics f/.actillnaar Mot/011 97·

(ii) The tension T, in· the string .BP.11.ct'ing in the direction pol. Lei th~ 1wo T (~ '(
('.81 i,e,,
in 1the dirccti9~ 9f x increasing, (Note that tlie string AP elastic strings be rasfeped to a A. a • 0 b 21
is -extended in the directlon,AP and so the tension T1 !n it acts !n • particle or mass m at O and
the. ~pposite .dfrcction PA.]· their other ends be nttached 10 7i '"') r.i.
,,t/J -- ·
. By Jfookc's law, T1='=A - - and T2 =A' -.-- ·
· (1-.. x) 1
(l'-x) two fixed points A and .B so A ~ I' e
,.\, ' ' 0 · II that the strings are ·taut and ;
Hence,by Newton's ·seco1\d law of motion (Pc=111j'), the equa- OA=a, OB=b. If I and/' nre the natural lengths of the·strings
tio o of moii9n .o'f foe p·articlc at fa·'is
1
. I . OA and OB respectively, then in the position• _'of 9quilibrium of
the partiqle at 0, ·
m d-d x A'(/'--;,;} A(/+.~)
2=T2-·T•=--..... -...... _ - - -
·. a' a
'
t · •
1te.nsion in the string OA=tensi'on in string 0.8='7',
I
lr.:,·
,\'x . ,\,': b ·(,I) Al' (as !iven). .
"'"'.,.--,
a a y ·. a ,=-,\/]
a Applyllig'Hookc's law, we have

I: =-x (f+~)·
dt/1 x =-m .i?+a)""-in (r'j, +7T) '
2
X (\\''')., X
.
.
. T , a-I· , b-1'

·a-·/.
=" ,=" ---P:-'
'
Frol)l T=A - -,. we1 have Tl=~a-M
2 from (2) 1
T(I+!') t',e" , I (T+i\)=i\a
,·. =-·-;inr- .-:, ·... (3) A' T+A
: i.c,,
r=-n-.'
sho1Ying that the motion of the pnrticle is· simple ,harmonic with ... (2)
centre arthc origia ·o., 1 ' • ·1 I ,\ T+i\
, · Since ~c·hnvc g'ive1\ only a slight displ:1tement the particle tJ Sirn1 ar y rr= )- . ,(3) ·- ..
. foword's 11, therefore during the i:intire 11'\lltibn of the panicl{! both Now .suppose the p,article is slightly pulled toward~ .8 and
th~ striQSS remnl11 t:till niid so the entire motion·of the particle is then let go. ft begins to move towards Q. Let Pb<; the position.of
governed by the eql.latim\ (3). · Thus .'.the jHl\",ide makes Sll)ail the pai:ticle arter any time 1, where ,Qp.,.x, The. direction 'OP is
oscillations in S.l.i.M. iibi1ut·O :ind the timc.-pdod of 01\e c,)m• that or:,: inc'reasilig and the direction PO is that of x decreaslns.
plete osci_li'ation · · At P there Rre two h_prizon1nl forces acting on rhe particle,
· . , ~ · f mW , . (i) The tensioi, T 1 i,1 lhe string AP ucting i1t th'e direction hi
.. '·,. =:.:Jrt'(l+t')/111l1'} ':"' 2 "' L'f(T+'.'P) 7.e,, in the' dircc1ion of;,: decreasing,,
'·Rernarl<:" in ~rdcr tlrnt the ei1tirc m.otio/1 or the particle ,h0uld . The ten~io11 Ti in rhc string BP ac1i11g in the direction.
P.B i.e., in the di~ection of.~ i11qrca,ins,
re111ojin)n\ple hiirm,)'nic with centre: at d, jth•; partigk must be
· · , · .11+x-l b.-x-1'
pu!lc,d .10.waras.B• ,~nly \1~t6 that distance ithicli doc:; not nllow By H?oke s lnw, T1=A , T~=,\ --:-y-·
1
·thcis,r111g ,Ob .to bei:om~ slack, · ': . ,·. • · I '· H'ehce by Newton•~ second i'n,~ of moti;n (P.=mf), the equ:1.•
. • I~:-:.· sn. T11·c /111h1 .,,f11.r1lc i1rb1g.1· arc /asw11/d 10 apui'lich' r,J tion o( moUon of the' particle at .P. ia ,
ma r III n11d rhc!• other !'11(/•.1• 10.)i.1w/ t}<iin~s s~ ·flint 1/w str/11.r,.,1: a,~ · 11 dsx .A (b·-x-1') ,\ (a+x-1) ,
mu · . Thr'' mod11/:1,1· of (111c/d.i A, the IMslo;1 ;T, and la11g1/, n and b. 1 .d1 i=T1-Ti"""";"'-•[i"'_ .. · ..,. ....... -,---·
SM1;: 1h111 the p~rlorl n/a11 ·n.1i://{nU011 a/;rig /i11r .of' 1/J(' ,1•1ri11gs ·,1,c ·
l.r = ··yrA x-.T
A
l· ["
' rr.om . (.I), .........
A lb-I')
r .. =,\·-:-r-..
(a-I;]

., :,:.~: ,,. ·:· ~ •


]\/l ..
1110,b , ,~-. · -;.:
. A r.J. ,\] i:· •. ;.;, . , ..;',,,
1 =-1-:,._···'.1--·-· ,\', ··"":·'\,·· ·"•'(rrom:·(2)and
:lcr-:-:~,t{r(•h/ ·. [Mw11t Jl)l1I, R~. Ii~) Cl • • ' •
........
.)
.·.
:1 ~!' ;: -:i ::•."': ', ,' {.: ~ ':: '.\ :«"'. . •• :.; 1 ;:,f:lr:::•,~;. ·:,' ,., > ;~; ;, :' ''. : ·! ::-,; ;..: ~ :.. ' , '.,, ;·~;) ;5~{ :; ,. ' . '·.•,•:•:~:•·.·:,;\~'., ' ',·••r'·, :J·,U·~?~::~~:,::-• ~:,..."10:•I!
. ' , :~~·'.•",• "'
'·~: ,: ~ i•:' ''.• ·~ '" ~ .
,,.,...,.;•~l, ,",' l '•;, ,:..,,:,!'.(}:\'' ,:,
''•'' ,,.,.,..,,.,-'••

98
• .. D,1•namia Rec.tilinerir Motion
99

.\
.\
..-~ l the eqµation 9f n;otiQn 01'. the particle at ( is'
(T.;.>.) (a~b) ,
-~,-: . ,.. - -··--;n;--
.. .r, di.-. ·;,. .
,n - .., -T,.=-.... .~
d12 a
dl,-c .(T-'-A) (a+b) .1
rlii ""' - · " ,nab · · .~,. ... (4) or 'JTf
d 1x ;,. ·
""-am.-., t.J .•;(
~howlng !hat the ~otion of !he p:vpcle is simple harmonic. wilh
ccntr• nt· the ori~in 0: · wl)ich fepr1l5en1s r4 s." H. M."wilh cel)tri: nt O.and amplitude O{J,
Since we h!!ve given onl:i.n slight displuc~rnent \o the particle
towards B, therefore during the entire ·motion of the particle bolh
If. 1,· be the tiine from B to ·o, th.en
1,=tXtime period of the S.'H. ·M. represented by(!)
.
l
the.strins.s re.main 1au1·and 1ne entire molion n.f the particle is
goverl)ed by the equation (4) .. ThiJ, ·the particle mak~s small
_.t·:·· 2" ")(am)
-··,' y(}../aw,1=2 T '(,
· ... (2)
osci1101h?ns Ins. H. f',L about O and the lime period·or one com• Now multiplying bo1h sides of ·(I) 0;1 2(dx/dl) and. then'
plete oscillation.
· ;1..,,. 2.,,.-· • [
=.jµ'-;i'((T+AJ ('o+b)/mab)= 2.,.,
nrab
(T+i\) (a+b)
Ex. 51. An 1:!/tutlc .ming of natural /q(lgth (a+b) where a> b
integrating, we hnve
(~r =-¾,; .. ·
'l'. 1 +.k, w.her~.~. is n C?nstant.

and mi:;dulu.r of clq.rticll;• >. has a particle cf moss III n11at:ked .10. i 1 But at the point' B:, ,\'.=OB .nnd dx/d1~~'
·11 •• • r.b·
at ri,(ilstarice a from one end., wll!ch i.rfixed to a. P,Oint·A of a sm901h 0=-.:.-b~+t-·
(111/ .
or k,.=-•
/1//1
. harirantaf p/a11e. TM ·atlu!r t:nd of the string Is fixed· to a ,point B
so th.at 1/,e .string ls Ju.rt u11stretcliqd.- (f the pdrtlr:11' he t,dd at B d:,;
-
(d1 - - v·-,:·
-am •: • t
)f: .~ (,:~ . ' ). ... ;3)
and tlU?ll,fl'll'ar~d, .rlww that it will osrlllatt to and fro . til1'011g/J n
If V inhc vetocit)' ot' \he ·r,ardcie at 0,. whe·re· ,-er=O, tl\ell
dlsiancf b ( v'..~;±.x'!?l fa a per/()dil' time Tl' (../a+../b)\l(n1/A;.
va . from (3), we have
Sol, Let ,,f8 be an .cla~tie .4.:_._, - - · '"'-1 .,
v:=.'.'...,bQ lor . V=J(1-),b,
.stringormitural lengtha+b .. " ? ;, " all/• ,1111 . ,.,,..4)
1
attached to IWO li,iQd points " At tho poi'nt o, tic tensi110 in cith~r o.f °ihe. two portionslof ·.
A and :S distant· a+b apnrt. the string is ieround (he velocity of' the particle is V (o the· eft ·
L~t a. particle or mass m be .I ~t:,lt~
or 0, due 10 which th1 pnrtic\c·movcs 1ow11rds the lefl of O. As
n\\aehed io t~e poinl -9 of A Cl' ' ·
'1 ;
- ~
07.• .. ;.
... .
... f:
- ..
·;
the p~rticle moves to \·he lefl of O·, th~ s1rii1g OA becomes·st ck'
tbc string .such that OA..., o, 'If) nnd the string. OR is S\rctched. ' .
d'a'~b nnd a> ·b. : f . If Q is the position 01'1he p:i.r1iclc between O nnd A, (s·eejig,
. • When the par1ic~e is h\!d
~ tt . . . . . -.. . . .
,.,., I ,, ::~ 1··, L ,it 11ny lill'1e I, iirtce it st;.lrlS,l~Oving fr,1m. 0 10 the left I' it
llt B, l "e·
" "O, r II t.!" A"'
v bf ic . ;. .;s....=-:.i,.. · .. ·f
1
string.is SIJCICn~d )'lhile. I ~-
,;· ,;,~ ------,;,
,,,,/
A 6
.,l'· :ind O"(J=.1•, lnen rhe t't•nsiqn in th~ string QB is 1}1=A ~.. net n~
,,or1i"on O,B is sln·ek and ~o 11/hc.n ,111c purlicle j~ rcleascd_(rt)J11 11, tnw~rdsO onct 1h~ teton in ri1e stri~g QA=O bedause it is st,clt
It moves lowilrds O ·srar.1ing rrom reH ut B, l;lic equation cl' inotion i:f tru<r.o.rtkl~ at Q is
. :ir Pis lhi: posjfio·n of the particle between· 0 and lJ, 1,sec ng. d'y . Ai" . .
nt any time t after iii .release from 1) ond OP=x, • 1l1cn the /JI ~7,, ",~ - 7,:1 "-" -· ,t
tension in r~e . AP .hr,=>.
. sfring . ·.:.acting
a . . O a11J the ten•
towarJs
siort \r1 the strin& PB i~ zero h.ec~usc it is slack,
or
:1 J' ..
.14)
..,....,.,......,___________________,....___________---"
·1'
i ~t
WU ~
$
Fff,..,.....
. .....J .
¼1;'i. ~!li~~9,{\i~.,-!Al,,H,i\ ~'.·(•'I:·:.';; ;-:,i: \,
.,,,,,,,.,,.,.,, .•.•., .. '

-~;fii{iii{:';;JJJiUJlRWUilJ~ilDi<iiWlf~;:; {i:· . :)1Wii.~l.in1g~iiimJ~a~\~~.\ I,.; ;,:i ·::: ~ ;:, ; ,;:i};;.;,;:!hfS~;,; ~f 1~·~1<mw~w;,ur~;:sni.t, l)!.-O :.-~,, V;-, \ ,iw~

t
.· roo Dyramics

(
R.ectlllnear Mot Ion 101
Multiplyios botJ1-.sidcs bf (4) by'2(dy/dt) 'and then integrating,
we have . · Let one end of the s.t.rlng OA of
d )2 I· ,·;._' , · ·

I
.natuipt length a be auached to the
( dyi =-b""-yi+D,
. m
where D is,a constant.
. ·fixed.point O and a pa:r\ic!e .of mass
. · ; (dy)2 >, . m be attached to the olhcr end A. Due .:, :
But at 0,
Y.?f_O, aod · di .= V'=amr· I
. f~ the _weight mg of the particle the
I
I
I
ming OA is stretched arid)f B ls the
. am
~.'b'=-2-,o-b
. bm •
or
·
D=~ b'·
am !)<isition of equilibrium.of the par/ic!c 9 lo

such that A.B=d, 'then the tension To


(<!.l)o~~(~-lb i)·
. dt . · m a in the string will baJattce· the weight .
a1··. !·.,
I Cl ~
of.the -particle·
9r .·(dy)'·
dt.
.('2:- 2·)·
a l. · ... (S:1 l ;,, mg=Ts. Ar.
·If. the p~rticle comes. to instantaneous rest :n the point C
between. 0.-and
• . .
A ·such
i
thnt OC=c, th<;njll
' •
C, .v= c.and dy,'dl=O.
or mg=:11. -=A-· '·
· OA . a· .•,tr)\
,.AB' d
~:• _:_,t.l"-~ rt
T~e particle ls pulled do1~n t6 a
from (5), -we tt'aye
-point C such that BC=c ai,.d (1:ieti '}· · . .·at\· . . 1hi. 8

0=~· ,.am a .
(b~· ). . :• J(b i)
' . · >. --~• . or ·c=-h ..., • ·
•a 1
i'eleased. At the. p'olnt ·C, the tension· »l{I .. r,,.p, .1
ln the striog 'ls·.greater .th;..iv the weight
From' C the particle rei'ra~es its path ·and co11.1es to in~t:i.nt•· c,f the particlO a:nd so the particle m5.
anrus .rest ·at E. .
1
. I
1
· . starts.movin'gvertically upwards with i,,,:l, o o,
... · The particle thus os:illates .. to and -fr o through a distanc~ velocity zero· t\t. C. Let P be.the .position of tho· pimlcle a.t e.ny
time 1,where !JP.=x., The tension in 1hc string when the particle
EC BO+OC=b+;=b-+•h )(~)=
-.Ja
b hi +vb .
a is at P is Tp=>. ~+?=, acting.vertically upwards.
· The e~uation (4) represents as; '.H·, with ccnlre at 0, a

amplitude' OC and ~iine period, T' =1.,,. / J(;~)=2., J(b';} ·T_he resultant force acting 01\ th~ pan!cle nt P In the ver,tically
.. , d'irect1on=
upwar1.1s ·· Tp-mg ..,.",'(d+x) >.d >.x .
- - -mg•-+--mg
· If 1, be the t·imc from. 0 lo C, -we, have a . a a I
·r. /(brn) '
1
• .· • : · ".
' [ ,d . . ].. .
~ 1~=!-.(T')=r J T · ··: 0 =mg, a. from (I)·,
Hence the required period hi. time. fo~ ml)king n cc•mplete 'A'lso the ·acce!er~tio~ of th~ p.articlo
,, •
at P Is d 2xfdt 2 in thl I
·osqillation bct'weeri !J a·nd C 1 direc!.ion or -~ ·increo.siing. I.e .. : '\'ii 'the vertically downwards
=2°,(limefro~\ lJ to C),=:2 (11+1.i) I . direction.

=2 fLf.
11 7i +2J(am) J(!;m)]
11 . ,· J(mT ).
T ="'.(va+v'b) by,Newtp·n•s1law, th.¢ equi.tlor\ or motion of Pis. glv~n by
/l~.'{ >.x , · d 1.~ ,\
· · :1•-§ 16. Particle ~ullpeno'ed by all elastic strlni,i. A f1article of Ill d/f.., _(I I Or ([ii'•~ ':"cim X, , .. (2)
mass 1/1 is suspended {ror,•1 a'-':ftxed poii1t'br ,a Ugh/ ,e/asJh: string of This cquatip1, hold,& good so l·ong as \Ile· te1uion· oper'ntes
!1.atura/ /engi.h,'a ,a,:it/ 1noa'ulus of e!a~lt'cir,v >.,. Tire partic!e is pulled i e., when I hii ~\'ring is extended be/ohd·hs-i.1:ilura\ l.cngth. ,\
do.tvtt a ,lluli.lri i'ire'lfire of the s(riiig aitd ··,eleaied; 10 · disctiss 1/.e, ..-..
motion. . . ._ (Meerut 1988 S) Equaiirn, (2J is the standard .:qua1ion. or a S.H.'wL with
, .
•:entre a.t th~ orii;in ..B and· the amplitude of the motion is .BC=c,
,'

., :: ;:-:1 ~! • ' :,; ,:j\\;:;;;::,::


,. ;,:•:::~r . :',; ;~,;,, :~· ·: "

-------- -- -~--- _, ... , ... .,,. ,;,,,,·. ,.,,,;,,, 14 .·Ji';\;:,:tv•,,,, •.v~::-;<:

~77'·--,,..,-.-,-
,_.:,,,.;.

~)
•. !-'
I •• ¾,
;.;
i:i 102 D;•11amlc1
~) R.ectilin~ar M otio11 103
~~
~ •... , . 'ii
{j, If Ii ,,_ 2a i.e .. if.Ji ~ 1U.'. then the ·particle, o.fter C.Pm'lilg· to·
The periodic time T of !he S.H.M. represented by the equa• '<
instantaueo\\s rest, wilt retrace· its· patb i.e., it \vii! fall• frcoly
tion (2) ls given by ~ under gravity upto ,(and below A it will move in s:H.M: ti!! 'it
T=2rr /J(,~J=2rr J(°t)· cornei to iostantnneous rest at C. ·· \ · '
Ii li=2a= AA', then the particle will just cbo;ie to rest at .:-1'
remains ·,irn pie harmonic a, long and ,~·ill rhcn move downward~, retrac'ing its .path... ·
as there Is tension in rhe min·g i.e.; as long the particle n this case the maxirnum height attained by 1he partick
in the region from C to A. · =c+.d+.2a. · ... (8)
In case the string becomes slack ~uring rh·e molion of 11',: lf 11>2a i.e .. if h>AA', then the panicle will ris{ above A'
particle, the particle will begin 10 move freely. under gravity. also and so the siring \\',ill again be~ome slrctcbed lllld the p11rii·
Now there are two cas~s. ck will ni:uiin begin tp move: in'tiiDple h'arruo,nic, m9t_ion,. After
Case r. Ir BC~AB I.e., e,,;;d, In-this case the particle will to instan1acieous re.st th_e 11article will rctraee:its path.
not rise above t1 nnd it will come 10 insLariraneous rest before N lli'ustrntlre Examples· . . ·. .
just reaching A. The whole rnotio.n will be S.H.M. with cen,re o·. E.x. !'-2 (a): .-111 e/a.:tic string_ 11·(/hout weight, o/, wh/cit:-
B, amplitude EC and period T given by (3). 11,1srretd1ed /e11gtlt is, I ai1d modu/11s o/ ela'st/city is the ll'elg'ht. of
/ Case JI .. rr DC>Al:l I.e., c>d, ln tbls case the particle will 11 o:, •is :ms_pended bJ• clue e11d"and ·a mass 111 o:.
i,' a1taclied ,to the
· rise above A, and the morion ·will bo simple harmonic upto ti and 01/ii:r ('/Id, Sh,J11' that the thtU/ o/ a small )'ertlcal oscf/!atic111 is
o.bove A the particle will move freely 4nder gravity. 2rr•./(TJ1fiiig), · (Meerut.1971, 76, 78, 7S>J
Mu!tlplylng both side·s or (2) by 2 (dx/dtf and then integ~:.- Sol, OA=I is the natunJ,le.ngth of a.strijlg whose oopod'is

ting, we hav~
. (dx)'
. ·Tr =-ii'iiix
>, • i •1•1,,
, h ., .
w cro,, 1s e.const:i.nt.
lixed 0.1 0, lJ is tlie position of equilibrium· of, a parti~le of mass
oz... 1m?ched ro 1~~ ot_her·. :nd of. !he ming.. 0 • .,
But Rt c; ;,:aa BC-c and dx/dt=iO, I Coni.1der1~g rhe rqutlibrium o: .rbe po.rticll.l at .B, · ·
w~ have nlf'=thc tension 7'a in the sfrin;," OB. · · ·
- ·,
. ·I
,,
~
0 ,I
""'.'"'.' iiiii C
·, /,
·I·< 01' •=am e··.
,\ ..
AB
111g=11g _..
.~
l
dx)" ... - I ... (I) I
:, (._:
dt,
,1
,1111
o'
(c''l"x''). •
llcc:.iuse mouulus or e!:isticil 1 of th: strlns i,. 1 • f·
· · ~
I... -ti,Now.!hc.o,ir ·from..{4J,
~-;vi! n tll b~ 11g.
Vi., the voloci1y of th~ piirlic!J at A, where ,'(..-.•.• fl,4. 1
. >low .uppose the' partic!11 is pu t:ed ~!ightfy ·
'wq huvc . np10 C (S<.i that lrC'<.,!B), · :ind .th~n ltt go, .1,·.
v,,..1._ or V,uJ[-~- ' II )' upll'urds wI th. vcllll:'jty " r.
am, {c 1 -•tfi)
· the direction of V J:iclng vcrticli.Jly upwards,
am
·
.. ,(S, (ci-d 1)], . start~ n11.w1ng ' v~nii:n
Hn1 :it C l.:t f' b.: its pN,iti,,n :It nnr ii-,int .t, · ~ f
\\'h<!-r~ BP~•-~. ·r11~ dirc1:th111 JJP {$ thm. ,,r .1· t pJ.:t ~r.,.
!,
If fl is tho hci;:hl. 19 which the pa.rtlc\e rises above If, tlieii incr~·.1~in~ 1\1Hi tile dir,•~tfon 'PB is th,11•· ,,t
-.1·: ~ ; .I •
• "'"' v• _,,1 (c 1 --cl')
·-T ................ .
J~,·1·~,.1sinl,l. ,i.: I' lhfr~ :ire w,1 for~,-~ :1~tin~ ,,1i 'lfl:J
2g • 'l.amg ' ... (6! tlle;,11r1icli:; . . · . · ·• \'' .. ·;·:
,.. ,
p nivlclecJ Ii 6. 2a. (il Th~ 11·,•Hn mg :l(!tin:1 l'~rii1::1~iy"J,,11·n11·ards /,,•,.,. in
<liT\¥Cfi1.,n ,Jl ,\··in . .'r\.'asi1\i.:,. ·. 1, J
AlsCJ i·n this.ca~~ the n'1aximum·llcigh1 auuin~i.J by 111~ ' ·

"le cJ.urlna:. l1s ·entire mutlun . . ·, . · ., ") I . · ... .'-!.8~-1· .


:lllu I.ii .1 \1' i~n;"rn Tr'".flg, .... r'·7 i·n I ()J>, n.:ti!I:,! .l't!I·•
=Cl/ . .f,,/JA·i·b,.

-- ...
.,,_·c•l·-d--Fh,. (,·, upwards/,,• .. ·iii th,• Jir~•,;ri~in ~,!".\ 1fo,·r11asln!:,
••' , , . It+,,,
·'nI'~•'
.,:.f.

.'·.(
!Ii'
~-·
• •• .J· ••.

·,•.~, ••.• ,' •''' ~•.J;;c., •1'. ,,, • • • •


iUU-it~rt!~. "~)~:;t;1w.uaRa~W,HUHtW~i:, i;;; ~.;;,~-:Ht:i~: :r~~t~H( ;:.!it~NXUI~th<.;.;.,;;: •'' .. ,,'.: '.' :~1 '; :\• ;~•:r.W~H},)l't!r.~Wm·.~.1;; ,:n;.,,(~ :,· ,(':'H~ ,.. ;.·~•t, ,.

;! ..
I:,,.
'JLi:
I
.,04 i Dynallllclr
· ' ,. · . I · I Recti/i11'ear Motion 105
;Hence by N~wton•~. second law of motion,. tl1e equation o~•
1otion of the particle at P Is . (If
By J\lewton's second law of motlon, 1hc ·equatitn of motion
d 2x · · AB+x AB x of the particle at P is
m Jir.=mg-11g - ;- =mg-11g -11g
1 6 1
1 d 1x A (d+x)
=f1g r• X ·
·'.'
[
from ~I), mg,;=11gT . AB] Ill 7ti=---,-. 7mg,
dix [Note that the weight mg oi; the particle has been
ifr,- = x, . . "(:l,'1 taken with the +Ive sign because i-\, is nctlng vertically
which is the equation or.a simple harmonic motiot) with centre ddwn11ards I.e., in .the direction or x increasing.)
at ttle origin E and· (!roplitude EC. ·
! •

'Ax
7 +mg•
/
,:

.. since BC<AB; th~r·erore during the entire rnotiol\ of or 111 -


··particle' the string .will no'i be~onwslac.k. .
. Ax [ Ari
Thus 'the entire motion of the llar.tlcle is· governed. by =,-7 ,. :.· ·, =mg ] . .
eql}atlon (2) and the particle wi!Unake oscillations in .1irople
mdnic motion about the 0.entre :B. . ,11~ >. . ' g
.'. - '1=.-- • .i:= .. ,d_ x, (from (l )) .
'fhdir:ne of one. oscHlation : "11 1m ,·.
•..
= 21'VI-\
~=·1 .· . .. 2,t J('/1!)
2?T,----= - . . Hen~e the m'olion of th.e p~riicle i:; simple harmonic about
· ;:;Tf?igflm). · ng .
...Ex. '52.'(,b~. ,\1 light e,!astlc sirtni.'o/ ~atura/ length I is liu11r; by
·one end ·qnd to the othei and are iled successively particles .o/ masses
the c~ntre Band ils ~.e'riod· is I.e., 2rr.J( -:;;?fra; i )·
' . , n:i1 an~mz. U t1 '0'nd I~ be the p~rio.ds and c1, Ci the statical axte11•
· Blll according to the question, the period is. 11 when cl=c 1
. •,' sil'JIIS correspondin°g.10 these. MO weights, prove Iha/ · ahd the period is 11 when d=ci,
. . ·g(11i-1 2~)=4"1 (c 1 -c2 ), [Rohilk~and 1985: ·. I 1,;,:21v(c1/g) and 11='211,J(ci/g),
Sol. One end of a string10A. .of natural length I is attached so1that · 111 -1i1 ;,.(4,,.1/g) (c1-r,)
to a ji.,:ed point 0. Ut l3 be the position or o or · g (t, 1 - (r,-c,).
equilibrium of,a. partJci'e of ri1ass m ·atiached to
,tho oth'er end •or the Ttien A.B is the Ex, SJ .. ii Ill/US m haJ1gs from a light spring ,am[ ls given a
sta:iica( .exte'nsioh lh . the· string c;orresponding sn,oll .1wllca/ .displacement, {f I ls the le11gtli of the, spring, whM
to .this pa~~iclci·of mass ·~i. · 'Let AB_=¢. .. , tile .rysie11i'i's ill ~(JtlWbrAi111 and' II 'the· nlll'itber
I,
..'Ia the equl\ll:irilim .position . of the particle t <J/ osci/latio11s per ;·eco1uf, show thal the 11a1v,a1
,r mass mat B, the tension T,='A (d/1) ,. in ,the Ail l-•(g/41'1 111 1 ),
lt11g1/, n.f the spring Is
stdog OB ba:la'uees the ~eight mg or tlle. par• Slll. Lci'OA~" be lhc ..no.tural. length of Q

ticle,. · 1 • • the spring which c;,:,tcnds 1i1 11' length OB=/ f,i
.j., Ad . A Ta I 1 · when u 'pi\niclc or mass,111 ru1\g! i11 cquili•
,!
1=mg. qr im .... (l) . t ll i:i:- ~• brium: In the position of cquilibl'ium or lhc, tI
:". N.ow suppose .the,• pa~ticl~ at B. is ktigbtl)'
pulled d<?wn upto C and then let go. Let P be
"'J .;
r "'S
t purticle al B .. the \Cl\oinh To i'n thll, spring i's
>, {(1-a)/a} nnd iL b:llanGe~ the weight
\he par\icle.
111g of I.' A J
l·~

, ! I: i• r,
t f
••.the pQ,s,itl~11.<lf t,!J.~,.P..article. at fflY .. tim.e_J.where BP=x, When the I ,', A(({,7:a)/(1}"':'IIIS',. ·.: 1,,.. ( [)
particle is~'~tK't1~e:'i~nsion '.r/l'X' riie"stii'~g Pis A d'j X. , Now su'ppose the particle at. is slightly
vcrti<;ally ·upwards. - pulled down upto C and then let go, !t inov~s '11'1.j C j-Q
,towards B starti1\g nt r~St rrom C, Let P be
,,:n

hi
,li!j
,,if
;I ·:::,,:,.:·:
~
.....
·:,/::'.: =;:;•::~:•:::• ;,, ~·•;' I , :•:. :>\:, ·::::·:;:1:.::·: ::>.'.'.'·l .. :··:
"ti
-----------•T"1"'1"T,r;>T.".T~....--.:r
.J . . ,.. .,·.:-~•.•: t,:,: .
:':·;,, .. ,

\i
~ij
i
;
i~
·..... :
OG. • · {·
tho r,ositjon q('thc purticle'aflcr any time
. . ·.
whcro BP=.'(, When
1,
. D;Jttamics Nec1ill11~ar Motiun
velocity zero at c. Let P. b/th1;·· poslt\op of o·
07
I. •
.
I
'

-1

: i ', . \ ·. . l-i1x-" _the particle n.ftcr al\)' tjm~- !, wli'i:r¢ Bl'=,>:,


t~e µ~rtlq!e 1.~· 0.1 .fl, lhc·tc'hpon 'l"p l,n the spring OP1 is >. -j;f-, · Noto thAt we 'hlive · .taken B . .a\. ~he origin,
' '
I\Ctfog vertically upwnrds I.~ .. in the direction oh:·decreasing.
i '
W~en the pa:ticle is 11
P','\hero.a_rc.two (orces a
act111g upon 11 : ,
By Newton's, second la~v of morion, the equation of motion . :. ·op-OA·. e+x
or the pt1nicie IH P is · (i)·
. the tension Tl'=A • -
OA- · =A ..--
a
A I
d 1.-: H·.\'-a t-a As in the string'OP, (tCtiitgvertically.upwnt·ds I.a.,

.t t
·
111--·=111rt• .. ·t. --····· =11w-A --·--
d1 1 in the direction.of x .decreasing, and (ii) tne

',), ~
11 · "'
.11 a ' ·;~

';t.{
I.,. weight mg of the lMtictc aCti\l!l . vertief.lly
8
•=-f,
"',j ,;l \''
'1
[from (I!). downwards i.e., in th .dii'cc1lonol'xincreijs\ng.
1/>.v: >, · ..........
---,·w••-•.\'m- g :; [ .
lroni!I),-. A ]• J Hence by Newto 's second-Jaw•o( motion;
dt· r1111 t~-a ' o · ,11J1 a i: the eq·untion · of moli'.011 oft-he pnriielc ai 'f, is ·,
C
·.
.
""'i
H.:11cc the motion of the p:irticl~ is s/inplc· harmonic, ,I/th ,lj,; d;x e+x ;,.,e ;,.,.v; •
:ii m cfit=mg-A. -;-•""mt-,j" '-;t
•centre 111 the 1?rlgi•11.B 1111d the lime period T (/,I!,,· tll~ lim~ for
[.. fr'om. (!), ·1111,1=-·a J·
;,.,,r -: . . >.e,
one complele; osollli11lon) ... ~1r
,
·J(l-a)
!J
seconds.
,I, ' ,
= ·-,t t

Sine~ 11 is giv.:n to be the number of oscillari,111s per s~conll.


thi:rcl'orc II, r- I or 11 2 .r 1 "" I
,12.,2 . >, • g,. . [ • • from.' · (l') ,-·=·-·
-=:--x=--:-.:,. A g] .
dt (llll 11' ',· Cl//1-!J ·
. 4,r~(J-a)•, !! Thus the equatibn of motion of the pa.rt)qle is.
or .n~ ·--·-·-=1
fl
or ·/-a ....
·
,-,-·u
•;,r II . d;x . g . .
·11, 1 =--;- ,,:, ,.. (2)
\l/' u'= /....4.JL..,
niu• which. is the equu1lOti or n,'siJhplo hurmonii; motion with. COllll'C
,,I'I 11~
' gives
I tllc 1rnn1r11 I 1ens),'<
. ' ' r1 t) I' t I1~ spru1g.
' · 111 the origil1· Band amplillld<: EC. The equation (2) governs the
. ·i::x. 5.:1. ,·I ht1m:1• pt1t'lfrf<, }!ffac/uul tu 11 Ji.w:tl point hr 1111 mol ion of the Jlnrticlc so long us the stri11g. dQes· not become·
~lack.' · ·· · ·
e(d,rth' string l1t///,l.!.Y J)•,,,,1,1•, .1·tretl'lth1g tfl,• J'lri11g by a ,;11111111!,1· i•. Ir
I.~ dr11ll'11 doll'// b,1· 1111 m~ll!/01111/ dit/(//1c·1•.f' 0111/ rlim let go; d,•ter• ,•··· Si11ccF,... .- 2 =4aa=+lvc; thercfo1·e/> ~ l.e., JJC ~ A.B, So
111111<• tfl~ lwl,11l1111.t 11'/ilt'h It 11'/II ar/.s1• i/.f':-c 1r:•<l<W, i: bi•l11g 1/ic whe11 the purticle, · whitc .niovini,: ji\ simpl~ hi\rit\onJc· motion; p
///1.Sll'C•/l'/it't/ lt•nJU/i ,lf'i //1<• ,\'/l'ing; rcucl1es the ·point ,·I, its velocily is_'iwt zer.,, · But.o.i A tho stri11~ t: Ii
beeo,ues slnck und so.abovi.: ,i the purtlol..: will mµv~ freely_ uud.er
801. L.:1 (),.,,..Jab~ the natural length ol' u1t .:ln~1ic $iring.·
wl,-111,'un~ 1111d i, li.~1·J 111 0, L,•, B' b~ lhe p,1sition ..:,f ~quilibrium
''./_,(~:·
'.'
gravity. :-'·· · ·. 1 · · · · ti
of' a particle,,/' 111a~s 111 aunc/i,•J 1~1- il\e ,11her ¢11<l or tllc si'ri11~. Jt ~ -~~; ' Let us .lir~1 f'lnd 11tq.-vcl\1~jty ut ,1 fpr the ~.1-1:M,: giyeh by (2). ;,
i~ i;iv~n thnr 1/J,wq', in'ilic p11si/11)1\ ·ol' ,·qullibri11111 o[' tlie par7id,• hoth sides.ofl:;):hy'2 (d.,;/c/lJ :rn~·i1)Jeg1·utin.t w,r,t. 't\,
:ti /J, the 1cnsio11 'f'11 in the ~1ri11g OlJ is ,\(c/11) nnll ii h:ilnn~~s 1h,· we gel . ('h1.:.·. )· "'•' .....eg- x .1-k, \,/here/.;
, y is n conslnn!,
2 ..
.
11clglll /Ilg p( lh~ imrlidt, i I

, 11/g.t.,\ (1•,.aJ..
Nmv supp'11s1: 1ho p.irticlc is pulkd do\\'n t,, n p;>illl C, ,u~l1.
._ .. \ !)
ii'
I',I
JJut Ill(.',

1'=(lf..)1•.
' (,/ '

'
.
;,;.,.1Jc,.;,j',11u; (~(/1:)_,;(i, .'l:li,,rcrdr~ () .......... (K·)r-t
.\~.
'•
(''

.
.
i
tl
111:,11 /,'('. ;: unJ ll1cn,.l.:t LIii, I_! lllt-1\'<'S t,>\\ards .lJ .Jl:irlilll! ,\\"ilh · 'I ur ct' ·,
.}

&lit!Siei

;n, >fl'
#14 HWA ■

.1 •::;~•'.::· rj'
•.
-· ·w♦@NHII
,_,,,,,t).< \lr·r,·r,.•,,.,,,_.,.,,.r;>', .. ~L• .. t-•- •-·

pvlf:!(;fac;;:.;,:,:,;.~i,;~ijSl~~IHi.,,;,,,;,;.,,;,;,:;,;.;,i;:,;/,i;:::i:ir,m~i,,,,..:,:_:;.,.:•: •,;,:,: ;.,,:;,,,.;;,v;:,,,:,:,.;,:,,t;,>,1,;!,tn>!,•,',s),1,/>,>'.""'"'"'"""•''"·''•'""

I''
1{.
-.
·1'
.::,. ' <;,,\•

. Ra'~ll/inear Motion ·I 109


108 . Dynamics
' 1Wheil the particle Is nt P, the.re nre thre() fotces act\ng \lt)Cl\l
dx
.-'- = __
( dt
)i.
g x2 + g..,- /2 = g.. ( fl 1t:
, e ·-· e e .. ,(3)
The equation (3) gives the velocity of the paniclc at 1i1iy poitlt Ii) ThetensionT,=>. inthcstringAP nctlng in
, a1
from C to A.. -Let v1 be. the ·velocity- 'of .thJ particle at -A. Th'en at the _directiou PA I.e., hi the of :e decrenslpg,
·d.J{ dx )2 1 . . .
A , x.=-:e an \;di =Vi , The:eror_e, from (3), we have (ii) The tension T,=-A --.--•• 111 the string /JP acting in

V1 9 =(: (.f.'--e 2 ),,;,[.·.4ae . ·(•·;· J the dkection PB I.a., in the of;,: ln;rcn.sh\g,
.. e e .- . (iii) The weight me of the, particle acting vertically down•
=4ag, the qitectiqn of v1 ~eing v~rtica.lly upwards. (Ji· wa,ds i.e.., in the direction of x increasing. /
Above_A the motion of the t:;!l,-Ttiole is freely· undyr gravity. Hence by New.ton's second law· or motio1/ the equation of
· ff the·particle rises to a height ll"a.bqv_e A,,we have motion of the particle nt P ls
a+x-a1 · b-x-
0=v 12 -2gh,. · [using tbe formula v1 =ti 2 +2fs] m ~x-di ::..-A··-" --1-A --.----+mg
t a1 ·
=:4ag--2gli; (',' "it1 1 =4ag),
·2gh_=41Jg or h=-2a. · ;,., a-a,+;,., b-b, >.x .\,,:
= - -:a;- T -m~--'ii( -ii;
Hence the _tota\.hei_gh,_t to which the part"icle 'rises above C
. , =CB-+BA+h=J-+:e+2a, :!->.(!..+~-);,:
111 01 ·
(by(I)]
Ex,.55_. A h~avy pa,'1ic(e Is al/ached ro one point of a,utitfon11'
elastic' s1ri11g, The ends.of rhe s/r,i11g are ,attached to two potnrs 111 =-i. (~~~J) x.
t a vertical lille, $hoiv- that.the period of a ved,.cal ·osci/laiion ill
which the string - ren1ains,. taut is 21T.-./(mh(2A), where i. Is -d2 = - -i,. (a,+ Gil)
dl X
-·01-0-1· x, w I110,
' 1 ' h .
1s t e equn\1on o r 010 (.10n 'o r'
I m •
coejjicl~nt of elnstlciry_ of rhe string and 11 the harmonic mcal'l' of a•S.H.M. with cen-irc at the origin 0 . .This equation of motion
unsire.tched lengths of the two par/s of Iha string. ·holds good so long as both the_ portions or the.string reinahi taut.
· Sol, Let a pariicl~ of mass m · be attached io a But the initial displacement given to the p11r~icle below O being
poilit O of a s\rinir:"hosc e.n,ds have,_b.eei, f~stenec\t'to s'mall, both the portions of the string ·must ren,ain ta\lt for ever.
two·ffxed points A and .8 m a vertical line. i'.he Bence this eqt111.tio1,,gov{nu the e1Hire motion of the particle.
·m•, Thus the entire motion ,of the particle is simplo harmonic about
string is taut and. the par.tic 1e is 'in equilibrium at O; }i
Let OA=a and OBr=b. Also let. a, and b1 be the
;•.
;t the centre O and the time period of one complete oscillation
nntura·l l·eiigths of the sfrctchep portions. OA and 0.8
0
= 2,,_ J{•p1_111b1;, }= J{. in (2111b1) }=i1r J(mh)•
. · A(a1+b1) _ .!A (a 1 +b,)
11
2,\
of ihe string. · - · · · , b .
r,
'Consid'ering the equilJbrium of the particle at O1 where Iii= ·_.r,~b'-.· is-the harmonic _mean bi;,l ween a1 and bi,
· 01 •,-
~
I
we h_ave the. :n:sultant upward force= th~ r~sultan1
down'.(•f(rd rorcc . I · · Ex. 56. A lig/ir elastic string of natural length I has ona
i.e., the. tensi,rn in 0.4= the· tension in OB+ the 1'
·e,1 extremity fixed ~I
a paint O atid the other a!tachcd to, a stone, 1,,e
·.. , _ · '::~ig~t of the pa.rlicle !) weight of which in equilibrMn. 11•01.i/d txt_end the string to a lr.nfth
A ta-a,);,A (b-b b.'.Jl+mg, I,. Show that ·tf tlie sloMe be dropped fro111 r<>sl at .0, It !I'll/ come
:e . . 01 ' I ' . .. (I)!
\
,p d~p/h. yl (1?-1 2)· bel9w tlie equi/i'brium
to' it1stant'a11eous res/ 11/
Now. suppo~~ the particle is_ sl)ghtly displaced towards Band position, [K,anpl!r. 1978; Mc_crut ~9, 84 P..:~.? ;p, AllnbabDd 75)
{ ·1w $()1. QA=! is the n~ti.iral lcngth of a ;trincr \\'hose on,c end is
1
hen,lct g~:·o·~ring this' slight djspla&:.Ment of the particle both
,1he, portions' of. the -siri-ng' re.main ltt'ut.· Let.? be the position or
? fixed ~t 0, ·B is t_hc positi~n of'e,quilibrium of a stone of inass 111
..
I he p~r.ticle after any time~•. whe.~ OJ'f x. ')r•
''

I

'.."••:, :,::;:?!?-::•:,:' ·' :;,.~::••'·1~•f;";:'.~!~1;~i;'-• ,;,,v;,•-::


;•,•.u~,<:;:g. •~:, \ ,' >\' ,;•, ,., • ,. '.' :• \\ •! :, ,; l.~;'i~~l': /~~' 1·:.; • • .•}• •f: •~i \ .; nYf~~•:,,,{~•i"; ,; :1 .1}-:~::iGim:;:, ,! ', ·1'.' ,,,f~

··-~""-"'"''""""--
:.~-
-'---~'--'----,-~---

' .. ,,,,.,.,,.,;,:• '.·'/::,·.,:·.- .'.:•~·:~:i: ·.:;.;,~::~~:-~,'.-:,,: ., .:. :;:"~\ ~l~ \:;}; ,_,_ ...; ... ~;;:-, .•:
n
~•i
:1
:E
1! ll0 D;111amirs Rectilinear Motlon. 1n
;:~
ntt,nohcd to· t lie other rncl of the ~t_rlng 11nd} 011= fi, When the
~ -~
••.
5to.ne rests ·nt .B, the te1,slon Ta or the string bnlances the. weight
[Note that lhc 'force.acting in.the'· dir~clion or'.x fncrea~lng
~ ' •~l •

the stone, TJ,erefore


has been taken·with+ive sign.and th~.t in the direct/!>n of.x. de_c,r.el\•
· sing with -ive sign], · • ·

II
1
"u1-1r.
To=·--:,- =mg, d2x 11 · ,·
i ... (I) Thus dT:'.t !: = --
1m .ll, . .,... (2)
where A is lhe mod11lus or ~lnsticity ol' 1\1c string.
Now the .swne is drupp~d ri-,on, 0, It
whieh is the cqu11,tion of R.S,H:M. with centre at· the... origlJ.i B. ·
The. equation (2) holdf good .so long as the st;rini;;' is. _slrctched f,a.,
forth.emotion ofihe stone bet'wecn A. and C. ·
'.
I f:ills the• dlstnncc 01{ (=I) freely und~i'
gravity, If 111 'be tlie velocity. gained by
of Multiplying (2) bt 2 (dx{tl;) and integrating·w,ry 'i', w~!gc! .'
l ·1 o'i
!
i
the stone nt A, wo hnve \'1=v'('i.g/) 'down.•
wnrds. . When the stone falls below A, -the l : 11
dx)i·
(d-t . :=--,- ,:1:i+D, whe1·e D
i>i
Ill, .
· · • ·. .
1s a constant".
•• •

~tring begin~ 16 · .:x1cnd beyond its naturnl At' A, X= :...u1 -f) a:nd d.~/dt~y(?.gl) :. . ,. \
length· llnd the . t~nsiori begins to opernte, .A . ), ', 1'' ·. ,;,,.. : ' •
,'1
. During the fall from A lo B, the rorce , of 2,-, 2gl=--, , or. D=2gl+ 1-m Oi•-I)·,.
m (/·1 -/JV+D
'8 (dx)~ ·· · 1., 1,., · ·
tension nctlng vcrtic:illy l\P,wards r~marns .· h >, . i· .• A (/ ·
less Jhan the weight or the stone acting
"':rtl,;:ally downwards, . T,hercrpre duifos'•
8
'p
~t ' ·:i:j Th us,
.
w.c. iwe r,..,. =-...- -.,; +Ji•,·-, _t-:- )'. · •
· , r.,t 1111. · · I'll , •
(
.. 31
. "'.j ' The equation (3)/give~ ye,locity of'lhc s\~ne o.t any poi rt1bet'..
tlwfall from A. to .B the velocity of the.
stone goes on increasing, Whcir the· stone f "tl' ll'een .1 nnd C'. Ar
' ,\
q,'
.i:=a:; d.rfdt=O, Therefore (3) grves
' .,:,. ,\ ..
'begins to rail below .8,.its vclo~i'iy goes on c ,, 0'"",-1.,... a~.1.2a/:J.. _ (/1-.')Q
. decreasing because now the rorce· of tension cxce.eds the weight.or nr I.' ·"' ~: hu ·
•,the sto_ne. 'Let !he StClncr:omc to· instan1anoous re~t nt C, where· . or a.'-~ 2!l'I +-.!,..,., U1 •:- I )1 ,,,,.'o
J3C..,.fJ, \ : . , ; U1 111•••1) I '

Pming lhe motion of'.tl\c xio1rc below A, let P.be its p·osition
··[·. 1:, ..
· ·: rrom(l),·Tm""'T;:::;t·
g]
llfler a_ny li~.e,!t., 'IVh_crc Bf\.;.,x, (Note th~t .we hnvc t1'.lm1·_ihc posi •.
· lion of eq\J1l1br1um B of the-Slone ns or,grn. .The d1rcct1oh EP is
1
or l1-u• ,j-/ 1 -l=l,+I'
• '
. thnt or X lncrcnsind n11<;l the dir~ctihn PIJ js' thnt of;,: deci:ensin,!J. or a =(/ 11 /) (l 1 (l)•=l?-·P,
1

tlie WH1c is nt P, there Rrc11wc forces :icring upon ii :


' I,,..,,.,
Tl ie t~ns11ln .,.. I Ui-:·X)-,/
· T" 1n l st1•1ng.
. 1.1c . OP· tlctrng
' '.in
:. a,.:y'(j/ 1~-.
E~. 57. AUKhl e/a,rtir srri11g .who.re
nM eiul fixed ro n point O, and to th<' .oilier fml is-',attt1ched a
//GIi/i'ii/ lc>11g1!i is t ·eig/:1
has
the direction 01' l.e.Jin the dir,•ction of.~ dec1·e~si11g. H"ilfr/, in eg11ilihri11111 11·011/d. prod11r" a11 ·e.-:te!1lio11 e.• . Sliow hfJt ,'f
· (ii) The wcigh~/111.{( or the sio11e nctini,1 v..:ltically, down~ords 1/te 11•r/g/J1 Ne let fall from rrsl at 0, It· will rome· to ·st y t11•
u,,, in the'directlon or.,· incrcnsins, s1a11ta11eo11i·l,1.' ~1 ·" p~lnt <1i'mw1 ,1(2~e-1•."') bclu11•·1irc• posi lot;<>/
Henco by New1·,)n'S.Mc,111d law or motion (P=m/), the equ11•
equi/ihri11m. I , · ·· ·
lloll of motion or thc·slone at 1'1• 'is Sol. Proceed :,,~ in.th!.: .prccecli11s eX.\l!TlPI.~ So. Tnkt> !=a,
d~.,. . (/,+.t-J, .. I I (/,-/) ~,,..
l1•-·I•·,(! or ,,~,1•+t1 .. Then the rcq11i,r..:d tl1stnnccr:•v(/1°·· (l)
m diU ":111i:-,\ ·-·- 'j .. =/Jig-,\ - , - -7 v'((r•i•a/2 · a'1 J,·,y'(2ae-1-c•').
>.x Ex .. 58. If /igli1 elll.r1i1• sid11K 1'1/ na.,1w,t1 h•nJ!/h a lta.r une
f"'l:1--,
t rrl'\111 JIJJ. rxtrl'111i1_1' Ji ..ml at n ,11oi111 0 amt 1/J,· ot/Jrr n11ached to <I. bodr of
mass m. 1'hr r1;1ti/ibrti11nlrnx1h 11/ thr ,wrinK 11·irh the hod;• a11arhel)

___________.....____ f

..... IIIIIIIR1l&t

·, '. •; :~• ;·::, ·. ::•· , ·•;tr•:·},.,;t:~·i,\:,.:~M~•'<~N'~.'iJi't('i:•~•:A;,;~;. ,~;.;1 '•:.: ,: ,,,: :: ,.1/!\•'.r»~•»C't,l:O!❖'/)}t,r)~•;•:-:.J, }·!, 1>;<<:'(f:1, t;< ;~, ••'i"Nf.}';~RfSM«'ill:,!n:ON•'..<;~~n;.-:~,~
f@..:~J.S~~~~'Ht;~1~~~!f~l~~:i.~irtm~~~9::;t~~~~~~n1.~i~''.t=~rm--v.·1: : ',.".1'l!i~~t~~i~!~l'l~~FH1!~; -~i"f,'.'.:~:HJ!{~~.tU!·Mi~mur~~Wf~Y V!1'.t;?~~n~~:-:•·,'' ,. ;· :'
........... ,

~mu.wwimmw1w~~,itW~Rti!U:WJil!JJUfm~;;::i)ift;;;;::~::;;~;H~$~);;Ul~:Uia;i~llDt,·~.t;:, ;,\)~W,•;\:,<•:{OJ~lt)}:,1f!;,,f,'!(tWr,:•11;:1,:·h'!•.',!l!.,''

,\'.

·1
.1· 112 .. Dynamics t

1: l.r a -.sec·e. . ,"//1.o'w that If I/le f;od)''_'b~ r(roj1ped .from rest at O it P.ecti//urar A·foi Ion m
' w/11 come 10 lnsta111at1eo11s'res1 {J.t a depth a u,1i'O-belq»: Iha posi'tion
of equllibrtum: ·. . · · Hence by Newton's second law of motion (/'r..m.f), the equo.,
· Sol. Proceed as'ili E;o.mpte ~6. T'ake l=a and· l1 =a sec 8.. tion of motio11 of'the po.rtlcle at Pis
We have then, the required depth below the equilibriu1 position , d1.~ rng .
._.,J(n; se(:)1 o·~•a')=.a./(sec2 8-l)=a tan p. m -dii
l
""'mg--
• .a
ta+.~)=- mgx
. -,
ii
- - __ x, 59.· · .,4.rl~eavy. ·partlc(e.ls at'tac/1ed to .,ne end of an elastic Tl d~:"< g .
striug, ·tlte oiher'end of which Is jixi?'d. The ·mod11/11s of e!asticlty 1US 'Jii=-,i'X' ,f,(I) ,I

· of ,tl:.e string Is eq,ual /() the ivetiht of the pa,iic/e. Tile srrl11g is whioh.:is the equation of r. S.H.M, with .c)!ll.trc,:11.t· 'tM oriSin· 0
~/'OW/I vertlcall).I down: ill/ Ii ls'./aur 11ni'es'• ,II:- natural lan.gth ai1d a.nd. the amplitu'de EC= 2a whieh:is grcnter tl;l.a:,;dA:B=a, .
t.hen- let go, Show that the particle wi'/1 re11Jri1 10 this point /11 time Multiplying both sides o:t.(1) by 2 (dx/dt)· and lntegTlltins
J(i)e; +2\/'J l wl,~re ~ i.s the n,atura/ le!i_gt,'1 ~( the string. w.r.t .. 1, we hnve · . · /

. · [~uckno_1Y 197G; Kll'npur h; Agl'a 80; :vtwut 88] (ti (Jidx)~ =·-. ga" 1
,"C'
.
+k, where kin, co11st,\llL'
. ' .'

: So'!. Let OA=a be the nat·;ral lengthlor ol\ 01 ro At the point C, x==iC=1a, µnd the ve.locity dxjdl=O; -

I
·el~ti-c string who;e on,e end is'flxed at O. Let I, . :. k.,;l.4(12. '
B e tl.ie position of equili,briutn of a particle •)f a a '·
•m ss.1,:, attached, to the other end of, the string dx)J
ai g
'""a ; '.
· a ·· AB=d, . lf Tn is the 'ten's ion ii, the stri11g A .. A ( (4a--.~i), ., .(2)
0 , 1hen.by_Hooke.'s law, . · · Tnki11g .~ci.unrc roM or (2), we huvo.
-1. ·. OJJ-OA. · d
To=~ -o;r- f:' >,a.' .
d,
1
}1'?-j(t)
v(4a~-x'>t .
.,. ,wiet-c
~-Is lli.e_mo.·Jui,u.~ (.l.r el_a'stl:i,yof the string. . J ale, the -ivc ~ign h11s be~n ;aken beonllse Ilw piirticle is rno:4,ng i1,
C n~iderll\g t_he eC!,Utl1b_r1un1 of t~.e-ptt.rticle a.t B. °l , . '1- the direction of·x. decrcaslhg·. .,,.. \ ·· ·
w have .
m ·=Ta...,;,, ~.dmg ~. ,-. [,.:
. . a a. .
. '. · .
),=:mg, as 'g.iv~·n]
. i.
'T1!J
.
7p
r :c
9
Separating the variaoefi, we have ·
.
. dt.=- J(a) ax •· • \
:;t v(4<'1~-x•)' j . . ·... (_3)
·. >◄.ow 'the, part.icle is ~ui'.t:.d fown to._ a.J!>oin.t ('.
Ir /1 be ihe tim.: rrom C to A, tlwn l1W/gmtlng (3)· from C to
A, we get · ·
h.thi.t0'C=4a,nhi:!1henlet'go. lt.stam/novini;1
arcs ~_:with 'velc,elty zerci :r.'t'<;', Let be: :h~. .
siti(.lll of the f'H\rticle·at.time 1,· wlie.rc P>=x c
.
-i
l>4Nl
. .""

or
Joi •d1=- .J(ag.... )fta· -vf4T-.l
1

1,= /(
ci.i: -

a.... )[ cos- 1 X
1
11

] •. ,:
.
1
)

•(rfote t.hat ~e bo.v.c ;nken the p~sitio11 ~f q,iilibtiun: B ~s origin. ' " g _a '"
!
Tpe d1rec\lol\ BP 1, 1ha1 of x 1ncrc11sing n11d t\·.c d1rcc11nn PB is
at of x decreasing.) ·
':·jff} (~os- 1 (-·½)-cw' (1)1•j(}). ~;.
1ii, Lee 1· 1 be th~ivclocity of the particle nt A. The11 at .A•
Whcn-the·pariicle is nt P, there are two fc;,•es :,.cling upon it.
, ;,:."'1•-·a nnd (dxfd1) 2 ... ,,.1~. • ·
J,:
.i,;: " . Tl. ie tCI\Sl(H\
1,1)· • . .
iiig
=·a (17-rf
' ) .tr, l I\~ ,1n:1g
• OP net• S~\ fl\lll\ (2), wi.: hRVCPi;:-J(g/a)'.(4a1 1) . -a'
or · l' 1 =,/(lagJ, tlic diNction or 1•1.bcin!'.I vcrtlclllly upward$.
iri the.direction PO i.e.-; in the: direction. of ,"C dccrcnsing.
.
·Thus the ·vet(.lcity 14 .-/ is . \l/3a,(,') 1f11d I~ in t.lie'upw:trds dircc.
of _the piirticle .~ctin~ vcn(cnlly down-
.tlon sq thatJ~e. eartipl~-rl.scs ubove '.1,;; . ·:Sln<::A .rhe tension pf t~e
,,.,,,,M"''""' of ;t>1trcr-eiisin°g;. stri\\g vanhhc.~ .!\( A, therefoN _:\.! )f the sin,r,ile 'ham)Ol\iq motion
' I
...,...
ceases and the p::.:·,icl~ ll'h.~n 1·.i,itig nbovc A n1vves fr~ely u11der

__;______....;.-------~---------=· ..
.«i ;

ti,.
,. \11~'1'!"1t"rf",'7'f!'1,,ie£AF.o"d~ibP::3,

[i
··rl.
M,l ~ , . ,.,.,;
"· ,.,::•:::· ';'::::::::::: :.-~,: •.-;'j_-:•:~~::~;•:\"I",• ~.,,.. ;,;
IM:

--------~---~~'."1',,'
----.-.·.-: ~------
-·~-~.J.~- ·~-~---~-

;/'(' ··:-;i::::,;!.;:.;~~! ;:~;,(~!j;lli\~ ;,: '. •.~:.. ·J. ·' ,,__,·;,./ ,4. '.. :11HUFJX•~•;~...,
,;,:,.,:,:,;~:.•:, '
:,', ',-,' ..•,•;·.s·:•,•,

i
~
~ I 14
~ Dy:ram.frs
R~ct/ll11ear Motion · 11 S

l1
gra-vi(y, Thu~ the p4r1/cre rising from A with velMi.ty y'(3ag)
moves upwardnllJ !his velocity/~ destroyed. The t'ime I; for this
mM/on Is given by
' •' · j(3a)
. During the motioi~·of tbc·_pRrticle bclo.w /4, let Pbeits·p~slt(o<1
after any tlcM 1,. where BP=x. [N'o)e'.fh~t we ha.v~ tn,ken the p~s)•
tlo·n or equilibrium B ·or the pa,rti_c,le:us origin. The 1llre0Ho·c:.e:P 1s
.l 0=v'OagJ-111:, so rhat'r,-1.. F". • that .of x i_ncreasi~ o.~1·
the. directlori'PB inhat of~ ~ecreas~?!.t'-)
. Conditions being the same, rhc equni time 11 _Is taken by the When the pnrtrcle 1s at }.',there are. two fo.rccs nc1mg upon.it.
\ particle in falling freely back ro A. : Prom .(to C the purllcle wil I (i)· The tensi011 Tp..:-\1.t-: in 1he1.wlng OP; acting in ·t~e
t·:ike the same t!mJ:·;, as it takes from Clo A.. Thus t~e Whole .
time tqken ·by the ):lut!clc to rct1irn to .C=2 (/1 -,: 11) direclion PO I.e .. in tni: direction of.>: decreasing, . , . .

[j(}.) .~-~ J(¥) J~ J(; )[;~+2vJ }'


(ii) The weiglit mg oft.iJe.particre iicti~s vertically downwards
==2 i.e., in !he dircc!ir.11 -~r x incre:1sing: . · , .,
·Ex. 60, A heavy partlr.fo of ,n(lss 111 !s a11ached to one e11d o.f
Hence by Ne11·to:'I'~ second' luw. of 11101ion, ·-r~c cquatioh l1f·
an. l!{asr/c' string of natural lengr/1·1. w!ia.re orher endl,r fixed at 0. motion M tile ·particlo: .nt P is . .
d 1:t d+,i:
TIie p,arl/c/e ts ihen, ler.fa(/ /rim, ·u.st.at O. Show '.frat, part of tit<•
moi/011 Is shnple haimonlc, and rltar, (f the grl!atest depth of the
f 'ifii.=.'.i1g-A . 1, .
particle brlow O Is l'·Mr 1 (0/2), tlte. modulus of e/asfii:/ly of the Ad >.;: AX h.. (
=m1;-7-:7•-•·---, y, I).,<.
Jtr.lt1R is smg tanU o, ' [Meerut !98RJ ' , . I \
(!Z;r, ). ',, I(
Sol. Let Oh,../ be the nRt11r11I length of o dr 1 _,._ Im x-:,· J .1:,. "' (~)

['.'·, fr9rn
' ,\. t]
nn elastlc string wlws,,: one end is t-ixcu nt o. 0 jl.W•O
Let B be the_ position of equilibrium of'l1 / •· (l ), 7iii;."d
rrnr1icl~
1 or m11ss m 11ttached to 1hc othc, encl I
or 1hc string und lei· rf.B..,,d, ,l11'the equ/ii• A The equation (2) rcrresen1S n. S. H. M. witli ·ccnt're n,t. the
, _B_and 1?.mpl!tude DC. He_ric~ !hc:'mQtion or the partlcle below
brlum posltlon: 111 JJ, the tension r.11 /11 the <t r.
string OB bnlanccs the weight· lllfr or 1'10 pnrll· ~ 4l
A
Q'
11' ,.~ simple f,nrmo 11ic, · · . .. · ·
clel. Thci'cCorc, 8 Multiplying (2) by 2 (d.rfdr) and· in1cgr1ttjng w,r,t, •,•,· we get
(dx)'
!,,
. d
Tn=A •·· -mg,
, ""7
r:I /.7 Ji = ... g'd'.:t 1
-
·
D, where Dis a yonstarl_t:
· ·
, • I . , ", (l)
where A is 11,~ modulus of. clasiici(.y ·or 1hc ["'J At rhe ;1oiiH .{, .~=--d .and the veiocity=dx/dr,,,,'-.;.(2g/),
siring, Now the !)llr!iqlc is dropped 111 rel1 . c .', D=2gl+gd.. . . · .
Crom O. Tt falls tho dis·:ince OA freely under gr11vi1y, If ,, be
1
. • ( 1·
. , . we novll,, ve o,clty)'= di
1 (di)' =-;t .1:•+?gl•l•g1, .n: . ... · .'
. ·... J)
the,velqcity gnihed ·by i{ :it A, we·hRve v,..,,y(2g/) in rh~ d1,w11•
ward dir~cttqn,.;When,thc particle fulls below A, the ~Iring 'begins .. _The above equanorl (3) gives tJ1e vclocjty l~r the.•partl.cle at ~n'y
to extend bcyo1\'~ lt.~ niltural lt1,glh nnd th~· tension begins to. op~• pomt between A and c: 1At C,x=(JC=•OC-OB,,;,l,cot 2 ½11-(l•Hd).
rate, Puring the fall from A to D tne force of tension acting verfi• and dxfdt=O . . Thm·fore (3) gives· ·
call~ llp~v~rds·remainUess•lhan th~ wciahr of the pnrlicfe acting
v~r11ciilly downward~, Therefore during the fall from A to E tile
0=-~ f(I co~ i6-l)-d_]2+2g/.i.'xd
I

or
v~locily the partlclc·goes. on.increasing, When'the particle begins
to fatt below B; its vcl_ocily goes on
decreasing becau~c now the
=~-f ((/t·o1i !O-/f+d~-2/d(co1s'!II-J))+1Kl+,wl
force of l'ension ~xceeds th_e weight oft he particle. L,er the pnrtic/c
come rci /n~t.1'ntabenus ..resr ,I\! ·c, where 'OC=i cnr• ~8. ns givrn,
""'. - l~·
r ,
(I cot' •W-(2-'i;rl cot 2
~8 · ·
] .

·[ir (I CO'.' ~B-+)_ 2-:--2,q/',i:Otl ,~] ; [ ·:. ~,·;,,..~/ by (l) J


·!\
-~§.~:;:~;1:t;~:,::.;,};:~i::;~i$~}-~;i;,!: :. ·i;.:;,!~(~~•:;;;;fr{~~f!(J.!1~}!i~!~ij!~~!,f;♦,·. t,;. \: ;, .; ;, ;:,;!; 1:~t.;:,:,:b;.;Ct:iM1N,,1';\1..;;,,,,t;;;111: ,V7,"\".',•·c,-w,>. H,

~--
/Ucli/i11eai' ·Mo/ion· ·
\
,,.. . .: 117
lb 6 Dynamic.
Dy Newt(!n's second .!Aw of m\Hion, the cou~tp11 of motion
· _ ;mg/I C0(1 '/_,.// '2tllg',C0l~ ,U0!
of the particle at P is ·
A..,. (/-cotl~)-=7i1= (C<Sti"le~i)T rl 1x .'.1.mg
i' -....- 2ing c:ot 1 0 . ;i i 111 dii=-mg-T,=mg-2mg a =-;-·0 - .~.
I .{dos! -!ii-Rini tBJi' sin !B ~ d 1x 2g
·te sin~ M!
m:rr,4 co~
=½·+-'1 -·---.~-'-·=Jmg,
.,
1
I-,-a=fmg
sin• 8
cos,. 11• cos· " tan• e, which is the equation. or n .S. ·H, M,
cfi"l= ""ii'"" x. ' .. ,(2)
with centre at the point C
. '.Ex.'0111'· end iof fl /(gilt el,a~tfa ,Nrlng _of natural lcng_tli q and amplltu<;ie cp, · • 1 . •

aid i11od1.1!11.rof t'lasticil/,21t1g is'a11dcli.ed to a fixed po/111 ti a11rl 'Multiplying \2) by 2(dxfdl) and Integrating w.r.t. 't\ .we get
dx ) 2 2g. 2 , · , ·
tlr,e othlr _c1fd 10 a partl.dla 0/1,:iass m. The p.artlcle i11ilially hel1i
n I rest tit. A,Jsl~t fall.· S90.w .'.hat t,he .greatest extension of '/i1 ( Tt· "'' -a..:.:
+k; w~ere k ts a c~n.staot.
Sl!ing is '!a (I+ ,/5) during tha fflotlo11 and show that the partfrlc At Hie point B,,the veJocity '
1v II reach back A ,ag(lill afier n lime (,r +-21. tan-': 2) -/(2a/g), •
,'j =,J;~/d1=:..;(2ga} and x= .. (i.,,.
' S~l. AB=a i~ the natural _length o'r An elastic st1•ing whose '~ • • 2g a1 , · ' 2ga Sag
· d · o e end is fixed ~t A, i,_et C be the posi\iO.n Pf equilibrium of a k =2ga+a• 4 = ;2ga ,'l-:-4-,':"',T.

'I
p r!i'cle Or rriuss 'm ntt?.ched to· U~c 'ot,~e1· . A I" ;be/ •o I ·W e· have ( dx '2g x 1 +Sag
r,·)~ ,.. __ ---•. .
i: end ,of· the string and_ let •BC=d. · {n /he · .! a. 2 ... (3).
IJ pos\t'ion Of· equilibrium- of ihc part;cle at , The ~quat ion (3) gives the vel6city of. ,the. parlicl.~...11;t, aoy
n
I'.'
, . d ' Q
a
point between Band 1). At-D, x=CD and dx/dt=O, So·putting
C; the tension Tc='A 2,111[( "'in the s,tl'ing d.~/dt=O in (3), we )lave ·
1.r ., a .
. 2 1 5
ra x. +
• 0' = _..1,_
~j AC b~l1'111~·cs t~e 1velght mg o{ the particle. °g or

· ··
. ' '
:. m,rr=2mg (d/a) or d=a(l,
' '
,
... (l).
.
8
Tc j a'
el 1 2

Now 1l1e partic)e .is droppe<;i, at rest d or ,-:=;vS=C'D.


from A. lt ralls. the,, distance A:iJ freely ·C C
th~ greatest extensip11 of'-the string_ .
under. gravit'y:,' :I( v, be the ·vel9city ·gaioed ,,,;,3 T,, i=;,BC+CD=!a-H'avSa;> ½a (I +v.S);
it .B., we hijye ,Vi.=-/(2gti) In 1-he downward
dir.ectlon·. Wh.cn the particle falls below B,
;the.' siring ;begins· 'to• ex't<H)d ·beyond it5 1mg
Now from .(3), we_ 1i'ave'(i )\• 2% [! ~~•-•,\' l 1

iia11,1ral le.ngth 'Tiid. the Jens{on


1
-begins to . 0
{ . :, dx
ili"'", J(2g)Jrs
a l'4 a_-x·, t he +1ve
. ·,
1 :,]
Slgll,
h as. b~en I ,
ll\~Cn,
. 'operate,: ':o~r1ng_ )lie fal! from ,B ,\0 C·th; velocity or the particle because the particle Is moving in the dl1·cc:tion or ;{incrensing .
. , ,goes 0~ incrca.si,ng as the tension remains less, than the W<;lght of
thd particle and. When the. p~rtlcle bcj'!ins to (al! below·C,.its veio• Scpa.ra._;i~g, lhc vu_riubl~i, we l\ui•e di J(~i) v'TTt1;~:;l
city' goeS',011 decreasing bec:i.use now· th~ fo,rce ,,f tension exceeds· ~C.I•
is tile time i'rom /J lo 'D, 1hcn
!lj!l tli'e weight.of the particle, Le! the particle conie to lnstRnln11eous
Ir 11
/(U. .,. ) J(0\ 1
't/;{ •
·1]{ rest at .D, ·· · . · · ' r
/l
'It= 1)/i
-~:,.._
v[,1:i"'--.1.';J_
'
!.
· :'. Duril"I!! the.motion: ~J t1,c particle below B, let/' be, ts posi•
tion. after any time 1, whe:re· CP==x, If T;, be tile tension in the
\
(Jf
·"

>1> <.:'d(
I,:.•
, 2g

lg
-111i

r,
-• ) 'S/11" 1
II i•
l ,. •';;i,y(~5U9,
'X
•• • - - -} ] ' " ' ',\J/~
.,,:·1
,,.fl · , . · .. ·. ·. d+x·.,:· .... ,1,a+.r ,.. . . , ·
l( ·r1r1n_g AP.,,\l/d':hl>iVP,•.Tr""A ·-_-C1-~1·.'.l111g,-• :;.(',, a 7t,ni; wrt1c:1lly 1111• ""J(~)[ ,in•' l.+~il.1·
1
y.5]=J(;g)(;-j. l:lll"
1
1) •
!\:1,
~I
10
'H
l
:I.
'"nrd·s..
..
li
•ilfl,
I

lli
ll 1

:-::;:,?::::; · :,:~ :.. ;:~1,;;;1:~;5::,;.,.;<.;t.


.-:•:!i;m::·•
l!l
H ,_,,, ... ,,.,,,,.,,~mrr:".il'!
'-' -~~.;· __........ ·

.:_ :,-:;.;.:;;,:,,, ,: ,.~:;:.':·,.:; :,·.~-

·;11•
_,:) ,~· ·a
11 S ./'t;.'111/n/lC.S
'}foc1//111car· Mo/ion
.
JJ!)
. '
~ .... =J(~ . 1.g/~ (::+c~1-
2
2)..,.,J(i..)(:!+:!-tan:
1
2g ''2 2
1 2°i
' · /
be)·ond its natural ·length and the teus'ion begiris to dpetufo.: TJJ.·c
ve!ocit~ or the weight continues iri'cieas'inji'u,plo C, aim ·wJ1ich if.
=: J(;/{) (11-m1-I 2), ~
1
, sta:rts decrea.1ing. Suppose t-he w.eight .comes. t'o instantanc.ous, rest ·
And if 12 is the ti 111e from A lo B, (.while at D, where CD.=n. . , · . ~ ..
fr~c!y \Jllcl~r During the 111otion of th.e weight-oelow· .B, let.P oe-its·p,osition
~rayity), then · , •
1,..... jf~a)
. af1er any time 1; where CP.=x. (Noti'thJir. ·wt·· have· taken Cas
· · · l
o=O.t,+ 2 gr.,O ·
or
· ·
(i" ·. I ,, of
origin and CP is the directloit x· Jn~'taslng;J, lf T.r be the· tension
the totnl time to return Quck to A=2 (ti1ue from A to /J) i; tl:ie string .-fP, w.e have Tp-_WHj'~ acti)i&' :ve;;i-aiilLy· ,upwards, .
"'•2 (ld·li)=2[j(!g) (~-fun-' ~H- JW')] , The equation of motion: of this ~eight w/<4.atris
=-J(;") [1r-lnn·· 2+2).-1
· IH
4g
1 cl;,i;
d1~=4 w-111
l ¼l+x· 1
7-=·4 w-:;i.w-w,r
I x· I,

This proyes the required result. • J 11· (!Zx x d'x · 4g-


Ex. 6:t A li,r;ltt l!ia,,4·tlc .w,i11g A.{3. n/ l~11q1'1. I l,r fixd. at, A w1d or 4g-Jii=-11• / or dr~=-7X,
Is J·uch that If r; 1i·1tll(ht "' .bt au,u:hcd /o /3, t/u!.strlllg wl// b~. J·ti:I!•
tched to a le11g1/i 2/. If n 11•rlg/11 i_w be a!(CJf/1,,d to, B and l:u .fall wb ich J; the equation 'of a S. H'.-. M. With' c-enite if Pl.\: orj.gln C,
from the /ev<!I of A prova 1'101 (/ J. rite pmpli 111,de of the S,fl,M. t/iar 11rid air.plilude CD (=o). The equntion'(2) ·btilds good1ssi long as
f'nsuqs ts Jl/4; (Ii) tht d/stmrcl! 1lirr111gh whtclt Ir Jal/J' ts 2/; and (IU) the string is s:re1ched.U., for.ih.e motion• ofth~ werg!U frcrm B .,.
tltr; perlo'd ofosc;ll/at(o11 fa ' • ::· w·D. "
J'(')(4•'?•
4g
2 :_,J)..
v ·• .,.,.,+ .rm ~ · (Mceru.t 1!>8!S,. 8RS].
1
2(1xfdt) and integraUng w.r.t, •i 1., we get
Sol. AB=/ ls tl11.1·nn1ural longtll of an elaf:Hic 1strlng whose dx ) 4g ., · · ·
-
( (I .. -~, ;,: 1,+k, wheie 1.is u con-s1a111.
one Clld is fixed · o.t A. Let ,\ be the modulus or I'll 16.<l,o 1 . ' .

elasticity of the sering, If a 1velght iy · be. nttu,ched At B, -~= -¼f DJ1df>;/:lt..;. v._(2g/). i
to tho other und or th!,1 Sf~lna, It cxtc,nds, the ,.stdt'\s
to·a !on11llt 21 while h11n:Ji,p'g 111 eq-uJlibrluin. There.
'
!Ore
21-'/
11•=A •··-- •=A,

,
I
.
'
I
<
,
4g l
2gl=_-T' 16 1•.+k
. (,/x~I 4g. ,9 4g(9 •')•
,
or
·. 9
/e= 4 ,zl,.
·
I
Thus, w~ ltav~ ir;J =-7 -x_t+4z'':"T' i6 IJ-x
1
I , ... (I) ·_ . . . ;(3)

't-low in tJiwvctu!ll probJum '11 parth.ile of weight The equation (3) gives' velocity M,auy po.int between; .!J and
liv or mi1ss" •.l{'w/g~ is nua~hcd to the free end or' t.11c e,'i;.,, /1.', [), At D, ;,;.--=a, dx/df:.:O. Thercfore.(3) giv'es
stri~g •. Let C be thµ,· poshion. of eq!Jillbrium of
tltis weight ¼w, The.n,:conslcleririg the · equilibrium
C;'
~
O=·T· 4g (916
P-a 2 or a~¾I, · )· ·
of this-weight nt C, 1~<:¢ h,we
·
-liv=,\
. --w-
.BC · BC
I · · I ['.' A"awJ
:tt Hence thn.mplitudc a.ot'tJ;e s.. }LM .- tl'ln-t·cns.ucs is ii,
Also. th(! total. disrnnoe through which· .tire 1•1cight fulls
""AB-\-BC-!-Cl)= I+ ll+¼/=2/; ··,
BC=i:ll, "'i · kt t, he 1·he lime ia.ken · by 1/ie weight \lo · Jrce-1'y
t)
Now the w.ei11ht.l·11• is dropped f'rom I/, It falls.I he distance undcl' gravity rrom A to fl. 1
A.8 (=I) freely under graiv/ty, If VJ be the velocity' gainc:d by lh,is The11 usi11i; !11~ formula 1•=1t·•·J), we ·•get ,
w.~ight nt B, we have ·v1=v(2gl) in the downward direction, v(2.~l),::O+gtJ or t, =vPl{g),
Wf1cn tbis weigllt Calls belbw B, the string oegins to extend /1.g,iin lei 11 be the. time !ti'ke'n· by tire. w~ig\H1tb.Jull·rr.u~1~.,Bdtl
I) wbile moving in S.IL,M, Prom (3),.011 taklnfsq,va'te,.root, we
' .. . .. . ,,, '

,,.,,.,·._ , .~:..-,r,··.<• ,. .
.,,.t:\_<'."t,·,•,·.-,, ,':\~·,.,.\n.•, ,. ,·,• .-,•.•,•.-.-.,,.

!20
-Rect.i/i11ear Motldn
1 I . ·12i:· . •
'&ct <!!.•=+J(. l·g)J··.(2:..
· dt
4
·' ,l(i
/~~~I)
' ' ver/ical/:,, above :O"pnd o·A=.a. S(tOHf that its _velqc./1J!. will be.. ,.~ro.
al B, wliere 0E=3a, (Mcorut.77, 83)
where the +tve sign h~~ been takeh because the weigh! i~ moving Calculate afso th<! litne from A to .8.
In the direction· of:-< lncr.easfn;,:. S~parating the vi.rinbles, we :;;et
Sol. Let OC-o, be the nnture.l length A 1~/•o
·(·l). dx·
'J'
4g "]f'' -,-,
' n-,r ) =,dt.
.
of aa elastic string suspended froni the !ixe<J
poii1t 0. The modulus of ela.s_tlcity ,\ of tin, .
siring, is given to be equul to 2111g, where m Q

. Integrating.from ·E to D,"v..•e ·get i's the mas.1 of the particle uttnciled to. the

f o
J('·)J -/{-to·
t 1 dt= ·
'· ·
-
4g
3
.·dx
· 1/I _,.,..,___
-111 11- xi)
..
.
1
other cnd··of th'c string. · ·
Jf'D is the position (1f equilibrium ol'ths
0 0

. J(')[
t2= , •4g ;=J:.::·]~//<
Sin•l-r/ ,_
11
'(')['
4g ;11r 1 I -Sin• 1 (--
particle such that CD=b, then at D th¢ te11~
sion TD in the string OD bala.nc.es the weisht a Cl
)·r ,.
. = J( l ½"+sin- J" .
I 1 . of the particle, .
4g
1
tr . ' ' I b b
to. fS.:l from A to D=t: +t~
· ·mg= r,i=-i\ -=2mg,.-'
a.
C c.~•v.
.
Hence 'the'tota.l time 'tak~n
J( )r '·' ,
q I

~u:
I 7i>
(21) · I
. ;;.;·J. :g_· +...~g l iltr-r:sin- J 1
_!-
or b=a/2, ,
Th~ particle, is let fall from A wlierc .
b
[)
0
I
0A=a, Th~a thc-niolio11 from A to C w.ill "1<9
=J( 4~)H+si,11:.\ ¼+2~2.} . . . 1 3 be freely under gravity ..
· Now a.fter i:nstt1ntaneous rest, at D,' the ·'.\leigbt begin!. to If Vis the veloci1y of the particle gained
at the point C, then· ""Q
..• · ti;1ove upwards', Fro.in ,D t'l ..B'it lrl9ve-. In S.H,M, whose equation. B
.is (2), At .B the string. becom~~· slaok,aod S.H.M. ceases. The j,n=0+2g,2a or ll-2v'(ag), .. , (I)
vel.ocity of the weight at 'B !&· ./(2gl) 1upwards.. Above 'E the in the downw:1rd direction.
1
welght rises rreely un~~r gravity ,al'\d·comes to-ic.stanta neous rest As the particle mo11e;, bclo\v C, the string besii{s to extend
at A, Thus it oscill.at~s again and agr.in betwee.n A and D. beyond its nntural 'IJ!11g1h artd' the ten.sion ·begin~ to operate, Tiu:
lfqe t•ime-per.!,od. ot Q1:1-e· completo oscfllation= 2. ti me frn1:i veloc'ity of ih~ particle continues increttSing \lpto D after. wl1ich
to,D=21( 1
t1+i~),:")()g){ .r.+~~2+2 1}}· Sitl-
it s.tarts cl~creHsing. Suppose thnt lhe piuticlc comes to instan•
tilneou~_.rcsi at .8, During the motion·b~low C, iet P be tpe posi~
Ex •.'(i3, A heavy pdrtlq/e of niass m is _attached 10 1ion or the particle 01 any tim~ 1, where D,,,.,,,,y, lf Tp is tbc ten•
(111 elaslic Jlring cf lll:1/Ul'Q[ length·./ ft.; whose,modu/11.i sion'in th~ ,;trinn OP, ive
ts eql{_q'f 10 'the ll'elght oj'1/ie particle ancf the o'riier end I is fi.ted-at I· , · T · : b +s
·-a, .i\Ct/1\g
· VCillCa
· II y U)IIVl\f dS,
0, _Tlre'.partlcle, Is let /all fro·m p, .s111,w thpt a part of 1/ie mor/011 1•=•

fs simple harmon/c and that the. g_reatP.sl depth of tl,e particle be/011· The equ1\\io1) ot'
m•,ii1\n o(the particle o.'1 I' i,
Q is (2+:'\13) I ft .. Shq10lia_l.'t/lls:dep1/i 'ts atralnCfl li111e in d.1.v . . b+x
[-/2f11-cor 1 (JJ;,;3))-/(//g) monds, (Luckoo1;1· l91lUJ Ill• --, = n14 - Jr"" 1ng.- ,\,. ·-·
I1t' ' a
. '·. ' Sol. Proceed us in tile preceding example, ' I
~n,g- ,,,1 ,, ~a
Ex, '64:A particle '01 ri1dss in Is 'awi,lted IQ one Cl/ti,?/ a(1 elvslii: ... , -=-. ·-a- ... - 2.ui.c" .-~
·r-1\;

... siring of natural- length a· and •m.odu/u.s, ·of elas!lc/ty 2111g, ,.-t:ose ii;,"( f 2g
·oriier. ~ni:fJU/:i;~'(i'.qr;-0,
. . ' . ·. :TMii.C!r:H~l.:Js}/.~1 fall from rl, wli,•11 .-1 is-~ or
:~~: ' .... ' . ·,,
ii1]:=:-'7ii' x, ;:.· ... ~ J)
' : ~.... '

~ .-~
M
iit1
~·I
:J
:is.Bm1r{i,1•··,~::•:,":, ·· . \;•.: ;;,::;~~~ii:t;i:;;-:1 ,, 'i:Pfrit.:1:;::: ,, :"~:11::/~1::,:: ,; ;; ::11:,;::::,;: ':; . ·;•:; '·~: !: ···::;··::;r'.::• . ' ~::ii··;','..

, ,n,,,.,.,,.c,,,,,,,,, 1,.1.(,1,,_IJl•.••-i•••r;rr-,,..t - ~
·,'.::,,::-::_..:· .. · :;._;;

~~,, ',;•.::•;•,·,•· ;._::,;.,

'1

~
~
·ti' ·~
I'I , ..
I :2.l o.),'1111mf<l· !/;:
lJ
}frc:1iii11itur i\'1'iti,m

tl\e titne from,-( lO l.J=l1-t-l•1 :'


'l
.'\:
1 which represent$ a S. }l. M. wilh ten!r~ :11 /) nn<l hold~ goi,<J
I
for th,: motion frdm C 11, fJ. , . . = Zv'(a/g)-1·.V(a/i.g)'. (,r/Z+ sin · 1 ( l /.3)J
II
-Mul!iplyl11g both sid\/s..~,r (2.l by 2 (tf.tlt/1) :md 1hcn i111egra-
w~ lrnve
~Jv(a/2g) t~v'i+.,,+2 sin"' (!13>J.
.F:x. 65. Tll'O boditf of'inosses M w1d M', ura attuc/1ed:io.tl
(,d·")i
, (ff
11,1 t'•'-k
. ,., -·-'-· ., Q ~ I I k 'is n CO!lSlillll /01,w <'!Id of w1 ~1.·1s1ic stf l11g .111/zose· up.per e11d Is .fixed wul"/1011g,At.
rrst; M' fulls aj(; .l'liow1l1a1 the ;/1~•1a11~e of M fi'?111 1/1~. 11pper c
l3ut at .C, .1: =- OC-..:: -b= -n/.1 ~nd (d.l'/li1yi.,. 1·11 =4•·,~-. of th,• ;·/rl111; at time I is b-i·b·H' cos {v'(g/b) t}. il'li'ere a is tit<! 1u11··
,, n! { .
4as= •-•~.. _;r +k or l<=·~qg. 11:e1ched h•oglli of-1/le .i1rr11g, b c,iid C Iii~ ~/iSl/7/lCtS b)',;W/11'.ch ,., ~OU J
be s/relclwd 11'!'-'II s11pporli1J!J M 1711d NI' respeNiw?.l,1•: · . ·
dt);
( -j,•,t
2ll
""I_...!..
9'
.~l "°'• - (I,',
· I · (Cuckn'ow J 97 f) ·
.. a 2\
0Ae-:1n b9 llicri:..turai ·. l<i1i;th. of · 0 ·1·. ,,o
(,dt-d.~)" =-
S,il. Let
vr 2g(',V rr--.~'
., . . )-
."1 u11 cl:Lstic string suspcn.ded fi·om.thc.tlxed. po.Int
a~ ... (.3}
o. rr e· is the positil,n ;of i:quilibriuin. of lhc .a a, .
If the pal'ticlc com~s tu inslnntancrnu~ rc~t ol 8 wher;: /.IE·=x,. p11rticle or mass ii'! ana9bed ro tile lower. cntl
(my), tltc'n or 111c string a11tl :AB":oli', then''
ot lJ, .~ ....~, ani.J d.~/rl/-o,. Thcr.:forc l'ro1n··:(3), \\'C have A A
. ;ig (9
ii"' :j° 2
a •••~•1•
,. ) . .
, glVII\!( Xt (I,
·:·' .~ "" ,\ ::!.1.!
.,\i,, a "'" ,\ a~- ... ( I J . i, f"8· 'i,
.., Simibrly M'g=A :·... 8 B
'N,nv 0.£/,., OC+ CD .,.'DlJ.,. n-!• }tt•l ..,,1;••· Ja, ... {2)
\Yhich prnl'cs 1/!e llrsr pun of the qu1'.Stiun. Adding {1) and (2)j1 we 7p •
h ,) . "19
'l'o find r!J~• 1/mc from A to U• ( -~1-1-,\,(') "~,\ . --·
. " · a l
1pNg·
11' 11 i., thi: time from ti ro C', \hen [rorn .1·=111•Hf1\
. •· 211 .... o_,•.Jrt1'• 11"'"'1vraJgJ.
,Thus 'he siring wHl b~ sti'elchcd by' the dis, .e,.'ttie.'•O
Now rn,m (3), \\IC huvc tanco ii-H· whc11 Sllpportlng b~.lh thi: tr.l\ss.cs M und· M' · llt the
\)' . " )'
. J(·~'(/•··•·•,\--
low~r end. Let OC b~ the sti'elched lrng1h o!' the string when
0

both tho m,isscs M ai1d M' arc attached to its lowcr_en.d;_ Theo
.,
:~
th,: ,-iv~ slg;1 ha$ been lal;~11.b~c1tusc th..: pnrticl~ is nrnving AC'=b+c and so JJC=AC~,411.d.b-i•c•-b=c,. ~!
rile dinic1/u11 or,\! incr~a~fng Now when M' falls" oft'-ni C, 1l;e, mnss M will bcgl11 to 111ovc 3
or. . 1(n ) . · d1·
dt ,,,, 1 . •_ ·- .........;. .. •
towards B starting wit Ii \:'c_lo,city z~ro a~ C, Le_t· /' be·. the nosithrn I il
2g v'tta'-.¥·) of the pi\rticlc of mas~ M·ai any t_ime 1, where B-P=x.
c: tu B, lhc limo l:1 ~·0111 (.' to
~,
lllf~gmlillg Cron) lJ is ¥lvcn ~y Ir Tl' b..: the tension· it\ the ..str.lng OP, then
l~=,;J(q-"-) f"'1' """",1('!:,:_-.. ,,... :,;t
Tp=A· ':'. ..,::•,· Acting vcriic~lly upwards.,
2g· ....... "'' v'(~/7 -.,..)
)]'Iv;~ u ·. . . r
"" J( a ) r
2'] · lsill'•J · 3'iii
· ( ·~
.,, ,
the equation of nHHio11 of lh~ 1n1nicle or, inu~s M ul /' is
·> • ' '
0
. .., ; ·( 'ig )' , l· -•
~Ill 1-·..~Ill
• -1
1
( .... 1)]
3
Ma;,~•,
d·x. \1g--·1·l'=1•g-.,
l/

b Ax
'

=✓(~) '[i •i ~1 11-i ... =Mg···A a-;r

''.••.:·; ·. ",'.~:.,~i't-!~ffHJ};t1 >H~11::•:i;•(;

.,,,·,•·.1•.• , .t,:•t,<1'-<•"
tllllm~w;~n t: j; ;; : ;u iWHHlUUJUiUDJUti!'.¾tUi!:;;; ;";:; m~; !::,; :: }~ i~·~ ;W~~! ~:mmum!, ~; ~; ;: ;:·; f,, .\ ',: ·:; (~: ;;; ::~; ;~~; l~!,I ~ ii~H~mii-H;.HI~: •;·n~:, \~: ,'1 :•!.'\ ,., •; j ~· f~•;:Tj :;

124 Dy11uh1ics
m Reclili11aar Mo.lion 125
=Mtf-Mg~~g .-:, [·: from (1), Mg=1E]
Mg• I
,,
· ~·
r:.
ro!. ·
Let u smuoth light pulley
be suspended from a lixed point Q 01 C

1
'= - I i x;_ ;r by a ipring OA of nal\1rnl length/
d 2x g ond modl)l,u:1· of elasticity A=;ng,
. d7 =·-b-
I"~! '
x, .. '
(3'
~ ,' l\ Let B be the position of ~quilibrlurn
of the· ,pulley when masses m 1, and
which represents a S. H. ·M, wi1,h.centre al B c1nd amplitude BC.
·, .
11!
m2 hang at the ends of a light inex, ..\;/ ,4, A
Multip,lying bo1h Si'dcs of (3) by• 2 (dxidt) and tben .inte, tensible · string passing round 'the d
ci
grating ·.w.r.t. 1 1", we have rulley: Let T be· the tension in th.~ T
816
inextet\sibl~ string passing r.9tmd the ' ' 8
. (dx) i
. dr =:...
g i ., . ,. •
6 x. +,<,. where» 1s a ~011sta111, pull~y. · Let us first find the value ''
,.,13' _T'~ z/,
of T. , /1 p
.But al the point C, :x=BC=c and dx/dt=O. Let f be 1he common ~ccclcJ'O• r
21.tg
0=-:(glb) c 2 +k •or k=(g/b) ,.i. lion of tli~ par1icles m1• m, whjch
hang nt the ends of a light inexten,
,nt,,?

.'(dx)
2
dt =} (ci-x•) · ;~; sible string passing round 1hc pulley, Jf 1111 > 11111 th~ 11 l he cq 11a,
\Ii
1t lions of motion of,ni 1, m, arc
or ·cl."< J· u . 2 ).
d1=- .b . . c_-x '
(g ) )(· .\ ."·
__;j ( .
111,g-T=m,f a1td T.-111,~=111,,/
, I , 2m11111 ·
the -ive sign has been tak~n since tbe particle is moving in 1i1e So,ving, .WC £Cl T=(m1 +ii,·;; g=Mg,
direction of ,x decreasing. 2m1m1 h , .
w herr. M =-+·- =l 11e armon1c me£\n between m1 and 1111,
; ln1 l'1-J ·

Jl
d I=-;
J( ib.) ::J(ci"::...:,:2),separaingl
d;,: l' he,varia
. bl' cs.
.Now the pressure on the pulley=2T=2Mg an'iJ. iherefore'the

I3_ui at
C, '.=0 a.t\d·x=c.; . D- 0. .
1
Jntegrating·, l=;'-/(b/g) cos- (x/c)°+D, )Y.here Dis a co11stan1.
• pulley, which itself is light, behaves like a pnrticle 'of mass 2M.
Now the problem reduce.1 to the vertical motion o~ a m11ss
!Iii
p
fl
ol
I= v(b/g)

x=BP~a .cos {..j(g/b) 1},


cps- 1 .(x/c)
1 2M attac_hed to th~ end A o.f the s.tring OA 1Yhose other end i.1
fixed at!)." Jf B.is the ~q1;ilibrium posiiion of t~e nrn.ss 2M and
1

AB=d, the·:i the tension TR in the spring OB is ~(cl,/), acting ver,,


•1! .I , ·.·.the r'~·quired di~·tanc~ 01:'tbe _par.licle ~f mass M at \imc ,·1 ticn\ly npw:irds .
i ·r Olil the point O . · , For cq·.1ilibrium of the pulley of mass 2M nl \he roint B,
· · ,;:.'OP·=OA.;,AB+BP=a+b+c cos (y(g/b) r). ' I . . d d
11·! II Ex. 66. '\i s111uo1h light pulley Is sus~cnded fro~1 afixad-µoi,,-r
1
wciave 'fM.r:-Ta-A.i'-ngf
I
•)
i.f .l bi•·(I spr/11g,of 110111ml ia>}gth ·l 011<! 111odulus _of elcrstl~ity 111g. If ,, 2MI •
or• r.JIIIZ - - · _. I
I. 11. sse.r.m 1 anil 111, hq11g al //1e eucls ·of ·.a. /lgl11 inexrmsible siring ;n \ . I , .. (J)
· µ· ·~s/'.1g ro111.1d rlt~ 11111/e;:, .ihq11· _,_hat· .lhP. p11/lc,1·. cxccut~s s_,·11,pl~ Now let the particle of mass 2M ),c.,sllghtly·pullcd down.and
l!\!, i, m1011/c 11,ur/011 about a c~111rc 11·hose def,"f h · bc/01,· rh~ i;o1111 nf thrn let go, . If I' is the t,osition of.th!~ ~!1rtl~_1~. at tim~ .. , .such
~l' sµc11sio11'./sJ {I +q~•f/11)}, .\\'lier.~: M. 'r-Jha harmonic mea11 b~- that .81'=,,.v, the11 th~ tenslon ·!n thc·sprd,g.OP:.. . ·.
11ce11111 1 ~h<i'1i'i1:" ,_.., ,·. ·.·.· . ,:: ...... :.· ·,,.TMccrut 1981, 84, OS SJ , d+x d+.~ ·, , .cl
If
·_ill
·I . .. I ;'-~C- -{f:··:;
\~ ;·i·
=Tr=-,\ '"'T =11g, . ,..( .. , (\Ctll\g vct11cul\y·up1ynr $,

l
!.. l·l,
I' -~~ 1(

\'.

;,,:_:·. .. -:~: ~: ~-; :'.:: ', .. ' :-;,,:,;,:,:,··,


.,'',.'.',•
:'·~');:::;~~:~::r;·•·; ·..... ~;:-.:
,_::•:,..
I).~!::·::~.',• • : '. ~; ~: '. ~ \: ' • I •, •:::-:,:::!:_:::::-::
1111
lo -----=.,,.-;r;<-'7'7,""""'.r:-:-7"1"'"'"'"",~•:a;t:\7,..,-,;,..-,";::;!:~

:~:r~-~::;::-,-------~ ... - . .- -~-----. .•


---):)\J]::::~;\::)\.:, ·:., -UT/):.? --.- )\?U--(-·
:~'.::;/;:·:- ·
.. , ,!~', ,::; .
·,.,·::-:1,
.•," --~1.;:;1 ..

::j
~i11
'I
;J
;/ 126 Rec!l/i'ncar Morion l 27
:1 Dynamics
•:.., .... Rut 111 .1, -~= OAa=n ,nnd. d:r/dt=O,
The cqua1h,n or mo1ion of the pl1lley is ,given hy
''f d'.i•. ~ '!
~/J' •(I/!'= "Ir'{;--
'i p o.,,;•~-'-A
a '
or· A=-~'.
a
d ':X J
~•-2Mg •- n/f · ✓-" .,. 2Mg-·off f
II!,;
·-y ug .
.\'= --T x, (did.~):.. 2 ('..'I: '.C/l) '
- ~ /t - - -

( I )l which gives. 1hc v~locil)' of rarticlc at nny di~tance x, from the


ff fl,
cen1re tif,force 0.
•·• -2.MI .'/, From (2), we hnve 01, taking square ro;i!'
whk)l represents(\ simple l\:1r111onic motio11.nhout the C~lllrC B.
Hence thC'f'lUllcy executes simple harmonic motio~ witli
d"'
iii= - )(2'')
ti .j(a·". .:. ,;) . . '
centre RI the rioil'lt B whMe depth below 1he point OI" rnsr,e,1slon· [Here _:ive $is·;,
i~ 1nker'i'since· th'c· P.ri 1icl,c i~ 111ovi1jg in· the
1
0 is given b}' direction of ;i: d~cr-easing].'
I
0 lJ,.. 0 rt 1-11.B·,,_,f+d Separating the vnriubles, we g~t
;d,1-~;.'
' I!
.~'..,.,_/ (1 +~":!)
I! di••• -j(-°2-I' ,j·(-:-· )· dx,
µ.~ 0-•X · ,,

-J(~,.) f}{i'!..x).
!i ·J f~ Motlo11 utulcr _lnnrs'c sq11ore h1w. I . .
,-t parrf<•/e mr.i,•e.r In a' .r1ralgh1 line under 011 a11rac1/o11 1011·11rrfs ·i:_ntegrath1g, I= <(.i:•f•:B, where a · i
ll fi;~rd po/11/ 011 t/1<1 //mr, tl'h/1•/i 1•arfr,Unl'.irsi•l,1• as· the ,,·quarr o.f !he co11stnnt qri11te,1?ralion, • , .
dlst1111c·e. from 1/ir Ji.rl/'d po/111. ~f 1hr par//ele 1v~., P111ting .Y=n co~" 11., so lhflt_d.x,.. "'-2(7 cos 11,iin O <10; we hiw
10! ini•r.rllf('ntr 1l1i- 1111)/lc,11.
j( -P. )f\ /(. cos r'·.'" ,' ,."

·
(Lucknow I977, Mee rut BJS, 84, 86, 87S) Ii:
a
-;,- I -c; C(lSi 8i () .-211 SIil () cos {I ,ftl+B
(7-(1 ' .
Lc't n parricle ~tnrt from .rm from~ point A such 1l1a1 OA,.,..n,
where O is 1hc fixed point-(1.e., the.centre force) on tl1c .line of =aj(~) .J 2 cos d0,1..B=f/ J(fµ) .J (I •!-COS 28).
1
{I

nn? i~ fo_ke.n ns origin .. Lee P. be t.~c position of_ th_e particle nt


· nny'timc·.t, sucll ·1ha1 OP=.t, Then 1hc ·:iccel,:ration at
1
""'a j( 2:,-. . B·-. f ::•~=-:(7.
a ) ( . ~) . . J(_2aµ/\ ..re+srn.q
. . .
cC1.~ 111+/J
fnli (ml's 0, where 1, is n con5lnnt., •0
,.,,17J(¾)-[8-t--/(1_~:•COS' 0) •?OS I.I! +,E. . .· . . •
~ . ---L_;, !!:! •0 1
llu1 ,r=n c.os·' 8 me~n$ cos l.l=v(-~/a)·and.O=cos,- ../(.1·/(7).
,,t,·•······-.·-··{•.'.
C,., 1~·•

.: /' . '
P A
,', ,,,·,aj(f;,). [cps-•J(:-ri~J(1 ~}). J(~-)li B.,
;,;~: cquali1•,1yhl' moii,111 111' the pur1icle at. ti is Hut in/tinily ni A,,J..o nn:d·.\'=•O.>l=,1,, · ·. · · ·
' . II';·,: \
~ ..... =:i--• 1
, , 'd(''
.i
,),. '
· ·.-:"
. ...(1)
I
, '( ) . \
O=:aj !!. {Q ..i.QJ,: • .s· or." B=O · '

2/! •' ' , ·' , . , J•.:s•' i~


[-iv<!' ~ign has b~c11-i11kel\ b~911t1sc d'xidr~ is p1,sitivc i11 th~
direcl'IC1n of.~ incrensing while h~re 1•/.1·' nets ln the direction of s
uc1:1rcnsirig]. • · '.
,. 1=11)(;~)[ co~-iJ(~:)~J(i-'[;-) .)(t·)J' ·. . . / ) i:
[,
t!
which gives the time l'rnm tho inflinl rosiiion A to any point d1L
::
·) !i
Mult'lplyi11g·t,01h sidc,;·of(I) by 2(dxfrlt) and lhc11 int~gru. from Ihe c.:n1r~ or force.,
. . 11·.~.1. •,..
ting we have '(ds)~
-r· ,..,,,2,, . '
.~__ ,1,,ii,. where
///
A
. cn11s1an1 nf·
1s
1nn1 x
Putting .r=O in (3). the tiin~. 11 t:ik~,; h~1 the ral'ticl~ f'rom U
,.
i1Hcj!ration.. ·, to'Oi~givenby. I .

..........,~~... .:....----:""'·""-·----.... -........ ,


i:, ,.,. !'J • ,.,:-:•: ".• i,•.{r~ ~ :"·'~- 1

~~~;5~i.1m~,'1?,{i)~tlth'f}i~!i!U~!1t:m;·~fflT!'.i:~r,:t.;nt;~ ! t:~:-i-"f.1(t;;: '~H!!ltlfflnfJS?H~!.,.11'l~~l~1'mmu:-: ij:J:mtnt,'1 1•nt!tHJfJlfriHffHHt~~tHJ~N1 \·!!"f:"~!ff1·~t:· -:~~J; · '; ·: ;: i~ ;··,, ''.'• ,;:>.; ~,-:., ,J:>-i ', :t••r:;x-1·1~~Hff'~\~li~!n,;.H~J(;,;~•; 1: r;•.,;; •:, ·' '. ,. r:-:~•HP,-t])'{~,l~t',O,N~t•t+'•>:·f,.11: ·.~.). ;~'"''.>' 1; ~.' •:{,;tr,t1tffS'MlfiSlf!Stl'fUH;,p;"»•W'\'
f28 Dy1111111ic.i !l.ectl/l1ear M oilon 129
Let 11 be the whole tirne·or d~&cent frClm ,: ... a to.i=O. Th~11
• t,=a j( -aJ .
.. J./>
+o ].,,_,,2 j(. -2,,
an ).
... (4) ' at• O, 7.=0, l=t,. Putting these values in the·.relation (l) <;onncc-
ting x and 1, we lia:ve
Pµtting x=O in (2), we that-the vcl,ocity at O is infinile
1
and therefore the,par'ticle moves to the lef~of O. l3ut the acce\e- · j(an
. t,= '2.K·)1[ cos- 0+0= J· "21\J,·/(a'2K·.:
1 )· . ...p.)
~at ion o_n the par~ic!e' is towards .0, so
the o.rticle moves to the
, Now let 12 be the time from .-:=a to x=3a/4, Then putting
lefif O und'er ·~efo.rdii.tio.11 il'hich is inverse y proportio:,~f to the x;,,,Ja/4 and 1=12 in (1), we get, • ·. ·
squ r.e of.the .dlsta1,ce from O. Thc'rinrticle will conie to in1tan•
tan 6us rest. at· A', where OA' :• o·A =a; ·and ·then retrac~ irs· rath.
Th , the particle will oscillate bet iveen A nnd A'.
I,= J(u rlco~_(v'-f3) +")(3'4' ifl)1j= ✓-('1°.2K,)["6+4..,/3] .
1
a ') 1

A;;iain let 1:i be the time·from x=;:,i to x";;u/{, Then putting


·
Time 'or ~11c·co1nplcte osci I.Jation =4 x (pme from A to OJ x.--=a/4 and f"!'/o in (I), we get ··
1 =4t,=2rrv'(a 0/21i),
·, • • !
§ tZ. l\ilodorr or a, parll~ic l;nder the attraction or th,\ earth. to= J(;~)[
cos·• H-j(~. ~)l:J(~~Hi+ 1}, .
-~Ne·iioi1'~.Ja\v of gral'ltation, Whcn·a pariklc moves u:1der the Therefore if 1 ◄ -be the tio1c fro1fi x"l3a/4 to X=~/4, fc have
a:ttr ction of the..earti,, !hC a<;d:lerntion ncliDg 011 it !O\',nrus the a~·· ) ["' "1 "j(qo J ·. . .
t,=Jo,-f,= J( 2K, 3-6, =15 . 2Kj
cent C··Of the earth will be. i1s follows : · ·
1' '' When. t,lic particle rrto~cs (upwards,or' d01Vll\\':l.td~). out•
sido the surface o( tl1c carth,.-t.hc accclcralio 11 varies in•

You might also like